College Algebra - OpenStax CNX

1177
College Algebra Collection Editor: OpenStax College Algebra

Transcript of College Algebra - OpenStax CNX

College Algebra

Collection Editor:OpenStax College Algebra

College Algebra

Collection Editor:OpenStax College Algebra

Authors:OpenStax College

OpenStax CATOpenStax College Algebra

Online:< http://legacy.cnx.org/content/col11759/1.2/ >

OpenStax-CNX

This selection and arrangement of content as a collection is copyrighted by OpenStax College Algebra. It is licensed

under the Creative Commons Attribution License 4.0 (http://creativecommons.org/licenses/by/4.0/).

Collection structure revised: February 5, 2015

PDF generated: February 6, 2015

For copyright and attribution information for the modules contained in this collection, see p. 1162.

Table of Contents

Preface . . . . . . . . . . . . . . . . . . . . . . . . . . . . . . . . . . . . . . . . . . . . . . . . . . . . . . . . . . . . . . . . . . . . . . . . . . . . . . . . . . . . . . . . . . . . . . . 1

1 Prerequisites

1.1 Introduction to Prerequisites . . . . . . . . . . . . . . . . . . . . . . . . . . . . . . . . . . . . . . . . . . . . . . . . . . . . . . . . . . . . . . . . 91.2 Real Numbers: Algebra Essentials . . . . . . . . . . . . . . . . . . . . . . . . . . . . . . . . . . . . . . . . . . . . . . . . . . . . . . . . . . 101.3 Exponents and Scienti�c Notation . . . . . . . . . . . . . . . . . . . . . . . . . . . . . . . . . . . . . . . . . . . . . . . . . . . . . . . . . . 341.4 Radicals and Rational Expressions . . . . . . . . . . . . . . . . . . . . . . . . . . . . . . . . . . . . . . . . . . . . . . . . . . . . . . . . . . 561.5 Polynomials . . . . . . . . . . . . . . . . . . . . . . . . . . . . . . . . . . . . . . . . . . . . . . . . . . . . . . . . . . . . . . . . . . . . . . . . . . . . . . . . 711.6 Factoring Polynomials . . . . . . . . . . . . . . . . . . . . . . . . . . . . . . . . . . . . . . . . . . . . . . . . . . . . . . . . . . . . . . . . . . . . . . 831.7 Rational Expressions . . . . . . . . . . . . . . . . . . . . . . . . . . . . . . . . . . . . . . . . . . . . . . . . . . . . . . . . . . . . . . . . . . . . . . . 96Solutions . . . . . . . . . . . . . . . . . . . . . . . . . . . . . . . . . . . . . . . . . . . . . . . . . . . . . . . . . . . . . . . . . . . . . . . . . . . . . . . . . . . . . . . 113

2 Equations and Inequalities

2.1 Introduction to Equations and Inequalities . . . . . . . . . . . . . . . . . . . . . . . . . . . . . . . . . . . . .. . . . . . . . . . . . 1292.2 The Rectangular Coordinate Systems and Graphs . . . . . . . . . . . . . . . . . . . . . . . . . . . . . . . . . . . . . . . . . 1302.3 Linear Equations in One Variable . . . . . . . . . . . . . . . . . . . . . . . . . . . . . . . . . . . . . . . . . . . . . .. . . . . . . . . . . . 1302.4 Models and Applications . . . . . . . . . . . . . . . . . . . . . . . . . . . . . . . . . . . . . . . . . . . . . . . . . . . . . . . . . . . . . . . . . . 1552.5 Complex Numbers . . . . . . . . . . . . . . . . . . . . . . . . . . . . . . . . . . . . . . . . . . . . . . . . . . . . . . . . . . . . .. . . . . . . . . . . . 1682.6 Quadratic Equations . . . . . . . . . . . . . . . . . . . . . . . . . . . . . . . . . . . . . . . . . . . . . . . . . . . . . . . . . . . . . . . . . . . . . . 1842.7 Other Types of Equations . . . . . . . . . . . . . . . . . . . . . . . . . . . . . . . . . . . . . . . . . . . . . . . . . . . . . . . . . . . . . . . . . 2052.8 Linear Inequalities and Absolute Value Inequalities . . . . . . . . . . . . . . . . . . . . . . . . . . . . . . . . . . . . . . . . 223Solutions . . . . . . . . . . . . . . . . . . . . . . . . . . . . . . . . . . . . . . . . . . . . . . . . . . . . . . . . . . . . . . . . . . . . . . . . . . . . . . . . . . . . . . . 245

3 Functions3.1 Introduction to Functions . . . . . . . . . . . . . . . . . . . . . . . . . . . . . . . . . . . . . . . . . . . . . . . . . . . . . . . . . . . . . . . . . 2733.2 Functions and Function Notation . . . . . . . . . . . . . . . . . . . . . . . . . . . . . . . . . . . . . . . . . . . . . . . . . . . . . . . . . . 2743.3 Domain and Range . . . . . . . . . . . . . . . . . . . . . . . . . . . . . . . . . . . . . . . . . . . . . . . . . . . . . . . . . . . . . . . . . . . . . . . . 3263.4 Rates of Change and Behavior of Graphs . . . . . . . . . . . . . . . . . . . . . . . . . . . . . . . . . . . . . . . . . . . . . . . . . . 3263.5 Composition of Functions . . . . . . . . . . . . . . . . . . . . . . . . . . . . . . . . . . . . . . . . . . . . . . . . . . . . . . . . . . . . . . . . . 3263.6 Transformation of Functions . . . . . . . . . . . . . . . . . . . . . . . . . . . . . . . . . . . . . . . . . . . . . . . . . . . . . . . . . . . . . . . 3263.7 Absolute Value Functions . . . . . . . . . . . . . . . . . . . . . . . . . . . . . . . . . . . . . . . . . . . . . . . . . . . . . .. . . . . . . . . . . . 3263.8 Inverse Functions . . . . . . . . . . . . . . . . . . . . . . . . . . . . . . . . . . . . . . . . . . . . . . . . . . . . . . . . . . . . . . . . . . . . . . . . . . 326Solutions . . . . . . . . . . . . . . . . . . . . . . . . . . . . . . . . . . . . . . . . . . . . . . . . . . . . . . . . . . . . . . . . . . . . . . . . . . . . . . . . . . . . . . . 327

4 Linear Functions4.1 Introduction to Linear Functions . . . . . . . . . . . . . . . . . . . . . . . . . . . . . . . . . . . . . . . . . . . . . . . . . . . . . . . . . . 3354.2 Linear Functions . . . . . . . . . . . . . . . . . . . . . . . . . . . . . . . . . . . . . . . . . . . . . . . . . . . . . . . . . . . . . . . . . . . . . . . . . . 3354.3 Modeling with Linear Functions . . . . . . . . . . . . . . . . . . . . . . . . . . . . . . . . . . . . . . . . . . . . . . . . . . . . . . . . . . . 3354.4 Fitting Linear Models to Data . . . . . . . . . . . . . . . . . . . . . . . . . . . . . . . . . . . . . . . . . . . . . . . . . . . . . . . . . . . . . 335

5 Polynomial and Rational Functions

5.1 Introduction to Polynomial and Rational Functions . . . . . . . . . . . . . . . . . . . . . . . . . . . . . . . . . . . . . . . . 3375.2 Quadratic Functions . . . . . . . . . . . . . . . . . . . . . . . . . . . . . . . . . . . . . . . . . . . . . . . . . . . . . . . . . . . . . . . . . . . . . . . 3375.3 Power Functions and Polynomial Functions . . . . . . . . . . . . . . . . . . . . . . . . . . . . . . . . . . . . . . . . . . . . . . . . 3375.4 Graphs of Polynomial Functions . . . . . . . . . . . . . . . . . . . . . . . . . . . . . . . . . . . . . . . . . . . . . . . . . . . . . . . . . . . 3375.5 Dividing Polynomials . . . . . . . . . . . . . . . . . . . . . . . . . . . . . . . . . . . . . . . . . . . . . . . . . . . . . . . . . . . . . . . . . . . . . . 3375.6 Zeros of Polynomial Functions . . . . . . . . . . . . . . . . . . . . . . . . . . . . . . . . . . . . . . . . . . . . . . . . . . . . . . . . . . . . . 3375.7 Rational Functions . . . . . . . . . . . . . . . . . . . . . . . . . . . . . . . . . . . . . . . . . . . . . . . . . . . . . . . . . . . . . . . . . . . . . . . . 3375.8 Inverses and Radical Functions . . . . . . . . . . . . . . . . . . . . . . . . . . . . . . . . . . . . . . . . . . . . . . . . . . . . . . . . . . . . 3375.9 Modeling Using Variation . . . . . . . . . . . . . . . . . . . . . . . . . . . . . . . . . . . . . . . . . . . . . . . . . . . . . . . . . . . . . . . . . 337

6 Exponential and Logarithmic Functions

iv

6.1 Introduction to Exponential and Logarithmic Functions . . . . . . . . . . . . . . . . . . . . . . . . . . . . . . . . . . . 3396.2 Exponential Functions . . . . . . . . . . . . . . . . . . . . . . . . . . . . . . . . . . . . . . . . . . . . . . . . . . . . . . . . . . . . . . . . . . . . . 3406.3 Graphs of Exponential Functions . . . . . . . . . . . . . . . . . . . . . . . . . . . . . . . . . . . . . . . . . . . . . . . . . . . . . . . . . . 3666.4 Logarithmic Functions . . . . . . . . . . . . . . . . . . . . . . . . . . . . . . . . . . . . . . . . . . . . . . . . . . . . . . . . .. . . . . . . . . . . . 3976.5 Graphs of Logarithmic Functions . . . . . . . . . . . . . . . . . . . . . . . . . . . . . . . . . . . . . . . . . . . . . . . . . . . . . . . . . . 4116.6 Logarithmic Properties . . . . . . . . . . . . . . . . . . . . . . . . . . . . . . . . . . . . . . . . . . . . . . . . . . . . . . . . . . . . . . . . . . . . 4486.7 Exponential and Logarithmic Equations . . . . . . . . . . . . . . . . . . . . . . . . . . . . . . . . . . . . . . . . . . . . . . . . . . . 4656.8 Exponential and Logarithmic Models . . . . . . . . . . . . . . . . . . . . . . . . . . . . . . . . . . . . . . . . . . . . . . . . . . . . . . 4856.9 Fitting Exponential Models to Data . . . . . . . . . . . . . . . . . . . . . . . . . . . . . . . . . . . . . . . . . . . . . . . . . . . . . . . 511Solutions . . . . . . . . . . . . . . . . . . . . . . . . . . . . . . . . . . . . . . . . . . . . . . . . . . . . . . . . . . . . . . . . . . . . . . . . . . . . . . . . . . . . . . . 545

7 Systems of Equations and Inequalities

7.1 Introduction to Systems of Equations and Inequalities . . . . . . . . . . . . . . . . . . . . . . . . . . . . . . . . . . . . . 5997.2 Systems of Linear Equations: Two Variables . . . . . . . . . . . . . . . . . . . . . . . . . . . . . . . . . . . . . . . . . . . . . . . 6007.3 Systems of Linear Equations: Three Variables . . . . . . . . . . . . . . . . . . . . . . . . . . . . . . . . . . . . . . . . . . . . . 6327.4 Systems of Nonlinear Equations and Inequalities: Two Variables . . . . . . . . . . . . . . . . . . . . . . . . . . . 6517.5 Partial Fractions . . . . . . . . . . . . . . . . . . . . . . . . . . . . . . . . . . . . . . . . . . . . . . . . . . . . . . . . . . . . . . . . . . . . . . . . . . 6717.6 Matrices and Matrix Operations . . . . . . . . . . . . . . . . . . . . . . . . . . . . . . . . . . . . . . . . . . . . . . . . . . . . . . . . . . . 6867.7 Solving Systems with Gaussian Elimination . . . . . . . . . . . . . . . . . . . . . . . . . . . . . . . . . . . .. . . . . . . . . . . . 7067.8 Solving Systems with Inverses . . . . . . . . . . . . . . . . . . . . . . . . . . . . . . . . . . . . . . . . . . . . . . . . . . . . . . . . . . . . . 7277.9 Solving Systems with Cramer's Rule . . . . . . . . . . . . . . . . . . . . . . . . . . . . . . . . . . . . . . . . . . . . . . . . . . . . . . . 750Solutions . . . . . . . . . . . . . . . . . . . . . . . . . . . . . . . . . . . . . . . . . . . . . . . . . . . . . . . . . . . . . . . . . . . . . . . . . . . . . . . . . . . . . . . 781

8 Analytic Geometry

8.1 Introduction to Analytic Geometry . . . . . . . . . . . . . . . . . . . . . . . . . . . . . . . . . . . . . . . . . . . . . . . . . . . . . . . . 8098.2 The Ellipse . . . . . . . . . . . . . . . . . . . . . . . . . . . . . . . . . . . . . . . . . . . . . . . . . . . . . . . . . . . . . . . . . . . . . . . . . . . . . . . . 8108.3 The Hyperbola . . . . . . . . . . . . . . . . . . . . . . . . . . . . . . . . . . . . . . . . . . . . . . . . . . . . . . . . . . . . . . . . . . . . . . . . . . . . 8398.4 The Parabola . . . . . . . . . . . . . . . . . . . . . . . . . . . . . . . . . . . . . . . . . . . . . . . . . . . . . . . . . . . . . . . . . . . . . . . . . . . . . 8698.5 Rotation of Axes . . . . . . . . . . . . . . . . . . . . . . . . . . . . . . . . . . . . . . . . . . . . . . . . . . . . . . . . . . . . . . . . . . . . . . . . . . 8968.6 Conic Sections in Polar Coordinates . . . . . . . . . . . . . . . . . . . . . . . . . . . . . . . . . . . . . . . . . . . . . . . . . . . . . . . 919Solutions . . . . . . . . . . . . . . . . . . . . . . . . . . . . . . . . . . . . . . . . . . . . . . . . . . . . . . . . . . . . . . . . . . . . . . . . . . . . . . . . . . . . . . . 942

9 Sequences, Probability, and Counting Theory

9.1 Introduction to Sequences, Probability and Counting Theory . . . . . . . . . . . . . . . . . . . . . . . . . . . . . . 9979.2 Sequences and Their Notations . . . . . . . . . . . . . . . . . . . . . . . . . . . . . . . . . . . . . . . . . . . . . . . . . . . . . . . . . . . . 9989.3 Arithmetic Sequences . . . . . . . . . . . . . . . . . . . . . . . . . . . . . . . . . . . . . . . . . . . . . . . . . . . . . . . . . .. . . . . . . . . . . 10239.4 Geometric Sequences . . . . . . . . . . . . . . . . . . . . . . . . . . . . . . . . . . . . . . . . . . . . . . . . . . . . . . . . . . . . . . . . . . . . . 10419.5 Series and Their Notations . . . . . . . . . . . . . . . . . . . . . . . . . . . . . . . . . . . . . . . . . . . . . . . . . . . . . . . . . . . . . . . 10569.6 Counting Principles . . . . . . . . . . . . . . . . . . . . . . . . . . . . . . . . . . . . . . . . . . . . . . . . . . . . . . . . . . . . . . . . . . . . . . 10739.7 Binomial Theorem . . . . . . . . . . . . . . . . . . . . . . . . . . . . . . . . . . . . . . . . . . . . . . . . . . . . . . . . . . . . .. . . . . . . . . . . 10889.8 Probability . . . . . . . . . . . . . . . . . . . . . . . . . . . . . . . . . . . . . . . . . . . . . . . . . . . . . . . . . . . . . . . . . . . . . . . . . . . . . . . 1099Solutions . . . . . . . . . . . . . . . . . . . . . . . . . . . . . . . . . . . . . . . . . . . . . . . . . . . . . . . . . . . . . . . . . . . . . . . . . . . . . . . . . . . . . . 1120

Glossary . . . . . . . . . . . . . . . . . . . . . . . . . . . . . . . . . . . . . . . . . . . . . . . . . . . . . . . . . . . . . . . . . . . . . . . . . . . . . . . . . . . . . . . . . . . 1142Index . . . . . . . . . . . . . . . . . . . . . . . . . . . . . . . . . . . . . . . . . . . . . . . . . . . . . . . . . . . . . . . . . . . . . . . . . . . . . . . . . . . . . . . . . . . . . . 1155Attributions . . . . . . . . . . . . . . . . . . . . . . . . . . . . . . . . . . . . . . . . . . . . . . . . . . . . . . . . . . . . . . . . . . . . . . . . . . . . . . . . . . . . . . 1162

Available for free at Connexions <http://legacy.cnx.org/content/col11759/1.2>

Preface1

Welcome to College Algebra, an OpenStax College resource. This textbook has been created with severalgoals in mind: accessibility, customization, and student engagement�all while encouraging students towardhigh levels of academic scholarship. Instructors and students alike will �nd that this textbook o�ers a strongfoundation in college algebra in an accessible format.

About OpenStax College

OpenStax College is a non-pro�t organization committed to improving student access to quality learningmaterials. Our free textbooks go through a rigorous editorial publishing process. Our texts are developedand peer-reviewed by educators to ensure they are readable, accurate, and meet the scope and sequencerequirements of today's college courses. Unlike traditional textbooks, OpenStax College resources live onlineand are owned by the community of educators using them. Through our partnerships with companies andfoundations committed to reducing costs for students, OpenStax College is working to improve access tohigher education for all. OpenStax College is an initiative of Rice University and is made possible throughthe generous support of several philanthropic foundations. OpenStax College textbooks are used at manycolleges and universities around the world. Please go to https://openstaxcollege.org/pages/adoptions to seeour rapidly expanding number of adoptions.

About OpenStax College's Resources

OpenStax College resources provide quality academic instruction. Three key features set our materials apartfrom others: they can be customized by instructors for each class, they are a "living" resource that growsonline through contributions from educators, and they are available free or for minimal cost.

Customization

OpenStax College learning resources are designed to be customized for each course. Our textbooks provide asolid foundation on which instructors can build, and our resources are conceived and written with �exibility inmind. Instructors can select the sections most relevant to their curricula and create a textbook that speaksdirectly to the needs of their classes and student body. Teachers are encouraged to expand on existingexamples by adding unique context via geographically localized applications and topical connections.

College Algebra can be easily customized using our online platform(http://cnx.org/content/col11759/latest/). Simply select the content most relevant to your currentsemester and create a textbook that speaks directly to the needs of your class. College Algebra is organizedas a collection of sections that can be rearranged, modi�ed, and enhanced through localized examples orto incorporate a speci�c theme to your course. This customization feature will ensure that your textbooktruly re�ects the goals of your course.

1This content is available online at <http://legacy.cnx.org/content/m53195/1.1/>.

Available for free at Connexions <http://legacy.cnx.org/content/col11759/1.2>

1

2

Curation

To broaden access and encourage community curation, College Algebra is �open source� licensed under aCreative Commons Attribution (CC-BY) license. The mathematics community is invited to submit feedbackto enhance and strengthen the material and keep it current and relevant for today's students. Submit yoursuggestions to [email protected], and check in on edition status, alternate versions, errata, and newson the StaxDash at http://openstaxcollege.org.

Cost

Our textbooks are available for free online, and in low-cost print and e-book editions.

About College Algebra

For many students, this course is a terminal course. A derivative of Precalculus, College Algebra wasdeveloped for a broad range of �rst- and second-year college students. Since algebra courses vary from oneinstitution to the next, we've developed a comprehensive book that covers more ground than an instructorcould likely cover in a typical one- or two-semester course; but instructors should �nd, almost without fail,that the topics they wish to include in their syllabus are covered in the text.

College Algebra is designed to give the student a comprehensive and working knowledge of the subject.As a result, applications are not contrived but rather suited with real world examples to meet the variedmajors. Chapters are speci�cally tailored to the particular scope, sequence, and pedagogy of a college algebracourse.

Coverage and Scope

OpenStax College College Algebra follows a traditional approach in its presentation of content. Concepts aredeveloped for students with student-friendly visual connections and concrete examples students can relateto.

Chapters 1 and 2 provide both a review and foundation for study of Functions that begins in Chapter 3.The authors recognize that while some institutions may �nd this material a prerequisite, other institutionshave told us that they have a cohort that need the prerequisite skills built into the course.

• Chapter 1: Prerequisites• Chapter 2: Equations and Inequalities

Chapters 3-6: The Algebraic Functions

• Chapter 3: Functions• Chapter 4: Linear Functions• Chapter 5: Polynomial and Rational Functions• Chapter 6: Exponential and Logarithm Functions

Chapters 7-9: Further Study in College Algebra

• Chapter 7: Systems of Equations and Inequalities• Chapter 8: Analytic Geometry• Chapter 9: Sequences, Probability, and Counting Theory

All chapters are broken down into multiple sections, the titles of which can be viewed in the Table of Contents.

Available for free at Connexions <http://legacy.cnx.org/content/col11759/1.2>

3

Development Overview

Openstax College Algebra is the product of a collaborative e�ort by a group of dedicated authors, editors,and instructors whose collective passion for this project has resulted in a text that is remarkably uni�ed inpurpose and voice. Special thanks is due to our Lead Author, Jay Abramson of Arizona State University,who provided the overall vision for the book and oversaw the development of each and every chapter, drawingup the initial blueprint, reading numerous drafts, and assimilating �eld reviews into actionable revision plansfor our authors and editors.

All thirteen chapters follow a new and innovative instructional design, and great care has been takento maintain a consistent voice from cover to cover. New features have been introduced to �esh out theinstruction, all of the graphics have been re-done in a more contemporary style, and much of the contenthas been revised, replaced, or supplemented to bring the text more in line with mainstream approaches toteaching college algebra.

Accuracy of the Content

We have taken great pains to ensure the validity and accuracy of this text. Each chapter's manuscriptunderwent at least two rounds of review and revision by a panel of active instructors. Then, prior topublication, a separate team of experts checked all text, examples, and graphics for mathematical accuracy;multiple reviewers were assigned to each chapter to minimize the chances of any error escaping notice. A thirdteam of experts was responsible for the accuracy of the Answer Key, dutifully re-working every solution toeradicate any lingering errors. Finally, the editorial team conducted a multi-round post-production review toensure the integrity of the content in its �nal form. The Solutions Manual, which was written and developedafter the Student Edition, has also been rigorously checked for accuracy following a process similar to thatdescribed above. Incidentally, the act of writing out solutions step-by-step served as yet another round ofvalidation for the Answer Key in the back of the Student Edition.

In spite of the e�orts described above, we acknowledge the possibility that�as with any textbook�someerrata have slipped past the guards. We encourage users to report errors via our Errata page.

Pedagogical Foundations and Features

Learning Objectives

Each chapter is divided into multiple sections (or modules), each of which is organized around a set oflearning objectives. The learning objectives are listed explicitly at the beginning of each section, and are thefocal point of every instructional element

Narrative text

Narrative text is used to introduce key concepts, terms, and de�nitions, to provide real-world context, and toprovide transitions between topics and examples. Throughout this book, we rely on a few basic conventionsto highlight the most important ideas:

• Key terms are boldfaced, typically when �rst introduced and/or when formally de�ned• Key concepts and de�nitions are called out in a blue box for easy reference.• Key equations, formulas, theorems, identities, etc. are assigned a number, which appears near the

right margin. Occasionally the text may refer back to an equation or formula by its number.

Available for free at Connexions <http://legacy.cnx.org/content/col11759/1.2>

4

Example

Each learning objective is supported by one or more worked examples, which demonstrate the problem-solving approaches that students must master. Typically, we include multiple Examples for each learningobjective in order to model di�erent approaches to the same type of problem, or to introduce similar problemsof increasing complexity.

All Examples follow a simple two- or three-part format. First, we pose a problem or question. Next, wedemonstrate the Solution, spelling out the steps along the way. Finally (for select Examples), we concludewith an Analysis re�ecting on the broader implications of the Solution just shown.

Figures

Openstax College Algebra contains many �gures and illustrations, the vast majority of which are graphsand diagrams. Art throughout the text adheres to a clear, understated style, drawing the eye to the mostimportant information in each �gure while minimizing visual distractions. Color contrast is employed withdiscretion to distinguish between the di�erent functions or features of a graph.

Supporting Features

Four small but important features, each marked by a distinctive icon, serve to support Examples.

A �How To� is a list of steps necessary to solve a certain type of problem. A How To typicallyprecedes an Example that proceeds to demonstrate the steps in action.

A �Try It� exercise immediately follows an Example or a set of related Examples, providing thestudent with an immediate opportunity to solve a similar problem. In the Web View version of the text,students can click an Answer link directly below the question to check their understanding. In the PDF,answers to the Try-It exercises are located in the Answer Key.

A Q&Amay appear at any point in the narrative, but most often follows an Example. This featurepre-empts misconceptions by posing a commonly asked yes/no question, followed by a detailed answer andexplanation.

The �Media� icon appears at the conclusion of each section, just prior to the Section Exercises.This icon marks a list of links to online video tutorials that reinforce the concepts and skills introduced inthe section.

Disclaimer: While we have selected tutorials that closely align to our learning objectives, we did notproduce these tutorials, nor were they speci�cally produced or tailored to accompany Openstax Col-

Available for free at Connexions <http://legacy.cnx.org/content/col11759/1.2>

5

lege Algebra. We are deeply grateful to James Sousa for compiling his incredibly robust and excel-lent library of video tutorials, which he has made available to the public under a CC-BY-SA license athttp://mathispower4u.yolasite.com/. Most or all of the videos to which we link in our �Media� feature (plusmany more) are found in the Algebra 2 video library at the above site.

Section Exercises

Each section of every chapter concludes with a well-rounded set of exercises that can be assigned as homeworkor used selectively for guided practice. With over 4600 exercises across the 9 chapters, instructors shouldhave plenty to choose from2.

Section Exercises are organized by question type, and generally appear in the following order:

• Verbal questions assess conceptual understanding of key terms and concepts.• Algebraic problems require students to apply algebraic manipulations demonstrated in the section.• Graphical problems assess students' ability to interpret or produce a graph.• Numeric problems require the student perform calculations or computations.• Technology problems encourage exploration through use of a graphing utility, either to visualize or

verify algebraic results or to solve problems via an alternative to the methods demonstrated in thesection.

• Extensions pose problems more challenging than the Examples demonstrated in the section. Theyrequire students to synthesize multiple learning objectives or apply critical thinking to solve complexproblems.

• Real-World Applications present realistic problem scenarios from �elds such as physics, geology,biology, �nance, and the social sciences.

Chapter Review Features

Each chapter concludes with a review of the most important takeaways, as well as additional practiceproblems that students can use to prepare for exams.

• Key Terms provides a formal de�nition for each bold-faced term in the chapter.• Key Equations presents a compilation of formulas, theorems, and standard-form equations.• Key Concepts summarizes the most important ideas introduced in each section, linking back to the

relevant Example(s) in case students need to review.• Chapter Review Exercises include 40-80 practice problems that recall the most important concepts

from each section.• Practice Test includes 25-50 problems assessing the most important learning objectives from the

chapter. Note that the practice test is not organized by section, and may be more heavily weightedtoward cumulative objectives as opposed to the foundational objectives covered in the opening sections.

• Answer Key includes the answers to all Try It exercises and every other exercise from the SectionExercises, Chapter Review Exercises, and Practice Test.

Ancillaries

OpenStax projects o�er an array of ancillaries for students and instructors. Currently the following resourcesare available.

• Instructor's Solutions Manual• Student's Solutions Manual• PowerPoint Slides

Please visit http://openstaxcollege.org to view an up-to-date list of the Learning Resources for this titleand to �nd information on accessing these resources.

24,649 total exercises. Includes Chapter Reviews and Practice Tests.

Available for free at Connexions <http://legacy.cnx.org/content/col11759/1.2>

6

Online Homework

WebAssign is an independent online homework and assessment solution �rst launchedat North Carolina State University in 1997. Today, WebAssign is an employee-owned bene�t corporationand participates in the education of over a million students each year. WebAssign empowers faculty todeliver fully customizable assignments and high quality content to their students in an interactive onlineenvironment. WebAssign supports College Algebra with hundreds of problems covering every concept inthe course, each containing algorithmically-generated values and links directly to the eBook providing acompletely integrated online learning experience.

Learningpod is the best place to �nd high-quality practice and homework questions. Through ourpartnership with OpenStax College we o�er easy-to-use assignment and reporting tools for professors and abeautiful practice experience for students. You can �nd questions directly from this textbook on Learning-pod.com or through the OpenStax mobile app. Look for our links at the end of each chapter!Practice questions on the Learningpod website: www.learningpod.comDownload the OpenStax Companion Workbooks app (iOS): http://bit.ly/openstaxworkbooks

About Our Team

Lead Author, Senior Content Expert

Jay Abramson has been teaching Precalculus for 33 years, the last 14 at Arizona State University, wherehe is a principal lecturer in the School of Mathematics and Statistics. His accomplishments at ASU includeco-developing the university's �rst hybrid and online math courses as well as an extensive library of videolectures and tutorials. In addition, he has served as a contributing author for two of Pearson Education'smath programs, NovaNet Precalculus and Trigonometry. Prior to coming to ASU, Jay taught at Texas StateTechnical College and Amarillo College. He received Teacher of the Year awards at both institutions.

Contributing Authors

• Valeree Falduto, Palm Beach State College• Rachael Gross, Towson University• David Lippman, Pierce College• Melonie Rasmussen, Pierce College• Rick Norwood, East Tennessee State University• Nicholas Belloit, Florida State College Jacksonville• Jean-Marie Magnier, Spring�eld Technical Community College• Harold Whipple• Christina Fernandez

Faculty Reviewers and Consultants

• Phil Clark, Scottsdale Community College• Michael Cohen, Hofstra University• Matthew Goodell, SUNY Ulster• Lance Hemlow, Raritan Valley Community College• Dongrin Kim, Arizona State University• Cynthia Landrigan, Eerie Community College• Wendy Lightheart, Lane Community College• Chinenye Ofodile, Albany State University• Carl Penziul, Tompkins-Cortland Community College• Sandra Nite, Texas A&M University

Available for free at Connexions <http://legacy.cnx.org/content/col11759/1.2>

7

• Eugenia Peterson, Richard J. Daley College• Rhonda Porter, Albany State University• Michael Price, University of Oregon• William Radulovich, Florida State College Jacksonville• Camelia Salajean, City Colleges of Chicago• Katy Shields, Oakland Community College• Nathan Schrenk, ECPI University• Pablo Suarez, Delaware State University• Allen Wolmer, Atlanta Jewish Academy

The following faculty contributed to the development of OpenStax Precalculus, the text from which thisproduct was updated and derived.

Honorable Mention

• Nina Alketa, Cecil College• Kiran Bhutani, Catholic University of America• Brandie Biddy, Cecil College• Lisa Blank, Lyme Central School• Bryan Blount, Kentucky Wesleyan College• Jessica Bolz, The Bryn Mawr School• Sheri Boyd, Rollins College• Sarah Brewer, Alabama School of Math and Science• Charles Buckley, St. Gregory's University• Michael Cohen, Hofstra University• Kenneth Crane, Texarkana College• Rachel Cywinski, Alamo Colleges• Nathan Czuba• Srabasti Dutta, Ashford University• Kristy Erickson, Cecil College• Nicole Fernandez, Georgetown University / Kent State University• David French, Tidewater Community College• Douglas Furman, SUNY Ulster• Lance Hemlow, Raritan Valley Community College• Erinn Izzo, Nicaragua Christian Academy• John Ja�e• Jerry Jared, Blue Ridge School• Stan Kopec, Mount Wachusett Community College• Kathy Kovacs• Cynthia Landrigan, Erie Community College• Sara Lenhart, Christopher Newport University• Wendy Lightheart, Lane Community College• Joanne Manville, Bunker Hill Community College• Karla McCavit, Albion College• Cynthia McGinnis, Northwest Florida State College• Lana Neal, University of Texas at Austin• Steven Purtee, Valencia College• Rhonda Porter, Albany State University• William Radulovich, Florida State College Jacksonville• Alice Ramos, Bethel College• Nick Reynolds, Montgomery Community College• Amanda Ross, A. A. Ross Consulting and Research, LLC• Erica Rutter, Arizona State University

Available for free at Connexions <http://legacy.cnx.org/content/col11759/1.2>

8

• Sutandra Sarkar, Georgia State University• Willy Schild, Wentworth Institute of Technology• Todd Stephen, Cleveland State University• Scott Sykes, University of West Georgia• Linda Tansil, Southeast Missouri State University• John Thomas, College of Lake County• Diane Valade, Piedmont Virginia Community College• Allen Wolmer, Atlanta Jewish Academy

Available for free at Connexions <http://legacy.cnx.org/content/col11759/1.2>

Chapter 1

Prerequisites

1.1 Introduction to Prerequisites1

Figure 1.1: Credit: Andreas Kambanls

1This content is available online at <http://legacy.cnx.org/content/m51240/1.2/>.

Available for free at Connexions <http://legacy.cnx.org/content/col11759/1.2>

9

10 CHAPTER 1. PREREQUISITES

It's a cold day in Antarctica. In fact, it's always a cold day in Antarctica. Earth's southernmost continent,Antarctica experiences the coldest, driest, and windiest conditions known. The coldest temperature everrecorded, over one hundred degrees below zero on the Celsius scale, was recorded by remote satellite. It isno surprise then, that no native human population can survive the harsh conditions. Only explorers andscientists brave the environment for any length of time.

Measuring and recording the characteristics of weather conditions in in Antarctica requires a use of di�er-ent kinds of numbers. Calculating with them and using them to make predictions requires an understandingof relationships among numbers. In this chapter, we will review sets of numbers and properties of operationsused to manipulate numbers. This understanding will serve as prerequisite knowledge throughout our studyof algebra and trigonometry.

1.2 Real Numbers: Algebra Essentials2

It is often said that mathematics is the language of science. If this is true, then the language of mathematicsis numbers. The earliest use of numbers occurred 100 centuries ago in the Middle East to count, or enumerateitems. Farmers, cattlemen, and tradesmen used tokens, stones, or markers to signify a single quantity�asheaf of grain, a head of livestock, or a �xed length of cloth, for example. Doing so made commerce possible,leading to improved communications and the spread of civilization.

Three to four thousand years ago, Egyptians introduced fractions. They �rst used them to show recip-rocals. Later, they used them to represent the amount when a quantity was divided into equal parts.

But what if there were no cattle to trade or an entire crop of grain was lost in a �ood? How couldsomeone indicate the existence of nothing? From earliest times, people had thought of a �base state� whilecounting and used various symbols to represent this null condition. However, it was not until about the �fthcentury A.D. in India that zero was added to the number system and used as a numeral in calculations.

Clearly, there was also a need for numbers to represent loss or debt. In India, in the seventh century A.D.,negative numbers were used as solutions to mathematical equations and commercial debts. The oppositesof the counting numbers expanded the number system even further.

Because of the evolution of the number system, we can now perform complex calculations using these andother categories of real numbers. In this section, we will explore sets of numbers, calculations with di�erentkinds of numbers, and the use of numbers in expressions.

1.2.1 Classifying a Real Number

The numbers we use for counting, or enumerating items, are the natural numbers: 1, 2, 3, 4, 5, and so on.We describe them in set notation as {1, 2, 3, ...}where the ellipsis (. . .) indicates that the numbers continueto in�nity. The natural numbers are, of course, also called the counting numbers. Any time we enumeratethe members of a team, count the coins in a collection, or tally the trees in a grove, we are using the set ofnatural numbers. The set of whole numbers is the set of natural numbers plus zero: {0, 1, 2, 3, ...}.

The set of integers adds the opposites of the natural numbers to the set of wholenumbers: {...,−3,−2,−1, 0, 1, 2, 3, ...}.It is useful to note that the set of integers is made up of three dis-tinct subsets: negative integers, zero, and positive integers. In this sense, the positive integers are just thenatural numbers. Another way to think about it is that the natural numbers are a subset of the integers.

negative integers

. . . ,−3,−2,−1,zero

0,positive integers

1, 2, 3, · · · (1.1)

The set of rational numbers is written as {mn |m and n are integers and n 6= 0}.Notice from the de�ni-tion that rational numbers are fractions (or quotients) containing integers in both the numerator and thedenominator, and the denominator is never 0. We can also see that every natural number, whole number,and integer is a rational number with a denominator of 1.

2This content is available online at <http://legacy.cnx.org/content/m51239/1.2/>.

Available for free at Connexions <http://legacy.cnx.org/content/col11759/1.2>

11

Because they are fractions, any rational number can also be expressed in decimal form. Any rationalnumber can be represented as either:

1. a terminating decimal: 158 = 1.875,or

2. a repeating decimal: 411 = 0.36363636 · · · = 0.36

We use a line drawn over the repeating block of numbers instead of writing the group multiple times.

Example 1.1Writing Integers as Rational NumbersWrite each of the following as a rational number.

a. 7b. 0c. �8

SolutionWrite a fraction with the integer in the numerator and 1 in the denominator.

a. 7 = 71

b. 0 = 01

c. −8 = − 81

Try It:

Exercise 1.2.1 (Solution on p. 113.)

Write each of the following as a rational number.

a.11b.3c.�4

Example 1.2Identifying Rational NumbersWrite each of the following rational numbers as either a terminating or repeating decimal.

a. − 57

b. 155

c. 1325

SolutionWrite each fraction as a decimal by dividing the numerator by the denominator.

a. − 57 = −0.714285,a repeating decimal

b. 155 = 3 (or 3.0), a terminating decimal

c. 1325 = 0.52, a terminating decimal

Available for free at Connexions <http://legacy.cnx.org/content/col11759/1.2>

12 CHAPTER 1. PREREQUISITES

Try It:

Exercise 1.2.2 (Solution on p. 113.)

Write each of the following rational numbers as either a terminating or repeating decimal.

a. 6817

b. 813

c.− 1720

1.2.1.1 Irrational Numbers

At some point in the ancient past, someone discovered that not all numbers are rational numbers. A builder,for instance, may have found that the diagonal of a square with unit sides was not 2 or even 3

2 ,but wassomething else. Or a garment maker might have observed that the ratio of the circumference to the diameterof a roll of cloth was a little bit more than 3, but still not a rational number. Such numbers are said tobe irrational because they cannot be written as fractions. These numbers make up the set of irrationalnumbers. Irrational numbers cannot be expressed as a fraction of two integers. It is impossible to describethis set of numbers by a single rule except to say that a number is irrational if it is not rational. So we writethis as shown.

{h|h is not a rational number} (1.2)

Example 1.3Di�erentiating Rational and Irrational NumbersDetermine whether each of the following numbers is rational or irrational. If it is rational, determinewhether it is a terminating or repeating decimal.

a.√

25b. 33

9

c.√

11d. 17

34e. 0.3033033303333 . . .

Solution

a.√

25 : This can be simpli�ed as√

25 = 5.Therefore,√

25 is rational.b. 33

9 : Because it is a fraction, 339 is a rational number. Next, simplify and divide.

339

=

11

)33)93

=113

= 3.6 (1.3)

So, 339 is rational and a repeating decimal.

c.√

11 : This cannot be simpli�ed any further. Therefore,√

11 is an irrational number.d. 17

34 : Because it is a fraction, 1734 is a rational number. Simplify and divide.

1734

=

1

)17)34

2

=12

= 0.5 (1.4)

So, 1734 is rational and a terminating decimal.

Available for free at Connexions <http://legacy.cnx.org/content/col11759/1.2>

13

e. 0.3033033303333 . . . is not a terminating decimal. Also note that there is no repeating patternbecause the group of 3s increases each time. Therefore it is neither a terminating nor arepeating decimal and, hence, not a rational number. It is an irrational number.

Try It:

Exercise 1.2.3 (Solution on p. 113.)

Determine whether each of the following numbers is rational or irrational. If it is rational,determine whether it is a terminating or repeating decimal.

a. 777

b.√

81c.4.27027002700027 . . .d. 91

13

e.√

39

1.2.1.2 Real Numbers

Given any number n, we know that n is either rational or irrational. It cannot be both. The sets of rationaland irrational numbers together make up the set of real numbers. As we saw with integers, the realnumbers can be divided into three subsets: negative real numbers, zero, and positive real numbers. Eachsubset includes fractions, decimals, and irrational numbers according to their algebraic sign (+ or �). Zerois considered neither positive nor negative.

The real numbers can be visualized on a horizontal number line with an arbitrary point chosen as 0,with negative numbers to the left of 0 and positive numbers to the right of 0. A �xed unit distance is thenused to mark o� each integer (or other basic value) on either side of 0. Any real number corresponds to aunique position on the number line.The converse is also true: Each location on the number line correspondsto exactly one real number. This is known as a one-to-one correspondence. We refer to this as the realnumber line as shown in Figure 1.2.

Figure 1.2: The real number line

Example 1.4Classifying Real NumbersClassify each number as either positive or negative and as either rational or irrational. Does thenumber lie to the left or the right of 0 on the number line?

a. − 103

Available for free at Connexions <http://legacy.cnx.org/content/col11759/1.2>

14 CHAPTER 1. PREREQUISITES

b.√

5c. −√

289d. −6πe. 0.615384615384 . . .

Solution

a. − 103 is negative and rational. It lies to the left of 0 on the number line.

b.√

5 is positive and irrational. It lies to the right of 0.

c. −√

289 = −√

172 = −17 is negative and rational. It lies to the left of 0.d. −6π is negative and irrational. It lies to the left of 0.e. 0.615384615384 . . . is a repeating decimal so it is rational and positive. It lies to the right of

0.

Try It:

Exercise 1.2.4 (Solution on p. 113.)

Classify each number as either positive or negative and as either rational or irrational.Does the number lie to the left or the right of 0 on the number line?

a.√

73b.−11.411411411 . . .c. 47

19

d.−√

52

e.6.210735

1.2.1.3 Sets of Numbers as Subsets

Beginning with the natural numbers, we have expanded each set to form a larger set, meaning that there isa subset relationship between the sets of numbers we have encountered so far. These relationships becomemore obvious when seen as a diagram, such as Figure 1.3.

Available for free at Connexions <http://legacy.cnx.org/content/col11759/1.2>

15

Figure 1.3: Sets of numbersN : the set of natural numbersW : the set of whole numbersI : the set of integersQ: the set of rational numbersQ�: the set of irrational numbers

A General Note: The set of natural numbers includes the numbers used forcounting: {1, 2, 3, ...}.The set of whole numbers is the set of natural numbers plus zero: {0, 1, 2, 3, ...}.The set of integers adds the negative natural numbers to the set of wholenumbers: {...,−3,−2,−1, 0, 1, 2, 3, ...}.The set of rational numbers includes fractions written as {mn |m and n are integers and n 6= 0}.The set of irrational numbers is the set of numbers that are not rational, are nonrepeating, andare nonterminating: {h|h is not a rational number}.

Example 1.5Di�erentiating the Sets of NumbersClassify each number as being a natural number (N), whole number (W ), integer (I), rationalnumber (Q), and/or irrational number (Q').

a.√

36b. 8

3

c.√

73d. −6e. 3.2121121112 . . .

Solution

Available for free at Connexions <http://legacy.cnx.org/content/col11759/1.2>

16 CHAPTER 1. PREREQUISITES

N W I Q Q'

a.√

36 = 6 X X X X

b. 83 = 2.6 X

c.√

73 X

d. �6 X X

e. 3.2121121112... X

Table 1.1

Try It:

Exercise 1.2.5 (Solution on p. 113.)

Classify each number as being a natural number (N), whole number (W ), integer (I),rational number (Q), and/or irrational number (Q').

a.− 357

b.0c.√

169d.√

24e.4.763763763 . . .

1.2.2 Performing Calculations Using the Order of Operations

When we multiply a number by itself, we square it or raise it to a power of 2. For example, 42 = 4 ·4 = 16.Wecan raise any number to any power. In general, the exponential notation anmeans that the number orvariable a is used as a factorn times.

an =n factors

a · a · a · · · · · a (1.5)

In this notation, an is read as the nth power of a, where a is called the base andn is called the exponent. Aterm in exponential notation may be part of a mathematical expression, which is a combination of numbersand operations. For example, 24 + 6 · 2

3 − 42 is a mathematical expression.To evaluate a mathematical expression, we perform the various operations. However, we do not perform

them in any random order. We use the order of operations. This is a sequence of rules for evaluatingsuch expressions.

Recall that in mathematics we use parentheses ( ), brackets [ ], and braces { } to group numbers andexpressions so that anything appearing within the symbols is treated as a unit. Additionally, fractionbars, radicals, and absolute value bars are treated as grouping symbols. When evaluating a mathematicalexpression, begin by simplifying expressions within grouping symbols.

The next step is to address any exponents or radicals. Afterward, perform multiplication and divisionfrom left to right and �nally addition and subtraction from left to right.

Let's take a look at the expression provided.

Available for free at Connexions <http://legacy.cnx.org/content/col11759/1.2>

17

24 + 6 · 23− 42 (1.6)

There are no grouping symbols, so we move on to exponents or radicals. The number 4 is raised to a powerof 2, so simplify 42 as 16.

24 + 6 · 23 − 42

24 + 6 · 23 − 16

(1.7)

Next, perform multiplication or division, left to right.

24 + 6 · 23 − 16

24 + 4− 16

(1.8)

Lastly, perform addition or subtraction, left to right.

24 + 4− 16

28− 16

12

(1.9)

Therefore, 24 + 6 · 23 − 42 = 12.

For some complicated expressions, several passes through the order of operations will be needed. Forinstance, there may be a radical expression inside parentheses that must be simpli�ed before the parenthesesare evaluated. Following the order of operations ensures that anyone simplifying the same mathematicalexpression will get the same result.

A General Note: Operations in mathematical expressions must be evaluated in a systematicorder, which can be simpli�ed using the acronym PEMDAS:

P(arentheses)E(xponents)M(ultiplication) and D(ivision)A(ddition) and S(ubtraction)

How To: Given a mathematical expression, simplify it using the order of operations.

Step 1.Simplify any expressions within grouping symbols.Step 2.Simplify any expressions containing exponents or radicals.Step 3.Perform any multiplication and division in order, from left to right.Step 4.Perform any addition and subtraction in order, from left to right.

Example 1.6Using the Order of OperationsUse the order of operations to evaluate each of the following expressions.

a. (3 · 2)2 − 4 (6 + 2)b. 52−4

7 −√

11− 2

Available for free at Connexions <http://legacy.cnx.org/content/col11759/1.2>

18 CHAPTER 1. PREREQUISITES

c. 6− |5− 8|+ 3 (4− 1)d. 14−3·2

2·5−32

e. 7 (5 · 3)− 2[(6− 3)− 42

]+ 1

Solution

a.

(3 · 2)2 − 4 (6 + 2) = (6)2 − 4 (8) Simplify parentheses

= 36− 4 (8) Simplify exponent

= 36− 32 Simplify multiplication

= 4 Simplify subtraction

b.

52

7 −√

11− 2 = 52−47 −

√9 Simplify grouping symbols (radical)

= 52−47 − 3 Simplify radical

= 25−47 − 3 Simplify exponent

= 217 − 3 Simplify subtraction in numerator

= 3− 3 Simplify division

= 0 Simplify subtraction

Note that in

the �rst step, the radical is treated as a grouping symbol, like parentheses. Also, in the thirdstep, the fraction bar is considered a grouping symbol so the numerator is considered to begrouped.

c.

6− |5− 8|+ 3 (4− 1) = 6− | − 3|+ 3 (3) Simplify inside grouping symbols

= 6− 3 + 3 (3) Simplify absolute value

= 6− 3 + 9 Simplify multiplication

= 3 + 9 Simplify subtraction

= 12 Simplify addition

d.

14−3·22·5−32 = 14−3·2

2·5−9 Simplify exponent

= 14−610−9 Simplify products

= 81 Simplify di�erences

= 8 Simplify quotient

In this example, the

fraction bar separates the numerator and denominator, which we simplify separately untilthe last step.

e.

7 (5 · 3)− 2[(6− 3)− 42

]+ 1 = 7 (15)− 2

[(3)− 42

]+ 1 Simplify inside parentheses

= 7 (15)− 2 (3− 16) + 1 Simplify exponent

= 7 (15)− 2 (−13) + 1 Subtract

= 105 + 26 + 1 Multiply

= 132 Add

Try It:

Exercise 1.2.6 (Solution on p. 113.)

Use the order of operations to evaluate each of the following expressions.

Available for free at Connexions <http://legacy.cnx.org/content/col11759/1.2>

19

a.√

52 − 42 + 7(5− 4)2

b.1 + 7·5−8·49−6

c.|1.8− 4.3|+ 0.4√

15 + 10d. 12

[5 · 32 − 72

]+ 1

3 · 92

e.[(3− 8)2 − 4

]− (3− 8)

1.2.3 Using Properties of Real Numbers

For some activities we perform, the order of certain operations does not matter, but the order of otheroperations does. For example, it does not make a di�erence if we put on the right shoe before the left orvice-versa. However, it does matter whether we put on shoes or socks �rst. The same thing is true foroperations in mathematics.

1.2.3.1

1.2.3.1.1 Commutative Properties

The commutative property of addition states that numbers may be added in any order without a�ectingthe sum.

a+ b = b+ a (1.10)

We can better see this relationship when using real numbers.

(−2) + 7 = 5 and 7 + (−2) = 5 (1.11)

Similarly, the commutative property of multiplication states that numbers may be multiplied in anyorder without a�ecting the product.

a · b = b · a (1.12)

Again, consider an example with real numbers.

(−11) · (−4) = 44 and (−4) · (−11) = 44 (1.13)

It is important to note that neither subtraction nor division is commutative. For example, 17− 5 is not thesame as 5− 17. Similarly, 20÷ 5 6= 5÷ 20.

1.2.3.1.2 Associative Properties

The associative property of multiplication tells us that it does not matter how we group numbers whenmultiplying. We can move the grouping symbols to make the calculation easier, and the product remainsthe same.

a (bc) = (ab) c (1.14)

Consider this example.

(3 · 4) · 5 = 60 and 3 · (4 · 5) = 60 (1.15)

The associative property of addition tells us that numbers may be grouped di�erently without a�ectingthe sum.

Available for free at Connexions <http://legacy.cnx.org/content/col11759/1.2>

20 CHAPTER 1. PREREQUISITES

a+ (b+ c) = (a+ b) + c (1.16)

This property can be especially helpful when dealing with negative integers. Consider this example.

[15 + (−9)] + 23 = 29 and 15 + [(−9) + 23] = 29 (1.17)

Are subtraction and division associative? Review these examples.

8− (3− 15) ?= (8− 3)− 15 64÷ (8÷ 4) ?= (64÷ 8)÷ 4

8− (−12) = 5− 15 64÷ 2 ?= 8÷ 4

20 6= 20− 10 32 6= 2

(1.18)

As we can see, neither subtraction nor division is associative.

1.2.3.1.3 Distributive Property

The distributive property states that the product of a factor times a sum is the sum of the factor timeseach term in the sum.

a · (b+ c) = a · b+ a · c (1.19)

This property combines both addition and multiplication (and is the only property to do so). Let us consideran example.

Note that 4 is outside the grouping symbols, so we distribute the 4 by multiplying it by 12, multiplying itby �7, and adding the products.

To be more precise when describing this property, we say that multiplication distributes over addition.The reverse is not true, as we can see in this example.

6 + (3 · 5) ?= (6 + 3) · (6 + 5)

6 + (15) ?= (9) · (11)

21 6= 99

(1.20)

Multiplication does not distribute over subtraction, and division distributes over neither addition nor sub-traction.

A special case of the distributive property occurs when a sum of terms is subtracted.

a− b = a+ (−b) (1.21)

For example, consider the di�erence 12 − (5 + 3) .We can rewrite the di�erence of the two terms12 and (5 + 3) by turning the subtraction expression into addition of the opposite. So instead ofsubtracting (5 + 3) ,we add the opposite.

12 + (−1) · (5 + 3) (1.22)

Available for free at Connexions <http://legacy.cnx.org/content/col11759/1.2>

21

Now, distribute − 1 and simplify the result.

12− (5 + 3) = 12 + (−1) · (5 + 3)

= 12 + [(−1) · 5 + (−1) · 3]

= 12 + (−8)

= 4

(1.23)

This seems like a lot of trouble for a simple sum, but it illustrates a powerful result that will be useful oncewe introduce algebraic terms. To subtract a sum of terms, change the sign of each term and add the results.With this in mind, we can rewrite the last example.

12− (5 + 3) = 12 + (−5− 3)

= 12 + (−8)

= 4

(1.24)

1.2.3.1.4 Identity Properties

The identity property of addition states that there is a unique number, called the additive identity (0)that, when added to a number, results in the original number.

a+ 0 = a (1.25)

The identity property of multiplication states that there is a unique number, called the multiplicativeidentity (1) that, when multiplied by a number, results in the original number.

a · 1 = a (1.26)

For example, we have (−6) + 0 = −6 and 23 ·1 = 23.There are no exceptions for these properties; they workfor every real number, including 0 and 1.

1.2.3.1.5 Inverse Properties

The inverse property of addition states that, for every real number a, there is a unique number, called theadditive inverse (or opposite), denoted−a, that, when added to the original number, results in the additiveidentity, 0.

a+ (−a) = 0 (1.27)

For example, if a = −8,the additive inverse is 8, since (−8) + 8 = 0.The inverse property of multiplication holds for all real numbers except 0 because the reciprocal of

0 is not de�ned. The property states that, for every real number a, there is a unique number, called themultiplicative inverse (or reciprocal), denoted 1

a ,that, when multiplied by the original number, results in themultiplicative identity, 1.

a · 1a

= 1 (1.28)

For example, if a = − 23 ,the reciprocal, denoted

1a ,is −

32 because

a · 1a

=(−2

3

)·(−3

2

)= 1 (1.29)

Available for free at Connexions <http://legacy.cnx.org/content/col11759/1.2>

22 CHAPTER 1. PREREQUISITES

A General Note: The following properties hold for real numbers a, b, and c.

Addition Multiplication

Commutative Property a+ b = b+ a a · b = b · aAssociative Property a+ (b+ c) = (a+ b) + c a (bc) = (ab) c

Distributive Property a · (b+ c) = a · b+ a · cIdentity Property There exists a unique real

number called the additiveidentity, 0, such that, for anyreal number a

a+ 0 = a (1.30)

There exists a unique realnumber called the multiplica-tive identity, 1, such that, forany real number a

a · 1 = a (1.31)

Inverse Property Every real number a has anadditive inverse, or opposite,denoted �a, such that

a+ (−a) = 0 (1.32)

Every nonzero real number ahas a multiplicative inverse,or reciprocal, denoted 1

a ,suchthat

a ·(

1a

)= 1 (1.33)

Table 1.2

Example 1.7Using Properties of Real NumbersUse the properties of real numbers to rewrite and simplify each expression. State which propertiesapply.

a. 3 · 6 + 3 · 4b. (5 + 8) + (−8)c. 6− (15 + 9)d. 4

7 ·(

23 ·

74

)e. 100 · [0.75 + (−2.38)]

Solution

a.

3 · 6 + 3 · 4 = 3 · (6 + 4) Distributive property

= 3 · 10 Simplify

= 30 Simplify

b.

(5 + 8) + (−8) = 5 + [8 + (−8)] Associative property of addition

= 5 + 0 Inverse property of addition

= 5 Identity property of addition

c.

6− (15 + 9) = 6 + [(−15) + (−9)] Distributive property

= 6 + (−24) Simplify

= −18 Simplify

Available for free at Connexions <http://legacy.cnx.org/content/col11759/1.2>

23

d.

47 ·(

23 ·

74

)= 4

7 ·(

74 ·

23

)Commutative property of multiplication

=(

47 ·

74

)· 2

3 Associative property of multiplication

= 1 · 23 Inverse property of multiplication

= 23 Identity property of multiplication

e.

100 · [0.75 + (−2.38)] = 100 · 0.75 + 100 · (−2.38) Distributive property

= 75 + (−238) Simplify

= −163 Simplify

Try It:

Exercise 1.2.7 (Solution on p. 113.)

Use the properties of real numbers to rewrite and simplify each expression. State whichproperties apply.

a.(− 23

5

)·[11 ·

(− 5

23

)]b. 5 · (6.2 + 0.4)c. 18− (7− 15)d. 17

18 + ·[

49 +

(− 17

18

)]e. 6 · (−3) + 6 · 3

1.2.3.2 Evaluating Algebraic Expressions

So far, the mathematical expressions we have seen have involved real numbers only. In mathematics, wemay see expressions such asx+ 5, 4

3πr3,or√

2m3n2. In the expressionx+ 5,5 is called a constant because itdoes not vary and x is called a variable because it does. (In naming the variable, ignore any exponents orradicals containing the variable.) An algebraic expression is a collection of constants and variables joinedtogether by the algebraic operations of addition, subtraction, multiplication, and division.

We have already seen some real number examples of exponential notation, a shorthand method of writingproducts of the same factor. When variables are used, the constants and variables are treated the same way.

(−3)5 = (−3) · (−3) · (−3) · (−3) · (−3) x5 = x · x · x · x · x(2 · 7)3 = (2 · 7) · (2 · 7) · (2 · 7) (yz)3 = (yz) · (yz) · (yz)

(1.34)

In each case, the exponent tells us how many factors of the base to use, whether the base consists of constantsor variables.

Any variable in an algebraic expression may take on or be assigned di�erent values. When that happens,the value of the algebraic expression changes. To evaluate an algebraic expression means to determine thevalue of the expression for a given value of each variable in the expression. Replace each variable in theexpression with the given value, then simplify the resulting expression using the order of operations. Ifthe algebraic expression contains more than one variable, replace each variable with its assigned value andsimplify the expression as before.

Example 1.8Describing Algebraic ExpressionsList the constants and variables for each algebraic expression.

Available for free at Connexions <http://legacy.cnx.org/content/col11759/1.2>

24 CHAPTER 1. PREREQUISITES

(a) x + 5(b) 4

3πr3

(c)√

2m3n2

Solution

Constants Variables

a. x + 5 5 x

b. 43πr

3 43 , π r

c.√

2m3n2 2 m,n

Table 1.3

Try It:

Exercise 1.2.8 (Solution on p. 114.)

List the constants and variables for each algebraic expression.

a.2πr (r + h)b.2(L + W )c.4y3 + y

Example 1.9Evaluating an Algebraic Expression at Di�erent ValuesEvaluate the expression 2x− 7 for each value for x.

a. x = 0b. x = 1c. x = 1

2d. x = −4

Solution

a. Substitute 0 forx.

2x− 7 = 2 (0)− 7

= 0− 7

= −7

(1.35)

b. Substitute 1 forx.

2x− 7 = 2 (1)− 7

= 2− 7

= −5

(1.36)

Available for free at Connexions <http://legacy.cnx.org/content/col11759/1.2>

25

c. Substitute 12 forx.

2x− 7 = 2(

12

)− 7

= 1− 7

= −6

(1.37)

d. Substitute − 4 forx.

2x− 7 = 2 (−4)− 7

= −8− 7

= −15

(1.38)

Try It:

Exercise 1.2.9 (Solution on p. 114.)

Evaluate the expression 11− 3y for each value for y.

a. y = 2b. y = 0c. y = 2

3d. y = −5

Example 1.10Evaluating Algebraic ExpressionsEvaluate each expression for the given values.

a. x+ 5 forx = −5b. t

2t−1 for t = 10c. 4

3πr3 for r = 5

d. a+ ab+ b fora = 11, b = −8e.√

2m3n2 form = 2, n = 3

Solution

a. Substitute − 5 forx.

x+ 5 = (−5) + 5

= 0(1.39)

b. Substitute 10 for t.

t2t−1 = (10)

2(10)−1

= 1020−1

= 1019

(1.40)

Available for free at Connexions <http://legacy.cnx.org/content/col11759/1.2>

26 CHAPTER 1. PREREQUISITES

c. Substitute 5 for r.

43πr

3 = 43π(5)3

= 43π (125)

= 5003 π

(1.41)

d. Substitute 11 for a and �8 for b.

a+ ab+ b = (11) + (11) (−8) + (−8)

= 11− 88− 8

= −85

(1.42)

e. Substitute 2 form and 3 forn.

√2m3n2 =

√2(2)3(3)2

=√

2 (8) (9)

=√

144

= 12

(1.43)

Try It:

Exercise 1.2.10 (Solution on p. 114.)

Evaluate each expression for the given values.

a. y+3y−3 for y = 5

b. 7− 2t for t = −2c. 1

3πr2 for r = 11

d.(p2q)3for p = −2, q = 3

e. 4 (m− n)− 5 (n−m) form = 23 , n = 1

3

1.2.3.3 Formulas

An equation is a mathematical statement indicating that two expressions are equal. The expressions canbe numerical or algebraic. The equation is not inherently true or false, but only a proposition. The valuesthat make the equation true, the solutions, are found using the properties of real numbers and other results.For example, the equation 2x+ 1 = 7 has the unique solutionx = 3 because when we substitute 3 forx in theequation, we obtain the true statement 2 (3) + 1 = 7.

A formula is an equation expressing a relationship between constant and variable quantities. Very often,the equation is a means of �nding the value of one quantity (often a single variable) in terms of anotheror other quantities. One of the most common examples is the formula for �nding the areaA of a circle interms of the radius r of the circle:A = πr2.For any value of r,the areaA can be found by evaluating theexpressionπr2.

Available for free at Connexions <http://legacy.cnx.org/content/col11759/1.2>

27

Example 1.11Using a FormulaA right circular cylinder with radius r and heighth has the surface areaS (in square units) given bythe formulaS = 2πr (r + h) . See Figure 1.4. Find the surface area of a cylinder with radius 6 in.and height 9 in. Leave the answer in terms ofπ.

Figure 1.4: Right circular cylinder

SolutionEvaluate the expression 2πr (r + h) for r = 6 andh = 9.

S = 2πr (r + h)

= 2π (6) [(6) + (9)]

= 2π (6) (15)

= 180π

(1.44)

The surface area is 180π square inches.

Try It:

Exercise 1.2.11 (Solution on p. 114.)

A photograph with length L and width W is placed in a matte of width 8 cen-timeters (cm). The area of the matte (in square centimeters, or cm2) is found to

Available for free at Connexions <http://legacy.cnx.org/content/col11759/1.2>

28 CHAPTER 1. PREREQUISITES

beA = (L+ 16) (W + 16)−L·W. See Figure 1.5. Find the area of a matte for a photographwith length 32 cm and width 24 cm.

Figure 1.5

1.2.3.4 Simplifying Algebraic Expressions

Sometimes we can simplify an algebraic expression to make it easier to evaluate or to use in some other way.To do so, we use the properties of real numbers. We can use the same properties in formulas because theycontain algebraic expressions.

Example 1.12Simplifying Algebraic ExpressionsSimplify each algebraic expression.

a. 3x− 2y + x− 3y − 7b. 2r − 5 (3− r) + 4

Available for free at Connexions <http://legacy.cnx.org/content/col11759/1.2>

29

c.(4t− 5

4s)−(

23 t+ 2s

)d. 2mn− 5m+ 3mn+ n

Solution

a.3x− 2y + x− 3y − 7 = 3x+ x− 2y − 3y − 7 Commutative property of addition

= 4x− 5y − 7 Simplify

b.

2r − 5 (3− r) + 4 = 2r − 15 + 5r + 4 Distributive property

= 2r + 5y − 15 + 4 Commutative property of addition

= 7r − 11 Simplify

c.

4t− 4(t− 5

4s)−(

23 t+ 2s

)= 4t− 5

4s−23 t− 2s Distributive property

= 4t− 23 t−

54s− 2s Commutative property of addition

= 103 t−

134 s Simplify

d.mn− 5m+ 3mn+ n = 2mn+ 3mn− 5m+ n Commutative property of addition

= 5mn− 5m+ n Simplify

Try It:

Exercise 1.2.12 (Solution on p. 114.)

Simplify each algebraic expression.

a. 23y − 2(

43y + z

)b. 5t − 2− 3

t + 1c.4p (q − 1) + q (1− p)d.9r − (s+ 2r) + (6− s)

Example 1.13Simplifying a FormulaA rectangle with lengthL and widthW has a perimeterP given byP = L + W + L + W. Simplifythis expression.

Solution

P = L+W + L+W

P = L+ L+W +W Commutative property of addition

P = 2L+ 2W Simplify

P = 2 (L+W ) Distributive property

(1.45)

Available for free at Connexions <http://legacy.cnx.org/content/col11759/1.2>

30 CHAPTER 1. PREREQUISITES

Try It:

Exercise 1.2.13 (Solution on p. 114.)

If the amountP is deposited into an account paying simple interest r for time t,the totalvalue of the depositA is given byA = P +Prt. Simplify the expression. (This formula willbe explored in more detail later in the course.)

Media: Access these online resources for additional instruction and practice with real numbers.

• Simplify an Expression3

• Evaluate an Expression14

• Evaluate an Expression25

1.2.4 Key Concepts

• Rational numbers may be written as fractions or terminating or repeating decimals. See Example 1.1and Example 1.2.

• Determine whether a number is rational or irrational by writing it as a decimal. See Example 1.3.• The rational numbers and irrational numbers make up the set of real numbers. See Example 1.4. A

number can be classi�ed as natural, whole, integer, rational, or irrational. See Example 1.5.• The order of operations is used to evaluate expressions. See Example 1.6.• The real numbers under the operations of addition and multiplication obey basic rules, known as the

properties of real numbers. These are the commutative properties, the associative properties, thedistributive property, the identity properties, and the inverse properties. See Example 1.7.

• Algebraic expressions are composed of constants and variables that are combined using addition, sub-traction, multiplication, and division. See Example 1.8. They take on a numerical value when evaluatedby replacing variables with constants. See Example 1.9, Example 1.10, and Example 1.12

• Formulas are equations in which one quantity is represented in terms of other quantities. They maybe simpli�ed or evaluated as any mathematical expression. See Example 1.11 and Example 1.13.

1.2.5

1.2.5.1 Verbal

Exercise 1.2.14 (Solution on p. 114.)

Is√

2 an example of a rational terminating, rational repeating, or irrational number? Tell why it�ts that category.

Exercise 1.2.15What is the order of operations? What acronym is used to describe the order of operations, andwhat does it stand for?

Exercise 1.2.16 (Solution on p. 114.)

What do the Associative Properties allow us to do when following the order of operations? Explainyour answer.

3http://openstaxcollege.org/l/simexpress4http://openstaxcollege.org/l/ordofoper15http://openstaxcollege.org/l/ordofoper2

Available for free at Connexions <http://legacy.cnx.org/content/col11759/1.2>

31

1.2.5.2 Numeric

For the following exercises, simplify the given expression.

Exercise 1.2.1710 + 2 × (5− 3)Exercise 1.2.18 (Solution on p. 114.)

6÷ 2−(81÷ 32

)Exercise 1.2.1918 + (6− 8)3

Exercise 1.2.20 (Solution on p. 114.)

−2 ×[16÷ (8− 4)2

]2Exercise 1.2.214− 6 + 2 × 7Exercise 1.2.22 (Solution on p. 115.)

3 (5− 8)Exercise 1.2.234 + 6− 10÷ 2Exercise 1.2.24 (Solution on p. 115.)

12÷ (36÷ 9) + 6Exercise 1.2.25(4 + 5)2 ÷ 3Exercise 1.2.26 (Solution on p. 115.)

3− 12 × 2 + 19Exercise 1.2.272 + 8 × 7÷ 4Exercise 1.2.28 (Solution on p. 115.)

5 + (6 + 4)− 11Exercise 1.2.299− 18÷ 32

Exercise 1.2.30 (Solution on p. 115.)

14 × 3÷ 7− 6Exercise 1.2.319− (3 + 11) × 2Exercise 1.2.32 (Solution on p. 115.)

6 + 2 × 2− 1Exercise 1.2.3364÷ (8 + 4 × 2)Exercise 1.2.34 (Solution on p. 115.)

9 + 4(22)

Exercise 1.2.35(12÷ 3 × 3)2

Exercise 1.2.36 (Solution on p. 115.)

25÷ 52 − 7Exercise 1.2.37(15− 7) × (3− 7)

Available for free at Connexions <http://legacy.cnx.org/content/col11759/1.2>

32 CHAPTER 1. PREREQUISITES

Exercise 1.2.38 (Solution on p. 115.)

2 × 4− 9 (−1)Exercise 1.2.3942 − 25 × 1

5

Exercise 1.2.40 (Solution on p. 115.)

12 (3− 1)÷ 6

1.2.5.3 Algebraic

For the following exercises, solve for the variable.

Exercise 1.2.418 (x+ 3) = 64Exercise 1.2.42 (Solution on p. 115.)

4y + 8 = 2yExercise 1.2.43(11a+ 3)− 18a = −4Exercise 1.2.44 (Solution on p. 115.)

4z − 2z (1 + 4) = 36Exercise 1.2.454y(7− 2)2 = −200Exercise 1.2.46 (Solution on p. 115.)

−(2x)2 + 1 = −3Exercise 1.2.478 (2 + 4)− 15b = b

Exercise 1.2.48 (Solution on p. 115.)

2 (11c− 4) = 36Exercise 1.2.494 (3− 1)x = 4Exercise 1.2.50 (Solution on p. 115.)14

(8w − 42

)= 0

For the following exercises, simplify the expression.

Exercise 1.2.514x+ x (13− 7)Exercise 1.2.52 (Solution on p. 115.)

2y − (4)2y − 11

Exercise 1.2.53a23 (64)− 12a÷ 6Exercise 1.2.54 (Solution on p. 115.)

8b− 4b (3) + 1Exercise 1.2.555l ÷ 3l × (9− 6)Exercise 1.2.56 (Solution on p. 115.)

7z − 3 + z × 62

Exercise 1.2.574 × 3 + 18x÷ 9− 12

Available for free at Connexions <http://legacy.cnx.org/content/col11759/1.2>

33

Exercise 1.2.58 (Solution on p. 115.)

9 (y + 8)− 27Exercise 1.2.59(

96 t− 4

)2

Exercise 1.2.60 (Solution on p. 115.)

6 + 12b− 3 × 6bExercise 1.2.6118y − 2 (1 + 7y)Exercise 1.2.62 (Solution on p. 115.)(

49

)2 × 27xExercise 1.2.638 (3−m) + 1 (−8)Exercise 1.2.64 (Solution on p. 115.)

9x+ 4x (2 + 3)− 4 (2x+ 3x)Exercise 1.2.6552 − 4 (3x)

1.2.5.4 Real-World Applications

For the following exercises, consider this scenario: Fred earns $40 mowing lawns. He spends $10 on mp3s,puts half of what is left in a savings account, and gets another $5 for washing his neighbor's car.

Exercise 1.2.66 (Solution on p. 115.)

Write the expression that represents the number of dollars Fred keeps (and does not put in hissavings account). Remember the order of operations.

Exercise 1.2.67How much money does Fred keep?

For the following exercises, solve the given problem.

Exercise 1.2.68 (Solution on p. 115.)

According to the U.S. Mint, the diameter of a quarter is 0.955 inches. The circumference of thequarter would be the diameter multiplied byπ. Is the circumference of a quarter a whole number,a rational number, or an irrational number?

Exercise 1.2.69Jessica and her roommate, Adriana, have decided to share a change jar for joint expenses. Jessicaput her loose change in the jar �rst, and then Adriana put her change in the jar. We know thatit does not matter in which order the change was added to the jar. What property of additiondescribes this fact?

For the following exercises, consider this scenario: There is a mound of g pounds of gravel in a quarry.Throughout the day, 400 pounds of gravel is added to the mound. Two orders of 600 pounds are sold andthe gravel is removed from the mound. At the end of the day, the mound has 1,200 pounds of gravel.

Exercise 1.2.70 (Solution on p. 115.)

Write the equation that describes the situation.

Exercise 1.2.71Solve for g.

For the following exercise, solve the given problem.

Available for free at Connexions <http://legacy.cnx.org/content/col11759/1.2>

34 CHAPTER 1. PREREQUISITES

Exercise 1.2.72 (Solution on p. 116.)

Ramon runs the marketing department at his company. His department gets a budget every year,and every year, he must spend the entire budget without going over. If he spends less than thebudget, then his department gets a smaller budget the following year. At the beginning of thisyear, Ramon got $2.5 million for the annual marketing budget. He must spend the budget suchthat 2, 500, 000− x = 0.What property of addition tells us what the value of x must be?

1.2.5.5 Technology

For the following exercises, use a graphing calculator to solve for x. Round the answers to the nearesthundredth.

Exercise 1.2.730.5(12.3)2 − 48x = 3

5

Exercise 1.2.74 (Solution on p. 116.)

(0.25− 0.75)2x− 7.2 = 9.9

1.2.5.6 Extensions

Exercise 1.2.75If a whole number is not a natural number, what must the number be?

Exercise 1.2.76 (Solution on p. 116.)

Determine whether the statement is true or false: The multiplicative inverse of a rational numberis also rational.

Exercise 1.2.77Determine whether the statement is true or false: The product of a rational and irrational numberis always irrational.

Exercise 1.2.78 (Solution on p. 116.)

Determine whether the simpli�ed expression is rational or irrational:√−18− 4 (5) (−1).

Exercise 1.2.79Determine whether the simpli�ed expression is rational or irrational:

√−16 + 4 (5) + 5.

Exercise 1.2.80 (Solution on p. 116.)

The division of two whole numbers will always result in what type of number?

Exercise 1.2.81What property of real numbers would simplify the following expression: 4 + 7 (x− 1)?

1.3 Exponents and Scienti�c Notation6

Mathematicians, scientists, and economists commonly encounter very large and very small numbers. But itmay not be obvious how common such �gures are in everyday life. For instance, a pixel is the smallest unitof light that can be perceived and recorded by a digital camera. A particular camera might record an imagethat is 2,048 pixels by 1,536 pixels, which is a very high resolution picture. It can also perceive a color depth(gradations in colors) of up to 48 bits per frame, and can shoot the equivalent of 24 frames per second. Themaximum possible number of bits of information used to �lm a one-hour (3,600-second) digital �lm is thenan extremely large number.

6This content is available online at <http://legacy.cnx.org/content/m51241/1.2/>.

Available for free at Connexions <http://legacy.cnx.org/content/col11759/1.2>

35

Using a calculator, we enter 2, 048 × 1, 536 × 48 × 24 × 3, 600 and press ENTER. The calculator displays1.304596316E13. What does this mean? The �E13� portion of the result represents the exponent 13 of ten,so there are a maximum of approximately 1.3 × 1013 bits of data in that one-hour �lm. In this section, wereview rules of exponents �rst and then apply them to calculations involving very large or small numbers.

1.3.1 Using the Product Rule of Exponents

Consider the productx3 · x4.Both terms have the same base, x, but they are raised to di�erent exponents.Expand each expression, and then rewrite the resulting expression.

x3 · x4 =3 factors

x · x · x ·4 factors

x · x · x · x

=7 factors

x · x · x · x · x · x · x= x7

(1.46)

The result is thatx3 · x4 = x3+4 = x7.Notice that the exponent of the product is the sum of the exponents of the terms. In other words, when

multiplying exponential expressions with the same base, we write the result with the common base and addthe exponents. This is the product rule of exponents.

am · an = am+n (1.47)

Now consider an example with real numbers.

23 · 24 = 23+4 = 27 (1.48)

We can always check that this is true by simplifying each exponential expression. We �nd that 23 is 8, 24 is16, and 27 is 128. The product 8 · 16 equals 128, so the relationship is true. We can use the product rule ofexponents to simplify expressions that are a product of two numbers or expressions with the same base butdi�erent exponents.

A General Note: For any real number a and natural numbersm andn,the product rule ofexponents states that

am · an = am+n (1.49)

Example 1.14Using the Product RuleWrite each of the following products with a single base. Do not simplify further.

a. t5 · t3b. (−3)5 · (−3)c. x2 · x5 · x3

SolutionUse the product rule to simplify each expression.

a. t5 · t3 = t5+3 = t8

b. (−3)5 · (−3) = (−3)5 · (−3)1 = (−3)5+1 = (−3)6

c. x2 · x5 · x3

Available for free at Connexions <http://legacy.cnx.org/content/col11759/1.2>

36 CHAPTER 1. PREREQUISITES

At �rst, it may appear that we cannot simplify a product of three factors. However, using theassociative property of multiplication, begin by simplifying the �rst two.

x2 · x5 · x3 =(x2 · x5

)· x3 =

(x2+5

)· x3 = x7 · x3 = x7+3 = x10 (1.50)

Notice we get the same result by adding the three exponents in one step.

x2 · x5 · x3 = x2+5+3 = x10 (1.51)

Try It:

Exercise 1.3.1 (Solution on p. 116.)

Write each of the following products with a single base. Do not simplify further.

a.k6 · k9

b.(

2y

)4

·(

2y

)c.t3 · t6 · t5

1.3.2 Using the Quotient Rule of Exponents

The quotient rule of exponents allows us to simplify an expression that divides two numbers with the samebase but di�erent exponents. In a similar way to the product rule, we can simplify an expression such

as ym

yn ,wherem > n.Consider the example y9

y5 .Perform the division by canceling common factors.

y9

y5 = y·y·y·y·y·y·y·y·yy·y·y·y·y

= )y·)y·)y·)y·)y·y·y·y·y)y·)y·)y·)y·)y

= y·y·y·y1

= y4

(1.52)

Notice that the exponent of the quotient is the di�erence between the exponents of the divisor and dividend.

am

an= am−n (1.53)

In other words, when dividing exponential expressions with the same base, we write the result with thecommon base and subtract the exponents.

y9

y5= y9−5 = y4 (1.54)

For the time being, we must be aware of the conditionm > n.Otherwise, the di�erencem− n could be zeroor negative. Those possibilities will be explored shortly. Also, instead of qualifying variables as nonzero eachtime, we will simplify matters and assume from here on that all variables represent nonzero real numbers.

Available for free at Connexions <http://legacy.cnx.org/content/col11759/1.2>

37

A General Note: For any real number a and natural numbersm andn,such thatm > n,thequotient rule of exponents states that

am

an= am−n (1.55)

Example 1.15Using the Quotient RuleWrite each of the following products with a single base. Do not simplify further.

a. (−2)14

(−2)9

b. t23

t15

c.(z√

2)5

z√

2

SolutionUse the quotient rule to simplify each expression.

a. (−2)14

(−2)9= (−2)14−9 = (−2)5

b. t23

t15 = t23−15 = t8

c.(z√

2)5

z√

2=(z√

2)5−1

=(z√

2)4

Try It:

Exercise 1.3.2 (Solution on p. 116.)

Write each of the following products with a single base. Do not simplify further.

a. s75

s68

b. (−3)6

−3

c.(ef2)5

(ef2)3

1.3.3 Using the Power Rule of Exponents

Suppose an exponential expression is raised to some power. Can we simplify the result? Yes. To do this,

we use the power rule of exponents. Consider the expression(x2)3.The expression inside the parentheses

is multiplied twice because it has an exponent of 2. Then the result is multiplied three times because theentire expression has an exponent of 3.

(x2)3 =

3 factors(x2)·(x2)·(x2)

=

3 factors2 factors︷︸︸︷x · x

·2 factors︷︸︸︷

x · x

·2 factors︷︸︸︷

x · x

= x · x · x · x · x · x= x6

(1.56)

Available for free at Connexions <http://legacy.cnx.org/content/col11759/1.2>

38 CHAPTER 1. PREREQUISITES

The exponent of the answer is the product of the exponents:(x2)3 = x2·3 = x6. In other words, when raising

an exponential expression to a power, we write the result with the common base and the product of theexponents.

(am)n = am·n (1.57)

Be careful to distinguish between uses of the product rule and the power rule. When using the product rule,di�erent terms with the same bases are raised to exponents. In this case, you add the exponents. Whenusing the power rule, a term in exponential notation is raised to a power. In this case, you multiply theexponents.

Product Rule Power Rule

53 · 54 = 53+4 = 57 but(53)4 = 53·4 =

512

x5 · x2 = x5+2 = x7 but(x5)2 = x5·2 =

x10

(3a)7 · (3a)10 = (3a)7+10 = (3a)17but

((3a)7

)10

= (3a)7·10 =

(3a)70

(1.58)

A General Note: For any real number a and positive integersm andn,the power rule of expo-nents states that

(am)n = am·n (1.59)

Example 1.16Using the Power RuleWrite each of the following products with a single base. Do not simplify further.

a.(x2)7

b.(

(2t)5)3

c.(

(−3)5)11

SolutionUse the power rule to simplify each expression.

a.(x2)7 = x2·7 = x14

b.(

(2t)5)3

= (2t)5·3 = (2t)15

c.(

(−3)5)11

= (−3)5·11 = (−3)55

Try It:

Exercise 1.3.3 (Solution on p. 116.)

Write each of the following products with a single base. Do not simplify further.

Available for free at Connexions <http://legacy.cnx.org/content/col11759/1.2>

39

a.(

(3y)8)3

b.(t5)7

c.(

(−g)4)4

1.3.4 Using the Zero Exponent Rule of Exponents

Return to the quotient rule. We made the condition thatm > n so that the di�erencem − nwould neverbe zero or negative. What would happen ifm = n?In this case, we would use the zero exponent rule ofexponents to simplify the expression to 1. To see how this is done, let us begin with an example.

t8

t8=

)t8

)t8= 1 (1.60)

If we were to simplify the original expression using the quotient rule, we would have

t8

t8= t8−8 = t0 (1.61)

If we equate the two answers, the result is t0 = 1.This is true for any nonzero real number, or any variablerepresenting a real number.

a0 = 1 (1.62)

The sole exception is the expression 00.This appears later in more advanced courses, but for now, we willconsider the value to be unde�ned.

A General Note: For any nonzero real number a,the zero exponent rule of exponents statesthat

a0 = 1 (1.63)

Example 1.17Using the Zero Exponent RuleSimplify each expression using the zero exponent rule of exponents.

a. c3

c3

b. −3x5

x5

c.(j2k)4

(j2k)·(j2k)3

d.5(rs2)2

(rs2)2

SolutionUse the zero exponent and other rules to simplify each expression.

a.

c3

c3 = c3−3

= c3−3

= c3−3

Available for free at Connexions <http://legacy.cnx.org/content/col11759/1.2>

40 CHAPTER 1. PREREQUISITES

b.

−3x5

x5 = −3 · x5

x5

= −3 · x5−5

= −3 · x0

= −3 · 1= −3

c.

(j2k)4

(j2k)·(j2k)3= (j2k)4

(j2k)1+3 Use the product rule in the denominator.

= (j2k)4

(j2k)4Simplify.

=(j2k)4−4

Use the quotient rule.

=(j2k)0

Simplify.

= 1

d.

5(rs2)2

(rs2)2= 5

(rs2)2−2

Use the quotient rule.

= 5(rs2)0

Simplify.

= 5 · 1 Use the zero exponent rule.

= 5 Simplify.

Try It:

Exercise 1.3.4 (Solution on p. 116.)

Simplify each expression using the zero exponent rule of exponents.

a. t7

t7

b.(de2)11

2(de2)11

c.w4·w2

w6

d. t3·t4t2·t5

1.3.5 Using the Negative Rule of Exponents

Another useful result occurs if we relax the condition thatm > n in the quotient rule even further. For

example, can we simplify h3

h5 ?Whenm < n�that is, where the di�erencem − n is negative�we can use thenegative rule of exponents to simplify the expression to its reciprocal.

Divide one exponential expression by another with a larger exponent. Use our example, h3

h5 .

h3

h5 = h·h·hh·h·h·h·h

= )h·)h·)h)h·)h·)h·h·h

= 1h·h

= 1h2

(1.64)

If we were to simplify the original expression using the quotient rule, we would have

Available for free at Connexions <http://legacy.cnx.org/content/col11759/1.2>

41

h3

h5 = h3−5

= h−2(1.65)

Putting the answers together, we haveh−2 = 1h2 .This is true for any nonzero real number, or any variable

representing a nonzero real number.A factor with a negative exponent becomes the same factor with a positive exponent if it is moved across

the fraction bar�from numerator to denominator or vice versa.

a−n = 1an and an = 1

a−n (1.66)

We have shown that the exponential expression an is de�ned whenn is a natural number, 0, or the negativeof a natural number. That means that an is de�ned for any integern.Also, the product and quotient rulesand all of the rules we will look at soon hold for any integern.

A General Note: For any nonzero real number a and natural numbern,the negative rule ofexponents states that

a−n =1an

(1.67)

Example 1.18Using the Negative Exponent RuleWrite each of the following quotients with a single base. Do not simplify further. Write answerswith positive exponents.

a. θ3

θ10

b. z2·zz4

c.(−5t3)4

(−5t3)8

Solution

a. θ3

θ10 = θ3−10 = θ−7 = 1θ7

b. z2·zz4 = z2+1

z4 = z3

z4 = z3−4 = z−1 = 1z

c.(−5t3)4

(−5t3)8=(−5t3

)4−8 =(−5t3

)−4 = 1(−5t3)4

Try It:

Exercise 1.3.5 (Solution on p. 116.)

Write each of the following quotients with a single base. Do not simplify further. Writeanswers with positive exponents.

a. (−3t)2

(−3t)8

b. f47

f49·f

c. 2k4

5k7

Available for free at Connexions <http://legacy.cnx.org/content/col11759/1.2>

42 CHAPTER 1. PREREQUISITES

Example 1.19Using the Product and Quotient RulesWrite each of the following products with a single base. Do not simplify further. Write answerswith positive exponents.

a. b2 · b−8

b. (−x)5 · (−x)−5

c. −7z(−7z)5

Solution

a. b2 · b−8 = b2−8 = b−6 = 1b6

b. (−x)5 · (−x)−5 = (−x)5−5 = (−x)0 = 1c. −7z

(−7z)5= (−7z)1

(−7z)5= (−7z)1−5 = (−7z)−4 = 1

(−7z)4

Try It:

Exercise 1.3.6 (Solution on p. 116.)

Write each of the following products with a single base. Do not simplify further. Writeanswers with positive exponents.

a.t−11 · t6b. 2512

2513

1.3.6 Finding the Power of a Product

To simplify the power of a product of two exponential expressions, we can use the power of a product rule ofexponents, which breaks up the power of a product of factors into the product of the powers of the factors.For instance, consider (pq)3

.We begin by using the associative and commutative properties of multiplicationto regroup the factors.

(pq)3 =3 factors

(pq) · (pq) · (pq)= p · q · p · q · p · q

=3 factors

p · p · p ·3 factors

q · q · q= p3 · q3

(1.68)

In other words, (pq)3 = p3 · q3.

A General Note: For any real numbers a and b and any integern,the power of a product rule ofexponents states that

(ab)n = anbn (1.69)

Available for free at Connexions <http://legacy.cnx.org/content/col11759/1.2>

43

Example 1.20Using the Power of a Product RuleSimplify each of the following products as much as possible using the power of a product rule.Write answers with positive exponents.

a.(ab2)3

b. (2t)15

c.(−2w3

)3d. 1

(−7z)4

e.(e−2f2

)7SolutionUse the product and quotient rules and the new de�nitions to simplify each expression.

a.(ab2)3 = (a)3 ·

(b2)3 = a1·3 · b2·3 = a3b6

b. 2t15 = (2)15 · (t)15 = 215t15 = 32, 768t15

c.(−2w3

)3 = (−2)3 ·(w3)3 = −8 · w3·3 = −8w9

d. 1(−7z)4

= 1(−7)4·(z)4 = 1

2,401z4

e.(e−2f2

)7 =(e−2)7 · (f2

)7 = e−2·7 · f2·7 = e−14f14 = f14

e14

Try It:

Exercise 1.3.7 (Solution on p. 116.)

Simplify each of the following products as much as possible using the power of a productrule. Write answers with positive exponents.

a.(g2h3

)5b.(5t)3

c.(−3y5

)3d. 1

(a6b7)3

e.(r3s−2

)4

1.3.7 Finding the Power of a Quotient

To simplify the power of a quotient of two expressions, we can use the power of a quotient rule, which statesthat the power of a quotient of factors is the quotient of the powers of the factors. For example, let's lookat the following example.

(e−2f2

)7=f14

e14(1.70)

Let's rewrite the original problem di�erently and look at the result.

(e−2f2

)7 =(f2

e2

)7

= f14

e14

(1.71)

Available for free at Connexions <http://legacy.cnx.org/content/col11759/1.2>

44 CHAPTER 1. PREREQUISITES

It appears from the last two steps that we can use the power of a product rule as a power of a quotient rule.

(e−2f2

)7 =(f2

e2

)7

= (f2)7

(e2)7

= f2·7

e2·7

= f14

e14

(1.72)

A General Note: For any real numbers a and b and any integern,the power of a quotient ruleof exponents states that

(ab

)n=an

bn(1.73)

Example 1.21Using the Power of a Quotient RuleSimplify each of the following quotients as much as possible using the power of a quotient rule.Write answers with positive exponents.

a.(

4z11

)3b.(pq3

)6

c.(−1t2

)27

d.(j3k−2

)4e.(m−2n−2

)3Solution

a.(

4z11

)3 = (4)3

(z11)3= 64

z11·3 = 64z33

b.(pq3

)6

= (p)6

(q3)6= p1·6

q3·6 = p6

q18

c.(−1t2

)27 = (−1)27

(t2)27= −1

t2·27 = −1t54 = − 1

t54

d.(j3k−2

)4 =(j3

k2

)4

= (j3)4

(k2)4= j3·4

k2·4 = j12

k8

e.(m−2n−2

)3 =(

1m2n2

)3 = (1)3

(m2n2)3= 1

(m2)3(n2)3= 1

m2·3·n2·3 = 1m6n6

Try It:

Exercise 1.3.8 (Solution on p. 117.)

Simplify each of the following quotients as much as possible using the power of a quotientrule. Write answers with positive exponents.

a.(b5

c

)3

b.(

5u8

)4c.(−1w3

)35

Available for free at Connexions <http://legacy.cnx.org/content/col11759/1.2>

45

d.(p−4q3

)8e.(c−5d−3

)4

1.3.8 Simplifying Exponential Expressions

Recall that to simplify an expression means to rewrite it by combing terms or exponents; in other words, towrite the expression more simply with fewer terms. The rules for exponents may be combined to simplifyexpressions.

Example 1.22Simplifying Exponential ExpressionsSimplify each expression and write the answer with positive exponents only.

a.(6m2n−1

)3b. 175 · 17−4 · 17−3

c.(u−1vv−1

)2

d.(−2a3b−1

) (5a−2b2

)e.(x2√

2)4(

x2√

2)−4

f.(3w2)5

(6w−2)2

Solution

a.

(6m2n−1

)3 = (6)3(m2)3(

n−1)3

The power of a product rule

= 63m2·3n−1·3 The power rule

= 216m6n−3 Simplify.

= 216m6

n3 The negative exponent rule

b.

175 · 17−4 · 17−3 = 175−4−3 The product rule

= 17−2 Simplify.

= 1172 or 1

289 The negative exponent rule

c.

(u−1vv−1

)2

= (u−1v)2

(v−1)2The power of a quotient rule

= u−2v2

v−2 The power of a product rule

= u−2v2−(−2) The quotient rule

= u−2v4 Simplify.

= v4

u2 The negative exponent rule

d.

(−2a3b−1

) (5a−2b2

)= −2 · 5 · a3 · a−2 · b−1 · b2 Commutative and associative laws of multiplication

= −10 · a3−2 · b−1+2 The product rule

= −10ab Simplify.

e.

(x2√

2)4(

x2√

2)−4

=(x2√

2)4−4

The product rule

=(x2√

2)0

Simplify.

= 1 The zero exponent rule

Available for free at Connexions <http://legacy.cnx.org/content/col11759/1.2>

46 CHAPTER 1. PREREQUISITES

f.

(3w2)5

(6w−2)2=

(3)5·(w2)5

(6)2·(w−2)2The power of a product rule

= 35w2·5

62w−2·2 The power rule

= 243w10

36w−4 Simplify.

= 27w10−(−4)

4 The quotient rule and reduce fraction

= 27w14

4 Simplify.

Try It:

Exercise 1.3.9 (Solution on p. 117.)

Simplify each expression and write the answer with positive exponents only.

a.(2uv−2

)−3

b.x8 · x−12 · xc.(e2f−3

f−1

)2

d.(9r−5s3

) (3r6s−4

)e.(

49 tw

−2)−3( 4

9 tw−2)3

f.(2h2k)4

(7h−1k2)2

1.3.9 Using Scienti�c Notation

Recall at the beginning of the section that we found the number 1.3× 1013 when describing bits of informationin digital images. Other extreme numbers include the width of a human hair, which is about 0.00005 m, andthe radius of an electron, which is about 0.00000000000047 m. How can we e�ectively work read, compare,and calculate with numbers such as these?

A shorthand method of writing very small and very large numbers is called scienti�c notation, in whichwe express numbers in terms of exponents of 10. To write a number in scienti�c notation, move the decimalpoint to the right of the �rst digit in the number. Write the digits as a decimal number between 1 and 10.Count the number of places n that you moved the decimal point. Multiply the decimal number by 10 raisedto a power of n. If you moved the decimal left as in a very large number,n is positive. If you moved thedecimal right as in a small large number,n is negative.

For example, consider the number 2,780,418. Move the decimal left until it is to the right of the �rstnonzero digit, which is 2.

Weobtain 2.780418 by moving the decimal point 6 places to the left. Therefore, the exponent of 10 is 6,and it is positive because we moved the decimal point to the left. This is what we should expect for a largenumber.

2.780418 × 106 (1.74)

Available for free at Connexions <http://legacy.cnx.org/content/col11759/1.2>

47

Working with small numbers is similar. Take, for example, the radius of an electron, 0.00000000000047 m.Perform the same series of steps as above, except move the decimal point to the right.

Becareful not to include the leading 0 in your count. We move the decimal point 13 places to the right,so the exponent of 10 is 13. The exponent is negative because we moved the decimal point to the right. Thisis what we should expect for a small number.

4.7 × 10−13 (1.75)

A General Note: A number is written in scienti�c notation if it is written in the form a ×10n,where 1 ≤ |a| < 10 andn is an integer.

Example 1.23Converting Standard Notation to Scienti�c NotationWrite each number in scienti�c notation.

a. Distance to Andromeda Galaxy from Earth: 24,000,000,000,000,000,000,000 mb. Diameter of Andromeda Galaxy: 1,300,000,000,000,000,000,000 mc. Number of stars in Andromeda Galaxy: 1,000,000,000,000d. Diameter of electron: 0.00000000000094 me. Probability of being struck by lightning in any single year: 0.00000143

Solution

a.

24, 000, 000, 000, 000, 000, 000, 000 m

24, 000, 000, 000, 000, 000, 000, 000 m

←22 places

2.4 × 1022 m

b.

1, 300, 000, 000, 000, 000, 000, 000 m

1, 300, 000, 000, 000, 000, 000, 000 m

←21 places

1.3 × 1021 m

c.

1, 000, 000, 000, 000

1, 000, 000, 000, 000

←12 places

1 × 1012

d.

0.00000000000094 m

0.00000000000094 m

→6 places

9.4 × 10−13 m

Available for free at Connexions <http://legacy.cnx.org/content/col11759/1.2>

48 CHAPTER 1. PREREQUISITES

e.

0.00000143

0.00000143→6 places

1.43 × 10−6

AnalysisObserve that, if the given number is greater than 1, as in examples a�c, the exponent of 10 ispositive; and if the number is less than 1, as in examples d�e, the exponent is negative.

Try It:

Exercise 1.3.10 (Solution on p. 117.)

Write each number in scienti�c notation.

a.U.S. national debt per taxpayer (April 2014): $152,000b.World population (April 2014): 7,158,000,000c.World gross national income (April 2014): $85,500,000,000,000d.Time for light to travel 1 m: 0.00000000334 se.Probability of winning lottery (match 6 of 49 possible numbers): 0.0000000715

1.3.9.1 Converting from Scienti�c to Standard Notation

To convert a number in scienti�c notation to standard notation, simply reverse the process. Move thedecimaln places to the right ifn is positive orn places to the left ifn is negative and add zeros as needed.Remember, ifn is positive, the value of the number is greater than 1, and ifn is negative, the value of thenumber is less than one.

Example 1.24Converting Scienti�c Notation to Standard NotationConvert each number in scienti�c notation to standard notation.

a. 3.547 × 1014

b. −2 × 106

c. 7.91 × 10−7

d. −8.05 × 10−12

Solution

a.

3.547 × 1014

3.54700000000000→14 places

354, 700, 000, 000, 000

b.

−2 × 106

−2.000000→6 places

−2, 000, 000

Available for free at Connexions <http://legacy.cnx.org/content/col11759/1.2>

49

c.

7.91 × 10−7

0000007.91→7 places

0.000000791

d.

−8.05 × 10−12

−000000000008.05→12 places

−0.00000000000805

Try It:

Exercise 1.3.11 (Solution on p. 117.)

Convert each number in scienti�c notation to standard notation.

a.7.03 × 105

b.−8.16 × 1011

c.−3.9 × 10−13

d.8 × 10−6

1.3.9.2 Using Scienti�c Notation in Applications

Scienti�c notation, used with the rules of exponents, makes calculating with large or small numbers mucheasier than doing so using standard notation. For example, suppose we are asked to calculate the number ofatoms in 1 L of water. Each water molecule contains 3 atoms (2 hydrogen and 1 oxygen). The average drop ofwater contains around 1.32 × 1021 molecules of water and 1 L of water holds about 1.22 × 104 average drops.Therefore, there are approximately 3 ·

(1.32 × 1021

)·(1.22 × 104

)≈ 4.83 × 1025 atoms in 1 L of water.

We simply multiply the decimal terms and add the exponents. Imagine having to perform the calculationwithout using scienti�c notation!

When performing calculations with scienti�c notation, be sure to write the answer in proper scienti�cnotation. For example, consider the product

(7 × 104

)·(5 × 106

)= 35 × 1010.The answer is not in proper

scienti�c notation because 35 is greater than 10. Consider 35 as 3.5 × 10.That adds a ten to the exponentof the answer.

(35) × 1010 = (3.5 × 10) × 1010 = 3.5 ×(10 × 1010

)= 3.5 × 1011 (1.76)

Example 1.25Using Scienti�c NotationPerform the operations and write the answer in scienti�c notation.

a.(8.14 × 10−7

) (6.5 × 1010

)b.(4 × 105

)÷(−1.52 × 109

)c.(2.7 × 105

) (6.04 × 1013

)d.(1.2 × 108

)÷(9.6 × 105

)e.(3.33 × 104

) (−1.05 × 107

) (5.62 × 105

)Available for free at Connexions <http://legacy.cnx.org/content/col11759/1.2>

50 CHAPTER 1. PREREQUISITES

Solution

a.

(8.14 × 10−7

) (6.5 × 1010

)= (8.14 × 6.5)

(10−7 × 1010

) Commutative and associative

properties of multiplication

= (52.91)(103)

Product rule of exponents

= 5.291 × 104 Scienti�c notation

b.

(4 × 105

)÷(−1.52 × 109

)=

(4

−1.52

)(105

109

) Commutative and associative

properties of multiplication

= (−2.63)(10−4

)Quotient rule of exponents

= −2.63 × 10−4 Scienti�c notation

c.

(2.7 × 105

) (6.04 × 1013

)= (2.7 × 6.04)

(105 × 1013

) Commutative and associative

properties of multiplication

= (16.308)(1018

)Product rule of exponents

= 1.6308 × 1019 Scienti�c notation

d.

(1.2 × 108

)÷(9.6 × 105

)=

(1.29.6

) (108

105

) Commutative and associative

properties of multiplication

= (0.125)(103)

Quotient rule of exponents

= 1.25 × 102 Scienti�c notation

e.

(3.33 × 104

) (−1.05 × 107

) (5.62 × 105

)= [3.33 × (−1.05) × 5.62]

(104 × 107 × 105

)≈ (−19.65)

(1016

)= −1.965 × 1017

Try It:

Exercise 1.3.12 (Solution on p. 117.)

Perform the operations and write the answer in scienti�c notation.

a.(−7.5 × 108

) (1.13 × 10−2

)b.(1.24 × 1011

)÷(1.55 × 1018

)c.(3.72 × 109

) (8 × 103

)d.(9.933 × 1023

)÷(−2.31 × 1017

)e.(−6.04 × 109

) (7.3 × 102

) (−2.81 × 102

)Example 1.26Applying Scienti�c Notation to Solve ProblemsIn April 2014, the population of the United States was about 308,000,000 people. The nationaldebt was about $17,547,000,000,000. Write each number in scienti�c notation, rounding �gures totwo decimal places, and �nd the amount of the debt per U.S. citizen. Write the answer in bothscienti�c and standard notations.

SolutionThe population was 308, 000, 000 = 3.08 × 108.

Available for free at Connexions <http://legacy.cnx.org/content/col11759/1.2>

51

The national debt was $17, 547, 000, 000, 000 ≈ $1.75 × 1013.To �nd the amount of debt per citizen, divide the national debt by the number of citizens.

(1.75 × 1013

)÷(3.08 × 108

)=

(1.753.08

)·(

1013

108

)≈ 0.57 × 105

= 5.7 × 104

(1.77)

The debt per citizen at the time was about $5.7 × 104,or $57,000.

Try It:

Exercise 1.3.13 (Solution on p. 117.)

An average human body contains around 30,000,000,000,000 red blood cells. Each cellmeasures approximately 0.000008 m long. Write each number in scienti�c notation and�nd the total length if the cells were laid end-to-end. Write the answer in both scienti�cand standard notations.

Media: Access these online resources for additional instruction and practice with exponents andscienti�c notation.

• Exponential Notation7• Properties of Exponents8• Zero Exponent9• Simplify Exponent Expressions10

• Quotient Rule for Exponents11• Scienti�c Notation12• Converting to Decimal Notation13

1.3.10 Key Equations

Rules of ExponentsFor nonzero real numbers a and b and integersm andn

Product rule am · an = am+n

Quotient rule am

an = am−n

Power rule (am)n = am·n

Zero exponent rule a0 = 1

Negative rule a−n = 1an

Power of a product rule (a · b)n = an · bn

Power of a quotient rule(ab

)n = an

bn

7http://openstaxcollege.org/l/exponnot8http://openstaxcollege.org/l/exponprops9http://openstaxcollege.org/l/zeroexponent

10http://openstaxcollege.org/l/exponexpres11http://openstaxcollege.org/l/quotofexpon12http://openstaxcollege.org/l/scienti�cnota13http://openstaxcollege.org/l/decimalnota

Available for free at Connexions <http://legacy.cnx.org/content/col11759/1.2>

52 CHAPTER 1. PREREQUISITES

Table 1.4

1.3.11 Key Concepts

• Products of exponential expressions with the same base can be simpli�ed by adding exponents. SeeExample 1.14.

• Quotients of exponential expressions with the same base can be simpli�ed by subtracting exponents.See Example 1.15.

• Powers of exponential expressions with the same base can be simpli�ed by multiplying exponents. SeeExample 1.16.

• An expression with exponent zero is de�ned as 1. See Example 1.17.• An expression with a negative exponent is de�ned as a reciprocal. See Example 1.18 and Example 1.19.• The power of a product of factors is the same as the product of the powers of the same factors. See

Example 1.20.• The power of a quotient of factors is the same as the quotient of the powers of the same factors. See

Example 1.21.• The rules for exponential expressions can be combined to simplify more complicated expressions. See

Example 1.22.• Scienti�c notation uses powers of 10 to simplify very large or very small numbers. See Example 1.23

and Example 1.24.• Scienti�c notation may be used to simplify calculations with very large or very small numbers. See

Example 1.25 and Example 1.26.

1.3.12 Section Exercises

1.3.12.1 Verbal

Exercise 1.3.14 (Solution on p. 117.)

Is 23 the same as 32?Explain.Exercise 1.3.15When can you add two exponents?

Exercise 1.3.16 (Solution on p. 117.)

What is the purpose of scienti�c notation?

Exercise 1.3.17Explain what a negative exponent does.

1.3.12.2 Numeric

For the following exercises, simplify the given expression. Write answers with positive exponents.

Exercise 1.3.18 (Solution on p. 117.)

92

Exercise 1.3.1915−2

Exercise 1.3.20 (Solution on p. 117.)

32 × 33

Exercise 1.3.2144 ÷ 4

Available for free at Connexions <http://legacy.cnx.org/content/col11759/1.2>

53

Exercise 1.3.22 (Solution on p. 117.)(22)−2

Exercise 1.3.23(5− 8)0

Exercise 1.3.24 (Solution on p. 117.)

113 ÷ 114

Exercise 1.3.2565 × 6−7

Exercise 1.3.26 (Solution on p. 117.)(80)2

Exercise 1.3.275−2 ÷ 52

For the following exercises, write each expression with a single base. Do not simplify further. Write answerswith positive exponents.

Exercise 1.3.28 (Solution on p. 118.)

42 × 43 ÷ 4−4

Exercise 1.3.29612

69

Exercise 1.3.30 (Solution on p. 118.)(123 × 12

)10

Exercise 1.3.31106 ÷

(1010

)−2

Exercise 1.3.32 (Solution on p. 118.)

7−6 × 7−3

Exercise 1.3.33(33 ÷ 34

)5For the following exercises, express the decimal in scienti�c notation.

Exercise 1.3.34 (Solution on p. 118.)

0.0000314

Exercise 1.3.35148,000,000

For the following exercises, convert each number in scienti�c notation to standard notation.

Exercise 1.3.36 (Solution on p. 118.)

1.6 × 1010

Exercise 1.3.379.8 × 10−9

Available for free at Connexions <http://legacy.cnx.org/content/col11759/1.2>

54 CHAPTER 1. PREREQUISITES

1.3.12.3 Algebraic

For the following exercises, simplify the given expression. Write answers with positive exponents.

Exercise 1.3.38 (Solution on p. 118.)a3a2

a

Exercise 1.3.39mn2

m−2

Exercise 1.3.40 (Solution on p. 118.)(b3c4

)2Exercise 1.3.41(x−3

y2

)−5

Exercise 1.3.42 (Solution on p. 118.)

ab2 ÷ d−3

Exercise 1.3.43(w0x5

)−1

Exercise 1.3.44 (Solution on p. 118.)m4

n0

Exercise 1.3.45y−4

(y2)2

Exercise 1.3.46 (Solution on p. 118.)p−4q2

p2q−3

Exercise 1.3.47(l × w)2

Exercise 1.3.48 (Solution on p. 118.)(y7)3 ÷ x14

Exercise 1.3.49(a23

)2Exercise 1.3.50 (Solution on p. 118.)

52m÷ 50m

Exercise 1.3.51(16√x)2

y−1

Exercise 1.3.52 (Solution on p. 118.)23

(3a)−2

Exercise 1.3.53(ma6

)2 1m3a2

Exercise 1.3.54 (Solution on p. 118.)(b−3c

)3Exercise 1.3.55(x2y13 ÷ y0

)2Exercise 1.3.56 (Solution on p. 118.)(9z3)−2

y

Available for free at Connexions <http://legacy.cnx.org/content/col11759/1.2>

55

1.3.12.4 Real-World Applications

Exercise 1.3.57To reach escape velocity, a rocket must travel at the rate of 2.2 × 106 ft/min. Rewrite the rate instandard notation.

Exercise 1.3.58 (Solution on p. 118.)

A dime is the thinnest coin in U.S. currency. A dime's thickness measures 2.2 × 106 m. Rewritethe number in standard notation.

Exercise 1.3.59The average distance between Earth and the Sun is 92,960,000 mi. Rewrite the distance usingscienti�c notation.

Exercise 1.3.60 (Solution on p. 118.)

A terabyte is made of approximately 1,099,500,000,000 bytes. Rewrite in scienti�c notation.

Exercise 1.3.61The Gross Domestic Product (GDP) for the United States in the �rst quarter of 2014 was $1.71496×1013.Rewrite the GDP in standard notation.

Exercise 1.3.62 (Solution on p. 118.)

One picometer is approximately 3.397 × 10−11 in. Rewrite this length using standard notation.

Exercise 1.3.63The value of the services sector of the U.S. economy in the �rst quarter of 2012 was $10,633.6billion. Rewrite this amount in scienti�c notation.

1.3.12.5 Technology

For the following exercises, use a graphing calculator to simplify. Round the answers to the nearest hundredth.

Exercise 1.3.64 (Solution on p. 118.)(123m33

4−3

)2

Exercise 1.3.65173 ÷ 152x3

1.3.12.6 Extensions

For the following exercises, simplify the given expression. Write answers with positive exponents.

Exercise 1.3.66 (Solution on p. 118.)(32

a3

)−2(a4

22

)2

Exercise 1.3.67(62 − 24

)2 ÷ (xy)−5

Exercise 1.3.68 (Solution on p. 118.)m2n3

a2c−3 · a−7n−2

m2c4

Exercise 1.3.69(x6y3

x3y−3 · y−7

x−3

)10

Exercise 1.3.70 (Solution on p. 118.)((ab2c)−3

b−3

)2

Available for free at Connexions <http://legacy.cnx.org/content/col11759/1.2>

56 CHAPTER 1. PREREQUISITES

Exercise 1.3.71Avogadro's constant is used to calculate the number of particles in a mole. A mole is a basicunit in chemistry to measure the amount of a substance. The constant is 6.0221413 × 1023.WriteAvogadro's constant in standard notation.

Exercise 1.3.72 (Solution on p. 118.)

Planck's constant is an important unit of measure in quantum physics. It describes the relationshipbetween energy and frequency. The constant is written as 6.62606957 × 10−34.Write Planck'sconstant in standard notation.

1.4 Radicals and Rational Expressions14

A hardware store sells 16-ft ladders and 24-ft ladders. A window is located 12 feet above the ground. Aladder needs to be purchased that will reach the window from a point on the ground 5 feet from the building.To �nd out the length of ladder needed, we can draw a right triangle as shown in Figure 1.6, and use thePythagorean Theorem.

Figure 1.6

14This content is available online at <http://legacy.cnx.org/content/m51242/1.2/>.

Available for free at Connexions <http://legacy.cnx.org/content/col11759/1.2>

57

a2 + b2 = c2

52 + 122 = c2

169 = c2

(1.78)

Now, we need to �nd out the length that, when squared, is 169, to determine which ladder to choose. Inother words, we need to �nd a square root. In this section, we will investigate methods of �nding solutionsto problems such as this one.

1.4.1 Evaluating Square Roots

When the square root of a number is squared, the result is the original number. Since 42 = 16,the square rootof 16 is 4.The square root function is the inverse of the squaring function just as subtraction is the inverse ofaddition. To undo squaring, we take the square root.

In general terms, if a is a positive real number, then the square root of a is a number that, when multipliedby itself, gives a.The square root could be positive or negative because multiplying two negative numbersgives a positive number. The principal square root is the nonnegative number that when multiplied byitself equals a.The square root obtained using a calculator is the principal square root.

The principal square root of a is written as√a.The symbol is called a radical, the term under the symbol

is called the radicand, and the entire expression is called a radical expression.

A General Note: The principal square root of a is the nonnegative number that, whenmultiplied by itself, equals a. It is written as a radical expression, with a symbol called a radicalover the term called the radicand:

√a.

Q&A: Does√

25 = ±5?

No. Although both 52 and (−5)2are 25,the radical symbol implies only a nonnegative root, the

principal square root. The principal square root of 25 is√

25 = 5.

Example 1.27Evaluating Square RootsEvaluate each expression.

a.√

100b.√√

16c.√

25 + 144d.√

49−√

81

Solution

a.√

100 = 10 because 102 = 100b.√√

16 =√

4 = 2 because 42 = 16 and 22 = 4c.√

25 + 144 =√

169 = 13 because 132 = 169d.√

49−√

81 = 7− 9 = −2 because 72 = 49 and 92 = 81

Available for free at Connexions <http://legacy.cnx.org/content/col11759/1.2>

58 CHAPTER 1. PREREQUISITES

Q&A: For√

25 + 144,can we �nd the square roots before adding?

No.√

25 +√

144 = 5 + 12 = 17.This is not equivalent to√

25 + 144 = 13.The order of operationsrequires us to add the terms in the radicand before �nding the square root.

Try It:

Exercise 1.4.1 (Solution on p. 118.)

Evaluate each expression.

a.√

225b.√√

81c.√

25− 9d.√

36 +√

121

1.4.2 Using the Product Rule to Simplify Square Roots

To simplify a square root, we rewrite it such that there are no perfect squares in the radicand. There areseveral properties of square roots that allow us to simplify complicated radical expressions. The �rst rulewe will look at is the product rule for simplifying square roots, which allows us to separate the square rootof a product of two numbers into the product of two separate rational expressions. For instance, we canrewrite

√15 as

√3·√

5.We can also use the product rule to express the product of multiple radical expressionsas a single radical expression.

A General Note: If a and b are nonnegative, the square root of the product ab is equal to theproduct of the square roots of a and b.

√ab =

√a ·√b (1.79)

How To: Given a square root radical expression, use the product rule to simplify it.

1.Factor any perfect squares from the radicand.2.Write the radical expression as a product of radical expressions.3.Simplify.

Example 1.28Using the Product Rule to Simplify Square RootsSimplify the radical expression.

a.√

300b.√

162a5b4

Solution

a.

√100 · 3 Factor perfect square from radicand.

√100 ·

√3 Write radical expression as product of radical expressions.

10√

3 Simplify.

Available for free at Connexions <http://legacy.cnx.org/content/col11759/1.2>

59

b.

√81a4b4 · 2a Factor perfect square from radicand.

√81a4b4 ·

√2a Write radical expression as product of radical expressions.

9a2b2√

2a Simplify.

Try It:

Exercise 1.4.2 (Solution on p. 119.)

Simplify√

50x2y3z.

How To: Given the product of multiple radical expressions, use the product rule tocombine them into one radical expression.

1.Express the product of multiple radical expressions as a single radical expression.2.Simplify.

Example 1.29Using the Product Rule to Simplify the Product of Multiple Square RootsSimplify the radical expression.√12 ·√

3

Solution√12 · 3 Express the product as a single radical expression.√

36 Simplify.

6

Try It:

Exercise 1.4.3 (Solution on p. 119.)

Simplify√

50x ·√

2x assumingx > 0.

1.4.3 Using the Quotient Rule to Simplify Square Roots

Just as we can rewrite the square root of a product as a product of square roots, so too can we rewrite thesquare root of a quotient as a quotient of square roots, using the quotient rule for simplifying square roots.It can be helpful to separate the numerator and denominator of a fraction under a radical so that we can

take their square roots separately. We can rewrite√

52 as

√5√2.

A General Note: The square root of the quotient ab is equal to the quotient of the square rootsof a and b,where b 6= 0.

√a

b=√a√b

(1.80)

Available for free at Connexions <http://legacy.cnx.org/content/col11759/1.2>

60 CHAPTER 1. PREREQUISITES

How To: Given a radical expression, use the quotient rule to simplify it.

1.Write the radical expression as the quotient of two radical expressions.2.Simplify the numerator and denominator.

Example 1.30Using the Quotient Rule to Simplify Square RootsSimplify the radical expression.√

536

Solution√5√36

Write as quotient of two radical expressions.√

56 Simplify denominator.

Try It:

Exercise 1.4.4 (Solution on p. 119.)

Simplify√

2x2

9y4 .

Example 1.31Using the Quotient Rule to Simplify an Expression with Two Square RootsSimplify the radical expression.√

234x11y√26x7y

Solution√234x11y26x7y Combine numerator and denominator into one radical expression.√

9x4 Simplify fraction.

3x2 Simplify square root.

Try It:

Exercise 1.4.5 (Solution on p. 119.)

Simplify√

9a5b14√3a4b5

.

1.4.4 Adding and Subtracting Square Roots

We can add or subtract radical expressions only when they have the same radicand and when they havethe same radical type such as square roots. For example, the sum of

√2 and 3

√2 is 4

√2.However, it is often

possible to simplify radical expressions, and that may change the radicand. The radical expression√

18 canbe written with a 2 in the radicand, as 3

√2,so√

2 +√

18 =√

2 + 3√

2 = 4√

2.

How To: Given a radical expression requiring addition or subtraction of square roots,solve.

Available for free at Connexions <http://legacy.cnx.org/content/col11759/1.2>

61

1.Simplify each radical expression.2.Add or subtract expressions with equal radicands.

Example 1.32Adding Square RootsAdd 5

√12 + 2

√3.

SolutionWe can rewrite 5

√12 as 5

√4 · 3.According the product rule, this becomes 5

√4√

3.The square rootof√

4 is 2, so the expression becomes 5 (2)√

3,which is 10√

3.Now we can the terms have the sameradicand so we can add.

10√

3 + 2√

3 = 12√

3

Try It:

Exercise 1.4.6 (Solution on p. 119.)

Add√

5 + 6√

20.

Example 1.33Subtracting Square RootsSubtract 20

√72a3b4c − 14

√8a3b4c.

SolutionRewrite each term so they have equal radicands.

20√

72a3b4c = 20√

9√

4√

2√a√a2

√(b2)2√

c

= 20 (3) (2) |a|b2√

2ac

= 120|a|b2√

2ac

(1.81)

14√

8a3b4c = 14√

2√

4√a√a2

√(b2)2√

c

= 14 (2) |a|b2√

2ac

= 28|a|b2√

2ac

(1.82)

Now the terms have the same radicand so we can subtract.

120|a|b2√

2ac − 28|a|b2√

2ac= 92|a|b2√

2ac (1.83)

Try It:

Exercise 1.4.7 (Solution on p. 119.)

Subtract 3√

80x − 4√

45x.

Available for free at Connexions <http://legacy.cnx.org/content/col11759/1.2>

62 CHAPTER 1. PREREQUISITES

1.4.5 Rationalizing Denominators

When an expression involving square root radicals is written in simplest form, it will not contain a radicalin the denominator. We can remove radicals from the denominators of fractions using a process calledrationalizing the denominator.

We know that multiplying by 1 does not change the value of an expression. We use this property ofmultiplication to change expressions that contain radicals in the denominator. To remove radicals from thedenominators of fractions, multiply by the form of 1 that will eliminate the radical.

For a denominator containing a single term, multiply by the radical in the denominator over itself. In

other words, if the denominator is b√c,multiply by

√c√c.

For a denominator containing the sum or di�erence of a rational and an irrational term, multiply thenumerator and denominator by the conjugate of the denominator, which is found by changing the sign ofthe radical portion of the denominator. If the denominator is a+ b

√c,then the conjugate is a− b

√c.

How To: Given an expression with a single square root radical term in the denomi-nator, rationalize the denominator.

a.Multiply the numerator and denominator by the radical in the denominator.b.Simplify.

Example 1.34Rationalizing a Denominator Containing a Single Term

Write 2√

33√

10in simplest form.

SolutionThe radical in the denominator is

√10. So multiply the fraction by

√10√10.Then simplify.

2√

33√

10·√

10√10

2√

3030√30

15

(1.84)

Try It:

Exercise 1.4.8 (Solution on p. 119.)

Write 12√

3√2in simplest form.

How To: Given an expression with a radical term and a constant in the denominator,rationalize the denominator.

1.Find the conjugate of the denominator.2.Multiply the numerator and denominator by the conjugate.3.Use the distributive property.4.Simplify.

Available for free at Connexions <http://legacy.cnx.org/content/col11759/1.2>

63

Example 1.35Rationalizing a Denominator Containing Two TermsWrite 4

1+√

5in simplest form.

SolutionBegin by �nding the conjugate of the denominator by writing the denominator and changing the

sign. So the conjugate of 1 +√

5 is 1−√

5.Then multiply the fraction by 1−√

51−√

5.

41+√

5· 1−

√5

1−√

54−4√

5−4 Use the distributive property.

√5− 1 Simplify.

(1.85)

Try It:

Exercise 1.4.9 (Solution on p. 119.)

Write 72+√

3in simplest form.

1.4.6 Using Rational Roots

Although square roots are the most common rational roots, we can also �nd cube roots, 4th roots, 5th roots,and more. Just as the square root function is the inverse of the squaring function, these roots are the inverseof their respective power functions. These functions can be useful when we need to determine the numberthat, when raised to a certain power, gives a certain number.

1.4.6.1 Understanding nth Roots

Suppose we know that a3 = 8.We want to �nd what number raised to the 3rd power is equal to 8. Since 23 =8,we say that 2 is the cube root of 8.

The nth root of a is a number that, when raised to the nth power, gives a.For example, − 3 is the 5throot of − 243 because (−3)5 = −243. If a is a real number with at least one nth root, then the principal nthroot of a is the number with the same sign as a that, when raised to the nth power, equals a.

The principal nth root of a is written as n√a,wheren is a positive integer greater than or equal to 2. In

the radical expression,n is called the index of the radical.

A General Note: If a is a real number with at least one nth root, then the principal nthroot of a,written as n

√a,is the number with the same sign as a that, when raised to the nth power,

equals a.The index of the radical isn.

Example 1.36Simplifying nth RootsSimplify each of the following:

a. 5√−32

b. 4√

4 · 4√

1, 024

c. − 3

√8x6

125

d. 8 4√

3− 4√

48

Available for free at Connexions <http://legacy.cnx.org/content/col11759/1.2>

64 CHAPTER 1. PREREQUISITES

Solution

a. 5√−32 = −2 because (−2)5 = −32

b. First, express the product as a single radical expression. 4√

4, 096 = 8 because 84 = 4, 096

c.− 3√

8x63√125

Write as quotient of two radical expressions.

−2x2

5 Simplify.

d.8 4√

3− 2 4√

3 Simplify to get equal radicands.

6 4√

3 Add.

Try It:

Exercise 1.4.10 (Solution on p. 119.)

Simplify.

a. 3√−216

b. 34√804√5

c.6 3√

9, 000 + 7 3√

576

1.4.6.2 Using Rational Exponents

Radical expressions can also be written without using the radical symbol. We can use rational (fractional)exponents. The index must be a positive integer. If the indexn is even, then a cannot be negative.

a1n = n

√a (1.86)

We can also have rational exponents with numerators other than 1. In these cases, the exponent must bea fraction in lowest terms. We raise the base to a power and take an nth root. The numerator tells us thepower and the denominator tells us the root.

amn =

(n√a)m = n

√am (1.87)

All of the properties of exponents that we learned for integer exponents also hold for rational exponents.

A General Note: Rational exponents are another way to express principal nth roots. Thegeneral form for converting between a radical expression with a radical symbol and one with arational exponent is

amn =

(n√a)m = n

√am (1.88)

How To: Given an expression with a rational exponent, write the expression as aradical.

1.Determine the power by looking at the numerator of the exponent.2.Determine the root by looking at the denominator of the exponent.3.Using the base as the radicand, raise the radicand to the power and use the root as the index.

Available for free at Connexions <http://legacy.cnx.org/content/col11759/1.2>

65

Example 1.37Writing Rational Exponents as RadicalsWrite 343

23 as a radical. Simplify.

SolutionThe 2 tells us the power and the 3 tells us the root.

34323 =

(3√

343)2

= 3√

3432

We know that 3√

343 = 7 because 73 = 343.Because the cube root is easy to �nd, it is easiest to�nd the cube root before squaring for this problem. In general, it is easier to �nd the root �rst andthen raise it to a power.

34323 =

(3√

343)2

= 72 = 49

Try It:

Exercise 1.4.11 (Solution on p. 119.)

Write 952 as a radical. Simplify.

Example 1.38Writing Radicals as Rational ExponentsWrite 4

7√a2using a rational exponent.

SolutionThe power is 2 and the root is 7, so the rational exponent will be 2

7 .We get 4

a27.Using properties

of exponents, we get 47√a2

= 4a−27 .

Try It:

Exercise 1.4.12 (Solution on p. 119.)

Writex

√(5y)9

using a rational exponent.

Example 1.39Simplifying Rational ExponentsSimplify:

a. 5(

2x34

)(3x

15

)b.(

169

)− 12

Solution

a.

30x34x

15 Multiply the coe�cients.

30x34 + 1

5 Use properties of exponents.

30x1920 Simplify.

Available for free at Connexions <http://legacy.cnx.org/content/col11759/1.2>

66 CHAPTER 1. PREREQUISITES

b.

(916

) 12 Use de�nition of negative exponents.√

916 Rewrite as a radical.√

9√16

Use the quotient rule.

34 Simplify.

Try It:

Exercise 1.4.13 (Solution on p. 119.)

Simplify (8x)13

(14x

65

).

Media: Access these online resources for additional instruction and practice with radicals andrational exponents.

• Radicals15• Rational Exponents16• Simplify Radicals17

• Rationalize Denominator18

1.4.7 Key Concepts

• The principal square root of a number a is the nonnegative number that when multiplied by itselfequals a. See Example 1.27.

• If a and b are nonnegative, the square root of the product ab is equal to the product of the square rootsof a and b See Example 1.28 and Example 1.29.

• If a and b are nonnegative, the square root of the quotient ab is equal to the quotient of the square rootsof a and b See Example 1.30 and Example 1.31.

• We can add and subtract radical expressions if they have the same radicand and the same index. SeeExample 1.32 and Example 1.33.

• Radical expressions written in simplest form do not contain a radical in the denominator. To eliminatethe square root radical from the denominator, multiply both the numerator and the denominator bythe conjugate of the denominator. See Example 1.34 and Example 1.35.

• The principal nth root of a is the number with the same sign as a that when raised to the nth powerequals a.These roots have the same properties as square roots. See Example 1.36.

• Radicals can be rewritten as rational exponents and rational exponents can be rewritten as radicals.See Example 1.37 and Example 1.38.

• The properties of exponents apply to rational exponents. See Example 1.39.

15http://openstaxcollege.org/l/introradical16http://openstaxcollege.org/l/rationexpon17http://openstaxcollege.org/l/simpradical18http://openstaxcollege.org/l/rationdenom

Available for free at Connexions <http://legacy.cnx.org/content/col11759/1.2>

67

1.4.8 Section Exercises

1.4.8.1 Verbal

Exercise 1.4.14 (Solution on p. 119.)

What does it mean when a radical does not have an index? Is the expression equal to the radicand?Explain.

Exercise 1.4.15Where would radicals come in the order of operations? Explain why.

Exercise 1.4.16 (Solution on p. 119.)

Every number will have two square roots. What is the principal square root?

Exercise 1.4.17Can a radical with a negative radicand have a real square root? Why or why not?

1.4.8.2 Numeric

For the following exercises, simplify each expression.

Exercise 1.4.18 (Solution on p. 119.)√256

Exercise 1.4.19√√256

Exercise 1.4.20 (Solution on p. 119.)√4 (9 + 16)

Exercise 1.4.21√289−

√121

Exercise 1.4.22 (Solution on p. 119.)√196

Exercise 1.4.23√1

Exercise 1.4.24 (Solution on p. 119.)√98

Exercise 1.4.25√2764

Exercise 1.4.26 (Solution on p. 119.)√815

Exercise 1.4.27√800

Exercise 1.4.28 (Solution on p. 119.)√169 +

√144

Exercise 1.4.29√850

Exercise 1.4.30 (Solution on p. 119.)18√162

Exercise 1.4.31√192

Available for free at Connexions <http://legacy.cnx.org/content/col11759/1.2>

68 CHAPTER 1. PREREQUISITES

Exercise 1.4.32 (Solution on p. 119.)

14√

6− 6√

24Exercise 1.4.3315√

5 + 7√

45Exercise 1.4.34 (Solution on p. 120.)√

150Exercise 1.4.35√

96100

Exercise 1.4.36 (Solution on p. 120.)(√42) (√

30)

Exercise 1.4.3712√

3− 4√

75Exercise 1.4.38 (Solution on p. 120.)√

4225

Exercise 1.4.39√405324

Exercise 1.4.40 (Solution on p. 120.)√360361

Exercise 1.4.415

1+√

3

Exercise 1.4.42 (Solution on p. 120.)8

1−√

17

Exercise 1.4.434√

16Exercise 1.4.44 (Solution on p. 120.)3√

128 + 3 3√

2Exercise 1.4.455

√−32243

Exercise 1.4.46 (Solution on p. 120.)15 4√125

4√5

Exercise 1.4.473 3√−432 + 3

√16

1.4.8.3 Algebraic

For the following exercises, simplify each expression.

Exercise 1.4.48 (Solution on p. 120.)√400x4

Exercise 1.4.49√4y2

Exercise 1.4.50 (Solution on p. 120.)√49p

Available for free at Connexions <http://legacy.cnx.org/content/col11759/1.2>

69

Exercise 1.4.51(144p2q6

) 12

Exercise 1.4.52 (Solution on p. 120.)

m52√

289Exercise 1.4.539√

3m2 +√

27Exercise 1.4.54 (Solution on p. 120.)

3√ab2 − b

√a

Exercise 1.4.554√

2n√16n4

Exercise 1.4.56 (Solution on p. 120.)√225x3

49x

Exercise 1.4.573√

44z +√

99zExercise 1.4.58 (Solution on p. 120.)√

50y8

Exercise 1.4.59√490bc2

Exercise 1.4.60 (Solution on p. 120.)√3214d

Exercise 1.4.61q

32√

63pExercise 1.4.62 (Solution on p. 120.)√

81−√

3x

Exercise 1.4.63√20

121d4

Exercise 1.4.64 (Solution on p. 120.)

w32√

32− w 32√

50Exercise 1.4.65√

108x4 +√

27x4

Exercise 1.4.66 (Solution on p. 120.)√12x

2+2√

3

Exercise 1.4.67√147k3

Exercise 1.4.68 (Solution on p. 120.)√125n10

Exercise 1.4.69√42q36q3

Exercise 1.4.70 (Solution on p. 120.)√81m

361m2

Exercise 1.4.71√72c− 2

√2c

Available for free at Connexions <http://legacy.cnx.org/content/col11759/1.2>

70 CHAPTER 1. PREREQUISITES

Exercise 1.4.72 (Solution on p. 120.)√144

324d2

Exercise 1.4.733√

24x6 + 3√

81x6

Exercise 1.4.74 (Solution on p. 120.)

4

√162x6

16x4

Exercise 1.4.753√

64yExercise 1.4.76 (Solution on p. 120.)3√

128z3 − 3√−16z3

Exercise 1.4.775√

1, 024c10

1.4.8.4 Real-World Applications

Exercise 1.4.78 (Solution on p. 120.)

A guy wire for a suspension bridge runs from the ground diagonally to the top of the closest pylonto make a triangle. We can use the Pythagorean Theorem to �nd the length of guy wire needed.The square of the distance between the wire on the ground and the pylon on the ground is 90,000feet. The square of the height of the pylon is 160,000 feet. So the length of the guy wire can befound by evaluating

√90, 000 + 160, 000.What is the length of the guy wire?

Exercise 1.4.79A car accelerates at a rate of 6−

√4√tm/s

2where t is the time in seconds after the car moves from

rest. Simplify the expression.

1.4.8.5 Extensions

For the following exercises, simplify each expression.

Exercise 1.4.80 (Solution on p. 120.)√8−√

164−√

2− 2

12

Exercise 1.4.814

32−16

32

813

Exercise 1.4.82 (Solution on p. 120.)√mn3

a2√c−3 ·

a−7n−2√m2c4

Exercise 1.4.83a

a−√c

Exercise 1.4.84 (Solution on p. 121.)x√

64y+4√y√

128y

Exercise 1.4.85(√250x2√100b3

)(7√b√

125x

)Exercise 1.4.86 (Solution on p. 121.)√

3√64+ 4√256√64+√

256

Available for free at Connexions <http://legacy.cnx.org/content/col11759/1.2>

71

1.5 Polynomials19

Earl is building a doghouse, whose front is in the shape of a square topped with a triangle. There will bea rectangular door through which the dog can enter and exit the house. Earl wants to �nd the area of thefront of the doghouse so that he can purchase the correct amount of paint. Using the measurements of thefront of the house, shown in Figure 1.7, we can create an expression that combines several variable terms,allowing us to solve this problem and others like it.

Figure 1.7

First �nd the area of the square in square feet.

A = s2

= (2x)2

= 4x2

(1.89)

Then �nd the area of the triangle in square feet.

A = 12bh

= 12 (2x)

(32

)= 3

2x

(1.90)

Next �nd the area of the rectangular door in square feet.

19This content is available online at <http://legacy.cnx.org/content/m51246/1.2/>.

Available for free at Connexions <http://legacy.cnx.org/content/col11759/1.2>

72 CHAPTER 1. PREREQUISITES

A = lw

= x · 1= x

(1.91)

The area of the front of the doghouse can be found by adding the areas of the square and the triangle, andthen subtracting the area of the rectangle. When we do this, we get 4x2 + 3

2x− x ft2,or 4x2 + 1

2x ft2.

In this section, we will examine expressions such as this one, which combine several variable terms.

1.5.1 Identifying the Degree and Leading Coe�cient of Polynomials

The formula just found is an example of a polynomial, which is a sum of or di�erence of terms, eachconsisting of a variable raised to a nonnegative integer power. A number multiplied by a variable raised toan exponent, such as 384π,is known as a coe�cient. Coe�cients can be positive, negative, or zero, and canbe whole numbers, decimals, or fractions. Each product aix

i,such as 384πw,is a term of a polynomial. Ifa term does not contain a variable, it is called a constant.

A polynomial containing only one term, such as 5x4,is called a monomial. A polynomial containing twoterms, such as 2x− 9,is called a binomial. A polynomial containing three terms, such as − 3x2 + 8x− 7,iscalled a trinomial.

We can �nd the degree of a polynomial by identifying the highest power of the variable that occurs inthe polynomial. The term with the highest degree is called the leading term because it is usually written�rst. The coe�cient of the leading term is called the leading coe�cient. When a polynomial is writtenso that the powers are descending, we say that it is in standard form.

A General Note: A polynomial is an expression that can be written in the form

anxn + ...+ a2x

2 + a1x+ a0 (1.92)

Each real number ai is called a coe�cient. The number a0 that is not multiplied by a variableis called a constant. Each product aix

i is a term of a polynomial. The highest power of thevariable that occurs in the polynomial is called the degree of a polynomial. The leading term isthe term with the highest power, and its coe�cient is called the leading coe�cient.

How To: Given a polynomial expression, identify the degree and leading coe�cient.

1.Find the highest power of x to determine the degree.2.Identify the term containing the highest power of x to �nd the leading term.3.Identify the coe�cient of the leading term.

Example 1.40Identifying the Degree and Leading Coe�cient of a PolynomialFor the following polynomials, identify the degree, the leading term, and the leading coe�cient.

Available for free at Connexions <http://legacy.cnx.org/content/col11759/1.2>

73

a. 3 + 2x2 − 4x3

b. 5t5 − 2t3 + 7tc. 6p− p3 − 2

Solution

a. The highest power of x is 3, so the degree is 3. The leading term is the term containing thatdegree, − 4x3.The leading coe�cient is the coe�cient of that term, − 4.

b. The highest power of t is 5,so the degree is 5.The leading term is the term containing thatdegree, 5t5.The leading coe�cient is the coe�cient of that term, 5.

c. The highest power of p is 3,so the degree is 3.The leading term is the term containing thatdegree, − p3,The leading coe�cient is the coe�cient of that term, − 1.

Try It:

Exercise 1.5.1 (Solution on p. 121.)

Identify the degree, leading term, and leading coe�cient of the polynomial 4x2−x6+2x−6.

1.5.2 Adding and Subtracting Polynomials

We can add and subtract polynomials by combining like terms, which are terms that contain the samevariables raised to the same exponents. For example, 5x2 and − 2x2 are like terms, and can be added toget 3x2,but 3x and 3x2 are not like terms, and therefore cannot be added.

How To: Given multiple polynomials, add or subtract them to simplify the expres-sions.

1.Combine like terms.2.Simplify and write in standard form.

Example 1.41Adding PolynomialsFind the sum.(

12x2 + 9x− 21)

+(4x3 + 8x2 − 5x+ 20

)Solution

4x3 +(12x2 + 8x2

)+ (9x− 5x) + (−21 + 20) Combine like terms.

4x3 + 20x2 + 4x− 1 Simplify.

AnalysisWe can check our answers to these types of problems using a graphing calculator. To check,graph the problem as given along with the simpli�ed answer. The two graphs should be equivalent.Be sure to use the same window to compare the graphs. Using di�erent windows can make theexpressions seem equivalent when they are not.

Available for free at Connexions <http://legacy.cnx.org/content/col11759/1.2>

74 CHAPTER 1. PREREQUISITES

Try It:

Exercise 1.5.2 (Solution on p. 121.)

Find the sum.(2x3 + 5x2 − x+ 1

)+(2x2 − 3x− 4

)Example 1.42Subtracting PolynomialsFind the di�erence.(

7x4 − x2 + 6x+ 1)−(5x3 − 2x2 + 3x+ 2

)Solution

7x4 − 5x3 +(−x2 + 2x2

)+ (6x− 3x) + (1− 2) Combine like terms.

7x4 − 5x3 + x2 + 3x− 1 Simplify.

AnalysisNote that �nding the di�erence between two polynomials is the same as adding the opposite ofthe second polynomial to the �rst.

Try It:

Exercise 1.5.3 (Solution on p. 121.)

Find the di�erence.(−7x3 − 7x2 + 6x− 2

)−(4x3 − 6x2 − x+ 7

)

1.5.3 Multiplying Polynomials

Multiplying polynomials is a bit more challenging than adding and subtracting polynomials. We must usethe distributive property to multiply each term in the �rst polynomial by each term in the second polynomial.We then combine like terms. We can also use a shortcut called the FOIL method when multiplying binomials.Certain special products follow patterns that we can memorize and use instead of multiplying the polynomialsby hand each time. We will look at a variety of ways to multiply polynomials.

1.5.3.1 Multiplying Polynomials Using the Distributive Property

To multiply a number by a polynomial, we use the distributive property. The number must be distributedto each term of the polynomial. We can distribute the 2 in 2 (x+ 7) to obtain the equivalent expression 2x+14.When multiplying polynomials, the distributive property allows us to multiply each term of the �rstpolynomial by each term of the second. We then add the products together and combine like terms tosimplify.

How To: Given the multiplication of two polynomials, use the distributive propertyto simplify the expression.

1.Multiply each term of the �rst polynomial by each term of the second.2.Combine like terms.3.Simplify.

Available for free at Connexions <http://legacy.cnx.org/content/col11759/1.2>

75

Example 1.43Multiplying Polynomials Using the Distributive PropertyFind the product.

(2x+ 1)(3x2 − x+ 4

)Solution

2x(3x2 − x+ 4

)+ 1

(3x2 − x+ 4

)Use the distributive property.(

6x3 − 2x2 + 8x)

+(3x2 − x+ 4

)Multiply.

6x3 +(−2x2 + 3x2

)+ (8x− x) + 4 Combine like terms.

6x3 + x2 + 7x+ 4 Simplify.

AnalysisWe can use a table to keep track of our work, as shown in Table 1.5. Write one polynomial acrossthe top and the other down the side. For each box in the table, multiply the term for that row bythe term for that column. Then add all of the terms together, combine like terms, and simplify.

3x2 −x +4

2x 6x3 −2x2 8x

+1 3x2 −x 4

Table 1.5

Try It:

Exercise 1.5.4 (Solution on p. 121.)

Find the product.

(3x+ 2)(x3 − 4x2 + 7

)1.5.3.2 Using FOIL to Multiply Binomials

A shortcut called FOIL is sometimes used to �nd the product of two binomials. It is called FOIL becausewe multiply the f irst terms, the outer terms, the inner terms, and then the last terms of each binomial.

Available for free at Connexions <http://legacy.cnx.org/content/col11759/1.2>

76 CHAPTER 1. PREREQUISITES

TheFOIL method arises out of the distributive property. We are simply multiplying each term of the �rstbinomial by each term of the second binomial, and then combining like terms.

How To: Given two binomials, use FOIL to simplify the expression.

1.Multiply the �rst terms of each binomial.2.Multiply the outer terms of the binomials.3.Multiply the inner terms of the binomials.4.Multiply the last terms of each binomial.5.Add the products.6.Combine like terms and simplify.

Example 1.44Using FOIL to Multiply BinomialsUse FOIL to �nd the product.

SolutionFind the product of the �rst terms.

Find the product of the outer terms.

Find the product of the inner terms.

Find the product of the last terms.

Available for free at Connexions <http://legacy.cnx.org/content/col11759/1.2>

77

6x2 + 6x− 54x− 54 Add the products.

6x2 + (6x− 54x)− 54 Combine like terms.

6x2 − 48x− 54 Simplify.

Try It:

Exercise 1.5.5 (Solution on p. 121.)

Use FOIL to �nd the product.

(x+ 7) (3x− 5)

1.5.3.3 Perfect Square Trinomials

Certain binomial products have special forms. When a binomial is squared, the result is called a perfectsquare trinomial. We can �nd the square by multiplying the binomial by itself. However, there is a specialform that each of these perfect square trinomials takes, and memorizing the form makes squaring binomialsmuch easier and faster. Let's look at a few perfect square trinomials to familiarize ourselves with the form.

(x+ 5)2 = x2 + 10x+ 25

(x− 3)2 = x2 − 6x+ 9

(4x− 1)2 = 4x2 − 8x+ 1

(1.93)

Notice that the �rst term of each trinomial is the square of the �rst term of the binomial and, similarly,the last term of each trinomial is the square of the last term of the binomial. The middle term is double theproduct of the two terms. Lastly, we see that the �rst sign of the trinomial is the same as the sign of thebinomial.

A General Note: When a binomial is squared, the result is the �rst term squared added todouble the product of both terms and the last term squared.

(x+ a)2 = (x+ a) (x+ a) = x2 + 2ax+ a2 (1.94)

How To: Given a binomial, square it using the formula for perfect square trinomials.

1.Square the �rst term of the binomial.2.Square the last term of the binomial.3.For the middle term of the trinomial, double the product of the two terms.4.Add and simplify.

Example 1.45Expanding Perfect Squares(3x− 8)2

.

SolutionBegin by squaring the �rst term and the last term. For the middle term of the trinomial, doublethe product of the two terms.

(3x)2 − 2 (3x) (8) + (−8)2(1.95)

Available for free at Connexions <http://legacy.cnx.org/content/col11759/1.2>

78 CHAPTER 1. PREREQUISITES

9x2 − 48x+ 64.

Try It:

Exercise 1.5.6 (Solution on p. 121.)

(4x− 1)2.

1.5.3.4 Di�erence of Squares

Another special product is called the di�erence of squares, which occurs when we multiply a bino-mial by another binomial with the same terms but the opposite sign. Let's see what happens when wemultiply (x+ 1) (x− 1) using the FOIL method.

(x+ 1) (x− 1) = x2 − x+ x− 1

= x2 − 1(1.96)

The middle term drops out, resulting in a di�erence of squares. Just as we did with the perfect squares,let's look at a few examples.

(x+ 5) (x− 5) = x2 − 25

(x+ 11) (x− 11) = x2 − 121

(2x+ 3) (2x− 3) = 4x2 − 9

(1.97)

Because the sign changes in the second binomial, the outer and inner terms cancel each other out, and weare left only with the square of the �rst term minus the square of the last term.

Q&A: Is there a special form for the sum of squares?

No. The di�erence of squares occurs because the opposite signs of the binomials cause the middleterms to disappear. There are no two binomials that multiply to equal a sum of squares.

A General Note: When a binomial is multiplied by a binomial with the same terms separatedby the opposite sign, the result is the square of the �rst term minus the square of the last term.

(a+ b) (a− b) = a2 − b2 (1.98)

How To: Given a binomial multiplied by a binomial with the same terms but theopposite sign, �nd the di�erence of squares.

1.Square the �rst term of the binomials.2.Square the last term of the binomials.3.Subtract the square of the last term from the square of the �rst term.

Available for free at Connexions <http://legacy.cnx.org/content/col11759/1.2>

79

Example 1.46Multiplying Binomials Resulting in a Di�erence of SquaresMultiply (9x+ 4) (9x− 4) .

SolutionSquare the �rst term to get (9x)2 = 81x2. Square the last term to get 42 = 16. Subtract the squareof the last term from the square of the �rst term to �nd the product of 81x2 − 16.

Try It:

Exercise 1.5.7 (Solution on p. 121.)

Multiply (2x+ 7) (2x− 7) .

1.5.4 Performing Operations with Polynomials of Several Variables

We have looked at polynomials containing only one variable. However, a polynomial can contain severalvariables. All of the same rules apply when working with polynomials containing several variables. Consideran example:

(a+ 2b) (4a− b− c)a (4a− b− c) + 2b (4a− b− c) Use the distributive property.

4a2 − ab− ac+ 8ab− 2b2 − 2bc Multiply.

4a2 + (−ab+ 8ab)− ac− 2b2 − 2bc Combine like terms.

4a2 + 7ab− ac− 2bc− 2b2 Simplify.

(1.99)

Example 1.47Multiplying Polynomials Containing Several VariablesMultiply (x+ 4) (3x− 2y + 5) .

SolutionFollow the same steps that we used to multiply polynomials containing only one variable.

x (3x− 2y + 5) + 4 (3x− 2y + 5) Use the distributive property.

3x2 − 2xy + 5x+ 12x− 8y + 20 Multiply.

3x2 − 2xy + (5x+ 12x)− 8y + 20 Combine like terms.

3x2 − 2xy + 17x− 8y + 20 Simplify.

(1.100)

Try It:

Exercise 1.5.8 (Solution on p. 121.)

(3x− 1) (2x+ 7y − 9) .

Available for free at Connexions <http://legacy.cnx.org/content/col11759/1.2>

80 CHAPTER 1. PREREQUISITES

Media: Access these online resources for additional instruction and practice with polynomials.

�Adding and Subtracting Polynomials20

�Multiplying Polynomials21

�Special Products of Polynomials22

1.5.5 Key Equations

perfect square trinomial (x+ a)2 = (x+ a) (x+ a) = x2 + 2ax+ a2

di�erence of squares (a+ b) (a− b) = a2 − b2

Table 1.6

1.5.6 Key Concepts

• A polynomial is a sum of terms each consisting of a variable raised to a non-negative integer power.The degree is the highest power of the variable that occurs in the polynomial. The leading term isthe term containing the highest degree, and the leading coe�cient is the coe�cient of that term. SeeExample 1.40.

• We can add and subtract polynomials by combining like terms. See Example 1.41 and Example 1.42.• To multiply polynomials, use the distributive property to multiply each term in the �rst polynomial

by each term in the second. Then add the products. See Example 1.43.• FOIL (First, Outer, Inner, Last) is a shortcut that can be used to multiply binomials. See Example 1.44.• Perfect square trinomials and di�erence of squares are special products. See Example 1.45 and Exam-

ple 1.46.• Follow the same rules to work with polynomials containing several variables. See Example 1.47.

1.5.7 Section Exercises

1.5.7.1 Verbal

Exercise 1.5.9 (Solution on p. 121.)

Evaluate the following statement: The degree of a polynomial in standard form is the exponent ofthe leading term. Explain why the statement is true or false.

Exercise 1.5.10Many times, multiplying two binomials with two variables results in a trinomial. This is not thecase when there is a di�erence of two squares. Explain why the product in this case is also abinomial.

Exercise 1.5.11 (Solution on p. 121.)

You can multiply polynomials with any number of terms and any number of variables using fourbasic steps over and over until you reach the expanded polynomial. What are the four steps?

Exercise 1.5.12State whether the following statement is true and explain why or why not: A trinomial is always ahigher degree than a monomial.

20http://openstaxcollege.org/l/addsubpoly21http://openstaxcollege.org/l/multiplpoly22http://openstaxcollege.org/l/specialpolyprod

Available for free at Connexions <http://legacy.cnx.org/content/col11759/1.2>

81

1.5.7.2 Algebraic

For the following exercises, identify the degree of the polynomial.

Exercise 1.5.13 (Solution on p. 121.)

7x− 2x2 + 13Exercise 1.5.1414m3 +m2 − 16m+ 8Exercise 1.5.15 (Solution on p. 121.)

−625a8 + 16b4

Exercise 1.5.16200p− 30p2m+ 40m3

Exercise 1.5.17 (Solution on p. 121.)

x2 + 4x+ 4Exercise 1.5.186y4 − y5 + 3y − 4

For the following exercises, �nd the sum or di�erence.

Exercise 1.5.19 (Solution on p. 121.)(12x2 + 3x

)−(8x2 − 19

)Exercise 1.5.20(4z3 + 8z2 − z

)+(−2z2 + z + 6

)Exercise 1.5.21 (Solution on p. 121.)(6w2 + 24w + 24

)−(3w2 − 6w + 3

)Exercise 1.5.22(7a3 + 6a2 − 4a− 13

)+(−3a3 − 4a2 + 6a+ 17

)Exercise 1.5.23 (Solution on p. 121.)(11b4 − 6b3 + 18b2 − 4b+ 8

)−(3b3 + 6b2 + 3b

)Exercise 1.5.24(49p2 − 25

)+(16p4 − 32p2 + 16

)For the following exercises, �nd the product.

Exercise 1.5.25 (Solution on p. 121.)

(4x+ 2) (6x− 4)Exercise 1.5.26(14c2 + 4c

) (2c2 − 3c

)Exercise 1.5.27 (Solution on p. 121.)(6b2 − 6

) (4b2 − 4

)Exercise 1.5.28(3d− 5) (2d+ 9)Exercise 1.5.29 (Solution on p. 121.)

(9v − 11) (11v − 9)Exercise 1.5.30(4t2 + 7t

) (−3t2 + 4

)Exercise 1.5.31 (Solution on p. 121.)

(8n− 4)(n2 + 9

)For the following exercises, expand the binomial.

Exercise 1.5.32(4x+ 5)2

Available for free at Connexions <http://legacy.cnx.org/content/col11759/1.2>

82 CHAPTER 1. PREREQUISITES

Exercise 1.5.33 (Solution on p. 121.)

(3y − 7)2

Exercise 1.5.34(12− 4x)2

Exercise 1.5.35 (Solution on p. 121.)

(4p+ 9)2

Exercise 1.5.36(2m− 3)2

Exercise 1.5.37 (Solution on p. 122.)

(3y − 6)2

Exercise 1.5.38(9b+ 1)2

For the following exercises, multiply the binomials.

Exercise 1.5.39 (Solution on p. 122.)

(4c+ 1) (4c− 1)Exercise 1.5.40(9a− 4) (9a+ 4)Exercise 1.5.41 (Solution on p. 122.)

(15n− 6) (15n+ 6)Exercise 1.5.42(25b+ 2) (25b− 2)Exercise 1.5.43 (Solution on p. 122.)

(4 + 4m) (4− 4m)Exercise 1.5.44(14p+ 7) (14p− 7)Exercise 1.5.45 (Solution on p. 122.)

(11q − 10) (11q + 10)For the following exercises, multiply the polynomials.

Exercise 1.5.46(2x2 + 2x+ 1

)(4x− 1)

Exercise 1.5.47 (Solution on p. 122.)(4t2 + t− 7

) (4t2 − 1

)Exercise 1.5.48(x− 1)

(x2 − 2x+ 1

)Exercise 1.5.49 (Solution on p. 122.)

(y − 2)(y2 − 4y − 9

)Exercise 1.5.50(6k − 5)

(6k2 + 5k − 1

)Exercise 1.5.51 (Solution on p. 122.)(3p2 + 2p− 10

)(p− 1)

Exercise 1.5.52(4m− 13)

(2m2 − 7m+ 9

)Exercise 1.5.53 (Solution on p. 122.)

(a+ b) (a− b)Exercise 1.5.54(4x− 6y) (6x− 4y)

Available for free at Connexions <http://legacy.cnx.org/content/col11759/1.2>

83

Exercise 1.5.55 (Solution on p. 122.)

(4t− 5u)2

Exercise 1.5.56(9m+ 4n− 1) (2m+ 8)Exercise 1.5.57 (Solution on p. 122.)

(4t− x) (t− x+ 1)Exercise 1.5.58(b2 − 1

) (a2 + 2ab+ b2

)Exercise 1.5.59 (Solution on p. 122.)

(4r − d) (6r + 7d)Exercise 1.5.60(x+ y)

(x2 − xy + y2

)

1.5.7.3 Real-World Applications

Exercise 1.5.61 (Solution on p. 122.)

A developer wants to purchase a plot of land to build a house. The area of the plot can bedescribed by the following expression: (4x+ 1) (8x− 3) where x is measured in meters. Multiplythe binomials to �nd the area of the plot in standard form.

Exercise 1.5.62A prospective buyer wants to know how much grain a speci�c silo can hold. The area of the �oorof the silo is (2x+ 9)2

.The height of the silo is 10x + 10,where x is measured in feet. Expand thesquare and multiply by the height to �nd the expression that shows how much grain the silo canhold.

1.5.7.4 Extensions

For the following exercises, perform the given operations.

Exercise 1.5.63 (Solution on p. 122.)

(4t− 7)2 (2t+ 1)−(4t2 + 2t+ 11

)Exercise 1.5.64(3b+ 6) (3b− 6)

(9b2 − 36

)Exercise 1.5.65 (Solution on p. 122.)(a2 + 4ac+ 4c2

) (a2 − 4c2

)

1.6 Factoring Polynomials23

Imagine that we are trying to �nd the area of a lawn so that we can determine how much grass seed topurchase. The lawn is the green portion in Figure 1.8.

23This content is available online at <http://legacy.cnx.org/content/m51247/1.2/>.

Available for free at Connexions <http://legacy.cnx.org/content/col11759/1.2>

84 CHAPTER 1. PREREQUISITES

Figure 1.8

The area of the entire region can be found using the formula for the area of a rectangle.

A = lw

= 10x · 6x= 60x2 units2

(1.101)

The areas of the portions that do not require grass seed need to be subtracted from the area of the entireregion. The two square regions each have an area ofA = s2 = 42 = 16 units2. The other rectangular regionhas one side of length 10x−8 and one side of length 4,giving an area ofA = lw = 4 (10x− 8) = 40x−32 units2.So the region that must be subtracted has an area of 2 (16) + 40x− 32 = 40x units2.

The area of the region that requires grass seed is found by subtracting 60x2 − 40x units2. This areacan also be expressed in factored form as 20x (3x− 2) units2. We can con�rm that this is an equivalentexpression by multiplying.

Many polynomial expressions can be written in simpler forms by factoring. In this section, we will lookat a variety of methods that can be used to factor polynomial expressions.

1.6.1 Factoring the Greatest Common Factor of a Polynomial

When we study fractions, we learn that the greatest common factor (GCF) of two numbers is the largestnumber that divides evenly into both numbers. For instance, 4 is the GCF of 16 and 20 because it is the largestnumber that divides evenly into both 16 and 20The GCF of polynomials works the same way: 4x is the GCFof 16x and 20x2 because it is the largest polynomial that divides evenly into both 16x and 20x2.

When factoring a polynomial expression, our �rst step should be to check for a GCF. Look for the GCFof the coe�cients, and then look for the GCF of the variables.

Available for free at Connexions <http://legacy.cnx.org/content/col11759/1.2>

85

A General Note: The greatest common factor (GCF) of polynomials is the largest polyno-mial that divides evenly into the polynomials.

How To: Given a polynomial expression, factor out the greatest common factor.

1.Identify the GCF of the coe�cients.2.Identify the GCF of the variables.3.Combine to �nd the GCF of the expression.4.Determine what the GCF needs to be multiplied by to obtain each term in the expression.5.Write the factored expression as the product of the GCF and the sum of the terms we needto multiply by.

Example 1.48Factoring the Greatest Common FactorFactor 6x3y3 + 45x2y2 + 21xy.

SolutionFirst, �nd the GCF of the expression. The GCF of 6, 45,and 21 is 3.The GCF

ofx3, x2,andx isx. (Note that the GCF of a set of expressions in the formxn will always be theexponent of lowest degree.) And the GCF of y3, y2,and y is y.Combine these to �nd the GCF of thepolynomial, 3xy.

Next, determine what the GCF needs to be multiplied by to obtain each term of the polynomial.We �nd that 3xy

(2x2y2

)= 6x3y3, 3xy (15xy) = 45x2y2,and 3xy (7) = 21xy.

Finally, write the factored expression as the product of the GCF and the sum of the terms weneeded to multiply by.

(3xy)(2x2y2 + 15xy + 7

)(1.102)

AnalysisAfter factoring, we can check our work by multiplying. Use the distributive property to con�rmthat (3xy)

(2x2y2 + 15xy + 7

)= 6x3y3 + 45x2y2 + 21xy.

Try It:

Exercise 1.6.1 (Solution on p. 122.)

Factorx(b2 − a

)+ 6

(b2 − a

)by pulling out the GCF.

1.6.2 Factoring a Trinomial with Leading Coe�cient 1

Although we should always begin by looking for a GCF, pulling out the GCF is not the only way thatpolynomial expressions can be factored. The polynomialx2 + 5x + 6 has a GCF of 1, but it can be writtenas the product of the factors (x+ 2) and (x+ 3) .

Trinomials of the formx2 + bx+ c can be factored by �nding two numbers with a product ofc and a sumof b.The trinomialx2+10x+16,for example, can be factored using the numbers 2 and 8 because the product ofthose numbers is 16 and their sum is 10.The trinomial can be rewritten as the product of (x+ 2) and (x+ 8) .

A General Note: A trinomial of the formx2 + bx + c can be written in factored formas (x+ p) (x+ q) where pq = c and p+ q = b.

Available for free at Connexions <http://legacy.cnx.org/content/col11759/1.2>

86 CHAPTER 1. PREREQUISITES

Q&A: Can every trinomial be factored as a product of binomials?

No. Some polynomials cannot be factored. These polynomials are said to be prime.

How To: Given a trinomial in the form x2 + bx+ c,factor it.

1.List factors of c.2.Find p and q,a pair of factors of cwith a sum of b.3.Write the factored expression (x+ p) (x+ q) .

Example 1.49Factoring a Trinomial with Leading Coe�cient 1Factorx2 + 2x− 15.

SolutionWe have a trinomial with leading coe�cient 1, b = 2,and c = −15.We need to �nd two numberswith a product of − 15 and a sum of 2. In Table 1.7, we list factors until we �nd a pair with thedesired sum.

Factors of − 15 Sum of Factors

1,−15 −14

−1, 15 14

3,−5 −2

−3, 5 2

Table 1.7

Now that we have identi�ed p and q as − 3 and 5,write the factored form as (x− 3) (x+ 5) .

AnalysisWe can check our work by multiplying. Use FOIL to con�rm that (x− 3) (x+ 5) = x2 + 2x− 15.

Q&A: Does the order of the factors matter?

No. Multiplication is commutative, so the order of the factors does not matter.

Try It:

Exercise 1.6.2 (Solution on p. 122.)

Factorx2 − 7x+ 6.

Available for free at Connexions <http://legacy.cnx.org/content/col11759/1.2>

87

1.6.3 Factoring by Grouping

Trinomials with leading coe�cients other than 1 are slightly more complicated to factor. For these trinomials,we can factor by grouping by dividing the x term into the sum of two terms, factoring each portion ofthe expression separately, and then factoring out the GCF of the entire expression. The trinomial 2x2 +5x+ 3 can be rewritten as (2x+ 3) (x+ 1) using this process. We begin by rewriting the original expressionas 2x2 + 2x+ 3x+ 3 and then factor each portion of the expression to obtain 2x (x+ 1) + 3 (x+ 1) .We thenpull out the GCF of (x+ 1) to �nd the factored expression.

A General Note: To factor a trinomial in the form ax2 + bx + c by grouping, we �nd twonumbers with a product of ac and a sum of b.We use these numbers to divide thex term into thesum of two terms and factor each portion of the expression separately, then factor out the GCF ofthe entire expression.

How To: Given a trinomial in the form ax2 + bx+ c,factor by grouping.

1.List factors of ac.2.Find p and q,a pair of factors of acwith a sum of b.3.Rewrite the original expression as ax2 + px+ qx+ c.4.Pull out the GCF of ax2 + px.5.Pull out the GCF of qx+ c.6.Factor out the GCF of the expression.

Example 1.50Factoring a Trinomial by GroupingFactor 5x2 + 7x− 6 by grouping.

SolutionWe have a trinomial with a = 5, b = 7,and c = −6.First, determine ac = −30.We need to �nd twonumbers with a product of − 30 and a sum of 7. In Table 1.8, we list factors until we �nd a pairwith the desired sum.

Factors of − 30 Sum of Factors

1,−30 −29

−1, 30 29

2,−15 −13

−2, 15 13

3,−10 −7

−3, 10 7

Table 1.8

So p = −3 and q = 10.

5x2 − 3x+ 10x− 6 Rewrite the original expression as ax2 + px+ qx+ c.

x (5x− 3) + 2 (5x− 3) Factor out the GCF of each part.

(5x− 3) (x+ 2) Factor out the GCF of the expression.

(1.103)

Available for free at Connexions <http://legacy.cnx.org/content/col11759/1.2>

88 CHAPTER 1. PREREQUISITES

AnalysisWe can check our work by multiplying. Use FOIL to con�rm that (5x− 3) (x+ 2) = 5x2 + 7x− 6.

: Exercise 1.6.3 (Solution on p. 122.)

Factor a. 2x2 + 9x+ 9 b. 6x2 + x− 1

1.6.4 Factoring a Perfect Square Trinomial

A perfect square trinomial is a trinomial that can be written as the square of a binomial. Recall that whena binomial is squared, the result is the square of the �rst term added to twice the product of the two termsand the square of the last term.

a2 + 2ab+ b2 = (a+ b)2

and

a2 − 2ab+ b2 = (a− b)2

(1.104)

We can use this equation to factor any perfect square trinomial.

A General Note: A perfect square trinomial can be written as the square of a binomial:

a2 + 2ab+ b2 = (a+ b)2(1.105)

How To: Given a perfect square trinomial, factor it into the square of a binomial.

1.Con�rm that the �rst and last term are perfect squares.2.Con�rm that the middle term is twice the product of ab.3.Write the factored form as (a+ b)2

.

Example 1.51Factoring a Perfect Square TrinomialFactor 25x2 + 20x+ 4.

SolutionNotice that 25x2 and 4 are perfect squares because 25x2 = (5x)2

and 4 = 22.Then check tosee if the middle term is twice the product of 5x and 2.The middle term is, indeed, twice theproduct: 2 (5x) (2) = 20x.Therefore, the trinomial is a perfect square trinomial and can be written

as (5x+ 2)2.

Try It:

Exercise 1.6.4 (Solution on p. 122.)

Factor 49x2 − 14x+ 1.

Available for free at Connexions <http://legacy.cnx.org/content/col11759/1.2>

89

1.6.5 Factoring a Di�erence of Squares

A di�erence of squares is a perfect square subtracted from a perfect square. Recall that a di�erence ofsquares can be rewritten as factors containing the same terms but opposite signs because the middle termscancel each other out when the two factors are multiplied.

a2 − b2 = (a+ b) (a− b) (1.106)

We can use this equation to factor any di�erences of squares.

A General Note: A di�erence of squares can be rewritten as two factors containing the sameterms but opposite signs.

a2 − b2 = (a+ b) (a− b) (1.107)

How To: Given a di�erence of squares, factor it into binomials.

1.Con�rm that the �rst and last term are perfect squares.2.Write the factored form as

(a+ b) (a− b) .

Example 1.52Factoring a Di�erence of SquaresFactor 9x2 − 25.

SolutionNotice that 9x2 and 25 are perfect squares because 9x2 = (3x)2

and 25 = 52.The polynomial repre-sents a di�erence of squares and can be rewritten as (3x+ 5) (3x− 5) .

Try It:

Exercise 1.6.5 (Solution on p. 122.)

Factor 81y2 − 100.

Q&A: Is there a formula to factor the sum of squares?

No. A sum of squares cannot be factored.

1.6.6 Factoring the Sum and Di�erence of Cubes

Now, we will look at two new special products: the sum and di�erence of cubes. Although the sum of squarescannot be factored, the sum of cubes can be factored into a binomial and a trinomial.

a3 + b3 = (a+ b)(a2 − ab+ b2

)(1.108)

Similarly, the sum of cubes can be factored into a binomial and a trinomial, but with di�erent signs.

a3 − b3 = (a− b)(a2 + ab+ b2

)(1.109)

Available for free at Connexions <http://legacy.cnx.org/content/col11759/1.2>

90 CHAPTER 1. PREREQUISITES

We can use the acronym SOAP to remember the signs when factoring the sum or di�erence of cubes. The�rst letter of each word relates to the signs: Same Opposite Always Positive. For example, consider thefollowing example.

x3 − 23 = (x− 2)(x2 + 2x+ 4

)(1.110)

The sign of the �rst 2 is the same as the sign betweenx3 − 23.The sign of the 2x term is opposite the signbetweenx3 − 23.And the sign of the last term, 4, is always positive.

A General Note: We can factor the sum of two cubes as

a3 + b3 = (a+ b)(a2 − ab+ b2

)(1.111)

We can factor the di�erence of two cubes as

a3 − b3 = (a− b)(a2 + ab+ b2

)(1.112)

How To: Given a sum of cubes or di�erence of cubes, factor it.

1.Con�rm that the �rst and last term are cubes, a3 + b3 or a3 − b3.2.For a sum of cubes, write the factored form as (a+ b)

(a2 − ab+ b2

).For a di�erence of cubes,

write the factored form as (a− b)(a2 + ab+ b2

).

Example 1.53Factoring a Sum of CubesFactorx3 + 512.

SolutionNotice thatx3 and 512 are cubes because 83 = 512.Rewrite the sum of cubes

as (x+ 8)(x2 − 8x+ 64

).

AnalysisAfter writing the sum of cubes this way, we might think we should check to see if the trinomialportion can be factored further. However, the trinomial portion cannot be factored, so we do notneed to check.

Try It:

Exercise 1.6.6 (Solution on p. 122.)

Factor the sum of cubes: 216a3 + b3.

Example 1.54Factoring a Di�erence of CubesFactor 8x3 − 125.

SolutionNotice that 8x3 and 125 are cubes because 8x3 = (2x)3

and 125 = 53.Write the di�erence of cubesas (2x− 5)

(4x2 + 10x+ 25

).

AnalysisJust as with the sum of cubes, we will not be able to further factor the trinomial portion.

Available for free at Connexions <http://legacy.cnx.org/content/col11759/1.2>

91

Try It:

Exercise 1.6.7 (Solution on p. 122.)

Factor the di�erence of cubes: 1, 000x3 − 1.

1.6.7 Factoring Expressions with Fractional or Negative Exponents

Expressions with fractional or negative exponents can be factored by pulling out a GCF. Look for the variableor exponent that is common to each term of the expression and pull out that variable or exponent raisedto the lowest power. These expressions follow the same factoring rules as those with integer exponents. For

instance, 2x14 + 5x

34 can be factored by pulling outx

14 and being rewritten asx

14

(2 + 5x

12

).

Example 1.55Factoring an Expression with Fractional or Negative Exponents

Factor 3x(x+ 2)−13 + 4(x+ 2)

23 .

SolutionFactor out the term with the lowest value of the exponent. In this case, that would be (x+ 2)−

13 .

(x+ 2)−13 (3x+ 4 (x+ 2)) Factor out the GCF.

(x+ 2)−13 (3x+ 4x+ 8) Simplify.

(x+ 2)−13 (7x+ 8)

(1.113)

Try It:

Exercise 1.6.8 (Solution on p. 122.)

Factor 2(5a− 1)34 + 7a(5a− 1)−

14 .

Media: Access these online resources for additional instruction and practice with factoring poly-nomials.

• Identify GCF24

• Factor Trinomials when a Equals 125

• Factor Trinomials when a is not equal to 126

• Factor Sum or Di�erence of Cubes27

24http://openstaxcollege.org/l/�ndgcftofact25http://openstaxcollege.org/l/facttrinom126http://openstaxcollege.org/l/facttrinom227http://openstaxcollege.org/l/sumdifcube

Available for free at Connexions <http://legacy.cnx.org/content/col11759/1.2>

92 CHAPTER 1. PREREQUISITES

1.6.8 Key Equations

di�erence of squares a2 − b2 = (a+ b) (a− b)perfect square trinomial a2 + 2ab+ b2 = (a+ b)2

sum of cubes a3 + b3 = (a+ b)(a2 − ab+ b2

)di�erence of cubes a3 − b3 = (a− b)

(a2 + ab+ b2

)Table 1.9

1.6.9

• The greatest common factor, or GCF, can be factored out of a polynomial. Checking for a GCF shouldbe the �rst step in any factoring problem. See Example 1.48.

• Trinomials with leading coe�cient 1 can be factored by �nding numbers that have a product of thethird term and a sum of the second term. See Example 1.49.

• Trinomials can be factored using a process called factoring by grouping. See Example 1.50.• Perfect square trinomials and the di�erence of squares are special products and can be factored using

equations. See Example 1.51 and Example 1.52.• The sum of cubes and the di�erence of cubes can be factored using equations. See Example 1.53 and

Example 1.54.• Polynomials containing fractional and negative exponents can be factored by pulling out a GCF. See

Example 1.55.

1.6.10

1.6.10.1 Verbal

Exercise 1.6.9 (Solution on p. 122.)

If the terms of a polynomial do not have a GCF, does that mean it is not factorable? Explain.

Exercise 1.6.10A polynomial is factorable, but it is not a perfect square trinomial or a di�erence of two squares.Can you factor the polynomial without �nding the GCF?

Exercise 1.6.11 (Solution on p. 123.)

How do you factor by grouping?

1.6.10.2 Algebraic

For the following exercises, �nd the greatest common factor.

Exercise 1.6.1214x+ 4xy − 18xy2

Exercise 1.6.13 (Solution on p. 123.)

49mb2 − 35m2ba+ 77ma2

Exercise 1.6.1430x3y − 45x2y2 + 135xy3

Available for free at Connexions <http://legacy.cnx.org/content/col11759/1.2>

93

Exercise 1.6.15 (Solution on p. 123.)

200p3m3 − 30p2m3 + 40m3

Exercise 1.6.1636j4k2 − 18j3k3 + 54j2k4

Exercise 1.6.17 (Solution on p. 123.)

6y4 − 2y3 + 3y2 − yFor the following exercises, factor by grouping.

Exercise 1.6.186x2 + 5x− 4Exercise 1.6.19 (Solution on p. 123.)

2a2 + 9a− 18Exercise 1.6.206c2 + 41c+ 63Exercise 1.6.21 (Solution on p. 123.)

6n2 − 19n− 11Exercise 1.6.2220w2 − 47w + 24Exercise 1.6.23 (Solution on p. 123.)

2p2 − 5p− 7For the following exercises, factor the polynomial.

Exercise 1.6.247x2 + 48x− 7Exercise 1.6.25 (Solution on p. 123.)

10h2 − 9h− 9Exercise 1.6.262b2 − 25b− 247Exercise 1.6.27 (Solution on p. 123.)

9d2 − 73d+ 8Exercise 1.6.2890v2 − 181v + 90Exercise 1.6.29 (Solution on p. 123.)

12t2 + t− 13Exercise 1.6.302n2 − n− 15Exercise 1.6.31 (Solution on p. 123.)

16x2 − 100Exercise 1.6.3225y2 − 196Exercise 1.6.33 (Solution on p. 123.)

121p2 − 169Exercise 1.6.344m2 − 9Exercise 1.6.35 (Solution on p. 123.)

361d2 − 81Exercise 1.6.36324x2 − 121

Available for free at Connexions <http://legacy.cnx.org/content/col11759/1.2>

94 CHAPTER 1. PREREQUISITES

Exercise 1.6.37 (Solution on p. 123.)

144b2 − 25c2

Exercise 1.6.3816a2 − 8a+ 1Exercise 1.6.39 (Solution on p. 123.)

49n2 + 168n+ 144Exercise 1.6.40121x2 − 88x+ 16Exercise 1.6.41 (Solution on p. 123.)

225y2 + 120y + 16Exercise 1.6.42m2 − 20m+ 100Exercise 1.6.43 (Solution on p. 123.)

m2 − 20m+ 100Exercise 1.6.4436q2 + 60q + 25

For the following exercises, factor the polynomials.

Exercise 1.6.45 (Solution on p. 123.)

x3 + 216Exercise 1.6.4627y3 − 8Exercise 1.6.47 (Solution on p. 123.)

125a3 + 343Exercise 1.6.48b3 − 8d3

Exercise 1.6.49 (Solution on p. 123.)

64x3 − 125Exercise 1.6.50729q3 + 1331Exercise 1.6.51 (Solution on p. 123.)

125r3 + 1, 728s3

Exercise 1.6.524x(x− 1)−

23 + 3(x− 1)

13

Exercise 1.6.53 (Solution on p. 123.)

3c(2c+ 3)−14 − 5(2c+ 3)

34

Exercise 1.6.543t(10t+ 3)

13 + 7(10t+ 3)

43

Exercise 1.6.55 (Solution on p. 123.)

14x(x+ 2)−25 + 5(x+ 2)

35

Exercise 1.6.569y(3y − 13)

15 − 2(3y − 13)

65

Exercise 1.6.57 (Solution on p. 123.)

5z(2z − 9)−32 + 11(2z − 9)−

12

Exercise 1.6.586d(2d+ 3)−

16 + 5(2d+ 3)

56

Available for free at Connexions <http://legacy.cnx.org/content/col11759/1.2>

95

1.6.10.3 Real-World Applications

For the following exercises, consider this scenario:Charlotte has appointed a chairperson to lead a city beauti�cation project. The �rst act is to install

statues and fountains in one of the city's parks. The park is a rectangle with an area of 98x2 + 105x− 27m2,as shown in the �gure below. The length and width of the park are perfect factors of the area.

Exercise 1.6.59 (Solution on p. 124.)

Factor by grouping to �nd the length and width of the park.

Exercise 1.6.60A statue is to be placed in the center of the park. The area of the base of the statue is 4x2 + 12x+9m2.Factor the area to �nd the lengths of the sides of the statue.

Exercise 1.6.61 (Solution on p. 124.)

At the northwest corner of the park, the city is going to install a fountain. The area of the base ofthe fountain is 9x2 − 25m2.Factor the area to �nd the lengths of the sides of the fountain.

For the following exercise, consider the following scenario:A school is installing a �agpole in the central plaza. The plaza is a square with side length 100 yd. as

shown in the �gure below. The �agpole will take up a square plot with areax2 − 6x+ 9yd2.

Available for free at Connexions <http://legacy.cnx.org/content/col11759/1.2>

96 CHAPTER 1. PREREQUISITES

Exercise 1.6.62Find the length of the base of the �agpole by factoring.

1.6.10.4 Extensions

For the following exercises, factor the polynomials completely.

Exercise 1.6.63 (Solution on p. 124.)

16x4 − 200x2 + 625Exercise 1.6.6481y4 − 256Exercise 1.6.65 (Solution on p. 124.)

16z4 − 2, 401a4

Exercise 1.6.665x(3x+ 2)−

24 + (12x+ 8)

32

Exercise 1.6.67 (Solution on p. 124.)(32x3 + 48x2 − 162x− 243

)−1

1.7 Rational Expressions28

A pastry shop has �xed costs of $280 per week and variable costs of $9 per box of pastries. The shop's costsper week in terms ofx,the number of boxes made, is 280+9x.We can divide the costs per week by the numberof boxes made to determine the cost per box of pastries.

280 + 9xx

(1.114)

Notice that the result is a polynomial expression divided by a second polynomial expression. In this section,we will explore quotients of polynomial expressions.

28This content is available online at <http://legacy.cnx.org/content/m51248/1.2/>.

Available for free at Connexions <http://legacy.cnx.org/content/col11759/1.2>

97

1.7.1 Simplifying Rational Expressions

The quotient of two polynomial expressions is called a rational expression. We can apply the properties offractions to rational expressions, such as simplifying the expressions by canceling common factors from thenumerator and the denominator. To do this, we �rst need to factor both the numerator and denominator.Let's start with the rational expression shown.

x2 + 8x+ 16x2 + 11x+ 28

(1.115)

We can factor the numerator and denominator to rewrite the expression.

(x+ 4)2

(x+ 4) (x+ 7)(1.116)

Then we can simplify that expression by canceling the common factor (x+ 4) .

x+ 4x+ 7

(1.117)

How To: Given a rational expression, simplify it.

1.Factor the numerator and denominator.2.Cancel any common factors.

Example 1.56Simplifying Rational Expressions

Simplify x2−9x2+4x+3 .

Solution

(x+3)(x−3)(x+3)(x+1) Factor the numerator and the denominator.

x−3x+1 Cancel common factor (x+ 3) .

(1.118)

AnalysisWe can cancel the common factor because any expression divided by itself is equal to 1.

Q&A: Can the x2 term be cancelled in Example 1.56?

No. A factor is an expression that is multiplied by another expression. Thex2 term is not a factorof the numerator or the denominator.

Try It:

Exercise 1.7.1 (Solution on p. 124.)

Simplify x−6x2−36 .

Available for free at Connexions <http://legacy.cnx.org/content/col11759/1.2>

98 CHAPTER 1. PREREQUISITES

1.7.2 Multiplying Rational Expressions

Multiplication of rational expressions works the same way as multiplication of any other fractions. Wemultiply the numerators to �nd the numerator of the product, and then multiply the denominators to �ndthe denominator of the product. Before multiplying, it is helpful to factor the numerators and denominatorsjust as we did when simplifying rational expressions. We are often able to simplify the product of rationalexpressions.

How To: Given two rational expressions, multiply them.

1.Factor the numerator and denominator.2.Multiply the numerators.3.Multiply the denominators.4.Simplify.

Example 1.57Multiplying Rational ExpressionsMultiply the rational expressions and show the product in simplest form:

(x+5)(x−1)3(x+6) · (2x−1)

(x+5) Factor the numerator and denominator.(x+5)(x−1)(2x−1)

3(x+6)(x+5) Multiply numerators and denominators.)(x+5)(x−1)(2x−1)

3(x+6))(x+5)Cancel common factors to simplify.

(x−1)(2x−1)3(x+6)

(1.119)

Solution

(x+5)(x−1)3(x+6) · (2x−1)

(x+5) Factor the numerator and denominator.(x+5)(x−1)(2x−1)

3(x+6)(x+5) Multiply numerators and denominators.)(x+5)(x−1)(2x−1)

3(x+6))(x+5)Cancel common factors to simplify.

(x−1)(2x−1)3(x+6)

(1.120)

Try It:

Exercise 1.7.2 (Solution on p. 124.)

Multiply the rational expressions and show the product in simplest form:

x2 + 11x+ 30x2 + 5x+ 6

· x2 + 7x+ 12x2 + 8x+ 16

(1.121)

Available for free at Connexions <http://legacy.cnx.org/content/col11759/1.2>

99

1.7.3 Dividing Rational Expressions

Division of rational expressions works the same way as division of other fractions. To divide a rationalexpression by another rational expression, multiply the �rst expression by the reciprocal of the second.

Using this approach, we would rewrite 1x ÷

x2

3 as the product 1x ·

3x2 .Once the division expression has been

rewritten as a multiplication expression, we can multiply as we did before.

1x· 3x2

=3x3

(1.122)

How To: Given two rational expressions, divide them.

1.Rewrite as the �rst rational expression multiplied by the reciprocal of the second.2.Factor the numerators and denominators.3.Multiply the numerators.4.Multiply the denominators.5.Simplify.

Example 1.58Dividing Rational ExpressionsDivide the rational expressions and express the quotient in simplest form:

2x2 + x− 6x2 − 1

÷ x2 − 4x2 + 2x+ 1

(1.123)

Solution

9x2 − 163x2 + 17x− 28

÷ 3x2 − 2x− 8x2 + 5x− 14

(1.124)

Try It:

Exercise 1.7.3 (Solution on p. 124.)

Divide the rational expressions and express the quotient in simplest form:

9x2 − 163x2 + 17x− 28

÷ 3x2 − 2x− 8x2 + 5x− 14

(1.125)

Available for free at Connexions <http://legacy.cnx.org/content/col11759/1.2>

100 CHAPTER 1. PREREQUISITES

1.7.4 Adding and Subtracting Rational Expressions

Adding and subtracting rational expressions works just like adding and subtracting numerical fractions. Toadd fractions, we need to �nd a common denominator. Let's look at an example of fraction addition.

524 + 1

40 = 25120 + 3

120

= 28120

= 730

(1.126)

We have to rewrite the fractions so they share a common denominator before we are able to add. We mustdo the same thing when adding or subtracting rational expressions.

The easiest common denominator to use will be the least common denominator, or LCD. The LCD isthe smallest multiple that the denominators have in common. To �nd the LCD of two rational expressions,we factor the expressions and multiply all of the distinct factors. For instance, if the factored denominatorswere (x+ 3) (x+ 4) and (x+ 4) (x+ 5) ,then the LCD would be (x+ 3) (x+ 4) (x+ 5) .

Once we �nd the LCD, we need to multiply each expression by the form of 1 that will change the denomina-tor to the LCD. We would need to multiply the expression with a denominator of (x+ 3) (x+ 4) by x+5

x+5 and

the expression with a denominator of (x+ 4) (x+ 5) by x+3x+3 .

How To: Given two rational expressions, add or subtract them.

1.Factor the numerator and denominator.2.Find the LCD of the expressions.3.Multiply the expressions by a form of 1 that changes the denominators to the LCD.4.Add or subtract the numerators.5.Simplify.

Example 1.59Adding Rational ExpressionsAdd the rational expressions:

5x

+6y

(1.127)

Solution

First, we have to �nd the LCD. In this case, the LCD will bexy.We then multiply each expressionby the appropriate form of 1 to obtainxy as the denominator for each fraction.

5x ·

yy + 6

y ·xx

5yxy + 6x

xy

(1.128)

Now that the expressions have the same denominator, we simply add the numerators to �nd thesum.

6x+ 5yxy

(1.129)

AnalysisMultiplying by y

y orxx does not change the value of the original expression because any number

divided by itself is 1, and multiplying an expression by 1 gives the original expression.

Available for free at Connexions <http://legacy.cnx.org/content/col11759/1.2>

101

Example 1.60Subtracting Rational ExpressionsSubtract the rational expressions:

6x2 + 4x+ 4

− 2x2 − 4

(1.130)

Solution

6(x+2)2

− 2(x+2)(x−2) Factor.

6(x+2)2

· x−2x−2 −

2(x+2)(x−2) ·

x+2x+2 Multiply each fraction to get LCD as denominator.

6(x−2)

(x+2)2(x−2)− 2(x+2)

(x+2)2(x−2)Multiply.

6x−12−(2x+4)

(x+2)2(x−2)Apply distributive property.

4x−16(x+2)2(x−2)

Subtract.

4(x−4)

(x+2)2(x−2)Simplify.

(1.131)

Q&A: Do we have to use the LCD to add or subtract rational expressions?

No. Any common denominator will work, but it is easiest to use the LCD.

Try It:

Exercise 1.7.4 (Solution on p. 124.)

Subtract the rational expressions: 3x+5 −

1x−3 .

1.7.5 Simplifying Complex Rational Expressions

A complex rational expression is a rational expression that contains additional rational expressions in thenumerator, the denominator, or both. We can simplify complex rational expressions by rewriting the numer-ator and denominator as single rational expressions and dividing. The complex rational expression a

1b +c

can

be simpli�ed by rewriting the numerator as the fraction a1 and combining the expressions in the denomi-

nator as 1+bcb .We can then rewrite the expression as a multiplication problem using the reciprocal of the

denominator. We get a1 ·b

1+bc ,which is equal to ab1+bc .

How To: Given a complex rational expression, simplify it.

1.Combine the expressions in the numerator into a single rational expression by adding orsubtracting.

2.Combine the expressions in the denominator into a single rational expression by adding orsubtracting.

3.Rewrite as the numerator divided by the denominator.

Available for free at Connexions <http://legacy.cnx.org/content/col11759/1.2>

102 CHAPTER 1. PREREQUISITES

4.Rewrite as multiplication.5.Multiply.6.Simplify.

Example 1.61Simplifying Complex Rational Expressions

Simplify:y+ 1

xxy

.

SolutionBegin by combining the expressions in the numerator into one expression.

y · xx + 1x Multiply by x

x to get LCD as denominator.xyx + 1

x

xy+1x Add numerators.

(1.132)

Now the numerator is a single rational expression and the denominator is a single rational expres-sion.

xy+1xxy

(1.133)

We can rewrite this as division, and then multiplication.

xy+1x ÷ x

y

xy+1x · yx Rewrite as multiplication.

y(xy+1)x2 Multiply.

(1.134)

Try It:

Exercise 1.7.5 (Solution on p. 124.)

Simplify:xy−

yx

y

Q&A: Can a complex rational expression always be simpli�ed?

Yes. We can always rewrite a complex rational expression as a simpli�ed rational expression.

Media: Access these online resources for additional instruction and practice with rational expres-sions.

• Simplify Rational Expressions29

• Multiply and Divide Rational Expressions30

• Add and Subtract Rational Expressions31

• Simplify a Complex Fraction32

29http://openstaxcollege.org/l/simpratexpress30http://openstaxcollege.org/l/multdivratex31http://openstaxcollege.org/l/addsubratex32http://openstaxcollege.org/l/complexfract

Available for free at Connexions <http://legacy.cnx.org/content/col11759/1.2>

103

1.7.6 Key Concepts

• Rational expressions can be simpli�ed by cancelling common factors in the numerator and denominator.See Example 1.56.

• We can multiply rational expressions by multiplying the numerators and multiplying the denominators.See Example 1.57.

• To divide rational expressions, multiply by the reciprocal of the second expression. See Example 1.58.• Adding or subtracting rational expressions requires �nding a common denominator. See Example 1.59

and Example 1.60.• Complex rational expressions have fractions in the numerator or the denominator. These expressions

can be simpli�ed. See Example 1.61.

1.7.7 Section Exercises

1.7.7.1 Verbal

Exercise 1.7.6 (Solution on p. 124.)

How can you use factoring to simplify rational expressions?

Exercise 1.7.7How do you use the LCD to combine two rational expressions?

Exercise 1.7.8 (Solution on p. 124.)

Tell whether the following statement is true or false and explain why: You only need to �nd theLCD when adding or subtracting rational expressions.

1.7.7.2 Algebraic

For the following exercises, simplify the rational expressions.

Exercise 1.7.9x2−16

x2−5x+4

Exercise 1.7.10 (Solution on p. 124.)y2+10y+25y2+11y+30

Exercise 1.7.116a2−24a+24

6a2−24

Exercise 1.7.12 (Solution on p. 124.)9b2+18b+9

3b+3

Exercise 1.7.13m−12m2−144

Exercise 1.7.14 (Solution on p. 124.)2x2+7x−44x2+2x−2

Exercise 1.7.156x2+5x−4

3x2+19x+20

Exercise 1.7.16 (Solution on p. 124.)a2+9a+18a2+3a−18

Exercise 1.7.173c2+25c−183c2−23c+14

Exercise 1.7.18 (Solution on p. 124.)12n2−29n−828n2−5n−3

Available for free at Connexions <http://legacy.cnx.org/content/col11759/1.2>

104 CHAPTER 1. PREREQUISITES

For the following exercises, multiply the rational expressions and express the product in simplest form.

Exercise 1.7.19x2−x−62x2+x−6 ·

2x2+7x−15x2−9

Exercise 1.7.20 (Solution on p. 124.)c2+2c−24c2+12c+36 ·

c2−10c+24c2−8c+16

Exercise 1.7.212d2+9d−35d2+10d+21 ·

3d2+2d−213d2+14d−49

Exercise 1.7.22 (Solution on p. 124.)10h2−9h−92h2−19h+24 ·

h2−16h+645h2−37h−24

Exercise 1.7.236b2+13b+6

4b2−9 · 6b2+31b−3018b2−3b−10

Exercise 1.7.24 (Solution on p. 124.)2d2+15d+25

4d2−25 · 2d2−15d+2525d2−1

Exercise 1.7.256x2−5x−50

15x2−44x−20 ·20x2−7x−62x2+9x+10

Exercise 1.7.26 (Solution on p. 124.)t2−1

t2+4t+3 ·t2+2t−15t2−4t+3

Exercise 1.7.272n2−n−156n2+13n−5 ·

12n2−13n+34n2−15n+9

Exercise 1.7.28 (Solution on p. 124.)36x2−25

6x2+65x+50 ·3x2+32x+2018x2+27x+10

For the following exercises, divide the rational expressions.

Exercise 1.7.293y2−7y−62y2−3y−9 ÷

y2+y−22y2+y−3

Exercise 1.7.30 (Solution on p. 124.)6p2+p−128p2+18p+9 ÷

6p2−11p+42p2+11p−6

Exercise 1.7.31q2−9

q2+6q+9 ÷q2−2q−3q2+2q−3

Exercise 1.7.32 (Solution on p. 124.)18d2+77d−1827d2−15d+2 ÷

3d2+29d−449d2−15d+4

Exercise 1.7.3316x2+18x−5532x2−36x−11 ÷

2x2+17x+304x2+25x+6

Exercise 1.7.34 (Solution on p. 125.)144b2−25

72b2−6b−10 ÷18b2−21b+536b2−18b−10

Exercise 1.7.3516a2−24a+94a2+17a−15 ÷

16a2−94a2+11a+6

Exercise 1.7.36 (Solution on p. 125.)22y2+59y+1012y2+28y−5 ÷

11y2+46y+824y2−10y+1

Exercise 1.7.379x2+3x−203x2−7x+4 ÷

6x2+4x−10x2−2x+1

For the following exercises, add and subtract the rational expressions, and then simplify.

Exercise 1.7.38 (Solution on p. 125.)4x + 10

y

Available for free at Connexions <http://legacy.cnx.org/content/col11759/1.2>

105

Exercise 1.7.39122q −

63p

Exercise 1.7.40 (Solution on p. 125.)4a+1 + 5

a−3

Exercise 1.7.41c+2

3 −c−4

4

Exercise 1.7.42 (Solution on p. 125.)y+3y−2 + y−3

y+1

Exercise 1.7.43x−1x+1 −

2x+32x+1

Exercise 1.7.44 (Solution on p. 125.)3zz+1 + 2z+5

z−2

Exercise 1.7.454pp+1 −

p+14p

Exercise 1.7.46 (Solution on p. 125.)xx+1 + y

y+1

For the following exercises, simplify the rational expression.

Exercise 1.7.476y−

4x

y

Exercise 1.7.48 (Solution on p. 125.)2a + 7

b

b

Exercise 1.7.49x4−

p8

p

Exercise 1.7.50 (Solution on p. 125.)3a + b

62b3a

Exercise 1.7.513

x+1 + 2x−1

x−1x+1

Exercise 1.7.52 (Solution on p. 125.)ab−

ba

a+bab

Exercise 1.7.532x3 + 4x

7x2

Exercise 1.7.54 (Solution on p. 125.)2c

c+2 + c−1c+1

2c+1c+1

Exercise 1.7.55xy−

yx

xy + y

x

Available for free at Connexions <http://legacy.cnx.org/content/col11759/1.2>

106 CHAPTER 1. PREREQUISITES

1.7.7.3 Real-World Applications

Exercise 1.7.56 (Solution on p. 125.)

Brenda is placing tile on her bathroom �oor. The area of the �oor is 15x2−8x−7 ft2. The area of onetile isx2−2x+ 1ft2.To �nd the number of tiles needed, simplify the rational expression: 15x2−8x−7

x2−2x+1 .

Exercise 1.7.57The area of Sandy's yard is 25x2 − 625 ft2. A patch of sod has an area ofx2 − 10x+ 25 ft2. Dividethe two areas and simplify to �nd how many pieces of sod Sandy needs to cover her yard.

Exercise 1.7.58 (Solution on p. 125.)

Aaron wants to mulch his garden. His garden isx2+18x+81 ft2. One bag of mulch coversx2−81 ft2.Divide the expressions and simplify to �nd how many bags of mulch Aaron needs to mulch hisgarden.

1.7.7.4 Extensions

For the following exercises, perform the given operations and simplify.

Exercise 1.7.59x2+x−6x2−2x−3 ·

2x2−3x−9x2−x−2 ÷

10x2+27x+18x2+2x+1

Exercise 1.7.60 (Solution on p. 125.)3y2−10y+33y2+5y−2

· 2y2−3y−202y2−y−15

y−4

Exercise 1.7.614a+12a−3 + 2a−3

2a+34a2+9

a

Exercise 1.7.62 (Solution on p. 125.)x2+7x+12x2+x−6 ÷

3x2+19x+288x2−4x−24 ÷

2x2+x−33x2+4x−7

Available for free at Connexions <http://legacy.cnx.org/content/col11759/1.2>

107

1.7.8 Chapter Review Exercises

1.7.8.1 Real Numbers: Algebra Essentials (Section 1.2)

For the following exercises, perform the given operations.

Exercise 1.7.63 (Solution on p. 125.)

(5− 3 · 2)2 − 6Exercise 1.7.6464÷ (2 · 8) + 14÷ 7Exercise 1.7.65 (Solution on p. 125.)

2 · 52 + 6÷ 2For the following exercises, solve the equation.

Exercise 1.7.665x+ 9 = −11Exercise 1.7.67 (Solution on p. 125.)

2y + 42 = 64For the following exercises, simplify the expression.

Exercise 1.7.689 (y + 2)÷ 3 · 2 + 1Exercise 1.7.69 (Solution on p. 125.)

3m (4 + 7)−mFor the following exercises, identify the number as rational, irrational, whole, or natural. Choose the mostdescriptive answer.

Exercise 1.7.7011

Exercise 1.7.71 (Solution on p. 125.)

0

Exercise 1.7.7256

Exercise 1.7.73 (Solution on p. 125.)√11

1.7.8.2 Exponents and Scienti�c Notation (Section 1.3)

For the following exercises, simplify the expression.

Exercise 1.7.7422 · 24

Exercise 1.7.75 (Solution on p. 125.)45

43

Exercise 1.7.76(a2

b3

)4

Exercise 1.7.77 (Solution on p. 125.)6a2·a0

2a−4

Exercise 1.7.78(xy)4

y3 · 2x5

Available for free at Connexions <http://legacy.cnx.org/content/col11759/1.2>

108 CHAPTER 1. PREREQUISITES

Exercise 1.7.79 (Solution on p. 125.)4−2x3y−3

2x0

Exercise 1.7.80(2x2

y

)−2

Exercise 1.7.81 (Solution on p. 125.)(16a3

b2

) (4ab−1

)−2

Exercise 1.7.82Write the number in standard notation: 2.1314 × 10−6

Exercise 1.7.83 (Solution on p. 126.)

Write the number in scienti�c notation: 16,340,000

1.7.8.3 Radicals and Rational Expressions (Section 1.4)

For the following exercises, �nd the principal square root.

Exercise 1.7.84√121

Exercise 1.7.85 (Solution on p. 126.)√196

Exercise 1.7.86√361

Exercise 1.7.87 (Solution on p. 126.)√75

Exercise 1.7.88√162

Exercise 1.7.89 (Solution on p. 126.)√3225

Exercise 1.7.90√8081

Exercise 1.7.91 (Solution on p. 126.)√49

1250

Exercise 1.7.922

4+√

2

Exercise 1.7.93 (Solution on p. 126.)

4√

3 + 6√

3Exercise 1.7.9412√

5− 13√

5Exercise 1.7.95 (Solution on p. 126.)5√−243

Exercise 1.7.963√2503√−8

Available for free at Connexions <http://legacy.cnx.org/content/col11759/1.2>

109

1.7.8.4 Polynomials (Section 1.5)

For the following exercises, perform the given operations and simplify.

Exercise 1.7.97 (Solution on p. 126.)(3x3 + 2x− 1

)+(4x2 − 2x+ 7

)Exercise 1.7.98(2y + 1)−

(2y2 − 2y − 5

)Exercise 1.7.99 (Solution on p. 126.)(2x2 + 3x− 6

)+(3x2 − 4x+ 9

)Exercise 1.7.100(6a2 + 3a+ 10

)−(6a2 − 3a+ 5

)Exercise 1.7.101 (Solution on p. 126.)

(k + 3) (k − 6)Exercise 1.7.102(2h+ 1) (3h− 2)Exercise 1.7.103 (Solution on p. 126.)

(x+ 1)(x2 + 1

)Exercise 1.7.104(m− 2)

(m2 + 2m− 3

)Exercise 1.7.105 (Solution on p. 126.)

(a+ 2b) (3a− b)Exercise 1.7.106(x+ y) (x− y)

1.7.8.5 Factoring Polynomials (Section 1.6)

For the following exercises, �nd the greatest common factor.

Exercise 1.7.107 (Solution on p. 126.)

81p+ 9pq − 27p2q2

Exercise 1.7.10812x2y + 4xy2 − 18xyExercise 1.7.109 (Solution on p. 126.)

88a3b+ 4a2b− 144a2

For the following exercises, factor the polynomial.

Exercise 1.7.1102x2 − 9x− 18Exercise 1.7.111 (Solution on p. 126.)

8a2 + 30a− 27Exercise 1.7.112d2 − 5d− 66Exercise 1.7.113 (Solution on p. 126.)

x2 + 10x+ 25Exercise 1.7.114y2 − 6y + 9Exercise 1.7.115 (Solution on p. 126.)

4h2 − 12hk + 9k2

Available for free at Connexions <http://legacy.cnx.org/content/col11759/1.2>

110 CHAPTER 1. PREREQUISITES

Exercise 1.7.116361x2 − 121Exercise 1.7.117 (Solution on p. 126.)

p3 + 216Exercise 1.7.1188x3 − 125Exercise 1.7.119 (Solution on p. 126.)

64q3 − 27p3

Exercise 1.7.1204x(x− 1)−

14 + 3(x− 1)

34

Exercise 1.7.121 (Solution on p. 126.)

3p(p+ 3)13 − 8(p+ 3)

43

Exercise 1.7.1224r(2r − 1)−

23 − 5(2r − 1)

13

1.7.8.6 Rational Expressions (Section 1.7)

For the following exercises, simplify the expression.

Exercise 1.7.123 (Solution on p. 126.)x2−x−12x2−8x+16

Exercise 1.7.1244y2−25

4y2−20y+25

Exercise 1.7.125 (Solution on p. 126.)2a2−a−32a2−6a−8 ·

5a2−19a−410a2−13a−3

Exercise 1.7.126d−4d2−9 ·

d−3d2−16

Exercise 1.7.127 (Solution on p. 126.)m2+5m+62m2−5m−3 ÷

2m2+3m−94m2−4m−3

Exercise 1.7.1284d2−7d−2

6d2−17d+10 ÷8d2+6d+16d2+7d−10

Exercise 1.7.129 (Solution on p. 126.)10x + 6

y

Exercise 1.7.13012

a2+2a+1 −3

a2−1

Exercise 1.7.131 (Solution on p. 126.)1d + 2

c6c+12d

dc

Exercise 1.7.1323x−

7y

2x

Available for free at Connexions <http://legacy.cnx.org/content/col11759/1.2>

111

1.7.9 Chapter Practice Test

For the following exercises, identify the number as rational, irrational, whole, or natural. Choose the mostdescriptive answer.

Exercise 1.7.133 (Solution on p. 127.)

−13Exercise 1.7.134√

2For the following exercises, evaluate the equations.

Exercise 1.7.135 (Solution on p. 127.)

2 (x+ 3)− 12 = 18Exercise 1.7.136y(3 + 3)2 − 26 = 10Exercise 1.7.137 (Solution on p. 127.)

Write the number in standard notation:3.1415 × 106

Exercise 1.7.138Write the number in scienti�c notation: 0.0000000212.

For the following exercises, simplify the expression.

Exercise 1.7.139 (Solution on p. 127.)

−2 · (2 + 3 · 2)2 + 144Exercise 1.7.1404 (x+ 3)− (6x+ 2)Exercise 1.7.141 (Solution on p. 127.)

35 · 3−3

Exercise 1.7.142 (23

)3

Exercise 1.7.143 (Solution on p. 127.)

8x3

(2x)2

Exercise 1.7.144(16y0

)2y−2

Exercise 1.7.145 (Solution on p. 127.)√441

Exercise 1.7.146√490

Exercise 1.7.147 (Solution on p. 127.)√9x16

Exercise 1.7.148√121b2

1+√b

Exercise 1.7.149 (Solution on p. 127.)

6√

24 + 7√

54− 12√

6Exercise 1.7.1503√−84√625

Available for free at Connexions <http://legacy.cnx.org/content/col11759/1.2>

112 CHAPTER 1. PREREQUISITES

Exercise 1.7.151 (Solution on p. 127.)(13q3 + 2q2 − 3

)−(6q2 + 5q − 3

)Exercise 1.7.152(6p2 + 2p+ 1

)+(9p2 − 1

)Exercise 1.7.153 (Solution on p. 127.)

(n− 2)(n2 − 4n+ 4

)Exercise 1.7.154(a− 2b) (2a+ b)

For the following exercises, factor the polynomial.

Exercise 1.7.155 (Solution on p. 127.)

16x2 − 81Exercise 1.7.156y2 + 12y + 36Exercise 1.7.157 (Solution on p. 127.)

27c3 − 1331Exercise 1.7.1583x(x− 6)−

14 + 2(x− 6)

34

For the following exercises, simplify the expression.

Exercise 1.7.159 (Solution on p. 127.)2z2+7z+3z2−9 · 4z2−15z+9

4z2−1

Exercise 1.7.160xy + 2

x

Exercise 1.7.161 (Solution on p. 127.)a2b−

2b9a

3a−2b6a

Available for free at Connexions <http://legacy.cnx.org/content/col11759/1.2>

113

Solutions to Exercises in Chapter 1

Solution to Exercise 1.2.1 (p. 11)

a. 111

b. 31

c. − 41

Solution to Exercise 1.2.2 (p. 12)

a. 4 (or 4.0), terminating;b. 0.615384,repeating;c. �0.85, terminating

Solution to Exercise 1.2.3 (p. 13)

a. rational and repeating;b. rational and terminating;c. irrational;d. rational and repeating;e. irrational

Solution to Exercise 1.2.4 (p. 14)

a. positive, irrational; rightb. negative, rational; leftc. positive, rational; rightd. negative, irrational; lefte. positive, rational; right

Solution to Exercise 1.2.5 (p. 16)

N W I Q Q'

a. − 357 X X

b. 0 X X X

c.√

169 X X X X

d.√

24 X

e. 4.763763763... X

Table 1.10

Solution to Exercise 1.2.6 (p. 18)

a. 10b. 2c. 4.5d. 25e. 26

Solution to Exercise 1.2.7 (p. 23)

a. 11, commutative property of multiplication, associative property of multiplication, inverse property ofmultiplication, identity property of multiplication;

Available for free at Connexions <http://legacy.cnx.org/content/col11759/1.2>

114 CHAPTER 1. PREREQUISITES

b. 33, distributive property;c. 26, distributive property;d. 4

9 , commutative property of addition, associative property of addition, inverse property of addition,identity property of addition;

e. 0, distributive property, inverse property of addition, identity property of addition

Solution to Exercise 1.2.8 (p. 24)

Constants Variables

a.2πr (r + h) 2, π r, h

b. 2(L + W) 2 L, W

c. 4y3 + y 4 y

Table 1.11

Solution to Exercise 1.2.9 (p. 25)

a. 5;b. 11;c. 9;d. 26

Solution to Exercise 1.2.10 (p. 26)

a. 4;b. 11;c. 121

3 π;d. 1728;e. 3

Solution to Exercise 1.2.11 (p. 27)1,152 cm2

Solution to Exercise 1.2.12 (p. 29)

a. − 2y − 2z or − 2 (y + z) ;b. 2

t − 1;c. 3pq − 4p+ q;d. 7r − 2s+ 6

Solution to Exercise 1.2.13 (p. 30)A = P (1 + rt)Solution to Exercise 1.2.14 (p. 30)irrational number. The square root of two does not terminate, and it does not repeat a pattern. It cannotbe written as a quotient of two integers, so it is irrational.Solution to Exercise 1.2.16 (p. 30)The Associative Properties state that the sum or product of multiple numbers can be grouped di�erentlywithout a�ecting the result. This is because the same operation is performed (either addition or subtraction),so the terms can be re-ordered.Solution to Exercise 1.2.18 (p. 31)−6

Available for free at Connexions <http://legacy.cnx.org/content/col11759/1.2>

115

Solution to Exercise 1.2.20 (p. 31)−2Solution to Exercise 1.2.22 (p. 31)−9Solution to Exercise 1.2.24 (p. 31)9Solution to Exercise 1.2.26 (p. 31)4Solution to Exercise 1.2.28 (p. 31)4Solution to Exercise 1.2.30 (p. 31)0Solution to Exercise 1.2.32 (p. 31)9Solution to Exercise 1.2.34 (p. 31)25Solution to Exercise 1.2.36 (p. 31)−6Solution to Exercise 1.2.38 (p. 31)17Solution to Exercise 1.2.40 (p. 32)4Solution to Exercise 1.2.42 (p. 32)−4Solution to Exercise 1.2.44 (p. 32)−6Solution to Exercise 1.2.46 (p. 32)±1Solution to Exercise 1.2.48 (p. 32)2Solution to Exercise 1.2.50 (p. 32)2Solution to Exercise 1.2.52 (p. 32)−14y − 11Solution to Exercise 1.2.54 (p. 32)−4b+ 1Solution to Exercise 1.2.56 (p. 32)43z − 3Solution to Exercise 1.2.58 (p. 32)9y + 45Solution to Exercise 1.2.60 (p. 33)−6b+ 6Solution to Exercise 1.2.62 (p. 33)16x3Solution to Exercise 1.2.64 (p. 33)9xSolution to Exercise 1.2.66 (p. 33)12 (40− 10) + 5Solution to Exercise 1.2.68 (p. 33)irrational number

Available for free at Connexions <http://legacy.cnx.org/content/col11759/1.2>

116 CHAPTER 1. PREREQUISITES

Solution to Exercise 1.2.70 (p. 33)g + 400− 2 (600) = 1200Solution to Exercise 1.2.72 (p. 33)inverse property of additionSolution to Exercise 1.2.74 (p. 34)68.4Solution to Exercise 1.2.76 (p. 34)trueSolution to Exercise 1.2.78 (p. 34)irrationalSolution to Exercise 1.2.80 (p. 34)rationalSolution to Exercise 1.3.1 (p. 36)

a. k15

b.(

2y

)5

c. t14

Solution to Exercise 1.3.2 (p. 37)

a. s7

b. (−3)5

c.(ef2)2

Solution to Exercise 1.3.3 (p. 38)

a. (3y)24

b. t35

c. (−g)16

Solution to Exercise 1.3.4 (p. 40)

a. 1b. 1

2c. 1d. 1

Solution to Exercise 1.3.5 (p. 41)

a. 1(−3t)6

b. 1f3

c. 25k3

Solution to Exercise 1.3.6 (p. 42)

a. t−5 = 1t5

b. 125

Solution to Exercise 1.3.7 (p. 43)

a. g10h15

b. 125t3

c. −27y15

d. 1a18b21

e. r12

s8

Available for free at Connexions <http://legacy.cnx.org/content/col11759/1.2>

117

Solution to Exercise 1.3.8 (p. 44)

a. b15

c3

b. 625u32

c. −1w105

d. q24

p32

e. 1c20d12

Solution to Exercise 1.3.9 (p. 46)

a. v6

8u3

b. 1x3

c. e4

f4

d. 27rs

e. 1f. 16h10

49

Solution to Exercise 1.3.10 (p. 48)

a. $1.52× 105

b. 7.158× 109

c. $8.55× 1013

d. 3.34× 10−9

e. 7.15× 10−8

Solution to Exercise 1.3.11 (p. 49)

a. 703, 000b. −816, 000, 000, 000c. −0.000 000 000 000 39d. 0.000008

Solution to Exercise 1.3.12 (p. 50)

a. −8.475 × 106

b. 8 × 10−8

c. 2.976 × 1013

d. −4.3 × 106

e. ≈ 1.24 × 1015

Solution to Exercise 1.3.13 (p. 51)Number of cells: 3× 1013;length of a cell: 8× 10−6 m; total length: 2.4× 108 m or 240, 000, 000m.Solution to Exercise 1.3.14 (p. 52)No, the two expressions are not the same. An exponent tells how many times you multiply the base. So 23 isthe same as 2× 2× 2,which is 8. 32 is the same as 3× 3,which is 9.Solution to Exercise 1.3.16 (p. 52)It is a method of writing very small and very large numbers.Solution to Exercise 1.3.18 (p. 52)81Solution to Exercise 1.3.20 (p. 52)243Solution to Exercise 1.3.22 (p. 52)116Solution to Exercise 1.3.24 (p. 53)111

Available for free at Connexions <http://legacy.cnx.org/content/col11759/1.2>

118 CHAPTER 1. PREREQUISITES

Solution to Exercise 1.3.26 (p. 53)1Solution to Exercise 1.3.28 (p. 53)49

Solution to Exercise 1.3.30 (p. 53)1240

Solution to Exercise 1.3.32 (p. 53)179

Solution to Exercise 1.3.34 (p. 53)3.14 × 10−5

Solution to Exercise 1.3.36 (p. 53)16,000,000,000Solution to Exercise 1.3.38 (p. 54)a4

Solution to Exercise 1.3.40 (p. 54)b6c8

Solution to Exercise 1.3.42 (p. 54)ab2d3

Solution to Exercise 1.3.44 (p. 54)m4

Solution to Exercise 1.3.46 (p. 54)q5

p6

Solution to Exercise 1.3.48 (p. 54)y21

x14

Solution to Exercise 1.3.50 (p. 54)25Solution to Exercise 1.3.52 (p. 54)72a2

Solution to Exercise 1.3.54 (p. 54)c3

b9

Solution to Exercise 1.3.56 (p. 54)y

81z6

Solution to Exercise 1.3.58 (p. 55)0.00135 mSolution to Exercise 1.3.60 (p. 55)1.0995× 1012

Solution to Exercise 1.3.62 (p. 55)0.00000000003397 in.Solution to Exercise 1.3.64 (p. 55)12,230,590,464m66

Solution to Exercise 1.3.66 (p. 55)a14

1296Solution to Exercise 1.3.68 (p. 55)na9cSolution to Exercise 1.3.70 (p. 55)

1a6b6c6

Solution to Exercise 1.3.72 (p. 56)0.000000000000000000000000000000000662606957Solution to Exercise 1.4.1 (p. 58)

a. 15

Available for free at Connexions <http://legacy.cnx.org/content/col11759/1.2>

119

b. 3c. 4d. 17

Solution to Exercise 1.4.2 (p. 59)5|x||y|

√2yz.Notice the absolute value signs around x and y? That's because their value must be positive!

Solution to Exercise 1.4.3 (p. 59)10|x|Solution to Exercise 1.4.4 (p. 60)x√

23y2 .We do not need the absolute value signs for y2 because that term will always be nonnegative.

Solution to Exercise 1.4.5 (p. 60)b4√

3abSolution to Exercise 1.4.6 (p. 61)13√

5Solution to Exercise 1.4.7 (p. 61)0Solution to Exercise 1.4.8 (p. 62)6√

6Solution to Exercise 1.4.9 (p. 63)14− 7

√3

Solution to Exercise 1.4.10 (p. 64)

a. −6b. 6c. 88 3

√9

Solution to Exercise 1.4.11 (p. 65)(√9)5

= 35 = 243Solution to Exercise 1.4.12 (p. 65)

x(5y)92

Solution to Exercise 1.4.13 (p. 66)

28x2315

Solution to Exercise 1.4.14 (p. 67)When there is no index, it is assumed to be 2 or the square root. The expression would only be equal tothe radicand if the index were 1.Solution to Exercise 1.4.16 (p. 67)The principal square root is the nonnegative root of the number.Solution to Exercise 1.4.18 (p. 67)16Solution to Exercise 1.4.20 (p. 67)10Solution to Exercise 1.4.22 (p. 67)14Solution to Exercise 1.4.24 (p. 67)7√

2Solution to Exercise 1.4.26 (p. 67)9√

55

Solution to Exercise 1.4.28 (p. 67)25Solution to Exercise 1.4.30 (p. 67)√

2

Available for free at Connexions <http://legacy.cnx.org/content/col11759/1.2>

120 CHAPTER 1. PREREQUISITES

Solution to Exercise 1.4.32 (p. 67)2√

6Solution to Exercise 1.4.34 (p. 68)5√

6Solution to Exercise 1.4.36 (p. 68)6√

35Solution to Exercise 1.4.38 (p. 68)215Solution to Exercise 1.4.40 (p. 68)6√

1019

Solution to Exercise 1.4.42 (p. 68)

− 1+√

172

Solution to Exercise 1.4.44 (p. 68)7 3√

2Solution to Exercise 1.4.46 (p. 68)15√

5Solution to Exercise 1.4.48 (p. 68)20x2

Solution to Exercise 1.4.50 (p. 68)7√p

Solution to Exercise 1.4.52 (p. 69)18m2

√m

Solution to Exercise 1.4.54 (p. 69)2b√a

Solution to Exercise 1.4.56 (p. 69)15x7Solution to Exercise 1.4.58 (p. 69)5y4√

2Solution to Exercise 1.4.60 (p. 69)4√

7d7d

Solution to Exercise 1.4.62 (p. 69)2√

2+2√

6x1−3x

Solution to Exercise 1.4.64 (p. 69)−w√

2wSolution to Exercise 1.4.66 (p. 69)3√x−√

3x2

Solution to Exercise 1.4.68 (p. 69)5n5√

5Solution to Exercise 1.4.70 (p. 69)9√m

19mSolution to Exercise 1.4.72 (p. 69)23dSolution to Exercise 1.4.74 (p. 70)3

4√2x2

2Solution to Exercise 1.4.76 (p. 70)6z 3√

2Solution to Exercise 1.4.78 (p. 70)500 feetSolution to Exercise 1.4.80 (p. 70)−5√

2−67

Available for free at Connexions <http://legacy.cnx.org/content/col11759/1.2>

121

Solution to Exercise 1.4.82 (p. 70)√mnc

a9cmnSolution to Exercise 1.4.84 (p. 70)2√

2x+√

24

Solution to Exercise 1.4.86 (p. 70)√3

3Solution to Exercise 1.5.1 (p. 73)The degree is 6, the leading term is − x6,and the leading coe�cient is − 1.Solution to Exercise 1.5.2 (p. 74)2x3 + 7x2 − 4x− 3Solution to Exercise 1.5.3 (p. 74)−11x3 − x2 + 7x− 9Solution to Exercise 1.5.4 (p. 75)3x4 − 10x3 − 8x2 + 21x+ 14Solution to Exercise 1.5.5 (p. 77)3x2 + 16x− 35Solution to Exercise 1.5.6 (p. 78)16x2 − 8x+ 1Solution to Exercise 1.5.7 (p. 79)4x2 − 49Solution to Exercise 1.5.8 (p. 79)6x2 + 21xy − 29x− 7y + 9Solution to Exercise 1.5.9 (p. 80)The statement is true. In standard form, the polynomial with the highest value exponent is placed �rstand is the leading term. The degree of a polynomial is the value of the highest exponent, which in standardform is also the exponent of the leading term.Solution to Exercise 1.5.11 (p. 80)Use the distributive property, multiply, combine like terms, and simplify.Solution to Exercise 1.5.13 (p. 81)2Solution to Exercise 1.5.15 (p. 81)8Solution to Exercise 1.5.17 (p. 81)2Solution to Exercise 1.5.19 (p. 81)4x2 + 3x+ 19Solution to Exercise 1.5.21 (p. 81)3w2 + 30w + 21Solution to Exercise 1.5.23 (p. 81)11b4 − 9b3 + 12b2 − 7b+ 8Solution to Exercise 1.5.25 (p. 81)24x2 − 4x− 8Solution to Exercise 1.5.27 (p. 81)24b4 − 48b2 + 24Solution to Exercise 1.5.29 (p. 81)99v2 − 202v + 99Solution to Exercise 1.5.31 (p. 81)8n3 − 4n2 + 72n− 36Solution to Exercise 1.5.33 (p. 81)9y2 − 42y + 49

Available for free at Connexions <http://legacy.cnx.org/content/col11759/1.2>

122 CHAPTER 1. PREREQUISITES

Solution to Exercise 1.5.35 (p. 82)16p2 + 72p+ 81Solution to Exercise 1.5.37 (p. 82)9y2 − 36y + 36Solution to Exercise 1.5.39 (p. 82)16c2 − 1Solution to Exercise 1.5.41 (p. 82)225n2 − 36Solution to Exercise 1.5.43 (p. 82)−16m2 + 16Solution to Exercise 1.5.45 (p. 82)121q2 − 100Solution to Exercise 1.5.47 (p. 82)16t4 + 4t3 − 32t2 − t+ 7Solution to Exercise 1.5.49 (p. 82)y3 − 6y2 − y + 18Solution to Exercise 1.5.51 (p. 82)3p3 − p2 − 12p+ 10Solution to Exercise 1.5.53 (p. 82)a2 − b2Solution to Exercise 1.5.55 (p. 83)16t2 − 40tu+ 25u2

Solution to Exercise 1.5.57 (p. 83)4t2 + x2 + 4t− 5tx− xSolution to Exercise 1.5.59 (p. 83)24r2 + 22rd− 7d2

Solution to Exercise 1.5.61 (p. 83)32x2 − 4x− 3m2

Solution to Exercise 1.5.63 (p. 83)32t3 − 100t2 + 40t+ 38Solution to Exercise 1.5.65 (p. 83)a4 + 4a3c− 16ac3 − 16c4

Solution to Exercise 1.6.1 (p. 85)(b2 − a

)(x+ 6)

Solution to Exercise 1.6.2 (p. 86)(x− 6) (x− 1)Solution to Exercise 1.6.3 (p. 88)a. (2x+ 3) (x+ 3) b. (3x− 1) (2x+ 1)Solution to Exercise 1.6.4 (p. 88)

(7x− 1)2

Solution to Exercise 1.6.5 (p. 89)(9y + 10) (9y − 10)Solution to Exercise 1.6.6 (p. 90)(6a+ b)

(36a2 − 6ab+ b2

)Solution to Exercise 1.6.7 (p. 91)(10x− 1)

(100x2 + 10x+ 1

)Solution to Exercise 1.6.8 (p. 91)

(5a− 1)−14 (17a− 2)

Available for free at Connexions <http://legacy.cnx.org/content/col11759/1.2>

123

Solution to Exercise 1.6.9 (p. 92)The terms of a polynomial do not have to have a common factor for the entire polynomial to be factorable.For example, 4x2 and − 9y2 don't have a common factor, but the whole polynomial is still factorable: 4x2 −9y2 = (2x+ 3y) (2x− 3y) .Solution to Exercise 1.6.11 (p. 92)Divide thex term into the sum of two terms, factor each portion of the expression separately, and thenfactor out the GCF of the entire expression.Solution to Exercise 1.6.13 (p. 92)7mSolution to Exercise 1.6.15 (p. 92)10m3

Solution to Exercise 1.6.17 (p. 93)ySolution to Exercise 1.6.19 (p. 93)(2a− 3) (a+ 6)Solution to Exercise 1.6.21 (p. 93)(3n− 11) (2n+ 1)Solution to Exercise 1.6.23 (p. 93)(p+ 1) (2p− 7)Solution to Exercise 1.6.25 (p. 93)(5h+ 3) (2h− 3)Solution to Exercise 1.6.27 (p. 93)(9d− 1) (d− 8)Solution to Exercise 1.6.29 (p. 93)(12t+ 13) (t− 1)Solution to Exercise 1.6.31 (p. 93)(4x+ 10) (4x− 10)Solution to Exercise 1.6.33 (p. 93)(11p+ 13) (11p− 13)Solution to Exercise 1.6.35 (p. 93)(19d+ 9) (19d− 9)Solution to Exercise 1.6.37 (p. 94)(12b+ 5c) (12b− 5c)Solution to Exercise 1.6.39 (p. 94)

(7n+ 12)2

Solution to Exercise 1.6.41 (p. 94)

(15y + 4)2

Solution to Exercise 1.6.43 (p. 94)

(5p− 12)2

Solution to Exercise 1.6.45 (p. 94)(x+ 6)

(x2 − 6x+ 36

)Solution to Exercise 1.6.47 (p. 94)(5a+ 7)

(25a2 − 35a+ 49

)Solution to Exercise 1.6.49 (p. 94)(4x− 5)

(16x2 + 20x+ 25

)Solution to Exercise 1.6.51 (p. 94)(5r + 12s)

(25r2 − 60rs+ 144s2

)Solution to Exercise 1.6.53 (p. 94)

(2c+ 3)−14 (−7c− 15)

Solution to Exercise 1.6.55 (p. 94)

(x+ 2)−25 (19x+ 10)

Available for free at Connexions <http://legacy.cnx.org/content/col11759/1.2>

124 CHAPTER 1. PREREQUISITES

Solution to Exercise 1.6.57 (p. 94)

(2z − 9)−32 (27z − 99)

Solution to Exercise 1.6.59 (p. 95)(14x− 3) (7x+ 9)Solution to Exercise 1.6.61 (p. 95)(3x+ 5) (3x− 5)Solution to Exercise 1.6.63 (p. 96)

(2x+ 5)2(2x− 5)2

Solution to Exercise 1.6.65 (p. 96)(4z2 + 49a2

)(2z + 7a) (2z − 7a)

Solution to Exercise 1.6.67 (p. 96)1

(4x+9)(4x−9)(2x+3)

Solution to Exercise 1.7.1 (p. 97)1

x+6Solution to Exercise 1.7.2 (p. 98)(x+5)(x+6)(x+2)(x+4)

Solution to Exercise 1.7.3 (p. 99)1Solution to Exercise 1.7.4 (p. 101)

2(x−7)(x+5)(x−3)

Solution to Exercise 1.7.5 (p. 102)x2−y2

xy2

Solution to Exercise 1.7.6 (p. 103)You can factor the numerator and denominator to see if any of the terms can cancel one another out.Solution to Exercise 1.7.8 (p. 103)True. Multiplication and division do not require �nding the LCD because the denominators can be combinedthrough those operations, whereas addition and subtraction require like terms.Solution to Exercise 1.7.10 (p. 103)y+5y+6

Solution to Exercise 1.7.12 (p. 103)3b+ 3Solution to Exercise 1.7.14 (p. 103)x+42x+2Solution to Exercise 1.7.16 (p. 103)a+3a−3Solution to Exercise 1.7.18 (p. 103)3n−87n−3Solution to Exercise 1.7.20 (p. 104)c−6c+6Solution to Exercise 1.7.22 (p. 104)1Solution to Exercise 1.7.24 (p. 104)d2−2525d2−1Solution to Exercise 1.7.26 (p. 104)t+5t+3Solution to Exercise 1.7.28 (p. 104)6x−56x+5Solution to Exercise 1.7.30 (p. 104)p+64p+3

Available for free at Connexions <http://legacy.cnx.org/content/col11759/1.2>

125

Solution to Exercise 1.7.32 (p. 104)2d+9d+11Solution to Exercise 1.7.34 (p. 104)12b+53b−1Solution to Exercise 1.7.36 (p. 104)4y−1y+4

Solution to Exercise 1.7.38 (p. 104)10x+4yxy

Solution to Exercise 1.7.40 (p. 105)9a−7

a2−2a−3Solution to Exercise 1.7.42 (p. 105)2y2−y+9y2−y−2

Solution to Exercise 1.7.44 (p. 105)5z2+z+5z2−z−2Solution to Exercise 1.7.46 (p. 105)x+2xy+yx+xy+y+1

Solution to Exercise 1.7.48 (p. 105)2b+7aab2

Solution to Exercise 1.7.50 (p. 105)18+ab

4bSolution to Exercise 1.7.52 (p. 105)a− bSolution to Exercise 1.7.54 (p. 105)3c2+3c−22c2+5c+2Solution to Exercise 1.7.56 (p. 106)

15x+7x−1

Solution to Exercise 1.7.58 (p. 106)x+9x−9Solution to Exercise 1.7.60 (p. 106)

1y+2

Solution to Exercise 1.7.62 (p. 106)4Solution to Exercise 1.7.63 (p. 107)−5Solution to Exercise 1.7.65 (p. 107)53Solution to Exercise 1.7.67 (p. 107)y = 24Solution to Exercise 1.7.69 (p. 107)32mSolution to Exercise 1.7.71 (p. 107)wholeSolution to Exercise 1.7.73 (p. 107)irrationalSolution to Exercise 1.7.75 (p. 107)16Solution to Exercise 1.7.77 (p. 107)a6

Solution to Exercise 1.7.79 (p. 107)x3

32y3

Available for free at Connexions <http://legacy.cnx.org/content/col11759/1.2>

126 CHAPTER 1. PREREQUISITES

Solution to Exercise 1.7.81 (p. 108)aSolution to Exercise 1.7.83 (p. 108)1.634 × 107

Solution to Exercise 1.7.85 (p. 108)14Solution to Exercise 1.7.87 (p. 108)5√

3Solution to Exercise 1.7.89 (p. 108)4√

25

Solution to Exercise 1.7.91 (p. 108)7√

250Solution to Exercise 1.7.93 (p. 108)10√

3Solution to Exercise 1.7.95 (p. 108)−3Solution to Exercise 1.7.97 (p. 109)3x3 + 4x2 + 6Solution to Exercise 1.7.99 (p. 109)5x2 − x+ 3Solution to Exercise 1.7.101 (p. 109)k2 − 3k − 18Solution to Exercise 1.7.103 (p. 109)x3 + x2 + x+ 1Solution to Exercise 1.7.105 (p. 109)3a2 + 5ab− 2b2

Solution to Exercise 1.7.107 (p. 109)9pSolution to Exercise 1.7.109 (p. 109)4a2

Solution to Exercise 1.7.111 (p. 109)(4a− 3) (2a+ 9)Solution to Exercise 1.7.113 (p. 109)

(x+ 5)2

Solution to Exercise 1.7.115 (p. 109)

(2h− 3k)2

Solution to Exercise 1.7.117 (p. 110)(p+ 6)

(p2 − 6p+ 36

)Solution to Exercise 1.7.119 (p. 110)(4q − 3p)

(16q2 + 12pq + 9p2

)Solution to Exercise 1.7.121 (p. 110)

(p+ 3)13 (−5p− 24)

Solution to Exercise 1.7.123 (p. 110)x+3x−4Solution to Exercise 1.7.125 (p. 110)12Solution to Exercise 1.7.127 (p. 110)m+2m−3Solution to Exercise 1.7.129 (p. 110)6x+10yxy

Available for free at Connexions <http://legacy.cnx.org/content/col11759/1.2>

127

Solution to Exercise 1.7.131 (p. 110)16Solution to Exercise 1.7.133 (p. 111)rationalSolution to Exercise 1.7.135 (p. 111)x = 12Solution to Exercise 1.7.137 (p. 111)3,141,500Solution to Exercise 1.7.139 (p. 111)16Solution to Exercise 1.7.141 (p. 111)9Solution to Exercise 1.7.143 (p. 111)2xSolution to Exercise 1.7.145 (p. 111)21Solution to Exercise 1.7.147 (p. 111)3√x

4Solution to Exercise 1.7.149 (p. 111)21√

6Solution to Exercise 1.7.151 (p. 112)13q3 − 4q2 − 5qSolution to Exercise 1.7.153 (p. 112)n3 − 6n2 + 12n− 8Solution to Exercise 1.7.155 (p. 112)(4x+ 9) (4x− 9)Solution to Exercise 1.7.157 (p. 112)(3c− 11)

(9c2 + 33c+ 121

)Solution to Exercise 1.7.159 (p. 112)4z−32z−1Solution to Exercise 1.7.161 (p. 112)3a+2b

3b

Available for free at Connexions <http://legacy.cnx.org/content/col11759/1.2>

128 CHAPTER 1. PREREQUISITES

Available for free at Connexions <http://legacy.cnx.org/content/col11759/1.2>

Chapter 2

Equations and Inequalities

2.1 Introduction to Equations and Inequalities1

Figure 2.1

1This content is available online at <http://legacy.cnx.org/content/m51251/1.2/>.

Available for free at Connexions <http://legacy.cnx.org/content/col11759/1.2>

129

130 CHAPTER 2. EQUATIONS AND INEQUALITIES

For most people, the term territorial possession indicates restrictions, usually dealing with trespassing orrite of passage and takes place in some foreign location. What most Americans do not realize is that fromSeptember through December, territorial possession dominates our lifestyles while watching the NFL. In thisarea, territorial possession is governed by the referees who make their decisions based on what the chainsreveal. If the ball is at point A (x1, y1) ,then it is up to the quarterback to decide which route to pointB (x2, y2) ,the end zone, is most feasible.

2.2 The Rectangular Coordinate Systems and Graphs2

2.3 Linear Equations in One Variable3

Caroline is a full-time college student planning a spring break vacation. To earn enough money for the trip,she has taken a part-time job at the local bank that pays $15.00/hr, and she opened a savings account withan initial deposit of $400 on January 15. She arranged for direct deposit of her payroll checks. If springbreak begins March 20 and the trip will cost approximately $2,500, how many hours will she have to workto earn enough to pay for her vacation? If she can only work 4 hours per day, how many days per week willshe have to work? How many weeks will it take? In this section, we will investigate problems like this andothers, which generate graphs like the line in Figure 2.2.

Figure 2.2

2This content is available online at <http://legacy.cnx.org/content/m51252/1.1/>.3This content is available online at <http://legacy.cnx.org/content/m51253/1.2/>.

Available for free at Connexions <http://legacy.cnx.org/content/col11759/1.2>

131

2.3.1 Solving Linear Equations in One Variable

A linear equation is an equation of a straight line, written in one variable. The only power of the variableis 1. Linear equations in one variable may take the form ax + b = 0 and are solved using basic algebraicoperations.

We begin by classifying linear equations in one variable as one of three types: identity, conditional, orinconsistent. An identity equation is true for all values of the variable. Here is an example of an identityequation.

3x = 2x+ x (2.1)

The solution set consists of all values that make the equation true. For this equation, the solution set isall real numbers because any real number substituted forxwill make the equation true.

A conditional equation is true for only some values of the variable. For example, if we are to solve theequation 5x+ 2 = 3x− 6,we have the following:

5x+ 2 = 3x− 6

2x = −8

x = −4

(2.2)

The solution set consists of one number: {−4}. It is the only solution and, therefore, we have solved aconditional equation.

An inconsistent equation results in a false statement. For example, if we are to solve 5x − 15 =5 (x− 4) ,we have the following:

5x− 15 = 5x− 20

5x− 15− 5x = 5x− 20− 5x Subtract 5x from both sides.

− 15 6= −20 False statement

(2.3)

Indeed, − 15 6= − 20.There is no solution because this is an inconsistent equation.Solving linear equations in one variable involves the fundamental properties of equality and basic algebraic

operations. A brief review of those operations follows.

A General Note: A linear equation in one variable can be written in the form

ax+ b = 0 (2.4)

where a and b are real numbers, a 6= 0.

How To: Given a linear equation in one variable, use algebra to solve it.

The following steps are used to manipulate an equation and isolate the unknown variable, so thatthe last line readsx = _________,if x is the unknown. There is no set order, as the steps useddepend on what is given:

1.We may add, subtract, multiply, or divide an equation by a number or an expression as longas we do the same thing to both sides of the equal sign. Note that we cannot divide by zero.

2.Apply the distributive property as needed: a (b+ c) = ab+ ac.3.Isolate the variable on one side of the equation.4.When the variable is multiplied by a coe�cient in the �nal stage, multiply both sides of theequation by the reciprocal of the coe�cient.

Available for free at Connexions <http://legacy.cnx.org/content/col11759/1.2>

132 CHAPTER 2. EQUATIONS AND INEQUALITIES

Example 2.1Solving an Equation in One VariableSolve the following equation: 2x+ 7 = 19.

SolutionThis equation can be written in the form ax + b = 0 by subtracting 19 from both sides. However,we may proceed to solve the equation in its original form by performing algebraic operations.

2x+ 7 = 19

2x = 12 Subtract 7 from both sides.

x = 6 Multiply both sides by 12 or divide by 2.

(2.5)

The solution isx = 6.

Try It:

Exercise 2.3.1 (Solution on p. 245.)

Solve the linear equation in one variable: 2x+ 1 = −9.

Example 2.2Solving an Equation Algebraically When the Variable Appears on Both SidesSolve the following equation: 4 (x− 3) + 12 = 15− 5 (x+ 6) .

SolutionApply standard algebraic properties.

4 (x− 3) + 12 = 15− 5 (x+ 6)

4x− 12 + 12 = 15− 5x− 30 Apply the distributive property.

4x = −15− 5x Combine like terms.

9x = −15 Place x− terms on one side and simplify.

x = − 159 Multiply both sides by 1

9 , the reciprocal of 9.

x = − 53

(2.6)

AnalysisThis problem requires the distributive property to be applied twice, and then the properties ofalgebra are used to reach the �nal line,x = − 5

3 .

Try It:

Exercise 2.3.2 (Solution on p. 245.)

Solve the equation in one variable: − 2 (3x− 1) + x = 14− x.

Available for free at Connexions <http://legacy.cnx.org/content/col11759/1.2>

133

2.3.2 Solving a Rational Equation

In this section, we look at rational equations that, after some manipulation, result in a linear equation. Ifan equation contains at least one rational expression, it is a considered a rational equation.

Recall that a rational number is the ratio of two numbers, such as 23 or

72 .A rational expression is

the ratio, or quotient, of two polynomials. Here are three examples.

x+ 1x2 − 4

,1

x− 3, or

4x2 + x− 2

(2.7)

Rational equations have a variable in the denominator in at least one of the terms. Our goal is to performalgebraic operations so that the variables appear in the numerator. In fact, we will eliminate all denominatorsby multiplying both sides of the equation by the least common denominator (LCD).

Finding the LCD is identifying an expression that contains the highest power of all of the factors in allof the denominators. We do this because when the equation is multiplied by the LCD, the common factorsin the LCD and in each denominator will equal one and will cancel out.

Example 2.3Solving a Rational EquationSolve the rational equation: 7

2x −5

3x = 223 .

SolutionWe have three denominators; 2x, 3x,and 3. The LCD must contain 2x, 3x,and 3. An LCDof 6x contains all three denominators. In other words, each denominator can be divided evenlyinto the LCD. Next, multiply both sides of the equation by the LCD 6x.

(6x)[

72x −

53x

]=[

223

](6x)

(6x)(

72x

)− (6x)

(5

3x

)=(

223

)(6x) Use the distributive property.(

)6x)(

7

)2x

)−(

)6x)(

5

)3x

)=(

22

)3

)()6x)

Cancel out the common factors.

3 (7)− 2 (5) = 22 (2x) Multiply remaining factors by each numerator.

21− 10 = 44x

11 = 44x1144 = x

14 = x

(2.8)

A common mistake made when solving rational equations involves �nding the LCD when one of the denom-inators is a binomial�two terms added or subtracted�such as (x+ 1) .Always consider a binomial as anindividual factor�the terms cannot be separated. For example, suppose a problem has three terms and thedenominators arex,x− 1,and 3x− 3.First, factor all denominators. We then havex,(x− 1) ,and 3 (x− 1) asthe denominators. (Note the parentheses placed around the second denominator.) Only the last twodenominators have a common factor of (x− 1) .Thex in the �rst denominator is separate from thex inthe (x− 1) denominators. An e�ective way to remember this is to write factored and binomial denomi-nators in parentheses, and consider each parentheses as a separate unit or a separate factor. The LCD inthis instance is found by multiplying together thex,one factor of (x− 1) ,and the 3. Thus, the LCD is thefollowing:

x (x− 1) 3 = 3x (x− 1) (2.9)

Available for free at Connexions <http://legacy.cnx.org/content/col11759/1.2>

134 CHAPTER 2. EQUATIONS AND INEQUALITIES

So, both sides of the equation would be multiplied by 3x (x− 1) .Leave the LCD in factored form, as thismakes it easier to see how each denominator in the problem cancels out.

Another example is a problem with two denominators, such asx andx2 + 2x.Once the second denomi-nator is factored asx2 + 2x = x (x+ 2) ,there is a common factor of x in both denominators and the LCDisx (x+ 2) .

Sometimes we have a rational equation in the form of a proportion; that is, when one fraction equalsanother fraction and there are no other terms in the equation.

a

b=c

d(2.10)

We can use another method of solving the equation without �nding the LCD: cross-multiplication. Wemultiply terms by crossing over the equal sign.

Multiply a (d) and b (c) ,which results in ad = bc.Any solution that makes a denominator in the original expression equal zero must be excluded from the

possibilities.

A General Note: A rational equation contains at least one rational expression where thevariable appears in at least one of the denominators.

How To: Given a rational equation, solve it.

1.Factor all denominators in the equation.2.Find and exclude values that set each denominator equal to zero.3.Find the LCD.4.Multiply the whole equation by the LCD. If the LCD is correct, there will be no denominatorsleft.

5.Solve the remaining equation.6.Make sure to check solutions back in the original equations to avoid a solution producing zeroin a denominator

Example 2.4Solving a Rational Equation without FactoringSolve the following rational equation:

2x− 3

2=

72x

(2.11)

Solution

We have three denominators:x,2,and 2x.No factoring is required. The product of the �rst twodenominators is equal to the third denominator, so, the LCD is 2x.Only one value is excluded from

Available for free at Connexions <http://legacy.cnx.org/content/col11759/1.2>

135

a solution set,x = 0.Next, multiply the whole equation (both sides of the equal sign) by 2x.

2x[

2x −

32

]=[

72x

]2x

2)x(

2

)x

)− )2x

(3

)2

)=(

7

)2x

))2x Distribute 2x

2 (2)− 3x = 7 Denominators cancel out.

4− 3x = 7

− 3x = 3

x = −1

or{−1}

(2.12)

The proposed solution isx = −1,which is not an excluded value, so the solution set contains onenumber,x = −1,or {−1}written in set notation.

Try It:

Exercise 2.3.3 (Solution on p. 245.)

Solve the rational equation: 23x = 1

4 −1

6x .

Example 2.5Solving a Rational Equation by Factoring the DenominatorSolve the following rational equation: 1

x = 110 −

34x .

SolutionFirst �nd the common denominator. The three denominators in factored form arex, 10 = 2 ·

5,and 4x = 2 · 2 · x.The smallest expression that is divisible by each one of the denominatorsis 20x.Onlyx = 0 is an excluded value. Multiply the whole equation by 20x.

20x(

1x

)=(

110 −

34x

)20x

20 = 2x− 15

35 = 2x352 = x

(2.13)

The solution isx = 352 .

Try It:

Exercise 2.3.4 (Solution on p. 245.)

Solve the rational equation: − 52x + 3

4x = − 74 .

Example 2.6Solving Rational Equations with a Binomial in the DenominatorSolve the following rational equations and state the excluded values:

Available for free at Connexions <http://legacy.cnx.org/content/col11759/1.2>

136 CHAPTER 2. EQUATIONS AND INEQUALITIES

a. 3x−6 = 5

x

b. xx−3 = 5

x−3 −12

c. xx−2 = 5

x−2 −12

Solution

a. The denominatorsx andx − 6 have nothing in common. Therefore, the LCD is theproductx (x− 6) .However, for this problem, we can cross-multiply.

3x−6 = 5

x

3x = 5 (x− 6) Distribute.

3x = 5x− 30

− 2x = −30

x = 15

(2.14)

The solution isx = 15.The excluded values arex = 6 andx = 0.b. The LCD is 2 (x− 3) .Multiply both sides of the equation by 2 (x− 3) .

2 (x− 3)[

xx−3

]=[

5x−3 −

12

]2 (x− 3)

2)(x−3)x

)x−3= 2)(x−3)5

)x−3− )2(x−3)

)2

2x = 10− (x− 3)

2x = 10− x+ 3

2x = 13− x3x = 13

x = 133

(2.15)

The solution isx = 133 .The excluded value isx = 3.

c. The least common denominator is 2 (x− 2) .Multiply both sides of the equation byx (x− 2) .

2 (x− 2)[

xx−2

]=[

5x−2 −

12

]2 (x− 2)

2x = 10− (x− 2)

2x = 12− x3x = 12

x = 4

(2.16)

The solution isx = 4.The excluded value isx = 2.

Try It:

Exercise 2.3.5 (Solution on p. 245.)

Solve −32x+1 = 4

3x+1 . State the excluded values.

Available for free at Connexions <http://legacy.cnx.org/content/col11759/1.2>

137

Example 2.7Solving a Rational Equation with Factored Denominators and Stating Excluded Val-uesSolve the rational equation after factoring the denominators: 2

x+1−1

x−1 = 2xx2−1 . State the excluded

values.

SolutionWe must factor the denominatorx2−1.We recognize this as the di�erence of squares, and factor itas (x− 1) (x+ 1) .Thus, the LCD that contains each denominator is (x− 1) (x+ 1) .Multiply thewhole equation by the LCD, cancel out the denominators, and solve the remaining equation.

(x− 1) (x+ 1)[

2x+1 −

1x−1

]=[

2x(x−1)(x+1)

](x− 1) (x+ 1)

2 (x− 1)− 1 (x+ 1) = 2x

2x− 2− x− 1 = 2x Distribute the negative sign.

− 3− x = 0

− 3 = x

(2.17)

The solution isx = −3.The excluded values arex = 1 andx = −1.

Try It:

Exercise 2.3.6 (Solution on p. 245.)

Solve the rational equation: 2x−2 + 1

x+1 = 1x2−x−2 .

2.3.3 Finding a Linear Equation

Perhaps the most familiar form of a linear equation is the slope-intercept form, written as y = mx +b,wherem = slope and b = y−intercept. Let us begin with the slope.

2.3.3.1 The Slope of a Line

The slope of a line refers to the ratio of the vertical change in y over the horizontal change in x betweenany two points on a line. It indicates the direction in which a line slants as well as its steepness. Slope issometimes described as rise over run.

m =y2 − y1

x2 − x1(2.18)

If the slope is positive, the line slants to the right. If the slope is negative, the line slants to the left. Asthe slope increases, the line becomes steeper. Some examples are shown in Figure 2.3. The lines indicatethe following slopes:m = −3,m = 2,andm = 1

3 .

Available for free at Connexions <http://legacy.cnx.org/content/col11759/1.2>

138 CHAPTER 2. EQUATIONS AND INEQUALITIES

Figure 2.3

A General Note: The slope of a line, m, represents the change in y over the change in x. Giventwo points, (x1, y1) and (x2, y2) ,the following formula determines the slope of a line containingthese points:

m =y2 − y1

x2 − x1(2.19)

Example 2.8Finding the Slope of a Line Given Two PointsFind the slope of a line that passes through the points (2,−1) and (−5, 3) .

Available for free at Connexions <http://legacy.cnx.org/content/col11759/1.2>

139

SolutionWe substitute the y-values and the x-values into the formula.

m = 3−(−1)−5−2

= 4−7

= − 47

(2.20)

The slope is − 47 .

Analysis

It does not matter which point is called (x1, y1) or (x2, y2) .As long as we are consistent withthe order of the y terms and the order of the x terms in the numerator and denominator, thecalculation will yield the same result.

Try It:

Exercise 2.3.7 (Solution on p. 245.)

Find the slope of the line that passes through the points (−2, 6) and (1, 4) .

Example 2.9Identifying the Slope and y-intercept of a Line Given an EquationIdentify the slope and y-intercept, given the equation y = − 3

4x− 4.

SolutionAs the line is in y = mx+ b form, the given line has a slope ofm = − 3

4 .The y-intercept is b = −4.

AnalysisThe y-intercept is the point at which the line crosses the y-axis. On the y-axis,x = 0.We canalways identify the y-intercept when the line is in slope-intercept form, as it will always equal b.Or, just substitutex = 0 and solve for y.

2.3.3.2 The Point-Slope Formula

Given the slope and one point on a line, we can �nd the equation of the line using the point-slope formula.

y − y1 = m (x− x1) (2.21)

This is an important formula, as it will be used in other areas of college algebra and often in calculus to�nd the equation of a tangent line. We need only one point and the slope of the line to use the formula.After substituting the slope and the coordinates of one point into the formula, we simplify it and write it inslope-intercept form.

A General Note: Given one point and the slope, the point-slope formula will lead to theequation of a line:

y − y1 = m (x− x1) (2.22)

Available for free at Connexions <http://legacy.cnx.org/content/col11759/1.2>

140 CHAPTER 2. EQUATIONS AND INEQUALITIES

Example 2.10Finding the Equation of a Line Given the Slope and One PointWrite the equation of the line with slopem = −3 and passing through the point (4, 8) .Write the�nal equation in slope-intercept form.

SolutionUsing the point-slope formula, substitute − 3 for m and the point (4, 8) for (x1, y1) .

y − y1 = m (x− x1)

y − 8 = −3 (x− 4)

y − 8 = −3x+ 12

y = −3x+ 20

(2.23)

AnalysisNote that any point on the line can be used to �nd the equation. If done correctly, the same �nalequation will be obtained.

Try It:

Exercise 2.3.8 (Solution on p. 245.)

Givenm = 4,�nd the equation of the line in slope-intercept form passing through thepoint (2, 5) .

Example 2.11Finding the Equation of a Line Passing Through Two Given PointsFind the equation of the line passing through the points (3, 4) and (0,−3) .Write the �nal equationin slope-intercept form.

SolutionFirst, we calculate the slope using the slope formula and two points.

m = −3−40−3

= −7−3

= 73

(2.24)

Next, we use the point-slope formula with the slope of 73 ,and either point. Let's pick the

point (3, 4) for (x1, y1) .

y − 4 = 73 (x− 3)

y − 4 = 73x− 7 Distribute the 7

3 .

y = 73x− 3

(2.25)

In slope-intercept form, the equation is written as y = 73x− 3.

Available for free at Connexions <http://legacy.cnx.org/content/col11759/1.2>

141

AnalysisTo prove that either point can be used, let us use the second point (0,−3) and see if we get thesame equation.

y − (−3) = 73 (x− 0)

y + 3 = 73x

y = 73x− 3

(2.26)

We see that the same line will be obtained using either point. This makes sense because we usedboth points to calculate the slope.

2.3.3.3 Standard Form of a Line

Another way that we can represent the equation of a line is in standard form. Standard form is given as

Ax+By = C (2.27)

whereA,B,andCare integers. The x- and y-terms are on one side of the equal sign and the constant termis on the other side.

Example 2.12Finding the Equation of a Line and Writing It in Standard FormFind the equation of the line withm = −6 and passing through the point

(14 ,−2

).Write the

equation in standard form.

SolutionWe begin using the point-slope formula.

y − (−2) = −6(x− 1

4

)y + 2 = −6x+ 3

2

(2.28)

From here, we multiply through by 2, as no fractions are permitted in standard form, and thenmove both variables to the left aside of the equal sign and move the constants to the right.

2 (y + 2) =(−6x+ 3

2

)2

2y + 4 = −12x+ 3

12x+ 2y = −1

(2.29)

This equation is now written in standard form.

Try It:

Exercise 2.3.9 (Solution on p. 245.)

Find the equation of the line in standard form with slopem = − 13 and passing through

the point(1, 1

3

).

Available for free at Connexions <http://legacy.cnx.org/content/col11759/1.2>

142 CHAPTER 2. EQUATIONS AND INEQUALITIES

2.3.3.4 Vertical and Horizontal Lines

The equations of vertical and horizontal lines do not require any of the preceding formulas, although we canuse the formulas to prove that the equations are correct. The equation of a vertical line is given as

x = c (2.30)

where c is a constant. The slope of a vertical line is unde�ned, and regardless of the y-value of any pointon the line, the x-coordinate of the point will be c.

Suppose that we want to �nd the equation of a line containing the followingpoints: (−3,−5) , (−3, 1) , (−3, 3) ,and (−3, 5) .First, we will �nd the slope.

m =5− 3

−3− (−3)=

20

(2.31)

Zero in the denominator means that the slope is unde�ned and, therefore, we cannot use the point-slopeformula. However, we can plot the points. Notice that all of the x-coordinates are the same and we �nd avertical line throughx = −3. See Figure 2.4.

The equation of a horizontal line is given as

y = c (2.32)

where c is a constant. The slope of a horizontal line is zero, and for any x-value of a point on the line, they-coordinate will be c.

Suppose we want to �nd the equation of a line that contains the following set ofpoints: (−2,−2) , (0,−2) , (3,−2) ,and (5,−2) .We can use the point-slope formula. First, we �nd the slopeusing any two points on the line.

m = −2−(−2)0−(−2)

= 02

= 0

(2.33)

Use any point for (x1, y1) in the formula, or use the y-intercept.

y − (−2) = 0 (x− 3)

y + 2 = 0

y = −2

(2.34)

The graph is a horizontal line through y = −2.Notice that all of the y-coordinates are the same. SeeFigure 2.4.

Available for free at Connexions <http://legacy.cnx.org/content/col11759/1.2>

143

Figure 2.4: The line x = −3 is a vertical line. The line y = −2 is a horizontal line.

Example 2.13Finding the Equation of a Line Passing Through the Given PointsFind the equation of the line passing through the given points: (1,−3) and (1, 4) .

SolutionThe x-coordinate of both points is 1. Therefore, we have a vertical line,x = 1.

Try It:

Exercise 2.3.10 (Solution on p. 245.)

Find the equation of the line passing through (−5, 2) and (2, 2) .

Available for free at Connexions <http://legacy.cnx.org/content/col11759/1.2>

144 CHAPTER 2. EQUATIONS AND INEQUALITIES

2.3.4 Determining Whether Graphs of Lines are Parallel or Perpendicular

Parallel lines have the same slope and di�erent y-intercepts. Lines that are parallel to each other will neverintersect. For example, Figure 2.5 shows the graphs of various lines with the same slope,m = 2.

Available for free at Connexions <http://legacy.cnx.org/content/col11759/1.2>

145

Figure 2.5: Parallel lines

Available for free at Connexions <http://legacy.cnx.org/content/col11759/1.2>

146 CHAPTER 2. EQUATIONS AND INEQUALITIES

All of the lines shown in the graph are parallel because they have the same slope and di�erent y-intercepts.Lines that are perpendicular intersect to form a 90◦-angle. The slope of one line is the negative re-

ciprocal of the other. We can show that two lines are perpendicular if the product of the two slopesis − 1 : m1 ·m2 = −1.For example, Figure 2.6 shows the graph of two perpendicular lines. One line has aslope of 3; the other line has a slope of − 1

3 .

m1 ·m2 = −1

3 ·(− 1

3

)= −1

(2.35)

Figure 2.6: Perpendicular lines

Example 2.14Graphing Two Equations, and Determining Whether the Lines are Parallel, Perpen-

dicular, or NeitherGraph the equations of the given lines, and state whether they are parallel, perpendicular, orneither: 3y = −4x+ 3 and 3x− 4y = 8.

SolutionThe �rst thing we want to do is rewrite the equations so that both equations are in slope-interceptform.

Available for free at Connexions <http://legacy.cnx.org/content/col11759/1.2>

147

First equation:

3y = −4x+ 3

y = − 43x+ 1

(2.36)

Second equation:

3x− 4y = 8

− 4y = −3x+ 8

y = 34x− 2

(2.37)

See the graph of both lines in Figure 2.7

Figure 2.7

From the graph, we can see that the lines appear perpendicular, but we must compare the

Available for free at Connexions <http://legacy.cnx.org/content/col11759/1.2>

148 CHAPTER 2. EQUATIONS AND INEQUALITIES

slopes.

m1 = − 43

m2 = 34

m1 ·m2 =(− 4

3

) (34

)= −1

(2.38)

The slopes are negative reciprocals of each other, con�rming that the lines are perpendicular.

Try It:

Exercise 2.3.11 (Solution on p. 245.)

Graph the two lines and determine whether they are parallel, perpendicular, orneither: 2y − x = 10 and 2y = x+ 4.

2.3.5 Writing the Equations of Lines Parallel or Perpendicular to a Given Line

As we have learned, determining whether two lines are parallel or perpendicular is a matter of �nding theslopes. To write the equation of a line parallel or perpendicular to another line, we follow the same principlesas we do for �nding the equation of any line. After �nding the slope, use the point-slope formula to writethe equation of the new line.

How To: Given an equation for a line, write the equation of a line parallel or per-pendicular to it.

1.Find the slope of the given line. The easiest way to do this is to write the equation in slope-intercept form.

2.Use the slope and the given point with the point-slope formula.3.Simplify the line to slope-intercept form and compare the equation to the given line.

Example 2.15Writing the Equation of a Line Parallel to a Given Line Passing Through a Given

PointWrite the equation of line parallel to a 5x+ 3y = 1 and passing through the point (3, 5) .

SolutionFirst, we will write the equation in slope-intercept form to �nd the slope.

5x+ 3y = 1

3y = −5x+ 1

y = − 53x+ 1

3

(2.39)

The slope ism = − 53 .The y-intercept is

13 ,but that really does not enter into our problem, as the

only thing we need for two lines to be parallel is the same slope. The one exception is that if they-intercepts are the same, then the two lines are the same line. The next step is to use this slopeand the given point with the point-slope formula.

y − 5 = − 53 (x− 3)

y − 5 = − 53x+ 5

y = − 53x+ 10

(2.40)

Available for free at Connexions <http://legacy.cnx.org/content/col11759/1.2>

149

The equation of the line is y = − 53x+ 10. See Figure 2.8.

Figure 2.8

Try It:

Exercise 2.3.12 (Solution on p. 246.)

Find the equation of the line parallel to 5x = 7+y and passing through the point (−1,−2) .

Example 2.16Finding the Equation of a Line Perpendicular to a Given Line Passing Through a

Given PointFind the equation of the line perpendicular to 5x− 3y + 4 = 0 (−4, 1) .

SolutionThe �rst step is to write the equation in slope-intercept form.

5x− 3y + 4 = 0

− 3y = −5x− 4

y = 53x+ 4

3

(2.41)

Available for free at Connexions <http://legacy.cnx.org/content/col11759/1.2>

150 CHAPTER 2. EQUATIONS AND INEQUALITIES

We see that the slope ism = 53 .This means that the slope of the line perpendicular to the given

line is the negative reciprocal, or − 35 .Next, we use the point-slope formula with this new slope and

the given point.

y − 1 = − 35 (x− (−4))

y − 1 = − 35x−

125

y = − 35x−

125 + 5

5

y = − 35x−

75

(2.42)

Media: Access these online resources for additional instruction and practice with linear equations.

• Solving rational equations4• Equation of a line given two points5

• Finding the Equation of a line perpendicular to another line through a given point6

• Finding the equation of a line parallel to another line through a given point7

2.3.6 Key Concepts

• We can solve linear equations in one variable in the form ax+b = 0 using standard algebraic properties.See Example 2.1 and Example 2.2.

• A rational expression is a quotient of two polynomials. We use the LCD to clear the fractions from anequation. See Example 2.3 and Example 2.4.

• All solutions to a rational equation should be veri�ed within the original equation to avoid an unde�nedterm, or zero in the denominator. See Example 2.5 and Example 2.6.

• Given two points, we can �nd the slope of a line using the slope formula. See Example 2.7.• We can identify the slope and y-intercept of an equation in slope-intercept form. See Example 2.8.• We can �nd the equation of a line given the slope and a point. See Example 2.9.• We can also �nd the equation of a line given two points. Find the slope and use the point-slope formula.

See Example 2.11.• The standard form of a line has no fractions. See Example 2.12.• Horizontal lines have a slope of zero and are de�ned as y = c,where c is a constant.• Vertical lines have an unde�ned slope (zero in the denominator), and are de�ned asx = c,where c is a

constant. See Example 2.13.• Parallel lines have the same slope and di�erent y-intercepts. See Example 2.15.• Perpendicular lines have slopes that are negative reciprocals of each other unless one is horizontal and

the other is vertical. See Example 2.16.

4http://openstaxcollege.org/l/rationaleqs5http://openstaxcollege.org/l/twopointsline6http://openstaxcollege.org/l/�ndperpline7http://openstaxcollege.org/l/�ndparaline

Available for free at Connexions <http://legacy.cnx.org/content/col11759/1.2>

151

2.3.7 Section Exercises

2.3.7.1 Verbal

Exercise 2.3.13 (Solution on p. 246.)

What does it mean when we say that two lines are parallel?

Exercise 2.3.14What is the relationship between the slopes of perpendicular lines (assuming neither is horizontalnor vertical)?

Exercise 2.3.15 (Solution on p. 246.)

How do we recognize when an equation, for example y = 4x+ 3,will be a straight line (linear) whengraphed?

Exercise 2.3.16What does it mean when we say that a linear equation is inconsistent?

Exercise 2.3.17 (Solution on p. 246.)

When solving the following equation:2

x−5 = 4x+1

explain why we must excludex = 5 andx = −1 as possible solutions from the solution set.

2.3.7.2 Algebraic

For the following exercises, solve the equation forx.

Exercise 2.3.187x+ 2 = 3x− 9Exercise 2.3.19 (Solution on p. 246.)

4x− 3 = 5Exercise 2.3.203 (x+ 2)− 12 = 5 (x+ 1)Exercise 2.3.21 (Solution on p. 246.)

12− 5 (x+ 3) = 2x− 5Exercise 2.3.2212 −

13x = 4

3

Exercise 2.3.23 (Solution on p. 246.)x3 −

34 = 2x+3

12

Exercise 2.3.2423x+ 1

2 = 316

Exercise 2.3.25 (Solution on p. 246.)

3 (2x− 1) + x = 5x+ 3Exercise 2.3.262x3 −

34 = x

6 + 214

Exercise 2.3.27 (Solution on p. 246.)x+2

4 −x−1

3 = 2For the following exercises, solve each rational equation forx. State all x-values that are excluded from thesolution set.

Exercise 2.3.283x −

13 = 1

6

Available for free at Connexions <http://legacy.cnx.org/content/col11759/1.2>

152 CHAPTER 2. EQUATIONS AND INEQUALITIES

Exercise 2.3.29 (Solution on p. 246.)

2− 3x+4 = x+2

x+4

Exercise 2.3.303

x−2 = 1x−1 + 7

(x−1)(x−2)

Exercise 2.3.31 (Solution on p. 246.)3xx−1 + 2 = 3

x−1

Exercise 2.3.325

x+1 + 1x−3 = −6

x2−2x−3

Exercise 2.3.33 (Solution on p. 247.)1x = 1

5 + 32x

For the following exercises, �nd the equation of the line using the point-slope formula.Write all the �nal equations using the slope-intercept form.

Exercise 2.3.34(0, 3) with a slope of 2

3

Exercise 2.3.35 (Solution on p. 247.)

(1, 2) with a slope of −45

Exercise 2.3.36x-intercept is 1, and (−2, 6)Exercise 2.3.37 (Solution on p. 247.)

y-intercept is 2, and (4,−1)Exercise 2.3.38(−3, 10) and (5,−6)Exercise 2.3.39 (Solution on p. 247.)

(1, 3) and (5, 5)Exercise 2.3.40parallel to y = 2x+ 5 and passes through the point (4, 3)Exercise 2.3.41 (Solution on p. 247.)

perpendicular to 3y = x− 4 and passes through the point (−2, 1).For the following exercises, �nd the equation of the line using the given information.

Exercise 2.3.42(−2, 0) and (−2, 5)Exercise 2.3.43 (Solution on p. 247.)

(1, 7) and (3, 7)Exercise 2.3.44The slope is unde�ned and it passes through the point (2, 3) .Exercise 2.3.45 (Solution on p. 247.)

The slope equals zero and it passes through the point (1,−4) .Exercise 2.3.46The slope is 3

4 and it passes through the point (1,4).

Exercise 2.3.47 (Solution on p. 247.)

(−1, 3) and (4,−5)

Available for free at Connexions <http://legacy.cnx.org/content/col11759/1.2>

153

2.3.7.3 Graphical

For the following exercises, graph the pair of equations on the same axes, and state whether they are parallel,perpendicular, or neither.

Exercise 2.3.48

y = 2x+ 7

y = −12 x− 4

Exercise 2.3.49 (Solution on p. 247.)

3x− 2y = 5

6y − 9x = 6

Exercise 2.3.50

y = 3x+14

y = 3x+ 2

Exercise 2.3.51 (Solution on p. 247.)

x = 4

y = −3

2.3.7.4 Numeric

For the following exercises, �nd the slope of the line that passes through the given points.

Exercise 2.3.52(5, 4) and (7, 9)Exercise 2.3.53 (Solution on p. 248.)

(−3, 2) and (4,−7)Exercise 2.3.54(−5, 4) and (2, 4)Exercise 2.3.55 (Solution on p. 248.)

(−1,−2) and (3, 4)Exercise 2.3.56(3,−2)and (3,−2)

For the following exercises, �nd the slope of the lines that pass through each pair of points and determinewhether the lines are parallel or perpendicular.

Exercise 2.3.57 (Solution on p. 248.)

(−1, 3) and (5, 1)

(−2, 3) and (0, 9)

Exercise 2.3.58

(2, 5) and (5, 9)

(−1,−1) and (2, 3)

Available for free at Connexions <http://legacy.cnx.org/content/col11759/1.2>

154 CHAPTER 2. EQUATIONS AND INEQUALITIES

2.3.7.5 Technology

For the following exercises, express the equations in slope intercept form (rounding each number to thethousandths place). Enter this into a graphing calculator as Y1, then adjust the ymin and ymax values foryour window to include where the y-intercept occurs. State your ymin and ymax values.

Exercise 2.3.59 (Solution on p. 248.)

0.537x− 2.19y = 100Exercise 2.3.604, 500x− 200y = 9, 528Exercise 2.3.61 (Solution on p. 248.)200−30y

x = 70

2.3.7.6 Extensions

Exercise 2.3.62Starting with the point-slope formula y − y1 = m (x− x1) ,solve this expression forx in termsofx1, y, y1,andm.

Exercise 2.3.63 (Solution on p. 248.)

Starting with the standard form of an equationAx + By = C,solve this expression for y in termsofA,B,C, andx.Then put the expression in slope-intercept form.

Exercise 2.3.64Use the above derived formula to put the following standard equation in slope intercept form: 7x−5y = 25.Exercise 2.3.65 (Solution on p. 248.)

Given that the following coordinates are the vertices of a rectangle, prove that this truly is arectangle by showing the slopes of the sides that meet are perpendicular.

(−1, 1) , (2, 0) , (3, 3) ,and (0, 4)Exercise 2.3.66Find the slopes of the diagonals in the previous exercise. Are they perpendicular?

2.3.7.7 Real-World Applications

Exercise 2.3.67 (Solution on p. 249.)

The slope for a wheelchair ramp for a home has to be 112 . If the vertical distance from the ground

to the door bottom is 2.5 ft, �nd the distance the ramp has to extend from the home in order tocomply with the needed slope.

Exercise 2.3.68If the pro�t equation for a small business sellingx number of item one and y number of item twois p = 3x+ 4y,�nd the y value when p = $453 and x = 75.

Available for free at Connexions <http://legacy.cnx.org/content/col11759/1.2>

155

For the following exercises, use this scenario: The cost of renting a car is $45/wk plus $0.25/mi traveledduring that week. An equation to represent the cost would be y = 45 + .25x,wherex is the number of milestraveled.

Exercise 2.3.69 (Solution on p. 249.)

What is your cost if you travel 50 mi?

Exercise 2.3.70If your cost were $63.75,how many miles were you charged for traveling?

Exercise 2.3.71 (Solution on p. 249.)

Suppose you have a maximum of $100 to spend for the car rental. What would be the maximumnumber of miles you could travel?

2.4 Models and Applications8

Figure 2.9: Credit: Kevin Dooley

8This content is available online at <http://legacy.cnx.org/content/m51254/1.2/>.

Available for free at Connexions <http://legacy.cnx.org/content/col11759/1.2>

156 CHAPTER 2. EQUATIONS AND INEQUALITIES

Josh is hoping to get an A in his college algebra class. He has scores of 75, 82, 95, 91, and 94 on his �rst�ve tests. Only the �nal exam remains, and the maximum of points that can be earned is 100. Is it possiblefor Josh to end the course with an A? A simple linear equation will give Josh his answer.

Many real-world applications can be modeled by linear equations. For example, a cell phone packagemay include a monthly service fee plus a charge per minute of talk-time; it costs a widget manufacturera certain amount to produce x widgets per month plus monthly operating charges; a car rental companycharges a daily fee plus an amount per mile driven. These are examples of applications we come across everyday that are modeled by linear equations. In this section, we will set up and use linear equations to solvesuch problems.

2.4.1 Setting up a Linear Equation to Solve a Real-World Application

To set up or model a linear equation to �t a real-world application, we must �rst determine the known quan-tities and de�ne the unknown quantity as a variable. Then, we begin to interpret the words as mathematicalexpressions using mathematical symbols. Let us use the car rental example above. In this case, a knowncost, such as $0.10/mi, is multiplied by an unknown quantity, the number of miles driven. Therefore, we canwrite 0.10x.This expression represents a variable cost because it changes according to the number of milesdriven.

If a quantity is independent of a variable, we usually just add or subtract it, according to the problem. Asthese amounts do not change, we call them �xed costs. Consider a car rental agency that charges $0.10/miplus a daily fee of $50. We can use these quantities to model an equation that can be used to �nd the dailycar rental costC.

C = 0.10x+ 50 (2.43)

When dealing with real-world applications, there are certain expressions that we can translate directly intomath. Table 2.1 lists some common verbal expressions and their equivalent mathematical expressions.

Verbal Translation to Math Operations

One number exceeds another by a x, x+ a

Twice a number 2x

One number is a more than another number x, x+ a

One number is a less than twice another number x, 2x− aThe product of a number and a, decreased by b ax− bThe quotient of a number and the number plus a isthree times the number

xx+a = 3x

The product of three times a number and the num-ber decreased by b is c

3x (x− b) = c

Table 2.1

How To: Given a real-world problem, model a linear equation to �t it.

1.Identify known quantities.2.Assign a variable to represent the unknown quantity.3.If there is more than one unknown quantity, �nd a way to write the second unknown in termsof the �rst.

4.Write an equation interpreting the words as mathematical operations.

Available for free at Connexions <http://legacy.cnx.org/content/col11759/1.2>

157

5.Solve the equation. Be sure the solution can be explained in words, including the units ofmeasure.

Example 2.17Modeling a Linear Equation to Solve an Unknown Number ProblemFind a linear equation to solve for the following unknown quantities: One number exceeds anothernumber by 17 and their sum is 31.Find the two numbers.

SolutionLetx equal the �rst number. Then, as the second number exceeds the �rst by 17, we can write thesecond number asx + 17.The sum of the two numbers is 31. We usually interpret the word is asan equal sign.

x+ (x+ 17) = 31

2x+ 17 = 31 Simplify and solve.

2x = 14

x = 7

x+ 17 = 7 + 17

= 24

(2.44)

The two numbers are 7 and 24.

Try It:

Exercise 2.4.1 (Solution on p. 249.)

Find a linear equation to solve for the following unknown quantities: One number is threemore than twice another number. If the sum of the two numbers is 36,�nd the numbers.

Example 2.18Setting Up a Linear Equation to Solve a Real-World ApplicationThere are two cell phone companies that o�er di�erent packages. Company A charges a monthlyservice fee of $34 plus $.05/min talk-time. Company B charges a monthly service fee of $40 plus$.04/min talk-time.

a. Write a linear equation that models the packages o�ered by both companies.b. If the average number of minutes used each month is 1,160, which company o�ers the better

plan?c. If the average number of minutes used each month is 420, which company o�ers the better

plan?d. How many minutes of talk-time would yield equal monthly statements from both companies?

Solution

a. The model for Company A can be written asA = 0.05x + 34.This includes the variablecost of 0.05x plus the monthly service charge of $34. Company B's package charges a highermonthly fee of $40, but a lower variable cost of 0.04x.Company B's model can be writtenasB = 0.04x+ $40.

Available for free at Connexions <http://legacy.cnx.org/content/col11759/1.2>

158 CHAPTER 2. EQUATIONS AND INEQUALITIES

b. If the average number of minutes used each month is 1,160, we have the following:

Company A = 0.05 (1, 160) + 34

= 58 + 34

= 92

Company B = 0.04 (1, 160) + 40

= 46.4 + 40

= 86.4

(2.45)

So, Company B o�ers the lower monthly cost of $86.40 as compared with the $92 monthlycost o�ered by Company A when the average number of minutes used each month is 1,160.

c. If the average number of minutes used each month is 420, we have the following:

Company A = 0.05 (420) + 34

= 21 + 34

= 55

Company B = 0.04 (420) + 40

= 16.8 + 40

= 56.8

(2.46)

If the average number of minutes used each month is 420, then Company A o�ers a lowermonthly cost of $55 compared to Company B's monthly cost of $56.80.

d. To answer the question of how many talk-time minutes would yield the same bill from bothcompanies, we should think about the problem in terms of (x, y) coordinates: At what pointare both the x-value and the y-value equal? We can �nd this point by setting the equationsequal to each other and solving for x.

0.05x+ 34 = 0.04x+ 40

0.01x = 6

x = 600

(2.47)

Check the x-value in each equation.

0.05 (600) + 34 = 64

0.04 (600) + 40 = 64(2.48)

Therefore, a monthly average of 600 talk-time minutes renders the plans equal. See Fig-ure 2.10

Available for free at Connexions <http://legacy.cnx.org/content/col11759/1.2>

159

Figure 2.10

Try It:

Exercise 2.4.2 (Solution on p. 249.)

Find a linear equation to model this real-world application: It costs ABC electronicscompany $2.50 per unit to produce a part used in a popular brand of desktop computers.The company has monthly operating expenses of $350 for utilities and $3,300 for salaries.What are the company's monthly expenses?

2.4.2 Using a Formula to Solve a Real-World Application

Many applications are solved using known formulas. The problem is stated, a formula is identi�ed, the knownquantities are substituted into the formula, the equation is solved for the unknown, and the problem's questionis answered. Typically, these problems involve two equations representing two trips, two investments, twoareas, and so on. Examples of formulas include the area of a rectangular region,A = LW ;the perimeter ofa rectangle,P = 2L+2W ;and the volume of a rectangular solid,V = LWH.When there are two unknowns,we �nd a way to write one in terms of the other because we can solve for only one variable at a time.

Example 2.19Solving an Application Using a FormulaIt takes Andrew 30 min to drive to work in the morning. He drives home using the same route, butit takes 10 min longer, and he averages 10 mi/h less than in the morning. How far does Andrewdrive to work?

Available for free at Connexions <http://legacy.cnx.org/content/col11759/1.2>

160 CHAPTER 2. EQUATIONS AND INEQUALITIES

SolutionThis is a distance problem, so we can use the formula d = rt,where distance equals rate multipliedby time. Note that when rate is given in mi/h, time must be expressed in hours. Consistent unitsof measurement are key to obtaining a correct solution.

First, we identify the known and unknown quantities. Andrew's morning drive to work takes30 min, or 1

2 h at rate r.His drive home takes 40 min, or 23 h, and his speed averages 10 mi/h less

than the morning drive. Both trips cover distance d.A table, such as Table 2.2, is often helpful forkeeping track of information in these types of problems.

d r t

To Work d r 12

To Home d r − 10 23

Table 2.2

Write two equations, one for each trip.

d = r(

12

)To work

d = (r − 10)(

23

)To home

(2.49)

As both equations equal the same distance, we set them equal to each other and solve for r.

r(

12

)= (r − 10)

(23

)12r = 2

3r −203

12r −

23r = − 20

3

− 16r = − 20

3

r = − 203 (−6)

r = 40

(2.50)

We have solved for the rate of speed to work, 40 mph. Substituting 40 into the rate on the returntrip yields 30 mi/h. Now we can answer the question. Substitute the rate back into either equationand solve for d.

d = 40(

12

)= 20

(2.51)

The distance between home and work is 20 mi.

AnalysisNote that we could have cleared the fractions in the equation by multiplying both sides of theequation by the LCD to solve for r.

r(

12

)= (r − 10)

(23

)6 × r

(12

)= 6 × (r − 10)

(23

)3r = 4 (r − 10)

3r = 4r − 40

− r = −40

r = 40

(2.52)

Available for free at Connexions <http://legacy.cnx.org/content/col11759/1.2>

161

Try It:

Exercise 2.4.3 (Solution on p. 249.)

On Saturday morning, it took Jennifer 3.6 h to drive to her mother's house for the weekend.On Sunday evening, due to heavy tra�c, it took Jennifer 4 h to return home. Her speedwas 5 mi/h slower on Sunday than on Saturday. What was her speed on Sunday?

Example 2.20Solving a Perimeter ProblemThe perimeter of a rectangular outdoor patio is 54 ft. The length is 3 ft greater than the width.What are the dimensions of the patio?

SolutionThe perimeter formula is standard:P = 2L + 2W.We have two unknown quantities, length andwidth. However, we can write the length in terms of the width asL = W + 3. Substitute theperimeter value and the expression for length into the formula. It is often helpful to make a sketchand label the sides as in Figure 2.11.

Figure 2.11

Available for free at Connexions <http://legacy.cnx.org/content/col11759/1.2>

162 CHAPTER 2. EQUATIONS AND INEQUALITIES

Now we can solve for the width and then calculate the length.

P = 2L+ 2W

54 = 2 (W + 3) + 2W

54 = 2W + 6 + 2W

54 = 4W + 6

48 = 4W

12 = W

(12 + 3) = L

15 = L

(2.53)

The dimensions areL = 15 ft andW = 12 ft.

Try It:

Exercise 2.4.4 (Solution on p. 249.)

Find the dimensions of a rectangle given that the perimeter is 110 cm and the length is 1cm more than twice the width.

Example 2.21Solving an Area ProblemThe perimeter of a tablet of graph paper is 48 in.2. The length is 6 in. more than the width. Findthe area of the graph paper.

SolutionThe standard formula for area isA = LW ;however, we will solve the problem using the perimeterformula. The reason we use the perimeter formula is because we know enough information aboutthe perimeter that the formula will allow us to solve for one of the unknowns. As both perimeterand area use length and width as dimensions, they are often used together to solve a problem suchas this one.

We know that the length is 6 in. more than the width, so we can write length asL = W +6. Substitute the value of the perimeter and the expression for length into the perimeter formulaand �nd the length.

P = 2L+ 2W

48 = 2 (W + 6) + 2W

48 = 2W + 12 + 2W

48 = 4W + 12

36 = 4W

9 = W

(9 + 6) = L

15 = L

(2.54)

Available for free at Connexions <http://legacy.cnx.org/content/col11759/1.2>

163

Now, we �nd the area given the dimensions ofL = 15 in. andW = 9 in.

A = LW

A = 15 (9)

= 135 in.2

(2.55)

The area is 135 in.2.

Try It:

Exercise 2.4.5 (Solution on p. 249.)

A game room has a perimeter of 70 ft. The length is �ve more than twice the width. Howmany ft2 of new carpeting should be ordered?

Example 2.22Solving a Volume ProblemFind the dimensions of a shipping box given that the length is twice the width, the height is 8 inches,and the volume is 1,600 in.3.

SolutionThe formula for the volume of a box is given asV = LWH,the product of length, width, andheight. We are given thatL = 2W,andH = 8.The volume is 1, 600 cubic inches.

V = LWH

1, 600 = (2W )W (8)

1, 600 = 16W 2

100 = W 2

10 = W

(2.56)

The dimensions areL = 20 in.,W = 10 in., andH = 8 in.

AnalysisNote that the square root ofW 2 would result in a positive and a negative value. However, becausewe are describing width, we can use only the positive result.

Media: Access these online resources for additional instruction and practice with models andapplications of linear equations.

• Problem solving using linear equations9

• Problem solving using equations10

• Finding the dimensions of area given the perimeter11

9http://openstaxcollege.org/l/lineqprobsolve10http://openstaxcollege.org/l/equationprsolve11http://openstaxcollege.org/l/permareasolve

Available for free at Connexions <http://legacy.cnx.org/content/col11759/1.2>

164 CHAPTER 2. EQUATIONS AND INEQUALITIES

• Find the distance between the cities using the distance = rate * time formula12

• Linear equation application (Write a cost equation)13

2.4.3 Key Concepts

• A linear equation can be used to solve for an unknown in a number problem. See Example 2.17.• Applications can be written as mathematical problems by identifying known quantities and assigning

a variable to unknown quantities. See Example 2.18.• There are many known formulas that can be used to solve applications. Distance problems, for example,

are solved using the d = rt formula. See Example 2.19.• Many geometry problems are solved using the perimeter formulaP = 2L + 2W,the area formulaA =

LW,or the volume formulaV = LWH. See Example 2.20, Example 2.21, and Example 2.22.

2.4.4 Section Exercises

2.4.4.1 Verbal

Exercise 2.4.6 (Solution on p. 249.)

To set up a model linear equation to �t real-world applications, what should always be the �rststep?

Exercise 2.4.7Use your own words to describe this equation where n is a number:

5 (n+ 3) = 2nExercise 2.4.8 (Solution on p. 249.)

If the total amount of money you had to invest was $2,000 and you depositx amount in one invest-ment, how can you represent the remaining amount?

Exercise 2.4.9If a man sawed a 10-ft board into two sections and one section wasn ft long, how long would theother section be in terms ofn?

Exercise 2.4.10 (Solution on p. 249.)

If Bill was traveling vmi/h, how would you represent Daemon's speed if he was traveling 10 mi/hfaster?

2.4.4.2 Real-World Applications

For the following exercises, use the information to �nd a linear algebraic equation model to use to answerthe question being asked.

Exercise 2.4.11Mark and Don are planning to sell each of their marble collections at a garage sale. If Don has 1more than 3 times the number of marbles Mark has, how many does each boy have to sell if thetotal number of marbles is 113?

Exercise 2.4.12 (Solution on p. 249.)

Beth and Ann are joking that their combined ages equal Sam's age. If Beth is twice Ann's age andSam is 69 yr old, what are Beth and Ann's ages?

12http://openstaxcollege.org/l/ratetimesolve13http://openstaxcollege.org/l/lineqappl

Available for free at Connexions <http://legacy.cnx.org/content/col11759/1.2>

165

Exercise 2.4.13Ben originally �lled out 8 more applications than Henry. Then each boy �lled out 3 additionalapplications, bringing the total to 28. How many applications did each boy originally �ll out?

For the following exercises, use this scenario: Two di�erent telephone carriers o�er the following plans thata person is considering. Company A has a monthly fee of $20 and charges of $.05/min for calls. CompanyB has a monthly fee of $5 and charges $.10/min for calls.

Exercise 2.4.14 (Solution on p. 249.)

Find the model of the total cost of Company A's plan, usingm for the minutes.

Exercise 2.4.15Find the model of the total cost of Company B's plan, usingm for the minutes.

Exercise 2.4.16 (Solution on p. 249.)

Find out how many minutes of calling would make the two plans equal.

Exercise 2.4.17If the person makes a monthly average of 200 min of calls, which plan should for the person choose?

For the following exercises, use this scenario: A wireless carrier o�ers the following plans that a person isconsidering. The Family Plan: $90 monthly fee, unlimited talk and text on up to 5 lines, and data chargesof $40 for each device for up to 2 GB of data per device. The Mobile Share Plan: $120 monthly fee for up to10 devices, unlimited talk and text for all the lines, and data charges of $35 for each device up to a sharedtotal of 10 GB of data. UseP for the number of devices that need data plans as part of their cost.

Exercise 2.4.18 (Solution on p. 249.)

Find the model of the total cost of the Family Plan.

Exercise 2.4.19Find the model of the total cost of the Mobile Share Plan.

Exercise 2.4.20 (Solution on p. 249.)

Assuming they stay under their data limit, �nd the number of devices that would make the twoplans equal in cost.

Exercise 2.4.21If a family has 3 smart phones, which plan should they choose?

For exercises 17 and 18, use this scenario: A retired woman has $50,000 to invest but needs to make $6,000a year from the interest to meet certain living expenses. One bond investment pays 15% annual interest.The rest of it she wants to put in a CD that pays 7%.

Exercise 2.4.22 (Solution on p. 249.)

If we letx be the amount the woman invests in the 15% bond, how much will she be able to investin the CD?

Exercise 2.4.23Set up and solve the equation for how much the woman should invest in each option to sustain a$6,000 annual return.

Exercise 2.4.24 (Solution on p. 249.)

Two planes �y in opposite directions. One travels 450 mi/h and the other 550 mi/h. How long willit take before they are 4,000 mi apart?

Exercise 2.4.25Ben starts walking along a path at 4 mi/h. One and a half hours after Ben leaves, his sister Amandabegins jogging along the same path at 6 mi/h. How long will it be before Amanda catches up toBen?

Available for free at Connexions <http://legacy.cnx.org/content/col11759/1.2>

166 CHAPTER 2. EQUATIONS AND INEQUALITIES

Exercise 2.4.26 (Solution on p. 249.)

Fiora starts riding her bike at 20 mi/h. After a while, she slows down to 12 mi/h, and maintainsthat speed for the rest of the trip. The whole trip of 70 mi takes her 4.5 h. For what distance didshe travel at 20 mi/h?

Exercise 2.4.27A chemistry teacher needs to mix a 30% salt solution with a 70% salt solution to make 20 qt ofa 40% salt solution. How many quarts of each solution should the teacher mix to get the desiredresult?

Exercise 2.4.28 (Solution on p. 249.)

Paul has $20,000 to invest. His intent is to earn 11% interest on his investment. He can invest partof his money at 8% interest and part at 12% interest. How much does Paul need to invest in eachoption to make get a total 11% return on his $20,000?

For the following exercises, use this scenario: A truck rental agency o�ers two kinds of plans. Plan A charges$75/wk plus $.10/mi driven. Plan B charges $100/wk plus $.05/mi driven.

Exercise 2.4.29Write the model equation for the cost of renting a truck with plan A.

Exercise 2.4.30 (Solution on p. 249.)

Write the model equation for the cost of renting a truck with plan B.

Exercise 2.4.31Find the number of miles that would generate the same cost for both plans.

Exercise 2.4.32 (Solution on p. 249.)

If Tim knows he has to travel 300 mi, which plan should he choose?

For the following exercises, use the given formulas to answer the questions.

Exercise 2.4.33A = P (1 + rt) is used to �nd the principal amount Pdeposited, earning r% interest, for t years.Use this to �nd what principal amount P David invested at a 3% rate for 20 yr ifA = $8, 000.Exercise 2.4.34 (Solution on p. 249.)

The formulaF = mv2

R relates force (F), velocity (v), mass (m), and resistance (R). FindRwhenm =45,v = 7,andF = 245.Exercise 2.4.35F = ma indicates that force (F) equals mass (m) times acceleration (a). Find the acceleration of amass of 50 kg if a force of 12 N is exerted on it.

Exercise 2.4.36 (Solution on p. 249.)

Sum = 11−r is the formula for an in�nite series sum. If the sum is 5, �nd r.

For the following exercises, solve for the given variable in the formula. After obtaining a new version of theformula, you will use it to solve a question.

Exercise 2.4.37Solve for W :P = 2L+ 2WExercise 2.4.38 (Solution on p. 249.)

Use the formula from the previous question to �nd the width,W,of a rectangle whose length is 15and whose perimeter is 58.

Exercise 2.4.39Solve for f : 1

p + 1q = 1

f

Exercise 2.4.40 (Solution on p. 250.)

Use the formula from the previous question to �nd f when p = 8 and q = 13.Exercise 2.4.41Solve form in the slope-intercept formula: y = mx+ b

Available for free at Connexions <http://legacy.cnx.org/content/col11759/1.2>

167

Exercise 2.4.42 (Solution on p. 250.)

Use the formula from the previous question to �ndmwhen the coordinates of the pointare (4, 7) and b = 12.Exercise 2.4.43The area of a trapezoid is given byA = 1

2h (b1 + b2) .Use the formula to �nd the area of a trapezoidwithh = 6, b1 = 14, and b2 = 8.Exercise 2.4.44 (Solution on p. 250.)

Solve for h:A = 12h (b1 + b2)

Exercise 2.4.45Use the formula from the previous question to �nd the height of a trapezoid withA = 150, b1 =19, and b2 = 11.Exercise 2.4.46 (Solution on p. 250.)

Find the dimensions of an American football �eld. The length is 200 ft more than the width, andthe perimeter is 1,040 ft. Find the length and width. Use the perimeter formulaP = 2L+ 2W.Exercise 2.4.47Distance equals rate times time, d = rt.Find the distance Tom travels if he is moving at a rate of55 mi/h for 3.5 h.

Exercise 2.4.48 (Solution on p. 250.)

Using the formula in the previous exercise, �nd the distance that Susan travels if she is moving ata rate of 60 mi/h for 6.75 h.

Exercise 2.4.49What is the total distance that two people travel in 3 h if one of them is riding a bike at 15 mi/hand the other is walking at 3 mi/h?

Exercise 2.4.50 (Solution on p. 250.)

If the area model for a triangle isA = 12bh,�nd the area of a triangle with a height of 16 in. and a

base of 11 in.

Exercise 2.4.51Solve for h:A = 1

2bh

Exercise 2.4.52 (Solution on p. 250.)

Use the formula from the previous question to �nd the height to the nearest tenth of a trianglewith a base of 15 and an area of 215.

Exercise 2.4.53The volume formula for a cylinder isV = πr2h.Using the symbolπ in your answer, �nd the volumeof a cylinder with a radius, r,of 4 cm and a height of 14 cm.

Exercise 2.4.54 (Solution on p. 250.)

Solve for h: V = πr2h

Exercise 2.4.55Use the formula from the previous question to �nd the height of a cylinder with a radius of 8 anda volume of 16πExercise 2.4.56 (Solution on p. 250.)

Solve for r: V = πr2h

Exercise 2.4.57Use the formula from the previous question to �nd the radius of a cylinder with a height of 36 anda volume of 324π.Exercise 2.4.58 (Solution on p. 250.)

The formula for the circumference of a circle isC = 2πr.Find the circumference of a circle with adiameter of 12 in. (diameter = 2r). Use the symbolπ in your �nal answer.

Available for free at Connexions <http://legacy.cnx.org/content/col11759/1.2>

168 CHAPTER 2. EQUATIONS AND INEQUALITIES

Exercise 2.4.59Solve the formula from the previous question forπ.Notice whyπ is sometimes de�ned as the ratioof the circumference to its diameter.

2.5 Complex Numbers14

Figure 2.12

Discovered by Benoit Mandelbrot around 1980, the Mandelbrot Set is one of the most recognizable fractalimages. The image is built on the theory of self-similarity and the operation of iteration. Zooming in ona fractal image brings many surprises, particularly in the high level of repetition of detail that appears asmagni�cation increases. The equation that generates this image turns out to be rather simple.

In order to better understand it, we need to become familiar with a new set of numbers. Keep in mindthat the study of mathematics continuously builds upon itself. Negative integers, for example, �ll a void leftby the set of positive integers. The set of rational numbers, in turn, �lls a void left by the set of integers.

14This content is available online at <http://legacy.cnx.org/content/m51255/1.2/>.

Available for free at Connexions <http://legacy.cnx.org/content/col11759/1.2>

169

The set of real numbers �lls a void left by the set of rational numbers. Not surprisingly, the set of realnumbers has voids as well. In this section, we will explore a set of numbers that �lls voids in the set of realnumbers and �nd out how to work within it.

2.5.1 Expressing Square Roots of Negative Numbers as Multiples of i

We know how to �nd the square root of any positive real number. In a similar way, we can �nd the squareroot of any negative number. The di�erence is that the root is not real. If the value in the radicand isnegative, the root is said to be an imaginary number. The imaginary number i is de�ned as the square rootof − 1.

√−1 = i (2.57)

So, using properties of radicals,

i2 =(√−1)2

= −1 (2.58)

We can write the square root of any negative number as a multiple of i.Consider the square root of − 49.

√−49 =

√49 · (−1)

=√

49√−1

= 7i

(2.59)

We use 7i and not − 7i because the principal root of 49 is the positive root.A complex number is the sum of a real number and an imaginary number. A complex number is expressed

in standard form when written a+biwhere a is the real part and b is the imaginary part. For example, 5+2i isa complex number. So, too, is 3 + 4i

√3.

Imaginary numbers di�er from real numbers in that a squared imaginary number produces a negative realnumber. Recall that when a positive real number is squared, the result is a positive real number and whena negative real number is squared, the result is also a positive real number. Complex numbers consist ofreal and imaginary numbers.

A General Note: A complex number is a number of the form a+ biwhere

• a is the real part of the complex number.• b is the imaginary part of the complex number.

If b = 0,then a+ bi is a real number. If a = 0 and b is not equal to 0, the complex number is called apure imaginary number. An imaginary number is an even root of a negative number.

How To: Given an imaginary number, express it in the standard form of a complexnumber.

1.Write√−a as

√a√−1.

2.Express√−1 as i.

3.Write√a · i in simplest form.

Available for free at Connexions <http://legacy.cnx.org/content/col11759/1.2>

170 CHAPTER 2. EQUATIONS AND INEQUALITIES

Example 2.23Expressing an Imaginary Number in Standard FormExpress

√−9 in standard form.

Solution

√−9 =

√9√−1

= 3i(2.60)

In standard form, this is 0 + 3i.

Try It:

Exercise 2.5.1 (Solution on p. 250.)

Express√−24 in standard form.

2.5.2 Plotting a Complex Number on the Complex Plane

We cannot plot complex numbers on a number line as we might real numbers. However, we can stillrepresent them graphically. To represent a complex number, we need to address the two components of thenumber. We use the complex plane, which is a coordinate system in which the horizontal axis representsthe real component and the vertical axis represents the imaginary component. Complex numbers are thepoints on the plane, expressed as ordered pairs (a, b) ,where a represents the coordinate for the horizontalaxis and b represents the coordinate for the vertical axis.

Let's consider the number − 2 + 3i.The real part of the complex number is − 2 and the imaginary partis 3. We plot the ordered pair (−2, 3) to represent the complex number − 2 + 3i,as shown in Figure 2.13.

Available for free at Connexions <http://legacy.cnx.org/content/col11759/1.2>

171

Figure 2.13

A General Note: In the complex plane, the horizontal axis is the real axis, and the verticalaxis is the imaginary axis, as shown in Figure 2.14.

Available for free at Connexions <http://legacy.cnx.org/content/col11759/1.2>

172 CHAPTER 2. EQUATIONS AND INEQUALITIES

Figure 2.14

How To: Given a complex number, represent its components on the complex plane.

1.Determine the real part and the imaginary part of the complex number.2.Move along the horizontal axis to show the real part of the number.3.Move parallel to the vertical axis to show the imaginary part of the number.4.Plot the point.

Example 2.24Plotting a Complex Number on the Complex PlanePlot the complex number 3− 4i on the complex plane.

SolutionThe real part of the complex number is 3,and the imaginary part is �4. We plot the orderedpair (3,−4) as shown in Figure 2.15.

Available for free at Connexions <http://legacy.cnx.org/content/col11759/1.2>

173

Figure 2.15

Try It:

Exercise 2.5.2 (Solution on p. 250.)

Plot the complex number − 4− i on the complex plane.

2.5.3 Adding and Subtracting Complex Numbers

Just as with real numbers, we can perform arithmetic operations on complex numbers. To add or subtractcomplex numbers, we combine the real parts and then combine the imaginary parts.

Available for free at Connexions <http://legacy.cnx.org/content/col11759/1.2>

174 CHAPTER 2. EQUATIONS AND INEQUALITIES

A General Note: Adding complex numbers:

(a+ bi) + (c+ di) = (a+ c) + (b+ d) i (2.61)

Subtracting complex numbers:

(a+ bi)− (c+ di) = (a− c) + (b− d) i (2.62)

How To: Given two complex numbers, �nd the sum or di�erence.

1.Identify the real and imaginary parts of each number.2.Add or subtract the real parts.3.Add or subtract the imaginary parts.

Example 2.25Adding and Subtracting Complex NumbersAdd or subtract as indicated.

1. (3− 4i) + (2 + 5i)2. (−5 + 7i)− (−11 + 2i)

SolutionWe add the real parts and add the imaginary parts.

1.

(3− 4i) + (2 + 5i) = 3− 4i+ 2 + 5i

= 3 + 2 + (−4i) + 5i

= (3 + 2) + (−4 + 5) i

= 5 + i

2.

(−5 + 7i)− (−11 + 2i) = −5 + 7i+ 11− 2i

= −5 + 11 + 7i− 2i

= (−5 + 11) + (7− 2) i

= 6 + 5i

Try It:

Exercise 2.5.3 (Solution on p. 251.)

Subtract 2 + 5i from 3−−4i.

2.5.4 Multiplying Complex Numbers

Multiplying complex numbers is much like multiplying binomials. The major di�erence is that we work withthe real and imaginary parts separately.

Available for free at Connexions <http://legacy.cnx.org/content/col11759/1.2>

175

2.5.4.1 Multiplying a Complex Number by a Real Number

Lets begin by multiplying a complex number by a real number. We distribute the real number just as wewould with a binomial. Consider, for example, 3 (6 + 2i):

How To: Given a complex number and a real number, multiply to �nd the product.

1.Use the distributive property.2.Simplify.

Example 2.26Multiplying a Complex Number by a Real NumberFind the product 4 (2 + 5i) .

SolutionDistribute the 4.

4 (2 + 5i) = (4 · 2) + (4 · 5i)= 8 + 20i

(2.63)

Try It:

Exercise 2.5.4 (Solution on p. 251.)

Find the product: 12 (5− 2i) .

2.5.4.2 Multiplying Complex Numbers Together

Now, lets multiply two complex numbers. We can use either the distributive property or more speci�callythe FOIL method because we are dealing with binomials. Recall that FOIL is an acronym for multiplyingFirst, Inner, Outer, and Last terms together. The di�erence with complex numbers is that when we get asquared term, i2,it equals − 1.

(a+ bi) (c+ di) = ac+ adi+ bci+ bdi2

= ac+ adi+ bci− bd i2 = −1

= (ac− bd) + (ad+ bc) i Group real terms and imaginary terms.

(2.64)

How To: Given two complex numbers, multiply to �nd the product.

1.Use the distributive property or the FOIL method.2.Remember that i2 = −1.

Available for free at Connexions <http://legacy.cnx.org/content/col11759/1.2>

176 CHAPTER 2. EQUATIONS AND INEQUALITIES

3.Group together the real terms and the imaginary terms

Example 2.27Multiplying a Complex Number by a Complex NumberMultiply (4 + 3i) (2− 5i) .

Solution

(4 + 3i) (2− 5i) = 4 (2)− 4 (5i) + 3i (2)− (3i) (5i)

= 8− 20i+ 6i− 15(i2)

= (8 + 15) + (−20 + 6) i

= 23− 14i

(2.65)

Try It:

Exercise 2.5.5 (Solution on p. 251.)

Multiply: (3− 4i) (2 + 3i) .

2.5.5 Dividing Complex Numbers

Dividing two complex numbers is more complicated than adding, subtracting, or multiplying because wecannot divide by an imaginary number, meaning that any fraction must have a real-number denominator towrite the answer in standard form a + bi.We need to �nd a term by which we can multiply the numeratorand the denominator that will eliminate the imaginary portion of the denominator so that we end up witha real number as the denominator. This term is called the complex conjugate of the denominator, whichis found by changing the sign of the imaginary part of the complex number. In other words, the complexconjugate of a+ bi is a− bi.For example, the product of a+ bi and a− bi is

(a+ bi) (a− bi) = a2 − abi+ abi− b2i2

= a2 + b2(2.66)

The result is a real number.Note that complex conjugates have an opposite relationship: The complex conjugate of a+bi is a−bi,and

the complex conjugate of a−bi is a+bi.Further, when a quadratic equation with real coe�cients has complexsolutions, the solutions are always complex conjugates of one another.

Suppose we want to divide c+ di by a+ bi,where neither a nor b equals zero. We �rst write the division asa fraction, then �nd the complex conjugate of the denominator, and multiply.

c+ di

a+ biwhere a 6= 0 and b 6= 0 (2.67)

Multiply the numerator and denominator by the complex conjugate of the denominator.

(c+ di)(a+ bi)

· (a− bi)(a− bi)

=(c+ di) (a− bi)(a+ bi) (a− bi)

(2.68)

Available for free at Connexions <http://legacy.cnx.org/content/col11759/1.2>

177

Apply the distributive property.

=ca− cbi+ adi− bdi2

a2 − abi+ abi− b2i2(2.69)

Simplify, remembering that i2 = −1.

= ca−cbi+adi−bd(−1)a2−abi+abi−b2(−1)

= (ca+bd)+(ad−cb)ia2+b2

(2.70)

A General Note: The complex conjugate of a complex number a+ bi is a− bi. It is found bychanging the sign of the imaginary part of the complex number. The real part of the number is leftunchanged.

• When a complex number is multiplied by its complex conjugate, the result is a real number.• When a complex number is added to its complex conjugate, the result is a real number.

Example 2.28Finding Complex ConjugatesFind the complex conjugate of each number.

1. 2 + i√

52. − 1

2 i

Solution

1. The number is already in the form a+ bi.The complex conjugate is a− bi,or 2− i√

5.2. We can rewrite this number in the form a+ bi as 0− 1

2 i.The complex conjugate is a− bi,or 0 +12 i.This can be written simply as 1

2 i.

AnalysisAlthough we have seen that we can �nd the complex conjugate of an imaginary number, in practicewe generally �nd the complex conjugates of only complex numbers with both a real and an imaginarycomponent. To obtain a real number from an imaginary number, we can simply multiply by i.

Try It:

Exercise 2.5.6 (Solution on p. 251.)

Find the complex conjugate of − 3 + 4i.

How To: Given two complex numbers, divide one by the other.

1.Write the division problem as a fraction.2.Determine the complex conjugate of the denominator.3.Multiply the numerator and denominator of the fraction by the complex conjugate of thedenominator.

4.Simplify.

Available for free at Connexions <http://legacy.cnx.org/content/col11759/1.2>

178 CHAPTER 2. EQUATIONS AND INEQUALITIES

Example 2.29Dividing Complex NumbersDivide: (2 + 5i) by (4− i) .SolutionWe begin by writing the problem as a fraction.

(2 + 5i)(4− i)

(2.71)

Then we multiply the numerator and denominator by the complex conjugate of the denominator.

(2 + 5i)(4− i)

· (4 + i)(4 + i)

(2.72)

To multiply two complex numbers, we expand the product as we would with polynomials (usingFOIL).

(2+5i)(4−i) ·

(4+i)(4+i) = 8+2i+20i+5i2

16+4i−4i−i2

= 8+2i+20i+5(−1)16+4i−4i−(−1) Because i2 = −1.

= 3+22i17

= 317 + 22

17 i Separate real and imaginary parts.

(2.73)

Note that this expresses the quotient in standard form.

2.5.6 Simplifying Powers of i

The powers of i are cyclic. Let's look at what happens when we raise i to increasing powers.

i1 = i

i2 = −1

i3 = i2 · i = −1 · i = −ii4 = i3 · i = −i · i = −i2 = − (−1) = 1

i5 = i4 · i = 1 · i = i

(2.74)

We can see that when we get to the �fth power of i,it is equal to the �rst power. As we continue to multiply iby increasing powers, we will see a cycle of four. Let's examine the next four powers of i.

i6 = i5 · i = i · i = i2 = −1

i7 = i6 · i = i2 · i = i3 = −ii8 = i7 · i = i3 · i = i4 = 1

i9 = i8 · i = i4 · i = i5 = i

(2.75)

The cycle is repeated continuously: i,−1,−i, 1,every four powers.

Example 2.30Simplifying Powers of iEvaluate: i35.

Available for free at Connexions <http://legacy.cnx.org/content/col11759/1.2>

179

SolutionSince i4 = 1,we can simplify the problem by factoring out as many factors of i4 as possible. To doso, �rst determine how many times 4 goes into 35: 35 = 4 · 8 + 3.

i35 = i4·8+3 = i4·8 · i3 =(i4)8 · i3 = 18 · i3 = i3 = −i (2.76)

Try It:

Exercise 2.5.7 (Solution on p. 251.)

Evaluate: i18

Q&A: Can we write i35 in other helpful ways?

As we saw in Example 2.30, we reduced i35 to i3 by dividing the exponent by 4 and using theremainder to �nd the simpli�ed form. But perhaps another factorization of i35 may be more useful.Table 2.3 shows some other possible factorizations.

Factorization of i35 i34 · i i33 · i2 i31 · i4 i19 · i16

Reduced form(i2)17 · i i33 · (−1) i31 · 1 i19 ·

(i4)4

Simpli�ed form (−1)17 · i −i33 i31 i19

Table 2.3

Each of these will eventually result in the answer we obtained above but may require several moresteps than our earlier method.

Media: Access these online resources for additional instruction and practice with complex num-bers.

• Adding and Subtracting Complex Numbers15

• Multiply Complex Numbers16

• Multiplying Complex Conjugates17

• Raising i to Powers18

15http://openstaxcollege.org/l/addsubcomplex16http://openstaxcollege.org/l/multiplycomplex17http://openstaxcollege.org/l/multcompconj18http://openstaxcollege.org/l/raisingi

Available for free at Connexions <http://legacy.cnx.org/content/col11759/1.2>

180 CHAPTER 2. EQUATIONS AND INEQUALITIES

2.5.7 Key Concepts

• The square root of any negative number can be written as a multiple of i. See Example 2.23.• To plot a complex number, we use two number lines, crossed to form the complex plane. The horizontal

axis is the real axis, and the vertical axis is the imaginary axis. See Example 2.24.• Complex numbers can be added and subtracted by combining the real parts and combining the imag-

inary parts. See Example 2.25.• Complex numbers can be multiplied and divided.

◦ To multiply complex numbers, distribute just as with polynomials. See Example 2.26 and Exam-ple 2.27.

◦ To divide complex numbers, multiply both numerator and denominator by the complex conjugateof the denominator to eliminate the complex number from the denominator. See Example 2.28and Example 2.29.

• The powers of i are cyclic, repeating every fourth one. See Example 2.30.

2.5.8 Section Exercises

2.5.8.1 Verbal

Exercise 2.5.8 (Solution on p. 251.)

Explain how to add complex numbers.

Exercise 2.5.9What is the basic principle in multiplication of complex numbers?

Exercise 2.5.10 (Solution on p. 251.)

Give an example to show that the product of two imaginary numbers is not always imaginary.

Exercise 2.5.11What is a characteristic of the plot of a real number in the complex plane?

2.5.8.2 Algebraic

For the following exercises, evaluate the algebraic expressions.

Exercise 2.5.12 (Solution on p. 251.)

If y = x2 + x− 4,evaluate y givenx = 2i.Exercise 2.5.13If y = x3 − 2,evaluate y givenx = i.

Exercise 2.5.14 (Solution on p. 251.)

If y = x2 + 3x+ 5,evaluate y givenx = 2 + i.

Exercise 2.5.15If y = 2x2 + x− 3,evaluate y givenx = 2− 3i.Exercise 2.5.16 (Solution on p. 252.)

If y = x+12−x ,evaluate y givenx = 5i.

Exercise 2.5.17If y = 1+2x

x+3 ,evaluate y givenx = 4i.

Available for free at Connexions <http://legacy.cnx.org/content/col11759/1.2>

181

2.5.8.3 Graphical

For the following exercises, plot the complex numbers on the complex plane.

Exercise 2.5.18 (Solution on p. 252.)

1− 2iExercise 2.5.19−2 + 3iExercise 2.5.20 (Solution on p. 252.)

i

Exercise 2.5.21−3− 4i

2.5.8.4 Numeric

For the following exercises, perform the indicated operation and express the result as a simpli�ed complexnumber.

Exercise 2.5.22 (Solution on p. 253.)

(3 + 2i) + (5− 3i)Exercise 2.5.23(−2− 4i) + (1 + 6i)Exercise 2.5.24 (Solution on p. 253.)

(−5 + 3i)− (6− i)Exercise 2.5.25(2− 3i)− (3 + 2i)Exercise 2.5.26 (Solution on p. 253.)

(−4 + 4i)− (−6 + 9i)Exercise 2.5.27(2 + 3i) (4i)Exercise 2.5.28 (Solution on p. 253.)

(5− 2i) (3i)Exercise 2.5.29(6− 2i) (5)Exercise 2.5.30 (Solution on p. 253.)

(−2 + 4i) (8)Exercise 2.5.31(2 + 3i) (4− i)Exercise 2.5.32 (Solution on p. 253.)

(−1 + 2i) (−2 + 3i)Exercise 2.5.33(4− 2i) (4 + 2i)Exercise 2.5.34 (Solution on p. 253.)

(3 + 4i) (3− 4i)Exercise 2.5.353+4i

2

Exercise 2.5.36 (Solution on p. 253.)6−2i

3

Available for free at Connexions <http://legacy.cnx.org/content/col11759/1.2>

182 CHAPTER 2. EQUATIONS AND INEQUALITIES

Exercise 2.5.37−5+3i

2i

Exercise 2.5.38 (Solution on p. 253.)6+4ii

Exercise 2.5.392−3i4+3i

Exercise 2.5.40 (Solution on p. 253.)3+4i2−iExercise 2.5.412+3i2−3i

Exercise 2.5.42 (Solution on p. 253.)√−9 + 3

√−16

Exercise 2.5.43−√−4− 4

√−25

Exercise 2.5.44 (Solution on p. 253.)2+√−12

2

Exercise 2.5.454+√−20

2

Exercise 2.5.46 (Solution on p. 254.)

i8

Exercise 2.5.47i15

Exercise 2.5.48 (Solution on p. 254.)

i22

2.5.8.5 Technology

For the following exercises, use a calculator to help answer the questions.

Exercise 2.5.49Evaluate (1 + i)k for k = 4, 8, and 12.Predict the value if k = 16.Exercise 2.5.50 (Solution on p. 254.)

Evaluate (1− i)k for k = 2, 6, and 10.Predict the value if k = 14.Exercise 2.5.51Evaluate(l + i)k − (l− i)kfor k = 4, 8, and 12.Predict the value for k = 16.Exercise 2.5.52 (Solution on p. 254.)

Show that a solution ofx6 + 1 = 0 is√

32 + 1

2 i.

Exercise 2.5.53Show that a solution ofx8 − 1 = 0 is

√2

2 +√

22 i.

Available for free at Connexions <http://legacy.cnx.org/content/col11759/1.2>

183

2.5.8.6 Extensions

For the following exercises, evaluate the expressions, writing the result as a simpli�ed complex number.

Exercise 2.5.54 (Solution on p. 254.)1i + 4

i3

Exercise 2.5.551i11 −

1i21

Exercise 2.5.56 (Solution on p. 254.)

i7(1 + i2

)Exercise 2.5.57i−3 + 5i7

Exercise 2.5.58 (Solution on p. 254.)(2+i)(4−2i)

(1+i)

Exercise 2.5.59(1+3i)(2−4i)

(1+2i)

Exercise 2.5.60 (Solution on p. 254.)(3+i)2

(1+2i)2

Exercise 2.5.613+2i2+i + (4 + 3i)Exercise 2.5.62 (Solution on p. 254.)4+ii + 3−4i

1−iExercise 2.5.633+2i1+2i −

2−3i3+i

Available for free at Connexions <http://legacy.cnx.org/content/col11759/1.2>

184 CHAPTER 2. EQUATIONS AND INEQUALITIES

2.6 Quadratic Equations19

Figure 2.16

The computer monitor on the left in Figure 2.16 is a 23.6-inch model and the one on the right is a 27-inchmodel. Proportionally, the monitors appear very similar. If there is a limited amount of space and we desirethe largest monitor possible, how do we decide which one to choose? In this section, we will learn how tosolve problems such as this using four di�erent methods.

2.6.1 Solving Quadratic Equations by Factoring

An equation containing a second-degree polynomial is called a quadratic equation. For example, equationssuch as 2x2 + 3x−1 = 0 andx2−4 = 0 are quadratic equations. They are used in countless ways in the �eldsof engineering, architecture, �nance, biological science, and, of course, mathematics.

Often the easiest method of solving a quadratic equation is factoring. Factoring means �nding expres-sions that can be multiplied together to give the expression on one side of the equation.

If a quadratic equation can be factored, it is written as a product of linear terms. Solving by factoringdepends on the zero-product property, which states that if a · b = 0,then a = 0 or b = 0,where a and b are realnumbers or algebraic expressions. In other words, if the product of two numbers or two expressions equalszero, then one of the numbers or one of the expressions must equal zero because zero multiplied by anythingequals zero.

Multiplying the factors expands the equation to a string of terms separated by plus or minus signs. So,in that sense, the operation of multiplication undoes the operation of factoring. For example, expand thefactored expression (x− 2) (x+ 3) by multiplying the two factors together.

(x− 2) (x+ 3) = x2 + 3x− 2x− 6

= x2 + x− 6(2.77)

19This content is available online at <http://legacy.cnx.org/content/m51256/1.2/>.

Available for free at Connexions <http://legacy.cnx.org/content/col11759/1.2>

185

The product is a quadratic expression. Set equal to zero,x2 + x− 6 = 0 is a quadratic equation. If we wereto factor the equation, we would get back the factors we multiplied.

The process of factoring a quadratic equation depends on the leading coe�cient, whether it is 1 or anotherinteger. We will look at both situations; but �rst, we want to con�rm that the equation is written in standardform, ax2 + bx + c = 0,where a, b, and c are real numbers, and a 6= 0.The equationx2 + x − 6 = 0 is instandard form.

We can use the zero-product property to solve quadratic equations in which we �rst have to factor outthe greatest common factor (GCF), and for equations that have special factoring formulas as well, suchas the di�erence of squares, both of which we will see later in this section.

A General Note: The zero-product property states

If a · b = 0, then a = 0 or b = 0, (2.78)

where a and b are real numbers or algebraic expressions.

A quadratic equation is an equation containing a second-degree polynomial; for example

ax2 + bx+ c = 0 (2.79)

where a, b, and c are real numbers, and if a 6= 0,it is in standard form.

2.6.1.1 Solving Quadratics with a Leading Coe�cient of 1

In the quadratic equationx2 + x − 6 = 0,the leading coe�cient, or the coe�cient ofx2,is 1. We have onemethod of factoring quadratic equations in this form.

How To: Given a quadratic equation with the leading coe�cient of 1, factor it.

1.Find two numbers whose product equals c and whose sum equals b.2.Use those numbers to write two factors of the form (x+ k) or (x− k) ,where k is one of thenumbers found in step 1. Use the numbers exactly as they are. In other words, if the twonumbers are 1 and − 2,the factors are (x+ 1) (x− 2) .

3.Solve using the zero-product property by setting each factor equal to zero and solving for thevariable.

Example 2.31Factoring and Solving a Quadratic with Leading Coe�cient of 1Factor and solve the equation:x2 + x− 6 = 0.

SolutionTo factorx2 + x− 6 = 0,we look for two numbers whose product equals − 6 and whose sum equals1. Begin by looking at the possible factors of − 6.

1 · (−6)

(−6) · 12 · (−3)

3 · (−2)

(2.80)

The last pair, 3 · (−2) sums to 1, so these are the numbers. Note that only one pair of numberswill work. Then, write the factors.

(x− 2) (x+ 3) = 0 (2.81)

Available for free at Connexions <http://legacy.cnx.org/content/col11759/1.2>

186 CHAPTER 2. EQUATIONS AND INEQUALITIES

To solve this equation, we use the zero-product property. Set each factor equal to zero and solve.

(x− 2) (x+ 3) = 0

(x− 2) = 0

x = 2

(x+ 3) = 0

x = −3

(2.82)

The two solutions arex = 2 andx = −3.We can see how the solutions relate to the graph inFigure 2.17. The solutions are the x-intercepts ofx2 + x− 6 = 0.

Available for free at Connexions <http://legacy.cnx.org/content/col11759/1.2>

187

Figure 2.17

Available for free at Connexions <http://legacy.cnx.org/content/col11759/1.2>

188 CHAPTER 2. EQUATIONS AND INEQUALITIES

Try It:

Exercise 2.6.1 (Solution on p. 254.)

Factor and solve the quadratic equation:x2 − 5x− 6 = 0.

Example 2.32Solve the Quadratic Equation by FactoringSolve the quadratic equation by factoring:x2 + 8x+ 15 = 0.

SolutionFind two numbers whose product equals 15 and whose sum equals 8.List the factors of 15.

1 · 15

3 · 5(−1) · (−15)

(−3) · (−5)

(2.83)

The numbers that add to 8 are 3 and 5. Then, write the factors, set each factor equal to zero, andsolve.

(x+ 3) (x+ 5) = 0

(x+ 3) = 0

x = −3

(x+ 5) = 0

x = −5

(2.84)

The solutions arex = −3 andx = −5.

Try It:

Exercise 2.6.2 (Solution on p. 254.)

Solve the quadratic equation by factoring:x2 − 4x− 21 = 0.

Example 2.33Using the Zero-Product Property to Solve a Quadratic Equation Written as the

Di�erence of SquaresSolve the di�erence of squares equation using the zero-product property:x2 − 9 = 0.

SolutionRecognizing that the equation represents the di�erence of squares, we can write the two factorsby taking the square root of each term, using a minus sign as the operator in one factor and a plus

Available for free at Connexions <http://legacy.cnx.org/content/col11759/1.2>

189

sign as the operator in the other. Solve using the zero-factor property.

x2 − 9 = 0

(x− 3) (x+ 3) = 0

(x− 3) = 0

x = 3

(x+ 3) = 0

x = −3

(2.85)

The solutions arex = 3 andx = −3.

Try It:

Exercise 2.6.3 (Solution on p. 254.)

Solve by factoring:x2 − 25 = 0.

2.6.1.2 Factoring and Solving a Quadratic Equation of Higher Order

When the leading coe�cient is not 1, we factor a quadratic equation using the method called grouping,which requires four terms. With the equation in standard form, let's review the grouping procedures:

1. With the quadratic in standard form, ax2 + bx+ c = 0,multiply a · c.2. Find two numbers whose product equals ac and whose sum equals b.3. Rewrite the equation replacing the bx term with two terms using the numbers found in step 1 as

coe�cients of x.4. Factor the �rst two terms and then factor the last two terms. The expressions in parentheses must be

exactly the same to use grouping.5. Factor out the expression in parentheses.6. Set the expressions equal to zero and solve for the variable.

Example 2.34Solving a Quadratic Equation Using GroupingUse grouping to factor and solve the quadratic equation: 4x2 + 15x+ 9 = 0.

SolutionFirst, multiply ac : 4 (9) = 36.Then list the factors of 36.

1 · 36

2 · 18

3 · 12

4 · 96 · 6

(2.86)

Available for free at Connexions <http://legacy.cnx.org/content/col11759/1.2>

190 CHAPTER 2. EQUATIONS AND INEQUALITIES

The only pair of factors that sums to 15 is 3+12.Rewrite the equation replacing the b term, 15x,withtwo terms using 3 and 12 as coe�cients of x. Factor the �rst two terms, and then factor the lasttwo terms.

4x2 + 3x+ 12x+ 9 = 0

x (4x+ 3) + 3 (4x+ 3) = 0

(4x+ 3) (x+ 3) = 0

(2.87)

Solve using the zero-product property.

(4x+ 3) (x+ 3) = 0

(4x+ 3) = 0

x = − 34

(x+ 3) = 0

x = −3

(2.88)

The solutions arex = − 34 ,x = −3. See Figure 2.18.

Available for free at Connexions <http://legacy.cnx.org/content/col11759/1.2>

191

Figure 2.18

Try It:

Exercise 2.6.4 (Solution on p. 254.)

Solve using factoring by grouping: 12x2 + 11x+ 2 = 0.

Example 2.35Solving a Higher Degree Quadratic Equation by FactoringSolve the equation by factoring: − 3x3 − 5x2 − 2x = 0.

Available for free at Connexions <http://legacy.cnx.org/content/col11759/1.2>

192 CHAPTER 2. EQUATIONS AND INEQUALITIES

SolutionThis equation does not look like a quadratic, as the highest power is 3, not 2. Recall that the �rstthing we want to do when solving any equation is to factor out the GCF, if one exists. And it doeshere. We can factor out − x from all of the terms and then proceed with grouping.

−3x3 − 5x2 − 2x = 0

−x(3x2 + 5x+ 2

)= 0

(2.89)

Use grouping on the expression in parentheses.

− x(3x2 + 3x+ 2x+ 2

)= 0

−x [3x (x+ 1) + 2 (x+ 1)] = 0

− x (3x+ 2) (x+ 1) = 0

(2.90)

Now, we use the zero-product property. Notice that we have three factors.

− x = 0

x = 0

3x+ 2 = 0

x = − 23

x+ 1 = 0

x = −1

(2.91)

The solutions arex = 0,x = − 23 ,andx = −1.

Try It:

Exercise 2.6.5 (Solution on p. 254.)

Solve by factoring:x3 + 11x2 + 10x = 0.

2.6.2 Using the Square Root Property

When there is no linear term in the equation, another method of solving a quadratic equation is by usingthe square root property, in which we isolate thex2 term and take the square root of the number on theother side of the equals sign. Keep in mind that sometimes we may have to manipulate the equation toisolate thex2 term so that the square root property can be used.

A General Note: With thex2 term isolated, the square root property states that:

ifx2 = k, thenx = ±√k (2.92)

where k is a nonzero real number.

Available for free at Connexions <http://legacy.cnx.org/content/col11759/1.2>

193

How To: Given a quadratic equation with an x2 term but no x term, use the squareroot property to solve it.

1.Isolate thex2 term on one side of the equal sign.2.Take the square root of both sides of the equation, putting a ± sign before the expression onthe side opposite the squared term.

3.Simplify the numbers on the side with the ± sign.

Example 2.36Solving a Simple Quadratic Equation Using the Square Root PropertySolve the quadratic using the square root property:x2 = 8.

SolutionTake the square root of both sides, and then simplify the radical. Remember to use a ± sign beforethe radical symbol.

x2 = 8

x = ±√

8

= ±2√

2

(2.93)

The solutions arex = 2√

2,x = −2√

2.

Example 2.37Solving a Quadratic Equation Using the Square Root PropertySolve the quadratic equation: 4x2 + 1 = 7

SolutionFirst, isolate thex2 term. Then take the square root of both sides.

4x2 + 1 = 7

4x2 = 6

x2 = 64

x = ±√

62

(2.94)

The solutions arex =√

62 ,x = −

√6

2 .

Try It:

Exercise 2.6.6 (Solution on p. 254.)

Solve the quadratic equation using the square root property: 3(x− 4)2 = 15.

Available for free at Connexions <http://legacy.cnx.org/content/col11759/1.2>

194 CHAPTER 2. EQUATIONS AND INEQUALITIES

2.6.3 Completing the Square

Not all quadratic equations can be factored or can be solved in their original form using the square rootproperty. In these cases, we may use a method for solving a quadratic equation known as completingthe square. Using this method, we add or subtract terms to both sides of the equation until we have aperfect square trinomial on one side of the equal sign. We then apply the square root property. To completethe square, the leading coe�cient, a, must equal 1. If it does not, then divide the entire equation by a.Then, we can use the following procedures to solve a quadratic equation by completing the square.

We will use the examplex2 + 4x+ 1 = 0 to illustrate each step.

1. Given a quadratic equation that cannot be factored, and with a = 1,�rst add or subtract the constantterm to the right sign of the equal sign.

x2 + 4x = −1 (2.95)

2. Multiply the b term by 12 and square it.

12 (4) = 2

22 = 4(2.96)

3. Add(

12b)2to both sides of the equal sign and simplify the right side. We have

x2 + 4x+ 4 = −1 + 4

x2 + 4x+ 4 = 3(2.97)

4. The left side of the equation can now be factored as a perfect square.

x2 + 4x+ 4 = 3

(x+ 2)2 = 3(2.98)

5. Use the square root property and solve. √(x+ 2)2 = ±

√3

x+ 2 = ±√

3

x = −2±√

3

(2.99)

6. The solutions arex = −2 +√

3,x = −2−√

3.

Example 2.38Solving a Quadratic by Completing the SquareSolve the quadratic equation by completing the square:x2 − 3x− 5 = 0.

SolutionFirst, move the constant term to the right side of the equal sign.

x2 − 3x = 5 (2.100)

Then, take 12 of the b term and square it.

12 (−3) = − 3

2(− 3

2

)2 = 94

(2.101)

Available for free at Connexions <http://legacy.cnx.org/content/col11759/1.2>

195

Add the result to both sides of the equal sign.

x2 − 3x+(− 3

2

)2 = 5 +(− 3

2

)2x2 − 3x+ 9

4 = 5 + 94

(2.102)

Factor the left side as a perfect square and simplify the right side.(x− 3

2

)2

=294

(2.103)

Use the square root property and solve.√(x− 3

2

)2 = ±√

294(

x− 32

)= ±

√292

x = 32 ±

√292

(2.104)

The solutions arex = 32 +

√292 ,x = 3

2 −√

292 .

Try It:

Exercise 2.6.7 (Solution on p. 254.)

Solve by completing the square:x2 − 6x = 13.

2.6.4 Using the Quadratic Formula

The fourth method of solving a quadratic equation is by using the quadratic formula, a formula thatwill solve all quadratic equations. Although the quadratic formula works on any quadratic equation instandard form, it is easy to make errors in substituting the values into the formula. Pay close attention whensubstituting, and use parentheses when inserting a negative number.

We can derive the quadratic formula by completing the square. We will assume that the leadingcoe�cient is positive; if it is negative, we can multiply the equation by−1 and obtain a positive a. Given ax2+bx+ c = 0,a 6= 0,we will complete the square as follows:

1. First, move the constant term to the right side of the equal sign:

ax2 + bx = −c (2.105)

2. As we want the leading coe�cient to equal 1, divide through by a:

x2 +b

ax = − c

a(2.106)

3. Then, �nd 12 of the middle term, and add

(12ba

)2= b2

4a2 to both sides of the equal sign:

x2 +b

ax+

b2

4a2=

b2

4a2− c

a(2.107)

5. Available for free at Connexions <http://legacy.cnx.org/content/col11759/1.2>

196 CHAPTER 2. EQUATIONS AND INEQUALITIES

4. Next, write the left side as a perfect square. Find the common denominator of the right side and writeit as a single fraction: (

x+b

2a

)2

=b2 − 4ac

4a2(2.108)

Now, use the square root property, which gives

x+ b2a = ±

√b2−4ac

4a2

x+ b2a = ±

√b2−4ac2a

(2.109)

6. Finally, add − b2a to both sides of the equation and combine the terms on the right side. Thus,

x =−b±

√b2 − 4ac

2a(2.110)

A General Note: Written in standard form, ax2 + bx + c = 0,any quadratic equation can besolved using the quadratic formula:

x =−b±

√b2 − 4ac

2a(2.111)

where a, b, and c are real numbers and a 6= 0.

How To: Given a quadratic equation, solve it using the quadratic formula

1.Make sure the equation is in standard form: ax2 + bx+ c = 0.2.Make note of the values of the coe�cients and constant term, a, b,and c.3.Carefully substitute the values noted in step 2 into the equation. To avoid needless errors, useparentheses around each number input into the formula.

4.Calculate and solve.

Example 2.39Solve the Quadratic Equation Using the Quadratic FormulaSolve the quadratic equation:x2 + 5x+ 1 = 0.

SolutionIdentify the coe�cients: a = 1, b = 5, c = 1.Then use the quadratic formula.

x = −(5)±√

(5)2−4(1)(1)

2(1)

= −5±√

25−42

= −5±√

212

(2.112)

Example 2.40Solving a Quadratic Equation with the Quadratic FormulaUse the quadratic formula to solvex2 + x+ 2 = 0.

SolutionFirst, we identify the coe�cients: a = 1, b = 1,and c = 2.

Available for free at Connexions <http://legacy.cnx.org/content/col11759/1.2>

197

Substitute these values into the quadratic formula.

x = −b±√b2−4ac

2a

= −(1)±√

(1)2−(4)·(1)·(2)

2·1

= −1±√

1−82

= −1±√−7

2

= −1±i√

72

(2.113)

The solutions to the equation arex = −1+i√

72 andx = −1−i

√7

2 or x = −12 + i

√7

2 andx = −12 −

i√

72 .

Try It:

Exercise 2.6.8 (Solution on p. 254.)

Solve the quadratic equation using the quadratic formula: 9x2 + 3x− 2 = 0.

2.6.5 The Discriminant

The quadratic formula not only generates the solutions to a quadratic equation, it tells us about thenature of the solutions when we consider the discriminant, or the expression under the radical, b2−4ac.Thediscriminant tells us whether the solutions are real numbers or complex numbers, and how many solutions ofeach type to expect. Table 2.4 relates the value of the discriminant to the solutions of a quadratic equation.

Value of Discriminant Results

b2 − 4ac = 0 One rational solution (double solution)

b2 − 4ac > 0,perfect square Two rational solutions

b2 − 4ac > 0,not a perfect square Two irrational solutions

b2 − 4ac < 0 Two complex solutions

Table 2.4

A General Note: For ax2 + bx + c = 0,where a,b,and c are real numbers, the discriminant isthe expression under the radical in the quadratic formula: b2− 4ac. It tells us whether the solutionsare real numbers or complex numbers and how many solutions of each type to expect.

Example 2.41Using the Discriminant to Find the Nature of the Solutions to a Quadratic EquationUse the discriminant to �nd the nature of the solutions to the following quadratic equations:

a. x2 + 4x+ 4 = 0b. 8x2 + 14x+ 3 = 0c. 3x2 − 5x− 2 = 0d. 3x2 − 10x+ 15 = 0

Available for free at Connexions <http://legacy.cnx.org/content/col11759/1.2>

198 CHAPTER 2. EQUATIONS AND INEQUALITIES

SolutionCalculate the discriminant b2 − 4ac for each equation and state the expected type of solutions.

a. x2 + 4x+ 4 = 0b2 − 4ac = (4)2 − 4 (1) (4) = 0.There will be one rational double solution.

b. 8x2 + 14x+ 3 = 0b2−4ac = (14)2−4 (8) (3) = 100.As 100 is a perfect square, there will be two rational solutions.

c. 3x2 − 5x− 2 = 0b2−4ac = (−5)2−4 (3) (−2) = 49.As 49 is a perfect square, there will be two rational solutions.

d. 3x2 − 10x+ 15 = 0b2 − 4ac = (−10)2 − 4 (3) (15) = −80.There will be two complex solutions.

2.6.6 Using the Pythagorean Theorem

One of the most famous formulas in mathematics is the Pythagorean Theorem. It is based on a righttriangle, and states the relationship among the lengths of the sides as a2 + b2 = c2,where a and b refer to thelegs of a right triangle adjacent to the 90 ◦ angle, and c refers to the hypotenuse. It has immeasurable uses inarchitecture, engineering, the sciences, geometry, trigonometry, and algebra, and in everyday applications.

We use the Pythagorean Theorem to solve for the length of one side of a triangle when we have thelengths of the other two. Because each of the terms is squared in the theorem, when we are solving for aside of a triangle, we have a quadratic equation. We can use the methods for solving quadratic equationsthat we learned in this section to solve for the missing side.

The Pythagorean Theorem is given as

a2 + b2 = c2 (2.114)

where a and b refer to the legs of a right triangle adjacent to the 90◦ angle, and c refers to the hypotenuse, asshown in Figure 2.19.

Available for free at Connexions <http://legacy.cnx.org/content/col11759/1.2>

199

Figure 2.19

Example 2.42Problem 1Finding the Length of the Missing Side of a Right TriangleFind the length of the missing side of the right triangle in Figure 2.20.

Figure 2.20

Available for free at Connexions <http://legacy.cnx.org/content/col11759/1.2>

200 CHAPTER 2. EQUATIONS AND INEQUALITIES

SolutionAs we have measurements for side b and the hypotenuse, the missing side is a.

a2 + b2 = c2

a2 + (4)2 = (12)2

a2 + 16 = 144

a2 = 128

a =√

128

= 8√

2

(2.115)

Try It:

Problem 2

Use the Pythagorean Theorem to solve the right triangle problem: Leg a measures 4 units,leg b measures 3 units. Find the length of the hypotenuse.

Solution

5 units

Media: Access these online resources for additional instruction and practice with quadraticequations.

• Solving Quadratic Equations by Factoring20

• The Zero-Product Property21• Completing the Square22• Quadratic Formula with Two Rational Solutions23

• Length of a leg of a right triangle24

2.6.7 Key Equations

quadratic formula x = −b±√b2−4ac

2a

Table 2.5

20http://openstaxcollege.org/l/quadreqfactor21http://openstaxcollege.org/l/zeroprodprop22http://openstaxcollege.org/l/complthesqr23http://openstaxcollege.org/l/quadrformrat24http://openstaxcollege.org/l/leglengthtri

Available for free at Connexions <http://legacy.cnx.org/content/col11759/1.2>

201

2.6.8 Key Concepts

• Many quadratic equations can be solved by factoring when the equation has a leading coe�cient of 1or if the equation is a di�erence of squares. The zero-factor property is then used to �nd solutions.See Example 2.31, Example 2.32, and Example 2.33.

• Many quadratic equations with a leading coe�cient other than 1 can be solved by factoring using thegrouping method. See Example 2.34 and Example 2.35.

• Another method for solving quadratics is the square root property. The variable is squared. We isolatethe squared term and take the square root of both sides of the equation. The solution will yield apositive and negative solution. See Example 2.36 and Example 2.37.

• Completing the square is a method of solving quadratic equations when the equation cannot be factored.See Example 2.38.

• A highly dependable method for solving quadratic equations is the quadratic formula, based on thecoe�cients and the constant term in the equation. See Example 2.39.

• The discriminant is used to indicate the nature of the roots that the quadratic equation will yield: realor complex, rational or irrational, and how many of each. See Example 2.40.

• The Pythagorean Theorem, among the most famous theorems in history, is used to solve right-triangleproblems and has applications in numerous �elds. Solving for the length of one side of a right trianglerequires solving a quadratic equation. See Example 2.41.

2.6.9 Section Exercises

2.6.9.1 Verbal

Exercise 2.6.9 (Solution on p. 254.)

How do we recognize when an equation is quadratic?

Exercise 2.6.10When we solve a quadratic equation, how many solutions should we always start out seeking?Explain why when solving a quadratic equation in the form ax2 + bx + c = 0we may graph theequation y = ax2 + bx+ c and have no zeroes (x-intercepts).

Exercise 2.6.11 (Solution on p. 254.)

When we solve a quadratic equation by factoring, why do we move all terms to one side, havingzero on the other side?

Exercise 2.6.12In the quadratic formula, what is the name of the expression under the radical sign b2 − 4ac,andhow does it determine the number of and nature of our solutions?

Exercise 2.6.13 (Solution on p. 254.)

Describe two scenarios where using the square root property to solve a quadratic equation wouldbe the most e�cient method.

2.6.9.2 Algebraic

For the following exercises, solve the quadratic equation by factoring.

Exercise 2.6.14x2 + 4x− 21 = 0Exercise 2.6.15 (Solution on p. 254.)

x2 − 9x+ 18 = 0Exercise 2.6.162x2 + 9x− 5 = 0

Available for free at Connexions <http://legacy.cnx.org/content/col11759/1.2>

202 CHAPTER 2. EQUATIONS AND INEQUALITIES

Exercise 2.6.17 (Solution on p. 254.)

6x2 + 17x+ 5 = 0Exercise 2.6.184x2 − 12x+ 8 = 0Exercise 2.6.19 (Solution on p. 254.)

3x2 − 75 = 0Exercise 2.6.208x2 + 6x− 9 = 0Exercise 2.6.21 (Solution on p. 254.)

4x2 = 9Exercise 2.6.222x2 + 14x = 36Exercise 2.6.23 (Solution on p. 255.)

5x2 = 5x+ 30Exercise 2.6.244x2 = 5xExercise 2.6.25 (Solution on p. 255.)

7x2 + 3x = 0Exercise 2.6.26x3 −

9x = 2

For the following exercises, solve the quadratic equation by using the square root property.

Exercise 2.6.27 (Solution on p. 255.)

x2 = 36Exercise 2.6.28x2 = 49Exercise 2.6.29 (Solution on p. 255.)

(x− 1)2 = 25Exercise 2.6.30(x− 3)2 = 7Exercise 2.6.31 (Solution on p. 255.)

(2x+ 1)2 = 9Exercise 2.6.32(x− 5)2 = 4

For the following exercises, solve the quadratic equation by completing the square. Show each step.

Exercise 2.6.33 (Solution on p. 255.)

x2 − 9x− 22 = 0Exercise 2.6.342x2 − 8x− 5 = 0Exercise 2.6.35 (Solution on p. 255.)

x2 − 6x = 13Exercise 2.6.36x2 + 2

3x−13 = 0

Exercise 2.6.37 (Solution on p. 255.)

2 + z = 6z2

Exercise 2.6.386p2 + 7p− 20 = 0

Available for free at Connexions <http://legacy.cnx.org/content/col11759/1.2>

203

Exercise 2.6.39 (Solution on p. 255.)

2x2 − 3x− 1 = 0For the following exercises, determine the discriminant, and then state how many solutions there are andthe nature of the solutions. Do not solve.

Exercise 2.6.402x2 − 6x+ 7 = 0Exercise 2.6.41 (Solution on p. 255.)

x2 + 4x+ 7 = 0Exercise 2.6.423x2 + 5x− 8 = 0Exercise 2.6.43 (Solution on p. 255.)

9x2 − 30x+ 25 = 0Exercise 2.6.442x2 − 3x− 7 = 0Exercise 2.6.45 (Solution on p. 255.)

6x2 − x− 2 = 0For the following exercises, solve the quadratic equation by using the quadratic formula. If the solutions arenot real, state No Real Solution.

Exercise 2.6.462x2 + 5x+ 3 = 0Exercise 2.6.47 (Solution on p. 255.)

x2 + x = 4Exercise 2.6.482x2 − 8x− 5 = 0Exercise 2.6.49 (Solution on p. 255.)

3x2 − 5x+ 1 = 0Exercise 2.6.50x2 + 4x+ 2 = 0Exercise 2.6.51 (Solution on p. 255.)

4 + 1x −

1x2 = 0

2.6.9.3 Technology

For the following exercises, enter the expressions into your graphing utility and �nd the zeroes to the equation(the x-intercepts) by using 2nd CALC 2:zero. Recall �nding zeroes will ask left bound (move your cursorto the left of the zero,enter), then right bound (move your cursor to the right of the zero,enter), then guess(move your cursor between the bounds near the zero, enter). Round your answers to the nearest thousandth.

Exercise 2.6.52Y1 = 4x2 + 3x− 2Exercise 2.6.53 (Solution on p. 255.)

Y1 = −3x2 + 8x− 1Exercise 2.6.54Y1 = 0.5x2 + x− 7Exercise 2.6.55 (Solution on p. 255.)

To solve the quadratic equationx2 + 5x− 7 = 4,we can graph these two equations

Available for free at Connexions <http://legacy.cnx.org/content/col11759/1.2>

204 CHAPTER 2. EQUATIONS AND INEQUALITIES

Y1 = x2 + 5x− 7

Y2 = 4and �nd the points of intersection. Recall 2nd CALC 5:intersection. Do this and �nd the

solutions to the nearest tenth.

Exercise 2.6.56To solve the quadratic equation 0.3x2 + 2x− 4 = 2,we can graph these two equations

Y1 = 0.3x2 + 2x− 4

Y2 = 2and �nd the points of intersection. Recall 2nd CALC 5:intersection. Do this and �nd the

solutions to the nearest tenth.

2.6.9.4 Extensions

Exercise 2.6.57 (Solution on p. 255.)

Beginning with the general form of a quadratic equation, ax2 + bx+ c = 0,solve for x by using thecompleting the square method , thus deriving the quadratic formula.

Exercise 2.6.58Show that the sum of the two solutions to the quadratic equation is −ba .

Exercise 2.6.59 (Solution on p. 255.)

A person has a garden that has a length 10 feet longer than the width. Set up a quadratic equationto �nd the dimensions of the garden if its area is 119 ft.2. Solve the quadratic equation to �nd thelength and width.

Exercise 2.6.60Abercrombie and Fitch stock had a price given asP = 0.2t2 − 5.6t + 50.2,where t is the time inmonths from 1999 to 2001. ( t = 1 is January 1999). Find the two months in which the price of thestock was $30.

Exercise 2.6.61 (Solution on p. 255.)

Suppose that an equation is given p = −2x2 + 280x− 1000,wherex represents the number of itemssold at an auction and p is the pro�t made by the business that ran the auction. How many itemssold would make this pro�t a maximum? Solve this by graphing the expression in your graphingutility and �nding the maximum using 2nd CALC maximum. To obtain a good window for thecurve, setx [0,200] and y [0,10000].

2.6.9.5 Real-World Applications

Exercise 2.6.62A formula for the normal systolic blood pressure for a man ageA,measured in mmHg, is givenasP = 0.006A2 − 0.02A + 120.Find the age to the nearest year of a man whose normal bloodpressure measures 125 mmHg.

Exercise 2.6.63 (Solution on p. 255.)

The cost function for a certain company isC = 60x+ 300 and the revenue is given byR = 100x−0.5x2.Recall that pro�t is revenue minus cost. Set up a quadratic equation and �nd two values ofx (production level) that will create a pro�t of $300.

Available for free at Connexions <http://legacy.cnx.org/content/col11759/1.2>

205

Exercise 2.6.64A falling object travels a distance given by the formula d = 5t + 16t2 ft., where t is measured inseconds. How long will it take for the object to traveled 74 ft.?

Exercise 2.6.65 (Solution on p. 256.)

A vacant lot is being converted into a community garden. The garden and the walkway around itsperimeter have an area of 378 ft2. Find the width of the walkway if the garden is 12 ft. wide by 15ft. long.

Exercise 2.6.66An epidemiological study of the spread of a certain in�uenza strain that hit a small school populationfound that the total number of students,P,who contracted the �u t days after it broke out is givenby the modelP = −t2 + 13t + 130,where 1 ≤ t ≤ 6.Find the day that 160 students had the �u.Recall that the restriction on t is at most 6.

2.7 Other Types of Equations25

We have solved linear equations, rational equations, and quadratic equations using several methods. However,there are many other types of equations, and we will investigate a few more types in this section. We will lookat equations involving rational exponents, polynomial equations, radical equations, absolute value equations,equations in quadratic form, and some rational equations that can be transformed into quadratics. Solving

25This content is available online at <http://legacy.cnx.org/content/m51258/1.2/>.

Available for free at Connexions <http://legacy.cnx.org/content/col11759/1.2>

206 CHAPTER 2. EQUATIONS AND INEQUALITIES

any equation, however, employs the same basic algebraic rules. We will learn some new techniques as theyapply to certain equations, but the algebra never changes.

2.7.1 Solving Equations Involving Rational Exponents

Rational exponents are exponents that are fractions, where the numerator is a power and the denominator isa root. For example, 16

12 is another way of writing

√16;8

13 is another way of writing 3

√8.The ability to work

with rational exponents is a useful skill, as it is highly applicable in calculus.We can solve equations in which a variable is raised to a rational exponent by raising both sides of the

equation to the reciprocal of the exponent. The reason we raise the equation to the reciprocal of the exponentis because we want to eliminate the exponent on the variable term, and a number multiplied by its reciprocalequals 1. For example, 2

3

(32

)= 1,3

(13

)= 1,and so on.

A General Note: A rational exponent indicates a power in the numerator and a root in thedenominator. There are multiple ways of writing an expression, a variable, or a number with arational exponent:

amn =

(a

1n

)m= (am)

1n = n

√am =

(n√a)m

(2.116)

Example 2.43Evaluating a Number Raised to a Rational ExponentEvaluate 8

23 .

SolutionWhether we take the root �rst or the power �rst depends on the number. It is easy to �nd the

cube root of 8, so rewrite 823 as

(8

13

)2

. (8

13

)2

= (2)2

= 4(2.117)

Try It:

Exercise 2.7.1 (Solution on p. 256.)

Evaluate 64−13 .

Example 2.44Solve the Equation Including a Variable Raised to a Rational ExponentSolve the equation in which a variable is raised to a rational exponent:x

54 = 32.

SolutionThe way to remove the exponent on x is by raising both sides of the equation to a power that isthe reciprocal of 5

4 ,which is 45 .

x54 = 32(

x54

) 45

= (32)45

x = (2)4The �fth root of 32 is 2.

= 16

(2.118)

Available for free at Connexions <http://legacy.cnx.org/content/col11759/1.2>

207

Try It:

Exercise 2.7.2 (Solution on p. 256.)

Solve the equationx32 = 125.

Example 2.45Solving an Equation Involving Rational Exponents and FactoringSolve 3x

34 = x

12 .

SolutionThis equation involves rational exponents as well as factoring rational exponents. Let us take thisone step at a time. First, put the variable terms on one side of the equal sign and set the equationequal to zero.

3x34 −

(x

12

)= x

12 −

(x

12

)3x

34 − x 1

2 = 0(2.119)

Now, it looks like we should factor the left side, but what do we factor out? We can always factorthe term with the lowest exponent. Rewritex

12 asx

24 .Then, factor outx

24 from both terms on the

left.

3x34 − x 2

4 = 0

x24

(3x

14 − 1

)= 0

(2.120)

Where didx14 come from? Remember, when we multiply two numbers with the same base, we

add the exponents. Therefore, if we multiplyx24 back in using the distributive property, we get the

expression we had before the factoring, which is what should happen. We need an exponent suchthat when added to 2

4 equals34 .Thus, the exponent on x in the parentheses is 1

4 .Let us continue. Now we have two factors and can use the zero factor theorem.

x24

(3x

14 − 1

)= 0

x24 = 0

x = 0

3x14 − 1 = 0

3x14 = 1

x14 = 1

3 Divide both sides by 3.(x

14

)4

=(

13

)4Raise both sides to the reciprocal of 1

4 .

x = 181

(2.121)

The two solutions arex = 0,x = 181 .

Available for free at Connexions <http://legacy.cnx.org/content/col11759/1.2>

208 CHAPTER 2. EQUATIONS AND INEQUALITIES

Try It:

Exercise 2.7.3 (Solution on p. 256.)

Solve: (x+ 5)32 = 8.

2.7.2 Solving Equations Using Factoring

We have used factoring to solve quadratic equations, but it is a technique that we can use with many typesof polynomial equations, which are equations that contain a string of terms including numerical coe�cientsand variables. When we are faced with an equation containing polynomials of degree higher than 2, we canoften solve them by factoring.

A General Note: A polynomial of degree n is an expression of the type

anxn + an−1x

n−1 + · · · + a2x2 + a1x+ a0 (2.122)

where n is a positive integer and an, . . . , a0 are real numbers and an 6= 0.

Setting the polynomial equal to zero gives a polynomial equation. The total number of solutions(real and complex) to a polynomial equation is equal to the highest exponent n.

Example 2.46Solving a Polynomial by FactoringSolve the polynomial by factoring: 5x4 = 80x2.

SolutionFirst, set the equation equal to zero. Then factor out what is common to both terms, the GCF.

5x4 − 80x2 = 0

5x2(x2 − 16

)= 0

(2.123)

Notice that we have the di�erence of squares in the factorx2− 16,which we will continue to factorand obtain two solutions. The �rst term, 5x2,generates, technically, two solutions as the exponentis 2, but they are the same solution.

5x2 = 0

x = 0

x2 − 16 = 0

(x− 4) (x+ 4) = 0

x = 4

x = −4

(2.124)

The solutions arex = 0 (double solution),x = 4,andx = −4.

AnalysisWe can see the solutions on the graph in Figure 2.21. The x-coordinates of the points wherethe graph crosses the x-axis are the solutions�the x-intercepts. Notice on the graph that at thesolutionx = 0,the graph touches the x-axis and bounces back. It does not cross the x-axis. This istypical of double solutions.

Available for free at Connexions <http://legacy.cnx.org/content/col11759/1.2>

209

Figure 2.21

Try It:

Exercise 2.7.4 (Solution on p. 256.)

Solve by factoring: 12x4 = 3x2.

Example 2.47Solve a Polynomial by GroupingSolve a polynomial by grouping:x3 + x2 − 9x− 9 = 0.

SolutionThis polynomial consists of 4 terms, which we can solve by grouping. Grouping procedures requirefactoring the �rst two terms and then factoring the last two terms. If the factors in the parentheses

Available for free at Connexions <http://legacy.cnx.org/content/col11759/1.2>

210 CHAPTER 2. EQUATIONS AND INEQUALITIES

are identical, we can continue the process and solve, unless more factoring is suggested.

x3 + x2 − 9x− 9 = 0

x2 (x+ 1)− 9 (x+ 1) = 0(x2 − 9

)(x+ 1) = 0

(2.125)

The grouping process ends here, as we can factorx2 − 9 using the di�erence of squares formula.(x2 − 9

)(x+ 1) = 0

(x− 3) (x+ 3) (x+ 1) = 0

x = 3

x = −3

x = −1

(2.126)

The solutions arex = 3,x = −3,andx = −1.Note that the highest exponent is 3 and we obtained3 solutions. We can see the solutions, the x-intercepts, on the graph in Figure 2.22.

Available for free at Connexions <http://legacy.cnx.org/content/col11759/1.2>

211

Figure 2.22

AnalysisWe looked at solving quadratic equations by factoring when the leading coe�cient is 1. Whenthe leading coe�cient is not 1, we solved by grouping. Grouping requires four terms, which weobtained by splitting the linear term of quadratic equations. We can also use grouping for somepolynomials of degree higher than 2, as we saw here, since there were already four terms.

Available for free at Connexions <http://legacy.cnx.org/content/col11759/1.2>

212 CHAPTER 2. EQUATIONS AND INEQUALITIES

2.7.3 Solving Radical Equations

Radical equations are equations that contain variables in the radicand (the expression under a radicalsymbol), such as

√3x+ 18 = x√x+ 3 = x− 3

√x+ 5−

√x− 3 = 2

(2.127)

Radical equations may have one or more radical terms, and are solved by eliminating each radical, oneat a time. We have to be careful when solving radical equations, as it is not unusual to �nd extraneoussolutions, roots that are not, in fact, solutions to the equation. These solutions are not due to a mistake inthe solving method, but result from the process of raising both sides of an equation to a power. However,checking each answer in the original equation will con�rm the true solutions.

A General Note: An equation containing terms with a variable in the radicand is called aradical equation.

How To: Given a radical equation, solve it.

1.Isolate the radical expression on one side of the equal sign. Put all remaining terms on theother side.

2.If the radical is a square root, then square both sides of the equation. If it is a cube root, thenraise both sides of the equation to the third power. In other words, for an nth root radical,raise both sides to the nth power. Doing so eliminates the radical symbol.

3.Solve the remaining equation.4.If a radical term still remains, repeat steps 1�2.5.Con�rm solutions by substituting them into the original equation.

Example 2.48Solving an Equation with One RadicalSolve

√15− 2x = x.

SolutionThe radical is already isolated on the left side of the equal side, so proceed to square both sides.

√15− 2x = x(√

15− 2x)2

= (x)2

15− 2x = x2

(2.128)

We see that the remaining equation is a quadratic. Set it equal to zero and solve.

0 = x2 + 2x− 15

= (x+ 5) (x− 3)

x = −5

x = 3

(2.129)

Available for free at Connexions <http://legacy.cnx.org/content/col11759/1.2>

213

The proposed solutions arex = −5 andx = 3.Let us check each solution back in the originalequation. First, checkx = −5.

√15− 2x = x√

15− 2 (−5) = −5√

25 = −5

5 6= −5

(2.130)

This is an extraneous solution. While no mistake was made solving the equation, we found asolution that does not satisfy the original equation.

Checkx = 3.

√15− 2x = x√

15− 2 (3) = 3√

9 = 3

3 = 3

(2.131)

The solution isx = 3.

Try It:

Exercise 2.7.5 (Solution on p. 256.)

Solve the radical equation:√x+ 3 = 3x− 1

Example 2.49Solving a Radical Equation Containing Two RadicalsSolve

√2x+ 3 +

√x− 2 = 4.

SolutionAs this equation contains two radicals, we isolate one radical, eliminate it, and then isolate thesecond radical.

√2x+ 3 +

√x− 2 = 4

√2x+ 3 = 4−

√x− 2 Subtract

√x− 2 from both sides.(√

2x+ 3)2

=(4−√x− 2

)2Square both sides.

(2.132)

Use the perfect square formula to expand the right side: (a− b)2 = a2 − 2ab+ b2.

2x+ 3 = (4)2 − 2 (4)√x− 2 +

(√x− 2

)22x+ 3 = 16− 8

√x− 2 + (x− 2)

2x+ 3 = 14 + x− 8√x− 2 Combine like terms.

x− 11 = −8√x− 2 Isolate the second radical.

(x− 11)2 =(−8√x− 2

)2Square both sides.

x2 − 22x+ 121 = 64 (x− 2)

(2.133)

Available for free at Connexions <http://legacy.cnx.org/content/col11759/1.2>

214 CHAPTER 2. EQUATIONS AND INEQUALITIES

Now that both radicals have been eliminated, set the quadratic equal to zero and solve.

x2 − 22x+ 121 = 64x− 128

x2 − 86x+ 249 = 0

(x− 3) (x− 83) = 0 Factor and solve.

x = 3

x = 83

(2.134)

The proposed solutions arex = 3 andx = 83.Check each solution in the original equation.

√2x+ 3 +

√x− 2 = 4

√2x+ 3 = 4−

√x− 2√

2 (3) + 3 = 4−√

(3)− 2√

9 = 4−√

1

3 = 3

(2.135)

One solution isx = 3.Checkx = 83.

√2x+ 3 +

√x− 2 = 4

√2x+ 3 = 4−

√x− 2√

2 (83) + 3 = 4−√

(83− 2)√

169 = 4−√

81

13 6= −5

(2.136)

The only solution isx = 3.We see thatx = 83 is an extraneous solution.

Try It:

Exercise 2.7.6 (Solution on p. 256.)

Solve the equation with two radicals:√

3x+ 7 +√x+ 2 = 1.

2.7.4 Solving an Absolute Value Equation

Next, we will learn how to solve an absolute value equation. To solve an equation such as |2x− 6| = 8,wenotice that the absolute value will be equal to 8 if the quantity inside the absolute value bars is 8 or −8.Thisleads to two di�erent equations we can solve independently.

2x− 6 = 8 or 2x− 6 = −8

2x = 14 2x = −2

x = 7 x = −1

(2.137)

Knowing how to solve problems involving absolute value functions is useful. For example, we may need toidentify numbers or points on a line that are at a speci�ed distance from a given reference point.

Available for free at Connexions <http://legacy.cnx.org/content/col11759/1.2>

215

A General Note: The absolute value of x is written as |x|. It has the following properties:

If x ≥ 0, then |x| = x.

If x < 0, then |x| = −x.(2.138)

For real numbersA andB,an equation of the form |A| = B,withB ≥ 0,will have solutions whenA =B orA = −B. IfB < 0,the equation |A| = B has no solution.

An absolute value equation in the form |ax+ b| = c has the following properties:

If c < 0, |ax+ b| = c has no solution.

If c = 0, |ax+ b| = c has one solution.

If c > 0, |ax+ b| = c has two solutions.

(2.139)

How To: Given an absolute value equation, solve it.

1.Isolate the absolute value expression on one side of the equal sign.2.If c > 0,write and solve two equations: ax+ b = c and ax+ b = −c.

Example 2.50Solving Absolute Value EquationsSolve the following absolute value equations:

(a) |6x+ 4| = 8(b) |3x+ 4| = −9(c) |3x− 5| − 4 = 6(d) | − 5x+ 10| = 0

Solution

(a) |6x+ 4| = 8

Write two equations and solve each:

6x+ 4 = 8 6x+ 4 = −8

6x = 4 6x = −12

x = 23 x = −2

The two solutions arex = 23 ,x = −2.

(b) |3x+ 4| = −9There is no solution as an absolute value cannot be negative.

(c) |3x− 5| − 4 = 6Isolate the absolute value expression and then write two equations.

|3x− 5| − 4 = 6

|3x− 5| = 10

3x− 5 = 10 3x− 5 = −10

3x = 15 3x = −5

x = 5 x = − 53

(2.140)

There are two solutions:x = 5,x = − 53 .

Available for free at Connexions <http://legacy.cnx.org/content/col11759/1.2>

216 CHAPTER 2. EQUATIONS AND INEQUALITIES

(d) | − 5x+ 10| = 0The equation is set equal to zero, so we have to write only one equation.

−5x+ 10 = 0

− 5x = −10

x = 2

(2.141)

There is one solution:x = 2.

Try It:

Exercise 2.7.7 (Solution on p. 256.)

Solve the absolute value equation: |1− 4x|+ 8 = 13.

2.7.5 Solving Other Types of Equations

There are many other types of equations in addition to the ones we have discussed so far. We will see more ofthem throughout the text. Here, we will discuss equations that are in quadratic form, and rational equationsthat result in a quadratic.

2.7.5.1 Solving Equations in Quadratic Form

Equations in quadratic form are equations with three terms. The �rst term has a power other than 2.The middle term has an exponent that is one-half the exponent of the leading term. The third term is aconstant. We can solve equations in this form as if they were quadratic. A few examples of these equationsincludex4 − 5x2 + 4 = 0, x6 + 7x3 − 8 = 0,andx

23 + 4x

13 + 2 = 0. In each one, doubling the exponent of

the middle term equals the exponent on the leading term. We can solve these equations by substituting avariable for the middle term.

A General Note: If the exponent on the middle term is one-half of the exponent on the leadingterm, we have an equation in quadratic form, which we can solve as if it were a quadratic. Wesubstitute a variable for the middle term to solve equations in quadratic form.

How To: Given an equation quadratic in form, solve it.

1.Identify the exponent on the leading term and determine whether it is double the exponenton the middle term.

2.If it is, substitute a variable, such as u, for the variable portion of the middle term.3.Rewrite the equation so that it takes on the standard form of a quadratic.4.Solve using one of the usual methods for solving a quadratic.5.Replace the substitution variable with the original term.6.Solve the remaining equation.

Example 2.51Solving a Fourth-degree Equation in Quadratic FormSolve this fourth-degree equation: 3x4 − 2x2 − 1 = 0.

Available for free at Connexions <http://legacy.cnx.org/content/col11759/1.2>

217

SolutionThis equation �ts the main criteria, that the power on the leading term is double the power onthe middle term. Next, we will make a substitution for the variable term in the middle. Letu =x2.Rewrite the equation in u.

3u2 − 2u− 1 = 0 (2.142)

Now solve the quadratic.

3u2 − 2u− 1 = 0

(3u+ 1) (u− 1) = 0(2.143)

Solve each factor and replace the original term for u.

3u+ 1 = 0

3u = −1

u = − 13

x2 = − 13

x = ±i√

13

(2.144)

u− 1 = 0

u = 1

x2 = 1

x = ±1

(2.145)

The solutions arex = ±i√

13 andx = ±1.

Try It:

Exercise 2.7.8 (Solution on p. 256.)

Solve using substitution:x4 − 8x2 − 9 = 0.

Example 2.52Solving an Equation in Quadratic Form Containing a BinomialSolve the equation in quadratic form: (x+ 2)2 + 11 (x+ 2)− 12 = 0.

SolutionThis equation contains a binomial in place of the single variable. The tendency is to expand whatis presented. However, recognizing that it �ts the criteria for being in quadratic form makes all thedi�erence in the solving process. First, make a substitution, lettingu = x + 2.Then rewrite theequation in u.

u2 + 11u− 12 = 0

(u+ 12) (u− 1) = 0(2.146)

Available for free at Connexions <http://legacy.cnx.org/content/col11759/1.2>

218 CHAPTER 2. EQUATIONS AND INEQUALITIES

Solve using the zero-factor property and then replace u with the original expression.

u+ 12 = 0

u = −12

x+ 2 = −12

x = −14

(2.147)

The second factor results in

u− 1 = 0

u = 1

x+ 2 = 1

x = −1

(2.148)

We have two solutions:x = −14,x = −1.

Try It:

Exercise 2.7.9 (Solution on p. 256.)

Solve: (x− 5)2 − 4 (x− 5)− 21 = 0.

2.7.5.2 Solving Rational Equations Resulting in a Quadratic

Earlier, we solved rational equations. Sometimes, solving a rational equation results in a quadratic. Whenthis happens, we continue the solution by simplifying the quadratic equation by one of the methods we haveseen. It may turn out that there is no solution.

Example 2.53Solving a Rational Equation Leading to a QuadraticSolve the following rational equation: −4x

x−1 + 4x+1 = −8

x2−1 .

SolutionWe want all denominators in factored form to �nd the LCD. Two of the denominators cannotbe factored further. However,x2 − 1 = (x+ 1) (x− 1) .Then, the LCD is (x+ 1) (x− 1) .Next, wemultiply the whole equation by the LCD.

(x+ 1) (x− 1)[−4xx−1 + 4

x+1

]=[

−8(x+1)(x−1)

](x+ 1) (x− 1)

− 4x (x+ 1) + 4 (x− 1) = −8

− 4x2 − 4x+ 4x− 4 = −8

− 4x2 + 4 = 0

− 4(x2 − 1

)= 0

− 4 (x+ 1) (x− 1) = 0

x = −1

x = 1

(2.149)

Available for free at Connexions <http://legacy.cnx.org/content/col11759/1.2>

219

In this case, either solution produces a zero in the denominator in the original equation. Thus,there is no solution.

Try It:

Exercise 2.7.10 (Solution on p. 256.)

Solve 3x+2x−2 + 1

x = −2x2−2x .

Media: Access these online resources for additional instruction and practice with di�erent typesof equations.

• Rational Equation with no Solution26

• Solving equations with rational exponents using reciprocal powers27

• Solving radical equations part 1 of 228• Solving radical equations part 2 of 229

2.7.6 Key Concepts

• Rational exponents can be rewritten several ways depending on what is most convenient for the prob-lem. To solve, both sides of the equation are raised to a power that will render the exponent on thevariable equal to 1. See Example 2.43, Example 2.44, and Example 2.45.

• Factoring extends to higher-order polynomials when it involves factoring out the GCF or factoring bygrouping. See Example 2.46 and Example 2.47.

• We can solve radical equations by isolating the radical and raising both sides of the equation to a powerthat matches the index. See Example 2.48 and Example 2.49.

• To solve absolute value equations, we need to write two equations, one for the positive value and onefor the negative value. See Example 2.50.

• Equations in quadratic form are easy to spot, as the exponent on the �rst term is double the exponenton the second term and the third term is a constant. We may also see a binomial in place of the singlevariable. We use substitution to solve. See Example 2.51 and Example 2.52.

• Solving a rational equation may also lead to a quadratic equation or an equation in quadratic form.See Example 2.53.

2.7.7 Section Exercises

2.7.7.1 Verbal

Exercise 2.7.11 (Solution on p. 256.)

In a radical equation, what does it mean if a number is an extraneous solution?

Exercise 2.7.12Explain why possible solutions must be checked in radical equations.

26http://openstaxcollege.org/l/rateqnosoln27http://openstaxcollege.org/l/ratexprecpexp28http://openstaxcollege.org/l/radeqsolvepart129http://openstaxcollege.org/l/radeqsolvepart2

Available for free at Connexions <http://legacy.cnx.org/content/col11759/1.2>

220 CHAPTER 2. EQUATIONS AND INEQUALITIES

Exercise 2.7.13 (Solution on p. 256.)

Your friend tries to calculate the value − 932 and keeps getting an ERROR message. What mistake

is he or she probably making?

Exercise 2.7.14Explain why |2x+ 5| = −7 has no solutions.Exercise 2.7.15 (Solution on p. 256.)

Explain how to change a rational exponent into the correct radical expression.

2.7.7.2 Algebraic

For the following exercises, solve the rational exponent equation. Use factoring where necessary.

Exercise 2.7.16x

23 = 16

Exercise 2.7.17 (Solution on p. 256.)

x34 = 27

Exercise 2.7.182x

12 − x 1

4 = 0Exercise 2.7.19 (Solution on p. 256.)

(x− 1)34 = 8

Exercise 2.7.20(x+ 1)

23 = 4

Exercise 2.7.21 (Solution on p. 256.)

x23 − 5x

13 + 6 = 0

Exercise 2.7.22x

73 − 3x

43 − 4x

13 = 0

For the following exercises, solve the following polynomial equations by grouping and factoring.

Exercise 2.7.23 (Solution on p. 256.)

x3 + 2x2 − x− 2 = 0Exercise 2.7.243x3 − 6x2 − 27x+ 54 = 0Exercise 2.7.25 (Solution on p. 256.)

4y3 − 9y = 0Exercise 2.7.26x3 + 3x2 − 25x− 75 = 0Exercise 2.7.27 (Solution on p. 256.)

m3 +m2 −m− 1 = 0Exercise 2.7.282x5 − 14x3 = 0Exercise 2.7.29 (Solution on p. 256.)

5x3 + 45x = 2x2 + 18For the following exercises, solve the radical equation. Be sure to check all solutions to eliminate extraneoussolutions.

Exercise 2.7.30√3x− 1− 2 = 0

Available for free at Connexions <http://legacy.cnx.org/content/col11759/1.2>

221

Exercise 2.7.31 (Solution on p. 256.)√x− 7 = 5

Exercise 2.7.32√x− 1 = x− 7

Exercise 2.7.33 (Solution on p. 256.)√3t+ 5 = 7

Exercise 2.7.34√t+ 1 + 9 = 7

Exercise 2.7.35 (Solution on p. 257.)√12− x = x

Exercise 2.7.36√2x+ 3−

√x+ 2 = 2

Exercise 2.7.37 (Solution on p. 257.)√3x+ 7 +

√x+ 2 = 1

Exercise 2.7.38√2x+ 3−

√x+ 1 = 1

For the following exercises, solve the equation involving absolute value.

Exercise 2.7.39 (Solution on p. 257.)

|3x− 4| = 8Exercise 2.7.40|2x− 3| = −2Exercise 2.7.41 (Solution on p. 257.)

|1− 4x| − 1 = 5Exercise 2.7.42|4x+ 1| − 3 = 6Exercise 2.7.43 (Solution on p. 257.)

|2x− 1| − 7 = −2Exercise 2.7.44|2x+ 1| − 2 = −3Exercise 2.7.45 (Solution on p. 257.)

|x+ 5| = 0Exercise 2.7.46−|2x+ 1| = −3

For the following exercises, solve the equation by identifying the quadratic form. Use a substitute variableand �nd all real solutions by factoring.

Exercise 2.7.47 (Solution on p. 257.)

x4 − 10x2 + 9 = 0Exercise 2.7.484(t− 1)2 − 9 (t− 1) = −2Exercise 2.7.49 (Solution on p. 257.)(x2 − 1

)2 +(x2 − 1

)− 12 = 0

Exercise 2.7.50(x+ 1)2 − 8 (x+ 1)− 9 = 0Exercise 2.7.51 (Solution on p. 257.)

(x− 3)2 − 4 = 0

Available for free at Connexions <http://legacy.cnx.org/content/col11759/1.2>

222 CHAPTER 2. EQUATIONS AND INEQUALITIES

2.7.7.3 Extensions

For the following exercises, solve for the unknown variable.

Exercise 2.7.52x−2 − x−1 − 12 = 0Exercise 2.7.53 (Solution on p. 257.)√|x|2 = x

Exercise 2.7.54t25 − t5 + 1 = 0Exercise 2.7.55 (Solution on p. 257.)

|x2 + 2x− 36| = 12

2.7.7.4 Real-World Applications

For the following exercises, use the model for the period of a pendulum,T,such thatT = 2π√

Lg ,where the

length of the pendulum is L and the acceleration due to gravity is g.

Exercise 2.7.56If the acceleration due to gravity is 9.8m/s2and the period equals 1 s, �nd the length to the nearestcm (100 cm = 1 m).

Exercise 2.7.57 (Solution on p. 257.)

If the gravity is 32 fts2 and the period equals 1 s, �nd the length to the nearest in. (12 in. = 1 ft).

Round your answer to the nearest in.

For the following exercises, use a model for body surface area, BSA, such thatBSA =√

wh3600 ,where w =

weight in kg and h = height in cm.

Exercise 2.7.58Find the height of a 72-kg female to the nearest cm whoseBSA = 1.8.Exercise 2.7.59 (Solution on p. 257.)

Find the weight of a 177-cm male to the nearest kg whoseBSA = 2.1.

Available for free at Connexions <http://legacy.cnx.org/content/col11759/1.2>

223

2.8 Linear Inequalities and Absolute Value Inequalities30

Figure 2.23

It is not easy to make the honor role at most top universities. Suppose students were required to carrya course load of at least 12 credit hours and maintain a grade point average of 3.5 or above. How couldthese honor roll requirements be expressed mathematically? In this section, we will explore various ways toexpress di�erent sets of numbers, inequalities, and absolute value inequalities.

2.8.1 Using Interval Notation

Indicating the solution to an inequality such asx ≥ 4 can be achieved in several ways.We can use a number line as shown in Figure 2.24. The blue ray begins atx = 4 and, as indicated by

the arrowhead, continues to in�nity, which illustrates that the solution set includes all real numbers greaterthan or equal to 4.

30This content is available online at <http://legacy.cnx.org/content/m51259/1.2/>.

Available for free at Connexions <http://legacy.cnx.org/content/col11759/1.2>

224 CHAPTER 2. EQUATIONS AND INEQUALITIES

Figure 2.24

We can use set-builder notation: {x|x ≥ 4},which translates to �all real numbers x such that x isgreater than or equal to 4.� Notice that braces are used to indicate a set.

The third method is interval notation, in which solution sets are indicated with parentheses or brackets.The solutions tox ≥ 4 are represented as [4,∞) .This is perhaps the most useful method, as it applies toconcepts studied later in this course and to other higher-level math courses.

The main concept to remember is that parentheses represent solutions greater or less than the number,and brackets represent solutions that are greater than or equal to or less than or equal to the number. Useparentheses to represent in�nity or negative in�nity, since positive and negative in�nity are not numbersin the usual sense of the word and, therefore, cannot be �equaled.� A few examples of an interval, or aset of numbers in which a solution falls, are [−2, 6) ,or all numbers between − 2 and 6,including − 2,but notincluding 6;(−1, 0) ,all real numbers between, but not including − 1 and 0;and (−∞, 1] ,all real numbers lessthan and including 1.Table 2.6 outlines the possibilities.

Set Indicated Set-Builder Notation Interval Notation

All real numbers between a andb, but not including a or b

{x|a < x < b} (a, b)

All real numbers greater than a,but not including a

{x|x > a} (a,∞)

All real numbers less than b, butnot including b

{x|x < b} (−∞, b)

All real numbers greater than a,including a

{x|x ≥ a} [a,∞)

All real numbers less than b, in-cluding b

{x|x ≤ b} (−∞, b]

All real numbers between a andb, including a

{x|a ≤ x < b} [a, b)

continued on next page

Available for free at Connexions <http://legacy.cnx.org/content/col11759/1.2>

225

All real numbers between a andb, including b

{x|a < x ≤ b} (a, b]

All real numbers between a andb, including a and b

{x|a ≤ x ≤ b} [a, b]

All real numbers less than a orgreater than b

{x|x < a andx > b} (−∞, a) ∪ (b,∞)

All real numbers {x|x is all real numbers} (−∞,∞)

Table 2.6

Example 2.54Using Interval Notation to Express All Real Numbers Greater Than or Equal to aUse interval notation to indicate all real numbers greater than or equal to − 2.

SolutionUse a bracket on the left of − 2 and parentheses after in�nity: [−2,∞) .The bracket indicatesthat − 2 is included in the set with all real numbers greater than − 2 to in�nity.

Try It:

Exercise 2.8.1 (Solution on p. 257.)

Use interval notation to indicate all real numbers between and including − 3 and 5.

Example 2.55Using Interval Notation to Express All Real Numbers Less Than or Equal to a or

Greater Than or Equal to bWrite the interval expressing all real numbers less than or equal to − 1 or greater than or equalto 1.

SolutionWe have to write two intervals for this example. The �rst interval must indicate all real numbersless than or equal to 1. So, this interval begins at−∞ and ends at−1,which is written as (−∞,−1] .

The second interval must show all real numbers greater than or equal to 1,which is writtenas [1,∞) .However, we want to combine these two sets. We accomplish this by inserting the unionsymbol,∪,between the two intervals.

(−∞,−1] ∪ [1,∞) (2.150)

Try It:

Exercise 2.8.2 (Solution on p. 257.)

Express all real numbers less than − 2 or greater than or equal to 3 in interval notation.

Available for free at Connexions <http://legacy.cnx.org/content/col11759/1.2>

226 CHAPTER 2. EQUATIONS AND INEQUALITIES

2.8.2 Using the Properties of Inequalities

When we work with inequalities, we can usually treat them similarly to but not exactly as we treat equalities.We can use the addition property and the multiplication property to help us solve them. The oneexception is when we multiply or divide by a negative number; doing so reverses the inequality symbol.

A General Note:

Addition Property If a < b, then a+ c < b+ c.

Multiplication Property If a < b and c > 0, then ac < bc.

If a < b and c < 0, then ac > bc.

(2.151)

These properties also apply to a ≤ b,a > b,and a ≥ b.

Example 2.56Demonstrating the Addition PropertyIllustrate the addition property for inequalities by solving each of the following:

(a) x− 15 < 4(b) 6 ≥ x− 1(c) x+ 7 > 9

SolutionThe addition property for inequalities states that if an inequality exists, adding or subtracting thesame number on both sides does not change the inequality.

a.

x− 15 < 4

x− 15 + 15 < 4 + 15 Add 15 to both sides.

x < 19

b.

6 ≥ x− 1

6 + 1 ≥ x− 1 + 1 Add 1 to both sides.

7 ≥ x

c.

x+ 7 > 9

x+ 7− 7 > 9− 7 Subtract 7 from both sides.

x > 2

Try It:

Exercise 2.8.3 (Solution on p. 257.)

Solve: 3x− 2 < 1.

Example 2.57Demonstrating the Multiplication PropertyIllustrate the multiplication property for inequalities by solving each of the following:

Available for free at Connexions <http://legacy.cnx.org/content/col11759/1.2>

227

a. 3x < 6b. −2x− 1 ≥ 5c. 5− x > 10

Solution

a.

3x < 613 (3x) < (6) 1

3

x < 2

b.

− 2x− 1 ≥ 5

− 2x ≥ 6(− 1

2

)(−2x) ≥ (6)

(− 1

2

)Multiply by − 1

2 .

x ≤ −3 Reverse the inequality.

c.

5− x > 10

− x > 5

(−1) (−x) > (5) (−1) Multiply by − 1.

x < − 5 Reverse the inequality.

Try It:

Exercise 2.8.4 (Solution on p. 257.)

Solve: 4x+ 7 ≥ 2x− 3.

2.8.3 Solving Inequalities in One Variable Algebraically

As the examples have shown, we can perform the same operations on both sides of an inequality, just as wedo with equations; we combine like terms and perform operations. To solve, we isolate the variable.

Example 2.58Solving an Inequality AlgebraicallySolve the inequality: 13− 7x ≥ 10x− 4.

SolutionSolving this inequality is similar to solving an equation up until the last step.

13− 7x ≥ 10x− 4

13− 17x ≥ −4 Move variable terms to one side of the inequality.

− 17x ≥ −17 Isolate the variable term.

x ≤ 1 Dividing both sides by − 17 reverses the inequality.

(2.152)

The solution set is given by the interval (−∞, 1] ,or all real numbers less than and including 1.

Available for free at Connexions <http://legacy.cnx.org/content/col11759/1.2>

228 CHAPTER 2. EQUATIONS AND INEQUALITIES

Try It:

Exercise 2.8.5 (Solution on p. 257.)

Solve the inequality and write the answer using interval notation: − x+ 4 < 12x+ 1.

Example 2.59Solving an Inequality with FractionsSolve the following inequality and write the answer in interval notation: − 3

4x ≥ −58 + 2

3x.

SolutionWe begin solving in the same way we do when solving an equation.

− 34x ≥ −

58 + 2

3x

− 34x−

23x ≥ −

58 Put variable terms on one side.

− 912x−

812x ≥ −

58 Write fractions with common denominator.

− 1712x ≥ −

58

x ≤ − 58

(− 12

17

)Multiplying by a negative number reverses the inequality.

x ≤ 1534

(2.153)

The solution set is the interval(−∞, 15

34

].

Try It:

Exercise 2.8.6 (Solution on p. 257.)

Solve the inequality and write the answer in interval notation: − 56x ≤

34 + 8

3x.

2.8.4 Understanding Compound Inequalities

A compound inequality includes two inequalities in one statement. A statement such as 4 < x ≤6means 4 < x andx ≤ 6.There are two ways to solve compound inequalities: separating them into twoseparate inequalities or leaving the compound inequality intact and performing operations on all three partsat the same time. We will illustrate both methods.

Example 2.60Solving a Compound InequalitySolve the compound inequality: 3 ≤ 2x+ 2 < 6.

SolutionThe �rst method is to write two separate inequalities: 3 ≤ 2x + 2 and 2x + 2 < 6.We solve themindependently.

3 ≤ 2x+ 2 and 2x+ 2 < 6

1 ≤ 2x 2x < 412 ≤ x x < 2

(2.154)

Then, we can rewrite the solution as a compound inequality, the same way the problem began.

12≤ x < 2 (2.155)

Available for free at Connexions <http://legacy.cnx.org/content/col11759/1.2>

229

In interval notation, the solution is written as[

12 , 2).

The second method is to leave the compound inequality intact, and perform solving procedureson the three parts at the same time.

3 ≤ 2x+ 2 < 6

1 ≤ 2x < 4 Isolate the variable term, and subtract 2 from all three parts.

12 ≤ x < 2 Divide through all three parts by 2.

(2.156)

We get the same solution:[

12 , 2).

Try It:

Exercise 2.8.7 (Solution on p. 257.)

Solve the compound inequality: 4 < 2x− 8 ≤ 10.

Example 2.61Solving a Compound Inequality with the Variable in All Three PartsSolve the compound inequality with variables in all three parts: 3 + x > 7x− 2 > 5x− 10.

SolutionLets try the �rst method. Write two inequalities:

3 + x > 7x− 2 and 7x− 2 > 5x− 10

3 > 6x− 2 2x− 2 > − 10

5 > 6x 2x > − 856 > x x > − 4

x < 56 − 4 < x

(2.157)

The solution set is − 4 < x < 56 or in interval notation

(−4, 5

6

).Notice that when we write the

solution in interval notation, the smaller number comes �rst. We read intervals from left to right,as they appear on a number line. See Figure 2.25.

Figure 2.25

Available for free at Connexions <http://legacy.cnx.org/content/col11759/1.2>

230 CHAPTER 2. EQUATIONS AND INEQUALITIES

Try It:

Exercise 2.8.8 (Solution on p. 257.)

Solve the compound inequality: 3y < 4− 5y < 5 + 3y.

2.8.5 Solving Absolute Value Inequalities

As we know, the absolute value of a quantity is a positive number or zero. From the origin, a point locatedat (−x, 0) has an absolute value ofx,as it is x units away. Consider absolute value as the distance from onepoint to another point. Regardless of direction, positive or negative, the distance between the two points isrepresented as a positive number or zero.

An absolute value inequality is an equation of the form

|A| < B, |A| ≤ B, |A| > B, or |A| ≥ B, (2.158)

Where A, and sometimes B, represents an algebraic expression dependent on a variable x. Solving theinequality means �nding the set of all x-values that satisfy the problem. Usually this set will be an intervalor the union of two intervals and will include a range of values.

There are two basic approaches to solving absolute value inequalities: graphical and algebraic. Theadvantage of the graphical approach is we can read the solution by interpreting the graphs of two equations.The advantage of the algebraic approach is that solutions are exact, as precise solutions are sometimesdi�cult to read from a graph.

Suppose we want to know all possible returns on an investment if we could earn some amount of moneywithin $200 of $600. We can solve algebraically for the set of x-values such that the distance betweenx and600 is less than 200. We represent the distance betweenx and 600 as |x−600|,and therefore, |x−600| ≤ 200 or

−200 ≤ x− 600 ≤ 200

−200 + 600 ≤ x− 600 + 600 ≤ 200 + 600

400 ≤ x ≤ 800

(2.159)

This means our returns would be between $400 and $800.To solve absolute value inequalities, just as with absolute value equations, we write two inequalities and

then solve them independently.

A General Note: For an algebraic expression X, and k > 0,an absolute value inequality isan inequality of the form

|X| < k is equivalent to − k < X < k

|X| > k is equivalent to X < − k or X > k(2.160)

These statements also apply to |X| ≤ k and |X| ≥ k.

Example 2.62Determining a Number within a Prescribed DistanceDescribe all valuesxwithin a distance of 4 from the number 5.

SolutionWe want the distance betweenx and 5 to be less than or equal to 4. We can draw a number line,such as in Figure 2.26, to represent the condition to be satis�ed.

Available for free at Connexions <http://legacy.cnx.org/content/col11759/1.2>

231

Figure 2.26

The distance fromx to 5 can be represented using an absolute value symbol, |x − 5|.Write thevalues ofx that satisfy the condition as an absolute value inequality.

|x− 5| ≤ 4 (2.161)

We need to write two inequalities as there are always two solutions to an absolute value equation.

x− 5 ≤ 4 and x− 5 ≥ −4

x ≤ 9 x ≥ 1(2.162)

If the solution set isx ≤ 9 andx ≥ 1,then the solution set is an interval including all real numbersbetween and including 1 and 9.

So |x− 5| ≤ 4 is equivalent to [1, 9] in interval notation.

Try It:

Exercise 2.8.9 (Solution on p. 257.)

Describe all x-values within a distance of 3 from the number 2.

Example 2.63Solving an Absolute Value InequalitySolve |x− 1| ≤ 3.

Solution

|x− 1| ≤ 3

−3 ≤ x− 1 ≤ 3

−2 ≤ x ≤ 4

[−2, 4]

(2.163)

Available for free at Connexions <http://legacy.cnx.org/content/col11759/1.2>

232 CHAPTER 2. EQUATIONS AND INEQUALITIES

Example 2.64Using a Graphical Approach to Solve Absolute Value InequalitiesGiven the equation y = − 1

2 |4x− 5|+ 3,determine the x-values for which the y-values are negative.

SolutionWe are trying to determine where y < 0,which is when − 1

2 |4x− 5|+ 3 < 0.We begin by isolatingthe absolute value.

− 12 |4x− 5| < − 3 Multiply both sides by �2, and reverse the inequality.

|4x− 5| > 6(2.164)

Next, we solve for the equality |4x− 5| = 6.

4x− 5 = 6 4x− 5 = −6

4x = 11 or 4x = −1

x = 114 x = − 1

4

(2.165)

Now, we can examine the graph to observe where the y-values are negative. We observe where thebranches are below the x-axis. Notice that it is not important exactly what the graph looks like, aslong as we know that it crosses the horizontal axis atx = − 1

4 andx = 114 ,and that the graph opens

downward. See Figure 2.27.

Available for free at Connexions <http://legacy.cnx.org/content/col11759/1.2>

233

Figure 2.27

Try It:

Exercise 2.8.10 (Solution on p. 257.)

Solve − 2|k − 4| ≤ −6.

Media: Access these online resources for additional instruction and practice with linear inequali-ties and absolute value inequalities.

• Interval notation31• How to solve linear inequalities32

• How to solve an inequality33

• Absolute value equations3431http://openstaxcollege.org/l/intervalnotn32http://openstaxcollege.org/l/solvelinineq33http://openstaxcollege.org/l/solveineq34http://openstaxcollege.org/l/absvaleq

Available for free at Connexions <http://legacy.cnx.org/content/col11759/1.2>

234 CHAPTER 2. EQUATIONS AND INEQUALITIES

• Compound inequalities35

• Absolute value inequalities36

2.8.6 Key Concepts

• Interval notation is a method to indicate the solution set to an inequality. Highly applicable in calculus,it is a system of parentheses and brackets that indicate what numbers are included in a set and whetherthe endpoints are included as well. See Table 2.6 and Example 2.55.

• Solving inequalities is similar to solving equations. The same algebraic rules apply, except for one:multiplying or dividing by a negative number reverses the inequality. See Example 2.56, Example 2.57,Example 2.58, and Example 2.59.

• Compound inequalities often have three parts and can be rewritten as two independent inequalities.Solutions are given by boundary values, which are indicated as a beginning boundary or an endingboundary in the solutions to the two inequalities. See Example 2.60 and Example 2.61.

• Absolute value inequalities will produce two solution sets due to the nature of absolute value. Wesolve by writing two equations: one equal to a positive value and one equal to a negative value. SeeExample 2.62 and Example 2.63.

• Absolute value inequalities can also be solved by graphing. At least we can check the algebraic solutionsby graphing, as we cannot depend on a visual for a precise solution. See Example 2.64.

2.8.7 Section Exercises

2.8.7.1 Verbal

Exercise 2.8.11 (Solution on p. 258.)

When solving an inequality, explain what happened from Step 1 to Step 2:

Step 1 − 2x > 6

Step 2 x < − 3

Exercise 2.8.12When solving an inequality, we arrive at:

x+ 2 < x+ 3

2 < 3Explain what our solution set is.

Exercise 2.8.13 (Solution on p. 258.)

When writing our solution in interval notation, how do we represent all the real numbers?

Exercise 2.8.14When solving an inequality, we arrive at:

x+ 2 > x+ 3

2 > 3Explain what our solution set is.

Exercise 2.8.15 (Solution on p. 258.)

Describe how to graph y = |x− 3|

35http://openstaxcollege.org/l/compndineqs36http://openstaxcollege.org/l/absvalineqs

Available for free at Connexions <http://legacy.cnx.org/content/col11759/1.2>

235

2.8.7.2 Algebraic

For the following exercises, solve the inequality. Write your �nal answer in interval notation.

Exercise 2.8.164x− 7 ≤ 9Exercise 2.8.17 (Solution on p. 258.)

3x+ 2 ≥ 7x− 1Exercise 2.8.18−2x+ 3 > x− 5Exercise 2.8.19 (Solution on p. 258.)

4 (x+ 3) ≥ 2x− 1Exercise 2.8.20− 1

2x ≤−54 + 2

5x

Exercise 2.8.21 (Solution on p. 258.)

−5 (x− 1) + 3 > 3x− 4− 4xExercise 2.8.22−3 (2x+ 1) > − 2 (x+ 4)Exercise 2.8.23 (Solution on p. 258.)x+3

8 −x+5

5 ≥ 310

Exercise 2.8.24x−1

3 + x+25 ≤ 3

5

For the following exercises, solve the inequality involving absolute value. Write your �nal answer in intervalnotation.

Exercise 2.8.25 (Solution on p. 259.)

|x+ 9| ≥ −6Exercise 2.8.26|2x+ 3| < 7Exercise 2.8.27 (Solution on p. 259.)

|3x− 1| > 11Exercise 2.8.28|2x+ 1|+ 1 ≤ 6Exercise 2.8.29 (Solution on p. 259.)

|x− 2|+ 4 ≥ 10Exercise 2.8.30| − 2x+ 7| ≤ 13Exercise 2.8.31 (Solution on p. 259.)

|x− 7| < − 4Exercise 2.8.32|x− 20| > − 1Exercise 2.8.33 (Solution on p. 259.)

|x−34 | < 2

For the following exercises, describe all the x-values within or including a distance of the given values.

Exercise 2.8.34Distance of 5 units from the number 7

Exercise 2.8.35 (Solution on p. 259.)

Distance of 3 units from the number 9

Available for free at Connexions <http://legacy.cnx.org/content/col11759/1.2>

236 CHAPTER 2. EQUATIONS AND INEQUALITIES

Exercise 2.8.36Distance of10 units from the number 4

Exercise 2.8.37 (Solution on p. 259.)

Distance of 11 units from the number 1

For the following exercises, solve the compound inequality. Express your answer using inequality signs, andthen write your answer using interval notation.

Exercise 2.8.38−4 < 3x+ 2 ≤ 18Exercise 2.8.39 (Solution on p. 259.)

3x+ 1 > 2x− 5 > x− 7Exercise 2.8.403y < 5− 2y < 7 + y

Exercise 2.8.41 (Solution on p. 259.)

2x− 5 < − 11 or 5x+ 1 ≥ 6Exercise 2.8.42x+ 7 < x+ 2

2.8.7.3 Graphical

For the following exercises, graph the function. Observe the points of intersection and shade the x-axisrepresenting the solution set to the inequality. Show your graph and write your �nal answer in intervalnotation.

Exercise 2.8.43 (Solution on p. 259.)

|x− 1| > 2Exercise 2.8.44|x+ 3| ≥ 5Exercise 2.8.45 (Solution on p. 260.)

|x+ 7| ≤ 4Exercise 2.8.46|x− 2| < 7Exercise 2.8.47 (Solution on p. 260.)

|x− 2| < 0For the following exercises, graph both straight lines (left-hand side being y1 and right-hand side being y2)on the same axes. Find the point of intersection and solve the inequality by observing where it is truecomparing the y-values of the lines.

Exercise 2.8.48x+ 3 < 3x− 4Exercise 2.8.49 (Solution on p. 261.)

x− 2 > 2x+ 1Exercise 2.8.50x+ 1 > x+ 4Exercise 2.8.51 (Solution on p. 262.)12x+ 1 > 1

2x− 5Exercise 2.8.524x+ 1 < 1

2x+ 3

Available for free at Connexions <http://legacy.cnx.org/content/col11759/1.2>

237

2.8.7.4 Numeric

For the following exercises, write the set in interval notation.

Exercise 2.8.53 (Solution on p. 263.)

{x| − 1 < x < 3}Exercise 2.8.54{x|x ≥ 7}Exercise 2.8.55 (Solution on p. 263.)

{x|x < 4}Exercise 2.8.56{x|x is all real numbers}

For the following exercises, write the interval in set-builder notation.

Exercise 2.8.57 (Solution on p. 263.)

(−∞, 6)Exercise 2.8.58(4,+∞)Exercise 2.8.59 (Solution on p. 263.)

[−3, 5)Exercise 2.8.60[−4, 1] ∪ [9,∞)

For the following exercises, write the set of numbers represented on the number line in interval notation.Exercise 2.8.61 (Solution on p. 263.)

Exercise 2.8.62

Exercise 2.8.63 (Solution on p. 263.)

2.8.7.5 Technology

For the following exercises, input the left-hand side of the inequality as a Y1 graph in your graphing utility.Enter y2 = the right-hand side. Entering the absolute value of an expression is found in the MATH menu,Num, 1:abs(. Find the points of intersection, recall (2nd CALC 5:intersection, 1st curve, enter, 2nd curve,enter, guess, enter). Copy a sketch of the graph and shade the x-axis for your solution set to the inequality.Write �nal answers in interval notation.

Available for free at Connexions <http://legacy.cnx.org/content/col11759/1.2>

238 CHAPTER 2. EQUATIONS AND INEQUALITIES

Exercise 2.8.64|x+ 2| − 5 < 2Exercise 2.8.65 (Solution on p. 263.)−12 |x+ 2| < 4Exercise 2.8.66|4x+ 1| − 3 > 2Exercise 2.8.67 (Solution on p. 264.)

|x− 4| < 3Exercise 2.8.68|x+ 2| ≥ 5

2.8.7.6 Extensions

Exercise 2.8.69 (Solution on p. 265.)

Solve |3x+ 1| = |2x+ 3|Exercise 2.8.70Solvex2 − x > 12Exercise 2.8.71 (Solution on p. 265.)x−5x+7 ≤ 0,x 6= −7Exercise 2.8.72p = −x2 + 130x − 3000 is a pro�t formula for a small business. Find the set of x-values that willkeep this pro�t positive.

2.8.7.7 Real-World Applications

Exercise 2.8.73 (Solution on p. 265.)

In chemistry the volume for a certain gas is given byV = 20T,where V is measured in cc and Tis temperature in ºC. If the temperature varies between 80ºC and 120ºC, �nd the set of volumevalues.

Exercise 2.8.74A basic cellular package costs $20/mo. for 60 min of calling, with an additional charge of $.30/minbeyond that time.. The cost formula would be

C = $20 + .30 (x− 60) .

If you have to keep your bill lower than $50, what is the maximum calling minutes you can use?

2.8.8 Chapter Review Exercises

2.8.8.1 The Rectangular Coordinate Systems and Graphs (Section 2.2)

For the following exercises, �nd the x-intercept and the y-intercept without graphing.

Exercise 2.8.75 (Solution on p. 265.)

4x− 3y = 12Exercise 2.8.762y − 4 = 3x

For the following exercises, solve for y in terms of x, putting the equation in slope�intercept form.

Available for free at Connexions <http://legacy.cnx.org/content/col11759/1.2>

239

Exercise 2.8.77 (Solution on p. 265.)

5x = 3y − 12Exercise 2.8.782x− 5y = 7

For the following exercises, �nd the distance between the two points.

Exercise 2.8.79 (Solution on p. 265.)

(−2, 5) (4,−1)Exercise 2.8.80(−12,−3) (−1, 5)Exercise 2.8.81 (Solution on p. 265.)

Find the distance between the two points (−71, 432) and (511,218) using your calculator, and roundyour answer to the nearest thousandth.

For the following exercises, �nd the coordinates of the midpoint of the line segment that joins the two givenpoints.

Exercise 2.8.82(−1, 5) and (4, 6)Exercise 2.8.83 (Solution on p. 265.)

(−13, 5) and (17, 18)For the following exercises, construct a table and graph the equation by plotting at least three points.

Exercise 2.8.84y = 1

2x+ 4Exercise 2.8.85 (Solution on p. 265.)

4x− 3y = 6

2.8.8.2 Linear Equations in One Variable (Section 2.3)

For the following exercises, solve forx.

Exercise 2.8.865x+ 2 = 7x− 8Exercise 2.8.87 (Solution on p. 266.)

3 (x+ 2)− 10 = x+ 4Exercise 2.8.887x− 3 = 5Exercise 2.8.89 (Solution on p. 266.)

12− 5 (x+ 1) = 2x− 5Exercise 2.8.902x3 −

34 = x

6 + 214

For the following exercises, solve forx. State all x-values that are excluded from the solution set.

Exercise 2.8.91 (Solution on p. 266.)x

x2−9 + 4x+3 = 3

x2−9 x 6= 3,−3Exercise 2.8.9212 + 2

x = 34

For the following exercises, �nd the equation of the line using the point-slope formula.

Exercise 2.8.93 (Solution on p. 266.)

Passes through these two points: (−2, 1) , (4, 2) .

Available for free at Connexions <http://legacy.cnx.org/content/col11759/1.2>

240 CHAPTER 2. EQUATIONS AND INEQUALITIES

Exercise 2.8.94Passes through the point (−3, 4) and has a slope of −1

3 .

Exercise 2.8.95 (Solution on p. 266.)

Passes through the point (−3, 4) and is parallel to the graph y = 23x+ 5.

Exercise 2.8.96Passes through these two points: (5, 1) , (5, 7) .

2.8.8.3 Models and Applications (Section 2.4)

For the following exercises, write and solve an equation to answer each question.

Exercise 2.8.97 (Solution on p. 266.)

The number of males in the classroom is �ve more than three times the number of females. If thetotal number of students is 73, how many of each gender are in the class?

Exercise 2.8.98A man has 72 ft. of fencing to put around a rectangular garden. If the length is 3 times the width,�nd the dimensions of his garden.

Exercise 2.8.99 (Solution on p. 267.)

A truck rental is $25 plus $.30/mi. Find out how many miles Ken traveled if his bill was $50.20.

2.8.8.4 Complex Numbers (Section 2.5)

For the following exercises, use the quadratic equation to solve.

Exercise 2.8.100x2 − 5x+ 9 = 0Exercise 2.8.101 (Solution on p. 267.)

2x2 + 3x+ 7 = 0For the following exercises, name the horizontal component and the vertical component.

Exercise 2.8.1024− 3iExercise 2.8.103 (Solution on p. 267.)

−2− iFor the following exercises, perform the operations indicated.

Exercise 2.8.104(9− i)− (4− 7i)Exercise 2.8.105 (Solution on p. 267.)

(2 + 3i)− (−5− 8i)Exercise 2.8.1062√−75 + 3

√25

Exercise 2.8.107 (Solution on p. 267.)√−16 + 4

√−9

Exercise 2.8.108−6i (i− 5)Exercise 2.8.109 (Solution on p. 267.)

(3− 5i)2

Available for free at Connexions <http://legacy.cnx.org/content/col11759/1.2>

241

Exercise 2.8.110√−4 ·

√−12

Exercise 2.8.111 (Solution on p. 267.)√−2(√−8−

√5)

Exercise 2.8.1122

5−3i

Exercise 2.8.113 (Solution on p. 267.)3+7ii

2.8.8.5 Quadratic Equations (Section 2.6)

For the following exercises, solve the quadratic equation by factoring.

Exercise 2.8.1142x2 − 7x− 4 = 0Exercise 2.8.115 (Solution on p. 267.)

3x2 + 18x+ 15 = 0Exercise 2.8.11625x2 − 9 = 0Exercise 2.8.117 (Solution on p. 267.)

7x2 − 9x = 0For the following exercises, solve the quadratic equation by using the square-root property.

Exercise 2.8.118x2 = 49Exercise 2.8.119 (Solution on p. 267.)

(x− 4)2 = 36For the following exercises, solve the quadratic equation by completing the square.

Exercise 2.8.120x2 + 8x− 5 = 0Exercise 2.8.121 (Solution on p. 267.)

4x2 + 2x− 1 = 0For the following exercises, solve the quadratic equation by using the quadratic formula. If the solutions arenot real, state No real solution.

Exercise 2.8.1222x2 − 5x+ 1 = 0Exercise 2.8.123 (Solution on p. 267.)

15x2 − x− 2 = 0For the following exercises, solve the quadratic equation by the method of your choice.

Exercise 2.8.124(x− 2)2 = 16Exercise 2.8.125 (Solution on p. 267.)

x2 = 10x+ 3

Available for free at Connexions <http://legacy.cnx.org/content/col11759/1.2>

242 CHAPTER 2. EQUATIONS AND INEQUALITIES

2.8.8.6 Other Types of Equations (Section 2.7)

For the following exercises, solve the equations.

Exercise 2.8.126x

32 = 27

Exercise 2.8.127 (Solution on p. 267.)

x12 − 4x

14 = 0

Exercise 2.8.1284x3 + 8x2 − 9x− 18 = 0Exercise 2.8.129 (Solution on p. 267.)

3x5 − 6x3 = 0Exercise 2.8.130√x+ 9 = x− 3

Exercise 2.8.131 (Solution on p. 267.)√3x+ 7 +

√x+ 2 = 1

Exercise 2.8.132|3x− 7| = 5Exercise 2.8.133 (Solution on p. 267.)

|2x+ 3| − 5 = 9

2.8.8.7 Linear Inequalities and Absolute Value Inequalities (Section 2.8)

For the following exercises, solve the inequality. Write your �nal answer in interval notation.

Exercise 2.8.1345x− 8 ≤ 12Exercise 2.8.135 (Solution on p. 267.)

−2x+ 5 > x− 7Exercise 2.8.136x−1

3 + x+25 ≤ 3

5

Exercise 2.8.137 (Solution on p. 267.)

|3x+ 2|+ 1 ≤ 9Exercise 2.8.138|5x− 1| > 14Exercise 2.8.139 (Solution on p. 267.)

|x− 3| < − 4For the following exercises, solve the compound inequality. Write your answer in interval notation.

Exercise 2.8.140−4 < 3x+ 2 ≤ 18Exercise 2.8.141 (Solution on p. 267.)

3y < 1− 2y < 5 + y

For the following exercises, graph as described.

Exercise 2.8.142Graph the absolute value function and graph the constant function. Observe the points of inter-section and shade the x-axis representing the solution set to the inequality. Show your graph andwrite your �nal answer in interval notation.|x+ 3| ≥ 5

Available for free at Connexions <http://legacy.cnx.org/content/col11759/1.2>

243

Exercise 2.8.143 (Solution on p. 267.)

Graph both straight lines (left-hand side being y1 and right-hand side being y2) on the same axes.Find the point of intersection and solve the inequality by observing where it is true comparing they-values of the lines. See the interval where the inequality is true.

x+ 3 < 3x− 4

2.8.9 Chapter Practice Test

Exercise 2.8.144 (Solution on p. 268.)

Graph the following: 2y = 3x+ 4.Exercise 2.8.145 (Solution on p. 269.)

Find the x- and y-intercepts for the following:

Exercise 2.8.146 (Solution on p. 269.)

Find the x- and y-intercepts of this equation, and sketch the graph of the line using just theintercepts plotted.

3x− 4y = 12Find the exact distance between (5,−3) and (−2, 8) .Find the coordinates of the midpoint of the line segmentjoining the two points.

Exercise 2.8.147 (Solution on p. 270.)

Write the interval notation for the set of numbers represented by {x|x ≤ 9}.Exercise 2.8.148Solve for x: 5x+ 8 = 3x− 10.Exercise 2.8.149 (Solution on p. 270.)

Solve for x: 3 (2x− 5)− 3 (x− 7) = 2x− 9.Exercise 2.8.150Solve for x: x2 + 1 = 4

x

Exercise 2.8.151 (Solution on p. 270.)

Solve for x: 5x+4 = 4 + 3

x−2 .

Exercise 2.8.152The perimeter of a triangle is 30 in. The longest side is 2 less than 3 times the shortest side andthe other side is 2 more than twice the shortest side. Find the length of each side.

Exercise 2.8.153 (Solution on p. 270.)

Solve for x. Write the answer in simplest radical form.x2

3 − x = −12

Exercise 2.8.154Solve: 3x− 8 ≤ 4.Exercise 2.8.155 (Solution on p. 270.)

Solve: |2x+ 3| < 5.Exercise 2.8.156Solve: |3x− 2| ≥ 4.

For the following exercises, �nd the equation of the line with the given information.

Exercise 2.8.157 (Solution on p. 270.)

Passes through the points (−4, 2) and (5,−3) .Exercise 2.8.158Has an unde�ned slope and passes through the point (4, 3) .

Available for free at Connexions <http://legacy.cnx.org/content/col11759/1.2>

244 CHAPTER 2. EQUATIONS AND INEQUALITIES

Exercise 2.8.159 (Solution on p. 270.)

Passes through the point (2, 1) and is perpendicular to y = −25 x+ 3.

Exercise 2.8.160Add these complex numbers: (3− 2i) + (4− i) .Exercise 2.8.161 (Solution on p. 270.)

Simplify:√−4 + 3

√−16.

Exercise 2.8.162Multiply: 5i (5− 3i) .Exercise 2.8.163 (Solution on p. 270.)

Divide: 4−i2+3i .

Exercise 2.8.164Solve this quadratic equation and write the two complex roots in a+ bi form:x2 − 4x+ 7 = 0.Exercise 2.8.165 (Solution on p. 270.)

Solve: (3x− 1)2 − 1 = 24.Exercise 2.8.166Solve:x2 − 6x = 13.Exercise 2.8.167 (Solution on p. 270.)

Solve: 4x2 − 4x− 1 = 0Exercise 2.8.168Solve:√

x− 7 = x− 7Exercise 2.8.169 (Solution on p. 271.)

Solve: 2 +√

12− 2x = x

Exercise 2.8.170Solve: (x− 1)

23 = 9

For the following exercises, �nd the real solutions of each equation by factoring.

Exercise 2.8.171 (Solution on p. 271.)

2x3 − x2 − 8x+ 4 = 0Exercise 2.8.172(x+ 5)2 − 3 (x+ 5)− 4 = 0

Available for free at Connexions <http://legacy.cnx.org/content/col11759/1.2>

245

Solutions to Exercises in Chapter 2

Solution to Exercise 2.3.1 (p. 132)x = −5Solution to Exercise 2.3.2 (p. 132)x = −3Solution to Exercise 2.3.3 (p. 135)x = 10

3Solution to Exercise 2.3.4 (p. 135)x = 1Solution to Exercise 2.3.5 (p. 136)x = − 7

17 .Excluded values arex = − 12 andx = − 1

3 .Solution to Exercise 2.3.6 (p. 137)x = 1

3Solution to Exercise 2.3.7 (p. 139)m = − 2

3Solution to Exercise 2.3.8 (p. 140)y = 4x− 3Solution to Exercise 2.3.9 (p. 141)x+ 3y = 2Solution to Exercise 2.3.10 (p. 143)Horizontal line: y = 2Solution to Exercise 2.3.11 (p. 148)Parallel lines: equations are written in slope-intercept form.

Available for free at Connexions <http://legacy.cnx.org/content/col11759/1.2>

246 CHAPTER 2. EQUATIONS AND INEQUALITIES

Figure 2.28

Solution to Exercise 2.3.12 (p. 149)y = 5x+ 3Solution to Exercise 2.3.13 (p. 151)It means they have the same slope.Solution to Exercise 2.3.15 (p. 151)The exponent of thex variable is 1. It is called a �rst-degree equation.Solution to Exercise 2.3.17 (p. 151)If we insert either value into the equation, they make an expression in the equation unde�ned (zero in thedenominator).Solution to Exercise 2.3.19 (p. 151)x = 2Solution to Exercise 2.3.21 (p. 151)x = 2

7Solution to Exercise 2.3.23 (p. 151)x = 6Solution to Exercise 2.3.25 (p. 151)x = 3Solution to Exercise 2.3.27 (p. 151)x = −14Solution to Exercise 2.3.29 (p. 151)x 6= −4;x = −3

Available for free at Connexions <http://legacy.cnx.org/content/col11759/1.2>

247

Solution to Exercise 2.3.31 (p. 152)x 6= 1;when we solve this we getx = 1,which is excluded, therefore NO solutionSolution to Exercise 2.3.33 (p. 152)x 6= 0;x = −5

2Solution to Exercise 2.3.35 (p. 152)y = −4

5 x+ 145

Solution to Exercise 2.3.37 (p. 152)y = −3

4 x+ 2Solution to Exercise 2.3.39 (p. 152)y = 1

2x+ 52

Solution to Exercise 2.3.41 (p. 152)y = −3x− 5Solution to Exercise 2.3.43 (p. 152)y = 7Solution to Exercise 2.3.45 (p. 152)y = −4Solution to Exercise 2.3.47 (p. 152)8x+ 5y = 7Solution to Exercise 2.3.49 (p. 153)

Parallel

Available for free at Connexions <http://legacy.cnx.org/content/col11759/1.2>

248 CHAPTER 2. EQUATIONS AND INEQUALITIES

Solution to Exercise 2.3.51 (p. 153)

PerpendicularSolution to Exercise 2.3.53 (p. 153)m = −9

7Solution to Exercise 2.3.55 (p. 153)m = 3

2Solution to Exercise 2.3.57 (p. 153)m1 = −1

3 , m2 = 3; Perpendicular.Solution to Exercise 2.3.59 (p. 154)y = 0.245x− 45.662.Answers may vary. ymin = −50, ymax = −40Solution to Exercise 2.3.61 (p. 154)y = −2.333x+ 6.667.Answers may vary. ymin = −10, ymax = 10Solution to Exercise 2.3.63 (p. 154)y = −A

B x+ CB

Solution to Exercise 2.3.65 (p. 154)

The slope for (−1, 1) to (0, 4) is 3.

The slope for (−1, 1) to (2, 0) is −13 .

The slope for (2, 0) to (3, 3) is 3.

The slope for (0, 4) to (3, 3) is −13 .

Available for free at Connexions <http://legacy.cnx.org/content/col11759/1.2>

249

Yes they are perpendicular.Solution to Exercise 2.3.67 (p. 154)30 ftSolution to Exercise 2.3.69 (p. 155)$57.50Solution to Exercise 2.3.71 (p. 155)220 miSolution to Exercise 2.4.1 (p. 157)11 and 25Solution to Exercise 2.4.2 (p. 159)C = 2.5x+ 3, 650Solution to Exercise 2.4.3 (p. 161)45 mi/hSolution to Exercise 2.4.4 (p. 162)L = 37 cm,W = 18 cmSolution to Exercise 2.4.5 (p. 163)250 ft2

Solution to Exercise 2.4.6 (p. 164)Answers may vary. Possible answers: We should de�ne in words what our variable is representing. Weshould declare the variable. A heading.Solution to Exercise 2.4.8 (p. 164)2, 000− xSolution to Exercise 2.4.10 (p. 164)v + 10Solution to Exercise 2.4.12 (p. 164)Ann: 23;Beth: 46Solution to Exercise 2.4.14 (p. 165)20 + 0.05mSolution to Exercise 2.4.16 (p. 165)300 minSolution to Exercise 2.4.18 (p. 165)90 + 40PSolution to Exercise 2.4.20 (p. 165)6 devicesSolution to Exercise 2.4.22 (p. 165)50, 000− xSolution to Exercise 2.4.24 (p. 165)4 hSolution to Exercise 2.4.26 (p. 166)She traveled for 2 h at 20 mi/h, or 40 miles.Solution to Exercise 2.4.28 (p. 166)$5,000 at 8% and $15,000 at 12%Solution to Exercise 2.4.30 (p. 166)B = 100 + .05xSolution to Exercise 2.4.32 (p. 166)Plan ASolution to Exercise 2.4.34 (p. 166)R = 9Solution to Exercise 2.4.36 (p. 166)r = 4

5 or 0.8

Available for free at Connexions <http://legacy.cnx.org/content/col11759/1.2>

250 CHAPTER 2. EQUATIONS AND INEQUALITIES

Solution to Exercise 2.4.38 (p. 166)

W = P−2L2 = 58−2(15)

2 = 14Solution to Exercise 2.4.40 (p. 166)

f = pqp+q = 8(13)

8+13 = 10421

Solution to Exercise 2.4.42 (p. 167)m = −5

4Solution to Exercise 2.4.44 (p. 167)h = 2A

b1+b2Solution to Exercise 2.4.46 (p. 167)length = 360 ft; width = 160 ftSolution to Exercise 2.4.48 (p. 167)405 miSolution to Exercise 2.4.50 (p. 167)A = 88 in.2

Solution to Exercise 2.4.52 (p. 167)28.7Solution to Exercise 2.4.54 (p. 167)h = V

πr2

Solution to Exercise 2.4.56 (p. 167)

r =√

Vπh

Solution to Exercise 2.4.58 (p. 167)C = 12πSolution to Exercise 2.5.1 (p. 170)√−24 = 0 + 2i

√6

Available for free at Connexions <http://legacy.cnx.org/content/col11759/1.2>

251

Solution to Exercise 2.5.2 (p. 173)

Solu-tion to Exercise 2.5.3 (p. 174)(3− 4i)− (2 + 5i) = 1− 9iSolution to Exercise 2.5.4 (p. 175)52 − iSolution to Exercise 2.5.5 (p. 176)18 + iSolution to Exercise 2.5.6 (p. 177)−3− 4iSolution to Exercise 2.5.7 (p. 179)−1Solution to Exercise 2.5.8 (p. 180)Add the real parts together and the imaginary parts together.Solution to Exercise 2.5.10 (p. 180)Possible answer: i times i equals 1, which is not imaginary.Solution to Exercise 2.5.12 (p. 180)−8 + 2i

Available for free at Connexions <http://legacy.cnx.org/content/col11759/1.2>

252 CHAPTER 2. EQUATIONS AND INEQUALITIES

Solution to Exercise 2.5.14 (p. 180)14 + 7iSolution to Exercise 2.5.16 (p. 180)− 23

29 + 1529 i

Solution to Exercise 2.5.18 (p. 181)

Solution to Exercise 2.5.20 (p. 181)

Available for free at Connexions <http://legacy.cnx.org/content/col11759/1.2>

253

Solution to Exercise 2.5.22 (p. 181)8− iSolution to Exercise 2.5.24 (p. 181)−11 + 4iSolution to Exercise 2.5.26 (p. 181)2− 5iSolution to Exercise 2.5.28 (p. 181)6 + 15iSolution to Exercise 2.5.30 (p. 181)−16 + 32iSolution to Exercise 2.5.32 (p. 181)−4− 7iSolution to Exercise 2.5.34 (p. 181)25Solution to Exercise 2.5.36 (p. 181)2− 2

3 iSolution to Exercise 2.5.38 (p. 182)4− 6iSolution to Exercise 2.5.40 (p. 182)25 + 11

5 iSolution to Exercise 2.5.42 (p. 182)15i

Available for free at Connexions <http://legacy.cnx.org/content/col11759/1.2>

254 CHAPTER 2. EQUATIONS AND INEQUALITIES

Solution to Exercise 2.5.44 (p. 182)1 + i

√3

Solution to Exercise 2.5.46 (p. 182)1Solution to Exercise 2.5.48 (p. 182)−1Solution to Exercise 2.5.50 (p. 182)128iSolution to Exercise 2.5.52 (p. 182)(√

32 + 1

2 i)6

= −1Solution to Exercise 2.5.54 (p. 183)3iSolution to Exercise 2.5.56 (p. 183)0Solution to Exercise 2.5.58 (p. 183)5− 5iSolution to Exercise 2.5.60 (p. 183)−2iSolution to Exercise 2.5.62 (p. 183)92 −

92 i

Solution to Exercise 2.6.1 (p. 188)(x− 6) (x+ 1) = 0;x = 6, x = −1Solution to Exercise 2.6.2 (p. 188)(x− 7) (x+ 3) = 0,x = 7,x = −3.Solution to Exercise 2.6.3 (p. 189)(x+ 5) (x− 5) = 0,x = −5,x = 5.Solution to Exercise 2.6.4 (p. 191)(3x+ 2) (4x+ 1) = 0,x = − 2

3 ,x = − 14

Solution to Exercise 2.6.5 (p. 192)x = 0, x = −10, x = −1Solution to Exercise 2.6.6 (p. 193)x = 4±

√5

Solution to Exercise 2.6.7 (p. 195)x = 3±

√22

Solution to Exercise 2.6.8 (p. 197)x = − 2

3 ,x = 13

Solution to Exercise 2.6.9 (p. 201)It is a second-degree equation (the highest variable exponent is 2).Solution to Exercise 2.6.11 (p. 201)We want to take advantage of the zero property of multiplication in the fact that if a · b = 0 then it mustfollow that each factor separately o�ers a solution to the product being zero: a = 0 or b = 0.Solution to Exercise 2.6.13 (p. 201)One, when no linear term is present (no x term), such asx2 = 16.Two, when the equation is already in the

form (ax+ b)2 = d.Solution to Exercise 2.6.15 (p. 201)x = 6,x = 3Solution to Exercise 2.6.17 (p. 201)x = −5

2 ,x = −13

Solution to Exercise 2.6.19 (p. 202)x = 5,x = −5

Available for free at Connexions <http://legacy.cnx.org/content/col11759/1.2>

255

Solution to Exercise 2.6.21 (p. 202)x = −3

2 ,x = 32

Solution to Exercise 2.6.23 (p. 202)x = −2,Solution to Exercise 2.6.25 (p. 202)x = 0,x = −3

7Solution to Exercise 2.6.27 (p. 202)x = −6,x = 6Solution to Exercise 2.6.29 (p. 202)x = 6,x = −4Solution to Exercise 2.6.31 (p. 202)x = 1,x = −2Solution to Exercise 2.6.33 (p. 202)x = −2,x = 11Solution to Exercise 2.6.35 (p. 202)x = 3±

√22

Solution to Exercise 2.6.37 (p. 202)z = 2

3 ,z = − 12

Solution to Exercise 2.6.39 (p. 203)

x = 3±√

174

Solution to Exercise 2.6.41 (p. 203)Not realSolution to Exercise 2.6.43 (p. 203)One rationalSolution to Exercise 2.6.45 (p. 203)Two real; rationalSolution to Exercise 2.6.47 (p. 203)

x = −1±√

172

Solution to Exercise 2.6.49 (p. 203)

x = 5±√

136

Solution to Exercise 2.6.51 (p. 203)

x = −1±√

178

Solution to Exercise 2.6.53 (p. 203)x ≈ 0.131 andx ≈ 2.535Solution to Exercise 2.6.55 (p. 203)x ≈ −6.7 andx ≈ 1.7Solution to Exercise 2.6.57 (p. 204)

ax2 + bx+ c = 0

x2 + bax = −c

a

x2 + bax+ b2

4a2 = −ca + b

4a2(x+ b

2a

)2= b2−4ac

4a2

x+ b2a = ±

√b2−4ac

4a2

x = −b±√b2−4ac

2a

Solution to Exercise 2.6.59 (p. 204)x (x+ 10) = 119;7 ft. and 17 ft.Solution to Exercise 2.6.61 (p. 204)maximum atx = 70

Available for free at Connexions <http://legacy.cnx.org/content/col11759/1.2>

256 CHAPTER 2. EQUATIONS AND INEQUALITIES

Solution to Exercise 2.6.63 (p. 204)The quadratic equation would be

(100x− 0.5x2

)− (60x+ 300) = 300.The two values ofx are 20 and 60.

Solution to Exercise 2.6.65 (p. 205)3 feetSolution to Exercise 2.7.1 (p. 206)14Solution to Exercise 2.7.2 (p. 207)25Solution to Exercise 2.7.3 (p. 208){−1}Solution to Exercise 2.7.4 (p. 209)x = 0,x = 1

2 ,x = − 12

Solution to Exercise 2.7.5 (p. 213)x = 1;extraneous solutionx = − 2

9Solution to Exercise 2.7.6 (p. 214)x = −2;extraneous solutionx = −1Solution to Exercise 2.7.7 (p. 216)x = −1,x = 3

2Solution to Exercise 2.7.8 (p. 217)x = −3, 3,−i, iSolution to Exercise 2.7.9 (p. 218)x = 2, x = 12Solution to Exercise 2.7.10 (p. 219)x = −1,x = 0is not a solution.Solution to Exercise 2.7.11 (p. 219)This is not a solution to the radical equation, it is a value obtained from squaring both sides and thuschanging the signs of an equation which has caused it not to be a solution in the original equation.Solution to Exercise 2.7.13 (p. 220)He or she is probably trying to enter negative 9, but taking the square root of − 9 is not a real number. Thenegative sign is in front of this, so your friend should be taking the square root of 9, cubing it, and thenputting the negative sign in front, resulting in − 27.Solution to Exercise 2.7.15 (p. 220)A rational exponent is a fraction: the denominator of the fraction is the root or index number and thenumerator is the power to which it is raised.Solution to Exercise 2.7.17 (p. 220)x = 81Solution to Exercise 2.7.19 (p. 220)x = 17Solution to Exercise 2.7.21 (p. 220)x = 8, x = 27Solution to Exercise 2.7.23 (p. 220)x = −2, 1,−1Solution to Exercise 2.7.25 (p. 220)y = 0, 3

2 ,−32

Solution to Exercise 2.7.27 (p. 220)m = 1,−1Solution to Exercise 2.7.29 (p. 220)x = 2

5Solution to Exercise 2.7.31 (p. 220)x = 32

Available for free at Connexions <http://legacy.cnx.org/content/col11759/1.2>

257

Solution to Exercise 2.7.33 (p. 221)t = 44

3Solution to Exercise 2.7.35 (p. 221)x = 3Solution to Exercise 2.7.37 (p. 221)x = −2Solution to Exercise 2.7.39 (p. 221)x = 4, −4

3Solution to Exercise 2.7.41 (p. 221)x = −5

4 ,74

Solution to Exercise 2.7.43 (p. 221)x = 3,−2Solution to Exercise 2.7.45 (p. 221)x = −5Solution to Exercise 2.7.47 (p. 221)x = 1,−1, 3,−3Solution to Exercise 2.7.49 (p. 221)x = 2,−2Solution to Exercise 2.7.51 (p. 221)x = 1, 5Solution to Exercise 2.7.53 (p. 222)All real numbersSolution to Exercise 2.7.55 (p. 222)x = 4, 6,−6,−8Solution to Exercise 2.7.57 (p. 222)10 in.Solution to Exercise 2.7.59 (p. 222)90 kgSolution to Exercise 2.8.1 (p. 225)[−3, 5]Solution to Exercise 2.8.2 (p. 225)(−∞,−2) ∪ [3,∞)Solution to Exercise 2.8.3 (p. 226)x < 1Solution to Exercise 2.8.4 (p. 227)x ≥ −5Solution to Exercise 2.8.5 (p. 228)(2,∞)Solution to Exercise 2.8.6 (p. 228)[− 3

14 ,∞)

Solution to Exercise 2.8.7 (p. 229)6 < x ≤ 9 or (6, 9]Solution to Exercise 2.8.8 (p. 230)(− 1

8 ,12

)Solution to Exercise 2.8.9 (p. 231)|x− 2| ≤ 3Solution to Exercise 2.8.10 (p. 233)k ≤ 1 or k ≥ 7;in interval notation, this would be (−∞, 1] ∪ [7,∞) .

Available for free at Connexions <http://legacy.cnx.org/content/col11759/1.2>

258 CHAPTER 2. EQUATIONS AND INEQUALITIES

So-lution to Exercise 2.8.11 (p. 234)When we divide both sides by a negative it changes the sign of both sides so the sense of the inequality signchanges.Solution to Exercise 2.8.13 (p. 234)(−∞,∞)Solution to Exercise 2.8.15 (p. 234)We start by �nding the x-intercept, or where the function = 0. Once we have that point, which is (3, 0) ,wegraph to the right the straight line graph y = x− 3,and then when we draw it to the left we plot positive yvalues, taking the absolute value of them.Solution to Exercise 2.8.17 (p. 235)(−∞, 3

4

]Solution to Exercise 2.8.19 (p. 235)[−13

2 ,∞)

Solution to Exercise 2.8.21 (p. 235)(−∞, 3)

Available for free at Connexions <http://legacy.cnx.org/content/col11759/1.2>

259

Solution to Exercise 2.8.23 (p. 235)(−∞,− 37

3

]Solution to Exercise 2.8.25 (p. 235)All real numbers (−∞,∞)Solution to Exercise 2.8.27 (p. 235)(−∞, −10

3

)∪ (4,∞)

Solution to Exercise 2.8.29 (p. 235)(−∞,−4] ∪ [8,+∞)Solution to Exercise 2.8.31 (p. 235)No solutionSolution to Exercise 2.8.33 (p. 235)(−5, 11)Solution to Exercise 2.8.35 (p. 235)[6, 12]Solution to Exercise 2.8.37 (p. 236)[−10, 12]Solution to Exercise 2.8.39 (p. 236)

x > − 6 and x > − 2 Take the intersection of two sets.

x > − 2, (−2,+∞)Solution to Exercise 2.8.41 (p. 236)

x < − 3 or x ≥ 1 Take the union of the two sets.

(−∞,−3)∪ [1,∞)Solution to Exercise 2.8.43 (p. 236)(−∞,−1) ∪ (3,∞)

Available for free at Connexions <http://legacy.cnx.org/content/col11759/1.2>

260 CHAPTER 2. EQUATIONS AND INEQUALITIES

Solution to Exercise 2.8.45 (p. 236)[−11,−3]

Available for free at Connexions <http://legacy.cnx.org/content/col11759/1.2>

261

Solution to Exercise 2.8.47 (p. 236)It is never less than zero. No solution.

Solution to Exercise 2.8.49 (p. 236)Where the blue line is above the orange line; point of intersection isx = −3.

(−∞,−3)

Available for free at Connexions <http://legacy.cnx.org/content/col11759/1.2>

262 CHAPTER 2. EQUATIONS AND INEQUALITIES

Solution to Exercise 2.8.51 (p. 236)Where the blue line is above the orange line; always. All real numbers.

(−∞,−∞)

Available for free at Connexions <http://legacy.cnx.org/content/col11759/1.2>

263

Solution to Exercise 2.8.53 (p. 237)(−1, 3)Solution to Exercise 2.8.55 (p. 237)(−∞, 4)Solution to Exercise 2.8.57 (p. 237){x|x < 6}Solution to Exercise 2.8.59 (p. 237){x| − 3 ≤ x < 5}Solution to Exercise 2.8.61 (p. 237)(−2, 1]Solution to Exercise 2.8.63 (p. 237)(−∞, 4]Solution to Exercise 2.8.65 (p. 238)Where the blue is below the orange; always. All real numbers. (−∞,+∞) .

Available for free at Connexions <http://legacy.cnx.org/content/col11759/1.2>

264 CHAPTER 2. EQUATIONS AND INEQUALITIES

Solution to Exercise 2.8.67 (p. 238)Where the blue is below the orange; (1, 7) .

Available for free at Connexions <http://legacy.cnx.org/content/col11759/1.2>

265

Solution to Exercise 2.8.69 (p. 238)x = 2, −4

5Solution to Exercise 2.8.71 (p. 238)(−7, 5]Solution to Exercise 2.8.73 (p. 238)

80 ≤ T ≤ 120

1, 600 ≤ 20T ≤ 2, 400[1, 600, 2, 400]

Solution to Exercise 2.8.75 (p. 238)x-intercept: (3, 0) ;y-intercept: (0,−4)Solution to Exercise 2.8.77 (p. 239)y = 5

3x+ 4Solution to Exercise 2.8.79 (p. 239)√

72 = 6√

2Solution to Exercise 2.8.81 (p. 239)620.097Solution to Exercise 2.8.83 (p. 239)midpoint is

(2, 23

2

)Solution to Exercise 2.8.85 (p. 239)

Available for free at Connexions <http://legacy.cnx.org/content/col11759/1.2>

266 CHAPTER 2. EQUATIONS AND INEQUALITIES

x y

0 −23 2

6 6

Table 2.7

Solution to Exercise 2.8.87 (p. 239)x = 4Solution to Exercise 2.8.89 (p. 239)x = 12

7Solution to Exercise 2.8.91 (p. 239)No solutionSolution to Exercise 2.8.93 (p. 239)y = 1

6x+ 43

Solution to Exercise 2.8.95 (p. 240)y = 2

3x+ 6

Available for free at Connexions <http://legacy.cnx.org/content/col11759/1.2>

267

Solution to Exercise 2.8.97 (p. 240)females 17, males 56Solution to Exercise 2.8.99 (p. 240)84 miSolution to Exercise 2.8.101 (p. 240)

x = −34 ±

i√

474

Solution to Exercise 2.8.103 (p. 240)horizontal component − 2;vertical component − 1Solution to Exercise 2.8.105 (p. 240)7 + 11iSolution to Exercise 2.8.107 (p. 240)16iSolution to Exercise 2.8.109 (p. 240)−16− 30iSolution to Exercise 2.8.111 (p. 241)−4− i

√10

Solution to Exercise 2.8.113 (p. 241)x = 7− 3iSolution to Exercise 2.8.115 (p. 241)x = −1,−5Solution to Exercise 2.8.117 (p. 241)x = 0, 9

7Solution to Exercise 2.8.119 (p. 241)x = 10,−2Solution to Exercise 2.8.121 (p. 241)

x = −1±√

54

Solution to Exercise 2.8.123 (p. 241)x = 2

5 ,−13

Solution to Exercise 2.8.125 (p. 241)x = 5± 2

√7

Solution to Exercise 2.8.127 (p. 242)x = 0, 256Solution to Exercise 2.8.129 (p. 242)x = 0,±

√2

Solution to Exercise 2.8.131 (p. 242)x = −2Solution to Exercise 2.8.133 (p. 242)x = 11

2 ,−17

2Solution to Exercise 2.8.135 (p. 242)(−∞, 4)Solution to Exercise 2.8.137 (p. 242)[−10

3 , 2]

Solution to Exercise 2.8.139 (p. 242)No solutionSolution to Exercise 2.8.141 (p. 242)(− 4

3 ,15

)Solution to Exercise 2.8.143 (p. 242)Where the blue is below the orange line; point of intersection isx = 3.5.

(3.5,∞)

Available for free at Connexions <http://legacy.cnx.org/content/col11759/1.2>

268 CHAPTER 2. EQUATIONS AND INEQUALITIES

Solution to Exercise 2.8.144 (p. 243)y = 3

2x+ 2

x y

0 2

2 5

4 8

Table 2.8

Available for free at Connexions <http://legacy.cnx.org/content/col11759/1.2>

269

Solution to Exercise 2.8.145 (p. 243)2x− 5y = 6Solution to Exercise 2.8.146 (p. 243)(0,−3)(4, 0)

Available for free at Connexions <http://legacy.cnx.org/content/col11759/1.2>

270 CHAPTER 2. EQUATIONS AND INEQUALITIES

Solution to Exercise 2.8.147 (p. 243)(−∞, 9]Solution to Exercise 2.8.149 (p. 243)x = −15Solution to Exercise 2.8.151 (p. 243)x 6= −4, 2;x = −5

2 , 1Solution to Exercise 2.8.153 (p. 243)

x = 3±√

32

Solution to Exercise 2.8.155 (p. 243)(−4, 1)Solution to Exercise 2.8.157 (p. 243)y = −5

9 x−29

Solution to Exercise 2.8.159 (p. 243)y = 5

2x− 4Solution to Exercise 2.8.161 (p. 244)14iSolution to Exercise 2.8.163 (p. 244)513 −

1413 i

Solution to Exercise 2.8.165 (p. 244)x = 2, −4

3

Available for free at Connexions <http://legacy.cnx.org/content/col11759/1.2>

271

Solution to Exercise 2.8.167 (p. 244)

x = 12 ±

√2

2Solution to Exercise 2.8.169 (p. 244)4Solution to Exercise 2.8.171 (p. 244)x = 1

2 , 2,−2

Available for free at Connexions <http://legacy.cnx.org/content/col11759/1.2>

272 CHAPTER 2. EQUATIONS AND INEQUALITIES

Available for free at Connexions <http://legacy.cnx.org/content/col11759/1.2>

Chapter 3

Functions

3.1 Introduction to Functions1

Figure 3.1: Standard and Poor's Index with dividends reinvested (credit "bull": modi�cation of workby Prayitno Hadinata; credit "graph": modi�cation of work by MeasuringWorth)

Toward the end of the twentieth century, the values of stocks of Internet and technology companies rosedramatically. As a result, the Standard and Poor's stock market average rose as well. Figure 3.1 tracksthe value of that initial investment of just under $100 over the 40 years. It shows that an investment thatwas worth less than $500 until about 1995 skyrocketed up to about $1100 by the beginning of 2000. That�ve-year period became known as the �dot-com bubble� because so many Internet startups were formed. Asbubbles tend to do, though, the dot-com bubble eventually burst. Many companies grew too fast and thensuddenly went out of business. The result caused the sharp decline represented on the graph beginning atthe end of 2000.

Notice, as we consider this example, that there is a de�nite relationship between the year and stockmarket average. For any year we choose, we can determine the corresponding value of the stock marketaverage. In this chapter, we will explore these kinds of relationships and their properties.

1This content is available online at <http://legacy.cnx.org/content/m51260/1.2/>.

Available for free at Connexions <http://legacy.cnx.org/content/col11759/1.2>

273

274 CHAPTER 3. FUNCTIONS

3.2 Functions and Function Notation2

A jetliner changes altitude as its distance from the starting point of a �ight increases. The weight of agrowing child increases with time. In each case, one quantity depends on another. There is a relationshipbetween the two quantities that we can describe, analyze, and use to make predictions. In this section, wewill analyze such relationships.

3.2.1 Determining Whether a Relation Represents a Function

A relation is a set of ordered pairs. The set consisting of the �rst components of each ordered pair iscalled the domain and the set consisting of the second components of each ordered pair is called the range.Consider the following set of ordered pairs. The �rst numbers in each pair are the �rst �ve natural numbers.The second number in each pair is twice that of the �rst.

{(1, 2) , (2, 4) , (3, 6) , (4, 8) , (5, 10)} (3.1)

The domain is {1, 2, 3, 4, 5}. The range is {2, 4, 6, 8, 10}.Note that each value in the domain is also known as an input value, or independent variable, and

is often labeled with the lowercase letterx.Each value in the range is also known as an output value, ordependent variable, and is often labeled lowercase letter y.

A function f is a relation that assigns a single element in the range to each element in the domain. Inother words, no x-values are repeated. For our example that relates the �rst �ve natural numbers tonumbers double their values, this relation is a function because each element in the domain, {1, 2, 3, 4, 5},is paired with exactly one element in the range, {2, 4, 6, 8, 10}.

Now let's consider the set of ordered pairs that relates the terms �even� and �odd� to the �rst �ve naturalnumbers. It would appear as

{(odd, 1) , (even, 2) , (odd, 3) , (even, 4) , (odd, 5)} (3.2)

Notice that each element in the domain, {even, odd} is not paired with exactly one element in the range,{1, 2, 3, 4, 5}. For example, the term �odd� corresponds to three values from the domain, {1, 3, 5} and theterm �even� corresponds to two values from the range, {2, 4}. This violates the de�nition of a function, sothis relation is not a function.

Figure 3.2 compares relations that are functions and not functions.

2This content is available online at <http://legacy.cnx.org/content/m51261/1.2/>.

Available for free at Connexions <http://legacy.cnx.org/content/col11759/1.2>

275

Figure 3.2: (a) This relationship is a function because each input is associated with a single output.Note that input q and r both give outputn. (b) This relationship is also a function. In this case, each inputis associated with a single output. (c) This relationship is not a function because input q is associatedwith two di�erent outputs.

A General Note: A function is a relation in which each possible input value leads to exactlyone output value. We say �the output is a function of the input.�

The input values make up the domain, and the output values make up the range.

How To: Given a relationship between two quantities, determine whether the rela-tionship is a function.

1.Identify the input values.2.Identify the output values.3.If each input value leads to only one output value, classify the relationship as a function. Ifany input value leads to two or more outputs, do not classify the relationship as a function.

Example 3.1Determining If Menu Price Lists Are FunctionsThe co�ee shop menu, shown in Figure 3.3 consists of items and their prices.

a. Is price a function of the item?b. Is the item a function of the price?

Available for free at Connexions <http://legacy.cnx.org/content/col11759/1.2>

276 CHAPTER 3. FUNCTIONS

Figure 3.3

Solution

a. Let's begin by considering the input as the items on the menu. The output values are thenthe prices. See Figure 3.4.

Figure 3.4

Each item on the menu has only one price, so the price is a function of the item.b. Two items on the menu have the same price. If we consider the prices to be the input values

and the items to be the output, then the same input value could have more than one outputassociated with it. See Figure 3.5.

Available for free at Connexions <http://legacy.cnx.org/content/col11759/1.2>

277

Figure 3.5

Therefore, the item is a not a function of price.

Example 3.2Determining If Class Grade Rules Are FunctionsIn a particular math class, the overall percent grade corresponds to a grade-point average. Is grade-point average a function of the percent grade? Is the percent grade a function of the grade-pointaverage? Table 3.1 shows a possible rule for assigning grade points.

Percent grade 0�56 57�61 62�66 67�71 72�77 78�86 87�91 92�100

Grade-point average 0.0 1.0 1.5 2.0 2.5 3.0 3.5 4.0

Table 3.1

SolutionFor any percent grade earned, there is an associated grade-point average, so the grade-point averageis a function of the percent grade. In other words, if we input the percent grade, the output is aspeci�c grade point average.

In the grading system given, there is a range of percent grades that correspond to the samegrade-point average. For example, students who receive a grade point average of 3.0 could have avariety of percent grades ranging from 78 all the way to 86. Thus, percent grade is not a functionof grade-point average.

Try It:

Exercise 3.2.1 (Solution on p. 327.)

Table 3.23 lists the �ve greatest baseball players of all time in order of rank.

3http://www.baseball-almanac.com/legendary/lisn100.shtml. Accessed 3/24/2014.

Available for free at Connexions <http://legacy.cnx.org/content/col11759/1.2>

278 CHAPTER 3. FUNCTIONS

Player Rank

Babe Ruth 1

Willie Mays 2

Ty Cobb 3

Walter Johnson 4

Hank Aaron 5

Table 3.2

a.Is the rank a function of the player name?b.Is the player name a function of the rank?

3.2.1.1 Using Function Notation

Once we determine that a relationship is a function, we need to display and de�ne the functional relationshipsso that we can understand and use them, and sometimes also so that we can program them into graphingcalculators and computers. There are various ways of representing functions. A standard function notationis one representation that facilitates working with functions.

To represent �height is a function of age,� we start by identifying the descriptive variables h for height anda for age. The letters f, g,andh are often used to represent functions just as we use x, y,andz to representnumbers and A, B, and C to represent sets.

h is f of a We name the function f ; height is a function of age.

h = f (a) We use parentheses to indicate the function input.

f (a) We name the function f ; the expression is read as �f of a.�

(3.3)

Remember, we can use any letter to name the function; the notationh (a) shows us thath depends on a.Thevalue amust be put into the functionh to get a result. The parentheses indicate that age is input into thefunction; they do not indicate multiplication.

We can also give an algebraic expression as the input to a function. For example f (a+ b) means ��rstadd a and b, and the result is the input for the function f.� The operations must be performed in this orderto obtain the correct result.

A General Note: The notation y = f (x) de�nes a function named f.This is readas “y is a function ofx.′′ The letterx represents the input value, or independent variable. Theletter y, or f (x) , represents the output value, or dependent variable.

Example 3.3Using Function Notation for Days in a MonthUse function notation to represent a function whose input is the name of a month and output isthe number of days in that month.

SolutionThe number of days in a month is a function of the name of the month, so if we name the functionf, we write days = f (month) or d = f (m) . The name of the month is the input to a �rule� thatassociates a speci�c number (the output) with each input.

Available for free at Connexions <http://legacy.cnx.org/content/col11759/1.2>

279

For example, f (March) = 31, because March has 31 days. The notation d = f (m) reminds us thatthe number of days, d (the output), is dependent on the name of the month,m (the input).

AnalysisNote that the inputs to a function do not have to be numbers; function inputs can be names ofpeople, labels of geometric objects, or any other element that determines some kind of output.However, most of the functions we will work with in this book will have numbers as inputs andoutputs.

Example 3.4Interpreting Function NotationA functionN = f (y) gives the number of police o�cers,N, in a town in year y.What

does f (2005) = 300 represent?

SolutionWhen we read f (2005) = 300, we see that the input year is 2005. The value for the output, thenumber of police o�cers (N) , is 300. Remember,N = f (y) .The statement f (2005) = 300 tells usthat in the year 2005 there were 300 police o�cers in the town.

Try It:

Exercise 3.2.2 (Solution on p. 327.)

Use function notation to express the weight of a pig in pounds as a function of its age indays d.

Q&A: Instead of a notation such as y = f (x) , could we use the same symbol for theoutput as for the function, such as y = y (x) , meaning �y is a function of x?�

Yes, this is often done, especially in applied subjects that use higher math, such as physics andengineering. However, in exploring math itself we like to maintain a distinction between a func-tion such as f, which is a rule or procedure, and the output ywe get by applying f to a particularinputx.This is why we usually use notation such as y = f (x) , P = W (d) , and so on.

3.2.1.2 Representing Functions Using Tables

A common method of representing functions is in the form of a table. The table rows or columns displaythe corresponding input and output values. In some cases, these values represent all we know about therelationship; other times, the table provides a few select examples from a more complete relationship.

Available for free at Connexions <http://legacy.cnx.org/content/col11759/1.2>

280 CHAPTER 3. FUNCTIONS

Table 3.3 lists the input number of each month (January = 1, February = 2, and so on) and the outputvalue of the number of days in that month. This information represents all we know about the monthsand days for a given year (that is not a leap year). Note that, in this table, we de�ne a days-in-a-monthfunction f whereD = f (m) identi�es months by an integer rather than by name.

Month number,m (input) 1 2 3 4 5 6 7 8 9 10 11 12

Days in month,D (output) 31 28 31 30 31 30 31 31 30 31 30 31

Table 3.3

Table 3.4 de�nes a functionQ = g (n) .Remember, this notation tells us that g is the name of the functionthat takes the inputn and gives the outputQ .

n 1 2 3 4 5

Q 8 6 7 6 8

Table 3.4

Table 3.5 displays the age of children in years and their corresponding heights. This table displays justsome of the data available for the heights and ages of children. We can see right away that this table doesnot represent a function because the same input value, 5 years, has two di�erent output values, 40 in. and42 in.

Age in years, a (input) 5 5 6 7 8 9 10

Height in inches, h (output) 40 42 44 47 50 52 54

Table 3.5

How To: Given a table of input and output values, determine whether the tablerepresents a function.

1.Identify the input and output values.2.Check to see if each input value is paired with only one output value. If so, the table representsa function.

Example 3.5Identifying Tables that Represent FunctionsWhich table, Table 3.6, Table 3.7, or Table 3.8, represents a function (if any)?

Input Output

2 1

5 3

8 6

Table 3.6

Available for free at Connexions <http://legacy.cnx.org/content/col11759/1.2>

281

Input Output

�3 5

0 1

4 5

Table 3.7

Input Output

1 0

5 2

5 4

Table 3.8

SolutionTable 3.6 and Table 3.7 de�ne functions. In both, each input value corresponds to exactly oneoutput value. Table 3.8 does not de�ne a function because the input value of 5 corresponds to twodi�erent output values.

When a table represents a function, corresponding input and output values can also be speci�edusing function notation.

The function represented by Table 3.6 can be represented by writing

f (2) = 1, f (5) = 3, and f (8) = 6 (3.4)

Similarly, the statements

g (−3) = 5, g (0) = 1, and g (4) = 5 (3.5)

represent the function in Table 3.7.Table 3.8 cannot be expressed in a similar way because it does not represent a function.

Try It:

Exercise 3.2.3 (Solution on p. 327.)

Does Table 3.9 represent a function?

Input Output

1 10

2 100

3 1000

Table 3.9

Available for free at Connexions <http://legacy.cnx.org/content/col11759/1.2>

282 CHAPTER 3. FUNCTIONS

3.2.2 Finding Input and Output Values of a Function

When we know an input value and want to determine the corresponding output value for a function, weevaluate the function. Evaluating will always produce one result because each input value of a functioncorresponds to exactly one output value.

When we know an output value and want to determine the input values that would produce that outputvalue, we set the output equal to the function's formula and solve for the input. Solving can produce morethan one solution because di�erent input values can produce the same output value.

3.2.2.1 Evaluation of Functions in Algebraic Forms

When we have a function in formula form, it is usually a simple matter to evaluate the function. Forexample, the function f (x) = 5 − 3x2 can be evaluated by squaring the input value, multiplying by 3, andthen subtracting the product from 5.

How To: Given the formula for a function, evaluate.

1.Replace the input variable in the formula with the value provided.2.Calculate the result.

Example 3.6Evaluating Functions at Speci�c ValuesEvaluate f (x) = x2 + 3x− 4 at

a. 2b. ac. a+ hd. f(a+h)−f(a)

h

SolutionReplace thex in the function with each speci�ed value.

a. Because the input value is a number, 2, we can use simple algebra to simplify.

f (2) = 22 + 3 (2)− 4

= 4 + 6− 4

= 6

(3.6)

b. In this case, the input value is a letter so we cannot simplify the answer any further.

f (a) = a2 + 3a− 4 (3.7)

c. With an input value of a+ h, we must use the distributive property.

f (a+ h) = (a+ h)2 + 3 (a+ h)− 4

= a2 + 2ah+ h2 + 3a+ 3h− 4(3.8)

d. In this case, we apply the input values to the function more than once, and then performalgebraic operations on the result. We already found that

f (a+ h) = a2 + 2ah+ h2 + 3a+ 3h− 4 (3.9)

and we know that

Available for free at Connexions <http://legacy.cnx.org/content/col11759/1.2>

283

f (a) = a2 + 3a− 4 (3.10)

Now we combine the results and simplify.

f(a+h)−f(a)h = (a2+2ah+h2+3a+3h−4)−(a2+3a−4)

h

= 2ah+h2+3hh

= h(2a+h+3)h Factor out h.

= 2a+ h+ 3 Simplify.

(3.11)

Example 3.7Evaluating FunctionsGiven the functionh (p) = p2 + 2p, evaluateh (4) .

SolutionTo evaluateh (4) , we substitute the value 4 for the input variable p in the given function.

h (p) = p2 + 2p

h (4) = (4)2 + 2 (4)

= 16 + 8

= 24

(3.12)

Therefore, for an input of 4, we have an output of 24.

Try It:

Exercise 3.2.4 (Solution on p. 327.)

Given the function g (m) =√m− 4, evaluate g (5) .

Example 3.8Solving FunctionsGiven the functionh (p) = p2 + 2p, solve forh (p) = 3.

Solution

h (p) = 3

p2 + 2p = 3 Substitute the original function h (p) = p2 + 2p.

p2 + 2p− 3 = 0 Subtract 3 from each side

(p+ 3)(p− 1) = 0 Factor.

(3.13)

If (p+ 3) (p− 1) = 0, either (p+ 3) = 0 or (p− 1) = 0 (or both of them equal 0). We will set eachfactor equal to 0 and solve for p in each case.

Available for free at Connexions <http://legacy.cnx.org/content/col11759/1.2>

284 CHAPTER 3. FUNCTIONS

(p+ 3) = 0, p = −3

(p− 1) = 0, p = 1(3.14)

This gives us two solutions. The outputh (p) = 3when the input is either p = 1 or p = −3.We canalso verify by graphing as in Figure 3.6. The graph veri�es thath (1) = h (−3) = 3 andh (4) = 24.

Figure 3.6

Available for free at Connexions <http://legacy.cnx.org/content/col11759/1.2>

285

Try It:

Exercise 3.2.5 (Solution on p. 327.)

Given the function g (m) =√m− 4, solve g (m) = 2.

3.2.2.2 Evaluating Functions Expressed in Formulas

Some functions are de�ned by mathematical rules or procedures expressed in equation form. If it is possibleto express the function output with a formula involving the input quantity, then we can de�ne a functionin algebraic form. For example, the equation 2n + 6p = 12 expresses a functional relationship betweennand p.We can rewrite it to decide if p is a function ofn.

How To: Given a function in equation form, write its algebraic formula.

1.Solve the equation to isolate the output variable on one side of the equal sign, with the otherside as an expression that involves only the input variable.

2.Use all the usual algebraic methods for solving equations, such as adding or subtracting thesame quantity to or from both sides, or multiplying or dividing both sides of the equation bythe same quantity.

Example 3.9Finding an Equation of a FunctionExpress the relationship 2n+ 6p = 12 as a function p = f (n) , if possible.

SolutionTo express the relationship in this form, we need to be able to write the relationship where p is afunction ofn, which means writing it as p = [expression involvingn] .

2n+ 6p = 12

6p = 12− 2n Subtract 2n from both sides.

p = 12−2n6 Divide both sides by 6 and simplify.

p = 126 −

2n6

p = 2− 13n

(3.15)

Therefore, p as a function ofn is written as

p = f (n) = 2− 13n (3.16)

Example 3.10Expressing the Equation of a Circle as a FunctionDoes the equationx2 + y2 = 1 represent a function withx as input and y as output? If so, expressthe relationship as a function y = f (x) .

SolutionFirst we subtractx2 from both sides.

y2 = 1− x2 (3.17)

Available for free at Connexions <http://legacy.cnx.org/content/col11759/1.2>

286 CHAPTER 3. FUNCTIONS

We now try to solve for y in this equation.

y = ±√

1− x2

= +√

1− x2 and −√

1− x2(3.18)

We get two outputs corresponding to the same input, so this relationship cannot be representedas a single function y = f (x) . If we graph both functions on a graphing calculator, we will get theupper and lower semicircles.

Try It:

Exercise 3.2.6 (Solution on p. 327.)

Ifx− 8y3 = 0, express y as a function ofx.

Q&A: Are there relationships expressed by an equation that do represent a functionbut that still cannot be represented by an algebraic formula?

Yes, this can happen. For example, given the equationx = y+ 2y, if we want to express y as a func-tion ofx, there is no simple algebraic formula involving onlyx that equals y.However, eachx doesdetermine a unique value for y, and there are mathematical procedures by which y can be found toany desired accuracy. In this case, we say that the equation gives an implicit (implied) rule for y asa function ofx, even though the formula cannot be written explicitly.

3.2.2.3 Evaluating a Function Given in Tabular Form

As we saw above, we can represent functions in tables. Conversely, we can use information in tables to writefunctions, and we can evaluate functions using the tables. For example, how well do our pets recall the fondmemories we share with them? There is an urban legend that a gold�sh has a memory of 3 seconds, but thisis just a myth. Gold�sh can remember up to 3 months, while the beta �sh has a memory of up to 5 months.And while a puppy's memory span is no longer than 30 seconds, the adult dog can remember for 5 minutes.This is meager compared to a cat, whose memory span lasts for 16 hours.

The function that relates the type of pet to the duration of its memory span is more easily visualizedwith the use of a table. See Table 3.10.4

Pet Memory span in hours

Puppy 0.008

Adult dog 0.083

Cat 16

continued on next page

4http://www.kgbanswers.com/how-long-is-a-dogs-memory-span/4221590. Accessed 3/24/2014.

Available for free at Connexions <http://legacy.cnx.org/content/col11759/1.2>

287

Gold�sh 2160

Beta �sh 3600

Table 3.10

At times, evaluating a function in table form may be more useful than using equations. Here let us callthe function P. The domain of the function is the type of pet and the range is a real number representingthe number of hours the pet's memory span lasts. We can evaluate the functionP at the input value of�gold�sh.� We would write P (gold�sh) = 2160. Notice that, to evaluate the function in table form, weidentify the input value and the corresponding output value from the pertinent row of the table. The tabularform for functionP seems ideally suited to this function, more so than writing it in paragraph or functionform.

How To: Given a function represented by a table, identify speci�c output and inputvalues.

1.Find the given input in the row (or column) of input values.2.Identify the corresponding output value paired with that input value.3.Find the given output values in the row (or column) of output values, noting every time thatoutput value appears.

4.Identify the input value(s) corresponding to the given output value.

Example 3.11Evaluating and Solving a Tabular FunctionUsing Table 3.11,

a. Evaluate g (3) .b. Solve g (n) = 6.

n 1 2 3 4 5

g (n) 8 6 7 6 8

Table 3.11

Solution

a. Evaluating g (3) means determining the output value of the function g for the input value ofn = 3. The table output value corresponding to n = 3 is 7, so g (3) = 7.

b. Solving g (n) = 6 means identifying the input values, n,that produce an output value of 6.Table 3.12 shows two solutions: n = 2 and n = 4.

n 1 2 3 4 5

g (n) 8 6 7 6 8

Table 3.12

When we input 2 into the function g, our output is 6. When we input 4 into the function g, ouroutput is also 6.

Available for free at Connexions <http://legacy.cnx.org/content/col11759/1.2>

288 CHAPTER 3. FUNCTIONS

Try It:

Exercise 3.2.7 (Solution on p. 327.)

Using Table 3.11, evaluate g (1) .

3.2.2.4 Finding Function Values from a Graph

Evaluating a function using a graph also requires �nding the corresponding output value for a given inputvalue, only in this case, we �nd the output value by looking at the graph. Solving a function equation usinga graph requires �nding all instances of the given output value on the graph and observing the correspondinginput value(s).

Example 3.12Reading Function Values from a GraphGiven the graph in Figure 3.7,

a. Evaluate f (2) .b. Solve f (x) = 4.

Available for free at Connexions <http://legacy.cnx.org/content/col11759/1.2>

289

Figure 3.7

Solution

a. To evaluate f (2) , locate the point on the curve wherex = 2, then read the y-coordinate ofthat point. The point has coordinates (2, 1) , so f (2) = 1. See Figure 3.8.

Available for free at Connexions <http://legacy.cnx.org/content/col11759/1.2>

290 CHAPTER 3. FUNCTIONS

Figure 3.8

b. To solve f (x) = 4, we �nd the output value 4 on the vertical axis. Moving horizontally alongthe line y = 4, we locate two points of the curve with output value 4 : (−1, 4) and (3, 4) .Thesepoints represent the two solutions to f (x) = 4 : x = −1 orx = 3.This means f (−1) =4 and f (3) = 4, or when the input is − 1 or3, the output is 4. See Figure 3.9.

Available for free at Connexions <http://legacy.cnx.org/content/col11759/1.2>

291

Figure 3.9

Try It:

Exercise 3.2.8 (Solution on p. 327.)

Using Figure 3.7, solve f (x) = 1.

3.2.3 Determining Whether a Function is One-to-One

Some functions have a given output value that corresponds to two or more input values. For example, in thestock chart shown in Figure 3.1 at the beginning of this chapter, the stock price was $1000 on �ve di�erent

Available for free at Connexions <http://legacy.cnx.org/content/col11759/1.2>

292 CHAPTER 3. FUNCTIONS

dates, meaning that there were �ve di�erent input values that all resulted in the same output value of $1000.However, some functions have only one input value for each output value, as well as having only one

output for each input. We call these functions one-to-one functions. As an example, consider a school thatuses only letter grades and decimal equivalents, as listed in Table 3.13.

Letter grade Grade point average

A 4.0

B 3.0

C 2.0

D 1.0

Table 3.13

This grading system represents a one-to-one function because each letter input yields one particulargrade-point average output and each grade-point average corresponds to one input letter.

To visualize this concept, let's look again at the two simple functions sketched in Figure 3.2(a) andFigure 3.2(b). The function in part (a) shows a relationship that is not a one-to-one function becauseinputs q and r both give outputn.The function in part (b) shows a relationship that is a one-to-one functionbecause each input is associated with a single output.

A General Note: A one-to-one function is a function in which each output value correspondsto exactly one input value. There are no repeated x- or y-values.

Example 3.13Determining Whether a Relationship Is a One-to-One FunctionIs the area of a circle a function of its radius? If yes, is the function one-to-one?

SolutionA circle of radius r has a unique area measure given byA = πr2,so for any input, r, there is onlyone output, A. The area is a function of radius r.

If the function is one-to-one, the output value, the area, must correspond to a unique inputvalue, the radius. Any area measureA is given by the formulaA = πr2.Because areas and radii

are positive numbers, there is exactly one solution:r =√

Aπ .So the area of a circle is a one-to-one

function of the circle's radius.

Try It:

Exercise 3.2.9 (Solution on p. 327.)

a.Is a balance a function of the bank account number?b.Is a bank account number a function of the balance?c.Is a balance a one-to-one function of the bank account number?

Try It:

Exercise 3.2.10 (Solution on p. 327.)

a.If each percent grade earned in a course translates to one letter grade, is the lettergrade a function of the percent grade?

b.If so, is the function one-to-one?

Available for free at Connexions <http://legacy.cnx.org/content/col11759/1.2>

293

3.2.4 Using the Vertical Line Test

As we have seen in some examples above, we can represent a function using a graph. Graphs display a greatmany input-output pairs in a small space. The visual information they provide often makes relationshipseasier to understand. By convention, graphs are typically constructed with the input values along thehorizontal axis and the output values along the vertical axis.

The most common graphs name the input valuex and the output value y, and we say y is a functionofx, or y = f (x) when the function is named f.The graph of the function is the set of all points (x, y) inthe plane that satis�es the equationy = f (x) . If the function is de�ned for only a few input values, then thegraph of the function consists of only a few points, where the x-coordinate of each point is an input value andthe y-coordinate of each point is the corresponding output value. For example, the black dots on the graphin Figure 3.10 tell us that f (0) = 2 and f (6) = 1.However, the set of all points (x, y) satisfying y = f (x) isa curve. The curve shown includes (0, 2) and (6, 1) because the curve passes through those points.

Figure 3.10

The vertical line test can be used to determine whether a graph represents a function. If we can drawany vertical line that intersects a graph more than once, then the graph does not de�ne a function becausea function has only one output value for each input value. See Figure 3.11.

Available for free at Connexions <http://legacy.cnx.org/content/col11759/1.2>

294 CHAPTER 3. FUNCTIONS

Figure 3.11

How To: Given a graph, use the vertical line test to determine if the graph representsa function.

1.Inspect the graph to see if any vertical line drawn would intersect the curve more than once.2.If there is any such line, determine that the graph does not represent a function.

Example 3.14Applying the Vertical Line TestWhich of the graphs in Figure 3.12 represent(s) a function y = f (x)?

Figure 3.12

SolutionIf any vertical line intersects a graph more than once, the relation represented by the graph is not afunction. Notice that any vertical line would pass through only one point of the two graphs shown

Available for free at Connexions <http://legacy.cnx.org/content/col11759/1.2>

295

in parts (a) and (b) of Figure 3.12. From this we can conclude that these two graphs representfunctions. The third graph does not represent a function because, at most x-values, a vertical linewould intersect the graph at more than one point, as shown in Figure 3.13.

Figure 3.13

Try It:

Exercise 3.2.11 (Solution on p. 327.)

Does the graph in Figure 3.14 represent a function?

Available for free at Connexions <http://legacy.cnx.org/content/col11759/1.2>

296 CHAPTER 3. FUNCTIONS

Figure 3.14

3.2.5 Using the Horizontal Line Test

Once we have determined that a graph de�nes a function, an easy way to determine if it is a one-to-onefunction is to use the horizontal line test. Draw horizontal lines through the graph. If any horizontal lineintersects the graph more than once, then the graph does not represent a one-to-one function.

How To: Given a graph of a function, use the horizontal line test to determine if thegraph represents a one-to-one function.

1.Inspect the graph to see if any horizontal line drawn would intersect the curve more than once.2.If there is any such line, determine that the function is not one-to-one.

Example 3.15Applying the Horizontal Line TestConsider the functions shown in Figure 3.12(a) and Figure 3.12(b). Are either of the functionsone-to-one?

Available for free at Connexions <http://legacy.cnx.org/content/col11759/1.2>

297

SolutionThe function in Figure 3.12(a) is not one-to-one. The horizontal line shown in Figure 3.15 intersectsthe graph of the function at two points (and we can even �nd horizontal lines that intersect it atthree points.)

Figure 3.15

The function in Figure 3.12(b) is one-to-one. Any horizontal line will intersect a diagonal lineat most once.

Try It:

Available for free at Connexions <http://legacy.cnx.org/content/col11759/1.2>

298 CHAPTER 3. FUNCTIONS

Exercise 3.2.12 (Solution on p. 327.)

Is the graph shown in Figure 3.13 one-to-one?

3.2.6 Identifying Basic Toolkit Functions

In this text, we will be exploring functions�the shapes of their graphs, their unique characteristics, theiralgebraic formulas, and how to solve problems with them. When learning to read, we start with the alphabet.When learning to do arithmetic, we start with numbers. When working with functions, it is similarly helpfulto have a base set of building-block elements. We call these our �toolkit functions,� which form a set of basicnamed functions for which we know the graph, formula, and special properties. Some of these functionsare programmed to individual buttons on many calculators. For these de�nitions we will usex as the inputvariable and y = f (x) as the output variable.

We will see these toolkit functions, combinations of toolkit functions, their graphs, and their transforma-tions frequently throughout this book. It will be very helpful if we can recognize these toolkit functions andtheir features quickly by name, formula, graph, and basic table properties. The graphs and sample tablevalues are included with each function shown in Table 3.14.

Toolkit Functions

Name Function Graph

Constant f (x) = c, where c is a constant

continued on next page

Available for free at Connexions <http://legacy.cnx.org/content/col11759/1.2>

299

Identity f (x) = x

continued on next page

Available for free at Connexions <http://legacy.cnx.org/content/col11759/1.2>

300 CHAPTER 3. FUNCTIONS

Absolute value f (x) = |x|

continued on next page

Available for free at Connexions <http://legacy.cnx.org/content/col11759/1.2>

301

Quadratic f (x) = x2

continued on next page

Available for free at Connexions <http://legacy.cnx.org/content/col11759/1.2>

302 CHAPTER 3. FUNCTIONS

Cubic f (x) = x3

continued on next page

Available for free at Connexions <http://legacy.cnx.org/content/col11759/1.2>

303

Reciprocal f (x) = 1x

continued on next page

Available for free at Connexions <http://legacy.cnx.org/content/col11759/1.2>

304 CHAPTER 3. FUNCTIONS

Reciprocal squared f (x) = 1x2

continued on next page

Available for free at Connexions <http://legacy.cnx.org/content/col11759/1.2>

305

Square root f (x) =√x

continued on next page

Available for free at Connexions <http://legacy.cnx.org/content/col11759/1.2>

306 CHAPTER 3. FUNCTIONS

Cube root f (x) = 3√x

Table 3.14

Media: Access the following online resources for additional instruction and practice with functions.

• Determine if a Relation is a Function5

• Vertical Line Test6• Introduction to Functions7

• Vertical Line Test on Graph8

• One-to-one Functions9• Graphs as One-to-one Functions10

5http://openstaxcollege.org/l/relationfunction6http://openstaxcollege.org/l/vertlinetest7http://openstaxcollege.org/l/introtofunction8http://openstaxcollege.org/l/vertlinegraph9http://openstaxcollege.org/l/onetoone

10http://openstaxcollege.org/l/graphonetoone

Available for free at Connexions <http://legacy.cnx.org/content/col11759/1.2>

307

3.2.7 Key Equations

Constant function f (x) = c,where c is a constant

Identity function f (x) = x

Absolute value function f (x) = |x|Quadratic function f (x) = x2

Cubic function f (x) = x3

Reciprocal function f (x) = 1x

Reciprocal squared function f (x) = 1x2

Square root function f (x) =√x

Cube root function f (x) = 3√x

Table 3.15

3.2.8 Key Concepts

• A relation is a set of ordered pairs. A function is a speci�c type of relation in which each domain value,or input, leads to exactly one range value, or output. See Example 3.1 and Example 3.2.

• Function notation is a shorthand method for relating the input to the output in the form y = f (x) . SeeExample 3.3 and Example 3.4.

• In tabular form, a function can be represented by rows or columns that relate to input and outputvalues. See Example 3.5.

• To evaluate a function, we determine an output value for a corresponding input value. Algebraic formsof a function can be evaluated by replacing the input variable with a given value. See Example 3.6 andExample 3.7.

• To solve for a speci�c function value, we determine the input values that yield the speci�c output value.See Example 3.8.

• An algebraic form of a function can be written from an equation. See Example 3.9 and Example 3.10.• Input and output values of a function can be identi�ed from a table. See Example 3.11.• Relating input values to output values on a graph is another way to evaluate a function. See Exam-

ple 3.12.• A function is one-to-one if each output value corresponds to only one input value. See Example 3.13.• A graph represents a function if any vertical line drawn on the graph intersects the graph at no more

than one point. See Example 3.14.• The graph of a one-to-one function passes the horizontal line test. See Example 3.15.

3.2.9 Section Exercises

3.2.9.1 Verbal

Exercise 3.2.13 (Solution on p. 327.)

What is the di�erence between a relation and a function?

Exercise 3.2.14What is the di�erence between the input and the output of a function?

Exercise 3.2.15 (Solution on p. 327.)

Why does the vertical line test tell us whether the graph of a relation represents a function?

Available for free at Connexions <http://legacy.cnx.org/content/col11759/1.2>

308 CHAPTER 3. FUNCTIONS

Exercise 3.2.16How can you determine if a relation is a one-to-one function?

Exercise 3.2.17 (Solution on p. 327.)

Why does the horizontal line test tell us whether the graph of a function is one-to-one?

3.2.9.2 Algebraic

For the following exercises, determine whether the relation represents a function.

Exercise 3.2.18{(a, b) , (c, d) , (a, c)}Exercise 3.2.19 (Solution on p. 327.)

{(a, b) , (b, c) , (c, c)}For the following exercises, determine whether the relation represents y as a function ofx.

Exercise 3.2.205x+ 2y = 10Exercise 3.2.21 (Solution on p. 327.)

y = x2

Exercise 3.2.22x = y2

Exercise 3.2.23 (Solution on p. 327.)

3x2 + y = 14Exercise 3.2.242x+ y2 = 6Exercise 3.2.25 (Solution on p. 327.)

y = −2x2 + 40xExercise 3.2.26y = 1

x

Exercise 3.2.27 (Solution on p. 327.)

x = 3y+57y−1

Exercise 3.2.28x =

√1− y2

Exercise 3.2.29 (Solution on p. 328.)

y = 3x+57x−1

Exercise 3.2.30x2 + y2 = 9Exercise 3.2.31 (Solution on p. 328.)

2xy = 1Exercise 3.2.32x = y3

Exercise 3.2.33 (Solution on p. 328.)

y = x3

Exercise 3.2.34y =√

1− x2

Exercise 3.2.35 (Solution on p. 328.)

x = ±√

1− y

Available for free at Connexions <http://legacy.cnx.org/content/col11759/1.2>

309

Exercise 3.2.36y = ±

√1− x

Exercise 3.2.37 (Solution on p. 328.)

y2 = x2

Exercise 3.2.38y3 = x2

For the following exercises, evaluate the function f at the indicatedvalues f (−3) , f (2) , f (−a) ,−f (a) , f (a+ h) .

Exercise 3.2.39 (Solution on p. 328.)

f (x) = 2x− 5Exercise 3.2.40f (x) = −5x2 + 2x− 1Exercise 3.2.41 (Solution on p. 328.)

f (x) =√

2− x+ 5Exercise 3.2.42f (x) = 6x−1

5x+2

Exercise 3.2.43 (Solution on p. 328.)

f (x) = |x− 1| − |x+ 1|Exercise 3.2.44Given the function g (x) = 5− x2, simplify g(x+h)−g(x)

h , h 6= 0.Exercise 3.2.45 (Solution on p. 328.)

Given the function g (x) = x2 + 2x, simplify g(x)−g(a)x−a , x 6= a.

Exercise 3.2.46Given the function k (t) = 2t− 1:

a. Evaluate k (2) .b. Solve k (t) = 7.

Exercise 3.2.47 (Solution on p. 328.)

Given the function f (x) = 8− 3x:

a. Evaluate f (−2) .b. Solve f (x) = −1.

Exercise 3.2.48Given the function p (c) = c2 + c:

a. Evaluate p (−3) .b. Solve p (c) = 2.

Exercise 3.2.49 (Solution on p. 328.)

Given the function f (x) = x2 − 3x:

a. Evaluate f (5) .b. Solve f (x) = 4.

Exercise 3.2.50Given the function f (x) =

√x+ 2:

a. Evaluate f (7) .

Available for free at Connexions <http://legacy.cnx.org/content/col11759/1.2>

310 CHAPTER 3. FUNCTIONS

b. Solve f (x) = 4.

Exercise 3.2.51 (Solution on p. 328.)

Consider the relationship 3r + 2t = 18.

a. Write the relationship as a function r = f (t) .b. Evaluate f (−3) .c. Solve f (t) = 2.

3.2.9.3 Graphical

For the following exercises, use the vertical line test to determine which graphs show relations that arefunctions.

Exercise 3.2.52

Available for free at Connexions <http://legacy.cnx.org/content/col11759/1.2>

311

Exercise 3.2.53 (Solution on p. 328.)

Available for free at Connexions <http://legacy.cnx.org/content/col11759/1.2>

312 CHAPTER 3. FUNCTIONS

Exercise 3.2.54

Available for free at Connexions <http://legacy.cnx.org/content/col11759/1.2>

313

Exercise 3.2.55 (Solution on p. 328.)

Exercise 3.2.56

Available for free at Connexions <http://legacy.cnx.org/content/col11759/1.2>

314 CHAPTER 3. FUNCTIONS

Exercise 3.2.57 (Solution on p. 328.)

Available for free at Connexions <http://legacy.cnx.org/content/col11759/1.2>

315

Exercise 3.2.58

Available for free at Connexions <http://legacy.cnx.org/content/col11759/1.2>

316 CHAPTER 3. FUNCTIONS

Exercise 3.2.59 (Solution on p. 328.)

Available for free at Connexions <http://legacy.cnx.org/content/col11759/1.2>

317

Exercise 3.2.60

Available for free at Connexions <http://legacy.cnx.org/content/col11759/1.2>

318 CHAPTER 3. FUNCTIONS

Exercise 3.2.61 (Solution on p. 328.)

Exercise 3.2.62

Available for free at Connexions <http://legacy.cnx.org/content/col11759/1.2>

319

Exercise 3.2.63 (Solution on p. 328.)

Exercise 3.2.64Given the following graph,

• Evaluate f (−1) .• Solve for f (x) = 3.

Exercise 3.2.65 (Solution on p. 328.)

Given the following graph,

• Evaluate f (0) .• Solve for f (x) = −3.

Available for free at Connexions <http://legacy.cnx.org/content/col11759/1.2>

320 CHAPTER 3. FUNCTIONS

Exercise 3.2.66Given the following graph,

• Evaluate f (4) .• Solve for f (x) = 1.

Available for free at Connexions <http://legacy.cnx.org/content/col11759/1.2>

321

For the following exercises, determine if the given graph is a one-to-one function.Exercise 3.2.67 (Solution on p. 328.)

Exercise 3.2.68

Available for free at Connexions <http://legacy.cnx.org/content/col11759/1.2>

322 CHAPTER 3. FUNCTIONS

Exercise 3.2.69 (Solution on p. 328.)

Exercise 3.2.70

Available for free at Connexions <http://legacy.cnx.org/content/col11759/1.2>

323

Exercise 3.2.71 (Solution on p. 328.)

3.2.9.4 Numeric

For the following exercises, determine whether the relation represents a function.

Exercise 3.2.72{(−1,−1) , (−2,−2) , (−3,−3)}Exercise 3.2.73 (Solution on p. 328.)

{(3, 4) , (4, 5) , (5, 6)}Exercise 3.2.74{(2, 5) , (7, 11) , (15, 8) , (7, 9)}

For the following exercises, determine if the relation represented in table form represents y as a function ofx.

Exercise 3.2.75 (Solution on p. 328.)

x 5 10 15

y 3 8 14

Table 3.16

Exercise 3.2.76

x 5 10 15

continued on next page

Available for free at Connexions <http://legacy.cnx.org/content/col11759/1.2>

324 CHAPTER 3. FUNCTIONS

y 3 8 8

Table 3.17

Exercise 3.2.77 (Solution on p. 329.)

x 5 10 10

y 3 8 14

Table 3.18

For the following exercises, use the function f represented in Table 3.19.

x 0 1 2 3 4 5 6 7 8 9

f (x) 74 28 1 53 56 3 36 45 14 47

Table 3.19

Exercise 3.2.78Evaluate f (3) .Exercise 3.2.79 (Solution on p. 329.)

Solve f (x) = 1.For the following exercises, evaluate the function f at the valuesf (−2) , f (−1) , f (0) , f (1) ,and f (2) .

Exercise 3.2.80f (x) = 4− 2xExercise 3.2.81 (Solution on p. 329.)

f (x) = 8− 3xExercise 3.2.82f (x) = 8x2 − 7x+ 3Exercise 3.2.83 (Solution on p. 329.)

f (x) = 3 +√x+ 3

Exercise 3.2.84f (x) = x−2

x+3

Exercise 3.2.85 (Solution on p. 329.)

f (x) = 3x

For the following exercises, evaluate the expressions, given functionsf, g,andh:

• f (x) = 3x− 2• g (x) = 5− x2

• h (x) = −2x2 + 3x− 1

Exercise 3.2.863f (1)− 4g (−2)Exercise 3.2.87 (Solution on p. 329.)

f(

73

)− h (−2)

Available for free at Connexions <http://legacy.cnx.org/content/col11759/1.2>

325

3.2.9.5 Technology

For the following exercises, graph y = x2 on the given viewing window. Determine the corresponding rangefor each viewing window. Show each graph.

Exercise 3.2.88[−0.1, 0.1]Exercise 3.2.89 (Solution on p. 329.)

[−10, 10]Exercise 3.2.90[−100, 100]

For the following exercises, graph y = x3 on the given viewing window. Determine the corresponding rangefor each viewing window. Show each graph.

Exercise 3.2.91 (Solution on p. 329.)

[−0.1, 0.1]Exercise 3.2.92[−10, 10]Exercise 3.2.93 (Solution on p. 330.)

[−100, 100]For the following exercises, graph y =

√x on the given viewing window. Determine the corresponding range

for each viewing window. Show each graph.

Exercise 3.2.94[0, 0.01]Exercise 3.2.95 (Solution on p. 331.)

[0, 100]Exercise 3.2.96[0, 10,000]

For the following exercises, graph y = 3√x on the given viewing window. Determine the corresponding range

for each viewing window. Show each graph.

Exercise 3.2.97 (Solution on p. 332.)

[−0.001, 0.001]Exercise 3.2.98[−1000, 1000]Exercise 3.2.99 (Solution on p. 333.)

[−1, 000, 000, 1,000,000]

3.2.9.6 Real-World Applications

Exercise 3.2.100The amount of garbage,G, produced by a city with population p is given byG = f (p) . G is mea-sured in tons per week, and p is measured in thousands of people.

a. The town of Tola has a population of 40,000 and produces 13 tons of garbage each week.Express this information in terms of the function f.

b. Explain the meaning of the statement f (5) = 2.

Exercise 3.2.101 (Solution on p. 334.)

The number of cubic yards of dirt,D, needed to cover a garden with area a square feet is givenbyD = g (a) .

Available for free at Connexions <http://legacy.cnx.org/content/col11759/1.2>

326 CHAPTER 3. FUNCTIONS

a. A garden with area 5000 ft2 requires 50 yd3 of dirt. Express this information in terms of thefunction g.

b. Explain the meaning of the statement g (100) = 1.

Exercise 3.2.102Let f (t) be the number of ducks in a lake t years after 1990. Explain the meaning of each statement:

a. f (5) = 30b. f (10) = 40

Exercise 3.2.103 (Solution on p. 334.)

Leth (t) be the height above ground, in feet, of a rocket t seconds after launching. Explain themeaning of each statement:

a. h (1) = 200b. h (2) = 350

Exercise 3.2.104Show that the function f (x) = 3(x− 5)2 + 7 is not one-to-one.

3.3 Domain and Range11

3.4 Rates of Change and Behavior of Graphs12

3.5 Composition of Functions13

3.6 Transformation of Functions14

3.7 Absolute Value Functions15

3.8 Inverse Functions16

11This content is available online at <http://legacy.cnx.org/content/m51262/1.1/>.12This content is available online at <http://legacy.cnx.org/content/m51263/1.1/>.13This content is available online at <http://legacy.cnx.org/content/m51265/1.1/>.14This content is available online at <http://legacy.cnx.org/content/m51266/1.1/>.15This content is available online at <http://legacy.cnx.org/content/m51267/1.1/>.16This content is available online at <http://legacy.cnx.org/content/m51268/1.1/>.

Available for free at Connexions <http://legacy.cnx.org/content/col11759/1.2>

327

Solutions to Exercises in Chapter 3

Solution to Exercise 3.2.1 (p. 277)a. yes; b. yes. (Note: If two players had been tied for, say, 4th place, then the name would not have been afunction of rank.)Solution to Exercise 3.2.2 (p. 279)w = f (d)Solution to Exercise 3.2.3 (p. 281)yesSolution to Exercise 3.2.4 (p. 283)g (5) = 1Solution to Exercise 3.2.5 (p. 285)m = 8Solution to Exercise 3.2.6 (p. 286)

y = f (x) =3√x2

Solution to Exercise 3.2.7 (p. 288)g (1) = 8Solution to Exercise 3.2.8 (p. 291)x = 0 orx = 2Solution to Exercise 3.2.9 (p. 292)a. yes, because each bank account has a single balance at any given time; b. no, because several bankaccount numbers may have the same balance; c. no, because the same output may correspond to more thanone input.Solution to Exercise 3.2.10 (p. 292)

a. Yes, letter grade is a function of percent grade;b. No, it is not one-to-one. There are 100 di�erent percent numbers we could get but only about �ve

possible letter grades, so there cannot be only one percent number that corresponds to each lettergrade.

Solution to Exercise 3.2.11 (p. 295)yesSolution to Exercise 3.2.12 (p. 297)No, because it does not pass the horizontal line test.Solution to Exercise 3.2.13 (p. 307)A relation is a set of ordered pairs. A function is a special kind of relation in which no two ordered pairshave the same �rst coordinate.Solution to Exercise 3.2.15 (p. 307)When a vertical line intersects the graph of a relation more than once, that indicates that for that inputthere is more than one output. At any particular input value, there can be only one output if the relationis to be a function.Solution to Exercise 3.2.17 (p. 308)When a horizontal line intersects the graph of a function more than once, that indicates that for that outputthere is more than one input. A function is one-to-one if each output corresponds to only one input.Solution to Exercise 3.2.19 (p. 308)functionSolution to Exercise 3.2.21 (p. 308)functionSolution to Exercise 3.2.23 (p. 308)functionSolution to Exercise 3.2.25 (p. 308)function

Available for free at Connexions <http://legacy.cnx.org/content/col11759/1.2>

328 CHAPTER 3. FUNCTIONS

Solution to Exercise 3.2.27 (p. 308)functionSolution to Exercise 3.2.29 (p. 308)functionSolution to Exercise 3.2.31 (p. 308)functionSolution to Exercise 3.2.33 (p. 308)functionSolution to Exercise 3.2.35 (p. 308)functionSolution to Exercise 3.2.37 (p. 309)not a functionSolution to Exercise 3.2.39 (p. 309)

f (−3) = −11; f (2) = −1; f (−a) = −2a− 5; −f (a) = −2a+ 5; f (a+ h) = 2a+ 2h− 5Solution to Exercise 3.2.41 (p. 309)

f (−3) =√

5 + 5; f (2) = 5; f (−a) =√

2 + a+ 5; −f (a) = −√

2− a− 5; f (a+ h) =√

2− a− h+5Solution to Exercise 3.2.43 (p. 309)

f (−3) = 2; f (2) = 1− 3 = −2; f (−a) = | − a− 1| − | − a+ 1|; −f (a) = −|a− 1| + |a+ 1|; f (a+ h) = |a+ h− 1| − |a+ h+ 1|Solution to Exercise 3.2.45 (p. 309)g(x)−g(a)x−a = x+ a+ 2, x 6= a

Solution to Exercise 3.2.47 (p. 309)a. f (−2) = 14; b.x = 3Solution to Exercise 3.2.49 (p. 309)a. f (5) = 10; b.x = −1 or x = 4Solution to Exercise 3.2.51 (p. 310)a. f (t) = 6− 2

3 t; b. f (−3) = 8; c. t = 6Solution to Exercise 3.2.53 (p. 311)not a functionSolution to Exercise 3.2.55 (p. 313)functionSolution to Exercise 3.2.57 (p. 314)functionSolution to Exercise 3.2.59 (p. 316)functionSolution to Exercise 3.2.61 (p. 318)functionSolution to Exercise 3.2.63 (p. 319)functionSolution to Exercise 3.2.65 (p. 319)a. f (0) = 1; b. f (x) = −3, x = −2 or x = 2Solution to Exercise 3.2.67 (p. 321)not a function so it is also not a one-to-one functionSolution to Exercise 3.2.69 (p. 322)one-to- one functionSolution to Exercise 3.2.71 (p. 323)function, but not one-to-oneSolution to Exercise 3.2.73 (p. 323)function

Available for free at Connexions <http://legacy.cnx.org/content/col11759/1.2>

329

Solution to Exercise 3.2.75 (p. 323)functionSolution to Exercise 3.2.77 (p. 324)not a functionSolution to Exercise 3.2.79 (p. 324)f (x) = 1, x = 2Solution to Exercise 3.2.81 (p. 324)

f (−2) = 14; f (−1) = 11; f (0) = 8; f (1) = 5; f (2) = 2Solution to Exercise 3.2.83 (p. 324)

f (−2) = 4; f (−1) = 4.414; f (0) = 4.732; f (1) = 4.5; f (2) = 5.236Solution to Exercise 3.2.85 (p. 324)

f (−2) = 19 ; f (−1) = 1

3 ; f (0) = 1; f (1) = 3; f (2) = 9Solution to Exercise 3.2.87 (p. 324)20Solution to Exercise 3.2.89 (p. 325)[0, 100]

Figure 3.16

Solution to Exercise 3.2.91 (p. 325)[−0.001, 0.001]

Available for free at Connexions <http://legacy.cnx.org/content/col11759/1.2>

330 CHAPTER 3. FUNCTIONS

Figure 3.17

Solution to Exercise 3.2.93 (p. 325)[−1, 000, 000, 1,000,000]

Available for free at Connexions <http://legacy.cnx.org/content/col11759/1.2>

331

Figure 3.18

Solution to Exercise 3.2.95 (p. 325)[0, 10]

Available for free at Connexions <http://legacy.cnx.org/content/col11759/1.2>

332 CHAPTER 3. FUNCTIONS

Figure 3.19

Solution to Exercise 3.2.97 (p. 325)[−0.1, 0.1]

Available for free at Connexions <http://legacy.cnx.org/content/col11759/1.2>

333

Figure 3.20

Solution to Exercise 3.2.99 (p. 325)[−100, 100]

Available for free at Connexions <http://legacy.cnx.org/content/col11759/1.2>

334 CHAPTER 3. FUNCTIONS

Figure 3.21

Solution to Exercise 3.2.101 (p. 325)a. g (5000) = 50; b. The number of cubic yards of dirt required for a garden of 100 square feet is 1.Solution to Exercise 3.2.103 (p. 326)a. The height of a rocket above ground after 1 second is 200 ft. b. the height of a rocket above ground after2 seconds is 350 ft.

Available for free at Connexions <http://legacy.cnx.org/content/col11759/1.2>

Chapter 4

Linear Functions

4.1 Introduction to Linear Functions1

4.2 Linear Functions2

4.3 Modeling with Linear Functions3

4.4 Fitting Linear Models to Data4

1This content is available online at <http://legacy.cnx.org/content/m51269/1.1/>.2This content is available online at <http://legacy.cnx.org/content/m51270/1.1/>.3This content is available online at <http://legacy.cnx.org/content/m51271/1.1/>.4This content is available online at <http://legacy.cnx.org/content/m51272/1.1/>.

Available for free at Connexions <http://legacy.cnx.org/content/col11759/1.2>

335

336 CHAPTER 4. LINEAR FUNCTIONS

Available for free at Connexions <http://legacy.cnx.org/content/col11759/1.2>

Chapter 5

Polynomial and Rational Functions

5.1 Introduction to Polynomial and Rational Functions1

5.2 Quadratic Functions2

5.3 Power Functions and Polynomial Functions3

5.4 Graphs of Polynomial Functions4

5.5 Dividing Polynomials5

5.6 Zeros of Polynomial Functions6

5.7 Rational Functions7

5.8 Inverses and Radical Functions8

5.9 Modeling Using Variation9

1This content is available online at <http://legacy.cnx.org/content/m51273/1.1/>.2This content is available online at <http://legacy.cnx.org/content/m51274/1.1/>.3This content is available online at <http://legacy.cnx.org/content/m51275/1.1/>.4This content is available online at <http://legacy.cnx.org/content/m51276/1.1/>.5This content is available online at <http://legacy.cnx.org/content/m51277/1.1/>.6This content is available online at <http://legacy.cnx.org/content/m51278/1.1/>.7This content is available online at <http://legacy.cnx.org/content/m51279/1.1/>.8This content is available online at <http://legacy.cnx.org/content/m51280/1.1/>.9This content is available online at <http://legacy.cnx.org/content/m51281/1.1/>.

Available for free at Connexions <http://legacy.cnx.org/content/col11759/1.2>

337

338 CHAPTER 5. POLYNOMIAL AND RATIONAL FUNCTIONS

Available for free at Connexions <http://legacy.cnx.org/content/col11759/1.2>

Chapter 6

Exponential and Logarithmic Functions

6.1 Introduction to Exponential and Logarithmic Functions1

Figure 6.1: Electron micrograph of E.Coli bacteria (credit: �Mattosaurus,� Wikimedia Commons)

1This content is available online at <http://legacy.cnx.org/content/m49356/1.2/>.

Available for free at Connexions <http://legacy.cnx.org/content/col11759/1.2>

339

340 CHAPTER 6. EXPONENTIAL AND LOGARITHMIC FUNCTIONS

Focus in on a square centimeter of your skin. Look closer. Closer still. If you could look closely enough,you would see hundreds of thousands of microscopic organisms. They are bacteria, and they are not onlyon your skin, but in your mouth, nose, and even your intestines. In fact, the bacterial cells in your body atany given moment outnumber your own cells. But that is no reason to feel bad about yourself. While somebacteria can cause illness, many are healthy and even essential to the body.

Bacteria commonly reproduce through a process called binary �ssion, during which one bacterial cellsplits into two. When conditions are right, bacteria can reproduce very quickly. Unlike humans and othercomplex organisms, the time required to form a new generation of bacteria is often a matter of minutes orhours, as opposed to days or years.2

For simplicity's sake, suppose we begin with a culture of one bacterial cell that can divide every hour.Table 6.1 shows the number of bacterial cells at the end of each subsequent hour. We see that the singlebacterial cell leads to over one thousand bacterial cells in just ten hours! And if we were to extrapolate thetable to twenty-four hours, we would have over 16 million!

Hour 0 1 2 3 4 5 6 7 8 9 10

Bacteria 1 2 4 8 16 32 64 128 256 512 1024

Table 6.1

In this chapter, we will explore exponential functions, which can be used for, among other things, modelinggrowth patterns such as those found in bacteria. We will also investigate logarithmic functions, which areclosely related to exponential functions. Both types of functions have numerous real-world applications whenit comes to modeling and interpreting data.

6.2 Exponential Functions3

India is the second most populous country in the world with a population of about 1.25 billion people in2013. The population is growing at a rate of about 1.2% each year4. If this rate continues, the populationof India will exceed China's population by the year 2031.When populations grow rapidly, we often say thatthe growth is �exponential,� meaning that something is growing very rapidly. To a mathematician, however,the term exponential growth has a very speci�c meaning. In this section, we will take a look at exponentialfunctions, which model this kind of rapid growth.

6.2.1 Identifying Exponential Functions

When exploring linear growth, we observed a constant rate of change�a constant number by which theoutput increased for each unit increase in input. For example, in the equation f (x) = 3x + 4,the slopetells us the output increases by 3 each time the input increases by 1. The scenario in the India populationexample is di�erent because we have a percent change per unit time (rather than a constant change) in thenumber of people.

6.2.1.1 De�ning an Exponential Function

A Harris Interactivity study found that the number of vegans in the United States doubled from 2009 to2011. In 2011, 2.5% of the population was vegan, adhering to a diet that does not include any animalproducts�no meat, poultry, �sh, dairy, or eggs. If this rate continues, vegans will make up 10% of the U.S.population in 2015, 40% in 2019, and 80% in 20505.

2Todar, PhD, Kenneth. Todar's Online Textbook of Bacteriology. http://textbookofbacteriology.net/growth_3.html.3This content is available online at <http://legacy.cnx.org/content/m49361/1.5/>.4http://www.worldometers.info/world-population/. Accessed February 24, 2014.5Bohanec, Hope. "U.S. vegan population doubles in only two years." Occupy for Animals.

http://www.occupyforanimals.org/us-vegan-population-doubles-in-only-two-years.html.

Available for free at Connexions <http://legacy.cnx.org/content/col11759/1.2>

341

What exactly does it mean to grow exponentially? What does the word double have in common withpercent increase? People toss these words around errantly. Are these words used correctly? The wordscertainly appear frequently in the media.

• Percent change refers to a change based on a percent of the original amount.• Exponential growth refers to an increase based on a constant multiplicative rate of change over

equal increments of time, that is, a percent increase of the original amount over time.• Exponential decay refers to a decrease based on a constant multiplicative rate of change over equal

increments of time, that is, a percent decrease of the original amount over time.

For us to gain a clear understanding of exponential growth, let us contrast exponential growth withlinear growth. We will construct two functions. The �rst function is exponential. We will start with aninput of 0, and increase each input by 1. We will double the corresponding consecutive outputs. The secondfunction is linear. We will start with an input of 0, and increase each input by 1. We will add 2 to thecorresponding consecutive outputs. See Table 6.2.

x f (x) = 2x g (x) = 2x

0 1 0

1 2 2

2 4 4

3 8 6

4 16 8

5 32 10

6 64 12

Table 6.2

From Table 6.2 we can infer that for these two functions, exponential growth dwarfs linear growth.

• Exponential growth refers to the original value from the range increases by the same percentageover equal increments found in the domain.

• Linear growth refers to the original value from the range increases by the same amount over equalincrements found in the domain.

Apparently, the di�erence between �the same percentage� and �the same amount� is quite signi�cant. Forexponential growth, over equal increments, the constant multiplicative rate of change resulted in doublingthe output whenever the input increased by one. For linear growth, the constant additive rate of changeover equal increments resulted in adding 2 to the output whenever the input was increased by one.

The general form of the exponential function is f (x) = abx, where a is any nonzero number, b is apositive real number not equal to 1.

• If b > 1,the function grows at a rate proportional to its size.• If 0 < b < 1, the function decays at a rate proportional to its size.

Let's look at the function f (x) = 2x from our example. We will create a table (Table 6.3) to determine thecorresponding outputs over an interval in the domain from − 3 to 3.

x −3 −2 −1 0 1 2 3

f (x) = 2x 2−3 = 18 2−2 = 1

4 2−1 = 12 20 = 1 21 = 2 22 = 4 23 = 8

Available for free at Connexions <http://legacy.cnx.org/content/col11759/1.2>

342 CHAPTER 6. EXPONENTIAL AND LOGARITHMIC FUNCTIONS

Table 6.3

Let us examine the graph of f by plotting the ordered pairs we observe on the table in Figure 6.2, andthen make a few observations.

Figure 6.2

Let's de�ne the behavior of the graph of the exponential function f (x) = 2x and highlight some its keycharacteristics.

• the domain is (−∞,∞) ,• the range is (0,∞) ,• asx→∞, f (x)→∞,• as x→ −∞, f (x)→ 0,• f (x) is always increasing,• the graph of f (x) will never touch the x-axis because base two raised to any exponent never has the

result of zero.• y = 0 is the horizontal asymptote.• the y-intercept is 1.

A General Note: For any real numberx,an exponential function is a function with the form

f (x) = abx (6.1)

Available for free at Connexions <http://legacy.cnx.org/content/col11759/1.2>

343

where

• a is the a non-zero real number called the initial value and• b is any positive real number such that b 6= 1.• The domain of f is all real numbers.• The range of f is all positive real numbers if a > 0.• The range of f is all negative real numbers if a < 0.• The y-intercept is (0, a) ,and the horizontal asymptote is y = 0.

Example 6.1Identifying Exponential FunctionsWhich of the following equations are not exponential functions?

• f (x) = 43(x−2)

• g (x) = x3

• h (x) =(

13

)x• j (x) = (−2)x

SolutionBy de�nition, an exponential function has a constant as a base and an independent variable asan exponent. Thus, g (x) = x3 does not represent an exponential function because the base is anindependent variable. In fact, g (x) = x3 is a power function.

Recall that the base b of an exponential function is always a constant, and b 6= 1.Thus, j (x) =(−2)x does not represent an exponential function because the base, − 2, is less than 0.

Try It:

Exercise 6.2.1 (Solution on p. 545.)

Which of the following equations represent exponential functions?

• f (x) = 2x2 − 3x+ 1• g (x) = 0.875x

• h (x) = 1.75x+ 2• j (x) = 1095.6−2x

6.2.2 Evaluating Exponential Functions

Recall that the base of an exponential function must be a positive real number other than 1.Why do we limitthe base b to positive values? To ensure that the outputs will be real numbers. Observe what happens if thebase is not positive:

• Let b = −9 andx = 12 .Then f (x) = f

(12

)= (−9)

12 =√−9,which is not a real number.

Why do we limit the base to positive values other than 1?Because base 1 results in the constant function.Observe what happens if the base is 1 :

• Let b = 1.Then f (x) = 1x = 1 for any value ofx.

Available for free at Connexions <http://legacy.cnx.org/content/col11759/1.2>

344 CHAPTER 6. EXPONENTIAL AND LOGARITHMIC FUNCTIONS

To evaluate an exponential function with the form f (x) = bx,we simply substitute xwith the given value,and calculate the resulting power. For example:

Let f (x) = 2x.What is f (3)?

f (x) = 2x

f (3) = 23 Substitute x = 3.

= 8 Evaluate the power.

(6.2)

To evaluate an exponential function with a form other than the basic form, it is important to follow theorder of operations. For example:

Let f (x) = 30(2)x.What is f (3)?

f (x) = 30(2)x

f (3) = 30(2)3Substitute x = 3.

= 30 (8) Simplify the power �rst.

= 240 Multiply.

(6.3)

Note that if the order of operations were not followed, the result would be incorrect:

f (3) = 30(2)3 6= 603 = 216, 000 (6.4)

Example 6.2Evaluating Exponential FunctionsLet f (x) = 5(3)x+1

.Evaluate f (2) without using a calculator.

SolutionFollow the order of operations. Be sure to pay attention to the parentheses.

f (x) = 5(3)x+1

f (2) = 5(3)2+1Substitute x = 2.

= 5(3)3Add the exponents.

= 5 (27) Simplify the power.

= 135 Multiply.

(6.5)

Try It:

Exercise 6.2.2 (Solution on p. 545.)

Letf (x) = 8(1.2)x−5.Evaluate f (3) using a calculator. Round to four decimal places.

Available for free at Connexions <http://legacy.cnx.org/content/col11759/1.2>

345

6.2.2.1 De�ning Exponential Growth

Because the output of exponential functions increases very rapidly, the term �exponential growth� is oftenused in everyday language to describe anything that grows or increases rapidly. However, exponential growthcan be de�ned more precisely in a mathematical sense. If the growth rate is proportional to the amountpresent, the function models exponential growth.

A General Note: A function that models exponential growth grows by a rate proportional tothe amount present. For any real numberx and any positive real numbers a and b such that b 6= 1,anexponential growth function has the form

f (x) = abx (6.6)

where

• a is the initial or starting value of the function.• b is the growth factor or growth multiplier per unitx.

In more general terms, we have an exponential function, in which a constant base is raised to a variableexponent. To di�erentiate between linear and exponential functions, let's consider two companies, A and B.Company A has 100 stores and expands by opening 50 new stores a year, so its growth can be represented bythe functionA (x) = 100 + 50x.Company B has 100 stores and expands by increasing the number of storesby 50% each year, so its growth can be represented by the function B (x) = 100(1 + 0.5)x.

A few years of growth for these companies are illustrated in Table 6.4.

Year, x Stores, Company A Stores, Company B

0 100 + 50 (0) = 100 100(1 + 0.5)0 = 100

1 100 + 50 (1) = 150 100(1 + 0.5)1 = 150

2 100 + 50 (2) = 200 100(1 + 0.5)2 = 225

3 100 + 50 (3) = 250 100(1 + 0.5)3 = 337.5

x A (x) = 100 + 50x B (x) = 100(1 + 0.5)x

Table 6.4

The graphs comparing the number of stores for each company over a �ve-year period are shown inFigure 6.3. We can see that, with exponential growth, the number of stores increases much more rapidlythan with linear growth.

Available for free at Connexions <http://legacy.cnx.org/content/col11759/1.2>

346 CHAPTER 6. EXPONENTIAL AND LOGARITHMIC FUNCTIONS

Figure 6.3: The graph shows the numbers of stores Companies A and B opened over a �ve-year period.Available for free at Connexions <http://legacy.cnx.org/content/col11759/1.2>

347

Notice that the domain for both functions is [0,∞) ,and the range for both functions is [100,∞) .Afteryear 1, Company B always has more stores than Company A.

Now we will turn our attention to the function representing the number of stores for Company B,B (x) =100(1 + 0.5)x. In this exponential function, 100 represents the initial number of stores, 0.50 represents thegrowth rate, and 1 + 0.5 = 1.5 represents the growth factor. Generalizing further, we can write this functionasB (x) = 100(1.5)x,where 100 is the initial value, 1.5 is called the base, andx is called the exponent.

Example 6.3Evaluating a Real-World Exponential ModelAt the beginning of this section, we learned that the population of India was about 1.25 billionin the year 2013, with an annual growth rate of about 1.2%.This situation is represented by thegrowth functionP (t) = 1.25(1.012)t, where t is the number of years since 2013.To the nearestthousandth, what will the population of India be in 2031?

SolutionTo estimate the population in 2031, we evaluate the models for t = 18, because 2031 is 18 yearsafter 2013. Rounding to the nearest thousandth,

P (18) = 1.25(1.012)18 ≈ 1.549 (6.7)

There will be about 1.549 billion people in India in the year 2031.

Try It:

Exercise 6.2.3 (Solution on p. 545.)

The population of China was about 1.39 billion in the year 2013, with an annualgrowth rate of about 0.6%.This situation is represented by the growth functionP (t) =1.39(1.006)t, where t is the number of years since 2013.To the nearest thousandth, whatwill the population of China be for the year 2031? How does this compare to the populationprediction we made for India in Example 6.3?

6.2.3 Finding Equations of Exponential Functions

In the previous examples, we were given an exponential function, which we then evaluated for a given input.Sometimes we are given information about an exponential function without knowing the function explicitly.We must use the information to �rst write the form of the function, then determine the constants a and b,andevaluate the function.

How To: Given two data points, write an exponential model.

1.If one of the data points has the form (0, a) , then a is the initial value. Using a, substitute thesecond point into the equation f (x) = a(b)x, and solve for b.

2.If neither of the data points have the form (0, a) , substitute both points into two equationswith the form f (x) = a(b)x. Solve the resulting system of two equations in two unknowns to�nd a and b.

3.Using the a and b found in the steps above, write the exponential function in the form f (x) =a(b)x.

Available for free at Connexions <http://legacy.cnx.org/content/col11759/1.2>

348 CHAPTER 6. EXPONENTIAL AND LOGARITHMIC FUNCTIONS

Example 6.4Writing an Exponential Model When the Initial Value Is KnownIn 2006, 80 deer were introduced into a wildlife refuge. By 2012, the population had grown to 180deer. The population was growing exponentially. Write an algebraic functionN (t) representingthe population (N) of deer over time t.

SolutionWe let our independent variable t be the number of years after 2006. Thus, the information givenin the problem can be written as input-output pairs: (0, 80) and (6, 180). Notice that by choosingour input variable to be measured as years after 2006, we have given ourselves the initial valuefor the function, a = 80.We can now substitute the second point into the equationN (t) = 80bt to�nd b :

N (t) = 80bt

180 = 80b6 Substitute using point (6, 180) .94 = b6 Divide and write in lowest terms.

b =(

94

) 16 Isolate b using properties of exponents.

b ≈ 1.1447 Round to 4 decimal places.

(6.8)

NOTE: Unless otherwise stated, do not round any intermediate calculations. Then round the�nal answer to four places for the remainder of this section.

The exponential model for the population of deer isN (t) = 80(1.1447)t. (Note that this expo-nential function models short-term growth. As the inputs gets large, the output will get increasinglylarger, so much so that the model may not be useful in the long term.)

We can graph our model to observe the population growth of deer in the refuge overtime. Notice that the graph in Figure 6.4 passes through the initial points given in theproblem, (0, 80) and (6, 180) .We can also see that the domain for the function is [0,∞) ,and therange for the function is [80,∞) .

Available for free at Connexions <http://legacy.cnx.org/content/col11759/1.2>

349

Figure 6.4: Graph showing the population of deer over time,N (t) = 80(1.1447)t,t years after 2006.Available for free at Connexions <http://legacy.cnx.org/content/col11759/1.2>

350 CHAPTER 6. EXPONENTIAL AND LOGARITHMIC FUNCTIONS

Try It:

Exercise 6.2.4 (Solution on p. 545.)

A wolf population is growing exponentially. In 2011, 129wolves were counted. By 2013, thepopulation had reached 236 wolves. What two points can be used to derive an exponentialequation modeling this situation? Write the equation representing the populationN ofwolves over time t.

Example 6.5Writing an Exponential Model When the Initial Value is Not KnownFind an exponential function that passes through the points (−2, 6) and (2, 1) .

SolutionBecause we don't have the initial value, we substitute both points into an equation of theform f (x) = abx, and then solve the system for a and b.

• Substituting (−2, 6) gives 6 = ab−2

• Substituting (2, 1) gives 1 = ab2

Use the �rst equation to solve for a in terms of b :

Substitute a in the second equation, and solve for b :

Use the value of b in the �rst equation to solve for the value of a :

Thus, the equation is f (x) = 2.4492(0.6389)x.

Available for free at Connexions <http://legacy.cnx.org/content/col11759/1.2>

351

We can graph our model to check our work. Notice that the graph in Figure 6.5 passes throughthe initial points given in the problem, (−2, 6) and (2, 1) .The graph is an example of an expo-nential decay function.

Figure 6.5: The graph of f (x) = 2.4492(0.6389)xmodels exponential decay.

Try It:

Exercise 6.2.5 (Solution on p. 545.)

Given the two points (1, 3) and (2, 4.5) ,�nd the equation of the exponential function thatpasses through these two points.

Available for free at Connexions <http://legacy.cnx.org/content/col11759/1.2>

352 CHAPTER 6. EXPONENTIAL AND LOGARITHMIC FUNCTIONS

Q&A: Do two points always determine a unique exponential function?

Yes, provided the two points are either both above the x-axis or both below the x-axis and havedi�erent x-coordinates. But keep in mind that we also need to know that the graph is, in fact, anexponential function. Not every graph that looks exponential really is exponential. We need toknow the graph is based on a model that shows the same percent growth with each unit increaseinx, which in many real world cases involves time.

How To: Given the graph of an exponential function, write its equation.

1.First, identify two points on the graph. Choose the y-intercept as one of the two pointswhenever possible. Try to choose points that are as far apart as possible to reduce round-o�error.

2.If one of the data points is the y-intercept (0, a), then a is the initial value. Using a, substitutethe second point into the equation f (x) = a(b)x, and solve for b.

3.If neither of the data points have the form (0, a) , substitute both points into two equationswith the form f (x) = a(b)x. Solve the resulting system of two equations in two unknowns to�nd a and b.

4.Write the exponential function, f (x) = a(b)x.

Example 6.6Writing an Exponential Function Given Its GraphFind an equation for the exponential function graphed in Figure 6.6.

Figure 6.6

SolutionWe can choose the y-intercept of the graph, (0, 3) , as our �rst point. This gives us the initialvalue, a = 3.Next, choose a point on the curve some distance away from (0, 3) that has integercoordinates. One such point is (2, 12) .

Available for free at Connexions <http://legacy.cnx.org/content/col11759/1.2>

353

y = abx Write the general form of an exponential equation.

y = 3bx Substitute the initial value 3 for a.

12 = 3b2 Substitute in 12 for y and 2 for x.

4 = b2 Divide by 3.

b = ±2 Take the square root.

(6.9)

Because we restrict ourselves to positive values of b, we will use b = 2. Substitute a and b into thestandard form to yield the equation f (x) = 3(2)x.

Try It:

Exercise 6.2.6 (Solution on p. 545.)

Find an equation for the exponential function graphed in Figure 6.7.

Figure 6.7

How To: Given two points on the curve of an exponential function, use a graphingcalculator to �nd the equation.

Available for free at Connexions <http://legacy.cnx.org/content/col11759/1.2>

354 CHAPTER 6. EXPONENTIAL AND LOGARITHMIC FUNCTIONS

1.Press [STAT].2.Clear any existing entries in columns L1 or L2.3.In L1, enter the x-coordinates given.4.In L2, enter the corresponding y-coordinates.5.Press [STAT] again. Cursor right to CALC, scroll down to ExpReg (Exponential Re-gression), and press [ENTER].

6.The screen displays the values of a and b in the exponential equation y = a · bx

Example 6.7Using a Graphing Calculator to Find an Exponential FunctionUse a graphing calculator to �nd the exponential equation that includes the

points (2, 24.8) and (5, 198.4) .

SolutionFollow the guidelines above. First press [STAT], [EDIT], [1: Edit. . .], and clear the lists L1and L2. Next, in the L1 column, enter the x-coordinates, 2 and 5. Do the same in the L2 columnfor the y-coordinates, 24.8 and 198.4.

Now press [STAT], [CALC], [0: ExpReg] and press [ENTER]. The values a = 6.2 and b =2will be displayed. The exponential equation is y = 6.2 · 2x.

Try It:

Exercise 6.2.7 (Solution on p. 545.)

Use a graphing calculator to �nd the exponential equation that includes the points (3,75.98) and (6, 481.07).

6.2.4 Applying the Compound-Interest Formula

Savings instruments in which earnings are continually reinvested, such as mutual funds and retirementaccounts, use compound interest. The term compounding refers to interest earned not only on theoriginal value, but on the accumulated value of the account.

The annual percentage rate (APR) of an account, also called the nominal rate, is the yearly interestrate earned by an investment account. The term nominal is used when the compounding occurs a numberof times other than once per year. In fact, when interest is compounded more than once a year, the e�ectiveinterest rate ends up being greater than the nominal rate! This is a powerful tool for investing.

We can calculate the compound interest using the compound interest formula, which is an exponentialfunction of the variables time t, principalP, APR r, and number of compounding periods in a yearn :

A (t) = P(

1 +r

n

)nt(6.10)

For example, observe Table 6.5, which shows the result of investing $1,000 at 10% for one year. Notice howthe value of the account increases as the compounding frequency increases.

Available for free at Connexions <http://legacy.cnx.org/content/col11759/1.2>

355

Frequency Value after 1 year

Annually $1100

Semiannually $1102.50

Quarterly $1103.81

Monthly $1104.71

Daily $1105.16

Table 6.5

A General Note: Compound interest can be calculated using the formula

A (t) = P(

1 +r

n

)nt(6.11)

where

• A (t) is the account value,• t is measured in years,• P is the starting amount of the account, often called the principal, or more generally presentvalue,• r is the annual percentage rate (APR) expressed as a decimal, and• n is the number of compounding periods in one year.

Example 6.8Calculating Compound InterestIf we invest $3,000 in an investment account paying 3% interest compounded quarterly, how muchwill the account be worth in 10 years?

SolutionBecause we are starting with $3,000,P = 3000.Our interest rate is 3%, so r = 0.03.Because weare compounding quarterly, we are compounding 4 times per year, son = 4.We want to know thevalue of the account in 10 years, so we are looking forA (10) ,the value when t = 10.

A (t) = P(1 + r

n

)ntUse the compound interest formula.

A (10) = 3000(1 + 0.03

4

)4·10Substitute using given values.

≈ $4045.05 Round to two decimal places.

(6.12)

The account will be worth about $4,045.05 in 10 years.

Try It:

Exercise 6.2.8 (Solution on p. 545.)

An initial investment of $100,000 at 12% interest is compounded weekly (use 52 weeks ina year). What will the investment be worth in 30 years?

Available for free at Connexions <http://legacy.cnx.org/content/col11759/1.2>

356 CHAPTER 6. EXPONENTIAL AND LOGARITHMIC FUNCTIONS

Example 6.9Using the Compound Interest Formula to Solve for the PrincipalA 529 Plan is a college-savings plan that allows relatives to invest money to pay for a child'sfuture college tuition; the account grows tax-free. Lily wants to set up a 529 account for her newgranddaughter and wants the account to grow to $40,000 over 18 years. She believes the accountwill earn 6% compounded semi-annually (twice a year). To the nearest dollar, how much will Lilyneed to invest in the account now?

SolutionThe nominal interest rate is 6%, so r = 0.06. Interest is compounded twice a year, so k = 2.

We want to �nd the initial investment,P, needed so that the value of the account will be worth$40,000 in 18 years. Substitute the given values into the compound interest formula, and solve forP.

A (t) = P(1 + r

n

)ntUse the compound interest formula.

40, 000 = P(1 + 0.06

2

)2(18)Substitute using given values A, r, n, and t.

40, 000 = P (1.03)36Simplify.

40,000(1.03)36

= P Isolate P.

P ≈ $13, 801 Divide and round to the nearest dollar.

(6.13)

Lily will need to invest $13,801 to have $40,000 in 18 years.

Try It:

Exercise 6.2.9 (Solution on p. 545.)

Refer to Example 6.9. To the nearest dollar, how much would Lily need to invest if theaccount is compounded quarterly?

6.2.5 Evaluating Functions with Base e

As we saw earlier, the amount earned on an account increases as the compounding frequency increases.Table 6.6 shows that the increase from annual to semi-annual compounding is larger than the increase frommonthly to daily compounding. This might lead us to ask whether this pattern will continue.

Examine the value of $1 invested at 100% interest for 1 year, compounded at various frequencies, listedin Table 6.6.

Available for free at Connexions <http://legacy.cnx.org/content/col11759/1.2>

357

Frequency A (t) =(1 + 1

n

)nValue

Annually(1 + 1

1

)1$2

Semiannually(1 + 1

2

)2$2.25

Quarterly(1 + 1

4

)4$2.441406

Monthly(1 + 1

12

)12$2.613035

Daily(1 + 1

365

)365$2.714567

Hourly(1 + 1

8766

)8766$2.718127

Once per minute(1 + 1

525960

)525960$2.718279

Once per second (1 + 31557600)31557600$2.718282

Table 6.6

These values appear to be approaching a limit asn increases without bound. In fact, asn gets largerand larger, the expression

(1 + 1

n

)napproaches a number used so frequently in mathematics that it has its

own name: the letter e.This value is an irrational number, which means that its decimal expansion goes onforever without repeating. Its approximation to six decimal places is shown below.

A General Note: The letter e represents the irrational number

(1 +

1n

)n, asn increases without bound (6.14)

The letter e is used as a base for many real-world exponential models. To work with base e, we usethe approximation, e ≈ 2.718282.The constant was named by the Swiss mathematician LeonhardEuler (1707�1783) who �rst investigated and discovered many of its properties.

Example 6.10Using a Calculator to Find Powers of eCalculate e3.14.Round to �ve decimal places.

SolutionOn a calculator, press the button labeled [ex] .The window shows [e^ ( ] . Type 3.14 and then closeparenthesis, [) .Press [ENTER]. Rounding to 5 decimal places, e3.14 ≈ 23.10387.Caution: Many sci-enti�c calculators have an �Exp� button, which is used to enter numbers in scienti�c notation. Itis not used to �nd powers of e.

Try It:

Exercise 6.2.10 (Solution on p. 545.)

Use a calculator to �nd e−0.5.Round to �ve decimal places.

Available for free at Connexions <http://legacy.cnx.org/content/col11759/1.2>

358 CHAPTER 6. EXPONENTIAL AND LOGARITHMIC FUNCTIONS

6.2.6 Investigating Continuous Growth

So far we have worked with rational bases for exponential functions. For most real-world phenomena,however, e is used as the base for exponential functions. Exponential models that use e as the base are calledcontinuous growth or decay models. We see these models in �nance, computer science, and most of thesciences, such as physics, toxicology, and �uid dynamics.

A General Note: For all real numbers t,and all positive numbers a and r,continuous growth ordecay is represented by the formula

A (t) = aert (6.15)

where

• a is the initial value,• r is the continuous growth rate per unit time,• and t is the elapsed time.

If r > 0 , then the formula represents continuous growth. If r < 0 , then the formula representscontinuous decay.

For business applications, the continuous growth formula is called the continuous compoundingformula and takes the form

A (t) = Pert (6.16)

where

• P is the principal or the initial invested,• r is the growth or interest rate per unit time,• and t is the period or term of the investment.

How To: Given the initial value, rate of growth or decay, and time t, solve a continuousgrowth or decay function.

1.Use the information in the problem to determine a, the initial value of the function.2.Use the information in the problem to determine the growth rate r.

a.If the problem refers to continuous growth, then r > 0.b.If the problem refers to continuous decay, then r < 0.

3.Use the information in the problem to determine the time t.4.Substitute the given information into the continuous growth formula and solve forA (t) .

Example 6.11Calculating Continuous GrowthA person invested $1,000 in an account earning a nominal 10% per year compounded continuously.How much was in the account at the end of one year?

SolutionSince the account is growing in value, this is a continuous compounding problem with growthrate r = 0.10.The initial investment was $1,000, soP = 1000.We use the continuous compoundingformula to �nd the value after t = 1 year:

Available for free at Connexions <http://legacy.cnx.org/content/col11759/1.2>

359

A (t) = Pert Use the continuous compounding formula.

= 1000(e)0.1Substitute known values for P, r, and t.

≈ 1105.17 Use a calculator to approximate.

(6.17)

The account is worth $1,105.17 after one year.

Try It:

Exercise 6.2.11 (Solution on p. 545.)

A person invests $100,000 at a nominal 12% interest per year compounded continuously.What will be the value of the investment in 30 years?

Example 6.12Calculating Continuous DecayRadon-222 decays at a continuous rate of 17.3% per day. How much will 100 mg of Radon-222decay to in 3 days?

SolutionSince the substance is decaying, the rate, 17.3%, is negative. So, r = − 0.173.The initial amountof radon-222 was 100mg, so a = 100.We use the continuous decay formula to �nd the value after t =3 days:

A (t) = aert Use the continuous growth formula.

= 100e−0.173(3) Substitute known values for a, r, and t.

≈ 59.5115 Use a calculator to approximate.

(6.18)

So 59.5115 mg of radon-222 will remain.

Try It:

Exercise 6.2.12 (Solution on p. 545.)

Using the data in Example 6.12, how much radon-222 will remain after one year?

Media: Access these online resources for additional instruction and practice with exponentialfunctions.

• Exponential Growth Function6

• Compound Interest7

6.2.7 Key Equations

6http://openstaxcollege.org/l/expgrowth7http://openstaxcollege.org/l/compoundint

Available for free at Connexions <http://legacy.cnx.org/content/col11759/1.2>

360 CHAPTER 6. EXPONENTIAL AND LOGARITHMIC FUNCTIONS

de�nition of the exponential function f (x) = bx, where b > 0, b 6= 1

de�nition of exponential growth f (x) = abx, where a > 0, b > 0, b 6= 1

compound interest formula

A (t) = P(1 + r

n

)nt, where

A (t) is the account value at time t

t is the number of years

P is the initial investment, often called the principal

r is the annual percentage rate (APR), or nominal rate

n is the number of compounding periods in one year

continuous growth formula A (t) = aert, wheretis the number of unit time periods of growthais the starting amount (in the continuous com-pounding formula a is replaced with P, the prin-cipal)eis the mathematical constant, e ≈ 2.718282

Table 6.7

6.2.8 Key Concepts

• An exponential function is de�ned as a function with a positive constant other than 1 raised to a variableexponent. See Example 6.1.

• A function is evaluated by solving at a speci�c value. See Example 6.2 and Example 6.3.• An exponential model can be found when the growth rate and initial value are known. See Example 6.4.• An exponential model can be found when the two data points from the model are known. See Exam-

ple 6.5.• An exponential model can be found using two data points from the graph of the model. See Example 6.6.• An exponential model can be found using two data points from the graph and a calculator. See

Example 6.7.• The value of an account at any time t can be calculated using the compound interest formula when the

principal, annual interest rate, and compounding periods are known. See Example 6.8.• The initial investment of an account can be found using the compound interest formula when the value

of the account, annual interest rate, compounding periods, and life span of the account are known. SeeExample 6.9.

• The number e is a mathematical constant often used as the base of real world exponential growth anddecay models. Its decimal approximation is e ≈ 2.718282.

• Scienti�c and graphing calculators have the key [ex] or [exp (x)] for calculating powers of e. See Exam-ple 6.10.

• Continuous growth or decay models are exponential models that use e as the base. Continuous growthand decay models can be found when the initial value and growth or decay rate are known. SeeExample 6.11 and Example 6.12.

6.2.9 Section Exercises

6.2.9.1 Verbal

Exercise 6.2.13 (Solution on p. 545.)

Explain why the values of an increasing exponential function will eventually overtake the valuesof an increasing linear function.

Available for free at Connexions <http://legacy.cnx.org/content/col11759/1.2>

361

Exercise 6.2.14Given a formula for an exponential function, is it possible to determine whether the function growsor decays exponentially just by looking at the formula? Explain.

Exercise 6.2.15 (Solution on p. 545.)

The Oxford Dictionary de�nes the word nominal as a value that is �stated or expressed but notnecessarily corresponding exactly to the real value.�8 Develop a reasonable argument for why theterm nominal rate is used to describe the annual percentage rate of an investment account thatcompounds interest.

6.2.9.2 Algebraic

For the following exercises, identify whether the statement represents an exponential function. Explain.

Exercise 6.2.16The average annual population increase of a pack of wolves is 25.

Exercise 6.2.17 (Solution on p. 545.)

A population of bacteria decreases by a factor of 18 every 24 hours.

Exercise 6.2.18The value of a coin collection has increased by 3.25% annually over the last 20 years.Exercise 6.2.19 (Solution on p. 545.)

For each training session, a personal trainer charges his clients $5 less than the previous trainingsession.

Exercise 6.2.20The height of a projectile at time t is represented by the functionh (t) = −4.9t2 + 18t+ 40.

For the following exercises, consider this scenario: For each year t,the population of a forest of trees isrepresented by the functionA (t) = 115(1.025)t. In a neighboring forest, the population of the same type oftree is represented by the functionB (t) = 82(1.029)t. (Round answers to the nearest whole number.)

Exercise 6.2.21 (Solution on p. 545.)

Which forest's population is growing at a faster rate?

Exercise 6.2.22Which forest had a greater number of trees initially? By how many?

Exercise 6.2.23 (Solution on p. 545.)

Assuming the population growth models continue to represent the growth of the forests, whichforest will have a greater number of trees after 20 years? By how many?

Exercise 6.2.24Assuming the population growth models continue to represent the growth of the forests, whichforest will have a greater number of trees after 100 years? By how many?

Exercise 6.2.25 (Solution on p. 545.)

Discuss the above results from the previous four exercises. Assuming the population growth modelscontinue to represent the growth of the forests, which forest will have the greater number of treesin the long run? Why? What are some factors that might in�uence the long-term validity of theexponential growth model?

For the following exercises, determine whether the equation represents exponential growth, exponential decay,or neither. Explain.

Exercise 6.2.26y = 300(1− t)5

8Oxford Dictionary. http://oxforddictionaries.com/us/de�nition/american_english/nomina.

Available for free at Connexions <http://legacy.cnx.org/content/col11759/1.2>

362 CHAPTER 6. EXPONENTIAL AND LOGARITHMIC FUNCTIONS

Exercise 6.2.27 (Solution on p. 546.)

y = 220(1.06)x

Exercise 6.2.28y = 16.5(1.025)

1x

Exercise 6.2.29 (Solution on p. 546.)

y = 11, 701(0.97)t

For the following exercises, �nd the formula for an exponential function that passes through the two pointsgiven.

Exercise 6.2.30(0, 6) and (3, 750)Exercise 6.2.31 (Solution on p. 546.)

(0, 2000) and (2, 20)Exercise 6.2.32(−1, 3

2

)and (3, 24)

Exercise 6.2.33 (Solution on p. 546.)

(−2, 6) and (3, 1)Exercise 6.2.34(3, 1) and (5, 4)

For the following exercises, determine whether the table could represent a function that is linear, exponential,or neither. If it appears to be exponential, �nd a function that passes through the points.

Exercise 6.2.35 (Solution on p. 546.)

x 1 2 3 4

f (x) 70 40 10 -20

Table 6.8

Exercise 6.2.36

x 1 2 3 4

h (x) 70 49 34.3 24.01

Table 6.9

Exercise 6.2.37 (Solution on p. 546.)

x 1 2 3 4

m (x) 80 61 42.9 25.61

Table 6.10

Exercise 6.2.38

Available for free at Connexions <http://legacy.cnx.org/content/col11759/1.2>

363

x 1 2 3 4

f (x) 10 20 40 80

Table 6.11

Exercise 6.2.39 (Solution on p. 546.)

x 1 2 3 4

g (x) -3.25 2 7.25 12.5

Table 6.12

For the following exercises, use the compound interest formula,A (t) = P(1 + r

n

)nt.

Exercise 6.2.40After a certain number of years, the value of an investment account is represented by the

equation 10, 250(1 + 0.04

12

)120.What is the value of the account?

Exercise 6.2.41 (Solution on p. 546.)

What was the initial deposit made to the account in the previous exercise?

Exercise 6.2.42How many years had the account from the previous exercise been accumulating interest?

Exercise 6.2.43 (Solution on p. 546.)

An account is opened with an initial deposit of $6,500 and earns 3.6% interest compounded semi-annually. What will the account be worth in 20 years?Exercise 6.2.44How much more would the account in the previous exercise have been worth if the interest werecompounding weekly?

Exercise 6.2.45 (Solution on p. 546.)

Solve the compound interest formula for the principal,P .

Exercise 6.2.46Use the formula found in the previous exercise to calculate the initial deposit of an account that isworth $14, 472.74 after earning 5.5% interest compounded monthly for 5 years. (Round to the nearestdollar.)

Exercise 6.2.47 (Solution on p. 546.)

How much more would the account in the previous two exercises be worth if it were earning interestfor 5more years?Exercise 6.2.48Use properties of rational exponents to solve the compound interest formula for the interest rate, r.

Exercise 6.2.49 (Solution on p. 546.)

Use the formula found in the previous exercise to calculate the interest rate for an account thatwas compounded semi-annually, had an initial deposit of $9,000 and was worth $13,373.53 after 10years.

Exercise 6.2.50Use the formula found in the previous exercise to calculate the interest rate for an account thatwas compounded monthly, had an initial deposit of $5,500, and was worth $38,455 after 30 years.

Available for free at Connexions <http://legacy.cnx.org/content/col11759/1.2>

364 CHAPTER 6. EXPONENTIAL AND LOGARITHMIC FUNCTIONS

For the following exercises, determine whether the equation represents continuous growth, continuous decay,or neither. Explain.

Exercise 6.2.51 (Solution on p. 546.)

y = 3742(e)0.75t

Exercise 6.2.52y = 150(e)

3.25t

Exercise 6.2.53 (Solution on p. 546.)

y = 2.25(e)−2t

Exercise 6.2.54Suppose an investment account is opened with an initial deposit of $12, 000 earning 7.2% interestcompounded continuously. How much will the account be worth after 30 years?Exercise 6.2.55 (Solution on p. 546.)

How much less would the account from Exercise 42 be worth after 30 years if it were compoundedmonthly instead?

6.2.9.3 Numeric

For the following exercises, evaluate each function. Round answers to four decimal places, if necessary.

Exercise 6.2.56f (x) = 2(5)x, for f (−3)Exercise 6.2.57 (Solution on p. 546.)

f (x) = −42x+3, for f (−1)Exercise 6.2.58f (x) = ex, for f (3)Exercise 6.2.59 (Solution on p. 546.)

f (x) = −2ex−1, for f (−1)Exercise 6.2.60f (x) = 2.7(4)−x+1 + 1.5, forf (−2)Exercise 6.2.61 (Solution on p. 546.)

f (x) = 1.2e2x − 0.3, for f (3)Exercise 6.2.62f (x) = − 3

2 (3)−x + 32 , for f (2)

6.2.9.4 Technology

For the following exercises, use a graphing calculator to �nd the equation of an exponential function giventhe points on the curve.

Exercise 6.2.63 (Solution on p. 546.)

(0, 3) and (3, 375)Exercise 6.2.64(3, 222.62) and (10, 77.456)Exercise 6.2.65 (Solution on p. 546.)

(20, 29.495) and (150, 730.89)Exercise 6.2.66(5, 2.909) and (13, 0.005)

Available for free at Connexions <http://legacy.cnx.org/content/col11759/1.2>

365

Exercise 6.2.67 (Solution on p. 546.)

(11, 310.035) and (25, 356.3652)

6.2.9.5 Extensions

Exercise 6.2.68The annual percentage yield (APY) of an investment account is a representation of the actualinterest rate earned on a compounding account. It is based on a compounding period of one year.Show that the APY of an account that compounds monthly can be found with the formulaAPY =(1 + r

12

)12 − 1.Exercise 6.2.69 (Solution on p. 546.)

Repeat the previous exercise to �nd the formula for the APY of an account that compounds daily.Use the results from this and the previous exercise to develop a function I (n) for the APY of anyaccount that compoundsn times per year.

Exercise 6.2.70Recall that an exponential function is any equation written in the form f (x) = a · bx suchthat a and b are positive numbers and b 6= 1. Any positive number b can be written as b = en forsome value of n. Use this fact to rewrite the formula for an exponential function that uses thenumber e as a base.

Exercise 6.2.71 (Solution on p. 546.)

In an exponential decay function, the base of the exponent is a value between 0 and 1. Thus,for some number b > 1, the exponential decay function can be written as f (x) = a ·

(1b

)x.Use this

formula, along with the fact that b = en, to show that an exponential decay function takes theform f (x) = a(e)−nx for some positive numbern .Exercise 6.2.72The formula for the amountA in an investment account with a nominal interest rate r at any time t isgiven byA (t) = a(e)rt,where a is the amount of principal initially deposited into an account thatcompounds continuously. Prove that the percentage of interest earned to principal at any time t canbe calculated with the formula I (t) = ert − 1.

6.2.9.6 Real-World Applications

Exercise 6.2.73 (Solution on p. 546.)

The fox population in a certain region has an annual growth rate of 9% per year. In the year 2012,there were 23,900 fox counted in the area. What is the fox population predicted to be in the year2020?

Exercise 6.2.74A scientist begins with 100 milligrams of a radioactive substance that decays exponentially. After35 hours, 50mg of the substance remains. How many milligrams will remain after 54 hours?

Exercise 6.2.75 (Solution on p. 547.)

In the year 1985, a house was valued at $110,000. By the year 2005, the value had appreciatedto $145,000. What was the annual growth rate between 1985 and 2005? Assume that the valuecontinued to grow by the same percentage. What was the value of the house in the year 2010?

Exercise 6.2.76A car was valued at $38,000 in the year 2007. By 2013, the value had depreciated to $11,000 Ifthe car's value continues to drop by the same percentage, what will it be worth by 2017?

Available for free at Connexions <http://legacy.cnx.org/content/col11759/1.2>

366 CHAPTER 6. EXPONENTIAL AND LOGARITHMIC FUNCTIONS

Exercise 6.2.77 (Solution on p. 547.)

Jamal wants to save $54,000 for a down payment on a home. How much will he need to invest inan account with 8.2% APR, compounding daily, in order to reach his goal in 5 years?

Exercise 6.2.78Kyoko has $10,000 that she wants to invest. Her bank has several investment accounts to choosefrom, all compounding daily. Her goal is to have $15,000 by the time she �nishes graduate schoolin 6 years. To the nearest hundredth of a percent, what should her minimum annual interest ratebe in order to reach her goal? (Hint: solve the compound interest formula for the interest rate.)

Exercise 6.2.79 (Solution on p. 547.)

Alyssa opened a retirement account with 7.25% APR in the year 2000. Her initial deposit was$13,500. How much will the account be worth in 2025 if interest compounds monthly? How muchmore would she make if interest compounded continuously?

Exercise 6.2.80An investment account with an annual interest rate of 7% was opened with an initial deposit of$4,000 Compare the values of the account after 9 years when the interest is compounded annually,quarterly, monthly, and continuously.

6.3 Graphs of Exponential Functions9

As we discussed in the previous section, exponential functions are used for many real-world applicationssuch as �nance, forensics, computer science, and most of the life sciences. Working with an equation thatdescribes a real-world situation gives us a method for making predictions. Most of the time, however, theequation itself is not enough. We learn a lot about things by seeing their pictorial representations, and thatis exactly why graphing exponential equations is a powerful tool. It gives us another layer of insight forpredicting future events.

6.3.1 Graphing Exponential Functions

Before we begin graphing, it is helpful to review the behavior of exponential growth. Recall the table of valuesfor a function of the form f (x) = bx whose base is greater than one. We'll use the function f (x) = 2x.Observehow the output values in Table 6.13 change as the input increases by 1.

x −3 −2 −1 0 1 2 3

f (x) = 2x 18

14

12 1 2 4 8

Table 6.13

Each output value is the product of the previous output and the base, 2.We call the base 2 the constantratio. In fact, for any exponential function with the form f (x) = abx, b is the constant ratio of the function.This means that as the input increases by 1, the output value will be the product of the base and the previousoutput, regardless of the value of a.

Notice from the table that

• the output values are positive for all values of x;• asx increases, the output values increase without bound; and• asx decreases, the output values grow smaller, approaching zero.

9This content is available online at <http://legacy.cnx.org/content/m49362/1.5/>.

Available for free at Connexions <http://legacy.cnx.org/content/col11759/1.2>

367

Figure 6.8 shows the exponential growth function f (x) = 2x.

Figure 6.8: Notice that the graph gets close to the x-axis, but never touches it.

The domain of f (x) = 2x is all real numbers, the range is (0,∞) , and the horizontal asymptote is y = 0.To get a sense of the behavior of exponential decay, we can create a table of values for a function of

the form f (x) = bx whose base is between zero and one. We'll use the function g (x) =(

12

)x.Observe how

the output values in Table 6.14 change as the input increases by 1.

Available for free at Connexions <http://legacy.cnx.org/content/col11759/1.2>

368 CHAPTER 6. EXPONENTIAL AND LOGARITHMIC FUNCTIONS

x −3 −2 −1 0 1 2 3

g (x) =(

12

)x 8 4 2 1 12

14

18

Table 6.14

Again, because the input is increasing by 1, each output value is the product of the previous output andthe base, or constant ratio 1

2 .Notice from the table that

• the output values are positive for all values ofx;• asx increases, the output values grow smaller, approaching zero; and• asx decreases, the output values grow without bound.

Figure 6.9 shows the exponential decay function, g (x) =(

12

)x.

Available for free at Connexions <http://legacy.cnx.org/content/col11759/1.2>

369

Figure 6.9

The domain of g (x) =(

12

)xis all real numbers, the range is (0,∞) ,and the horizontal asymptote is y = 0.

A General Note: An exponential function with the form f (x) = bx, b > 0, b 6= 1,has thesecharacteristics:

• one-to-one function

Available for free at Connexions <http://legacy.cnx.org/content/col11759/1.2>

370 CHAPTER 6. EXPONENTIAL AND LOGARITHMIC FUNCTIONS

• horizontal asymptote: y = 0• domain: (−−∞, ∞)• range: (0,∞)• x-intercept: none• y-intercept: (0, 1)• increasing if b > 1• decreasing if b < 1

Figure 6.10 compares the graphs of exponential growth and decay functions.

Figure 6.10

How To: Given an exponential function of the form f (x) = bx,graph the function.

1.Create a table of points.2.Plot at least 3 point from the table, including the y-intercept (0, 1) .3.Draw a smooth curve through the points.4.State the domain, (−∞,∞) ,the range, (0,∞) ,and the horizontal asymptote, y = 0.

Example 6.13Sketching the Graph of an Exponential Function of the Form f (x) = bx

Sketch a graph of f (x) = 0.25x. State the domain, range, and asymptote.

SolutionBefore graphing, identify the behavior and create a table of points for the graph.

• Since b = 0.25 is between zero and one, we know the function is decreasing. The left tail ofthe graph will increase without bound, and the right tail will approach the asymptote y = 0.

Available for free at Connexions <http://legacy.cnx.org/content/col11759/1.2>

371

• Create a table of points as in Table 6.15.

x −3 −2 −1 0 1 2 3

f (x) = 0.25x 64 16 4 1 0.25 0.0625 0.015625

Table 6.15

• Plot the y-intercept, (0, 1) ,along with two other points. We can use (−1, 4) and (1, 0.25) .

Draw a smooth curve connecting the points as in Figure 6.11.

Figure 6.11

The domain is (−∞,∞) ; the range is (0,∞) ; the horizontal asymptote is y = 0.

Try It:

Exercise 6.3.1 (Solution on p. 547.)

Sketch the graph of f (x) = 4x. State the domain, range, and asymptote.

Available for free at Connexions <http://legacy.cnx.org/content/col11759/1.2>

372 CHAPTER 6. EXPONENTIAL AND LOGARITHMIC FUNCTIONS

6.3.2 Graphing Transformations of Exponential Functions

Transformations of exponential graphs behave similarly to those of other functions. Just as with other parentfunctions, we can apply the four types of transformations�shifts, re�ections, stretches, and compressions�to the parent function f (x) = bx without loss of shape. For instance, just as the quadratic function maintainsits parabolic shape when shifted, re�ected, stretched, or compressed, the exponential function also maintainsits general shape regardless of the transformations applied.

6.3.2.1 Graphing a Vertical Shift

The �rst transformation occurs when we add a constant d to the parent function f (x) = bx, giving us avertical shift d units in the same direction as the sign. For example, if we begin by graphing a parentfunction, f (x) = 2x, we can then graph two vertical shifts alongside it, using d = 3 : the upward shift, g (x) =2x + 3 and the downward shift,h (x) = 2x − 3.Both vertical shifts are shown in Figure 6.12.

Available for free at Connexions <http://legacy.cnx.org/content/col11759/1.2>

373

Figure 6.12

Available for free at Connexions <http://legacy.cnx.org/content/col11759/1.2>

374 CHAPTER 6. EXPONENTIAL AND LOGARITHMIC FUNCTIONS

Observe the results of shifting f (x) = 2x vertically:

• The domain, (−∞,∞) remains unchanged.• When the function is shifted up 3 units to g (x) = 2x + 3 :

◦ The y-intercept shifts up 3 units to (0, 4) .◦ The asymptote shifts up 3 units to y = 3.◦ The range becomes (3,∞) .

• When the function is shifted down 3 units toh (x) = 2x − 3 :

◦ The y-intercept shifts down 3 units to (0,−2) .◦ The asymptote also shifts down 3 units to y = −3.◦ The range becomes (−3,∞) .

6.3.2.2 Graphing a Horizontal Shift

The next transformation occurs when we add a constant c to the input of the parent function f (x) = bx,giving us a horizontal shift c units in the opposite direction of the sign. For example, if we begin by graphingthe parent function f (x) = 2x, we can then graph two horizontal shifts alongside it, using c = 3 : the shiftleft, g (x) = 2x+3, and the shift right,h (x) = 2x−3.Both horizontal shifts are shown in Figure 6.13.

Figure 6.13

Observe the results of shifting f (x) = 2x horizontally:

Available for free at Connexions <http://legacy.cnx.org/content/col11759/1.2>

375

• The domain, (−∞,∞) ,remains unchanged.• The asymptote, y = 0,remains unchanged.• The y-intercept shifts such that:

◦ When the function is shifted left 3 units to g (x) = 2x+3,the y-intercept becomes (0, 8) .This isbecause 2x+3 = (8) 2x,so the initial value of the function is 8.

◦ When the function is shifted right 3 units toh (x) = 2x−3,the y-intercept becomes(0, 1

8

).Again,

see that 2x−3 =(

18

)2x,so the initial value of the function is 1

8 .

A General Note: For any constants c and d,the function f (x) = bx+c + d shifts the parentfunction f (x) = bx

• vertically d units, in the same direction of the sign of d.• horizontally c units, in the opposite direction of the sign of c.• The y-intercept becomes (0, bc + d) .• The horizontal asymptote becomes y = d.• The range becomes (d,∞) .• The domain, (−∞,∞) ,remains unchanged.

How To: Given an exponential function with the form f (x) = bx+c + d,graph thetranslation.

1.Draw the horizontal asymptote y = d.2.Identify the shift as (−c, d) . Shift the graph of f (x) = bx left c units if c is positive, andright c units ifc is negative.

3.Shift the graph of f (x) = bx up d units if d is positive, and down d units if d is negative.4.State the domain, (−∞,∞) ,the range, (d,∞) ,and the horizontal asymptote y = d.

Example 6.14Graphing a Shift of an Exponential FunctionGraph f (x) = 2x+1 − 3. State the domain, range, and asymptote.

SolutionWe have an exponential equation of the form f (x) = bx+c + d, with b = 2, c = 1, and d = −3.

Draw the horizontal asymptote y = d, so draw y = −3.Identify the shift as (−c, d) , so the shift is (−1,−3) .Shift the graph of f (x) = bx left 1 units and down 3 units.

Available for free at Connexions <http://legacy.cnx.org/content/col11759/1.2>

376 CHAPTER 6. EXPONENTIAL AND LOGARITHMIC FUNCTIONS

Figure 6.14

The domain is (−∞,∞) ; the range is (−3,∞) ; the horizontal asymptote is y = −3.

Try It:

Available for free at Connexions <http://legacy.cnx.org/content/col11759/1.2>

377

Exercise 6.3.2 (Solution on p. 547.)

Graph f (x) = 2x−1 + 3. State domain, range, and asymptote.

How To: Given an equation of the form f (x) = bx+c+d for x, use a graphing calculatorto approximate the solution.

• Press [Y=]. Enter the given exponential equation in the line headed �Y1=�.• Enter the given value for f (x) in the line headed �Y2=�.• Press [WINDOW]. Adjust the y-axis so that it includes the value entered for �Y2=�.• Press [GRAPH] to observe the graph of the exponential function along with the line for thespeci�ed value of f (x) .• To �nd the value ofx,we compute the point of intersection. Press [2ND] then [CALC]. Select�intersect� and press [ENTER] three times. The point of intersection gives the value of x forthe indicated value of the function.

Example 6.15Approximating the Solution of an Exponential EquationSolve 42 = 1.2(5)x + 2.8 graphically. Round to the nearest thousandth.

SolutionPress [Y=] and enter 1.2(5)x + 2.8 next to Y1=. Then enter 42 next to Y2=. For a window, usethe values �3 to 3 forx and �5 to 55 for y.Press [GRAPH]. The graphs should intersect somewherenearx = 2.

For a better approximation, press [2ND] then [CALC]. Select [5: intersect] and press [EN-TER] three times. The x-coordinate of the point of intersection is displayed as 2.1661943. (Youranswer may be di�erent if you use a di�erent window or use a di�erent value for Guess?) To thenearest thousandth,x ≈ 2.166.

Try It:

Exercise 6.3.3 (Solution on p. 548.)

Solve 4 = 7.85(1.15)x − 2.27 graphically. Round to the nearest thousandth.

6.3.2.3 Graphing a Stretch or Compression

While horizontal and vertical shifts involve adding constants to the input or to the function itself, a stretchor compression occurs when we multiply the parent function f (x) = bx by a constant |a| > 0.For example,if we begin by graphing the parent function f (x) = 2x,we can then graph the stretch, using a = 3,toget g (x) = 3(2)x as shown on the left in Figure 6.15, and the compression, using a = 1

3 ,to geth (x) = 13 (2)x as

shown on the right in Figure 6.15.

Available for free at Connexions <http://legacy.cnx.org/content/col11759/1.2>

378 CHAPTER 6. EXPONENTIAL AND LOGARITHMIC FUNCTIONS

Figure 6.15: (a) g (x) = 3(2)x stretches the graph of f (x) = 2x vertically by a factor of 3. (b)h (x) =13(2)x compresses the graph of f (x) = 2x vertically by a factor of 1

3.

A General Note: For any factor a > 0,the function f (x) = a(b)x

• is stretched vertically by a factor of a if |a| > 1.• is compressed vertically by a factor of a if |a| < 1.• has a y-intercept of (0, a) .• has a horizontal asymptote at y = 0, a range of (0,∞) , and a domain of (−∞,∞) ,which areunchanged from the parent function.

Example 6.16Graphing the Stretch of an Exponential Function

Sketch a graph of f (x) = 4(

12

)x. State the domain, range, and asymptote.

SolutionBefore graphing, identify the behavior and key points on the graph.

• Since b = 12 is between zero and one, the left tail of the graph will increase without bound

asx decreases, and the right tail will approach the x-axis asx increases.• Since a = 4,the graph of f (x) =

(12

)xwill be stretched by a factor of 4.

• Create a table of points as shown in Table 6.16.

x −3 −2 −1 0 1 2 3

f(x) = 4(

12

)x 32 16 8 4 2 1 0.5

Table 6.16

• Plot the y-intercept, (0, 4) ,along with two other points. We can use (−1, 8) and (1, 2) .

Available for free at Connexions <http://legacy.cnx.org/content/col11759/1.2>

379

Draw a smooth curve connecting the points, as shown in Figure 6.16.

Figure 6.16

The domain is (−∞,∞) ; the range is (0,∞) ; the horizontal asymptote is y = 0.

Try It:

Available for free at Connexions <http://legacy.cnx.org/content/col11759/1.2>

380 CHAPTER 6. EXPONENTIAL AND LOGARITHMIC FUNCTIONS

Exercise 6.3.4 (Solution on p. 548.)

Sketch the graph of f (x) = 12 (4)x. State the domain, range, and asymptote.

6.3.2.4 Graphing Re�ections

In addition to shifting, compressing, and stretching a graph, we can also re�ect it about the x-axis or they-axis. When we multiply the parent function f (x) = bx by − 1,we get a re�ection about the x-axis. Whenwe multiply the input by − 1,we get a re�ection about the y-axis. For example, if we begin by graphingthe parent function f (x) = 2x, we can then graph the two re�ections alongside it. The re�ection about thex-axis, g (x) = −2x,is shown on the left side of Figure 6.17, and the re�ection about the y-axish (x) = 2−x,is shown on the right side of Figure 6.17.

Figure 6.17: (a) g (x) = −2x re�ects the graph of f (x) = 2x about the x-axis. (b) g (x) = 2−x re�ectsthe graph of f (x) = 2x about the y-axis.

A General Note: The function f (x) = −bx

• re�ects the parent function f (x) = bx about the x-axis.• has a y-intercept of (0,−1) .• has a range of (−∞, 0)• has a horizontal asymptote at y = 0 and domain of (−∞,∞) ,which are unchanged from theparent function.

Available for free at Connexions <http://legacy.cnx.org/content/col11759/1.2>

381

The function f (x) = b−x

• re�ects the parent function f (x) = bx about the y-axis.• has a y-intercept of (0, 1) , a horizontal asymptote at y = 0, a range of (0,∞) , and a domainof (−∞,∞) , which are unchanged from the parent function.

Example 6.17Writing and Graphing the Re�ection of an Exponential FunctionFind and graph the equation for a function, g (x) ,that re�ects f (x) =

(14

)xabout the x-axis. State

its domain, range, and asymptote.

SolutionSince we want to re�ect the parent function f (x) =

(14

)xabout the x-axis, we multiply f (x) by −

1 to get, g (x) = −(

14

)x.Next we create a table of points as in Table 6.17.

x −3 −2 −1 0 1 2 3

g (x) = −(

14

)x −64 −16 −4 −1 −0.25 −0.0625 −0.0156

Table 6.17

Plot the y-intercept, (0,−1) ,along with two other points. We can use (−1,−4) and (1,−0.25) .Draw a smooth curve connecting the points:

Available for free at Connexions <http://legacy.cnx.org/content/col11759/1.2>

382 CHAPTER 6. EXPONENTIAL AND LOGARITHMIC FUNCTIONS

Figure 6.18

The domain is (−∞,∞) ; the range is (−∞, 0) ; the horizontal asymptote is y = 0.

Try It:

Exercise 6.3.5 (Solution on p. 549.)

Find and graph the equation for a function, g (x) , that re�ects f (x) = 1.25x about they-axis. State its domain, range, and asymptote.

6.3.2.5 Summarizing Translations of the Exponential Function

Now that we have worked with each type of translation for the exponential function, we can summarize themin Table 6.18 to arrive at the general equation for translating exponential functions.

Available for free at Connexions <http://legacy.cnx.org/content/col11759/1.2>

383

Available for free at Connexions <http://legacy.cnx.org/content/col11759/1.2>

384 CHAPTER 6. EXPONENTIAL AND LOGARITHMIC FUNCTIONS

Translations of the Parent Function f (x) = bx

Translation Form

Shift

• Horizontally c units to the left• Vertically d units up

f (x) = bx+c + d

Stretch and Compress

• Stretch if |a| > 1• Compression if 0 < |a| < 1

f (x) = abx

Re�ect about the x-axisf (x) = −bx

Re�ect about the y-axis

f (x) = b−x =(

1b

)x

General equation for all translationsf (x) = abx+c + d

Table 6.18

A General Note: A translation of an exponential function has the form

f (x) = abx+c + d (6.19)

Where the parent function, y = bx, b > 1,is

• shifted horizontally c units to the left.• stretched vertically by a factor of |a| if |a| > 0.• compressed vertically by a factor of |a| if 0 < |a| < 1.• shifted vertically d units.• re�ected about the x-axis when a < 0.

Note the order of the shifts, transformations, and re�ections follow the order of operations.

Example 6.18Writing a Function from a Description

Write the equation for the function described below. Give the horizontal asymptote, the domain,and the range.

• f (x) = ex is vertically stretched by a factor of 2 , re�ected across the y-axis, and then shiftedup 4 units.

SolutionWe want to �nd an equation of the general form f (x) = abx+c+d.We use the description providedto �nd a, b, c, and d.

Available for free at Connexions <http://legacy.cnx.org/content/col11759/1.2>

385

• We are given the parent function f (x) = ex, so b = e.• The function is stretched by a factor of 2, so a = 2.• The function is re�ected about the y-axis. We replacexwith − x to get: e−x.• The graph is shifted vertically 4 units, so d = 4.

Substituting in the general form we get,

f (x) = abx+c + d

= 2e−x+0 + 4

= 2e−x + 4

(6.20)

The domain is (−∞,∞) ; the range is (4,∞) ; the horizontal asymptote is y = 4.

Try It:

Exercise 6.3.6 (Solution on p. 549.)

Write the equation for function described below. Give the horizontal asymptote, thedomain, and the range.

• f (x) = ex is compressed vertically by a factor of 13 , re�ected across the x-axis and

then shifted down 2 units.

Media: Access this online resource for additional instruction and practice with graphing expo-nential functions.

• Graph Exponential Functions10

6.3.3 Key Equations

General Form for the Translation of the Parent Function f (x) = bx f (x) = abx+c + d

Table 6.19

6.3.4 Key Concepts

• The graph of the function f (x) = bx has a y-intercept at (0, 1) ,domain (−∞, ∞) ,range (0, ∞) , andhorizontal asymptote y = 0. See Example 6.13.

• If b > 1,the function is increasing. The left tail of the graph will approach the asymptote y = 0, andthe right tail will increase without bound.

• If 0 < b < 1, the function is decreasing. The left tail of the graph will increase without bound, and theright tail will approach the asymptote y = 0.

• The equation f (x) = bx + d represents a vertical shift of the parent function f (x) = bx.• The equation f (x) = bx+c represents a horizontal shift of the parent function f (x) = bx. See Exam-

ple 6.14.

10http://openstaxcollege.org/l/graphexpfunc

Available for free at Connexions <http://legacy.cnx.org/content/col11759/1.2>

386 CHAPTER 6. EXPONENTIAL AND LOGARITHMIC FUNCTIONS

• Approximate solutions of the equation f (x) = bx+c + d can be found using a graphing calculator. SeeExample 6.15.

• The equation f (x) = abx, where a > 0, represents a vertical stretch if |a| > 1 or compression if 0 < |a| <1 of the parent function f (x) = bx. See Example 6.16.

• When the parent function f (x) = bx is multiplied by − 1,the result, f (x) = −bx, is a re�ection aboutthe x-axis. When the input is multiplied by −1,the result, f (x) = b−x, is a re�ection about the y-axis.See Example 6.17.

• All translations of the exponential function can be summarized by the general equation f (x) = abx+c+d. See Table 6.15.

• Using the general equation f (x) = abx+c +d, we can write the equation of a function given its descrip-tion. See Example 6.18.

6.3.5 Section Exercises

6.3.5.1 Verbal

Exercise 6.3.7 (Solution on p. 549.)

What role does the horizontal asymptote of an exponential function play in telling us about theend behavior of the graph?

Exercise 6.3.8What is the advantage of knowing how to recognize transformations of the graph of a parentfunction algebraically?

6.3.5.2 Algebraic

Exercise 6.3.9 (Solution on p. 550.)

The graph of f (x) = 3x is re�ected about the y-axis and stretched vertically by a factor of 4.Whatis the equation of the new function, g (x)? State its y-intercept, domain, and range.

Exercise 6.3.10The graph of f (x) =

(12

)−xis re�ected about the y-axis and compressed vertically by a factor

of 15 .What is the equation of the new function, g (x)? State its y-intercept, domain, and range.

Exercise 6.3.11 (Solution on p. 550.)

The graph of f (x) = 10x is re�ected about the x-axis and shifted upward 7 units. What is theequation of the new function, g (x)? State its y-intercept, domain, and range.

Exercise 6.3.12The graph of f (x) = (1.68)x is shifted right 3 units, stretched vertically by a factor of 2,re�ectedabout the x-axis, and then shifted downward 3 units. What is the equation of the newfunction, g (x)? State its y-intercept (to the nearest thousandth), domain, and range.

Exercise 6.3.13 (Solution on p. 550.)

The graph of f (x) = − 12

(14

)x−2 + 4 is shifted left 2 units, stretched vertically by a factorof 4,re�ected about the x-axis, and then shifted downward 4 units. What is the equation of thenew function, g (x)? State its y-intercept, domain, and range.

Available for free at Connexions <http://legacy.cnx.org/content/col11759/1.2>

387

6.3.5.3 Graphical

For the following exercises, graph the function and its re�ection about the y-axis on the same axes, and givethe y-intercept.

Exercise 6.3.14f (x) = 3

(12

)xExercise 6.3.15 (Solution on p. 550.)

g (x) = −2(0.25)x

Exercise 6.3.16h (x) = 6(1.75)−x

For the following exercises, graph each set of functions on the same axes.

Exercise 6.3.17 (Solution on p. 550.)

f (x) = 3(

14

)x,g (x) = 3(2)x,andh (x) = 3(4)x

Exercise 6.3.18f (x) = 1

4 (3)x,g (x) = 2(3)x,andh (x) = 4(3)x

For the following exercises, match each function with one of the graphs in Figure 6.19.

Available for free at Connexions <http://legacy.cnx.org/content/col11759/1.2>

388 CHAPTER 6. EXPONENTIAL AND LOGARITHMIC FUNCTIONS

Figure 6.19

Exercise 6.3.19 (Solution on p. 551.)

f (x) = 2(0.69)x

Exercise 6.3.20f (x) = 2(1.28)x

Exercise 6.3.21 (Solution on p. 551.)

f (x) = 2(0.81)x

Available for free at Connexions <http://legacy.cnx.org/content/col11759/1.2>

389

Exercise 6.3.22f (x) = 4(1.28)x

Exercise 6.3.23 (Solution on p. 551.)

f (x) = 2(1.59)x

Exercise 6.3.24f (x) = 4(0.69)x

For the following exercises, use the graphs shown in Figure 6.20. All have the form f (x) = abx.

Figure 6.20

Available for free at Connexions <http://legacy.cnx.org/content/col11759/1.2>

390 CHAPTER 6. EXPONENTIAL AND LOGARITHMIC FUNCTIONS

Exercise 6.3.25 (Solution on p. 551.)

Which graph has the largest value for b?Exercise 6.3.26Which graph has the smallest value for b?Exercise 6.3.27 (Solution on p. 551.)

Which graph has the largest value for a?Exercise 6.3.28Which graph has the smallest value for a?

For the following exercises, graph the function and its re�ection about the x-axis on the same axes.

Exercise 6.3.29 (Solution on p. 551.)

f (x) = 12 (4)x

Exercise 6.3.30f (x) = 3(0.75)x − 1Exercise 6.3.31 (Solution on p. 552.)

f (x) = −4(2)x + 2For the following exercises, graph the transformation of f (x) = 2x.Give the horizontal asymptote, thedomain, and the range.

Exercise 6.3.32f (x) = 2−x

Exercise 6.3.33 (Solution on p. 553.)

h (x) = 2x + 3Exercise 6.3.34f (x) = 2x−2

For the following exercises, describe the end behavior of the graphs of the functions.

Exercise 6.3.35 (Solution on p. 554.)

f (x) = −5(4)x − 1Exercise 6.3.36f (x) = 3

(12

)x − 2Exercise 6.3.37 (Solution on p. 554.)

f (x) = 3(4)−x + 2For the following exercises, start with the graph of f (x) = 4x.Then write a function that results from thegiven transformation.

Exercise 6.3.38Shift f (x) 4 units upwardExercise 6.3.39 (Solution on p. 554.)

Shift f (x) 3 units downwardExercise 6.3.40Shift f (x) 2 units leftExercise 6.3.41 (Solution on p. 554.)

Shift f (x) 5 units rightExercise 6.3.42Re�ect f (x) about the x-axisExercise 6.3.43 (Solution on p. 554.)

Re�ect f (x) about the y-axisFor the following exercises, each graph is a transformation of y = 2x.Write an equation describing thetransformation.

Available for free at Connexions <http://legacy.cnx.org/content/col11759/1.2>

391

Exercise 6.3.44

Exercise 6.3.45 (Solution on p. 554.)

Available for free at Connexions <http://legacy.cnx.org/content/col11759/1.2>

392 CHAPTER 6. EXPONENTIAL AND LOGARITHMIC FUNCTIONS

Exercise 6.3.46

Available for free at Connexions <http://legacy.cnx.org/content/col11759/1.2>

393

For the following exercises, �nd an exponential equation for the graph.Exercise 6.3.47 (Solution on p. 554.)

Available for free at Connexions <http://legacy.cnx.org/content/col11759/1.2>

394 CHAPTER 6. EXPONENTIAL AND LOGARITHMIC FUNCTIONS

Exercise 6.3.48

Available for free at Connexions <http://legacy.cnx.org/content/col11759/1.2>

395

6.3.5.4 Numeric

For the following exercises, evaluate the exponential functions for the indicated value ofx.

Exercise 6.3.49 (Solution on p. 554.)

g (x) = 13 (7)x−2

for g (6) .Exercise 6.3.50f (x) = 4(2)x−1 − 2 for f (5) .Exercise 6.3.51 (Solution on p. 554.)

h (x) = − 12

(12

)x + 6 forh (−7) .

6.3.5.5 Technology

For the following exercises, use a graphing calculator to approximate the solutions of the equation. Roundto the nearest thousandth. f (x) = abx + d.

Exercise 6.3.52−50 = −

(12

)−x

Available for free at Connexions <http://legacy.cnx.org/content/col11759/1.2>

396 CHAPTER 6. EXPONENTIAL AND LOGARITHMIC FUNCTIONS

Exercise 6.3.53 (Solution on p. 554.)

116 = 14

(18

)xExercise 6.3.5412 = 2(3)x + 1Exercise 6.3.55 (Solution on p. 554.)

5 = 3(

12

)x−1 − 2Exercise 6.3.56−30 = −4(2)x+2 + 2

6.3.5.6 Extensions

Exercise 6.3.57 (Solution on p. 554.)

Explore and discuss the graphs ofF (x) = (b)x andG (x) =(

1b

)x.Then make a conjecture about

the relationship between the graphs of the functions bx and(

1b

)xfor any real number b > 0.

Exercise 6.3.58Prove the conjecture made in the previous exercise.

Exercise 6.3.59 (Solution on p. 554.)

Explore and discuss the graphs of f (x) = 4x, g (x) = 4x−2,andh (x) =(

116

)4x.Then make a

conjecture about the relationship between the graphs of the functions bx and(

1bn

)bx for any real

number n and real number b > 0.Exercise 6.3.60Prove the conjecture made in the previous exercise.

Available for free at Connexions <http://legacy.cnx.org/content/col11759/1.2>

397

6.4 Logarithmic Functions11

Figure 6.21: Devastation of March 11, 2011 earthquake in Honshu, Japan. (credit: Daniel Pierce)

In 2010, a major earthquake struck Haiti, destroying or damaging over 285,000 homes12. One year later,another, stronger earthquake devastated Honshu, Japan, destroying or damaging over 332,000 buildings,13

like those shown in Figure 6.21. Even though both caused substantial damage, the earthquake in 2011 was100 times stronger than the earthquake in Haiti. How do we know? The magnitudes of earthquakes aremeasured on a scale known as the Richter Scale. The Haitian earthquake registered a 7.0 on the RichterScale14 whereas the Japanese earthquake registered a 9.0.15

The Richter Scale is a base-ten logarithmic scale. In other words, an earthquake of magnitude 8 is nottwice as great as an earthquake of magnitude 4. It is 108−4 = 104 = 10, 000 times as great! In this lesson,we will investigate the nature of the Richter Scale and the base-ten function upon which it depends.

11This content is available online at <http://legacy.cnx.org/content/m49363/1.3/>.12http://earthquake.usgs.gov/earthquakes/eqinthenews/2010/us2010rja6/#summary. Accessed 3/4/2013.13http://earthquake.usgs.gov/earthquakes/eqinthenews/2011/usc0001xgp/#summary. Accessed 3/4/2013.14http://earthquake.usgs.gov/earthquakes/eqinthenews/2010/us2010rja6/. Accessed 3/4/2013.15http://earthquake.usgs.gov/earthquakes/eqinthenews/2011/usc0001xgp/#details. Accessed 3/4/2013.

Available for free at Connexions <http://legacy.cnx.org/content/col11759/1.2>

398 CHAPTER 6. EXPONENTIAL AND LOGARITHMIC FUNCTIONS

6.4.1 Converting from Logarithmic to Exponential Form

In order to analyze the magnitude of earthquakes or compare the magnitudes of two di�erent earthquakes,we need to be able to convert between logarithmic and exponential form. For example, suppose the amountof energy released from one earthquake were 500 times greater than the amount of energy released fromanother. We want to calculate the di�erence in magnitude. The equation that represents this problemis 10x = 500, wherex represents the di�erence in magnitudes on the Richter Scale. How would we solveforx?

We have not yet learned a method for solving exponential equations. None of the algebraic tools discussedso far is su�cient to solve 10x = 500.We know that 102 = 100 and 103 = 1000, so it is clear thatxmust besome value between 2 and 3, since y = 10x is increasing. We can examine a graph, as in Figure 6.22, tobetter estimate the solution.

Available for free at Connexions <http://legacy.cnx.org/content/col11759/1.2>

399

Figure 6.22

Estimating from a graph, however, is imprecise. To �nd an algebraic solution, we must introduce anew function. Observe that the graph in Figure 6.22 passes the horizontal line test. The exponentialfunction y = bx is one-to-one, so its inverse,x = by is also a function. As is the case with all inversefunctions, we simply interchangex and y and solve for y to �nd the inverse function. To represent y as afunction ofx, we use a logarithmic function of the form y = logb (x) .The base b logarithm of a number isthe exponent by which we must raise b to get that number.

We read a logarithmic expression as, �The logarithm with base b ofx is equal to y,� or, simpli�ed, �logbase b ofx is y.� We can also say, �b raised to the power of y isx,� because logs are exponents. For example,the base 2 logarithm of 32 is 5, because 5 is the exponent we must apply to 2 to get 32. Since 25 = 32, we

Available for free at Connexions <http://legacy.cnx.org/content/col11759/1.2>

400 CHAPTER 6. EXPONENTIAL AND LOGARITHMIC FUNCTIONS

can write log232 = 5.We read this as �log base 2 of 32 is 5.�We can express the relationship between logarithmic form and its corresponding exponential form as

follows:

logb (x) = y ⇔ by = x, b > 0, b 6= 1 (6.21)

Note that the base b is always positive.

Because logarithm is a function, it is most correctly written as logb (x) , using parentheses to denotefunction evaluation, just as we would with f (x) .However, when the input is a single variable or number, itis common to see the parentheses dropped and the expression written without parentheses, as logbx.Notethat many calculators require parentheses around thex.

We can illustrate the notation of logarithms as follows:

Notice that, comparing the logarithm function and the exponential function, the input and the outputare switched. This means y = logb (x) and y = bx are inverse functions.

A General Note: A logarithm base b of a positive numberx satis�es the following de�nition.

Forx > 0, b > 0, b 6= 1,

y = logb (x) is equivalent to by = x (6.22)

where,

• we read logb (x) as, �the logarithm with base b ofx� or the �log base b ofx.”• the logarithm y is the exponent to which bmust be raised to getx.

Also, since the logarithmic and exponential functions switch thex and y values, the domain andrange of the exponential function are interchanged for the logarithmic function. Therefore,

• the domain of the logarithm function with base b is (0,∞) .• the range of the logarithm function with base b is (−∞,∞) .

Q&A: Can we take the logarithm of a negative number?

No. Because the base of an exponential function is always positive, no power of that base can everbe negative. We can never take the logarithm of a negative number. Also, we cannot take thelogarithm of zero. Calculators may output a log of a negative number when in complex mode, butthe log of a negative number is not a real number.

Available for free at Connexions <http://legacy.cnx.org/content/col11759/1.2>

401

How To: Given an equation in logarithmic form logb (x) = y, convert it to exponentialform.

1.Examine the equation y = logbx and identify b, y, andx.2.Rewrite logbx = y as by = x.

Example 6.19Converting from Logarithmic Form to Exponential FormWrite the following logarithmic equations in exponential form.

a. log6

(√6)

= 12

b. log3 (9) = 2

SolutionFirst, identify the values of b, y, andx.Then, write the equation in the form by = x.

a. log6

(√6)

= 12 Here, b = 6, y = 1

2 , and x =√

6.Therefore, the equation log6

(√6)

= 12 is

equivalent to6

12 =√

6.

b. log3 (9) = 2 Here, b = 3, y = 2, and x = 9.Therefore, the equation log3 (9) = 2 is equivalentto 32 = 9.

Try It:

Exercise 6.4.1 (Solution on p. 555.)

Write the following logarithmic equations in exponential form.

a.log10 (1, 000, 000) = 6b.log5 (25) = 2

6.4.2 Converting from Exponential to Logarithmic Form

To convert from exponents to logarithms, we follow the same steps in reverse. We identify thebase b,exponentx,and output y.Then we writex = logb (y) .

Example 6.20Converting from Exponential Form to Logarithmic FormWrite the following exponential equations in logarithmic form.

a. 23 = 8b. 52 = 25c. 10−4 = 1

10,000

SolutionFirst, identify the values of b, y, andx.Then, write the equation in the formx = logb (y) .

a. 23 = 8 Here, b = 2, x = 3,and y = 8.Therefore, the equation 23 = 8 is equivalent to log2 (8) =3.

Available for free at Connexions <http://legacy.cnx.org/content/col11759/1.2>

402 CHAPTER 6. EXPONENTIAL AND LOGARITHMIC FUNCTIONS

b. 52 = 25 Here, b = 5, x = 2,and y = 25.Therefore, the equation 52 = 25 is equivalentto log5 (25) = 2.

c. 10−4 = 110,000 Here, b = 10, x = −4,and y = 1

10,000 .Therefore, the equation 10−4 = 110,000 is

equivalent to log10

(1

10,000

)= −4.

Try It:

Exercise 6.4.2 (Solution on p. 555.)

Write the following exponential equations in logarithmic form.

a.32 = 9b.53 = 125c.2−1 = 1

2

6.4.3 Evaluating Logarithms

Knowing the squares, cubes, and roots of numbers allows us to evaluate many logarithms mentally. Forexample, consider log28.We ask, �To what exponent must 2 be raised in order to get 8?� Because wealready know 23 = 8, it follows that log28 = 3.

Now consider solving log749 and log327mentally.

• We ask, �To what exponent must 7 be raised in order to get 49?� We know 72 = 49.Therefore, log749 =2

• We ask, �To what exponent must 3 be raised in order to get 27?� We know 33 = 27.Therefore, log327 =3

Even some seemingly more complicated logarithms can be evaluated without a calculator. For example, let'sevaluate log 2

3

49 mentally.

• We ask, �To what exponent must 23 be raised in order to get 4

9? � We know 22 = 4 and 32 = 9,so(

23

)2 =49 .Therefore, log 2

3

(49

)= 2.

How To: Given a logarithm of the form y = logb (x) ,evaluate it mentally.

1.Rewrite the argumentx as a power of b : by = x.2.Use previous knowledge of powers of b identify y by asking, �To what exponent should b be raisedin order to getx?�

Example 6.21Solving Logarithms MentallySolve y = log4 (64) without using a calculator.

SolutionFirst we rewrite the logarithm in exponential form: 4y = 64.Next, we ask, �To what exponent must4 be raised in order to get 64?�

We know

Available for free at Connexions <http://legacy.cnx.org/content/col11759/1.2>

403

43 = 64 (6.23)

Therefore,

log4 (64) = 3 (6.24)

Try It:

Exercise 6.4.3 (Solution on p. 555.)

Solve y = log121 (11) without using a calculator.

Example 6.22Evaluating the Logarithm of a ReciprocalEvaluate y = log3

(127

)without using a calculator.

SolutionFirst we rewrite the logarithm in exponential form: 3y = 1

27 .Next, we ask, �To what exponent must3 be raised in order to get 1

27?�We know 33 = 27,but what must we do to get the reciprocal, 1

27?Recall from working withexponents that b−a = 1

ba .We use this information to write

3−3 = 133

= 127

(6.25)

Therefore, log3

(127

)= −3.

Try It:

Exercise 6.4.4 (Solution on p. 555.)

Evaluate y = log2

(132

)without using a calculator.

6.4.4 Using Common Logarithms

Sometimes we may see a logarithm written without a base. In this case, we assume that the base is 10.In other words, the expression log (x) means log10 (x) .We call a base-10 logarithm a common logarithm.Common logarithms are used to measure the Richter Scale mentioned at the beginning of the section. Scalesfor measuring the brightness of stars and the pH of acids and bases also use common logarithms.

Available for free at Connexions <http://legacy.cnx.org/content/col11759/1.2>

404 CHAPTER 6. EXPONENTIAL AND LOGARITHMIC FUNCTIONS

A General Note: A common logarithm is a logarithm with base 10.We write log10 (x) simplyas log (x) .The common logarithm of a positive numberx satis�es the following de�nition.

Forx > 0,

y = log (x) is equivalent to 10y = x (6.26)

We read log (x) as, �the logarithm with base 10 ofx � or �log base 10 ofx.�

The logarithm y is the exponent to which 10must be raised to getx.

How To: Given a common logarithm of the form y = log (x) , evaluate it mentally.

1.Rewrite the argumentx as a power of 10 : 10y = x.2.Use previous knowledge of powers of 10 to identify y by asking, �To what exponent must 10 beraised in order to getx?�

Example 6.23Finding the Value of a Common Logarithm MentallyEvaluate y = log (1000) without using a calculator.

SolutionFirst we rewrite the logarithm in exponential form: 10y = 1000.Next, we ask, �To what exponentmust 10 be raised in order to get 1000?� We know

103 = 1000 (6.27)

Therefore, log (1000) = 3.

Try It:

Exercise 6.4.5 (Solution on p. 555.)

Evaluate y = log (1, 000, 000) .

How To: Given a common logarithm with the form y = log (x) ,evaluate it using acalculator.

1.Press [LOG].2.Enter the value given forx,followed by [ ) ].3.Press [ENTER].

Example 6.24Finding the Value of a Common Logarithm Using a CalculatorEvaluate y = log (321) to four decimal places using a calculator.

Solution

• Press [LOG].• Enter 321, followed by [ ) ].• Press [ENTER].

Available for free at Connexions <http://legacy.cnx.org/content/col11759/1.2>

405

Rounding to four decimal places, log (321) ≈ 2.5065.

AnalysisNote that 102 = 100 and that 103 = 1000. Since 321 is between 100 and 1000, we know

that log (321) must be between log (100) and log (1000) .This gives us the following:

100 < 321 < 1000

2 < 2.5065 < 3(6.28)

Try It:

Exercise 6.4.6 (Solution on p. 555.)

Evaluate y = log (123) to four decimal places using a calculator.

Example 6.25Rewriting and Solving a Real-World Exponential ModelThe amount of energy released from one earthquake was 500 times greater than the amount ofenergy released from another. The equation 10x = 500 represents this situation, wherex is thedi�erence in magnitudes on the Richter Scale. To the nearest thousandth, what was the di�erencein magnitudes?

SolutionWe begin by rewriting the exponential equation in logarithmic form.

10x = 500

log (500) = x Use the de�nition of the common log.(6.29)

Next we evaluate the logarithm using a calculator:

• Press [LOG].• Enter 500,followed by [ ) ].• Press [ENTER].• To the nearest thousandth, log (500) ≈ 2.699.

The di�erence in magnitudes was about 2.699.

Try It:

Exercise 6.4.7 (Solution on p. 555.)

The amount of energy released from one earthquake was 8,500 times greater than theamount of energy released from another. The equation 10x = 8500 represents this situa-tion, wherex is the di�erence in magnitudes on the Richter Scale. To the nearest thou-sandth, what was the di�erence in magnitudes?

Available for free at Connexions <http://legacy.cnx.org/content/col11759/1.2>

406 CHAPTER 6. EXPONENTIAL AND LOGARITHMIC FUNCTIONS

6.4.5 Using Natural Logarithms

The most frequently used base for logarithms is e.Base e logarithms are important in calculus and somescienti�c applications; they are called natural logarithms. The base e logarithm, loge (x) , has its ownnotation, ln (x) .

Most values of ln (x) can be found only using a calculator. The major exception is that, because thelogarithm of 1 is always 0 in any base, ln1 = 0.For other natural logarithms, we can use the ln key that canbe found on most scienti�c calculators. We can also �nd the natural logarithm of any power of e using theinverse property of logarithms.

A General Note: A natural logarithm is a logarithm with base e. We write loge (x) simplyas ln (x) . The natural logarithm of a positive number x satis�es the following de�nition.

Forx > 0,

y = ln (x) is equivalent to ey = x (6.30)

We read ln (x) as, �the logarithm with base e ofx� or �the natural logarithm ofx.�

The logarithm y is the exponent to which emust be raised to getx.

Since the functions y = ex and y = ln (x) are inverse functions, ln (ex) = x for allx and eln(x) =x forx > 0.

How To: Given a natural logarithm with the form y = ln (x) , evaluate it using acalculator.

1.Press [LN].2.Enter the value given forx, followed by [ ) ].3.Press [ENTER].

Example 6.26Evaluating a Natural Logarithm Using a CalculatorEvaluate y = ln (500) to four decimal places using a calculator.

Solution

• Press [LN].• Enter 500,followed by [ ) ].• Press [ENTER].

Rounding to four decimal places, ln (500) ≈ 6.2146

Try It:

Exercise 6.4.8 (Solution on p. 555.)

Evaluate ln (−500) .

Media: Access this online resource for additional instruction and practice with logarithms.

• Introduction to Logarithms16

16http://openstaxcollege.org/l/intrologarithms

Available for free at Connexions <http://legacy.cnx.org/content/col11759/1.2>

407

6.4.6 Key Equations

De�nition of the logarithmic function For x > 0, b > 0, b 6= 1,y = logb (x) if and only if by = x.

De�nition of the common logarithm For x > 0,y = log (x) if and only if 10y = x.

De�nition of the natural logarithm For x > 0,y = ln (x) if and only if ey = x.

Table 6.20

6.4.7 Key Concepts

• The inverse of an exponential function is a logarithmic function, and the inverse of a logarithmicfunction is an exponential function.

• Logarithmic equations can be written in an equivalent exponential form, using the de�nition of alogarithm. See Example 6.19.

• Exponential equations can be written in their equivalent logarithmic form using the de�nition of alogarithm See Example 6.20.

• Logarithmic functions with base b can be evaluated mentally using previous knowledge of powersof b. See Example 6.21 and Example 6.22.

• Common logarithms can be evaluated mentally using previous knowledge of powers of 10. See Exam-ple 6.23.

• When common logarithms cannot be evaluated mentally, a calculator can be used. See Example 6.24.• Real-world exponential problems with base 10 can be rewritten as a common logarithm and then eval-

uated using a calculator. See Example 6.25.• Natural logarithms can be evaluated using a calculator Example 6.26.

6.4.8 Section Exercises

6.4.8.1 Verbal

Exercise 6.4.9 (Solution on p. 555.)

What is a base b logarithm? Discuss the meaning by interpreting each part of the equivalentequations by = x and logbx = y for b > 0, b 6= 1.Exercise 6.4.10How is the logarithmic function f (x) = logbx related to the exponential function g (x) = bx?Whatis the result of composing these two functions?

Exercise 6.4.11 (Solution on p. 555.)

How can the logarithmic equation logbx = y be solved forx using the properties of exponents?

Exercise 6.4.12Discuss the meaning of the common logarithm. What is its relationship to a logarithm with base b,and how does the notation di�er?

Exercise 6.4.13 (Solution on p. 555.)

Discuss the meaning of the natural logarithm. What is its relationship to a logarithm with base b,and how does the notation di�er?

Available for free at Connexions <http://legacy.cnx.org/content/col11759/1.2>

408 CHAPTER 6. EXPONENTIAL AND LOGARITHMIC FUNCTIONS

6.4.8.2 Algebraic

For the following exercises, rewrite each equation in exponential form.

Exercise 6.4.14log4 (q) = m

Exercise 6.4.15 (Solution on p. 555.)

loga (b) = c

Exercise 6.4.16log16 (y) = x

Exercise 6.4.17 (Solution on p. 555.)

logx (64) = y

Exercise 6.4.18logy (x) = −11Exercise 6.4.19 (Solution on p. 555.)

log15 (a) = b

Exercise 6.4.20logy (137) = x

Exercise 6.4.21 (Solution on p. 555.)

log13 (142) = a

Exercise 6.4.22log (v) = t

Exercise 6.4.23 (Solution on p. 555.)

ln (w) = n

For the following exercises, rewrite each equation in logarithmic form.

Exercise 6.4.244x = y

Exercise 6.4.25 (Solution on p. 555.)

cd = k

Exercise 6.4.26m−7 = n

Exercise 6.4.27 (Solution on p. 555.)

19x = y

Exercise 6.4.28x−

1013 = y

Exercise 6.4.29 (Solution on p. 555.)

n4 = 103Exercise 6.4.30(

75

)m = n

Exercise 6.4.31 (Solution on p. 555.)

yx = 39100

Exercise 6.4.3210a = b

Exercise 6.4.33 (Solution on p. 556.)

ek = h

For the following exercises, solve forx by converting the logarithmic equation to exponential form.

Available for free at Connexions <http://legacy.cnx.org/content/col11759/1.2>

409

Exercise 6.4.34log3 (x) = 2Exercise 6.4.35 (Solution on p. 556.)

log2 (x) = −3Exercise 6.4.36log5 (x) = 2Exercise 6.4.37 (Solution on p. 556.)

log3 (x) = 3Exercise 6.4.38log2 (x) = 6Exercise 6.4.39 (Solution on p. 556.)

log9 (x) = 12

Exercise 6.4.40log18 (x) = 2Exercise 6.4.41 (Solution on p. 556.)

log6 (x) = −3Exercise 6.4.42log (x) = 3Exercise 6.4.43 (Solution on p. 556.)

ln (x) = 2For the following exercises, use the de�nition of common and natural logarithms to simplify.

Exercise 6.4.44log(1008

)Exercise 6.4.45 (Solution on p. 556.)

10log(32)

Exercise 6.4.462log (.0001)Exercise 6.4.47 (Solution on p. 556.)

eln(1.06)

Exercise 6.4.48ln(e−5.03

)Exercise 6.4.49 (Solution on p. 556.)

eln(10.125) + 4

6.4.8.3 Numeric

For the following exercises, evaluate the base b logarithmic expression without using a calculator.

Exercise 6.4.50log3

(127

)Exercise 6.4.51 (Solution on p. 556.)

log6

(√6)

Exercise 6.4.52log2

(18

)+ 4

Exercise 6.4.53 (Solution on p. 556.)

6log8 (4)

Available for free at Connexions <http://legacy.cnx.org/content/col11759/1.2>

410 CHAPTER 6. EXPONENTIAL AND LOGARITHMIC FUNCTIONS

For the following exercises, evaluate the common logarithmic expression without using a calculator.

Exercise 6.4.54log (10, 000)Exercise 6.4.55 (Solution on p. 556.)

log (0.001)Exercise 6.4.56log (1) + 7Exercise 6.4.57 (Solution on p. 556.)

2log(100−3

)For the following exercises, evaluate the natural logarithmic expression without using a calculator.

Exercise 6.4.58ln(e

13

)Exercise 6.4.59 (Solution on p. 556.)

ln (1)Exercise 6.4.60ln(e−0.225

)− 3

Exercise 6.4.61 (Solution on p. 556.)

25ln(e

25

)

6.4.8.4 Technology

For the following exercises, evaluate each expression using a calculator. Round to the nearest thousandth.

Exercise 6.4.62log (0.04)Exercise 6.4.63 (Solution on p. 556.)

ln (15)Exercise 6.4.64ln(

45

)Exercise 6.4.65 (Solution on p. 556.)

log(√

2)

Exercise 6.4.66ln(√

2)

6.4.8.5 Extensions

Exercise 6.4.67 (Solution on p. 556.)

Isx = 0 in the domain of the function f (x) = log (x)? If so, what is the value of the functionwhenx = 0?Verify the result.

Exercise 6.4.68Is f (x) = 0 in the range of the function f (x) = log (x)? If so, for what value ofx?Verify the result.

Exercise 6.4.69 (Solution on p. 556.)

Is there a numberx such that lnx = 2? If so, what is that number? Verify the result.

Exercise 6.4.70Is the following true: log3(27)

log4( 164 ) = −1?Verify the result.

Available for free at Connexions <http://legacy.cnx.org/content/col11759/1.2>

411

Exercise 6.4.71 (Solution on p. 556.)

Is the following true:ln(e1.725)

ln(1) = 1.725?Verify the result.

6.4.8.6 Real-World Applications

Exercise 6.4.72The exposure indexEI for a 35 millimeter camera is a measurement of the amount of light that

hits the �lm. It is determined by the equationEI = log2

(f2

t

), where f is the �f-stop� setting on

the camera, and t is the exposure time in seconds. Suppose the f-stop setting is 8 and the desiredexposure time is 2 seconds. What will the resulting exposure index be?

Exercise 6.4.73 (Solution on p. 556.)

Refer to the previous exercise. Suppose the light meter on a camera indicates anEI of − 2, andthe desired exposure time is 16 seconds. What should the f-stop setting be?

Exercise 6.4.74The intensity levels I of two earthquakes measured on a seismograph can be compared by theformula log I1I2 = M1 −M2 whereM is the magnitude given by the Richter Scale. In August 2009,an earthquake of magnitude 6.1 hit Honshu, Japan. In March 2011, that same region experiencedyet another, more devastating earthquake, this time with a magnitude of 9.0.17 How many timesgreater was the intensity of the 2011 earthquake? Round to the nearest whole number.

6.5 Graphs of Logarithmic Functions18

In Graphs of Exponential Functions (Section 6.3), we saw how creating a graphical representation of anexponential model gives us another layer of insight for predicting future events. How do logarithmic graphsgive us insight into situations? Because every logarithmic function is the inverse function of an exponentialfunction, we can think of every output on a logarithmic graph as the input for the corresponding inverseexponential equation. In other words, logarithms give the cause for an e�ect.

To illustrate, suppose we invest $2500 in an account that o�ers an annual interest rate of 5%,compoundedcontinuously. We already know that the balance in our account for any year t can be found with theequationA = 2500e0.05t.

But what if we wanted to know the year for any balance? We would need to create a corresponding newfunction by interchanging the input and the output; thus we would need to create a logarithmic model forthis situation. By graphing the model, we can see the output (year) for any input (account balance). Forinstance, what if we wanted to know how many years it would take for our initial investment to double?Figure 6.23 shows this point on the logarithmic graph.

17http://earthquake.usgs.gov/earthquakes/world/historical.php. Accessed 3/4/2014.18This content is available online at <http://legacy.cnx.org/content/m49364/1.5/>.

Available for free at Connexions <http://legacy.cnx.org/content/col11759/1.2>

412 CHAPTER 6. EXPONENTIAL AND LOGARITHMIC FUNCTIONS

Figure 6.23

In this section we will discuss the values for which a logarithmic function is de�ned, and then turn ourattention to graphing the family of logarithmic functions.

6.5.1 Finding the Domain of a Logarithmic Function

Before working with graphs, we will take a look at the domain (the set of input values) for which thelogarithmic function is de�ned.

Recall that the exponential function is de�ned as y = bx for any real numberx and constant b > 0, b 6= 1,where

• The domain of y is (−∞,∞) .• The range of y is (0,∞) .

In the last section we learned that the logarithmic function y = logb (x) is the inverse of the exponentialfunction y = bx. So, as inverse functions:

• The domain of y = logb (x) is the range of y = bx : (0,∞) .• The range of y = logb (x) is the domain of y = bx : (−∞,∞) .

Transformations of the parent function y = logb (x) behave similarly to those of other functions. Just as withother parent functions, we can apply the four types of transformations�shifts, stretches, compressions, andre�ections�to the parent function without loss of shape.

In Graphs of Exponential Functions (Section 6.3) we saw that certain transformations can change therange of y = bx. Similarly, applying transformations to the parent function y = logb (x) can change thedomain. When �nding the domain of a logarithmic function, therefore, it is important to remember that thedomain consists only of positive real numbers. That is, the argument of the logarithmic function must begreater than zero.

Available for free at Connexions <http://legacy.cnx.org/content/col11759/1.2>

413

For example, consider f (x) = log4 (2x− 3) .This function is de�ned for any values ofx such that theargument, in this case 2x − 3, is greater than zero. To �nd the domain, we set up an inequality and solveforx :

2x− 3 > 0 Show the argument greater than zero.

2x > 3 Add 3.

x > 1.5 Divide by 2.

(6.31)

In interval notation, the domain of f (x) = log4 (2x− 3) is (1.5,∞) .

How To: Given a logarithmic function, identify the domain.

1.Set up an inequality showing the argument greater than zero.2.Solve forx.3.Write the domain in interval notation.

Example 6.27Identifying the Domain of a Logarithmic ShiftWhat is the domain of f (x) = log2 (x+ 3)?

SolutionThe logarithmic function is de�ned only when the input is positive, so this function is de�nedwhenx+ 3 > 0. Solving this inequality,

x+ 3 > 0 The input must be positive.

x > − 3 Subtract 3.(6.32)

The domain of f (x) = log2 (x+ 3) is (−3,∞) .

Try It:

Exercise 6.5.1 (Solution on p. 556.)

What is the domain of f (x) = log5 (x− 2) + 1?

Example 6.28Identifying the Domain of a Logarithmic Shift and Re�ectionWhat is the domain of f (x) = log (5− 2x)?

SolutionThe logarithmic function is de�ned only when the input is positive, so this function is de�nedwhen 5−−2x > 0. Solving this inequality,

5− 2x > 0 The input must be positive.

− 2x > − 5 Subtract 5.

x < 52 Divide by − 2 and switch the inequality.

(6.33)

The domain of f (x) = log (5− 2x) is(−−∞, 5

2

).

Available for free at Connexions <http://legacy.cnx.org/content/col11759/1.2>

414 CHAPTER 6. EXPONENTIAL AND LOGARITHMIC FUNCTIONS

Try It:

Exercise 6.5.2 (Solution on p. 556.)

What is the domain of f (x) = log (x− 5) + 2?

6.5.2 Graphing Logarithmic Functions

Now that we have a feel for the set of values for which a logarithmic function is de�ned, we move onto graphing logarithmic functions. The family of logarithmic functions includes the parent function y =logb (x) along with all its transformations: shifts, stretches, compressions, and re�ections.

We begin with the parent function y = logb (x) .Because every logarithmic function of this form is theinverse of an exponential function with the form y = bx, their graphs will be re�ections of each other acrossthe line y = x.To illustrate this, we can observe the relationship between the input and output valuesof y = 2x and its equivalentx = log2 (y) in Table 6.21.

x −3 −2 −1 0 1 2 3

2x = y 18

14

12 1 2 4 8

log2 (y) = x −3 −2 −1 0 1 2 3

Table 6.21

Using the inputs and outputs from Table 6.21, we can build another table to observe the relationshipbetween points on the graphs of the inverse functions f (x) = 2x and g (x) = log2 (x) . See Table 6.22.

f (x) = 2x(−3, 1

8

) (−2, 1

4

) (−1, 1

2

)(0, 1) (1, 2) (2, 4) (3, 8)

g (x) = log2 (x)(

18 ,−3

) (14 ,−2

) (12 ,−1

)(1, 0) (2, 1) (4, 2) (8, 3)

Table 6.22

As we'd expect, the x- and y-coordinates are reversed for the inverse functions. Figure 6.24 shows thegraph of f and g.

Available for free at Connexions <http://legacy.cnx.org/content/col11759/1.2>

415

Figure 6.24: Notice that the graphs of f (x) = 2x and g (x) = log2 (x) are re�ections about the line y =x.

Observe the following from the graph:

• f (x) = 2x has a y-intercept at (0, 1) and g (x) = log2 (x) has an x- intercept at (1, 0) .• The domain of f (x) = 2x, (−∞,∞) , is the same as the range of g (x) = log2 (x) .• The range of f (x) = 2x, (0,∞) , is the same as the domain of g (x) = log2 (x) .

A General Note: For any real numberx and constant b > 0,b 6= 1, we can see the followingcharacteristics in the graph of f (x) = logb (x) :

• one-to-one function• vertical asymptote:x = 0

Available for free at Connexions <http://legacy.cnx.org/content/col11759/1.2>

416 CHAPTER 6. EXPONENTIAL AND LOGARITHMIC FUNCTIONS

• domain: (0,∞)• range: (−∞,∞)• x-intercept: (1, 0) and key point (b, 1)• y-intercept: none• increasing if b > 1• decreasing if 0 < b < 1

See Figure 6.25.

Figure 6.25

Figure 6.26 shows how changing the base b in f (x) = logb (x) can a�ect the graphs. Observethat the graphs compress vertically as the value of the base increases. (Note: recall that thefunction ln (x) has base e ≈ 2.718.)

Available for free at Connexions <http://legacy.cnx.org/content/col11759/1.2>

417

Figure 6.26: The graphs of three logarithmic functions with di�erent bases, all greater than 1.

How To: Given a logarithmic function with the form f (x) = logb (x) , graph the func-tion.

1.Draw and label the vertical asymptote,x = 0.2.Plot the x-intercept, (1, 0) .3.Plot the key point (b, 1) .4.Draw a smooth curve through the points.5.State the domain, (0,∞) ,the range, (−∞,∞) ,and the vertical asymptote,x = 0.

Example 6.29Graphing a Logarithmic Function with the Form f (x) = logb(x).Graph f (x) = log5 (x) . State the domain, range, and asymptote.

SolutionBefore graphing, identify the behavior and key points for the graph.

• Since b = 5 is greater than one, we know the function is increasing. The left tail of the graphwill approach the vertical asymptotex = 0, and the right tail will increase slowly withoutbound.

• The x-intercept is (1, 0) .

Available for free at Connexions <http://legacy.cnx.org/content/col11759/1.2>

418 CHAPTER 6. EXPONENTIAL AND LOGARITHMIC FUNCTIONS

• The key point (5, 1) is on the graph.• We draw and label the asymptote, plot and label the points, and draw a smooth curve through

the points (see Figure 6.27).

Figure 6.27

The domain is (0,∞) , the range is (−∞,∞) , and the vertical asymptote isx = 0.

Try It:

Exercise 6.5.3 (Solution on p. 557.)

Graph f (x) = log 15

(x) . State the domain, range, and asymptote.

Available for free at Connexions <http://legacy.cnx.org/content/col11759/1.2>

419

6.5.2.1 Graphing Transformations of Logarithmic Functions

As we mentioned in the beginning of the section, transformations of logarithmic graphs behave similarlyto those of other parent functions. We can shift, stretch, compress, and re�ect the parent function y =logb (x) without loss of shape.

6.5.2.2 Graphing a Horizontal Shift of f (x) = logb(x)

When a constant c is added to the input of the parent function f (x) = logb (x) , the result is a horizontalshift c units in the opposite direction of the sign on c.To visualize horizontal shifts, we can observe thegeneral graph of the parent functionf (x) = logb (x) and for c > 0 alongside the shift left, g (x) = logb (x+ c) ,and the shift right,h (x) = logb (x− c) . See Figure 6.28.

Figure 6.28

A General Note: For any constant c,the function f (x) = logb (x+ c)

• shifts the parent function y = logb (x) left c units if c > 0.• shifts the parent function y = logb (x) right c units if c < 0.• has the vertical asymptotex = −c.• has domain (−c,∞) .• has range (−∞,∞) .

How To: Given a logarithmic function with the form f (x) = logb (x+ c) , graph thetranslation.

Available for free at Connexions <http://legacy.cnx.org/content/col11759/1.2>

420 CHAPTER 6. EXPONENTIAL AND LOGARITHMIC FUNCTIONS

1.Identify the horizontal shift:

a.If c > 0,shift the graph of f (x) = logb (x) left c units.b.If c < 0,shift the graph of f (x) = logb (x) right c units.

2.Draw the vertical asymptotex = −c.3.Identify three key points from the parent function. Find new coordinates for the shiftedfunctions by subtracting c from thex coordinate.

4.Label the three points.5.The Domain is (−c,∞) ,the range is (−∞,∞) , and the vertical asymptote isx = −c.

Example 6.30Graphing a Horizontal Shift of the Parent Function y = logb(x)Sketch the horizontal shift f (x) = log3 (x− 2) alongside its parent function. Include the key pointsand asymptotes on the graph. State the domain, range, and asymptote.

SolutionSince the function is f (x) = log3 (x− 2) , we noticex+ (−2) = x−−2.

Thus c = −2,so c < 0.This means we will shift the function f (x) = log3 (x) right 2 units.The vertical asymptote isx = − (−2) orx = 2.Consider the three key points from the parent function,

(13 ,−1

),(1, 0) ,and (3, 1) .

The new coordinates are found by adding 2 to thex coordinates.Label the points

(73 ,−1

),(3, 0) ,and (5, 1) .

The domain is (2,∞) ,the range is (−∞,∞) ,and the vertical asymptote isx = 2.

Available for free at Connexions <http://legacy.cnx.org/content/col11759/1.2>

421

Figure 6.29

Try It:

Exercise 6.5.4 (Solution on p. 557.)

Sketch a graph of f (x) = log3 (x+ 4) alongside its parent function. Include the key pointsand asymptotes on the graph. State the domain, range, and asymptote.

6.5.2.3 Graphing a Vertical Shift of y = logb(x)

When a constant d is added to the parent function f (x) = logb (x) ,the result is a vertical shift d unitsin the direction of the sign on d.To visualize vertical shifts, we can observe the general graph of the parentfunction f (x) = logb (x) alongside the shift up, g (x) = logb (x)+d and the shift down,h (x) = logb (x)−d.SeeFigure 6.30.

Available for free at Connexions <http://legacy.cnx.org/content/col11759/1.2>

422 CHAPTER 6. EXPONENTIAL AND LOGARITHMIC FUNCTIONS

Figure 6.30

A General Note: For any constant d,the function f (x) = logb (x) + d

• shifts the parent function y = logb (x) up d units if d > 0.• shifts the parent function y = logb (x) down d units if d < 0.• has the vertical asymptotex = 0.• has domain (0,∞) .• has range (−∞,∞) .

How To: Given a logarithmic function with the form f (x) = logb (x) + d, graph thetranslation.

1.Identify the vertical shift:

• If d > 0, shift the graph of f (x) = logb (x) up d units.• If d < 0, shift the graph of f (x) = logb (x)down d units.

2.Draw the vertical asymptotex = 0.

Available for free at Connexions <http://legacy.cnx.org/content/col11759/1.2>

423

3.Identify three key points from the parent function. Find new coordinates for the shiftedfunctions by adding d to the y coordinate.

4.Label the three points.5.The domain is (0,∞) ,the range is (−∞,∞) ,and the vertical asymptote isx = 0.

Example 6.31Graphing a Vertical Shift of the Parent Function y = logb(x)Sketch a graph of f (x) = log3 (x) − 2 alongside its parent function. Include the key points andasymptote on the graph. State the domain, range, and asymptote.

SolutionSince the function is f (x) = log3 (x)− 2,we will notice d = −− 2.Thus d < 0.

This means we will shift the function f (x) = log3 (x) down 2 units.The vertical asymptote isx = 0.Consider the three key points from the parent function,

(13 ,−1

),(1, 0) ,and (3, 1) .

The new coordinates are found by subtracting 2 from the y coordinates.Label the points

(13 ,−3

),(1,−2) , and (3,−1) .

The domain is (0,∞) ,the range is (−∞,∞) , and the vertical asymptote isx = 0.

Available for free at Connexions <http://legacy.cnx.org/content/col11759/1.2>

424 CHAPTER 6. EXPONENTIAL AND LOGARITHMIC FUNCTIONS

Figure 6.31

The domain is (0,∞) ,the range is (−∞,∞) ,and the vertical asymptote isx = 0.

Try It:

Available for free at Connexions <http://legacy.cnx.org/content/col11759/1.2>

425

Exercise 6.5.5 (Solution on p. 558.)

Sketch a graph of f (x) = log2 (x)+2 alongside its parent function. Include the key pointsand asymptote on the graph. State the domain, range, and asymptote.

6.5.2.4 Graphing Stretches and Compressions of y = logb(x)

When the parent function f (x) = logb (x) is multiplied by a constant a > 0, the result is a vertical stretchor compression of the original graph. To visualize stretches and compressions, we set a > 1 and observethe general graph of the parent function f (x) = logb (x) alongside the vertical stretch, g (x) = alogb (x) andthe vertical compression,h (x) = 1

a logb (x) .See Figure 6.32.

Figure 6.32

A General Note: For any constant a > 1,the function f (x) = alogb (x)

• stretches the parent function y = logb (x) vertically by a factor of a if a > 1.• compresses the parent function y = logb (x) vertically by a factor of a if 0 < a < 1.

Available for free at Connexions <http://legacy.cnx.org/content/col11759/1.2>

426 CHAPTER 6. EXPONENTIAL AND LOGARITHMIC FUNCTIONS

• has the vertical asymptotex = 0.• has the x-intercept (1, 0) .• has domain (0,∞) .• has range (−∞,∞) .

How To: Given a logarithmic function with the form f (x) = alogb (x) ,a > 0,graph thetranslation.

1.Identify the vertical stretch or compressions:

• If |a| > 1,the graph of f (x) = logb (x) is stretched by a factor of a units.• If |a| < 1,the graph of f (x) = logb (x) is compressed by a factor of a units.

2.Draw the vertical asymptotex = 0.3.Identify three key points from the parent function. Find new coordinates for the shiftedfunctions by multiplying the y coordinates by a.

4.Label the three points.5.The domain is (0,∞) ,the range is (−∞,∞) ,and the vertical asymptote isx = 0.

Example 6.32Graphing a Stretch or Compression of the Parent Function y = logb(x)Sketch a graph of f (x) = 2log4 (x) alongside its parent function. Include the key points andasymptote on the graph. State the domain, range, and asymptote.

SolutionSince the function is f (x) = 2log4 (x) ,we will notice a = 2.

This means we will stretch the function f (x) = log4 (x) by a factor of 2.The vertical asymptote isx = 0.Consider the three key points from the parent function,

(14 ,−1

),(1, 0) , and (4, 1) .

The new coordinates are found by multiplying the y coordinates by 2.Label the points

(14 ,−2

),(1, 0) , and (4, 2) .

The domain is (0, ∞) , the range is (−∞,∞) , and the vertical asymptote isx = 0. See Fig-ure 6.33.

Available for free at Connexions <http://legacy.cnx.org/content/col11759/1.2>

427

Figure 6.33

The domain is (0,∞) , the range is (−∞,∞) , and the vertical asymptote isx = 0.

Try It:

Exercise 6.5.6 (Solution on p. 559.)

Sketch a graph of f (x) = 12 log4 (x) alongside its parent function. Include the key points

and asymptote on the graph. State the domain, range, and asymptote.

Example 6.33Combining a Shift and a StretchSketch a graph of f (x) = 5log (x+ 2) . State the domain, range, and asymptote.

SolutionRemember: what happens inside parentheses happens �rst. First, we move the graph left 2 units,then stretch the function vertically by a factor of 5, as in Figure 6.34. The vertical asymptote willbe shifted tox = −2.The x-intercept will be (−1, 0) .The domain will be (−2,∞) .Two points will

Available for free at Connexions <http://legacy.cnx.org/content/col11759/1.2>

428 CHAPTER 6. EXPONENTIAL AND LOGARITHMIC FUNCTIONS

help give the shape of the graph: (−1, 0) and (8, 5) .We chosex = 8 as the x-coordinate of one pointto graph because whenx = 8, x+ 2 = 10, the base of the common logarithm.

Figure 6.34

The domain is (−2,∞) ,the range is (−∞,∞) ,and the vertical asymptote isx = −2.

Try It:

Exercise 6.5.7 (Solution on p. 560.)

Sketch a graph of the function f (x) = 3log (x− 2) + 1. State the domain, range, andasymptote.

Available for free at Connexions <http://legacy.cnx.org/content/col11759/1.2>

429

6.5.2.5 Graphing Re�ections of f (x) = logb(x)

When the parent function f (x) = logb (x) is multiplied by − 1,the result is a re�ection about the x-axis.When the input is multiplied by − 1,the result is a re�ection about the y-axis. To visualize re�ections, werestrict b > 1, and observe the general graph of the parent function f (x) = logb (x) alongside the re�ectionabout the x-axis, g (x) = −logb (x) and the re�ection about the y-axis,h (x) = logb (−x) .

Figure 6.35

A General Note: The function f (x) = −logb (x)

• re�ects the parent function y = logb (x) about the x-axis.• has domain, (0,∞) , range, (−∞,∞) , and vertical asymptote,x = 0, which are unchangedfrom the parent function.

The function f (x) = logb (−x)

• re�ects the parent function y = logb (x) about the y-axis.

Available for free at Connexions <http://legacy.cnx.org/content/col11759/1.2>

430 CHAPTER 6. EXPONENTIAL AND LOGARITHMIC FUNCTIONS

• has domain (−∞, 0) .• has range, (−∞,∞) , and vertical asymptote,x = 0, which are unchanged from the parentfunction.

How To: Given a logarithmic function with the parent function f (x) = logb (x) , grapha translation.

If f (x) = −logb (x) If f (x) = logb (−x)

1.Draw the vertical asymptote,x = 0. 1.Draw the vertical asymptote,x = 0.

2.Plot the x-intercept, (1, 0) . 2.Plot the x-intercept, (1, 0) .

3.Re�ect the graph of the parentfunction f (x) = logb (x) about thex-axis.

3.Re�ect the graph of the parentfunction f (x) = logb (x) about they-axis.

4.Draw a smooth curve through the points. 4.Draw a smooth curve through the points.

5.State the domain, (0,∞) , therange, (−∞,∞) , and the verticalasymptotex = 0.

5.State the domain, (−∞, 0) , therange, (−∞,∞) , and the verticalasymptotex = 0.

Table 6.23

Example 6.34Graphing a Re�ection of a Logarithmic FunctionSketch a graph of f (x) = log (−x) alongside its parent function. Include the key points andasymptote on the graph. State the domain, range, and asymptote.

SolutionBefore graphing f (x) = log (−x) ,identify the behavior and key points for the graph.

• Since b = 10 is greater than one, we know that the parent function is increasing. Sincethe input value is multiplied by − 1,f is a re�ection of the parent graph about the y-axis.Thus, f (x) = log (−x) will be decreasing asxmoves from negative in�nity to zero, and theright tail of the graph will approach the vertical asymptotex = 0.

• The x-intercept is (−1, 0) .• We draw and label the asymptote, plot and label the points, and draw a smooth curve through

the points.

Available for free at Connexions <http://legacy.cnx.org/content/col11759/1.2>

431

Figure 6.36

The domain is (−∞, 0) ,the range is (−∞,∞) ,and the vertical asymptote isx = 0.

Try It:

Exercise 6.5.8 (Solution on p. 561.)

Graph f (x) = −log (−x) . State the domain, range, and asymptote.

How To: Given a logarithmic equation, use a graphing calculator to approximatesolutions.

• Press [Y=]. Enter the given logarithm equation or equations as Y1= and, if needed, Y2=.• Press [GRAPH] to observe the graphs of the curves and use [WINDOW] to �nd an appro-priate view of the graphs, including their point(s) of intersection.• To �nd the value ofx, we compute the point of intersection. Press [2ND] then [CALC].Select �intersect� and press [ENTER] three times. The point of intersection gives the valueofx,for the point(s) of intersection.

Available for free at Connexions <http://legacy.cnx.org/content/col11759/1.2>

432 CHAPTER 6. EXPONENTIAL AND LOGARITHMIC FUNCTIONS

Example 6.35Approximating the Solution of a Logarithmic EquationSolve 4ln (x) + 1 = −2ln (x− 1) graphically. Round to the nearest thousandth.

SolutionPress [Y=] and enter 4ln (x) + 1 next to Y1=. Then enter − 2ln (x− 1) next to Y2=. Fora window, use the values 0 to 5 forx and �10 to 10 for y.Press [GRAPH]. The graphs shouldintersect somewhere a little to right ofx = 1.

For a better approximation, press [2ND] then [CALC]. Select [5: intersect] and press [EN-TER] three times. The x-coordinate of the point of intersection is displayed as 1.3385297. (Youranswer may be di�erent if you use a di�erent window or use a di�erent value for Guess?) So, tothe nearest thousandth,x ≈ 1.339.

Try It:

Exercise 6.5.9 (Solution on p. 562.)

Solve 5log (x+ 2) = 4− log (x) graphically. Round to the nearest thousandth.

6.5.2.6 Summarizing Translations of the Logarithmic Function

Now that we have worked with each type of translation for the logarithmic function, we can summarize eachin Table 6.24 to arrive at the general equation for translating exponential functions.

Translations of the Parent Function y = logb (x)

Translation Form

Shift

• Horizontally c units to the left• Vertically d units up

y = logb (x+ c) + d

Stretch and Compress

• Stretch if |a| > 1• Compression if |a| < 1

y = alogb (x)

continued on next page

Available for free at Connexions <http://legacy.cnx.org/content/col11759/1.2>

433

Re�ect about the x-axis y = −logb (x)

Re�ect about the y-axis y = logb (−x)

General equation for all translations y = alogb (x+ c) + d

Table 6.24

A General Note: All translations of the parent logarithmic function, y = logb (x) , have theform

f (x) = alogb (x+ c) + d (6.34)

where the parent function, y = logb (x) , b > 1,is

• shifted vertically up d units.• shifted horizontally to the left c units.• stretched vertically by a factor of |a| if |a| > 0.• compressed vertically by a factor of |a| if 0 < |a| < 1.• re�ected about the x-axis when a < 0.

For f (x) = log (−x) , the graph of the parent function is re�ected about the y-axis.

Example 6.36Finding the Vertical Asymptote of a Logarithm GraphWhat is the vertical asymptote of f (x) = −2log3 (x+ 4) + 5?

SolutionThe vertical asymptote is atx = −4.

AnalysisThe coe�cient, the base, and the upward translation do not a�ect the asymptote. The shift of thecurve 4 units to the left shifts the vertical asymptote tox = −4.

Try It:

Exercise 6.5.10 (Solution on p. 562.)

What is the vertical asymptote of f (x) = 3 + ln (x− 1)?

Example 6.37Finding the Equation from a GraphFind a possible equation for the common logarithmic function graphed in Figure 6.37.

Available for free at Connexions <http://legacy.cnx.org/content/col11759/1.2>

434 CHAPTER 6. EXPONENTIAL AND LOGARITHMIC FUNCTIONS

Figure 6.37

SolutionThis graph has a vertical asymptote atx = − − 2 and has been vertically re�ected. We do notknow yet the vertical shift or the vertical stretch. We know so far that the equation will have form:

f (x) = −alog (x+ 2) + k (6.35)

It appears the graph passes through the points (−− 1, 1) and (2,−− 1) . Substituting (−− 1, 1) ,

1 = −alog (−1 + 2) + k Substitute (−1, 1) .

1 = −alog (1) + k Arithmetic.

1 = k log(1) = 0.

(6.36)

Next, substituting in (2,−− 1),

−1 = −alog (2 + 2) + 1 Plug in (2,−1) .

−2 = −alog (4) Arithmetic.

a = 2log(4) Solve for a.

(6.37)

Available for free at Connexions <http://legacy.cnx.org/content/col11759/1.2>

435

This gives us the equation f (x) = −− 2log(4) log (x+ 2) + 1.

AnalysisWe can verify this answer by comparing the function values in Table 6.25 with the points on thegraph in Figure 6.37.

x −1 0 1 2 3

f (x) 1 0 −0.58496 −1 −1.3219x 4 5 6 7 8

f (x) −1.5850 −1.8074 −2 −2.1699 −2.3219

Table 6.25

Try It:

Exercise 6.5.11 (Solution on p. 562.)

Give the equation of the natural logarithm graphed in Figure 6.38.

Available for free at Connexions <http://legacy.cnx.org/content/col11759/1.2>

436 CHAPTER 6. EXPONENTIAL AND LOGARITHMIC FUNCTIONS

Figure 6.38

Q&A: Is it possible to tell the domain and range and describe the end behavior of afunction just by looking at the graph?

Yes, if we know the function is a general logarithmic function. For example, look at the graph inFigure 6.38. The graph approachesx = −3 (or thereabouts) more and more closely, sox = −3 is, oris very close to, the vertical asymptote. It approaches from the right, so the domain is all pointsto the right, {x |x > − 3}.The range, as with all general logarithmic functions, is all real numbers.And we can see the end behavior because the graph goes down as it goes left and up as it goesright. The end behavior is that asx→ −3+, f (x)→ −∞ and asx→∞, f (x)→∞.

Media: Access these online resources for additional instruction and practice with graphing loga-rithms.

Available for free at Connexions <http://legacy.cnx.org/content/col11759/1.2>

437

• Graph an Exponential Function and Logarithmic Function19

• Match Graphs with Exponential and Logarithmic Functions20

• Find the Domain of Logarithmic Functions21

6.5.3 Key Equations

General Form for the Translation of the Par-ent Logarithmic Function f (x) = logb (x)

f (x) = alogb (x+ c) + d

Table 6.26

6.5.4 Key Concepts

• To �nd the domain of a logarithmic function, set up an inequality showing the argument greater thanzero, and solve forx. See Example 6.27 and Example 6.28

• The graph of the parent function f (x) = logb (x) has an x-interceptat (1, 0) ,domain (0,∞) ,range (−∞,∞) ,vertical asymptotex = 0,and

· if b > 1,the function is increasing.· if 0 < b < 1, the function is decreasing.

See Example 6.29.• The equation f (x) = logb (x+ c) shifts the parent function y = logb (x) horizontally

· left c units if c > 0.· right c units if c < 0.

See Example 6.30.• The equation f (x) = logb (x) + d shifts the parent function y = logb (x) vertically

· up d units if d > 0.· down d units if d < 0.

See Example 6.31.• For any constant a > 0, the equation f (x) = alogb (x)

· stretches the parent function y = logb (x) vertically by a factor of a if |a| > 1.· compresses the parent function y = logb (x) vertically by a factor of a if |a| < 1.

See Example 6.32 and Example 6.33.• When the parent function y = logb (x) is multiplied by − 1, the result is a re�ection about the x-axis.

When the input is multiplied by − 1, the result is a re�ection about the y-axis.

· The equation f (x) = −logb (x) represents a re�ection of the parent function about the x-axis.· The equation f (x) = logb (−x) represents a re�ection of the parent function about the y-axis.

See Example 6.34.

· A graphing calculator may be used to approximate solutions to some logarithmic equations SeeExample 6.35.

• All translations of the logarithmic function can be summarized by the general equation f (x) =alogb (x+ c) + d. See Table 6.24.

• Given an equation with the general form f (x) = alogb (x+ c) + d,we can identify the verticalasymptotex = −c for the transformation. See Example 6.36.

19http://openstaxcollege.org/l/graphexplog20http://openstaxcollege.org/l/matchexplog21http://openstaxcollege.org/l/domainlog

Available for free at Connexions <http://legacy.cnx.org/content/col11759/1.2>

438 CHAPTER 6. EXPONENTIAL AND LOGARITHMIC FUNCTIONS

• Using the general equation f (x) = alogb (x+ c)+d,we can write the equation of a logarithmic functiongiven its graph. See Example 6.37.

6.5.5 Section Exercises

6.5.5.1 Verbal

Exercise 6.5.12 (Solution on p. 562.)

The inverse of every logarithmic function is an exponential function and vice-versa. What doesthis tell us about the relationship between the coordinates of the points on the graphs of each?

Exercise 6.5.13What type(s) of translation(s), if any, a�ect the range of a logarithmic function?

Exercise 6.5.14 (Solution on p. 562.)

What type(s) of translation(s), if any, a�ect the domain of a logarithmic function?

Exercise 6.5.15Consider the general logarithmic function f (x) = logb (x) .Why can'tx be zero?

Exercise 6.5.16 (Solution on p. 562.)

Does the graph of a general logarithmic function have a horizontal asymptote? Explain.

6.5.5.2 Algebraic

For the following exercises, state the domain and range of the function.

Exercise 6.5.17f (x) = log3 (x+ 4)Exercise 6.5.18 (Solution on p. 562.)

h (x) = ln(

12 − x

)Exercise 6.5.19g (x) = log5 (2x+ 9)− 2Exercise 6.5.20 (Solution on p. 562.)

h (x) = ln (4x+ 17)− 5Exercise 6.5.21f (x) = log2 (12− 3x)− 3

For the following exercises, state the domain and the vertical asymptote of the function.

Exercise 6.5.22 (Solution on p. 562.)

f (x) = logb (x− 5)Exercise 6.5.23g (x) = ln (3− x)Exercise 6.5.24 (Solution on p. 562.)

f (x) = log (3x+ 1)Exercise 6.5.25f (x) = 3log (−x) + 2Exercise 6.5.26 (Solution on p. 562.)

g (x) = −ln (3x+ 9)− 7For the following exercises, state the domain, vertical asymptote, and end behavior of the function.

Exercise 6.5.27f (x) = ln (2− x)

Available for free at Connexions <http://legacy.cnx.org/content/col11759/1.2>

439

Exercise 6.5.28 (Solution on p. 562.)

f (x) = log(x− 3

7

)Exercise 6.5.29h (x) = −log (3x− 4) + 3Exercise 6.5.30 (Solution on p. 562.)

g (x) = ln (2x+ 6)− 5Exercise 6.5.31f (x) = log3 (15− 5x) + 6

For the following exercises, state the domain, range, and x- and y-intercepts, if they exist. If they do notexist, write DNE.

Exercise 6.5.32 (Solution on p. 563.)

h (x) = log4 (x− 1) + 1Exercise 6.5.33f (x) = log (5x+ 10) + 3Exercise 6.5.34 (Solution on p. 563.)

g (x) = ln (−x)− 2Exercise 6.5.35f (x) = log2 (x+ 2)− 5Exercise 6.5.36 (Solution on p. 563.)

h (x) = 3ln (x)− 9

6.5.5.3 Graphical

For the following exercises, match each function in Figure 6.39 with the letter corresponding to its graph.

Available for free at Connexions <http://legacy.cnx.org/content/col11759/1.2>

440 CHAPTER 6. EXPONENTIAL AND LOGARITHMIC FUNCTIONS

Figure 6.39

Exercise 6.5.37d (x) = log (x)Exercise 6.5.38 (Solution on p. 563.)

f (x) = ln (x)Exercise 6.5.39g (x) = log2 (x)Exercise 6.5.40 (Solution on p. 563.)

h (x) = log5 (x)Exercise 6.5.41j (x) = log25 (x)

Available for free at Connexions <http://legacy.cnx.org/content/col11759/1.2>

441

For the following exercises, match each function in Figure 6.40 with the letter corresponding to its graph.

Figure 6.40

Exercise 6.5.42 (Solution on p. 563.)

f (x) = log 13

(x)Exercise 6.5.43g (x) = log2 (x)Exercise 6.5.44 (Solution on p. 563.)

h (x) = log 34

(x)For the following exercises, sketch the graphs of each pair of functions on the same axis.

Exercise 6.5.45f (x) = log (x) and g (x) = 10x

Available for free at Connexions <http://legacy.cnx.org/content/col11759/1.2>

442 CHAPTER 6. EXPONENTIAL AND LOGARITHMIC FUNCTIONS

Exercise 6.5.46 (Solution on p. 563.)

f (x) = log (x) and g (x) = log 12

(x)Exercise 6.5.47f (x) = log4 (x) and g (x) = ln (x)Exercise 6.5.48 (Solution on p. 563.)

f (x) = ex and g (x) = ln (x)For the following exercises, match each function in Figure 6.41 with the letter corresponding to its graph.

Figure 6.41

Exercise 6.5.49f (x) = log4 (−x+ 2)Exercise 6.5.50 (Solution on p. 564.)

g (x) = −log4 (x+ 2)Exercise 6.5.51h (x) = log4 (x+ 2)

For the following exercises, sketch the graph of the indicated function.

Available for free at Connexions <http://legacy.cnx.org/content/col11759/1.2>

443

Exercise 6.5.52 (Solution on p. 564.)

f (x) = log2 (x+ 2)Exercise 6.5.53f (x) = 2log (x)Exercise 6.5.54 (Solution on p. 564.)

f (x) = ln (−x)Exercise 6.5.55g (x) = log (4x+ 16) + 4Exercise 6.5.56 (Solution on p. 565.)

g (x) = log (6− 3x) + 1Exercise 6.5.57h (x) = − 1

2 ln (x+ 1)− 3For the following exercises, write a logarithmic equation corresponding to the graph shown.

Exercise 6.5.58 (Solution on p. 565.)

Use y = log2 (x) as the parent function.

Exercise 6.5.59Use f (x) = log3 (x) as the parent function.

Available for free at Connexions <http://legacy.cnx.org/content/col11759/1.2>

444 CHAPTER 6. EXPONENTIAL AND LOGARITHMIC FUNCTIONS

Exercise 6.5.60 (Solution on p. 566.)

Use f (x) = log4 (x) as the parent function.

Available for free at Connexions <http://legacy.cnx.org/content/col11759/1.2>

445

Exercise 6.5.61Use f (x) = log5 (x) as the parent function.

Available for free at Connexions <http://legacy.cnx.org/content/col11759/1.2>

446 CHAPTER 6. EXPONENTIAL AND LOGARITHMIC FUNCTIONS

6.5.5.4 Technology

For the following exercises, use a graphing calculator to �nd approximate solutions to each equation.

Exercise 6.5.62 (Solution on p. 566.)

log (x− 1) + 2 = ln (x− 1) + 2Exercise 6.5.63log (2x− 3) + 2 = −log (2x− 3) + 5Exercise 6.5.64 (Solution on p. 566.)

ln (x− 2) = −ln (x+ 1)Exercise 6.5.652ln (5x+ 1) = 1

2 ln (−5x) + 1Exercise 6.5.66 (Solution on p. 566.)

13 log (1− x) = log (x+ 1) + 1

3

Available for free at Connexions <http://legacy.cnx.org/content/col11759/1.2>

447

6.5.5.5 Extensions

Exercise 6.5.67Let b be any positive real number such that b 6= 1.What must logb1 be equal to? Verify the result.

Exercise 6.5.68 (Solution on p. 566.)

Explore and discuss the graphs of f (x) = log 12

(x) and g (x) = −log2 (x) .Make a conjecture basedon the result.

Exercise 6.5.69Prove the conjecture made in the previous exercise.

Exercise 6.5.70 (Solution on p. 566.)

What is the domain of the function f (x) = ln(x+2x−4

)?Discuss the result.

Exercise 6.5.71Use properties of exponents to �nd the x-intercepts of the function f (x) =

log(x2 + 4x+ 4

)algebraically. Show the steps for solving, and then verify the result by

graphing the function.

Available for free at Connexions <http://legacy.cnx.org/content/col11759/1.2>

448 CHAPTER 6. EXPONENTIAL AND LOGARITHMIC FUNCTIONS

6.6 Logarithmic Properties22

Figure 6.42: The pH of hydrochloric acid is tested with litmus paper. (credit: David Berardan)

In chemistry, pH is used as a measure of the acidity or alkalinity of a substance. The pH scale runs from0 to 14. Substances with a pH less than 7 are considered acidic, and substances with a pH greater than 7are said to be alkaline. Our bodies, for instance, must maintain a pH close to 7.35 in order for enzymes towork properly. To get a feel for what is acidic and what is alkaline, consider the following pH levels of somecommon substances:

• Battery acid: 0.8• Stomach acid: 2.7

22This content is available online at <http://legacy.cnx.org/content/m49365/1.4/>.

Available for free at Connexions <http://legacy.cnx.org/content/col11759/1.2>

449

• Orange juice: 3.3• Pure water: 7 (at 25 ◦ C)• Human blood: 7.35• Fresh coconut: 7.8• Sodium hydroxide (lye): 14

To determine whether a solution is acidic or alkaline, we �nd its pH, which is a measure of the number ofactive positive hydrogen ions in the solution. The pH is de�ned by the following formula, where a is theconcentration of hydrogen ion in the solution

pH = −log ([H+])

= log(

1[H+]

) (6.38)

The equivalence of − log ([H+]) and log(

1[H+]

)is one of the logarithm properties we will examine in this

section.

6.6.1 Using the Product Rule for Logarithms

Recall that the logarithmic and exponential functions �undo� each other. This means that logarithms havesimilar properties to exponents. Some important properties of logarithms are given here. First, the followingproperties are easy to prove.

logb1 = 0

logbb = 1(6.39)

For example, log51 = 0 since 50 = 1.And log55 = 1 since 51 = 5.Next, we have the inverse property.

logb (bx) = x

blogbx = x, x > 0

(6.40)

For example, to evaluate log (100) ,we can rewrite the logarithm as log10

(102), and then apply the inverse

property logb (bx) = x to get log10

(102)

= 2.To evaluate eln(7),we can rewrite the logarithm as eloge7, and then apply the inverse property blogbx = x to

get eloge7 = 7.Finally, we have the one-to-one property.

logbM = logbN if and only if M = N (6.41)

We can use the one-to-one property to solve the equation log3 (3x) = log3 (2x+ 5) forx. Since the bases arethe same, we can apply the one-to-one property by setting the arguments equal and solving forx :

3x = 2x+ 5 Set the arguments equal.

x = 5 Subtract 2x.(6.42)

But what about the equation log3 (3x) + log3 (2x+ 5) = 2?The one-to-one property does not help us in thisinstance. Before we can solve an equation like this, we need a method for combining terms on the left sideof the equation.

Available for free at Connexions <http://legacy.cnx.org/content/col11759/1.2>

450 CHAPTER 6. EXPONENTIAL AND LOGARITHMIC FUNCTIONS

Recall that we use the product rule of exponents to combine the product of exponents by adding:xaxb =xa+b.We have a similar property for logarithms, called the product rule for logarithms, which says thatthe logarithm of a product is equal to a sum of logarithms. Because logs are exponents, and we multiply likebases, we can add the exponents. We will use the inverse property to derive the product rule below.

Given any real numberx and positive real numbers M,N, and b, where b 6= 1, we will show

logb (MN)=logb (M) + logb (N) . (6.43)

Letm = logbM andn = logbN. In exponential form, these equations are bm = M and bn = N. It follows that

logb (MN) = logb (bmbn) Substitute for M and N.

= logb (bm+n) Apply the product rule for exponents.

= m+ n Apply the inverse property of logs.

= logb (M) + logb (N) Substitute for m and n.

(6.44)

Note that repeated applications of the product rule for logarithms allow us to simplify the logarithm of theproduct of any number of factors. For example, consider logb (wxyz) .Using the product rule for logarithms,we can rewrite this logarithm of a product as the sum of logarithms of its factors:

logb (wxyz) = logbw + logbx+ logby + logbz (6.45)

A General Note: The product rule for logarithms can be used to simplify a logarithm ofa product by rewriting it as a sum of individual logarithms.

logb (MN) = logb (M) + logb (N) for b > 0 (6.46)

How To: Given the logarithm of a product, use the product rule of logarithms towrite an equivalent sum of logarithms.

1.Factor the argument completely, expressing each whole number factor as a product of primes.2.Write the equivalent expression by summing the logarithms of each factor.

Example 6.38Using the Product Rule for LogarithmsExpand log3 (30x (3x+ 4)) .

SolutionWe begin by factoring the argument completely, expressing 30 as a product of primes.

log3 (30x (3x+ 4)) = log3 (2 · 3 · 5 · x · (3x+ 4)) (6.47)

Next we write the equivalent equation by summing the logarithms of each factor.

log3 (30x (3x+ 4)) = log3 (2) + log3 (3) + log3 (5) + log3 (x) + log3 (3x+ 4) (6.48)

Available for free at Connexions <http://legacy.cnx.org/content/col11759/1.2>

451

Try It:

Exercise 6.6.1 (Solution on p. 566.)

Expand logb (8k) .

6.6.2 Using the Quotient Rule for Logarithms

For quotients, we have a similar rule for logarithms. Recall that we use the quotient rule of exponents tocombine the quotient of exponents by subtracting:x

ab = xa−b.The quotient rule for logarithms says that

the logarithm of a quotient is equal to a di�erence of logarithms. Just as with the product rule, we can usethe inverse property to derive the quotient rule.

Given any real numberx and positive real numbersM,N, and b, where b 6= 1, we will show

logb

(M

N

)=logb (M)− logb (N) . (6.49)

Letm = logbM andn = logbN. In exponential form, these equations are bm = M and bn = N. It follows that

logb(MN

)= logb

(bm

bn

)Substitute for M and N.

= logb (bm−n) Apply the quotient rule for exponents.

= m− n Apply the inverse property of logs.

= logb (M)− logb (N) Substitute for m and n.

(6.50)

For example, to expand log(

2x2+6x3x+9

),we must �rst express the quotient in lowest terms. Factoring and

canceling we get,

log(

2x2+6x3x+9

)= log

(2x(x+3)3(x+3)

)Factor the numerator and denominator.

= log(

2x3

)Cancel the common factors.

(6.51)

Next we apply the quotient rule by subtracting the logarithm of the denominator from the logarithm of thenumerator. Then we apply the product rule.

log(

2x3

)= log (2x)− log (3)

= log (2) + log (x)− log (3)(6.52)

A General Note: The quotient rule for logarithms can be used to simplify a logarithm ora quotient by rewriting it as the di�erence of individual logarithms.

logb

(M

N

)= logbM − logbN (6.53)

How To: Given the logarithm of a quotient, use the quotient rule of logarithms towrite an equivalent di�erence of logarithms.

1.Express the argument in lowest terms by factoring the numerator and denominator and can-celing common terms.

Available for free at Connexions <http://legacy.cnx.org/content/col11759/1.2>

452 CHAPTER 6. EXPONENTIAL AND LOGARITHMIC FUNCTIONS

2.Write the equivalent expression by subtracting the logarithm of the denominator from thelogarithm of the numerator.

3.Check to see that each term is fully expanded. If not, apply the product rule for logarithmsto expand completely.

Example 6.39Using the Quotient Rule for Logarithms

Expand log2

(15x(x−1)

(3x+4)(2−x)

).

SolutionFirst we note that the quotient is factored and in lowest terms, so we apply the quotient rule.

log2

(15x (x− 1)

(3x+ 4) (2− x)

)= log2 (15x (x− 1))− log2 ((3x+ 4) (2− x)) (6.54)

Notice that the resulting terms are logarithms of products. To expand completely, we apply theproduct rule, noting that the prime factors of the factor 15 are 3 and 5.

log2 (15x (x− 1))− log2 ((3x + 4) (2− x)) = [log2 (3) + log2 (5) + log2 (x) + log2 (x− 1)]− [log2 (3x + 4) + log2 (2− x)]

= log2 (3) + log2 (5) + log2 (x) + log2 (x− 1)− log2 (3x + 4)− log2 (2− x)(6.55)

AnalysisThere are exceptions to consider in this and later examples. First, because denominators must neverbe zero, this expression is not de�ned forx = − 4

3 andx = 2.Also, since the argument of a logarithmmust be positive, we note as we observe the expanded logarithm, thatx > 0,x > 1,x > − 4

3 ,andx < 2.Combining these conditions is beyond the scope of this section, and we will not considerthem here or in subsequent exercises.

Try It:

Exercise 6.6.2 (Solution on p. 566.)

Expand log3

(7x2+21x

7x(x−1)(x−2)

).

6.6.3 Using the Power Rule for Logarithms

We've explored the product rule and the quotient rule, but how can we take the logarithm of a power, suchasx2?One method is as follows:

logb(x2)

= logb (x · x)

= logbx+ logbx

= 2logbx

(6.56)

Notice that we used the product rule for logarithms to �nd a solution for the example above. By doingso, we have derived the power rule for logarithms, which says that the log of a power is equal to the

Available for free at Connexions <http://legacy.cnx.org/content/col11759/1.2>

453

exponent times the log of the base. Keep in mind that, although the input to a logarithm may not be writtenas a power, we may be able to change it to a power. For example,

100 = 102√

3 = 312 1

e = e−1 (6.57)

A General Note: The power rule for logarithms can be used to simplify the logarithm ofa power by rewriting it as the product of the exponent times the logarithm of the base.

logb (Mn) = nlogbM (6.58)

How To: Given the logarithm of a power, use the power rule of logarithms to writean equivalent product of a factor and a logarithm.

1.Express the argument as a power, if needed.2.Write the equivalent expression by multiplying the exponent times the logarithm of the base.

Example 6.40Expanding a Logarithm with PowersExpand log2x

5.

SolutionThe argument is already written as a power, so we identify the exponent, 5, and the base,x, andrewrite the equivalent expression by multiplying the exponent times the logarithm of the base.

log2

(x5)

= 5log2x (6.59)

Try It:

Exercise 6.6.3 (Solution on p. 566.)

Expand lnx2.

Example 6.41Rewriting an Expression as a Power before Using the Power RuleExpand log3 (25) using the power rule for logs.

SolutionExpressing the argument as a power, we get log3 (25) = log3

(52).

Next we identify the exponent, 2, and the base, 5, and rewrite the equivalent expression bymultiplying the exponent times the logarithm of the base.

log3

(52)

= 2log3 (5) (6.60)

Available for free at Connexions <http://legacy.cnx.org/content/col11759/1.2>

454 CHAPTER 6. EXPONENTIAL AND LOGARITHMIC FUNCTIONS

Try It:

Exercise 6.6.4 (Solution on p. 566.)

Expand ln(

1x2

).

Example 6.42Using the Power Rule in ReverseRewrite 4ln (x) using the power rule for logs to a single logarithm with a leading coe�cient of 1.

SolutionBecause the logarithm of a power is the product of the exponent times the logarithm of the base,it follows that the product of a number and a logarithm can be written as a power. For theexpression 4ln (x) ,we identify the factor, 4, as the exponent and the argument,x, as the base, andrewrite the product as a logarithm of a power: 4ln (x) = ln

(x4).

Try It:

Exercise 6.6.5 (Solution on p. 567.)

Rewrite 2log34 using the power rule for logs to a single logarithm with a leading coe�cientof 1.

6.6.4 Expanding Logarithmic Expressions

Taken together, the product rule, quotient rule, and power rule are often called �laws of logs.� Sometimeswe apply more than one rule in order to simplify an expression. For example:

logb(

6xy

)= logb (6x)− logby

= logb6 + logbx− logby(6.61)

We can use the power rule to expand logarithmic expressions involving negative and fractional exponents.Here is an alternate proof of the quotient rule for logarithms using the fact that a reciprocal is a negativepower:

logb(AC

)= logb

(AC−1

)= logb (A) + logb

(C−1

)= logbA+ (−1) logbC

= logbA− logbC

(6.62)

We can also apply the product rule to express a sum or di�erence of logarithms as the logarithm of aproduct.

With practice, we can look at a logarithmic expression and expand it mentally, writing the �nal answer.Remember, however, that we can only do this with products, quotients, powers, and roots�never withaddition or subtraction inside the argument of the logarithm.

Example 6.43Expanding Logarithms Using Product, Quotient, and Power Rules

Rewrite ln(x4y7

)as a sum or di�erence of logs.

Available for free at Connexions <http://legacy.cnx.org/content/col11759/1.2>

455

SolutionFirst, because we have a quotient of two expressions, we can use the quotient rule:

ln(x4y

7

)= ln

(x4y)− ln (7) (6.63)

Then seeing the product in the �rst term, we use the product rule:

ln(x4y)− ln (7) = ln

(x4)

+ ln (y)− ln (7) (6.64)

Finally, we use the power rule on the �rst term:

ln(x4)

+ ln (y)− ln (7) = 4ln (x) + ln (y)− ln (7) (6.65)

Try It:

Exercise 6.6.6 (Solution on p. 567.)

Expand log(x2y3

z4

).

Example 6.44Using the Power Rule for Logarithms to Simplify the Logarithm of a Radical Expres-sionExpand log (

√x) .

Solution

log (√x) = logx( 1

2 )

= 12 logx

(6.66)

Try It:

Exercise 6.6.7 (Solution on p. 567.)

Expand ln(

3√x2).

Q&A: Can we expand ln(x2 + y2

)?

No. There is no way to expand the logarithm of a sum or di�erence inside the argument of thelogarithm.

Available for free at Connexions <http://legacy.cnx.org/content/col11759/1.2>

456 CHAPTER 6. EXPONENTIAL AND LOGARITHMIC FUNCTIONS

Example 6.45Expanding Complex Logarithmic Expressions

Expand log6

(64x3(4x+1)

(2x−1)

).

SolutionWe can expand by applying the Product and Quotient Rules.

log6

(64x3(4x+1)

(2x−1)

)= log664 + log6x

3 + log6 (4x + 1)− log6 (2x− 1) Apply the Quotient Rule.

= log626 + log6x

3 + log6 (4x + 1)− log6 (2x− 1) Simplify by writing 64 as 26.

= 6log62 + 3log6x + log6 (4x + 1)− log6 (2x− 1) Apply the Power Rule.

(6.67)

Try It:

Exercise 6.6.8 (Solution on p. 567.)

Expand ln(√

(x−1)(2x+1)2

(x2−9)

).

6.6.5 Condensing Logarithmic Expressions

We can use the rules of logarithms we just learned to condense sums, di�erences, and products with thesame base as a single logarithm. It is important to remember that the logarithms must have the same baseto be combined. We will learn later how to change the base of any logarithm before condensing.

How To: Given a sum, di�erence, or product of logarithms with the same base, writean equivalent expression as a single logarithm.

1.Apply the power property �rst. Identify terms that are products of factors and a logarithm,and rewrite each as the logarithm of a power.

2.Next apply the product property. Rewrite sums of logarithms as the logarithm of a product.3.Apply the quotient property last. Rewrite di�erences of logarithms as the logarithm of aquotient.

Example 6.46Using the Product and Quotient Rules to Combine LogarithmsWrite log3 (5) + log3 (8)− log3 (2) as a single logarithm.

SolutionUsing the product and quotient rules

log3 (5) + log3 (8) = log3 (5 · 8) = log3 (40) (6.68)

This reduces our original expression to

log3 (40)− log3 (2) (6.69)

Then, using the quotient rule

Available for free at Connexions <http://legacy.cnx.org/content/col11759/1.2>

457

log3 (40)− log3 (2) = log3

(402

)= log3 (20) (6.70)

Try It:

Exercise 6.6.9 (Solution on p. 567.)

Condense log3− log4 + log5− log6.

Example 6.47Condensing Complex Logarithmic Expressions

Condense log2

(x2)

+ 12 log2 (x− 1)− 3log2

((x+ 3)2

).

SolutionWe apply the power rule �rst:

log2

(x2)

+12

log2 (x− 1)− 3log2

((x+ 3)2

)= log2

(x2)

+ log2

(√x− 1

)− log2

((x+ 3)6

)(6.71)

Next we apply the product rule to the sum:

log2

(x2)

+ log2

(√x− 1

)− log2

((x+ 3)6

)= log2

(x2√x− 1

)− log2

((x+ 3)6

)(6.72)

Finally, we apply the quotient rule to the di�erence:

log2

(x2√x− 1

)− log2

((x+ 3)6

)= log2

x2√x− 1

(x+ 3)6 (6.73)

Example 6.48Rewriting as a Single LogarithmRewrite 2logx− 4log (x+ 5) + 1

x log (3x+ 5) as a single logarithm.

SolutionWe apply the power rule �rst:

2logx− 4log (x+ 5) +1x

log (3x+ 5) = log(x2)− log

((x+ 5)4

)+ log

((3x+ 5)x

−1)

(6.74)

Next we apply the product rule to the sum:

log(x2)− log

((x+ 5)4

)+ log

((3x+ 5)x

−1)

= log(x2)− log

((x+ 5)4(3x+ 5)x

−1)

(6.75)

Finally, we apply the quotient rule to the di�erence:

log(x2)− log

((x+ 5)4(3x+ 5)x

−1)

= log

x2

(x+ 5)4(

(3x+ 5)x−1) (6.76)

Available for free at Connexions <http://legacy.cnx.org/content/col11759/1.2>

458 CHAPTER 6. EXPONENTIAL AND LOGARITHMIC FUNCTIONS

Try It:

Exercise 6.6.10 (Solution on p. 567.)

Rewrite log (5) + 0.5log (x)− log (7x− 1) + 3log (x− 1) as a single logarithm.

Try It:

Exercise 6.6.11 (Solution on p. 567.)

Condense 4 (3log (x) + log (x+ 5)− log (2x+ 3)) .

Example 6.49Applying of the Laws of LogsRecall that, in chemistry, pH = −log [H+] . If the concentration of hydrogen ions in a liquid isdoubled, what is the e�ect on pH?

SolutionSupposeC is the original concentration of hydrogen ions, andP is the original pH of the liquid.ThenP = − − log (C) . If the concentration is doubled, the new concentration is 2C. Then the pHof the new liquid is

pH = −log (2C) (6.77)

Using the product rule of logs

pH = −log (2C) = − (log (2) + log (C)) = −log (2)− log (C) (6.78)

SinceP = −− log (C) ,the new pH is

pH = P − log (2) ≈ P − 0.301 (6.79)

When the concentration of hydrogen ions is doubled, the pH decreases by about 0.301.

Try It:

Exercise 6.6.12 (Solution on p. 567.)

How does the pH change when the concentration of positive hydrogen ions is decreasedby half?

Available for free at Connexions <http://legacy.cnx.org/content/col11759/1.2>

459

6.6.6 Using the Change-of-Base Formula for Logarithms

Most calculators can evaluate only common and natural logs. In order to evaluate logarithms with a baseother than 10 or e,we use the change-of-base formula to rewrite the logarithm as the quotient of logarithmsof any other base; when using a calculator, we would change them to common or natural logs.

To derive the change-of-base formula, we use the one-to-one property and power rule for logarithms.Given any positive real numbersM, b, and n, wheren 6= 1 and b 6= 1,we show

logbM=lognMlognb

(6.80)

Let y = logbM.By taking the log basen of both sides of the equation, we arrive at an exponential form,namely by = M. It follows that

logn (by) = lognM Apply the one-to-one property.

ylognb = lognM Apply the power rule for logarithms.

y = lognMlognb

Isolate y.

logbM = lognMlognb

Substitute for y.

(6.81)

For example, to evaluate log536 using a calculator, we must �rst rewrite the expression as a quotient ofcommon or natural logs. We will use the common log.

log536 = log(36)log(5) Apply the change of base formula using base 10.

≈ 2.2266 Use a calculator to evaluate to 4 decimal places.(6.82)

A General Note: The change-of-base formula can be used to evaluate a logarithm with anybase.

For any positive real numbersM, b, andn, wheren 6= 1 and b 6= 1,

logbM=lognMlognb

. (6.83)

It follows that the change-of-base formula can be used to rewrite a logarithm with any base as thequotient of common or natural logs.

logbM =lnMlnb

(6.84)

and

logbM =logMlogb

(6.85)

How To: Given a logarithm with the form logbM, use the change-of-base formula torewrite it as a quotient of logs with any positive base n, where n 6= 1.

1.Determine the new basen, remembering that the common log, log (x) , has base 10, and thenatural log, ln (x) ,has base e.

2.Rewrite the log as a quotient using the change-of-base formula

• The numerator of the quotient will be a logarithm with basen and argumentM.

Available for free at Connexions <http://legacy.cnx.org/content/col11759/1.2>

460 CHAPTER 6. EXPONENTIAL AND LOGARITHMIC FUNCTIONS

• The denominator of the quotient will be a logarithm with basen and argument b.

Example 6.50Changing Logarithmic Expressions to Expressions Involving Only Natural LogsChange log53 to a quotient of natural logarithms.

SolutionBecause we will be expressing log53 as a quotient of natural logarithms, the new base,n = e.

We rewrite the log as a quotient using the change-of-base formula. The numerator of the quotientwill be the natural log with argument 3. The denominator of the quotient will be the natural logwith argument 5.

logbM = lnMlnb

log53 = ln3ln5

(6.86)

Try It:

Exercise 6.6.13 (Solution on p. 567.)

Change log0.58 to a quotient of natural logarithms.

Q&A: Can we change common logarithms to natural logarithms?

Yes. Remember that log9means log109. So, log9 = ln9ln10 .

Example 6.51Using the Change-of-Base Formula with a CalculatorEvaluate log2 (10) using the change-of-base formula with a calculator.

SolutionAccording to the change-of-base formula, we can rewrite the log base 2 as a logarithm of anyother base. Since our calculators can evaluate the natural log, we might choose to use the naturallogarithm, which is the log base e.

log210 = ln10ln2 Apply the change of base formula using base e.

≈ 3.3219 Use a calculator to evaluate to 4 decimal places.(6.87)

Try It:

Exercise 6.6.14 (Solution on p. 567.)

Evaluate log5 (100) using the change-of-base formula.

Media: Access these online resources for additional instruction and practice with laws of loga-rithms.

• The Properties of Logarithms2323http://openstaxcollege.org/l/proplog

Available for free at Connexions <http://legacy.cnx.org/content/col11759/1.2>

461

• Expand Logarithmic Expressions24

• Evaluate a Natural Logarithmic Expression25

6.6.7 Key Equations

The Product Rule for Logarithms logb (MN) = logb (M) + logb (N)

The Quotient Rule for Logarithms logb(MN

)= logbM − logbN

The Power Rule for Logarithms logb (Mn) = nlogbM

The Change-of-Base Formula logbM= lognMlognb

n > 0, n 6= 1, b 6= 1

Table 6.27

6.6.8 Key Concepts

• We can use the product rule of logarithms to rewrite the log of a product as a sum of logarithms. SeeExample 6.38.

• We can use the quotient rule of logarithms to rewrite the log of a quotient as a di�erence of logarithms.See Example 6.39.

• We can use the power rule for logarithms to rewrite the log of a power as the product of the exponentand the log of its base. See Example 6.40, Example 6.41, and Example 6.42.

• We can use the product rule, the quotient rule, and the power rule together to combine or expand alogarithm with a complex input. See Example 6.43, Example 6.44, and Example 6.45.

• The rules of logarithms can also be used to condense sums, di�erences, and products with the samebase as a single logarithm. See Example 6.46, Example 6.47, Example 6.48, and Example 6.49.

• We can convert a logarithm with any base to a quotient of logarithms with any other base using thechange-of-base formula. See Example 6.50.

• The change-of-base formula is often used to rewrite a logarithm with a base other than 10 and e as thequotient of natural or common logs. That way a calculator can be used to evaluate. See Example 6.51.

6.6.9 Section Exercises

6.6.9.1 Verbal

Exercise 6.6.15 (Solution on p. 567.)

How does the power rule for logarithms help when solving logarithms with the form logb ( n√x)?

Exercise 6.6.16What does the change-of-base formula do? Why is it useful when using a calculator?

6.6.9.2 Algebraic

For the following exercises, expand each logarithm as much as possible. Rewrite each expression as a sum,di�erence, or product of logs.

Exercise 6.6.17 (Solution on p. 567.)

logb (7x · 2y)

24http://openstaxcollege.org/l/expandlog25http://openstaxcollege.org/l/evaluatelog

Available for free at Connexions <http://legacy.cnx.org/content/col11759/1.2>

462 CHAPTER 6. EXPONENTIAL AND LOGARITHMIC FUNCTIONS

Exercise 6.6.18ln (3ab · 5c)Exercise 6.6.19 (Solution on p. 567.)

logb(

1317

)Exercise 6.6.20log4

(xz

w

)Exercise 6.6.21 (Solution on p. 567.)

ln(

14k

)Exercise 6.6.22log2 (yx)

For the following exercises, condense to a single logarithm if possible.

Exercise 6.6.23 (Solution on p. 567.)

ln (7) + ln (x) + ln (y)Exercise 6.6.24log3 (2) + log3 (a) + log3 (11) + log3 (b)Exercise 6.6.25 (Solution on p. 567.)

logb (28)− logb (7)Exercise 6.6.26ln (a)− ln (d)− ln (c)Exercise 6.6.27 (Solution on p. 567.)

−logb(

17

)Exercise 6.6.28

13 ln (8)

For the following exercises, use the properties of logarithms to expand each logarithm as much as possible.Rewrite each expression as a sum, di�erence, or product of logs.

Exercise 6.6.29 (Solution on p. 567.)

log(x15y13

z19

)Exercise 6.6.30ln(a−2

b−4c5

)Exercise 6.6.31 (Solution on p. 567.)

log(√

x3y−4)

Exercise 6.6.32ln(y√

y1−y

)Exercise 6.6.33 (Solution on p. 567.)

log(x2y3 3

√x2y5

)For the following exercises, condense each expression to a single logarithm using the properties of logarithms.

Exercise 6.6.34log(2x4)

+ log(3x5)

Exercise 6.6.35 (Solution on p. 567.)

ln(6x9)− ln

(3x2)

Exercise 6.6.362log (x) + 3log (x+ 1)Exercise 6.6.37 (Solution on p. 567.)

log (x)− 12 log (y) + 3log (z)

Available for free at Connexions <http://legacy.cnx.org/content/col11759/1.2>

463

Exercise 6.6.384log7 (c) + log7(a)

3 + log7(b)3

For the following exercises, rewrite each expression as an equivalent ratio of logs using the indicated base.

Exercise 6.6.39 (Solution on p. 567.)

log7 (15) to base eExercise 6.6.40log14 (55.875) to base 10

For the following exercises, suppose log5 (6) = a and log5 (11) = b.Use the change-of-base formula along withproperties of logarithms to rewrite each expression in terms of a and b. Show the steps for solving.

Exercise 6.6.41 (Solution on p. 567.)

log11 (5)Exercise 6.6.42log6 (55)Exercise 6.6.43 (Solution on p. 568.)

log11

(611

)

6.6.9.3 Numeric

For the following exercises, use properties of logarithms to evaluate without using a calculator.

Exercise 6.6.44log3

(19

)− 3log3 (3)

Exercise 6.6.45 (Solution on p. 568.)

6log8 (2) + log8(64)3log8(4)

Exercise 6.6.462log9 (3)− 4log9 (3) + log9

(1

729

)For the following exercises, use the change-of-base formula to evaluate each expression as a quotient ofnatural logs. Use a calculator to approximate each to �ve decimal places.

Exercise 6.6.47 (Solution on p. 568.)

log3 (22)Exercise 6.6.48log8 (65)Exercise 6.6.49 (Solution on p. 568.)

log6 (5.38)Exercise 6.6.50log4

(152

)Exercise 6.6.51 (Solution on p. 568.)

log 12

(4.7)

Available for free at Connexions <http://legacy.cnx.org/content/col11759/1.2>

464 CHAPTER 6. EXPONENTIAL AND LOGARITHMIC FUNCTIONS

6.6.9.4 Extensions

Exercise 6.6.52Use the product rule for logarithms to �nd allx values such that log12 (2x+ 6) + log12 (x+ 2) =

2. Show the steps for solving.

Exercise 6.6.53 (Solution on p. 568.)

Use the quotient rule for logarithms to �nd allx values such that log6 (x+ 2)−log6 (x− 3) = 1. Showthe steps for solving.

Exercise 6.6.54Can the power property of logarithms be derived from the power property of exponents using theequation bx = m? If not, explain why. If so, show the derivation.

Exercise 6.6.55 (Solution on p. 568.)

Prove that logb (n) = 1logn(b) for any positive integers b > 1 andn > 1.

Exercise 6.6.56Does log81 (2401) = log3 (7)?Verify the claim algebraically.

Available for free at Connexions <http://legacy.cnx.org/content/col11759/1.2>

465

6.7 Exponential and Logarithmic Equations26

Figure 6.43: Wild rabbits in Australia. The rabbit population grew so quickly in Australia that theevent became known as the �rabbit plague.� (credit: Richard Taylor, Flickr)

In 1859, an Australian landowner named Thomas Austin released 24 rabbits into the wild for hunting.Because Australia had few predators and ample food, the rabbit population exploded. In fewer than tenyears, the rabbit population numbered in the millions.

Uncontrolled population growth, as in the wild rabbits in Australia, can be modeled with exponentialfunctions. Equations resulting from those exponential functions can be solved to analyze and make predic-tions about exponential growth. In this section, we will learn techniques for solving exponential functions.

6.7.1 Using Like Bases to Solve Exponential Equations

The �rst technique involves two functions with like bases. Recall that the one-to-one property of exponentialfunctions tells us that, for any real numbers b, S, andT, where b > 0, b 6= 1,bS = bT if and only ifS = T.

In other words, when an exponential equation has the same base on each side, the exponents mustbe equal. This also applies when the exponents are algebraic expressions. Therefore, we can solve manyexponential equations by using the rules of exponents to rewrite each side as a power with the same base.

26This content is available online at <http://legacy.cnx.org/content/m49366/1.3/>.

Available for free at Connexions <http://legacy.cnx.org/content/col11759/1.2>

466 CHAPTER 6. EXPONENTIAL AND LOGARITHMIC FUNCTIONS

Then, we use the fact that exponential functions are one-to-one to set the exponents equal to one another,and solve for the unknown.

For example, consider the equation 34x−7 = 32x

3 .To solve forx, we use the division property of exponentsto rewrite the right side so that both sides have the common base, 3.Then we apply the one-to-one propertyof exponents by setting the exponents equal to one another and solving forx :

34x−7 = 32x

3

34x−7 = 32x

31 Rewrite 3 as 31.

34x−7 = 32x−1 Use the division property of exponents.

4x− 7 = 2x− 1 Apply the one-to-one property of exponents.

2x = 6 Subtract 2x and add 7 to both sides.

x = 3 Divide by 3.

(6.88)

A General Note: For any algebraic expressionsS and T, and any positive real number b 6= 1,

bS = bT if and only if S = T (6.89)

How To: Given an exponential equation with the form bS = bT , where S and T arealgebraic expressions with an unknown, solve for the unknown.

1.Use the rules of exponents to simplify, if necessary, so that the resulting equation has theform bS = bT .

2.Use the one-to-one property to set the exponents equal.3.Solve the resulting equation,S = T, for the unknown.

Example 6.52Solving an Exponential Equation with a Common BaseSolve 2x−1 = 22x−4.

Solution

2x−1 = 22x−4 The common base is 2.

x− 1 = 2x− 4 By the one-to-one property the exponents must be equal.

x = 3 Solve for x.

(6.90)

Try It:

Exercise 6.7.1 (Solution on p. 568.)

Solve 52x = 53x+2.

Available for free at Connexions <http://legacy.cnx.org/content/col11759/1.2>

467

6.7.1.1 Rewriting Equations So All Powers Have the Same Base

Sometimes the common base for an exponential equation is not explicitly shown. In these cases, we simplyrewrite the terms in the equation as powers with a common base, and solve using the one-to-one property.

For example, consider the equation 256 = 4x−5.We can rewrite both sides of this equation as a powerof 2.Then we apply the rules of exponents, along with the one-to-one property, to solve forx :

256 = 4x−5

28 =(22)x−5

Rewrite each side as a power with base 2.

28 = 22x−10 Use the one-to-one property of exponents.

8 = 2x− 10 Apply the one-to-one property of exponents.

18 = 2x Add 10 to both sides.

x = 9 Divide by 2.

(6.91)

How To: Given an exponential equation with unlike bases, use the one-to-one prop-erty to solve it.

1.Rewrite each side in the equation as a power with a common base.2.Use the rules of exponents to simplify, if necessary, so that the resulting equation has theform bS = bT .

3.Use the one-to-one property to set the exponents equal.4.Solve the resulting equation,S = T, for the unknown.

Example 6.53Solving Equations by Rewriting Them to Have a Common BaseSolve 8x+2 = 16x+1.

Solution

8x+2 = 16x+1(23)x+2 =

(24)x+1

Write 8 and 16 as powers of 2.

23x+6 = 24x+4 To take a power of a power, multiply exponents.

3x+ 6 = 4x+ 4 Use the one-to-one property to set the exponents equal.

x = 2 Solve for x.

(6.92)

Try It:

Exercise 6.7.2 (Solution on p. 568.)

Solve 52x = 253x+2.

Available for free at Connexions <http://legacy.cnx.org/content/col11759/1.2>

468 CHAPTER 6. EXPONENTIAL AND LOGARITHMIC FUNCTIONS

Example 6.54Solving Equations by Rewriting Roots with Fractional Exponents to Have a CommonBaseSolve 25x =

√2.

Solution

25x = 212 Write the square root of 2 as a power of 2.

5x = 12 Use the one-to-one property.

x = 110 Solve for x.

(6.93)

Try It:

Exercise 6.7.3 (Solution on p. 568.)

Solve 5x =√

5.

Q&A: Do all exponential equations have a solution? If not, how can we tell if there isa solution during the problem-solving process?

No. Recall that the range of an exponential function is always positive. While solving the equation,we may obtain an expression that is unde�ned.

Example 6.55Solving an Equation with Positive and Negative PowersSolve 3x+1 = −2.

SolutionThis equation has no solution. There is no real value ofx that will make the equation a truestatement because any power of a positive number is positive.

AnalysisFigure 6.44 shows that the two graphs do not cross so the left side is never equal to the right side.Thus the equation has no solution.

Available for free at Connexions <http://legacy.cnx.org/content/col11759/1.2>

469

Figure 6.44

Try It:

Exercise 6.7.4 (Solution on p. 568.)

Solve 2x = −100.

6.7.2 Solving Exponential Equations Using Logarithms

Sometimes the terms of an exponential equation cannot be rewritten with a common base. In these cases,we solve by taking the logarithm of each side. Recall, since log (a) = log (b) is equivalent to a = b, we may

Available for free at Connexions <http://legacy.cnx.org/content/col11759/1.2>

470 CHAPTER 6. EXPONENTIAL AND LOGARITHMIC FUNCTIONS

apply logarithms with the same base on both sides of an exponential equation.

How To: Given an exponential equation in which a common base cannot be found,solve for the unknown.

1.Apply the logarithm of both sides of the equation.

�If one of the terms in the equation has base 10, use the common logarithm.�If none of the terms in the equation has base 10, use the natural logarithm.

2.Use the rules of logarithms to solve for the unknown.

Example 6.56Solving an Equation Containing Powers of Di�erent BasesSolve 5x+2 = 4x.

Solution

5x+2 = 4x There is no easy way to get the powers to have the same base.

ln5x+2 = ln4x Take ln of both sides.

(x + 2) ln5 = xln4 Use laws of logs.

xln5 + 2ln5 = xln4 Use the distributive law.

xln5− xln4 = −2ln5 Get terms containing x on one side, terms without x on the other.

x (ln5− ln4) = −2ln5 On the left hand side, factor out an x.

xln(

54

)= ln

(125

)Use the laws of logs.

x =ln( 1

25)ln( 5

4)Divide by the coe�cient of x.

(6.94)

Try It:

Exercise 6.7.5 (Solution on p. 568.)

Solve 2x = 3x+1.

Q&A: Is there any way to solve 2x = 3x?

Yes. The solution isx = 0.

6.7.2.1 Equations Containing e

One common type of exponential equations are those with base e.This constant occurs again and again innature, in mathematics, in science, in engineering, and in �nance. When we have an equation with a base e oneither side, we can use the natural logarithm to solve it.

How To: Given an equation of the form y = Aekt, solve for t.

Available for free at Connexions <http://legacy.cnx.org/content/col11759/1.2>

471

1.Divide both sides of the equation byA.2.Apply the natural logarithm of both sides of the equation.3.Divide both sides of the equation by k.

Example 6.57Solve an Equation of the Form y = Aekt

Solve 100 = 20e2t.

Solution

100 = 20e2t

5 = e2t Divide by the coe�cient of the power.

ln5 = 2t Take ln of both sides. Use the fact that ln (x) and ex are inverse functions.

t = ln52 Divide by the coe�cient of t.

(6.95)

AnalysisUsing laws of logs, we can also write this answer in the form t = ln

√5.If we want a decimal

approximation of the answer, we use a calculator.

Try It:

Exercise 6.7.6 (Solution on p. 568.)

Solve 3e0.5t = 11.

Q&A: Does every equation of the form y = Aekt have a solution?

No. There is a solution when k 6= 0,and when y andA are either both 0 or neither 0, and they havethe same sign. An example of an equation with this form that has no solution is 2 = −3et.

Example 6.58Solving an Equation That Can Be Simpli�ed to the Form y = Aekt

Solve 4e2x + 5 = 12.

Solution

4e2x + 5 = 12

4e2x = 7 Combine like terms.

e2x = 74 Divide by the coe�cient of the power.

2x = ln(

74

)Take ln of both sides.

x = 12 ln(

74

)Solve for x.

(6.96)

Available for free at Connexions <http://legacy.cnx.org/content/col11759/1.2>

472 CHAPTER 6. EXPONENTIAL AND LOGARITHMIC FUNCTIONS

Try It:

Exercise 6.7.7 (Solution on p. 568.)

Solve 3 + e2t = 7e2t.

6.7.2.2 Extraneous Solutions

Sometimes the methods used to solve an equation introduce an extraneous solution, which is a solutionthat is correct algebraically but does not satisfy the conditions of the original equation. One such situationarises in solving when the logarithm is taken on both sides of the equation. In such cases, remember thatthe argument of the logarithm must be positive. If the number we are evaluating in a logarithm function isnegative, there is no output.

Example 6.59Solving Exponential Functions in Quadratic FormSolve e2x − ex = 56.

Solution

e2x − ex = 56

e2x − ex − 56 = 0 Get one side of the equation equal to zero.

(ex + 7) (ex − 8) = 0 Factor by the FOIL method.

ex + 7 = 0 or ex − 8 = 0 If a product is zero, then one factor must be zero.

ex = −7 or ex = 8 Isolate the exponentials.

ex = 8 Reject the equation in which the power equals a negative number.

x = ln8 Solve the equation in which the power equals a positive number.

(6.97)

AnalysisWhen we plan to use factoring to solve a problem, we always get zero on one side of the equation,because zero has the unique property that when a product is zero, one or both of the factorsmust be zero. We reject the equation ex = −7 because a positive number never equals a negativenumber. The solutionx = ln (−7) is not a real number, and in the real number system this solutionis rejected as an extraneous solution.

Try It:

Exercise 6.7.8 (Solution on p. 568.)

Solve e2x = ex + 2.

Q&A: Does every logarithmic equation have a solution?

No. Keep in mind that we can only apply the logarithm to a positive number. Always check forextraneous solutions.

Available for free at Connexions <http://legacy.cnx.org/content/col11759/1.2>

473

6.7.3 Using the De�nition of a Logarithm to Solve Logarithmic Equations

We have already seen that every logarithmic equation logb (x) = y is equivalent to the exponentialequation by = x.We can use this fact, along with the rules of logarithms, to solve logarithmic equationswhere the argument is an algebraic expression.

For example, consider the equation log2 (2) + log2 (3x− 5) = 3. To solve this equation, we can use rulesof logarithms to rewrite the left side in compact form and then apply the de�nition of logs to solve forx :

log2 (2) + log2 (3x− 5) = 3

log2 (2 (3x− 5)) = 3 Apply the product rule of logarithms.

log2 (6x− 10) = 3 Distribute.

23 = 6x− 10 Apply the de�nition of a logarithm.

8 = 6x− 10 Calculate 23.

18 = 6x Add 10 to both sides.

x = 3 Divide by 6.

(6.98)

A General Note: For any algebraic expressionS and real numbers b and c,where b > 0, b 6= 1,

logb (S) = c if and only if bc = S (6.99)

Example 6.60Using Algebra to Solve a Logarithmic EquationSolve 2lnx+ 3 = 7.

Solution

2lnx+ 3 = 7

2lnx = 4 Subtract 3.

lnx = 2 Divide by 2.

x = e2 Rewrite in exponential form.

(6.100)

Try It:

Exercise 6.7.9 (Solution on p. 568.)

Solve 6 + lnx = 10.

Example 6.61Using Algebra Before and After Using the De�nition of the Natural LogarithmSolve 2ln (6x) = 7.

Solution

Available for free at Connexions <http://legacy.cnx.org/content/col11759/1.2>

474 CHAPTER 6. EXPONENTIAL AND LOGARITHMIC FUNCTIONS

2ln (6x) = 7

ln (6x) = 72 Divide by 2.

6x = e(72 ) Use the de�nition of ln.

x = 16e

( 72 ) Divide by 6.

(6.101)

Try It:

Exercise 6.7.10 (Solution on p. 568.)

Solve 2ln (x+ 1) = 10.

Example 6.62Using a Graph to Understand the Solution to a Logarithmic EquationSolve lnx = 3.

Solution

lnx = 3

x = e3 Use the de�nition of the natural logarithm.(6.102)

Figure 6.45 represents the graph of the equation. On the graph, the x-coordinate of the point atwhich the two graphs intersect is close to 20. In other words e3 ≈ 20.A calculator gives a betterapproximation: e3 ≈ 20.0855.

Available for free at Connexions <http://legacy.cnx.org/content/col11759/1.2>

475

Figure 6.45: The graphs of y = lnx and y = 3 cross at the point (e3, 3,which is approximately (20.0855,3).

Try It:

Exercise 6.7.11 (Solution on p. 568.)

Use a graphing calculator to estimate the approximate solution to the logarithmicequation 2x = 1000 to 2 decimal places.

6.7.4 Using the One-to-One Property of Logarithms to Solve Logarithmic Equa-

tions

As with exponential equations, we can use the one-to-one property to solve logarithmic equations. Theone-to-one property of logarithmic functions tells us that, for any real numbersx > 0, S > 0, T > 0 and anypositive real number b, where b 6= 1,

logbS = logbT if and only if S = T. (6.103)

For example,

If log2 (x− 1) = log2 (8) , then x− 1 = 8. (6.104)

Available for free at Connexions <http://legacy.cnx.org/content/col11759/1.2>

476 CHAPTER 6. EXPONENTIAL AND LOGARITHMIC FUNCTIONS

So, ifx− 1 = 8,then we can solve forx,and we getx = 9.To check, we can substitutex = 9 into the originalequation: log2 (9− 1) = log2 (8) = 3. In other words, when a logarithmic equation has the same base oneach side, the arguments must be equal. This also applies when the arguments are algebraic expressions.Therefore, when given an equation with logs of the same base on each side, we can use rules of logarithmsto rewrite each side as a single logarithm. Then we use the fact that logarithmic functions are one-to-one toset the arguments equal to one another and solve for the unknown.

For example, consider the equation log (3x− 2)− log (2) = log (x+ 4) .To solve this equation, we can usethe rules of logarithms to rewrite the left side as a single logarithm, and then apply the one-to-one propertyto solve forx :

log (3x− 2)− log (2) = log (x+ 4)

log(

3x−22

)= log (x+ 4) Apply the quotient rule of logarithms.

3x−22 = x+ 4 Apply the one to one property of a logarithm.

3x− 2 = 2x+ 8 Multiply both sides of the equation by 2.

x = 10 Subtract 2x and add 2.

(6.105)

To check the result, substitutex = 10 into log (3x− 2)− log (2) = log (x+ 4) .

log (3 (10)− 2)− log (2) = log ((10) + 4)

log (28)− log (2) = log (14)

log(

282

)= log (14) The solution checks.

(6.106)

A General Note: For any algebraic expressionsS andT and any positive real number b,where b 6= 1,

logbS = logbT if and only if S = T (6.107)

Note, when solving an equation involving logarithms, always check to see if the answer is corrector if it is an extraneous solution.

How To: Given an equation containing logarithms, solve it using the one-to-oneproperty.

1.Use the rules of logarithms to combine like terms, if necessary, so that the resulting equationhas the form logbS = logbT.

2.Use the one-to-one property to set the arguments equal.3.Solve the resulting equation,S = T, for the unknown.

Example 6.63Solving an Equation Using the One-to-One Property of LogarithmsSolve ln

(x2)

= ln (2x+ 3) .

Solution

Available for free at Connexions <http://legacy.cnx.org/content/col11759/1.2>

477

ln (x2) = ln (2x + 3)

x2 = 2x + 3 Use the one-to-one property of the logarithm.

x2 − 2x− 3 = 0 Get zero on one side before factoring.

(x− 3) (x + 1) = 0 Factor using FOIL.

x− 3 = 0 or x + 1 = 0 If a product is zero, one of the factors must be zero.

x = 3 or x = −1 Solve for x.

(6.108)

AnalysisThere are two solutions:x = 3 orx = −1.The solutionx = −1 is negative, but it checks whensubstituted into the original equation because the argument of the logarithm functions is stillpositive.

Try It:

Exercise 6.7.12 (Solution on p. 569.)

Solve ln(x2)

= ln1.

6.7.5 Solving Applied Problems Using Exponential and Logarithmic Equations

In previous sections, we learned the properties and rules for both exponential and logarithmic functions. Wehave seen that any exponential function can be written as a logarithmic function and vice versa. We haveused exponents to solve logarithmic equations and logarithms to solve exponential equations. We are nowready to combine our skills to solve equations that model real-world situations, whether the unknown is inan exponent or in the argument of a logarithm.

One such application is in science, in calculating the time it takes for half of the unstable material in asample of a radioactive substance to decay, called its half-life. Table 6.28 lists the half-life for several of themore common radioactive substances.

Substance Use Half-life

gallium-67 nuclear medicine 80 hours

cobalt-60 manufacturing 5.3 years

technetium-99m nuclear medicine 6 hours

americium-241 construction 432 years

carbon-14 archeological dating 5,715 years

uranium-235 atomic power 703,800,000 years

Table 6.28

Available for free at Connexions <http://legacy.cnx.org/content/col11759/1.2>

478 CHAPTER 6. EXPONENTIAL AND LOGARITHMIC FUNCTIONS

We can see how widely the half-lives for these substances vary. Knowing the half-life of a substance allowsus to calculate the amount remaining after a speci�ed time. We can use the formula for radioactive decay:

A (t) = A0eln(0.5)

T t

A (t) = A0eln(0.5) t

T

A (t) = A0

(eln(0.5)

) tT

A (t) = A0

(12

) tT

(6.109)

where

• A0 is the amount initially present• T is the half-life of the substance• t is the time period over which the substance is studied• y is the amount of the substance present after time t

Example 6.64Using the Formula for Radioactive Decay to Find the Quantity of a SubstanceHow long will it take for ten percent of a 1000-gram sample of uranium-235 to decay?

Solution

y = 1000e ln(0.5)703,800,000 t

900 = 1000eln(0.5)

703,800,000t After 10% decays, 900 grams are left.

0.9 = eln(0.5)

703,800,000t Divide by 1000.

ln (0.9) = ln(e

ln(0.5)703,800,000

t)

Take ln of both sides.

ln (0.9) = ln(0.5)703,800,000 t ln

(eM)

= M

t = 703,800,000× ln(0.9)ln(0.5)years Solve for t.

t ≈ 106,979,777 years

(6.110)

AnalysisTen percent of 1000 grams is 100 grams. If 100 grams decay, the amount of uranium-235 remainingis 900 grams.

Try It:

Exercise 6.7.13 (Solution on p. 569.)

How long will it take before twenty percent of our 1000-gram sample of uranium-235 hasdecayed?

Media: Access these online resources for additional instruction and practice with exponential andlogarithmic equations.

• Solving Logarithmic Equations27

27http://openstaxcollege.org/l/solvelogeq

Available for free at Connexions <http://legacy.cnx.org/content/col11759/1.2>

479

• Solving Exponential Equations with Logarithms28

6.7.6 Key Equations

One-to-one property for exponential func-tions

For any algebraic expressions S and T and any pos-itive real number b, wherebS = bT if and only if S = T.

De�nition of a logarithm For any algebraic expression S and positive realnumbers b and c, where b 6= 1,logb (S) = c if and only if bc = S.

One-to-one property for logarithmic func-tions

For any algebraic expressions S and T and any pos-itive real number b, where b 6= 1,logbS = logbT if and only if S = T.

Table 6.29

6.7.7 Key Concepts

• We can solve many exponential equations by using the rules of exponents to rewrite each side as apower with the same base. Then we use the fact that exponential functions are one-to-one to set theexponents equal to one another and solve for the unknown.

• When we are given an exponential equation where the bases are explicitly shown as being equal, setthe exponents equal to one another and solve for the unknown. See Example 6.52.

• When we are given an exponential equation where the bases are not explicitly shown as being equal,rewrite each side of the equation as powers of the same base, then set the exponents equal to oneanother and solve for the unknown. See Example 6.53, Example 6.54, and Example 6.55.

• When an exponential equation cannot be rewritten with a common base, solve by taking the logarithmof each side. See Example 6.56.

• We can solve exponential equations with base e,by applying the natural logarithm of both sides becauseexponential and logarithmic functions are inverses of each other. See Example 6.57 and Example 6.58.

• After solving an exponential equation, check each solution in the original equation to �nd and eliminateany extraneous solutions. See Example 6.59.

• When given an equation of the form logb (S) = c, whereS is an algebraic expression, we can use thede�nition of a logarithm to rewrite the equation as the equivalent exponential equation bc = S, andsolve for the unknown. See Example 6.60 and Example 6.61.

• We can also use graphing to solve equations with the form logb (S) = c.We graph both equations y =logb (S) and y = c on the same coordinate plane and identify the solution as the x-value of the inter-secting point. See Example 6.62.

• When given an equation of the form logbS = logbT, whereS andT are algebraic expressions, we can usethe one-to-one property of logarithms to solve the equationS = T for the unknown. See Example 6.63.

• Combining the skills learned in this and previous sections, we can solve equations that model realworld situations, whether the unknown is in an exponent or in the argument of a logarithm. SeeExample 6.64.

28http://openstaxcollege.org/l/solveexplog

Available for free at Connexions <http://legacy.cnx.org/content/col11759/1.2>

480 CHAPTER 6. EXPONENTIAL AND LOGARITHMIC FUNCTIONS

6.7.8 Section Exercises

6.7.8.1 Verbal

Exercise 6.7.14 (Solution on p. 569.)

How can an exponential equation be solved?

Exercise 6.7.15When does an extraneous solution occur? How can an extraneous solution be recognized?

Exercise 6.7.16 (Solution on p. 569.)

When can the one-to-one property of logarithms be used to solve an equation? When can it notbe used?

6.7.8.2 Algebraic

For the following exercises, use like bases to solve the exponential equation.

Exercise 6.7.174−3v−2 = 4−v

Exercise 6.7.18 (Solution on p. 569.)

64 · 43x = 16Exercise 6.7.1932x+1 · 3x = 243Exercise 6.7.20 (Solution on p. 569.)

2−3n · 14 = 2n+2

Exercise 6.7.21625 · 53x+3 = 125Exercise 6.7.22 (Solution on p. 569.)

363b

362b = 2162−b

Exercise 6.7.23(164

)3n · 8 = 26

For the following exercises, use logarithms to solve.

Exercise 6.7.24 (Solution on p. 569.)

9x−10 = 1Exercise 6.7.252e6x = 13Exercise 6.7.26 (Solution on p. 569.)

er+10 − 10 = −42Exercise 6.7.272 · 109a = 29Exercise 6.7.28 (Solution on p. 569.)

−8 · 10p+7 − 7 = −24Exercise 6.7.297e3n−5 + 5 = −89Exercise 6.7.30 (Solution on p. 569.)

e−3k + 6 = 44Exercise 6.7.31−5e9x−8 − 8 = −62

Available for free at Connexions <http://legacy.cnx.org/content/col11759/1.2>

481

Exercise 6.7.32 (Solution on p. 569.)

−6e9x+8 + 2 = −74Exercise 6.7.332x+1 = 52x−1

Exercise 6.7.34 (Solution on p. 569.)

e2x − ex − 132 = 0Exercise 6.7.357e8x+8 − 5 = −95Exercise 6.7.36 (Solution on p. 569.)

10e8x+3 + 2 = 8Exercise 6.7.374e3x+3 − 7 = 53Exercise 6.7.38 (Solution on p. 569.)

8e−5x−2 − 4 = −90Exercise 6.7.3932x+1 = 7x−2

Exercise 6.7.40 (Solution on p. 569.)

e2x − ex − 6 = 0Exercise 6.7.413e3−3x + 6 = −31

For the following exercises, use the de�nition of a logarithm to rewrite the equation as an exponentialequation.

Exercise 6.7.42 (Solution on p. 569.)

log(

1100

)= −2

Exercise 6.7.43log324 (18) = 1

2

For the following exercises, use the de�nition of a logarithm to solve the equation.

Exercise 6.7.44 (Solution on p. 569.)

5log7n = 10Exercise 6.7.45−8log9x = 16Exercise 6.7.46 (Solution on p. 569.)

4 + log2 (9k) = 2Exercise 6.7.472log (8n+ 4) + 6 = 10Exercise 6.7.48 (Solution on p. 569.)

10− 4ln (9− 8x) = 6For the following exercises, use the one-to-one property of logarithms to solve.

Exercise 6.7.49ln (10− 3x) = ln (−4x)Exercise 6.7.50 (Solution on p. 569.)

log13 (5n− 2) = log13 (8− 5n)Exercise 6.7.51log (x+ 3)− log (x) = log (74)Exercise 6.7.52 (Solution on p. 569.)

ln (−3x) = ln(x2 − 6x

)Available for free at Connexions <http://legacy.cnx.org/content/col11759/1.2>

482 CHAPTER 6. EXPONENTIAL AND LOGARITHMIC FUNCTIONS

Exercise 6.7.53log4 (6−m) = log43mExercise 6.7.54 (Solution on p. 570.)

ln (x− 2)− ln (x) = ln (54)Exercise 6.7.55log9

(2n2 − 14n

)= log9

(−45 + n2

)Exercise 6.7.56 (Solution on p. 570.)

ln(x2 − 10

)+ ln (9) = ln (10)

For the following exercises, solve each equation forx.

Exercise 6.7.57log (x+ 12) = log (x) + log (12)Exercise 6.7.58 (Solution on p. 570.)

ln (x) + ln (x− 3) = ln (7x)Exercise 6.7.59log2 (7x+ 6) = 3Exercise 6.7.60 (Solution on p. 570.)

ln (7) + ln(2− 4x2

)= ln (14)

Exercise 6.7.61log8 (x+ 6)− log8 (x) = log8 (58)Exercise 6.7.62 (Solution on p. 570.)

ln (3)− ln (3− 3x) = ln (4)Exercise 6.7.63log3 (3x)− log3 (6) = log3 (77)

6.7.8.3 Graphical

For the following exercises, solve the equation forx,if there is a solution. Then graph both sides of theequation, and observe the point of intersection (if it exists) to verify the solution.

Exercise 6.7.64 (Solution on p. 570.)

log9 (x)− 5 = −4Exercise 6.7.65log3 (x) + 3 = 2Exercise 6.7.66 (Solution on p. 570.)

ln (3x) = 2Exercise 6.7.67ln (x− 5) = 1Exercise 6.7.68 (Solution on p. 571.)

log (4) + log (−5x) = 2Exercise 6.7.69−7 + log3 (4− x) = −6Exercise 6.7.70 (Solution on p. 571.)

ln (4x− 10)− 6 = −5Exercise 6.7.71log (4− 2x) = log (−4x)

Available for free at Connexions <http://legacy.cnx.org/content/col11759/1.2>

483

Exercise 6.7.72 (Solution on p. 572.)

log11

(−2x2 − 7x

)= log11 (x− 2)

Exercise 6.7.73ln (2x+ 9) = ln (−5x)Exercise 6.7.74 (Solution on p. 573.)

log9 (3− x) = log9 (4x− 8)Exercise 6.7.75log(x2 + 13

)= log (7x+ 3)

Exercise 6.7.76 (Solution on p. 574.)3

log2(10) − log (x− 9) = log (44)

Exercise 6.7.77ln (x)− ln (x+ 3) = ln (6)

For the following exercises, solve for the indicated value, and graph the situation showing the solution point.

Exercise 6.7.78 (Solution on p. 574.)

An account with an initial deposit of $6,500 earns 7.25% annual interest, compounded continuously.How much will the account be worth after 20 years?

Exercise 6.7.79The formula for measuring sound intensity in decibelsD is de�ned by the equationD =

10log(II0

), where I is the intensity of the sound in watts per square meter and I0 = 10−12 is the

lowest level of sound that the average person can hear. How many decibels are emitted from a jetplane with a sound intensity of 8.3 · 102 watts per square meter?

Exercise 6.7.80 (Solution on p. 575.)

The population of a small town is modeled by the equationP = 1650e0.5t where t is measured inyears. In approximately how many years will the town's population reach 20,000?

6.7.8.4 Technology

For the following exercises, solve each equation by rewriting the exponential expression using the indicatedlogarithm. Then use a calculator to approximatex to 3 decimal places.

Exercise 6.7.811000(1.03)t = 5000 using the common log.

Exercise 6.7.82 (Solution on p. 575.)

e5x = 17 using the natural logExercise 6.7.833(1.04)3t = 8 using the common log

Exercise 6.7.84 (Solution on p. 575.)

34x−5 = 38 using the common log

Exercise 6.7.8550e−0.12t = 10 using the natural log

For the following exercises, use a calculator to solve the equation. Unless indicated otherwise, round allanswers to the nearest ten-thousandth.

Exercise 6.7.86 (Solution on p. 575.)

7e3x−5 + 7.9 = 47Exercise 6.7.87ln (3) + ln (4.4x+ 6.8) = 2

Available for free at Connexions <http://legacy.cnx.org/content/col11759/1.2>

484 CHAPTER 6. EXPONENTIAL AND LOGARITHMIC FUNCTIONS

Exercise 6.7.88 (Solution on p. 575.)

log (−0.7x− 9) = 1 + 5log (5)Exercise 6.7.89Atmospheric pressureP in pounds per square inch is represented by the formulaP = 14.7e−0.21x,where x is the number of miles above sea level. To the nearest foot, how high is the peak of amountain with an atmospheric pressure of 8.369 pounds per square inch? (Hint: there are 5280 feetin a mile)

Exercise 6.7.90 (Solution on p. 575.)

The magnitude M of an earthquake is represented by the equationM = 23 log

(EE0

)whereE is

the amount of energy released by the earthquake in joules andE0 = 104.4 is the assigned minimalmeasure released by an earthquake. To the nearest hundredth, what would the magnitude be of anearthquake releasing 1.4 · 1013 joules of energy?

6.7.8.5 Extensions

Exercise 6.7.91Use the de�nition of a logarithm along with the one-to-one property of logarithms to prove thatblogbx = x.

Exercise 6.7.92 (Solution on p. 576.)

Recall the formula for continually compounding interest, y = Aekt.Use the de�nition of a logarithmalong with properties of logarithms to solve the formula for time t such that t is equal to a singlelogarithm.

Exercise 6.7.93Recall the compound interest formulaA = a

(1 + r

k

)kt.Use the de�nition of a logarithm along with

properties of logarithms to solve the formula for time t.

Exercise 6.7.94 (Solution on p. 576.)

Newton's Law of Cooling states that the temperature T of an object at any time t can be de-scribed by the equationT = Ts + (T0 − Ts) e−kt, where Ts is the temperature of the surroundingenvironment, T0 is the initial temperature of the object, and k is the cooling rate. Use the de�ni-tion of a logarithm along with properties of logarithms to solve the formula for time t such that t isequal to a single logarithm.

Available for free at Connexions <http://legacy.cnx.org/content/col11759/1.2>

485

6.8 Exponential and Logarithmic Models29

Figure 6.46: A nuclear research reactor inside the Neely Nuclear Research Center on the GeorgiaInstitute of Technology campus (credit: Georgia Tech Research Institute)

We have already explored some basic applications of exponential and logarithmic functions. In this section,we explore some important applications in more depth, including radioactive isotopes and Newton's Law ofCooling.

29This content is available online at <http://legacy.cnx.org/content/m49367/1.5/>.

Available for free at Connexions <http://legacy.cnx.org/content/col11759/1.2>

486 CHAPTER 6. EXPONENTIAL AND LOGARITHMIC FUNCTIONS

6.8.1 Modeling Exponential Growth and Decay

In real-world applications, we need to model the behavior of a function. In mathematical modeling, wechoose a familiar general function with properties that suggest that it will model the real-world phenomenonwe wish to analyze. In the case of rapid growth, we may choose the exponential growth function:

y = A0ekt (6.111)

where A0 is equal to the value at time zero, e is Euler's constant, and k is a positive constant that determinesthe rate (percentage) of growth. We may use the exponential growth function in applications involvingdoubling time, the time it takes for a quantity to double. Such phenomena as wildlife populations, �nancialinvestments, biological samples, and natural resources may exhibit growth based on a doubling time. In someapplications, however, as we will see when we discuss the logistic equation, the logistic model sometimes �tsthe data better than the exponential model.

On the other hand, if a quantity is falling rapidly toward zero, without ever reaching zero, then we shouldprobably choose the exponential decay model. Again, we have the form y = A0e

kt where A0 is the startingvalue, and e is Euler's constant. Now k is a negative constant that determines the rate of decay. We mayuse the exponential decay model when we are calculating half-life, or the time it takes for a substance toexponentially decay to half of its original quantity. We use half-life in applications involving radioactiveisotopes.

In our choice of a function to serve as a mathematical model, we often use data points gathered by carefulobservation and measurement to construct points on a graph and hope we can recognize the shape of thegraph. Exponential growth and decay graphs have a distinctive shape, as we can see in Figure 6.47 andFigure 6.48. It is important to remember that, although parts of each of the two graphs seem to lie on thex-axis, they are really a tiny distance above the x-axis.

Available for free at Connexions <http://legacy.cnx.org/content/col11759/1.2>

487

Figure 6.47: A graph showing exponential growth. The equation is y = 2e3x.

Available for free at Connexions <http://legacy.cnx.org/content/col11759/1.2>

488 CHAPTER 6. EXPONENTIAL AND LOGARITHMIC FUNCTIONS

Figure 6.48: A graph showing exponential decay. The equation is y = 3e−2x.

Exponential growth and decay often involve very large or very small numbers. To describe these numbers,we often use orders of magnitude. The order of magnitude is the power of ten, when the number isexpressed in scienti�c notation, with one digit to the left of the decimal. For example, the distance to thenearest star, Proxima Centauri, measured in kilometers, is 40,113,497,200,000 kilometers. Expressed inscienti�c notation, this is 4.01134972 × 1013. So, we could describe this number as having order of magnitude1013.

A General Note: An exponential function with the form y = A0ekt has the following charac-

teristics:

• one-to-one function• horizontal asymptote: y = 0

Available for free at Connexions <http://legacy.cnx.org/content/col11759/1.2>

489

• domain: (−−∞, ∞)• range: (0,∞)• x intercept: none• y-intercept: (0, A0)• increasing if k > 0 (see Figure 6.49)• decreasing if k < 0 (see Figure 6.49)

Figure 6.49: An exponential function models exponential growth when k > 0 and exponential decaywhen k < 0.

Example 6.65Graphing Exponential GrowthA population of bacteria doubles every hour. If the culture started with 10 bacteria, graph thepopulation as a function of time.

SolutionWhen an amount grows at a �xed percent per unit time, the growth is exponential. To �nd A0 weuse the fact that A0 is the amount at time zero, so A0 = 10. To �nd k, use the fact that after onehour (t = 1) the population doubles from 10 to 20. The formula is derived as follows

20 = 10ek·1

2 = ek Divide by 10

ln2 = k Take the natural logarithm

(6.112)

so k = ln (2) . Thus the equation we want to graph is y = 10e(ln2)t = 10(eln2)t = 10 · 2t.The graph

is shown in Figure 6.50.

Available for free at Connexions <http://legacy.cnx.org/content/col11759/1.2>

490 CHAPTER 6. EXPONENTIAL AND LOGARITHMIC FUNCTIONS

Figure 6.50: The graph of y = 10e(ln2)t.

AnalysisThe population of bacteria after ten hours is 10,240. We could describe this amount is being ofthe order of magnitude 104. The population of bacteria after twenty hours is 10,485,760 which is ofthe order of magnitude 107, so we could say that the population has increased by three orders ofmagnitude in ten hours.

Available for free at Connexions <http://legacy.cnx.org/content/col11759/1.2>

491

6.8.1.1 Half-Life

We now turn to exponential decay. One of the common terms associated with exponential decay, asstated above, is half-life, the length of time it takes an exponentially decaying quantity to decrease to halfits original amount. Every radioactive isotope has a half-life, and the process describing the exponentialdecay of an isotope is called radioactive decay.

To �nd the half-life of a function describing exponential decay, solve the following equation:

12A0 = Aoe

kt (6.113)

We �nd that the half-life depends only on the constant k and not on the starting quantity A0.The formula is derived as follows

12A0 = Aoe

kt

12 = ekt Divide by A0.

ln(

12

)= kt Take the natural log.

−ln (2) = kt Apply laws of logarithms.

− ln(2)k = t Divide by k.

(6.114)

Since t, the time, is positive, k must, as expected, be negative. This gives us the half-life formula

t = − ln (2)k

(6.115)

How To: Given the half-life, �nd the decay rate.

1.Write A = Aoekt.

2.Replace A by 12A0 and replace t by the given half-life.

3.Solve to �nd k. Express k as an exact value (do not round).

Note: It is also possible to �nd the decay rate using k = − ln(2)t .

Example 6.66Finding the Function that Describes Radioactive DecayThe half-life of carbon-14 is 5,730 years. Express the amount of carbon-14 remaining as a functionof time, t.

SolutionThis formula is derived as follows.

A = A0ekt The continuous growth formula.

0.5A0 = A0ek·5730 Substitute the half-life for t and 0.5A0 for f (t) .

0.5 = e5730k Divide by A0.

ln (0.5) = 5730k Take the natural log of both sides.

k = ln(0.5)5730 Divide by the coe�cient of k.

A = A0e( ln(0.5)

5730 )t Substitute for r in the continuous growth formula.

(6.116)

The function that describes this continuous decay is f (t) = A0e( ln(0.5)

5730 )t. We observe that the

coe�cient of t, ln(0.5)5730 ≈ −1.2097 is negative, as expected in the case of exponential decay.

Available for free at Connexions <http://legacy.cnx.org/content/col11759/1.2>

492 CHAPTER 6. EXPONENTIAL AND LOGARITHMIC FUNCTIONS

Try It:

Exercise 6.8.1 (Solution on p. 576.)

The half-life of plutonium-244 is 80,000,000 years. Find function gives the amount ofcarbon-14 remaining as a function of time, measured in years.

6.8.1.2 Radiocarbon Dating

The formula for radioactive decay is important in radiocarbon dating, which is used to calculate theapproximate date a plant or animal died. Radiocarbon dating was discovered in 1949 by Willard Libby, whowon a Nobel Prize for his discovery. It compares the di�erence between the ratio of two isotopes of carbonin an organic artifact or fossil to the ratio of those two isotopes in the air. It is believed to be accurate towithin about 1% error for plants or animals that died within the last 60,000 years.

Carbon-14 is a radioactive isotope of carbon that has a half-life of 5,730 years. It occurs in small quantitiesin the carbon dioxide in the air we breathe. Most of the carbon on earth is carbon-12, which has an atomicweight of 12 and is not radioactive. Scientists have determined the ratio of carbon-14 to carbon-12 in theair for the last 60,000 years, using tree rings and other organic samples of known dates�although the ratiohas changed slightly over the centuries.

As long as a plant or animal is alive, the ratio of the two isotopes of carbon in its body is close to theratio in the atmosphere. When it dies, the carbon-14 in its body decays and is not replaced. By comparingthe ratio of carbon-14 to carbon-12 in a decaying sample to the known ratio in the atmosphere, the date theplant or animal died can be approximated.

Since the half-life of carbon-14 is 5,730 years, the formula for the amount of carbon-14 remaining after tyears is

A ≈ A0e( ln(0.5)

5730 )t (6.117)

where

• A is the amount of carbon-14 remaining• A0 is the amount of carbon-14 when the plant or animal began decaying.

This formula is derived as follows:

A = A0ekt The continuous growth formula.

0.5A0 = A0ek·5730 Substitute the half-life for t and 0.5A0 for f (t) .

0.5 = e5730k Divide by A0.

ln (0.5) = 5730k Take the natural log of both sides.

k = ln(0.5)5730 Divide by the coe�cient of k.

A = A0e( ln(0.5)

5730 )t Substitute for r in the continuous growth formula.

(6.118)

To �nd the age of an object, we solve this equation for t :

t =ln(AA0

)−0.000121

(6.119)

Available for free at Connexions <http://legacy.cnx.org/content/col11759/1.2>

493

Out of necessity, we neglect here the many details that a scientist takes into consideration when doing carbon-14 dating, and we only look at the basic formula. The ratio of carbon-14 to carbon-12 in the atmosphere isapproximately 0.0000000001%. Let r be the ratio of carbon-14 to carbon-12 in the organic artifact or fossilto be dated, determined by a method called liquid scintillation. From the equation A ≈ A0e

−0.000121t weknow the ratio of the percentage of carbon-14 in the object we are dating to the percentage of carbon-14 inthe atmosphere is r = A

A0≈ e−0.000121t. We solve this equation for t, to get

t =ln (r)

−0.000121(6.120)

How To: Given the percentage of carbon-14 in an object, determine its age.

1.Express the given percentage of carbon-14 as an equivalent decimal, k.

2.Substitute for k in the equation t = ln(r)−0.000121 and solve for the age, t.

Example 6.67Finding the Age of a BoneA bone fragment is found that contains 20% of its original carbon-14. To the nearest year, howold is the bone?

SolutionWe substitute 20% = 0.20 for k in the equation and solve for t :

t = ln(r)−0.000121 Use the general form of the equation.

= ln(0.20)−0.000121 Substitute for r.

≈ 13301 Round to the nearest year.

(6.121)

The bone fragment is about 13,301 years old.

AnalysisThe instruments that measure the percentage of carbon-14 are extremely sensitive and, as wemention above, a scientist will need to do much more work than we did in order to be satis�ed.Even so, carbon dating is only accurate to about 1%, so this age should be given as 13,301 years±1% or 13,301 years± 133 years.

Try It:

Exercise 6.8.2 (Solution on p. 576.)

Cesium-137 has a half-life of about 30 years. If we begin with 200 mg of cesium-137, willit take more or less than 230 years until only 1 milligram remains?

6.8.1.3 Calculating Doubling Time

For decaying quantities, we determined how long it took for half of a substance to decay. For growingquantities, we might want to �nd out how long it takes for a quantity to double. As we mentioned above,the time it takes for a quantity to double is called the doubling time.

Given the basic exponential growth equationA = A0ekt, doubling time can be found by solving for

when the original quantity has doubled, that is, by solving 2A0 = A0ekt.

Available for free at Connexions <http://legacy.cnx.org/content/col11759/1.2>

494 CHAPTER 6. EXPONENTIAL AND LOGARITHMIC FUNCTIONS

The formula is derived as follows:

2A0 = A0ekt

2 = ekt Divide by A0.

ln2 = kt Take the natural logarithm.

t = ln2k Divide by the coe�cient of t.

(6.122)

Thus the doubling time is

t =ln2k

(6.123)

Example 6.68Finding a Function That Describes Exponential GrowthAccording to Moore's Law, the doubling time for the number of transistors that can be put on acomputer chip is approximately two years. Give a function that describes this behavior.

SolutionThe formula is derived as follows:

t = ln2k The doubling time formula.

2 = ln2k Use a doubling time of two years.

k = ln22 Multiply by k and divide by 2.

A = A0eln22 t Substitute k into the continuous growth formula.

(6.124)

The function isA = A0eln22 t.

Try It:

Exercise 6.8.3 (Solution on p. 576.)

Recent data suggests that, as of 2013, the rate of growth predicted by Moore's Law nolonger holds. Growth has slowed to a doubling time of approximately three years. Findthe new function that takes that longer doubling time into account.

6.8.2 Using Newton's Law of Cooling

Exponential decay can also be applied to temperature. When a hot object is left in surrounding air that isat a lower temperature, the object's temperature will decrease exponentially, leveling o� as it approachesthe surrounding air temperature. On a graph of the temperature function, the leveling o� will correspondto a horizontal asymptote at the temperature of the surrounding air. Unless the room temperature is zero,this will correspond to a vertical shift of the generic exponential decay function. This translation leadsto Newton's Law of Cooling, the scienti�c formula for temperature as a function of time as an object'stemperature is equalized with the ambient temperature

T (t) = aekt + Ts (6.125)

Available for free at Connexions <http://legacy.cnx.org/content/col11759/1.2>

495

This formula is derived as follows:

T (t) = Abct + Ts

T (t) = Aeln(bct) + Ts Laws of logarithms.

T (t) = Aectlnb + Ts Laws of logarithms.

T (t) = Aekt + Ts Rename the constant c ln b, calling it k.

(6.126)

A General Note: The temperature of an object,T, in surrounding air with temperatureTs willbehave according to the formula

T (t) = Aekt + Ts (6.127)

where

• t is time• A is the di�erence between the initial temperature of the object and the surroundings• k is a constant, the continuous rate of cooling of the object

How To: Given a set of conditions, apply Newton's Law of Cooling.

1.SetTs equal to the y-coordinate of the horizontal asymptote (usually the ambient tempera-ture).

2.Substitute the given values into the continuous growth formulaT (t) = Aekt + Ts to �nd theparametersA and k.

3.Substitute in the desired time to �nd the temperature or the desired temperature to �nd thetime.

Example 6.69Using Newton's Law of CoolingA cheesecake is taken out of the oven with an ideal internal temperature of 165 ◦F, and is placedinto a 35 ◦F refrigerator. After 10 minutes, the cheesecake has cooled to 150 ◦F. If we must waituntil the cheesecake has cooled to 70 ◦Fbefore we eat it, how long will we have to wait?

SolutionBecause the surrounding air temperature in the refrigerator is 35 degrees, the cheesecake's tem-perature will decay exponentially toward 35, following the equation

T (t) = Aekt + 35 (6.128)

We know the initial temperature was 165, soT (0) = 165.

165 = Aek0 + 35 Substitute (0, 165) .

A = 130 Solve for A.(6.129)

We were given another data point,T (10) = 150, which we can use to solve for k.

150 = 130ek10 + 35 Substitute (10, 150).

115 = 130ek10 Subtract 35.

115130 = e10k Divide by 130.

ln(

115130

)= 10k Take the natural log of both sides.

k =ln( 115

130 )10 = −0.0123 Divide by the coe�cient of k.

(6.130)

Available for free at Connexions <http://legacy.cnx.org/content/col11759/1.2>

496 CHAPTER 6. EXPONENTIAL AND LOGARITHMIC FUNCTIONS

This gives us the equation for the cooling of the cheesecake:T (t) = 130e−−0.0123t + 35.Now we can solve for the time it will take for the temperature to cool to 70 degrees.

70 = 130e−0.0123t + 35 Substitute in 70 for T (t) .

35 = 130e−0.0123t Subtract 35.

35130 = e−0.0123t Divide by 130.

ln(

35130

)= −0.0123t Take the natural log of both sides

t =ln( 35

130 )−0.0123 ≈ 106.68 Divide by the coe�cient of t.

(6.131)

It will take about 107 minutes, or one hour and 47 minutes, for the cheesecake to cool to 70 ◦F.

Try It:

Exercise 6.8.4 (Solution on p. 576.)

A pitcher of water at 40 degrees Fahrenheit is placed into a 70 degree room. One hourlater, the temperature has risen to 45 degrees. How long will it take for the temperatureto rise to 60 degrees?

6.8.3 Using Logistic Growth Models

Exponential growth cannot continue forever. Exponential models, while they may be useful in the shortterm, tend to fall apart the longer they continue. Consider an aspiring writer who writes a single line on dayone and plans to double the number of lines she writes each day for a month. By the end of the month, shemust write over 17 billion lines, or one-half-billion pages. It is impractical, if not impossible, for anyone towrite that much in such a short period of time. Eventually, an exponential model must begin to approachsome limiting value, and then the growth is forced to slow. For this reason, it is often better to use a modelwith an upper bound instead of an exponential growth model, though the exponential growth model isstill useful over a short term, before approaching the limiting value.

The logistic growth model is approximately exponential at �rst, but it has a reduced rate of growthas the output approaches the model's upper bound, called the carrying capacity. For constants a, b, andc, the logistic growth of a population over timex is represented by the model

f (x) =c

1 + ae−bx(6.132)

The graph in Figure 6.51 shows how the growth rate changes over time. The graph increases from left toright, but the growth rate only increases until it reaches its point of maximum growth rate, at which pointthe rate of increase decreases.

Available for free at Connexions <http://legacy.cnx.org/content/col11759/1.2>

497

Figure 6.51

A General Note: The logistic growth model is

f (x) =c

1 + ae−bx(6.133)

where

• c1+a is the initial value• c is the carrying capacity, or limiting value• b is a constant determined by the rate of growth.

Example 6.70Using the Logistic-Growth ModelAn in�uenza epidemic spreads through a population rapidly, at a rate that depends on two factors:The more people who have the �u, the more rapidly it spreads, and also the more uninfected peoplethere are, the more rapidly it spreads. These two factors make the logistic model a good one tostudy the spread of communicable diseases. And, clearly, there is a maximum value for the numberof people infected: the entire population.

Available for free at Connexions <http://legacy.cnx.org/content/col11759/1.2>

498 CHAPTER 6. EXPONENTIAL AND LOGARITHMIC FUNCTIONS

For example, at time t = 0 there is one person in a community of 1,000 people who has the�u. So, in that community, at most 1,000 people can have the �u. Researchers �nd that forthis particular strain of the �u, the logistic growth constant is b = 0.6030.Estimate the number ofpeople in this community who will have had this �u after ten days. Predict how many people inthis community will have had this �u after a long period of time has passed.

SolutionWe substitute the given data into the logistic growth model

f (x) =c

1 + ae−bx(6.134)

Because at most 1,000 people, the entire population of the community, can get the �u, we knowthe limiting value is c = 1000. To �nd a,we use the formula that the number of cases at time t =0 is c

1+a = 1, from which it follows that a = 999.This model predicts that, after ten days, the

number of people who have had the �u is f (x) = 10001+999e−0.6030x ≈ 293.8.Because the actual number

must be a whole number (a person has either had the �u or not) we round to 294. In the long term,the number of people who will contract the �u is the limiting value, c = 1000.

AnalysisRemember that, because we are dealing with a virus, we cannot predict with certainty the numberof people infected. The model only approximates the number of people infected and will not giveus exact or actual values.

The graph in Figure 6.52 gives a good picture of how this model �ts the data.

Available for free at Connexions <http://legacy.cnx.org/content/col11759/1.2>

499

Figure 6.52: The graph of f (x) = 10001+999e−0.6030x

Try It:

Exercise 6.8.5 (Solution on p. 576.)

Using the model in Example 6.70, estimate the number of cases of �u on day 15.

6.8.4 Choosing an Appropriate Model for Data

Now that we have discussed various mathematical models, we need to learn how to choose the appropriatemodel for the raw data we have. Many factors in�uence the choice of a mathematical model, among whichare experience, scienti�c laws, and patterns in the data itself. Not all data can be described by elementaryfunctions. Sometimes, a function is chosen that approximates the data over a given interval. For instance,suppose data were gathered on the number of homes bought in the United States from the years 1960 to2013. After plotting these data in a scatter plot, we notice that the shape of the data from the years 2000 to2013 follow a logarithmic curve. We could restrict the interval from 2000 to 2010, apply regression analysisusing a logarithmic model, and use it to predict the number of home buyers for the year 2015.

Three kinds of functions that are often useful in mathematical models are linear functions, exponentialfunctions, and logarithmic functions. If the data lies on a straight line, or seems to lie approximately along

Available for free at Connexions <http://legacy.cnx.org/content/col11759/1.2>

500 CHAPTER 6. EXPONENTIAL AND LOGARITHMIC FUNCTIONS

a straight line, a linear model may be best. If the data is non-linear, we often consider an exponential orlogarithmic model, though other models, such as quadratic models, may also be considered.

In choosing between an exponential model and a logarithmic model, we look at the way the data curves.This is called the concavity. If we draw a line between two data points, and all (or most) of the data betweenthose two points lies above that line, we say the curve is concave down. We can think of it as a bowl thatbends downward and therefore cannot hold water. If all (or most) of the data between those two points liesbelow the line, we say the curve is concave up. In this case, we can think of a bowl that bends upward andcan therefore hold water. An exponential curve, whether rising or falling, whether representing growth ordecay, is always concave up away from its horizontal asymptote. A logarithmic curve is always concave awayfrom its vertical asymptote. In the case of positive data, which is the most common case, an exponentialcurve is always concave up, and a logarithmic curve always concave down.

A logistic curve changes concavity. It starts out concave up and then changes to concave down beyonda certain point, called a point of in�ection.

After using the graph to help us choose a type of function to use as a model, we substitute points, andsolve to �nd the parameters. We reduce round-o� error by choosing points as far apart as possible.

Example 6.71Choosing a Mathematical ModelDoes a linear, exponential, logarithmic, or logistic model best �t the values listed in Table 6.30?Find the model, and use a graph to check your choice.

x 1 2 3 4 5 6 7 8 9

y 0 1.386 2.197 2.773 3.219 3.584 3.892 4.159 4.394

Table 6.30

SolutionFirst, plot the data on a graph as in Figure 6.53. For the purpose of graphing, round the data totwo signi�cant digits.

Available for free at Connexions <http://legacy.cnx.org/content/col11759/1.2>

501

Figure 6.53

Clearly, the points do not lie on a straight line, so we reject a linear model. If we draw a linebetween any two of the points, most or all of the points between those two points lie above the line,so the graph is concave down, suggesting a logarithmic model. We can try y = aln (bx) .Plugging inthe �rst point, (1,0) , gives 0 = alnb.We reject the case that a = 0 (if it were, all outputs would be0), so we know ln (b) = 0.Thus b = 1 and y = aln (x) .Next we can use the point (9,4.394) to solvefor a :

Available for free at Connexions <http://legacy.cnx.org/content/col11759/1.2>

502 CHAPTER 6. EXPONENTIAL AND LOGARITHMIC FUNCTIONS

y = aln (x)

4.394 = aln (9)

a = 4.394ln(9)

(6.135)

Because a = 4.394ln(9) ≈ 2,an appropriate model for the data is y = 2ln (x) .

To check the accuracy of the model, we graph the function together with the given points as inFigure 6.54.

Figure 6.54: The graph of y = 2lnx.

We can conclude that the model is a good �t to the data.

Available for free at Connexions <http://legacy.cnx.org/content/col11759/1.2>

503

Compare Figure 6.54 to the graph of y = ln(x2)shown in Figure 6.55.

Figure 6.55: The graph of y = ln`x2

´

The graphs appear to be identical whenx > 0. A quick check con�rms this conclusion: y =ln(x2)

= 2ln (x) forx > 0.However, ifx < 0, the graph of y = ln

(x2)includes a �extra� branch, as shown in Figure 6.56.

This occurs because, while y = 2ln (x) cannot have negative values in the domain (as such valueswould force the argument to be negative), the function y = ln

(x2)can have negative domain values.

Available for free at Connexions <http://legacy.cnx.org/content/col11759/1.2>

504 CHAPTER 6. EXPONENTIAL AND LOGARITHMIC FUNCTIONS

Figure 6.56

Try It:

Exercise 6.8.6 (Solution on p. 576.)

Does a linear, exponential, or logarithmic model best �t the data in Table 6.31? Find themodel.

x 1 2 3 4 5 6 7 8 9

y 3.297 5.437 8.963 14.778 24.365 40.172 66.231 109.196 180.034

Table 6.31

6.8.5 Expressing an Exponential Model in Base e

While powers and logarithms of any base can be used in modeling, the two most common bases are 10 and e. Inscience and mathematics, the base e is often preferred. We can use laws of exponents and laws of logarithmsto change any base to base e.

How To: Given a model with the form y = abx, change it to the form y = A0ekx.

1.Rewrite y = abx as y = aeln(bx).2.Use the power rule of logarithms to rewrite y as y = aexln(b) = aeln(b)x.3.Note that a = A0 and k = ln (b) in the equation y = A0e

kx.

Available for free at Connexions <http://legacy.cnx.org/content/col11759/1.2>

505

Example 6.72Changing to base eChange the function y = 2.5(3.1)x so that this same function is written in the form y = A0e

kx.

SolutionThe formula is derived as follows

y = 2.5(3.1)x

= 2.5eln(3.1x) Insert exponential and its inverse.

= 2.5exln3.1 Laws of logs.

= 2.5e(ln3.1)x Commutative law of multiplication

(6.136)

Try It:

Exercise 6.8.7 (Solution on p. 576.)

Change the function y = 3(0.5)x to one having e as the base.

Media: Access these online resources for additional instruction and practice with exponential andlogarithmic models.

• Logarithm Application � pH30

• Exponential Model � Age Using Half-Life31

• Newton's Law of Cooling32

• Exponential Growth Given Doubling Time33

• Exponential Growth � Find Initial Amount Given Doubling Time34

6.8.6 Key Equations

Half-life formula If A = A0ekt, k < 0, the half-life is t = − ln(2)

k .

Carbon-14 dating t =ln“

AA0

”−0.000121 .

A0 A is the amount of carbon-14 when the plant oranimal diedt is the amount of carbon-14 remaining todayis the age of the fossil in years

continued on next page

30http://openstaxcollege.org/l/logph31http://openstaxcollege.org/l/expmodelhalf32http://openstaxcollege.org/l/newtoncooling33http://openstaxcollege.org/l/expgrowthdbl34http://openstaxcollege.org/l/initialdouble

Available for free at Connexions <http://legacy.cnx.org/content/col11759/1.2>

506 CHAPTER 6. EXPONENTIAL AND LOGARITHMIC FUNCTIONS

Doubling time formula If A = A0ekt, k > 0, the doubling time is t = ln2

k

Newton's Law of Cooling T (t) = Aekt + Ts, where Ts is the ambienttemperature, A = T (0)− Ts, and k is the continu-ous rate of cooling.

Table 6.32

6.8.7 Key Concepts

• The basic exponential function is f (x) = abx. If b > 1,we have exponential growth; if 0 < b < 1, wehave exponential decay.

• We can also write this formula in terms of continuous growth asA = A0ekx, whereA0 is the starting

value. IfA0 is positive, then we have exponential growth when k > 0 and exponential decay when k <0. See Example 6.65.

• In general, we solve problems involving exponential growth or decay in two steps. First, we set up amodel and use the model to �nd the parameters. Then we use the formula with these parameters topredict growth and decay. See Example 6.66.

• We can �nd the age, t, of an organic artifact by measuring the amount, k, of carbon-14 remaining in

the artifact and using the formula t = ln(k)−0.000121 to solve for t. See Example 6.67.

• Given a substance's doubling time or half-time, we can �nd a function that represents its exponentialgrowth or decay. See Example 6.68.

• We can use Newton's Law of Cooling to �nd how long it will take for a cooling object to reach a desiredtemperature, or to �nd what temperature an object will be after a given time. See Example 6.69.

• We can use logistic growth functions to model real-world situations where the rate of growth changesover time, such as population growth, spread of disease, and spread of rumors. See Example 6.70.

• We can use real-world data gathered over time to observe trends. Knowledge of linear, exponential,logarithmic, and logistic graphs help us to develop models that best �t our data. See Example 6.71.

• Any exponential function with the form y = abx can be rewritten as an equivalent exponential functionwith the form y = A0e

kx where k = lnb. See Example 6.72.

6.8.8 Section Exercises

6.8.8.1 Verbal

Exercise 6.8.8 (Solution on p. 576.)

With what kind of exponential model would half-life be associated? What role does half-life playin these models?

Exercise 6.8.9What is carbon dating? Why does it work? Give an example in which carbon dating would beuseful.

Exercise 6.8.10 (Solution on p. 576.)

With what kind of exponential model would doubling time be associated? What role does doublingtime play in these models?

Exercise 6.8.11De�ne Newton's Law of Cooling. Then name at least three real-world situations where Newton'sLaw of Cooling would be applied.

Exercise 6.8.12 (Solution on p. 576.)

What is an order of magnitude? Why are orders of magnitude useful? Give an example to explain.

Available for free at Connexions <http://legacy.cnx.org/content/col11759/1.2>

507

6.8.8.2 Numeric

Exercise 6.8.13The temperature of an object in degrees Fahrenheit after t minutes is represented by theequation T (t) = 68e−0.0174t + 72. To the nearest degree, what is the temperature of the objectafter one and a half hours?

For the following exercises, use the logistic growth model f (x) = 1501+8e−2x .

Exercise 6.8.14 (Solution on p. 576.)

Find and interpret f (0) .Round to the nearest tenth.

Exercise 6.8.15Find and interpret f (4) .Round to the nearest tenth.

Exercise 6.8.16 (Solution on p. 576.)

Find the carrying capacity.

Exercise 6.8.17Graph the model.

Exercise 6.8.18Determine whether the data from the table could best be represented as a function that is linear,exponential, or logarithmic. Then write a formula for a model that represents the data.

Exercise 6.8.19 (Solution on p. 576.)

x �2 �1 0 1 2 3 4 5

f (x) 0.694 0.833 1 1.2 1.44 1.728 2.074 2.488

Table 6.33

Exercise 6.8.20Rewrite f (x) = 1.68(0.65)x as an exponential equation with base e to �ve signi�cant digits.

6.8.8.3 Technology

For the following exercises, enter the data from each table into a graphing calculator and graph the re-sulting scatter plots. Determine whether the data from the table could represent a function that is linear,exponential, or logarithmic.

Exercise 6.8.21 (Solution on p. 576.)

x 1 2 3 4 5 6 7 8 9 10

f (x) 2 4.079 5.296 6.159 6.828 7.375 7.838 8.238 8.592 8.908

Table 6.34

Exercise 6.8.22

x 1 2 3 4 5 6 7 8 9 10

f (x) 2.4 2.88 3.456 4.147 4.977 5.972 7.166 8.6 10.32 12.383

Available for free at Connexions <http://legacy.cnx.org/content/col11759/1.2>

508 CHAPTER 6. EXPONENTIAL AND LOGARITHMIC FUNCTIONS

Table 6.35

Exercise 6.8.23 (Solution on p. 577.)

x 4 5 6 7 8 9 10 11 12 13

f (x) 9.429 9.972 10.415 10.79 11.115 11.401 11.657 11.889 12.101 12.295

Table 6.36

Exercise 6.8.24

x 1.25 2.25 3.56 4.2 5.65 6.75 7.25 8.6 9.25 10.5

f (x) 5.75 8.75 12.68 14.6 18.95 22.25 23.75 27.8 29.75 33.5

Table 6.37

For the following exercises, use a graphing calculator and this scenario: the population of a �sh farm in t yearsis modeled by the equation P (t) = 1000

1+9e−0.6t .

Exercise 6.8.25 (Solution on p. 578.)

Graph the function.

Exercise 6.8.26What is the initial population of �sh?

Exercise 6.8.27 (Solution on p. 579.)

To the nearest tenth, what is the doubling time for the �sh population?

Exercise 6.8.28To the nearest whole number, what will the �sh population be after 2 years?Exercise 6.8.29 (Solution on p. 579.)

To the nearest tenth, how long will it take for the population to reach 900?Exercise 6.8.30What is the carrying capacity for the �sh population? Justify your answer using the graph ofP.

6.8.8.4 Extensions

Exercise 6.8.31 (Solution on p. 579.)

A substance has a half-life of 2.045 minutes. If the initial amount of the substance was 132.8 grams,how many half-lives will have passed before the substance decays to 8.3 grams? What is the totaltime of decay?

Exercise 6.8.32The formula for an increasing population is given byP (t) = P0e

rt whereP0 is the initial populationand r > 0.Derive a general formula for the time t it takes for the population to increase by a factorof M.

Exercise 6.8.33 (Solution on p. 579.)

Recall the formula for calculating the magnitude of an earthquake,M = 23 log

(SS0

). Show each

step for solving this equation algebraically for the seismic momentS.

Available for free at Connexions <http://legacy.cnx.org/content/col11759/1.2>

509

Exercise 6.8.34What is the y-intercept of the logistic growth model y = c

1+ae−rx ? Show the steps for calculation.What does this point tell us about the population?

Exercise 6.8.35 (Solution on p. 579.)

Prove that bx = exln(b) for positive b 6= 1.

6.8.8.5 Real-World Applications

For the following exercises, use this scenario: A doctor prescribes 125 milligrams of a therapeutic drug thatdecays by about 30% each hour.

Exercise 6.8.36To the nearest hour, what is the half-life of the drug?

Exercise 6.8.37 (Solution on p. 579.)

Write an exponential model representing the amount of the drug remaining in the patient's systemafter t hours. Then use the formula to �nd the amount of the drug that would remain in thepatient's system after 3 hours. Round to the nearest milligram.

Exercise 6.8.38Using the model found in the previous exercise, �nd f (10) and interpret the result. Round to thenearest hundredth.

For the following exercises, use this scenario: A tumor is injected with 0.5 grams of Iodine-125, which has adecay rate of 1.15% per day.

Exercise 6.8.39 (Solution on p. 579.)

To the nearest day, how long will it take for half of the Iodine-125 to decay?

Exercise 6.8.40Write an exponential model representing the amount of Iodine-125 remaining in the tumorafter t days. Then use the formula to �nd the amount of Iodine-125 that would remain in thetumor after 60 days. Round to the nearest tenth of a gram.

Exercise 6.8.41 (Solution on p. 579.)

A scientist begins with 250 grams of a radioactive substance. After 250minutes, the sample hasdecayed to 32 grams. Rounding to �ve signi�cant digits, write an exponential equation representingthis situation. To the nearest minute, what is the half-life of this substance?

Exercise 6.8.42The half-life of Radium-226 is 1590 years. What is the annual decay rate? Express the decimalresult to four signi�cant digits and the percentage to two signi�cant digits.

Exercise 6.8.43 (Solution on p. 579.)

The half-life of Erbium-165 is 10.4 hours. What is the hourly decay rate? Express the decimalresult to four signi�cant digits and the percentage to two signi�cant digits.

Exercise 6.8.44A wooden artifact from an archeological dig contains 60 percent of the carbon-14 that is presentin living trees. To the nearest year, about how many years old is the artifact? (The half-life ofcarbon-14 is 5730 years.)Exercise 6.8.45 (Solution on p. 579.)

A research student is working with a culture of bacteria that doubles in size every twenty minutes.The initial population count was 1350 bacteria. Rounding to �ve signi�cant digits, write an expo-nential equation representing this situation. To the nearest whole number, what is the populationsize after 3 hours?

Available for free at Connexions <http://legacy.cnx.org/content/col11759/1.2>

510 CHAPTER 6. EXPONENTIAL AND LOGARITHMIC FUNCTIONS

For the following exercises, use this scenario: A biologist recorded a count of 360 bacteria present in a cultureafter 5 minutes and 1000 bacteria present after 20 minutes.

Exercise 6.8.46To the nearest whole number, what was the initial population in the culture?

Exercise 6.8.47 (Solution on p. 580.)

Rounding to six signi�cant digits, write an exponential equation representing this situation. Tothe nearest minute, how long did it take the population to double?

For the following exercises, use this scenario: A pot of boiling soup with an internal temperatureof 100 ◦ Fahrenheit was taken o� the stove to cool in a 69 ◦ F room. After �fteen minutes, the internal tem-perature of the soup was 95 ◦ F.

Exercise 6.8.48Use Newton's Law of Cooling to write a formula that models this situation.

Exercise 6.8.49 (Solution on p. 580.)

To the nearest minute, how long will it take the soup to cool to 80 ◦ F?

Exercise 6.8.50To the nearest degree, what will the temperature be after 2 and a half hours?

For the following exercises, use this scenario: A turkey is taken out of the oven with an internal temperatureof 165 ◦ Fahrenheit and is allowed to cool in a 75 ◦ F room. After half an hour, the internal temperature ofthe turkey is 145 ◦ F.

Exercise 6.8.51 (Solution on p. 580.)

Write a formula that models this situation.

Exercise 6.8.52To the nearest degree, what will the temperature be after 50 minutes?

Exercise 6.8.53 (Solution on p. 580.)

To the nearest minute, how long will it take the turkey to cool to 110 ◦ F?

For the following exercises, �nd the value of the number shown on each logarithmic scale. Round all answersto the nearest thousandth.

Exercise 6.8.54

Exercise 6.8.55 (Solution on p. 580.)

Exercise 6.8.56Plot each set of approximate values of intensity of sounds on a logarithmic scale:

Whisper: 10−10 Wm2 ,Vacuum: 10−4 W

m2 ,Jet: 102 Wm2

Available for free at Connexions <http://legacy.cnx.org/content/col11759/1.2>

511

Exercise 6.8.57 (Solution on p. 580.)

Recall the formula for calculating the magnitude of an earthquake,M = 23 log

(SS0

).One earthquake

has magnitude 3.9 on the MMS scale. If a second earthquake has 750 times as much energy as the�rst, �nd the magnitude of the second quake. Round to the nearest hundredth.

For the following exercises, use this scenario: The equationN (t) = 5001+49e−0.7t models the number of people

in a town who have heard a rumor after t days.

Exercise 6.8.58How many people started the rumor?

Exercise 6.8.59 (Solution on p. 580.)

To the nearest whole number, how many people will have heard the rumor after 3 days?

Exercise 6.8.60As t increases without bound, what value doesN (t) approach? Interpret your answer.

For the following exercise, choose the correct answer choice.

Exercise 6.8.61 (Solution on p. 580.)

A doctor and injects a patient with 13milligrams of radioactive dye that decays exponentially.After 12minutes, there are 4.75milligrams of dye remaining in the patient's system. Which is anappropriate model for this situation?

A. f (t) = 13(0.0805)t

B. f (t) = 13e0.9195t

C. f (t) = 13e(−0.0839t)

D. f (t) = 4.751+13e−0.83925t

6.9 Fitting Exponential Models to Data35

In previous sections of this chapter, we were either given a function explicitly to graph or evaluate, or wewere given a set of points that were guaranteed to lie on the curve. Then we used algebra to �nd the equationthat �t the points exactly. In this section, we use a modeling technique called regression analysis to �nd acurve that models data collected from real-world observations. With regression analysis, we don't expectall the points to lie perfectly on the curve. The idea is to �nd a model that best �ts the data. Then we usethe model to make predictions about future events.

Do not be confused by the word model. In mathematics, we often use the terms function, equation, andmodel interchangeably, even though they each have their own formal de�nition. The term model is typicallyused to indicate that the equation or function approximates a real-world situation.

We will concentrate on three types of regression models in this section: exponential, logarithmic, andlogistic. Having already worked with each of these functions gives us an advantage. Knowing their formalde�nitions, the behavior of their graphs, and some of their real-world applications gives us the opportunityto deepen our understanding. As each regression model is presented, key features and de�nitions of itsassociated function are included for review. Take a moment to rethink each of these functions, re�ect on thework we've done so far, and then explore the ways regression is used to model real-world phenomena.

35This content is available online at <http://legacy.cnx.org/content/m49368/1.4/>.

Available for free at Connexions <http://legacy.cnx.org/content/col11759/1.2>

512 CHAPTER 6. EXPONENTIAL AND LOGARITHMIC FUNCTIONS

6.9.1 Building an Exponential Model from Data

As we've learned, there are a multitude of situations that can be modeled by exponential functions, such asinvestment growth, radioactive decay, atmospheric pressure changes, and temperatures of a cooling object.What do these phenomena have in common? For one thing, all the models either increase or decrease astime moves forward. But that's not the whole story. It's the way data increase or decrease that helps usdetermine whether it is best modeled by an exponential equation. Knowing the behavior of exponentialfunctions in general allows us to recognize when to use exponential regression, so let's review exponentialgrowth and decay.

Recall that exponential functions have the form y = abx or y = A0ekx.When performing regression anal-

ysis, we use the form most commonly used on graphing utilities, y = abx.Take a moment to re�ect on thecharacteristics we've already learned about the exponential function y = abx (assume a > 0 :

• bmust be greater than zero and not equal to one.• The initial value of the model is y = a.

◦ If b > 1, the function models exponential growth. Asx increases, the outputs of the model increaseslowly at �rst, but then increase more and more rapidly, without bound.

◦ If 0 < b < 1, the function models exponential decay. Asx increases, the outputs for the modeldecrease rapidly at �rst and then level o� to become asymptotic to the x-axis. In other words,the outputs never become equal to or less than zero.

As part of the results, your calculator will display a number known as the correlation coe�cient, labeled bythe variable r, or r2. (You may have to change the calculator's settings for these to be shown.) The valuesare an indication of the �goodness of �t� of the regression equation to the data. We more commonly use thevalue of r2 instead of r, but the closer either value is to 1, the better the regression equation approximatesthe data.

A General Note: Exponential regression is used to model situations in which growth beginsslowly and then accelerates rapidly without bound, or where decay begins rapidly and then slowsdown to get closer and closer to zero. We use the command �ExpReg� on a graphing utility to �tan exponential function to a set of data points. This returns an equation of the form,

y = abx

Note that:

• bmust be non-negative.• when b > 1, we have an exponential growth model.• when 0 < b < 1, we have an exponential decay model.

How To: Given a set of data, perform exponential regression using a graphing utility.

1.Use the STAT then EDIT menu to enter given data.

a.Clear any existing data from the lists.b.List the input values in the L1 column.c.List the output values in the L2 column.

2.Graph and observe a scatter plot of the data using the STATPLOT feature.

a.Use ZOOM [9] to adjust axes to �t the data.b.Verify the data follow an exponential pattern.

3.Find the equation that models the data.

a.Select �ExpReg� from the STAT then CALC menu.

Available for free at Connexions <http://legacy.cnx.org/content/col11759/1.2>

513

b.Use the values returned for a and b to record the model, y = abx.

4.Graph the model in the same window as the scatterplot to verify it is a good �t for the data.

Example 6.73Using Exponential Regression to Fit a Model to DataIn 2007, a university study was published investigating the crash risk of alcohol impaired driving.Data from 2,871 crashes were used to measure the association of a person's blood alcohol level(BAC) with the risk of being in an accident. Table 6.38 shows results from the study 36. Therelative risk is a measure of how many times more likely a person is to crash. So, for example, aperson with a BAC of 0.09 is 3.54 times as likely to crash as a person who has not been drinkingalcohol.

BAC 0 0.01 0.03 0.05 0.07 0.09

Relative Risk of Crashing 1 1.03 1.06 1.38 2.09 3.54

BAC 0.11 0.13 0.15 0.17 0.19 0.21

Relative Risk of Crashing 6.41 12.6 22.1 39.05 65.32 99.78

Table 6.38

a. Letx represent the BAC level, and let y represent the corresponding relative risk. Use expo-nential regression to �t a model to these data.

b. After 6 drinks, a person weighing 160 pounds will have a BAC of about 0.16.How many timesmore likely is a person with this weight to crash if they drive after having a 6-pack of beer?Round to the nearest hundredth.

Solution

a. Using the STAT then EDIT menu on a graphing utility, list the BAC values in L1 and therelative risk values in L2. Then use the STATPLOT feature to verify that the scatterplotfollows the exponential pattern shown in Figure 6.57:

36Source: Indiana University Center for Studies of Law in Action, 2007

Available for free at Connexions <http://legacy.cnx.org/content/col11759/1.2>

514 CHAPTER 6. EXPONENTIAL AND LOGARITHMIC FUNCTIONS

Figure 6.57

Use the �ExpReg� command from the STAT then CALC menu to obtain the exponentialmodel,

y = 0.58304829(2.20720213E10)x (6.137)

Converting from scienti�c notation, we have:

y = 0.58304829(22,072,021,300)x (6.138)

Notice that r2 ≈ 0.97which indicates the model is a good �t to the data. To see this,graph the model in the same window as the scatterplot to verify it is a good �t as shown inFigure 6.58:

Available for free at Connexions <http://legacy.cnx.org/content/col11759/1.2>

515

Figure 6.58

b. Use the model to estimate the risk associated with a BAC of 0.16. Substitute 0.16 forx in themodel and solve for y.

y = 0.58304829(22,072,021,300)x Use the regression model found in part (a).

= 0.58304829(22,072,021,300)0.16Substitute 0.16 for x.

≈ 26.35 Round to the nearest hundredth.

(6.139)

If a 160-pound person drives after having 6 drinks, he or she is about 26.35 times more likelyto crash than if driving while sober.

Available for free at Connexions <http://legacy.cnx.org/content/col11759/1.2>

516 CHAPTER 6. EXPONENTIAL AND LOGARITHMIC FUNCTIONS

Try It:

Exercise 6.9.1 (Solution on p. 580.)

Table 6.39 shows a recent graduate's credit card balance each month after graduation.

Month 1 2 3 4 5 6 7 8

Debt ($) 620.00 761.88 899.80 1039.93 1270.63 1589.04 1851.31 2154.92

Table 6.39

a.Use exponential regression to �t a model to these data.b.If spending continues at this rate, what will the graduate's credit card debt be oneyear after graduating?

Q&A: Is it reasonable to assume that an exponential regression model will representa situation inde�nitely?

No. Remember that models are formed by real-world data gathered for regression. It is usually rea-sonable to make estimates within the interval of original observation (interpolation). However, whena model is used to make predictions, it is important to use reasoning skills to determine whetherthe model makes sense for inputs far beyond the original observation interval (extrapolation).

6.9.2 Building a Logarithmic Model from Data

Just as with exponential functions, there are many real-world applications for logarithmic functions: intensityof sound, pH levels of solutions, yields of chemical reactions, production of goods, and growth of infants.As with exponential models, data modeled by logarithmic functions are either always increasing or alwaysdecreasing as time moves forward. Again, it is the way they increase or decrease that helps us determinewhether a logarithmic model is best.

Recall that logarithmic functions increase or decrease rapidly at �rst, but then steadily slow as time moveson. By re�ecting on the characteristics we've already learned about this function, we can better analyze realworld situations that re�ect this type of growth or decay. When performing logarithmic regression analysis,we use the form of the logarithmic function most commonly used on graphing utilities, y = a + bln (x) .Forthis function

• All input values,x,must be greater than zero.• The point (1, a) is on the graph of the model.• If b > 0,the model is increasing. Growth increases rapidly at �rst and then steadily slows over time.• If b < 0,the model is decreasing. Decay occurs rapidly at �rst and then steadily slows over time.

A General Note: Logarithmic regression is used to model situations where growth or decayaccelerates rapidly at �rst and then slows over time. We use the command �LnReg� on a graphingutility to �t a logarithmic function to a set of data points. This returns an equation of the form,

y = a+ bln (x) (6.140)

Note that

Available for free at Connexions <http://legacy.cnx.org/content/col11759/1.2>

517

• all input values,x,must be non-negative.• when b > 0,the model is increasing.• when b < 0,the model is decreasing.

How To: Given a set of data, perform logarithmic regression using a graphing utility.

1.Use the STAT then EDIT menu to enter given data.

a.Clear any existing data from the lists.b.List the input values in the L1 column.c.List the output values in the L2 column.

2.Graph and observe a scatter plot of the data using the STATPLOT feature.

a.Use ZOOM [9] to adjust axes to �t the data.b.Verify the data follow a logarithmic pattern.

3.Find the equation that models the data.

a.Select �LnReg� from the STAT then CALC menu.b.Use the values returned for a and b to record the model, y = a+ bln (x) .

4.Graph the model in the same window as the scatterplot to verify it is a good �t for the data.

Example 6.74Using Logarithmic Regression to Fit a Model to DataDue to advances in medicine and higher standards of living, life expectancy has been increasing inmost developed countries since the beginning of the 20th century.

Table 6.40 shows the average life expectancies, in years, of Americans from 1900�201037.

Year 1900 1910 1920 1930 1940 1950

Life Expectancy(Years) 47.3 50.0 54.1 59.7 62.9 68.2

Year 1960 1970 1980 1990 2000 2010

Life Expectancy(Years) 69.7 70.8 73.7 75.4 76.8 78.7

Table 6.40

a. Letx represent time in decades starting withx = 1 for the year 1900,x = 2 for the year 1910,and so on. Let y represent the corresponding life expectancy. Use logarithmic regression to �ta model to these data.

b. Use the model to predict the average American life expectancy for the year 2030.

Solution

a. Using the STAT then EDIT menu on a graphing utility, list the years using values 1�12 inL1 and the corresponding life expectancy in L2. Then use the STATPLOT feature to verifythat the scatterplot follows a logarithmic pattern as shown in Figure 6.59:

37Source: Center for Disease Control and Prevention, 2013

Available for free at Connexions <http://legacy.cnx.org/content/col11759/1.2>

518 CHAPTER 6. EXPONENTIAL AND LOGARITHMIC FUNCTIONS

Figure 6.59

Use the �LnReg� command from the STAT then CALC menu to obtain the logarithmic model,

y = 42.52722583 + 13.85752327ln (x) (6.141)

Next, graph the model in the same window as the scatterplot to verify it is a good �t asshown in Figure 6.60:

Available for free at Connexions <http://legacy.cnx.org/content/col11759/1.2>

519

Figure 6.60

b. To predict the life expectancy of an American in the year 2030, substitutex = 14 for the inthe model and solve for y :

y = 42.52722583 + 13.85752327ln (x) Use the regression model found in part (a).

= 42.52722583 + 13.85752327ln (14) Substitute 14 for x.

≈ 79.1 Round to the nearest tenth

(6.142)

If life expectancy continues to increase at this pace, the average life expectancy of an Americanwill be 79.1 by the year 2030.

Try It:

Exercise 6.9.2 (Solution on p. 580.)

Sales of a video game released in the year 2000 took o� at �rst, but then steadily slowedas time moved on. Table 6.41 shows the number of games sold, in thousands, from theyears 2000�2010.

Available for free at Connexions <http://legacy.cnx.org/content/col11759/1.2>

520 CHAPTER 6. EXPONENTIAL AND LOGARITHMIC FUNCTIONS

Year 2000 2001 2002 2003 2004 2005

Number Sold (thousands) 142 149 154 155 159 161

Year 2006 2007 2008 2009 2010 -

Number Sold (thousands) 163 164 164 166 167 -

Table 6.41

a.Letx represent time in years starting withx = 1 for the year 2000. Let y representthe number of games sold in thousands. Use logarithmic regression to �t a model tothese data.

b.If games continue to sell at this rate, how many games will sell in 2015? Round tothe nearest thousand.

6.9.3 Building a Logistic Model from Data

Like exponential and logarithmic growth, logistic growth increases over time. One of the most notabledi�erences with logistic growth models is that, at a certain point, growth steadily slows and the functionapproaches an upper bound, or limiting value. Because of this, logistic regression is best for modelingphenomena where there are limits in expansion, such as availability of living space or nutrients.

It is worth pointing out that logistic functions actually model resource-limited exponential growth. Thereare many examples of this type of growth in real-world situations, including population growth and spreadof disease, rumors, and even stains in fabric. When performing logistic regression analysis, we use theform most commonly used on graphing utilities:

y =c

1 + ae−bx(6.143)

Recall that:

• c1+a is the initial value of the model.

• when b > 0, the model increases rapidly at �rst until it reaches its point of maximum growth

rate,(

ln(a)b , c2

).At that point, growth steadily slows and the function becomes asymptotic to the

upper bound y = c.• c is the limiting value, sometimes called the carrying capacity, of the model.

A General Note: Logistic regression is used to model situations where growth acceleratesrapidly at �rst and then steadily slows to an upper limit. We use the command �Logistic� on agraphing utility to �t a logistic function to a set of data points. This returns an equation of theform

y =c

1 + ae−bx(6.144)

Note that

• The initial value of the model is c1+a .

• Output values for the model grow closer and closer to y = c as time increases.

Available for free at Connexions <http://legacy.cnx.org/content/col11759/1.2>

521

How To: Given a set of data, perform logistic regression using a graphing utility.

1.Use the STAT then EDIT menu to enter given data.

a.Clear any existing data from the lists.b.List the input values in the L1 column.c.List the output values in the L2 column.

2.Graph and observe a scatter plot of the data using the STATPLOT feature.

a.Use ZOOM [9] to adjust axes to �t the data.b.Verify the data follow a logistic pattern.

3.Find the equation that models the data.

a.Select �Logistic� from the STAT then CALC menu.b.Use the values returned for a, b, and c to record the model, y = c

1+ae−bx .

4.Graph the model in the same window as the scatterplot to verify it is a good �t for the data.

Example 6.75Using Logistic Regression to Fit a Model to DataMobile telephone service has increased rapidly in America since the mid 1990s. Today, almost allresidents have cellular service. Table 6.42 shows the percentage of Americans with cellular servicebetween the years 1995 and 2012 38.

Year Americans with Cellular Service (%) Year Americans with Cellular Service (%)

1995 12.69 2004 62.852

1996 16.35 2005 68.63

1997 20.29 2006 76.64

1998 25.08 2007 82.47

1999 30.81 2008 85.68

2000 38.75 2009 89.14

2001 45.00 2010 91.86

2002 49.16 2011 95.28

2003 55.15 2012 98.17

Table 6.42

a. Letx represent time in years starting withx = 0 for the year 1995. Let y represent the corre-sponding percentage of residents with cellular service. Use logistic regression to �t a modelto these data.

b. Use the model to calculate the percentage of Americans with cell service in the year 2013.Round to the nearest tenth of a percent.

c. Discuss the value returned for the upper limit, c.What does this tell you about the model?What would the limiting value be if the model were exact?

Solution

38Source: The World Bank, 2013

Available for free at Connexions <http://legacy.cnx.org/content/col11759/1.2>

522 CHAPTER 6. EXPONENTIAL AND LOGARITHMIC FUNCTIONS

a. Using the STAT then EDIT menu on a graphing utility, list the years using values 0�15 in L1and the corresponding percentage in L2. Then use the STATPLOT feature to verify that thescatterplot follows a logistic pattern as shown in Figure 6.61:

Figure 6.61

Use the �Logistic� command from the STAT then CALC menu to obtain the logistic model,

y =105.7379526

1 + 6.88328979e−0.2595440013x(6.145)

Next, graph the model in the same window as shown in Figure 6.62 the scatterplot to verifyit is a good �t:

Available for free at Connexions <http://legacy.cnx.org/content/col11759/1.2>

523

Figure 6.62

b. To approximate the percentage of Americans with cellular service in the year 2013,substitutex = 18 for the in the model and solve for y :

y = 105.73795261+6.88328979e−0.2595440013x Use the regression model found in part (a).

= 105.73795261+6.88328979e−0.2595440013(18) Substitute 18 for x.

≈ 99.3 Round to the nearest tenth

(6.146)

According to the model, about 98.8% of Americans had cellular service in 2013.c. The model gives a limiting value of about 105. This means that the maximum possible

percentage of Americans with cellular service would be 105%, which is impossible. (Howcould over 100% of a population have cellular service?) If the model were exact, the limitingvalue would be c = 100 and the model's outputs would get very close to, but never actuallyreach 100%. After all, there will always be someone out there without cellular service!

Try It:

Exercise 6.9.3 (Solution on p. 580.)

Table 6.43 shows the population, in thousands, of harbor seals in the Wadden Sea overthe years 1997 to 2012.

Available for free at Connexions <http://legacy.cnx.org/content/col11759/1.2>

524 CHAPTER 6. EXPONENTIAL AND LOGARITHMIC FUNCTIONS

Year Seal Population (Thousands) Year Seal Population (Thousands)

1997 3.493 2005 19.590

1998 5.282 2006 21.955

1999 6.357 2007 22.862

2000 9.201 2008 23.869

2001 11.224 2009 24.243

2002 12.964 2010 24.344

2003 16.226 2011 24.919

2004 18.137 2012 25.108

Table 6.43

a.Letx represent time in years starting withx = 0 for the year 1997. Let y representthe number of seals in thousands. Use logistic regression to �t a model to these data.

b.Use the model to predict the seal population for the year 2020.c.To the nearest whole number, what is the limiting value of this model?

Media: Access this online resource for additional instruction and practice with exponentialfunction models.

• Exponential Regression on a Calculator39

Visit this website40 for additional practice questions from Learningpod.

6.9.4 Key Concepts

• Exponential regression is used to model situations where growth begins slowly and then acceleratesrapidly without bound, or where decay begins rapidly and then slows down to get closer and closer tozero.

• We use the command �ExpReg� on a graphing utility to �t function of the form y = abx to a set of datapoints. See Example 6.73.

• Logarithmic regression is used to model situations where growth or decay accelerates rapidly at �rstand then slows over time.

• We use the command �LnReg� on a graphing utility to �t a function of the form y = a + bln (x) to aset of data points. See Example 6.74.

• Logistic regression is used to model situations where growth accelerates rapidly at �rst and then steadilyslows as the function approaches an upper limit.

• We use the command �Logistic� on a graphing utility to �t a function of the form y = c1+ae−bx to a set

of data points. See Example 6.75.

39http://openstaxcollege.org/l/pregresscalc40http://openstaxcollege.org/l/PreCalcLPC04

Available for free at Connexions <http://legacy.cnx.org/content/col11759/1.2>

525

6.9.5 Section Exercises

6.9.5.1 Verbal

Exercise 6.9.4 (Solution on p. 580.)

What situations are best modeled by a logistic equation? Give an example, and state a case forwhy the example is a good �t.

Exercise 6.9.5What is a carrying capacity? What kind of model has a carrying capacity built into its formula?Why does this make sense?

Exercise 6.9.6 (Solution on p. 580.)

What is regression analysis? Describe the process of performing regression analysis on a graphingutility.

Exercise 6.9.7What might a scatterplot of data points look like if it were best described by a logarithmic model?

Exercise 6.9.8 (Solution on p. 580.)

What does the y-intercept on the graph of a logistic equation correspond to for a populationmodeled by that equation?

6.9.5.2 Graphical

For the following exercises, match the given function of best �t with the appropriate scatterplot in Figure 6.63through Figure 6.67. Answer using the letter beneath the matching graph.

Available for free at Connexions <http://legacy.cnx.org/content/col11759/1.2>

526 CHAPTER 6. EXPONENTIAL AND LOGARITHMIC FUNCTIONS

Figure 6.63

Available for free at Connexions <http://legacy.cnx.org/content/col11759/1.2>

527

Figure 6.64

Available for free at Connexions <http://legacy.cnx.org/content/col11759/1.2>

528 CHAPTER 6. EXPONENTIAL AND LOGARITHMIC FUNCTIONS

Figure 6.65

Available for free at Connexions <http://legacy.cnx.org/content/col11759/1.2>

529

Figure 6.66

Available for free at Connexions <http://legacy.cnx.org/content/col11759/1.2>

530 CHAPTER 6. EXPONENTIAL AND LOGARITHMIC FUNCTIONS

Figure 6.67

Exercise 6.9.9y = 10.209e−0.294x

Exercise 6.9.10 (Solution on p. 580.)

y = 5.598− 1.912ln (x)Exercise 6.9.11y = 2.104(1.479)x

Exercise 6.9.12 (Solution on p. 580.)

y = 4.607 + 2.733ln (x)Exercise 6.9.13y = 14.005

1+2.79e−0.812x

Available for free at Connexions <http://legacy.cnx.org/content/col11759/1.2>

531

6.9.5.3 Numeric

Exercise 6.9.14 (Solution on p. 580.)

To the nearest whole number, what is the initial value of a population modeled by the logisticequationP (t) = 175

1+6.995e−0.68t ?What is the carrying capacity?

Exercise 6.9.15Rewrite the exponential modelA (t) = 1550(1.085)x as an equivalent model with base e.Expressthe exponent to four signi�cant digits.

Exercise 6.9.16 (Solution on p. 580.)

A logarithmic model is given by the equationh (p) = 67.682−5.792ln (p) .To the nearest hundredth,for what value of p doesh (p) = 62?Exercise 6.9.17A logistic model is given by the equationP (t) = 90

1+5e−0.42t .To the nearest hundredth, for whatvalue of t doesP (t) = 45?Exercise 6.9.18 (Solution on p. 581.)

What is the y-intercept on the graph of the logistic model given in the previous exercise?

6.9.5.4 Technology

For the following exercises, use this scenario: The populationP of a koi pond overxmonths is modeled bythe functionP (x) = 68

1+16e−0.28x .

Exercise 6.9.19Graph the population model to show the population over a span of 3 years.Exercise 6.9.20 (Solution on p. 581.)

What was the initial population of koi?

Exercise 6.9.21How many koi will the pond have after one and a half years?

Exercise 6.9.22 (Solution on p. 581.)

How many months will it take before there are 20 koi in the pond?

Exercise 6.9.23 (Solution on p. 581.)

Use the intersect feature to approximate the number of months it will take before the populationof the pond reaches half its carrying capacity.

For the following exercises, use this scenario: The populationP of an endangered species habitat for wolvesis modeled by the functionP (x) = 558

1+54.8e−0.462x , where x is given in years.

Exercise 6.9.24Graph the population model to show the population over a span of 10 years.Exercise 6.9.25 (Solution on p. 581.)

What was the initial population of wolves transported to the habitat?

Exercise 6.9.26How many wolves will the habitat have after 3 years?Exercise 6.9.27 (Solution on p. 581.)

How many years will it take before there are 100wolves in the habitat?

Exercise 6.9.28Use the intersect feature to approximate the number of years it will take before the population ofthe habitat reaches half its carrying capacity.

For the following exercises, refer to Table 6.44.

Available for free at Connexions <http://legacy.cnx.org/content/col11759/1.2>

532 CHAPTER 6. EXPONENTIAL AND LOGARITHMIC FUNCTIONS

x 1 2 3 4 5 6

f(x) 1125 1495 2310 3294 4650 6361

Table 6.44

Exercise 6.9.29 (Solution on p. 581.)

Use a graphing calculator to create a scatter diagram of the data.

Exercise 6.9.30Use the regression feature to �nd an exponential function that best �ts the data in the table.

Exercise 6.9.31 (Solution on p. 582.)

Write the exponential function as an exponential equation with base e.

Exercise 6.9.32Graph the exponential equation on the scatter diagram.

Exercise 6.9.33 (Solution on p. 582.)

Use the intersect feature to �nd the value of x for which f (x) = 4000.For the following exercises, refer to Table 6.45.

x 1 2 3 4 5 6

f(x) 555 383 307 210 158 122

Table 6.45

Exercise 6.9.34Use a graphing calculator to create a scatter diagram of the data.

Exercise 6.9.35 (Solution on p. 582.)

Use the regression feature to �nd an exponential function that best �ts the data in the table.

Exercise 6.9.36Write the exponential function as an exponential equation with base e.

Exercise 6.9.37 (Solution on p. 582.)

Graph the exponential equation on the scatter diagram.

Exercise 6.9.38Use the intersect feature to �nd the value ofx for which f (x) = 250.

For the following exercises, refer to Table 6.46.

x 1 2 3 4 5 6

f(x) 5.1 6.3 7.3 7.7 8.1 8.6

Table 6.46

Exercise 6.9.39 (Solution on p. 583.)

Use a graphing calculator to create a scatter diagram of the data.

Exercise 6.9.40Use the LOGarithm option of the REGression feature to �nd a logarithmic function of the form y =a+ bln (x) that best �ts the data in the table.

Exercise 6.9.41 (Solution on p. 584.)

Use the logarithmic function to �nd the value of the function whenx = 10.

Available for free at Connexions <http://legacy.cnx.org/content/col11759/1.2>

533

Exercise 6.9.42Graph the logarithmic equation on the scatter diagram.

Exercise 6.9.43 (Solution on p. 584.)

Use the intersect feature to �nd the value ofx for which f (x) = 7.For the following exercises, refer to Table 6.47.

x 1 2 3 4 5 6 7 8

f(x) 7.5 6 5.2 4.3 3.9 3.4 3.1 2.9

Table 6.47

Exercise 6.9.44Use a graphing calculator to create a scatter diagram of the data.

Exercise 6.9.45 (Solution on p. 584.)

Use the LOGarithm option of the REGression feature to �nd a logarithmic function of the form y =a+ bln (x) that best �ts the data in the table.

Exercise 6.9.46Use the logarithmic function to �nd the value of the function whenx = 10.Exercise 6.9.47 (Solution on p. 584.)

Graph the logarithmic equation on the scatter diagram.

Exercise 6.9.48Use the intersect feature to �nd the value ofx for which f (x) = 8.

For the following exercises, refer to Table 6.48.

x 1 2 3 4 5 6 7 8 9 10

f(x) 8.7 12.3 15.4 18.5 20.7 22.5 23.3 24 24.6 24.8

Table 6.48

Exercise 6.9.49 (Solution on p. 585.)

Use a graphing calculator to create a scatter diagram of the data.

Exercise 6.9.50Use the LOGISTIC regression option to �nd a logistic growth model of the form y = c

1+ae−bx thatbest �ts the data in the table.

Exercise 6.9.51 (Solution on p. 586.)

Graph the logistic equation on the scatter diagram.

Exercise 6.9.52To the nearest whole number, what is the predicted carrying capacity of the model?

Exercise 6.9.53 (Solution on p. 587.)

Use the intersect feature to �nd the value of x for which the model reaches half its carrying capacity.

For the following exercises, refer to Table 6.49.

x 0 2 4 5 7 8 10 11 15 17

f (x) 12 28.6 52.8 70.3 99.9 112.5 125.8 127.9 135.1 135.9

Table 6.49

Available for free at Connexions <http://legacy.cnx.org/content/col11759/1.2>

534 CHAPTER 6. EXPONENTIAL AND LOGARITHMIC FUNCTIONS

Exercise 6.9.54Use a graphing calculator to create a scatter diagram of the data.

Exercise 6.9.55 (Solution on p. 588.)

Use the LOGISTIC regression option to �nd a logistic growth model of the form y = c1+ae−bx that

best �ts the data in the table.

Exercise 6.9.56Graph the logistic equation on the scatter diagram.

Exercise 6.9.57 (Solution on p. 588.)

To the nearest whole number, what is the predicted carrying capacity of the model?

Exercise 6.9.58Use the intersect feature to �nd the value of x for which the model reaches half its carrying capacity.

6.9.5.5 Extensions

Exercise 6.9.59 (Solution on p. 588.)

Recall that the general form of a logistic equation for a population is given byP (t) = c1+ae−bt , such

that the initial population at time t = 0 isP (0) = P0. Show algebraically that c−P (t)P (t) = c−P0

P0e−bt.

Exercise 6.9.60Use a graphing utility to �nd an exponential regression formula f (x) and a logarithmic regressionformula g (x) for the points (1.5, 1.5) and (8.5, 8.5) .Round all numbers to 6 decimal places. Graphthe points and both formulas along with the line y = x on the same axis. Make a conjecture aboutthe relationship of the regression formulas.

Exercise 6.9.61 (Solution on p. 588.)

Verify the conjecture made in the previous exercise. Round all numbers to six decimal places whennecessary.

Exercise 6.9.62Find the inverse function f−1 (x) for the logistic function f (x) = c

1+ae−bx . Show all steps.

Exercise 6.9.63 (Solution on p. 589.)

Use the result from the previous exercise to graph the logistic model P (t) = 201+4e−0.5t along with

its inverse on the same axis. What are the intercepts and asymptotes of each function?

6.9.6 Chapter Review Exercises

6.9.6.1 Exponential Functions (Section 6.2)

Exercise 6.9.64 (Solution on p. 589.)

Determine whether the function y = 156(0.825)t represents exponential growth, exponential decay,or neither. Explain

Exercise 6.9.65The population of a herd of deer is represented by the functionA (t) = 205(1.13)t, where t is givenin years. To the nearest whole number, what will the herd population be after 6 years?Exercise 6.9.66 (Solution on p. 589.)

Find an exponential equation that passes through the points (2, 2.25) and (5, 60.75) .Exercise 6.9.67Determine whether Table 6.50 could represent a function that is linear, exponential, or neither. Ifit appears to be exponential, �nd a function that passes through the points.

Available for free at Connexions <http://legacy.cnx.org/content/col11759/1.2>

535

x 1 2 3 4

f(x) 3 0.9 0.27 0.081

Table 6.50

Exercise 6.9.68 (Solution on p. 589.)

A retirement account is opened with an initial deposit of $8,500 and earns 8.12% interest com-pounded monthly. What will the account be worth in 20 years?Exercise 6.9.69Hsu-Mei wants to save $5,000 for a down payment on a car. To the nearest dollar, how much willshe need to invest in an account now with 7.5%APR, compounded daily, in order to reach her goalin 3 years?Exercise 6.9.70 (Solution on p. 589.)

Does the equation y = 2.294e−0.654t represent continuous growth, continuous decay, or neither?Explain.

Exercise 6.9.71Suppose an investment account is opened with an initial deposit of $10,500 earning 6.25% interest,compounded continuously. How much will the account be worth after 25 years?

6.9.6.2 Graphs of Exponential Functions (Section 6.3)

Exercise 6.9.72 (Solution on p. 589.)

Graph the function f (x) = 3.5(2)x. State the domain and range and give the y-intercept.

Exercise 6.9.73Graph the function f (x) = 4

(18

)xand its re�ection about the y-axis on the same axes, and give

the y-intercept.

Exercise 6.9.74 (Solution on p. 590.)

The graph of f (x) = 6.5x is re�ected about the y-axis and stretched vertically by a factor of 7.Whatis the equation of the new function, g (x)? State its y-intercept, domain, and range.

Exercise 6.9.75The graph below shows transformations of the graph of f (x) = 2x.What is the equation for thetransformation?

Available for free at Connexions <http://legacy.cnx.org/content/col11759/1.2>

536 CHAPTER 6. EXPONENTIAL AND LOGARITHMIC FUNCTIONS

6.9.6.3 Logarithmic Functions (Section 6.4)

Exercise 6.9.76 (Solution on p. 590.)

Rewrite log17 (4913) = x as an equivalent exponential equation.

Exercise 6.9.77Rewrite ln (s) = t as an equivalent exponential equation.

Exercise 6.9.78 (Solution on p. 590.)

Rewrite a−25 = b as an equivalent logarithmic equation.

Exercise 6.9.79Rewrite e−3.5 = h as an equivalent logarithmic equation.

Exercise 6.9.80 (Solution on p. 590.)

Solve forx log64 (x) = 13 to exponential form.

Exercise 6.9.81Evaluate log5

(1

125

)without using a calculator.

Exercise 6.9.82 (Solution on p. 590.)

Evaluate log (0.000001) without using a calculator.

Available for free at Connexions <http://legacy.cnx.org/content/col11759/1.2>

537

Exercise 6.9.83Evaluate log (4.005) using a calculator. Round to the nearest thousandth.

Exercise 6.9.84 (Solution on p. 590.)

Evaluate ln(e−0.8648

)without using a calculator.

Exercise 6.9.85Evaluate ln

(3√

18)using a calculator. Round to the nearest thousandth.

6.9.6.4 Graphs of Logarithmic Functions (Section 6.5)

Exercise 6.9.86 (Solution on p. 590.)

Graph the function g (x) = log (7x+ 21)− 4.Exercise 6.9.87Graph the functionh (x) = 2ln (9− 3x) + 1.Exercise 6.9.88 (Solution on p. 591.)

State the domain, vertical asymptote, and end behavior of the function g (x) = ln (4x+ 20)− 17.

6.9.6.5 Logarithmic Properties (Section 6.6)

Exercise 6.9.89Rewrite ln (7r · 11st) in expanded form.

Exercise 6.9.90 (Solution on p. 591.)

Rewrite log8 (x) + log8 (5) + log8 (y) + log8 (13) in compact form.

Exercise 6.9.91Rewrite logm

(6783

)in expanded form.

Exercise 6.9.92 (Solution on p. 591.)

Rewrite ln (z)− ln (x)− ln (y) in compact form.

Exercise 6.9.93Rewrite ln

(1x5

)as a product.

Exercise 6.9.94 (Solution on p. 591.)

Rewrite − logy(

112

)as a single logarithm.

Exercise 6.9.95Use properties of logarithms to expand log

(r2s11

t14

).

Exercise 6.9.96 (Solution on p. 591.)

Use properties of logarithms to expand ln(

2b√

b+1b−1

).

Exercise 6.9.97Condense the expression 5ln (b) + ln (c) + ln(4−a)

2 to a single logarithm.

Exercise 6.9.98 (Solution on p. 591.)

Condense the expression 3log7v + 6log7w −log7u

3 to a single logarithm.

Exercise 6.9.99Rewrite log3 (12.75) to base e.Exercise 6.9.100 (Solution on p. 591.)

Rewrite 512x−17 = 125 as a logarithm. Then apply the change of base formula to solve forx usingthe common log. Round to the nearest thousandth.

Available for free at Connexions <http://legacy.cnx.org/content/col11759/1.2>

538 CHAPTER 6. EXPONENTIAL AND LOGARITHMIC FUNCTIONS

6.9.6.6 Exponential and Logarithmic Equations (Section 6.7)

Exercise 6.9.101Solve 2163x · 216x = 363x+2 by rewriting each side with a common base.

Exercise 6.9.102 (Solution on p. 591.)

Solve 125

( 1625 )−x−3 = 53 by rewriting each side with a common base.

Exercise 6.9.103Use logarithms to �nd the exact solution for 7 · 17−9x − 7 = 49. If there is no solution, write nosolution.

Exercise 6.9.104 (Solution on p. 591.)

Use logarithms to �nd the exact solution for 3e6n−2 + 1 = −60. If there is no solution, write nosolution.

Exercise 6.9.105Find the exact solution for 5e3x − 4 = 6 . If there is no solution, write no solution.Exercise 6.9.106 (Solution on p. 591.)

Find the exact solution for 2e5x−2 − 9 = −56. If there is no solution, write no solution.Exercise 6.9.107Find the exact solution for 52x−3 = 7x+1. If there is no solution, write no solution.

Exercise 6.9.108 (Solution on p. 591.)

Find the exact solution for e2x − ex − 110 = 0. If there is no solution, write no solution.Exercise 6.9.109Use the de�nition of a logarithm to solve. − 5log7 (10n) = 5.Exercise 6.9.110 (Solution on p. 591.)

47. Use the de�nition of a logarithm to �nd the exact solution for 9 + 6ln (a+ 3) = 33.Exercise 6.9.111Use the one-to-one property of logarithms to �nd an exact solution for log8 (7) + log8 (−4x) =

log8 (5) . If there is no solution, write no solution.Exercise 6.9.112 (Solution on p. 592.)

Use the one-to-one property of logarithms to �nd an exact solution for ln (5) + ln(5x2 − 5

)=

ln (56) . If there is no solution, write no solution.Exercise 6.9.113The formula for measuring sound intensity in decibelsD is de�ned by the equationD = 10log

(II0

),

where I is the intensity of the sound in watts per square meter and I0 = 10−12 is the lowest levelof sound that the average person can hear. How many decibels are emitted from a large orchestrawith a sound intensity of 6.3 · 10−3 watts per square meter?

Exercise 6.9.114 (Solution on p. 592.)

The population of a city is modeled by the equationP (t) = 256, 114e0.25t where t is measured inyears. If the city continues to grow at this rate, how many years will it take for the population toreach one million?

Exercise 6.9.115Find the inverse function f−1 for the exponential function f (x) = 2 · ex+1 − 5.Exercise 6.9.116 (Solution on p. 592.)

Find the inverse function f−1 for the logarithmic function f (x) = 0.25 · log2

(x3 + 1

).

Available for free at Connexions <http://legacy.cnx.org/content/col11759/1.2>

539

6.9.6.7 Exponential and Logarithmic Models (Section 6.8)

For the following exercises, use this scenario: A doctor prescribes 300milligrams of a therapeutic drug thatdecays by about 17% each hour.

Exercise 6.9.117To the nearest minute, what is the half-life of the drug?

Exercise 6.9.118 (Solution on p. 592.)

Write an exponential model representing the amount of the drug remaining in the patient's systemafter t hours. Then use the formula to �nd the amount of the drug that would remain in the patient'ssystem after 24 hours. Round to the nearest hundredth of a gram.

For the following exercises, use this scenario: A soup with an internal temperature of 350 ◦ Fahrenheit wastaken o� the stove to cool in a 71 ◦F room. After �fteen minutes, the internal temperature of the soupwas 175 ◦F.

Exercise 6.9.119Use Newton's Law of Cooling to write a formula that models this situation.

Exercise 6.9.120 (Solution on p. 592.)

How many minutes will it take the soup to cool to 85 ◦F?

For the following exercises, use this scenario: The equationN (t) = 12001+199e−0.625t models the number of people

in a school who have heard a rumor after t days.

Exercise 6.9.121How many people started the rumor?

Exercise 6.9.122 (Solution on p. 592.)

To the nearest tenth, how many days will it be before the rumor spreads to half the carryingcapacity?

Exercise 6.9.123What is the carrying capacity?

For the following exercises, enter the data from each table into a graphing calculator and graph the resultingscatter plots. Determine whether the data from the table would likely represent a function that is linear,exponential, or logarithmic.

Exercise 6.9.124 (Solution on p. 592.)

x 1 2 3 4 5 6 7 8 9 10

f(x) 3.05 4.42 6.4 9.28 13.46 19.52 28.3 41.04 59.5 86.28

Table 6.51

Exercise 6.9.125

x 0.5 1 3 5 7 10 12 13 15 17 20

f(x) 18.05 17 15.33 14.55 14.04 13.5 13.22 13.1 12.88 12.69 12.45

Table 6.52

Exercise 6.9.126 (Solution on p. 592.)

Find a formula for an exponential equation that goes through the points (−2, 100) and (0, 4) .Thenexpress the formula as an equivalent equation with base e.

Available for free at Connexions <http://legacy.cnx.org/content/col11759/1.2>

540 CHAPTER 6. EXPONENTIAL AND LOGARITHMIC FUNCTIONS

6.9.6.8 Fitting Exponential Models to Data

Exercise 6.9.127What is the carrying capacity for a population modeled by the logistic equationP (t) =

250,0001 + 499e−0.45t ?What is the initial population for the model?

Exercise 6.9.128 (Solution on p. 592.)

The population of a culture of bacteria is modeled by the logistic equationP (t) = 14,2501 + 29e−0.62t ,

where t is in days. To the nearest tenth, how many days will it take the culture to reach 75% of itscarrying capacity?

For the following exercises, use a graphing utility to create a scatter diagram of the data given in the table.Observe the shape of the scatter diagram to determine whether the data is best described by an exponential,logarithmic, or logistic model. Then use the appropriate regression feature to �nd an equation that modelsthe data. When necessary, round values to �ve decimal places.

Exercise 6.9.129

x 1 2 3 4 5 6 7 8 9 10

f(x) 409.4 260.7 170.4 110.6 74 44.7 32.4 19.5 12.7 8.1

Table 6.53

Exercise 6.9.130 (Solution on p. 592.)

x 0.15 0.25 0.5 0.75 1 1.5 2 2.25 2.75 3 3.5

f(x) 36.21 28.88 24.39 18.28 16.5 12.99 9.91 8.57 7.23 5.99 4.81

Table 6.54

Exercise 6.9.131

x 0 2 4 5 7 8 10 11 15 17

f(x) 9 22.6 44.2 62.1 96.9 113.4 133.4 137.6 148.4 149.3

Table 6.55

6.9.7 Practice Test

Exercise 6.9.132 (Solution on p. 593.)

The population of a pod of bottlenose dolphins is modeled by the functionA (t) = 8(1.17)t,where t is given in years. To the nearest whole number, what will the pod population beafter 3 years?Exercise 6.9.133Find an exponential equation that passes through the points (0, 4) and (2, 9).

Available for free at Connexions <http://legacy.cnx.org/content/col11759/1.2>

541

Exercise 6.9.134 (Solution on p. 594.)

Drew wants to save $2,500 to go to the next World Cup. To the nearest dollar, how much will heneed to invest in an account now with 6.25%APR, compounding daily, in order to reach his goalin 4 years?Exercise 6.9.135An investment account was opened with an initial deposit of $9,600 and earns 7.4% interest, com-pounded continuously. How much will the account be worth after 15 years?Exercise 6.9.136 (Solution on p. 594.)

Graph the function f (x) = 5(0.5)−x and its re�ection across the y-axis on the same axes, and givethe y-intercept.

Exercise 6.9.137The graph below shows transformations of the graph of f (x) =

(12

)x.What is the equation for the

transformation?

Exercise 6.9.138 (Solution on p. 594.)

Rewrite log8.5 (614.125) = a as an equivalent exponential equation.

Exercise 6.9.139Rewrite e

12 = m as an equivalent logarithmic equation.

Available for free at Connexions <http://legacy.cnx.org/content/col11759/1.2>

542 CHAPTER 6. EXPONENTIAL AND LOGARITHMIC FUNCTIONS

Exercise 6.9.140 (Solution on p. 594.)

Solve forx by converting the logarithmic equation log 17

(x) = 2 to exponential form.Exercise 6.9.141Evaluate log (10,000,000) without using a calculator.Exercise 6.9.142 (Solution on p. 594.)

Evaluate ln (0.716) using a calculator. Round to the nearest thousandth.

Exercise 6.9.143Graph the function g (x) = log (12− 6x) + 3.Exercise 6.9.144 (Solution on p. 594.)

State the domain, vertical asymptote, and end behavior of the function f (x) = log5 (39− 13x)+7.Exercise 6.9.145Rewrite log (17a · 2b) as a sum.Exercise 6.9.146 (Solution on p. 594.)

Rewrite logt (96)− logt (8) in compact form.

Exercise 6.9.147Rewrite log8

(a

1b

)as a product.

Exercise 6.9.148 (Solution on p. 594.)

Use properties of logarithm to expand ln(y3z2 · 3

√x− 4

).

Exercise 6.9.149Condense the expression 4ln (c) + ln (d) + ln(a)

3 + ln(b+3)3 to a single logarithm.

Exercise 6.9.150 (Solution on p. 594.)

Rewrite 163x−5 = 1000 as a logarithm. Then apply the change of base formula to solve forx usingthe natural log. Round to the nearest thousandth.

Exercise 6.9.151Solve

(181

)x · 1243 =

(19

)−3x−1by rewriting each side with a common base.

Exercise 6.9.152 (Solution on p. 594.)

Use logarithms to �nd the exact solution for − 9e10a−8 − 5 = −41. If there is no solution, writeno solution.

Exercise 6.9.153Find the exact solution for 10e4x+2 + 5 = 56. If there is no solution, write no solution.Exercise 6.9.154 (Solution on p. 594.)

Find the exact solution for − 5e−4x−1 − 4 = 64. If there is no solution, write no solution.Exercise 6.9.155Find the exact solution for 2x−3 = 62x−1. If there is no solution, write no solution.

Exercise 6.9.156 (Solution on p. 594.)

Find the exact solution for e2x − ex − 72 = 0. If there is no solution, write no solution.Exercise 6.9.157Use the de�nition of a logarithm to �nd the exact solution for 4log (2n)− 7 = −11Exercise 6.9.158 (Solution on p. 595.)

Use the one-to-one property of logarithms to �nd an exact solution for log(4x2 − 10

)+ log (3) =

log (51) If there is no solution, write no solution.Exercise 6.9.159The formula for measuring sound intensity in decibelsD is de�ned by the equationD =

10log(II0

),where I is the intensity of the sound in watts per square meter and I0 = 10−12 is the

lowest level of sound that the average person can hear. How many decibels are emitted from a rockconcert with a sound intensity of 4.7 · 10−1 watts per square meter?

Available for free at Connexions <http://legacy.cnx.org/content/col11759/1.2>

543

Exercise 6.9.160 (Solution on p. 595.)

A radiation safety o�cer is working with 112 grams of a radioactive substance. After 17 days, thesample has decayed to 80 grams. Rounding to �ve signi�cant digits, write an exponential equationrepresenting this situation. To the nearest day, what is the half-life of this substance?

Exercise 6.9.161Write the formula found in the previous exercise as an equivalent equation with base e.Express theexponent to �ve signi�cant digits.

Exercise 6.9.162 (Solution on p. 595.)

A bottle of soda with a temperature of 71 ◦ Fahrenheit was taken o� a shelf and placed in arefrigerator with an internal temperature of 35 ◦ F.After ten minutes, the internal temperature ofthe soda was 63 ◦ F.Use Newton's Law of Cooling to write a formula that models this situation. Tothe nearest degree, what will the temperature of the soda be after one hour?

Exercise 6.9.163The population of a wildlife habitat is modeled by the equationP (t) = 360

1+6.2e−0.35t , where t isgiven in years. How many animals were originally transported to the habitat? How many yearswill it take before the habitat reaches half its capacity?

Exercise 6.9.164 (Solution on p. 595.)

Enter the data from Table 6.56 into a graphing calculator and graph the resulting scatter plot. De-termine whether the data from the table would likely represent a function that is linear, exponential,or logarithmic.

x 1 2 3 4 5 6 7 8 9 10

f(x) 3 8.55 11.79 14.09 15.88 17.33 18.57 19.64 20.58 21.42

Table 6.56

Exercise 6.9.165The population of a lake of �sh is modeled by the logistic equationP (t) = 16,120

1+25e−0.75t , where t istime in years. To the nearest hundredth, how many years will it take the lake to reach 80% of itscarrying capacity?

For the following exercises, use a graphing utility to create a scatter diagram of the data given in the table.Observe the shape of the scatter diagram to determine whether the data is best described by an exponential,logarithmic, or logistic model. Then use the appropriate regression feature to �nd an equation that modelsthe data. When necessary, round values to �ve decimal places.

Exercise 6.9.166 (Solution on p. 596.)

x 1 2 3 4 5 6 7 8 9 10

f(x) 20 21.6 29.2 36.4 46.6 55.7 72.6 87.1 107.2 138.1

Table 6.57

Exercise 6.9.167

x 3 4 5 6 7 8 9 10 11 12 13

f(x) 13.98 17.84 20.01 22.7 24.1 26.15 27.37 28.38 29.97 31.07 31.43

Available for free at Connexions <http://legacy.cnx.org/content/col11759/1.2>

544 CHAPTER 6. EXPONENTIAL AND LOGARITHMIC FUNCTIONS

Table 6.58

Exercise 6.9.168 (Solution on p. 597.)

x 0 0.5 1 1.5 2 3 4 5 6 7 8

f(x) 2.2 2.9 3.9 4.8 6.4 9.3 12.3 15 16.2 17.3 17.9

Table 6.59

Available for free at Connexions <http://legacy.cnx.org/content/col11759/1.2>

545

Solutions to Exercises in Chapter 6

Solution to Exercise 6.2.1 (p. 343)g (x) = 0.875x andj (x) = 1095.6−2x represent exponential functions.Solution to Exercise 6.2.2 (p. 344)5.5556Solution to Exercise 6.2.3 (p. 347)About 1.548 billion people; by the year 2031, India's population will exceed China's by about 0.001 billion,or 1 million people.Solution to Exercise 6.2.4 (p. 350)(0, 129) and (2, 236) ; N (t) = 129(1.3526)t

Solution to Exercise 6.2.5 (p. 351)f (x) = 2(1.5)x

Solution to Exercise 6.2.6 (p. 353)

f (x) =√

2(√

2)x.Answers may vary due to round-o� error. The answer should be very close

to 1.4142(1.4142)x.Solution to Exercise 6.2.7 (p. 354)y ≈ 12 · 1.85x

Solution to Exercise 6.2.8 (p. 355)about $3,644,675.88Solution to Exercise 6.2.9 (p. 356)$13,693Solution to Exercise 6.2.10 (p. 357)e−0.5 ≈ 0.60653Solution to Exercise 6.2.11 (p. 359)$3,659,823.44Solution to Exercise 6.2.12 (p. 359)3.77E-26 (This is calculator notation for the number written as 3.77×10−26 in scienti�c notation. While theoutput of an exponential function is never zero, this number is so close to zero that for all practical purposeswe can accept zero as the answer.)Solution to Exercise 6.2.13 (p. 360)Linear functions have a constant rate of change. Exponential functions increase based on a percent of theoriginal.Solution to Exercise 6.2.15 (p. 361)When interest is compounded, the percentage of interest earned to principal ends up being greater than theannual percentage rate for the investment account. Thus, the annual percentage rate does not necessarilycorrespond to the real interest earned, which is the very de�nition of nominal.Solution to Exercise 6.2.17 (p. 361)exponential; the population decreases by a proportional rate. .Solution to Exercise 6.2.19 (p. 361)not exponential; the charge decreases by a constant amount each visit, so the statement represents a linearfunction. .Solution to Exercise 6.2.21 (p. 361)The forest represented by the functionB (t) = 82(1.029)t.Solution to Exercise 6.2.23 (p. 361)After t = 20 years, forest A will have 43more trees than forest B.Solution to Exercise 6.2.25 (p. 361)Answers will vary. Sample response: For a number of years, the population of forest A will increasinglyexceed forest B, but because forest B actually grows at a faster rate, the population will eventually becomelarger than forest A and will remain that way as long as the population growth models hold. Some factors

Available for free at Connexions <http://legacy.cnx.org/content/col11759/1.2>

546 CHAPTER 6. EXPONENTIAL AND LOGARITHMIC FUNCTIONS

that might in�uence the long-term validity of the exponential growth model are drought, an epidemic thatculls the population, and other environmental and biological factors.Solution to Exercise 6.2.27 (p. 361)exponential growth; The growth factor, 1.06, is greater than 1.Solution to Exercise 6.2.29 (p. 362)exponential decay; The decay factor, 0.97, is between 0 and 1.Solution to Exercise 6.2.31 (p. 362)f (x) = 2000(0.1)x

Solution to Exercise 6.2.33 (p. 362)

f (x) =(

16

)− 35(

16

) x5 ≈ 2.93(0.699)x

Solution to Exercise 6.2.35 (p. 362)LinearSolution to Exercise 6.2.37 (p. 362)NeitherSolution to Exercise 6.2.39 (p. 363)LinearSolution to Exercise 6.2.41 (p. 363)$10, 250Solution to Exercise 6.2.43 (p. 363)$13, 268.58Solution to Exercise 6.2.45 (p. 363)

P = A (t) ·(1 + r

n

)−ntSolution to Exercise 6.2.47 (p. 363)$4, 572.56Solution to Exercise 6.2.49 (p. 363)4%Solution to Exercise 6.2.51 (p. 364)continuous growth; the growth rate is greater than 0.Solution to Exercise 6.2.53 (p. 364)continuous decay; the growth rate is less than 0.Solution to Exercise 6.2.55 (p. 364)$669.42Solution to Exercise 6.2.57 (p. 364)f (−1) = −4Solution to Exercise 6.2.59 (p. 364)f (−1) ≈ −0.2707Solution to Exercise 6.2.61 (p. 364)f (3) ≈ 483.8146Solution to Exercise 6.2.63 (p. 364)y = 3 · 5xSolution to Exercise 6.2.65 (p. 364)y ≈ 18 · 1.025x

Solution to Exercise 6.2.67 (p. 364)y ≈ 0.2 · 1.95x

Solution to Exercise 6.2.69 (p. 365)

APY = A(t)−aa =

a(1+ r365 )365(1)−a

a =ah(1+ r

365 )365−1i

a =(1 + r

365

)365 − 1;I (n) =(1 + r

n

)n − 1Solution to Exercise 6.2.71 (p. 365)Let f be the exponential decay function f (x) = a ·

(1b

)xsuch that b > 1.Then for some numbern > 0,f (x) =

a ·(

1b

)x = a(b−1)x = a

((en)−1

)x= a(e−n)x = a(e)−nx.

Available for free at Connexions <http://legacy.cnx.org/content/col11759/1.2>

547

Solution to Exercise 6.2.73 (p. 365)47, 622 foxSolution to Exercise 6.2.75 (p. 365)1.39%; $155, 368.09Solution to Exercise 6.2.77 (p. 366)$35, 838.76Solution to Exercise 6.2.79 (p. 366)$82, 247.78; $449.75Solution to Exercise 6.3.1 (p. 371)The domain is (−∞,∞) ; the range is (0,∞) ; the horizontal asymptote is y = 0.

So-lution to Exercise 6.3.2 (p. 376)The domain is (−∞,∞) ; the range is (3,∞) ; the horizontal asymptote is y = 3.

Available for free at Connexions <http://legacy.cnx.org/content/col11759/1.2>

548 CHAPTER 6. EXPONENTIAL AND LOGARITHMIC FUNCTIONS

So-lution to Exercise 6.3.3 (p. 377)x ≈ −1.608Solution to Exercise 6.3.4 (p. 379)The domain is (−∞,∞) ; the range is (0,∞) ; the horizontal asymptote is y = 0.

Available for free at Connexions <http://legacy.cnx.org/content/col11759/1.2>

549

Solution to Exercise 6.3.5 (p. 382)The domain is (−∞,∞) ; the range is (0,∞) ; the horizontal asymptote is y = 0.

So-lution to Exercise 6.3.6 (p. 385)f (x) = − 1

3ex − 2; the domain is (−∞,∞) ; the range is (−∞, 2) ; the horizontal asymptote is y = 2.

Available for free at Connexions <http://legacy.cnx.org/content/col11759/1.2>

550 CHAPTER 6. EXPONENTIAL AND LOGARITHMIC FUNCTIONS

Solution to Exercise 6.3.7 (p. 386)An asymptote is a line that the graph of a function approaches, asx either increases or decreases withoutbound. The horizontal asymptote of an exponential function tells us the limit of the function's values as theindependent variable gets either extremely large or extremely small.Solution to Exercise 6.3.9 (p. 386)g (x) = 4(3)−x; y-intercept: (0, 4) ; Domain: all real numbers; Range: all real numbers greater than 0.Solution to Exercise 6.3.11 (p. 386)g (x) = −10x + 7; y-intercept: (0, 6) ; Domain: all real numbers; Range: all real numbers less than 7.Solution to Exercise 6.3.13 (p. 386)g (x) = 2

(14

)x; y-intercept: (0, 2) ; Domain: all real numbers; Range: all real numbers greater than 0.Solution to Exercise 6.3.15 (p. 387)

y-intercept: (0,−2)

Available for free at Connexions <http://legacy.cnx.org/content/col11759/1.2>

551

Solution to Exercise 6.3.17 (p. 387)

Solu-tion to Exercise 6.3.19 (p. 388)BSolution to Exercise 6.3.21 (p. 388)ASolution to Exercise 6.3.23 (p. 389)ESolution to Exercise 6.3.25 (p. 389)DSolution to Exercise 6.3.27 (p. 390)C

Available for free at Connexions <http://legacy.cnx.org/content/col11759/1.2>

552 CHAPTER 6. EXPONENTIAL AND LOGARITHMIC FUNCTIONS

Solution to Exercise 6.3.29 (p. 390)

Solu-

Available for free at Connexions <http://legacy.cnx.org/content/col11759/1.2>

553

tion to Exercise 6.3.31 (p. 390)

Solu-

Available for free at Connexions <http://legacy.cnx.org/content/col11759/1.2>

554 CHAPTER 6. EXPONENTIAL AND LOGARITHMIC FUNCTIONS

tion to Exercise 6.3.33 (p. 390)

Hori-zontal asymptote:h (x) = 3; Domain: all real numbers; Range: all real numbers strictly greater than 3.Solution to Exercise 6.3.35 (p. 390)Asx→∞, f (x)→ −∞ ; As x→ −∞, f (x)→ −1Solution to Exercise 6.3.37 (p. 390)Asx→∞ , f (x)→ 2; Asx→ −∞ , f (x)→∞Solution to Exercise 6.3.39 (p. 390)f (x) = 4x − 3Solution to Exercise 6.3.41 (p. 390)f (x) = 4x−5

Solution to Exercise 6.3.43 (p. 390)f (x) = 4−x

Solution to Exercise 6.3.45 (p. 391)y = −2x + 3Solution to Exercise 6.3.47 (p. 393)y = −2(3)x + 7Solution to Exercise 6.3.49 (p. 395)g (6) = 800 + 1

3 ≈ 800.3333Solution to Exercise 6.3.51 (p. 395)h (−7) = −58Solution to Exercise 6.3.53 (p. 395)x ≈ −2.953Solution to Exercise 6.3.55 (p. 396)x ≈ −0.222Solution to Exercise 6.3.57 (p. 396)The graph ofG (x) =

(1b

)xis the refelction about the y-axis of the graph ofF (x) = bx; For any real

number b > 0 and function f (x) = bx,the graph of(

1b

)xis the the re�ection about the y-axis,F (−x) .

Available for free at Connexions <http://legacy.cnx.org/content/col11759/1.2>

555

Solution to Exercise 6.3.59 (p. 396)The graphs of g (x) andh (x) are the same and are a horizontal shift to the right of the graph of f (x) ; Forany real number n, real number b > 0, and function f (x) = bx, the graph of

(1bn

)bx is the horizontal

shift f (x− n) .Solution to Exercise 6.4.1 (p. 401)

a. log10 (1, 000, 000) = 6 is equivalent to 106 = 1, 000, 000b. log5 (25) = 2 is equivalent to 52 = 25

Solution to Exercise 6.4.2 (p. 402)

a. 32 = 9 is equivalent to log3 (9) = 2b. 53 = 125 is equivalent to log5 (125) = 3c. 2−1 = 1

2 is equivalent to log2

(12

)= −1

Solution to Exercise 6.4.3 (p. 403)

log121 (11) = 12 (recalling that

√121 = (121)

12 = 11)

Solution to Exercise 6.4.4 (p. 403)log2

(132

)= −5

Solution to Exercise 6.4.5 (p. 404)log (1, 000, 000) = 6Solution to Exercise 6.4.6 (p. 405)log (123) ≈ 2.0899Solution to Exercise 6.4.7 (p. 405)The di�erence in magnitudes was about 3.929.Solution to Exercise 6.4.8 (p. 406)It is not possible to take the logarithm of a negative number in the set of real numbers.Solution to Exercise 6.4.9 (p. 407)A logarithm is an exponent. Speci�cally, it is the exponent to which a base b is raised to produce a givenvalue. In the expressions given, the base b has the same value. The exponent, y,in the expression by can alsobe written as the logarithm, logbx,and the value ofx is the result of raising b to the power of y.Solution to Exercise 6.4.11 (p. 407)Since the equation of a logarithm is equivalent to an exponential equation, the logarithm can be convertedto the exponential equation by = x, and then properties of exponents can be applied to solve forx.Solution to Exercise 6.4.13 (p. 407)The natural logarithm is a special case of the logarithm with base b in that the natural log always hasbase e.Rather than notating the natural logarithm as loge (x) ,the notation used is ln (x) .Solution to Exercise 6.4.15 (p. 408)ac = bSolution to Exercise 6.4.17 (p. 408)xy = 64Solution to Exercise 6.4.19 (p. 408)15b = aSolution to Exercise 6.4.21 (p. 408)13a = 142Solution to Exercise 6.4.23 (p. 408)en = wSolution to Exercise 6.4.25 (p. 408)logc (k) = dSolution to Exercise 6.4.27 (p. 408)log19y = xSolution to Exercise 6.4.29 (p. 408)logn (103) = 4

Available for free at Connexions <http://legacy.cnx.org/content/col11759/1.2>

556 CHAPTER 6. EXPONENTIAL AND LOGARITHMIC FUNCTIONS

Solution to Exercise 6.4.31 (p. 408)logy

(39100

)= x

Solution to Exercise 6.4.33 (p. 408)ln (h) = kSolution to Exercise 6.4.35 (p. 409)x = 2−3 = 1

8Solution to Exercise 6.4.37 (p. 409)x = 33 = 27Solution to Exercise 6.4.39 (p. 409)

x = 912 = 3

Solution to Exercise 6.4.41 (p. 409)x = 6−3 = 1

216Solution to Exercise 6.4.43 (p. 409)x = e2

Solution to Exercise 6.4.45 (p. 409)32Solution to Exercise 6.4.47 (p. 409)1.06Solution to Exercise 6.4.49 (p. 409)14.125Solution to Exercise 6.4.51 (p. 409)

12Solution to Exercise 6.4.53 (p. 409)4Solution to Exercise 6.4.55 (p. 410)−3Solution to Exercise 6.4.57 (p. 410)−12Solution to Exercise 6.4.59 (p. 410)0Solution to Exercise 6.4.61 (p. 410)10Solution to Exercise 6.4.63 (p. 410)2.708Solution to Exercise 6.4.65 (p. 410)0.151Solution to Exercise 6.4.67 (p. 410)No, the function has no de�ned value forx = 0.To verify, supposex = 0 is in the domain of the

function f (x) = log (x) .Then there is some numbern such thatn = log (0) .Rewriting as an exponentialequation gives: 10n = 0, which is impossible since no such real numbern exists. Therefore,x = 0 is not thedomain of the function f (x) = log (x) .Solution to Exercise 6.4.69 (p. 410)Yes. Suppose there exists a real numberx such that lnx = 2.Rewriting as an exponential equation givesx =e2, which is a real number. To verify, letx = e2.Then, by de�nition, ln (x) = ln

(e2)

= 2.Solution to Exercise 6.4.71 (p. 411)

No; ln (1) = 0, soln(e1.725)

ln(1) is unde�ned.

Solution to Exercise 6.4.73 (p. 411)2Solution to Exercise 6.5.1 (p. 413)(2,∞)

Available for free at Connexions <http://legacy.cnx.org/content/col11759/1.2>

557

Solution to Exercise 6.5.2 (p. 414)(5,∞)Solution to Exercise 6.5.3 (p. 418)

Thedomain is (0,∞) ,the range is (−∞,∞) , and the vertical asymptote isx = 0.

Available for free at Connexions <http://legacy.cnx.org/content/col11759/1.2>

558 CHAPTER 6. EXPONENTIAL AND LOGARITHMIC FUNCTIONS

Solution to Exercise 6.5.4 (p. 421)

Thedomain is (−4,∞) ,the range (−∞,∞) ,and the asymptotex = −− 4.

Available for free at Connexions <http://legacy.cnx.org/content/col11759/1.2>

559

Solution to Exercise 6.5.5 (p. 424)

Thedomain is (0,∞) ,the range is (−∞,∞) ,and the vertical asymptote isx = 0.

Available for free at Connexions <http://legacy.cnx.org/content/col11759/1.2>

560 CHAPTER 6. EXPONENTIAL AND LOGARITHMIC FUNCTIONS

Solution to Exercise 6.5.6 (p. 427)

Thedomain is (0,∞) ,the range is (−∞,∞) ,and the vertical asymptote isx = 0.

Available for free at Connexions <http://legacy.cnx.org/content/col11759/1.2>

561

Solution to Exercise 6.5.7 (p. 428)

Thedomain is (2,∞) ,the range is (−∞,∞) ,and the vertical asymptote isx = 2.

Available for free at Connexions <http://legacy.cnx.org/content/col11759/1.2>

562 CHAPTER 6. EXPONENTIAL AND LOGARITHMIC FUNCTIONS

Solution to Exercise 6.5.8 (p. 431)

Thedomain is (−∞, 0) ,the range is (−∞,∞) ,and the vertical asymptote isx = 0.Solution to Exercise 6.5.9 (p. 432)x ≈ 3.049Solution to Exercise 6.5.10 (p. 433)x = 1Solution to Exercise 6.5.11 (p. 435)f (x) = 2ln (x+ 3)− 1Solution to Exercise 6.5.12 (p. 438)Since the functions are inverses, their graphs are mirror images about the line y = x. So for everypoint (a, b) on the graph of a logarithmic function, there is a corresponding point (b, a) on the graph ofits inverse exponential function.Solution to Exercise 6.5.14 (p. 438)Shifting the function right or left and re�ecting the function about the y-axis will a�ect its domain.Solution to Exercise 6.5.16 (p. 438)No. A horizontal asymptote would suggest a limit on the range, and the range of any logarithmic functionin general form is all real numbers.Solution to Exercise 6.5.18 (p. 438)Domain:

(−∞, 1

2

); Range: (−∞,∞)

Solution to Exercise 6.5.20 (p. 438)Domain:

(− 17

4 ,∞)

; Range: (−∞,∞)Solution to Exercise 6.5.22 (p. 438)Domain: (5,∞) ; Vertical asymptote:x = 5Solution to Exercise 6.5.24 (p. 438)Domain:

(− 1

3 ,∞)

; Vertical asymptote:x = − 13

Solution to Exercise 6.5.26 (p. 438)Domain: (−3,∞) ; Vertical asymptote:x = −3Solution to Exercise 6.5.28 (p. 438)

Domain:(

37 ,∞

); Vertical asymptote:x = 3

7 ; End behavior: asx →(

37

)+, f (x) → −∞ and asx →

∞, f (x)→∞

Available for free at Connexions <http://legacy.cnx.org/content/col11759/1.2>

563

Solution to Exercise 6.5.30 (p. 439)Domain: (−3,∞) ; Vertical asymptote:x = −3; End behavior: asx → −3+, f (x) → −∞ and asx →∞, f (x)→∞Solution to Exercise 6.5.32 (p. 439)Domain: (1,∞) ; Range: (−∞,∞) ; Vertical asymptote:x = 1; x-intercept:

(54 , 0)

; y-intercept: DNESolution to Exercise 6.5.34 (p. 439)Domain: (−∞, 0) ; Range: (−∞,∞) ; Vertical asymptote:x = 0; x-intercept:

(−e2, 0

); y-intercept: DNE

Solution to Exercise 6.5.36 (p. 439)Domain: (0,∞) ; Range: (−∞,∞) ; Vertical asymptote: x = 0; x-intercept:

(e3, 0

); y-intercept: DNE

Solution to Exercise 6.5.38 (p. 440)BSolution to Exercise 6.5.40 (p. 440)CSolution to Exercise 6.5.42 (p. 441)BSolution to Exercise 6.5.44 (p. 441)CSolution to Exercise 6.5.46 (p. 442)

Solu-

Available for free at Connexions <http://legacy.cnx.org/content/col11759/1.2>

564 CHAPTER 6. EXPONENTIAL AND LOGARITHMIC FUNCTIONS

tion to Exercise 6.5.48 (p. 442)

Solu-tion to Exercise 6.5.50 (p. 442)CSolution to Exercise 6.5.52 (p. 443)

Available for free at Connexions <http://legacy.cnx.org/content/col11759/1.2>

565

Solution to Exercise 6.5.54 (p. 443)

Solution to Exercise 6.5.56 (p. 443)

Available for free at Connexions <http://legacy.cnx.org/content/col11759/1.2>

566 CHAPTER 6. EXPONENTIAL AND LOGARITHMIC FUNCTIONS

Solution to Exercise 6.5.58 (p. 443)f (x) = log2 (− (x− 1))Solution to Exercise 6.5.60 (p. 444)f (x) = 3log4 (x+ 2)Solution to Exercise 6.5.62 (p. 446)x = 2Solution to Exercise 6.5.64 (p. 446)x ≈ 2.303Solution to Exercise 6.5.66 (p. 446)x ≈ −0.472Solution to Exercise 6.5.68 (p. 447)The graphs of f (x) = log 1

2(x) and g (x) = −log2 (x) appear to be the same; Conjecture: for any positive

base b 6= 1, logb (x) = −log 1b

(x) .Solution to Exercise 6.5.70 (p. 447)Recall that the argument of a logarithmic function must be positive, so we determine where x+2

x−4 > 0 . Fromthe graph of the function f (x) = x+2

x−4 , note that the graph lies above the x-axis on the interval (−∞,−2) andagain to the right of the vertical asymptote, that is (4,∞) .Therefore, the domain is (−∞,−2) ∪ (4,∞) .

So-lution to Exercise 6.6.1 (p. 451)logb2 + logb2 + logb2 + logbk = 3logb2 + logbkSolution to Exercise 6.6.2 (p. 452)log3 (x+ 3)− log3 (x− 1)− log3 (x− 2)Solution to Exercise 6.6.3 (p. 453)2lnx

Available for free at Connexions <http://legacy.cnx.org/content/col11759/1.2>

567

Solution to Exercise 6.6.4 (p. 454)−2ln (x)Solution to Exercise 6.6.5 (p. 454)log316Solution to Exercise 6.6.6 (p. 455)2logx+ 3logy − 4logzSolution to Exercise 6.6.7 (p. 455)

23 lnxSolution to Exercise 6.6.8 (p. 456)

12 ln (x− 1) + ln (2x+ 1)− ln (x+ 3)− ln (x− 3)Solution to Exercise 6.6.9 (p. 457)log(

3·54·6)

; can also be written log(

58

)by reducing the fraction to lowest terms.

Solution to Exercise 6.6.10 (p. 458)

log(

5(x−1)3√x

(7x−1)

)Solution to Exercise 6.6.11 (p. 458)

logx12(x+5)4

(2x+3)4; this answer could also be written log

(x3(x+5)(2x+3)

)4

.

Solution to Exercise 6.6.12 (p. 458)The pH increases by about 0.301.Solution to Exercise 6.6.13 (p. 460)

ln8ln0.5Solution to Exercise 6.6.14 (p. 460)

ln100ln5 ≈

4.60511.6094 = 2.861

Solution to Exercise 6.6.15 (p. 461)Any root expression can be rewritten as an expression with a rational exponent so that the power rule can

be applied, making the logarithm easier to calculate. Thus, logb(x

1n

)= 1

n logb (x) .Solution to Exercise 6.6.17 (p. 461)logb (2) + logb (7) + logb (x) + logb (y)Solution to Exercise 6.6.19 (p. 462)logb (13)− logb (17)Solution to Exercise 6.6.21 (p. 462)−kln (4)Solution to Exercise 6.6.23 (p. 462)ln (7xy)Solution to Exercise 6.6.25 (p. 462)logb (4)Solution to Exercise 6.6.27 (p. 462)logb (7)Solution to Exercise 6.6.29 (p. 462)15log (x) + 13log (y)− 19log (z)Solution to Exercise 6.6.31 (p. 462)

32 log (x)− 2log (y)Solution to Exercise 6.6.33 (p. 462)

83 log (x) + 14

3 log (y)Solution to Exercise 6.6.35 (p. 462)ln(2x7)

Solution to Exercise 6.6.37 (p. 462)

log(xz3√y

)Solution to Exercise 6.6.39 (p. 463)

log7 (15) = ln(15)ln(7)

Available for free at Connexions <http://legacy.cnx.org/content/col11759/1.2>

568 CHAPTER 6. EXPONENTIAL AND LOGARITHMIC FUNCTIONS

Solution to Exercise 6.6.41 (p. 463)

log11 (5) = log5(5)log5(11) = 1

b

Solution to Exercise 6.6.43 (p. 463)

log11

(611

)=

log5( 611 )

log5(11) = log5(6)−log5(11)log5(11) = a−b

b = ab − 1

Solution to Exercise 6.6.45 (p. 463)3Solution to Exercise 6.6.47 (p. 463)2.81359Solution to Exercise 6.6.49 (p. 463)0.93913Solution to Exercise 6.6.51 (p. 463)−2.23266Solution to Exercise 6.6.53 (p. 464)

x = 4; By the quotient rule:log6 (x+ 2)− log6 (x− 3) = log6

(x+2x−3

)= 1.

Rewriting as an exponential equation and solving forx :

61 = x+2x−3

0 = x+2x−3 − 6

0 = x+2x−3 −

6(x−3)(x−3)

0 = x+2−6x+18x−3

0 = x−4x−3

x = 4

Checking, we �nd that log6 (4 + 2)− log6 (4− 3) = log6 (6)− log6 (1) is de�ned, sox = 4.Solution to Exercise 6.6.55 (p. 464)

Let b andn be positive integers greater than 1.Then, by the change-of-base formula, logb (n) = logn(n)logn(b) =

1logn(b) .

Solution to Exercise 6.7.1 (p. 466)x = −2Solution to Exercise 6.7.2 (p. 467)x = −1Solution to Exercise 6.7.3 (p. 468)x = 1

2Solution to Exercise 6.7.4 (p. 469)The equation has no solution.Solution to Exercise 6.7.5 (p. 470)x = ln3

ln(23)

Solution to Exercise 6.7.6 (p. 471)

t = 2ln(

113

)or ln

(113

)2Solution to Exercise 6.7.7 (p. 472)

t = ln(

1√2

)= − 1

2 ln (2)Solution to Exercise 6.7.8 (p. 472)x = ln2Solution to Exercise 6.7.9 (p. 473)x = e4

Solution to Exercise 6.7.10 (p. 474)x = e5 − 1

Available for free at Connexions <http://legacy.cnx.org/content/col11759/1.2>

569

Solution to Exercise 6.7.11 (p. 475)x ≈ 9.97Solution to Exercise 6.7.12 (p. 477)x = 1 orx = −1Solution to Exercise 6.7.13 (p. 478)

t = 703, 800, 000× ln(0.8)ln(0.5) years ≈ 226, 572, 993 years.

Solution to Exercise 6.7.14 (p. 480)Determine �rst if the equation can be rewritten so that each side uses the same base. If so, the exponentscan be set equal to each other. If the equation cannot be rewritten so that each side uses the same base,then apply the logarithm to each side and use properties of logarithms to solve.Solution to Exercise 6.7.16 (p. 480)The one-to-one property can be used if both sides of the equation can be rewritten as a single logarithmwith the same base. If so, the arguments can be set equal to each other, and the resulting equation canbe solved algebraically. The one-to-one property cannot be used when each side of the equation cannot berewritten as a single logarithm with the same base.Solution to Exercise 6.7.18 (p. 480)x = − 1

3Solution to Exercise 6.7.20 (p. 480)n = −1Solution to Exercise 6.7.22 (p. 480)b = 6

5Solution to Exercise 6.7.24 (p. 480)x = 10Solution to Exercise 6.7.26 (p. 480)No solutionSolution to Exercise 6.7.28 (p. 480)p = log

(178

)− 7

Solution to Exercise 6.7.30 (p. 480)

k = − ln(38)3

Solution to Exercise 6.7.32 (p. 480)

x =ln( 38

3 )−8

9Solution to Exercise 6.7.34 (p. 481)x = ln12Solution to Exercise 6.7.36 (p. 481)

x =ln( 3

5 )−3

8Solution to Exercise 6.7.38 (p. 481)no solutionSolution to Exercise 6.7.40 (p. 481)x = ln (3)Solution to Exercise 6.7.42 (p. 481)10−2 = 1

100Solution to Exercise 6.7.44 (p. 481)n = 49Solution to Exercise 6.7.46 (p. 481)k = 1

36Solution to Exercise 6.7.48 (p. 481)x = 9−e

8Solution to Exercise 6.7.50 (p. 481)n = 1

Available for free at Connexions <http://legacy.cnx.org/content/col11759/1.2>

570 CHAPTER 6. EXPONENTIAL AND LOGARITHMIC FUNCTIONS

Solution to Exercise 6.7.52 (p. 481)No solutionSolution to Exercise 6.7.54 (p. 482)No solutionSolution to Exercise 6.7.56 (p. 482)x = ± 10

3Solution to Exercise 6.7.58 (p. 482)x = 10Solution to Exercise 6.7.60 (p. 482)x = 0Solution to Exercise 6.7.62 (p. 482)x = 3

4Solution to Exercise 6.7.64 (p. 482)x = 9

So-lution to Exercise 6.7.66 (p. 482)

x = e2

3 ≈ 2.5

Available for free at Connexions <http://legacy.cnx.org/content/col11759/1.2>

571

So-lution to Exercise 6.7.68 (p. 482)x = −5

So-lution to Exercise 6.7.70 (p. 482)x = e+10

4 ≈ 3.2

Available for free at Connexions <http://legacy.cnx.org/content/col11759/1.2>

572 CHAPTER 6. EXPONENTIAL AND LOGARITHMIC FUNCTIONS

So-lution to Exercise 6.7.72 (p. 482)No solution

Available for free at Connexions <http://legacy.cnx.org/content/col11759/1.2>

573

So-lution to Exercise 6.7.74 (p. 483)x = 11

5 ≈ 2.2

Available for free at Connexions <http://legacy.cnx.org/content/col11759/1.2>

574 CHAPTER 6. EXPONENTIAL AND LOGARITHMIC FUNCTIONS

So-lution to Exercise 6.7.76 (p. 483)x = 101

11 ≈ 9.2

So-lution to Exercise 6.7.78 (p. 483)about $27, 710.24

Available for free at Connexions <http://legacy.cnx.org/content/col11759/1.2>

575

So-lution to Exercise 6.7.80 (p. 483)about 5 years

So-lution to Exercise 6.7.82 (p. 483)

ln(17)5 ≈ 0.567

Solution to Exercise 6.7.84 (p. 483)

x = log(38)+5log(3)4log(3) ≈ 2.078

Solution to Exercise 6.7.86 (p. 483)x ≈ 2.2401Solution to Exercise 6.7.88 (p. 483)x ≈ −44655.7143

Available for free at Connexions <http://legacy.cnx.org/content/col11759/1.2>

576 CHAPTER 6. EXPONENTIAL AND LOGARITHMIC FUNCTIONS

Solution to Exercise 6.7.90 (p. 484)about 5.83Solution to Exercise 6.7.92 (p. 484)

t = ln((

yA

) 1k

)Solution to Exercise 6.7.94 (p. 484)

t = ln((

T−Ts

T0−Ts

)− 1k

)Solution to Exercise 6.8.1 (p. 492)f (t) = A0e

−0.0000000087t

Solution to Exercise 6.8.2 (p. 493)less than 230 years, 229.3157 to be exactSolution to Exercise 6.8.3 (p. 494)

f (t) = A0eln23 t

Solution to Exercise 6.8.4 (p. 496)6.026 hoursSolution to Exercise 6.8.5 (p. 499)895 cases on day 15Solution to Exercise 6.8.6 (p. 504)Exponential. y = 2e0.5x.Solution to Exercise 6.8.7 (p. 505)y = 3e(ln0.5)x

Solution to Exercise 6.8.8 (p. 506)Half-life is a measure of decay and is thus associated with exponential decay models. The half-life of asubstance or quantity is the amount of time it takes for half of the initial amount of that substance orquantity to decay.Solution to Exercise 6.8.10 (p. 506)Doubling time is a measure of growth and is thus associated with exponential growth models. The doublingtime of a substance or quantity is the amount of time it takes for the initial amount of that substance orquantity to double in size.Solution to Exercise 6.8.12 (p. 506)An order of magnitude is the nearest power of ten by which a quantity exponentially grows. It is also anapproximate position on a logarithmic scale; Sample response: Orders of magnitude are useful when makingcomparisons between numbers that di�er by a great amount. For example, the mass of Saturn is 95 timesgreater than the mass of Earth. This is the same as saying that the mass of Saturn is about 102 times, or 2orders of magnitude greater, than the mass of Earth.Solution to Exercise 6.8.14 (p. 507)f (0) ≈ 16.7; The amount initially present is about 16.7 units.Solution to Exercise 6.8.16 (p. 507)150Solution to Exercise 6.8.19 (p. 507)exponential; f (x) = 1.2x

Solution to Exercise 6.8.21 (p. 507)logarithmic

Available for free at Connexions <http://legacy.cnx.org/content/col11759/1.2>

577

So-lution to Exercise 6.8.23 (p. 508)logarithmic

Available for free at Connexions <http://legacy.cnx.org/content/col11759/1.2>

578 CHAPTER 6. EXPONENTIAL AND LOGARITHMIC FUNCTIONS

So-

Available for free at Connexions <http://legacy.cnx.org/content/col11759/1.2>

579

lution to Exercise 6.8.25 (p. 508)

Solu-tion to Exercise 6.8.27 (p. 508)about 1.4 yearsSolution to Exercise 6.8.29 (p. 508)about 7.3 yearsSolution to Exercise 6.8.31 (p. 508)4 half-lives; 8.18minutesSolution to Exercise 6.8.33 (p. 508)

M = 23 log

(SS0

)log(SS0

)= 3

2M

SS0

= 103M2

S = S0103M2

Solution to Exercise 6.8.35 (p. 509)Let y = bx for some non-negative real number b such that b 6= 1.Then,

ln (y) = ln (bx)

ln (y) = xln (b)

eln(y) = exln(b)

y = exln(b)

Solution to Exercise 6.8.37 (p. 509)A = 125e(−0.3567t);A ≈ 43mgSolution to Exercise 6.8.39 (p. 509)about 60 daysSolution to Exercise 6.8.41 (p. 509)f (t) = 250e(−0.00914t); half-life: about 76minutesSolution to Exercise 6.8.43 (p. 509)r ≈ −0.0667, So the hourly decay rate is about 6.67%

Available for free at Connexions <http://legacy.cnx.org/content/col11759/1.2>

580 CHAPTER 6. EXPONENTIAL AND LOGARITHMIC FUNCTIONS

Solution to Exercise 6.8.45 (p. 509)f (t) = 1350e(0.03466t); after 3 hours:P (180) ≈ 691, 200Solution to Exercise 6.8.47 (p. 510)f (t) = 256e(0.068110t); doubling time: about 10minutesSolution to Exercise 6.8.49 (p. 510)about 88minutesSolution to Exercise 6.8.51 (p. 510)T (t) = 90e(−0.008377t) + 75, where t is in minutes.Solution to Exercise 6.8.53 (p. 510)about 113minutesSolution to Exercise 6.8.55 (p. 510)log (x) = 1.5; x ≈ 31.623Solution to Exercise 6.8.57 (p. 510)MMS magnitude: 5.82Solution to Exercise 6.8.59 (p. 511)N (3) ≈ 71Solution to Exercise 6.8.61 (p. 511)CSolution to Exercise 6.9.1 (p. 516)

a. The exponential regression model that �ts these data is y = 522.88585984(1.19645256)x.b. If spending continues at this rate, the graduate's credit card debt will be $4,499.38 after one year.

Solution to Exercise 6.9.2 (p. 519)

a. The logarithmic regression model that �ts these data is y = 141.91242949 + 10.45366573ln (x)b. If sales continue at this rate, about 171,000 games will be sold in the year 2015.

Solution to Exercise 6.9.3 (p. 523)

a. The logistic regression model that �ts these data is y = 25.656659791+6.113686306e−0.3852149008x .

b. If the population continues to grow at this rate, there will be about 25,634 seals in 2020.c. To the nearest whole number, the carrying capacity is 25,657.

Solution to Exercise 6.9.4 (p. 525)Logistic models are best used for situations that have limited values. For example, populations cannot growinde�nitely since resources such as food, water, and space are limited, so a logistic model best describespopulations.Solution to Exercise 6.9.6 (p. 525)Regression analysis is the process of �nding an equation that best �ts a given set of data points. To performa regression analysis on a graphing utility, �rst list the given points using the STAT then EDIT menu. Nextgraph the scatter plot using the STAT PLOT feature. The shape of the data points on the scatter graph canhelp determine which regression feature to use. Once this is determined, select the appropriate regressionanalysis command from the STAT then CALC menu.Solution to Exercise 6.9.8 (p. 525)The y-intercept on the graph of a logistic equation corresponds to the initial population for the populationmodel.Solution to Exercise 6.9.10 (p. 530)CSolution to Exercise 6.9.12 (p. 530)BSolution to Exercise 6.9.14 (p. 531)P (0) = 22 ; 175

Available for free at Connexions <http://legacy.cnx.org/content/col11759/1.2>

581

Solution to Exercise 6.9.16 (p. 531)p ≈ 2.67Solution to Exercise 6.9.18 (p. 531)y-intercept: (0, 15)Solution to Exercise 6.9.20 (p. 531)4 koiSolution to Exercise 6.9.22 (p. 531)about 6.8months.Solution to Exercise 6.9.23 (p. 531)

Solution to Exercise 6.9.25 (p. 531)10wolvesSolution to Exercise 6.9.27 (p. 531)about 5.4 years.

Available for free at Connexions <http://legacy.cnx.org/content/col11759/1.2>

582 CHAPTER 6. EXPONENTIAL AND LOGARITHMIC FUNCTIONS

Solution to Exercise 6.9.29 (p. 532)

Solu-tion to Exercise 6.9.31 (p. 532)f (x) = 776.682e0.3549x

Solution to Exercise 6.9.33 (p. 532)When f (x) = 4000,x ≈ 4.6.

So-lution to Exercise 6.9.35 (p. 532)f (x) = 731.92(0.738)x

Available for free at Connexions <http://legacy.cnx.org/content/col11759/1.2>

583

Solution to Exercise 6.9.37 (p. 532)

Solu-

Available for free at Connexions <http://legacy.cnx.org/content/col11759/1.2>

584 CHAPTER 6. EXPONENTIAL AND LOGARITHMIC FUNCTIONS

tion to Exercise 6.9.39 (p. 532)

Solu-tion to Exercise 6.9.41 (p. 532)f (10) ≈ 9.5Solution to Exercise 6.9.43 (p. 533)When f (x) = 7, x ≈ 2.7.

So-lution to Exercise 6.9.45 (p. 533)f (x) = 7.544− 2.268ln (x)

Available for free at Connexions <http://legacy.cnx.org/content/col11759/1.2>

585

Solution to Exercise 6.9.47 (p. 533)

Solu-

Available for free at Connexions <http://legacy.cnx.org/content/col11759/1.2>

586 CHAPTER 6. EXPONENTIAL AND LOGARITHMIC FUNCTIONS

tion to Exercise 6.9.49 (p. 533)

Solu-

Available for free at Connexions <http://legacy.cnx.org/content/col11759/1.2>

587

tion to Exercise 6.9.51 (p. 533)

Solu-tion to Exercise 6.9.53 (p. 533)When f (x) = 12.5, x ≈ 2.1.

Available for free at Connexions <http://legacy.cnx.org/content/col11759/1.2>

588 CHAPTER 6. EXPONENTIAL AND LOGARITHMIC FUNCTIONS

So-lution to Exercise 6.9.55 (p. 534)f (x) = 136.068

1+10.324e−0.480x

Solution to Exercise 6.9.57 (p. 534)about136Solution to Exercise 6.9.59 (p. 534)Working with the left side of the equation, we see that it can be rewritten as ae−bt :

c−P (t)P (t) =

c− c

1+ae−btc

1+ae−bt=

c(1+ae−bt)−c

1+ae−btc

1+ae−bt=

c(1+ae−bt−1)1+ae−bt

c

1+ae−bt= 1 + ae−bt − 1 = ae−bt

Working with the right side of the equation we show that it can also be rewritten as ae−bt.But �rst notethat when t = 0, P0 = c

1+ae−b(0) = c1+a .Therefore,

c−P0P0

e−bt =c− c

1+ac

1+ae−bt =

c(1+a)−c1+a

c1+a

e−bt =c(1+a−1)

1+ac

1+ae−bt = (1 + a− 1) e−bt = ae−bt

Thus, c−P (t)P (t) = c−P0

P0e−bt.

Solution to Exercise 6.9.61 (p. 534)First rewrite the exponential with base e: f (x) = 1.034341e0.247800x. Then test to verify that f (g (x)) =x,taking rounding error into consideration:

Available for free at Connexions <http://legacy.cnx.org/content/col11759/1.2>

589

g (f (x)) = 4.035510ln(1.034341e0.247800x

)− 0.136259

= 4.03551(ln (1.034341) + ln

(e0.2478x

))− 0.136259

= 4.03551 (ln (1.034341) + 0.2478x)− 0.136259

= 0.136257 + 0.999999x− 0.136259

= −0.000002 + 0.999999x

≈ 0 + x

= x

Solution to Exercise 6.9.63 (p. 534)

Thegraph ofP (t) has a y-intercept at (0, 4) and horizontal asymptotes at y = 0 and y = 20. The graphofP−1 (t) has an x- intercept at (4, 0) and vertical asymptotes at x = 0 and x = 20.Solution to Exercise 6.9.64 (p. 534)exponential decay; The growth factor, 0.825, is between 0 and 1.Solution to Exercise 6.9.66 (p. 534)y = 0.25(3)x

Solution to Exercise 6.9.68 (p. 535)$42, 888.18Solution to Exercise 6.9.70 (p. 535)continuous decay; the growth rate is negative.Solution to Exercise 6.9.72 (p. 535)domain: all real numbers; range: all real numbers strictly greater than zero; y-intercept: (0, 3.5);

Available for free at Connexions <http://legacy.cnx.org/content/col11759/1.2>

590 CHAPTER 6. EXPONENTIAL AND LOGARITHMIC FUNCTIONS

Solution to Exercise 6.9.74 (p. 535)g (x) = 7(6.5)−x; y-intercept: (0, 7) ; Domain: all real numbers; Range: all real numbers greater than 0.Solution to Exercise 6.9.76 (p. 536)17x = 4913Solution to Exercise 6.9.78 (p. 536)logab = − 2

5Solution to Exercise 6.9.80 (p. 536)

x = 6413 = 4

Solution to Exercise 6.9.82 (p. 536)log (0.000001) = −6Solution to Exercise 6.9.84 (p. 537)ln(e−0.8648

)= −0.8648

Solution to Exercise 6.9.86 (p. 537)

Available for free at Connexions <http://legacy.cnx.org/content/col11759/1.2>

591

Solution to Exercise 6.9.88 (p. 537)Domain:x > − 5; Vertical asymptote:x = −5; End behavior: asx → −5+, f (x) → −∞ and asx →∞, f (x)→∞.Solution to Exercise 6.9.90 (p. 537)log8 (65xy)Solution to Exercise 6.9.92 (p. 537)

ln(zxy

)Solution to Exercise 6.9.94 (p. 537)logy (12)Solution to Exercise 6.9.96 (p. 537)

ln (2) + ln (b) + ln(b+1)−ln(b−1)2

Solution to Exercise 6.9.98 (p. 537)

log7

(v3w6

3√u

)Solution to Exercise 6.9.100 (p. 537)

x =log(125)log(5) +17

12 = 53

Solution to Exercise 6.9.102 (p. 538)x = −3Solution to Exercise 6.9.104 (p. 538)no solutionSolution to Exercise 6.9.106 (p. 538)no solutionSolution to Exercise 6.9.108 (p. 538)x = ln (11)

Available for free at Connexions <http://legacy.cnx.org/content/col11759/1.2>

592 CHAPTER 6. EXPONENTIAL AND LOGARITHMIC FUNCTIONS

Solution to Exercise 6.9.110 (p. 538)a = e4 − 3Solution to Exercise 6.9.112 (p. 538)x = ± 9

5Solution to Exercise 6.9.114 (p. 538)about 5.45 yearsSolution to Exercise 6.9.116 (p. 538)f−1 (x) = 3

√24x − 1

Solution to Exercise 6.9.118 (p. 539)f (t) = 300(0.83)t; f (24) ≈ 3.43 gSolution to Exercise 6.9.120 (p. 539)about 45minutesSolution to Exercise 6.9.122 (p. 539)about 8.5 daysSolution to Exercise 6.9.124 (p. 539)exponential

Solution to Exercise 6.9.126 (p. 539)y = 4(0.2)x; y = 4e-1.609438x

Solution to Exercise 6.9.128 (p. 540)about 7.2 daysSolution to Exercise 6.9.130 (p. 540)logarithmic; y = 16.68718− 9.71860ln (x)

Available for free at Connexions <http://legacy.cnx.org/content/col11759/1.2>

593

Available for free at Connexions <http://legacy.cnx.org/content/col11759/1.2>

594 CHAPTER 6. EXPONENTIAL AND LOGARITHMIC FUNCTIONS

Solution to Exercise 6.9.132 (p. 540)About 13 dolphins.Solution to Exercise 6.9.134 (p. 540)$1, 947Solution to Exercise 6.9.136 (p. 541)y-intercept: (0, 5)

Solution to Exercise 6.9.138 (p. 541)8.5a = 614.125Solution to Exercise 6.9.140 (p. 542)

x =(

17

)2 = 149

Solution to Exercise 6.9.142 (p. 542)ln (0.716) ≈ −0.334Solution to Exercise 6.9.144 (p. 542)Domain:x < 3; Vertical asymptote:x = 3; End behavior:x→ 3−, f (x)→ −∞ andx→ −∞, f (x)→∞Solution to Exercise 6.9.146 (p. 542)logt (12)Solution to Exercise 6.9.148 (p. 542)

3 ln (y) + 2ln (z) + ln(x−4)3

Solution to Exercise 6.9.150 (p. 542)

x =ln(1000)ln(16) +5

3 ≈ 2.497Solution to Exercise 6.9.152 (p. 542)

a = ln(4)+810

Solution to Exercise 6.9.154 (p. 542)no solution

Available for free at Connexions <http://legacy.cnx.org/content/col11759/1.2>

595

Solution to Exercise 6.9.156 (p. 542)x = ln (9)Solution to Exercise 6.9.158 (p. 542)

x = ± 3√

32

Solution to Exercise 6.9.160 (p. 543)f (t) = 112e−.019792t; half-life: about 35 daysSolution to Exercise 6.9.162 (p. 543)T (t) = 36e−0.025131t + 35;T (60) ≈ 43oFSolution to Exercise 6.9.164 (p. 543)logarithmic

Available for free at Connexions <http://legacy.cnx.org/content/col11759/1.2>

596 CHAPTER 6. EXPONENTIAL AND LOGARITHMIC FUNCTIONS

Available for free at Connexions <http://legacy.cnx.org/content/col11759/1.2>

597

Solution to Exercise 6.9.166 (p. 543)exponential; y = 15.10062(1.24621)x

Solution to Exercise 6.9.168 (p. 544)logistic; y = 18.41659

1+7.54644e−0.68375x

Available for free at Connexions <http://legacy.cnx.org/content/col11759/1.2>

598 CHAPTER 6. EXPONENTIAL AND LOGARITHMIC FUNCTIONS

Available for free at Connexions <http://legacy.cnx.org/content/col11759/1.2>

Chapter 7

Systems of Equations and Inequalities

7.1 Introduction to Systems of Equations and Inequalities1

Figure 7.1: Enigma machines like this one, once owned by Italian dictator Benito Mussolini, were usedby government and military o�cials for enciphering and deciphering top-secret communications duringWorld War II. (credit: Dave Addey, Flickr)

1This content is available online at <http://legacy.cnx.org/content/m49418/1.4/>.

Available for free at Connexions <http://legacy.cnx.org/content/col11759/1.2>

599

600 CHAPTER 7. SYSTEMS OF EQUATIONS AND INEQUALITIES

By 1943, it was obvious to the Nazi regime that defeat was imminent unless it could build a weapon withunlimited destructive power, one that had never been seen before in the history of the world. In September,Adolf Hitler ordered German scientists to begin building an atomic bomb. Rumors and whispers beganto spread from across the ocean. Refugees and diplomats told of the experiments happening in Norway.However, Franklin D. Roosevelt wasn't sold, and even doubted British Prime Minister Winston Churchill'swarning. Roosevelt wanted undeniable proof. Fortunately, he soon received the proof he wanted when agroup of mathematicians cracked the �Enigma� code, proving beyond a doubt that Hitler was building anatomic bomb. The next day, Roosevelt gave the order that the United States begin work on the same.

The Enigma is perhaps the most famous cryptographic device ever known. It stands as an example ofthe pivotal role cryptography has played in society. Now, technology has moved cryptanalysis to the digitalworld.

Many ciphers are designed using invertible matrices as the method of message transference, as �nding theinverse of a matrix is generally part of the process of decoding. In addition to knowing the matrix and itsinverse, the receiver must also know the key that, when used with the matrix inverse, will allow the messageto be read.

In this chapter, we will investigate matrices and their inverses, and various ways to use matrices to solvesystems of equations. First, however, we will study systems of equations on their own: linear and nonlinear,and then partial fractions. We will not be breaking any secret codes here, but we will lay the foundation forfuture courses.

7.2 Systems of Linear Equations: Two Variables2

Figure 7.2: (credit: Thomas Sørenes)

2This content is available online at <http://legacy.cnx.org/content/m49420/1.4/>.

Available for free at Connexions <http://legacy.cnx.org/content/col11759/1.2>

601

A skateboard manufacturer introduces a new line of boards. The manufacturer tracks its costs, which is theamount it spends to produce the boards, and its revenue, which is the amount it earns through sales of itsboards. How can the company determine if it is making a pro�t with its new line? How many skateboardsmust be produced and sold before a pro�t is possible? In this section, we will consider linear equations withtwo variables to answer these and similar questions.

7.2.1 Introduction to Systems of Equations

In order to investigate situations such as that of the skateboard manufacturer, we need to recognize that weare dealing with more than one variable and likely more than one equation. A system of linear equationsconsists of two or more linear equations made up of two or more variables such that all equations in thesystem are considered simultaneously. To �nd the unique solution to a system of linear equations, we must�nd a numerical value for each variable in the system that will satisfy all equations in the system at the sametime. Some linear systems may not have a solution and others may have an in�nite number of solutions. Inorder for a linear system to have a unique solution, there must be at least as many equations as there arevariables. Even so, this does not guarantee a unique solution.

In this section, we will look at systems of linear equations in two variables, which consist of two equationsthat contain two di�erent variables. For example, consider the following system of linear equations in twovariables.

2x+ y = 15

3x−−y = 5(7.1)

The solution to a system of linear equations in two variables is any ordered pair that satis�es each equationindependently. In this example, the ordered pair (4, 7) is the solution to the system of linear equations. Wecan verify the solution by substituting the values into each equation to see if the ordered pair satis�es bothequations. Shortly we will investigate methods of �nding such a solution if it exists.

2 (4) + (7) = 15 True

3 (4)− (7) = 5 True(7.2)

In addition to considering the number of equations and variables, we can categorize systems of linearequations by the number of solutions. A consistent system of equations has at least one solution. Aconsistent system is considered to be an independent system if it has a single solution, such as theexample we just explored. The two lines have di�erent slopes and intersect at one point in the plane. Aconsistent system is considered to be a dependent system if the equations have the same slope and thesame y-intercepts. In other words, the lines coincide so the equations represent the same line. Every pointon the line represents a coordinate pair that satis�es the system. Thus, there are an in�nite number ofsolutions.

Another type of system of linear equations is an inconsistent system, which is one in which theequations represent two parallel lines. The lines have the same slope and di�erent y-intercepts. There areno points common to both lines; hence, there is no solution to the system.

A General Note: There are three types of systems of linear equations in two variables, andthree types of solutions.

• An independent system has exactly one solution pair (x, y) .The point where the two linesintersect is the only solution.• An inconsistent system has no solution. Notice that the two lines are parallel and will neverintersect.• A dependent system has in�nitely many solutions. The lines are coincident. They are thesame line, so every coordinate pair on the line is a solution to both equations.

Available for free at Connexions <http://legacy.cnx.org/content/col11759/1.2>

602 CHAPTER 7. SYSTEMS OF EQUATIONS AND INEQUALITIES

Figure 7.3 compares graphical representations of each type of system.

Figure 7.3

How To: Given a system of linear equations and an ordered pair, determine whetherthe ordered pair is a solution.

1.Substitute the ordered pair into each equation in the system.2.Determine whether true statements result from the substitution in both equations; if so, theordered pair is a solution.

Example 7.1Determining Whether an Ordered Pair Is a Solution to a System of EquationsDetermine whether the ordered pair (5, 1) is a solution to the given system of equations.

x+ 3y = 8

2x− 9 = y(7.3)

Solution

Substitute the ordered pair (5, 1) into both equations.

(5) + 3 (1) = 8

8 = 8 True

2 (5)− 9 = (1)

1=1 True

(7.4)

The ordered pair (5, 1) satis�es both equations, so it is the solution to the system.

Available for free at Connexions <http://legacy.cnx.org/content/col11759/1.2>

603

AnalysisWe can see the solution clearly by plotting the graph of each equation. Since the solution is anordered pair that satis�es both equations, it is a point on both of the lines and thus the point ofintersection of the two lines. See Figure 7.4.

Figure 7.4

Try It:

Exercise 7.2.1 (Solution on p. 781.)

Determine whether the ordered pair (8, 5) is a solution to the following system.

5x− 4y = 20

2x+ 1 = 3y(7.5)

Available for free at Connexions <http://legacy.cnx.org/content/col11759/1.2>

604 CHAPTER 7. SYSTEMS OF EQUATIONS AND INEQUALITIES

7.2.2 Solving Systems of Equations by Graphing

There are multiple methods of solving systems of linear equations. For a system of linear equationsin two variables, we can determine both the type of system and the solution by graphing the system ofequations on the same set of axes.

Example 7.2Solving a System of Equations in Two Variables by GraphingSolve the following system of equations by graphing. Identify the type of system.

2x+ y = −8

x− y = −1(7.6)

Solution

Solve the �rst equation for y.

2x+ y = −8

y = −2x− 8(7.7)

Solve the second equation for y.

x− y = −1

y = x+ 1(7.8)

Graph both equations on the same set of axes as in Figure 7.5.

Available for free at Connexions <http://legacy.cnx.org/content/col11759/1.2>

605

Figure 7.5

The lines appear to intersect at the point (−3,−2) .We can check to make sure that this is thesolution to the system by substituting the ordered pair into both equations.

2 (−3) + (−2) = −8

− 8 = −8 True

(−3)− (−2) = −1

− 1 = −1 True

(7.9)

The solution to the system is the ordered pair (−3,−2) ,so the system is independent.

Try It:

Exercise 7.2.2 (Solution on p. 781.)

Solve the following system of equations by graphing.

2x− 5y = −25

−4x+ 5y = 35(7.10)

Available for free at Connexions <http://legacy.cnx.org/content/col11759/1.2>

606 CHAPTER 7. SYSTEMS OF EQUATIONS AND INEQUALITIES

Q&A: Can graphing be used if the system is inconsistent or dependent?

Yes, in both cases we can still graph the system to determine the type of system and solution. If thetwo lines are parallel, the system has no solution and is inconsistent. If the two lines are identical,the system has in�nite solutions and is a dependent system.

7.2.3 Solving Systems of Equations by Substitution

Solving a linear system in two variables by graphing works well when the solution consists of integer values,but if our solution contains decimals or fractions, it is not the most precise method. We will considertwo more methods of solving a system of linear equations that are more precise than graphing. Onesuch method is solving a system of equations by the substitution method, in which we solve one of theequations for one variable and then substitute the result into the second equation to solve for the secondvariable. Recall that we can solve for only one variable at a time, which is the reason the substitution methodis both valuable and practical.

How To: Given a system of two equations in two variables, solve using the substitutionmethod.

1.Solve one of the two equations for one of the variables in terms of the other.2.Substitute the expression for this variable into the second equation, then solve for the remainingvariable.

3.Substitute that solution into either of the original equations to �nd the value of the �rstvariable. If possible, write the solution as an ordered pair.

4.Check the solution in both equations.

Example 7.3Solving a System of Equations in Two Variables by SubstitutionSolve the following system of equations by substitution.

− x+ y = −5

2x− 5y = 1(7.11)

Solution

First, we will solve the �rst equation for y.

−x+ y = −5

y = x− 5(7.12)

Now we can substitute the expressionx− 5 for y in the second equation.

2x− 5y = 1

2x− 5 (x− 5) = 1

2x− 5x+ 25 = 1

− 3x = −24

x = 8

(7.13)

Available for free at Connexions <http://legacy.cnx.org/content/col11759/1.2>

607

Now, we substitutex = 8 into the �rst equation and solve for y.

− (8) + y = −5

y = 3(7.14)

Our solution is (8, 3) .Check the solution by substituting (8, 3) into both equations.

− x+ y = −5

− (8) + (3) = −5 True

2x− 5y = 1

2 (8)− 5 (3) = 1 True

(7.15)

Try It:

Exercise 7.2.3 (Solution on p. 781.)

Solve the following system of equations by substitution.

x = y + 3

4 = 3x− 2y(7.16)

Q&A: Can the substitution method be used to solve any linear system in two variables?

Yes, but the method works best if one of the equations contains a coe�cient of 1 or �1 so that wedo not have to deal with fractions.

7.2.4 Solving Systems of Equations in Two Variables by the Addition Method

A third method of solving systems of linear equations is the addition method. In this method, weadd two terms with the same variable, but opposite coe�cients, so that the sum is zero. Of course, not allsystems are set up with the two terms of one variable having opposite coe�cients. Often we must adjustone or both of the equations by multiplication so that one variable will be eliminated by addition.

How To: Given a system of equations, solve using the addition method.

1.Write both equations with x- and y-variables on the left side of the equal sign and constantson the right.

2.Write one equation above the other, lining up corresponding variables. If one of the variablesin the top equation has the opposite coe�cient of the same variable in the bottom equation,add the equations together, eliminating one variable. If not, use multiplication by a nonzeronumber so that one of the variables in the top equation has the opposite coe�cient of the samevariable in the bottom equation, then add the equations to eliminate the variable.

3.Solve the resulting equation for the remaining variable.

Available for free at Connexions <http://legacy.cnx.org/content/col11759/1.2>

608 CHAPTER 7. SYSTEMS OF EQUATIONS AND INEQUALITIES

4.Substitute that value into one of the original equations and solve for the second variable.5.Check the solution by substituting the values into the other equation.

Example 7.4Solving a System by the Addition MethodSolve the given system of equations by addition.

x+ 2y = −1

−x+ y = 3(7.17)

Solution

Both equations are already set equal to a constant. Notice that the coe�cient ofx in the secondequation, �1, is the opposite of the coe�cient ofx in the �rst equation, 1. We can add the twoequations to eliminatexwithout needing to multiply by a constant.

x+ 2y = −1

−x+ y = 3

3y = 2(7.18)

Now that we have eliminatedx, we can solve the resulting equation for y.

3y = 2

y = 23

(7.19)

Then, we substitute this value for y into one of the original equations and solve forx.

− x+ y = 3

− x+ 23 = 3

− x = 3− 23

− x = 73

x = − 73

(7.20)

The solution to this system is(− 7

3 ,23

).

Check the solution in the �rst equation.

x+ 2y = −1(− 7

3

)+ 2

(23

)=

− 73 + 4

3 =

− 33 =

− 1 = −1 True

(7.21)

AnalysisWe gain an important perspective on systems of equations by looking at the graphical represen-tation. See Figure 7.6 to �nd that the equations intersect at the solution. We do not need to ask

Available for free at Connexions <http://legacy.cnx.org/content/col11759/1.2>

609

whether there may be a second solution because observing the graph con�rms that the system hasexactly one solution.

Figure 7.6

Example 7.5Using the Addition Method When Multiplication of One Equation Is RequiredSolve the given system of equations by the addition method.

3x+ 5y = −11

x− 2y = 11

(7.22)

Solution

Adding these equations as presented will not eliminate a variable. However, we see that the �rstequation has 3x in it and the second equation hasx. So if we multiply the second equation by−3, thex-terms will add to zero.

x− 2y = 11

−3 (x− 2y) = −3 (11) Multiply both sides by − 3.

− 3x+ 6y = −33 Use the distributive property.

(7.23)

Available for free at Connexions <http://legacy.cnx.org/content/col11759/1.2>

610 CHAPTER 7. SYSTEMS OF EQUATIONS AND INEQUALITIES

Now, let's add them.

3x+ 5y = −11

−3x+ 6y = −33_______________

11y = −44

y = −4

(7.24)

For the last step, we substitute y = −4 into one of the original equations and solve forx.

3x+ 5y = −11

3x+ 5 (−4) = −11

3x− 20 = −11

3x = 9

x = 3

(7.25)

Our solution is the ordered pair (3,−4) . See Figure 7.7. Check the solution in the original secondequation.

x− 2y = 11

(3)− 2 (−4) = 3 + 8

= 11 True

(7.26)

Available for free at Connexions <http://legacy.cnx.org/content/col11759/1.2>

611

Figure 7.7

Try It:

Exercise 7.2.4 (Solution on p. 781.)

Solve the system of equations by addition.

2x− 7y = 2

3x+ y = −20(7.27)

Example 7.6Using the Addition Method When Multiplication of Both Equations Is RequiredSolve the given system of equations in two variables by addition.

2x+ 3y = −16

5x− 10y = 30(7.28)

Available for free at Connexions <http://legacy.cnx.org/content/col11759/1.2>

612 CHAPTER 7. SYSTEMS OF EQUATIONS AND INEQUALITIES

Solution

One equation has 2x and the other has 5x.The least common multiple is 10x so we will have tomultiply both equations by a constant in order to eliminate one variable. Let's eliminatex bymultiplying the �rst equation by−5 and the second equation by 2.

− 5 (2x+ 3y) = −5 (−16)

− 10x− 15y = 80

2 (5x− 10y) = 2 (30)

10x− 20y = 60

(7.29)

Then, we add the two equations together.

−10x− 15y = 80

10x− 20y = 60________________

− 35y = 140

y = −4

(7.30)

Substitute y = −4 into the original �rst equation.

2x+ 3 (−4) = −16

2x− 12 = −16

2x = −4

x = −2

(7.31)

The solution is (−2,−4) .Check it in the other equation.

5x− 10y = 30

5 (−2)− 10 (−4) = 30

− 10 + 40 = 30

30 = 30

(7.32)

See Figure 7.8.

Available for free at Connexions <http://legacy.cnx.org/content/col11759/1.2>

613

Figure 7.8

Example 7.7Using the Addition Method in Systems of Equations Containing FractionsSolve the given system of equations in two variables by addition.

x3 + y

6 = 3x2 −

y4 = 1

(7.33)

Solution

First clear each equation of fractions by multiplying both sides of the equation by the least commondenominator.

Available for free at Connexions <http://legacy.cnx.org/content/col11759/1.2>

614 CHAPTER 7. SYSTEMS OF EQUATIONS AND INEQUALITIES

6(x3 + y

6

)= 6 (3)

2x+ y = 18

4(x2 −

y4

)= 4 (1)

2x− y = 4

(7.34)

Now multiply the second equation by − 1 so that we can eliminate the x-variable.

−1 (2x− y) = −1 (4)

− 2x+ y = −4(7.35)

Add the two equations to eliminate the x-variable and solve the resulting equation.

2x+ y = 18

−2x+ y = −4_____________

2y = 14

y = 7

(7.36)

Substitute y = 7 into the �rst equation.

2x+ (7) = 18

2x = 11

x = 112

= 7.5

(7.37)

The solution is(

112 , 7

).Check it in the other equation.

x2 −

y4 = 1

1122 −

74 = 1

114 −

74 = 144 = 1

(7.38)

Try It:

Exercise 7.2.5 (Solution on p. 781.)

Solve the system of equations by addition.

2x+ 3y = 8

3x+ 5y = 10(7.39)

Available for free at Connexions <http://legacy.cnx.org/content/col11759/1.2>

615

7.2.5 Identifying Inconsistent Systems of Equations Containing Two Variables

Now that we have several methods for solving systems of equations, we can use the methods to identifyinconsistent systems. Recall that an inconsistent system consists of parallel lines that have the sameslope but di�erent y-intercepts. They will never intersect. When searching for a solution to an inconsistentsystem, we will come up with a false statement, such as 12 = 0.

Example 7.8Solving an Inconsistent System of EquationsSolve the following system of equations.

x = 9− 2y

x+ 2y = 13(7.40)

Solution

We can approach this problem in two ways. Because one equation is already solved forx, the mostobvious step is to use substitution.

x+ 2y = 13

(9− 2y) + 2y = 13

9 + 0y = 13

9 = 13

(7.41)

Clearly, this statement is a contradiction because 9 6= 13.Therefore, the system has no solution.The second approach would be to �rst manipulate the equations so that they are both in slope-

intercept form. We manipulate the �rst equation as follows.

x = 9− 2y

2y = −x+ 9

y = − 12x+ 9

2

(7.42)

We then convert the second equation expressed to slope-intercept form.

x+ 2y = 13

2y = −x+ 13

y = − 12x+ 13

2

(7.43)

Comparing the equations, we see that they have the same slope but di�erent y-intercepts. There-fore, the lines are parallel and do not intersect.

y = − 12x+ 9

2

y = − 12x+ 13

2

(7.44)

AnalysisWriting the equations in slope-intercept form con�rms that the system is inconsistent because alllines will intersect eventually unless they are parallel. Parallel lines will never intersect; thus, the

Available for free at Connexions <http://legacy.cnx.org/content/col11759/1.2>

616 CHAPTER 7. SYSTEMS OF EQUATIONS AND INEQUALITIES

two lines have no points in common. The graphs of the equations in this example are shown inFigure 7.9.

Figure 7.9

Try It:

Exercise 7.2.6 (Solution on p. 781.)

Solve the following system of equations in two variables.

2y − 2x = 2

2y − 2x = 6(7.45)

7.2.6 Expressing the Solution of a System of Dependent Equations Containing

Two Variables

Recall that a dependent system of equations in two variables is a system in which the two equationsrepresent the same line. Dependent systems have an in�nite number of solutions because all of the points

Available for free at Connexions <http://legacy.cnx.org/content/col11759/1.2>

617

on one line are also on the other line. After using substitution or addition, the resulting equation will be anidentity, such as 0 = 0.

Example 7.9Finding a Solution to a Dependent System of Linear EquationsFind a solution to the system of equations using the addition method.

x+ 3y = 2

3x+ 9y = 6(7.46)

Solution

With the addition method, we want to eliminate one of the variables by adding the equations. Inthis case, let's focus on eliminatingx. If we multiply both sides of the �rst equation by − 3, thenwe will be able to eliminate thex-variable.

x+ 3y = 2

(−3) (x+ 3y) = (−3) (2)

− 3x− 9y = −6

(7.47)

Now add the equations.

− 3x− 9y = −6

+ 3x+ 9y = 6______________

0 = 0

(7.48)

We can see that there will be an in�nite number of solutions that satisfy both equations.

AnalysisIf we rewrote both equations in the slope-intercept form, we might know what the solution wouldlook like before adding. Let's look at what happens when we convert the system to slope-interceptform.

x+ 3y = 2

3y = −x+ 2

y = − 13x+ 2

3

3x+ 9y = 6

9y = −3x+ 6

y = − 39x+ 6

9

y = − 13x+ 2

3

(7.49)

See Figure 7.10. Notice the results are the same. The general solution to the systemis(x, − 1

3x+ 23

).

Available for free at Connexions <http://legacy.cnx.org/content/col11759/1.2>

618 CHAPTER 7. SYSTEMS OF EQUATIONS AND INEQUALITIES

Figure 7.10

Try It:

Exercise 7.2.7 (Solution on p. 781.)

Solve the following system of equations in two variables.

y − 2x = 5

−3y + 6x = −15

(7.50)

Available for free at Connexions <http://legacy.cnx.org/content/col11759/1.2>

619

7.2.7 Using Systems of Equations to Investigate Pro�ts

Using what we have learned about systems of equations, we can return to the skateboard manufacturingproblem at the beginning of the section. The skateboard manufacturer's revenue function is the func-tion used to calculate the amount of money that comes into the business. It can be represented by theequationR = xp, wherex =quantity and p =price. The revenue function is shown in orange in Figure 7.11.

The cost function is the function used to calculate the costs of doing business. It includes �xed costs,such as rent and salaries, and variable costs, such as utilities. The cost function is shown in blue in Figure 7.11.Thex-axis represents quantity in hundreds of units. The y-axis represents either cost or revenue in hundredsof dollars.

Figure 7.11

The point at which the two lines intersect is called the break-even point. We can see from the graphthat if 700 units are produced, the cost is $3,300 and the revenue is also $3,300. In other words, the companybreaks even if they produce and sell 700 units. They neither make money nor lose money.

The shaded region to the right of the break-even point represents quantities for which the company makesa pro�t. The shaded region to the left represents quantities for which the company su�ers a loss. The pro�tfunction is the revenue function minus the cost function, written asP (x) = R (x)−C (x) .Clearly, knowingthe quantity for which the cost equals the revenue is of great importance to businesses.

Available for free at Connexions <http://legacy.cnx.org/content/col11759/1.2>

620 CHAPTER 7. SYSTEMS OF EQUATIONS AND INEQUALITIES

Example 7.10Finding the Break-Even Point and the Pro�t Function Using SubstitutionGiven the cost functionC (x) = 0.85x + 35, 000 and the revenue functionR (x) = 1.55x,�nd thebreak-even point and the pro�t function.

SolutionWrite the system of equations using y to replace function notation.

y = 0.85x+ 35, 000

y = 1.55x

(7.51)

Substitute the expression 0.85x+35, 000 from the �rst equation into the second equation and solveforx.

0.85x+ 35, 000 = 1.55x

35, 000 = 0.7x

50, 000 = x

(7.52)

Then, we substitute x = 50, 000 into either the cost function or the revenue function.

1.55 (50, 000) = 77, 500 (7.53)

The break-even point is (50, 000, 77, 500) .The pro�t function is found using the formulaP (x) = R (x)− C (x) .

P (x) = 1.55x− (0.85x+ 35, 000)

= 0.7x− 35, 000(7.54)

The pro�t function isP (x) = 0.7x− 35, 000.

AnalysisThe cost to produce 50,000 units is $77,500, and the revenue from the sales of 50,000 units isalso $77,500. To make a pro�t, the business must produce and sell more than 50,000 units. SeeFigure 7.12.

Available for free at Connexions <http://legacy.cnx.org/content/col11759/1.2>

621

Figure 7.12

We see from the graph in Figure 7.13 that the pro�t function has a negative value until x =50, 000, when the graph crosses the x-axis. Then, the graph emerges into positive y-values andcontinues on this path as the pro�t function is a straight line. This illustrates that the break-evenpoint for businesses occurs when the pro�t function is 0. The area to the left of the break-evenpoint represents operating at a loss.

Available for free at Connexions <http://legacy.cnx.org/content/col11759/1.2>

622 CHAPTER 7. SYSTEMS OF EQUATIONS AND INEQUALITIES

Figure 7.13

Example 7.11Writing and Solving a System of Equations in Two VariablesThe cost of a ticket to the circus is $25.00 for children and $50.00 for adults. On a certain day,attendance at the circus is 2, 000 and the total gate revenue is $70, 000.How many children and howmany adults bought tickets?

SolutionLet c = the number of children and a = the number of adults in attendance.

The total number of people is 2, 000.We can use this to write an equation for the number ofpeople at the circus that day.

c+ a = 2, 000 (7.55)

The revenue from all children can be found by multiplying $25.00 by the number of children, 25c.Therevenue from all adults can be found by multiplying $50.00 by the number of adults, 50a.The totalrevenue is$70, 000.We can use this to write an equation for the revenue.

25c+ 50a = 70, 000 (7.56)

Available for free at Connexions <http://legacy.cnx.org/content/col11759/1.2>

623

We now have a system of linear equations in two variables.

c+ a = 2, 000

25c+ 50a = 70, 000(7.57)

In the �rst equation, the coe�cient of both variables is 1. We can quickly solve the �rst equationfor either c or a.We will solve for a.

c+ a = 2, 000

a = 2, 000− c(7.58)

Substitute the expression 2, 000− c in the second equation for a and solve for c.

25c+ 50 (2, 000− c) = 70, 000

25c+ 100, 000− 50c = 70, 000

− 25c = −30, 000

c = 1, 200

(7.59)

Substitute c = 1, 200 into the �rst equation to solve for a.

1, 200 + a = 2, 000

a = 800(7.60)

We �nd that 1, 200 children and 800 adults bought tickets to the circus that day.

Try It:

Exercise 7.2.8 (Solution on p. 781.)

Meal tickets at the circus cost $4.00 for children and $12.00 for adults. If 1, 650meal ticketswere bought for a total of $14, 200, how many children and how many adults bought mealtickets?

Media: Access these online resources for additional instruction and practice with systems of linearequations.

• Solving Systems of Equations Using Substitution3• Solving Systems of Equations Using Elimination4• Applications of Systems of Equations5

3http://openstaxcollege.org/l/syssubst4http://openstaxcollege.org/l/syselim5http://openstaxcollege.org/l/sysapp

Available for free at Connexions <http://legacy.cnx.org/content/col11759/1.2>

624 CHAPTER 7. SYSTEMS OF EQUATIONS AND INEQUALITIES

7.2.8 Key Concepts

• A system of linear equations consists of two or more equations made up of two or more variables suchthat all equations in the system are considered simultaneously.

• The solution to a system of linear equations in two variables is any ordered pair that satis�es eachequation independently. See Example 7.1.

• Systems of equations are classi�ed as independent with one solution, dependent with an in�nite numberof solutions, or inconsistent with no solution.

• One method of solving a system of linear equations in two variables is by graphing. In this method,we graph the equations on the same set of axes. See Example 7.2.

• Another method of solving a system of linear equations is by substitution. In this method, we solvefor one variable in one equation and substitute the result into the second equation. See Example 7.3.

• A third method of solving a system of linear equations is by addition, in which we can eliminate avariable by adding opposite coe�cients of corresponding variables. See Example 7.4.

• It is often necessary to multiply one or both equations by a constant to facilitate elimination of avariable when adding the two equations together. See Example 7.5, Example 7.6, and Example 7.7.

• Either method of solving a system of equations results in a false statement for inconsistent systemsbecause they are made up of parallel lines that never intersect. See Example 7.8.

• The solution to a system of dependent equations will always be true because both equations describethe same line. See Example 7.9.

• Systems of equations can be used to solve real-world problems that involve more than one variable,such as those relating to revenue, cost, and pro�t. See Example 7.10 and Example 7.11.

7.2.9 Section Exercises

7.2.9.1 Verbal

Exercise 7.2.9 (Solution on p. 781.)

Can a system of linear equations have exactly two solutions? Explain why or why not.

Exercise 7.2.10If you are performing a break-even analysis for a business and their cost and revenue equations aredependent, explain what this means for the company's pro�t margins.

Exercise 7.2.11 (Solution on p. 781.)

If you are solving a break-even analysis and get a negative break-even point, explain what thissigni�es for the company?

Exercise 7.2.12If you are solving a break-even analysis and there is no break-even point, explain what this meansfor the company. How should they ensure there is a break-even point?

Exercise 7.2.13 (Solution on p. 782.)

Given a system of equations, explain at least two di�erent methods of solving that system.

7.2.9.2 Algebraic

For the following exercises, determine whether the given ordered pair is a solution to the system of equations.

Exercise 7.2.14

5x− y = 4

x+ 6y = 2and (4, 0)

Available for free at Connexions <http://legacy.cnx.org/content/col11759/1.2>

625

Exercise 7.2.15 (Solution on p. 782.)

−3x− 5y = 13

− x+ 4y = 10and(−6, 1)

Exercise 7.2.16

3x+ 7y = 1

2x+ 4y = 0and (2, 3)

Exercise 7.2.17 (Solution on p. 782.)

−2x+ 5y = 7

2x+ 9y = 7and(−1, 1)

Exercise 7.2.18

x+ 8y = 43

3x− 2y = −1and (3, 5)

For the following exercises, solve each system by substitution.

Exercise 7.2.19 (Solution on p. 782.)

x+ 3y = 5

2x+ 3y = 4

Exercise 7.2.20

3x− 2y = 18

5x+ 10y = −10

Exercise 7.2.21 (Solution on p. 782.)

4x+ 2y = −10

3x+ 9y = 0

Exercise 7.2.22

2x+ 4y = −3.8

9x− 5y = 1.3

Exercise 7.2.23 (Solution on p. 782.)

−2x+ 3y = 1.2

−3x− 6y = 1.8

Exercise 7.2.24

x− 0.2y = 1

−10x+ 2y = 5

Exercise 7.2.25 (Solution on p. 782.)

3x+ 5y = 9

30x+ 50y = −90

Exercise 7.2.26

− 3x+ y = 2

12x− 4y = −8

Available for free at Connexions <http://legacy.cnx.org/content/col11759/1.2>

626 CHAPTER 7. SYSTEMS OF EQUATIONS AND INEQUALITIES

Exercise 7.2.27 (Solution on p. 782.)12x+ 1

3y = 1616x+ 1

4y = 9

Exercise 7.2.28

− 14x+ 3

2y = 11

− 18x+ 1

3y = 3

For the following exercises, solve each system by addition.

Exercise 7.2.29 (Solution on p. 782.)

−2x+ 5y = −42

7x+ 2y = 30

Exercise 7.2.30

6x− 5y = −34

2x+ 6y = 4

Exercise 7.2.31 (Solution on p. 782.)

5x− y = −2.6

−4x− 6y = 1.4

Exercise 7.2.32

7x− 2y = 3

4x+ 5y = 3.25

Exercise 7.2.33 (Solution on p. 782.)

−x + 2y = −1

5x− 10y = 6

Exercise 7.2.34

7x+ 6y = 2

−28x− 24y = −8

Exercise 7.2.35 (Solution on p. 782.)56x+ 1

4y = 018x−

12y = − 43

120

Exercise 7.2.3613x+ 1

9y = 29

− 12x+ 4

5y = − 13

Exercise 7.2.37 (Solution on p. 782.)

−0.2x+ 0.4y = 0.6

x− 2y = −3

Available for free at Connexions <http://legacy.cnx.org/content/col11759/1.2>

627

Exercise 7.2.38

−0.1x+ 0.2y = 0.6

5x− 10y = 1

For the following exercises, solve each system by any method.

Exercise 7.2.39 (Solution on p. 782.)

5x+ 9y = 16

x+ 2y = 4

Exercise 7.2.40

6x− 8y = −0.6

3x+ 2y = 0.9

Exercise 7.2.41 (Solution on p. 782.)

5x− 2y = 2.25

7x− 4y = 3

Exercise 7.2.42

x− 512y = − 55

12

−6x+ 52y = 55

2

Exercise 7.2.43 (Solution on p. 782.)

7x− 4y = 76

2x+ 4y = 13

Exercise 7.2.44

3x+ 6y = 11

2x+ 4y = 9

Exercise 7.2.45 (Solution on p. 782.)73x−

16y = 2

− 216 x+ 3

12y = −3

Exercise 7.2.4612x+ 1

3y = 13

32x+ 1

4y = − 18

Exercise 7.2.47 (Solution on p. 782.)

2.2x+ 1.3y = −0.1

4.2x+ 4.2y = 2.1

Exercise 7.2.48

0.1x+ 0.2y = 2

0.35x− 0.3y = 0

Available for free at Connexions <http://legacy.cnx.org/content/col11759/1.2>

628 CHAPTER 7. SYSTEMS OF EQUATIONS AND INEQUALITIES

7.2.9.3 Graphical

For the following exercises, graph the system of equations and state whether the system is consistent,inconsistent, or dependent and whether the system has one solution, no solution, or in�nite solutions.

Exercise 7.2.49 (Solution on p. 782.)

3x− y = 0.6

x− 2y = 1.3

Exercise 7.2.50

−x+ 2y = 4

2x− 4y = 1

Exercise 7.2.51 (Solution on p. 782.)

x+ 2y = 7

2x+ 6y = 12

Exercise 7.2.52

3x− 5y = 7

x− 2y = 3

Exercise 7.2.53 (Solution on p. 782.)

3x− 2y = 5

−9x+ 6y = −15

7.2.9.4 Technology

For the following exercises, use the intersect function on a graphing device to solve each system. Round allanswers to the nearest hundredth.

Exercise 7.2.54

0.1x+ 0.2y = 0.3

−0.3x+ 0.5y = 1

Exercise 7.2.55 (Solution on p. 782.)

−0.01x+ 0.12y = 0.62

0.15x+ 0.20y = 0.52

Exercise 7.2.56

0.5x+ 0.3y = 4

0.25x− 0.9y = 0.46

Exercise 7.2.57 (Solution on p. 782.)

0.15x+ 0.27y = 0.39

−0.34x+ 0.56y = 1.8

Exercise 7.2.58

−0.71x+ 0.92y = 0.13

0.83x+ 0.05y = 2.1

Available for free at Connexions <http://legacy.cnx.org/content/col11759/1.2>

629

7.2.9.5 Extensions

For the following exercises, solve each system in terms ofA,B,C,D,E, andF whereA − −F are nonzeronumbers. Note thatA 6= B andAE 6= BD.

Exercise 7.2.59 (Solution on p. 782.)

x+ y = A

x− y = B

Exercise 7.2.60

x+Ay = 1

x+By = 1

Exercise 7.2.61 (Solution on p. 782.)

Ax+ y = 0

Bx+ y = 1

Exercise 7.2.62

Ax+By = C

x+ y = 1

Exercise 7.2.63 (Solution on p. 783.)

Ax+By = C

Dx+ Ey = F

7.2.9.6 Real-World Applications

For the following exercises, solve for the desired quantity.

Exercise 7.2.64A stu�ed animal business has a total cost of productionC = 12x+ 30 and a revenue functionR =20x.Find the break-even point.

Exercise 7.2.65 (Solution on p. 783.)

A fast-food restaurant has a cost of productionC (x) = 11x + 120 and a revenue functionR (x) =5x.When does the company start to turn a pro�t?

Exercise 7.2.66A cell phone factory has a cost of productionC (x) = 150x+ 10, 000 and a revenue functionR (x) =200x.What is the break-even point?

Exercise 7.2.67 (Solution on p. 783.)

A musician chargesC (x) = 64x+ 20, 000, where x is the total number of attendees at the concert.The venue charges $80 per ticket. After how many people buy tickets does the venue break even,and what is the value of the total tickets sold at that point?

Exercise 7.2.68A guitar factory has a cost of productionC (x) = 75x+50, 000. If the company needs to break evenafter 150 units sold, at what price should they sell each guitar? Round up to the nearest dollar,and write the revenue function.

For the following exercises, use a system of linear equations with two variables and two equations to solve.

Exercise 7.2.69 (Solution on p. 783.)

Find two numbers whose sum is 28 and di�erence is 13.

Available for free at Connexions <http://legacy.cnx.org/content/col11759/1.2>

630 CHAPTER 7. SYSTEMS OF EQUATIONS AND INEQUALITIES

Exercise 7.2.70A number is 9 more than another number. Twice the sum of the two numbers is 10. Find the twonumbers.

Exercise 7.2.71 (Solution on p. 783.)

The startup cost for a restaurant is $120,000, and each meal costs $10 for the restaurant to make.If each meal is then sold for $15, after how many meals does the restaurant break even?

Exercise 7.2.72A moving company charges a �at rate of $150, and an additional $5 for each box. If a taxi servicewould charge $20 for each box, how many boxes would you need for it to be cheaper to use themoving company, and what would be the total cost?

Exercise 7.2.73 (Solution on p. 783.)

A total of 1,595 �rst- and second-year college students gathered at a pep rally. The number offreshmen exceeded the number of sophomores by 15. How many freshmen and sophomores were inattendance?

Exercise 7.2.74276 students enrolled in a freshman-level chemistry class. By the end of the semester, 5 times thenumber of students passed as failed. Find the number of students who passed, and the number ofstudents who failed.

Exercise 7.2.75 (Solution on p. 783.)

There were 130 faculty at a conference. If there were 18 more women than men attending, howmany of each gender attended the conference?

Exercise 7.2.76A jeep and BMW enter a highway running east-west at the same exit heading in opposite directions.The jeep entered the highway 30 minutes before the BMW did, and traveled 7 mph slower thanthe BMW. After 2 hours from the time the BMW entered the highway, the cars were 306.5 milesapart. Find the speed of each car, assuming they were driven on cruise control.

Exercise 7.2.77 (Solution on p. 783.)

If a scientist mixed 10% saline solution with 60% saline solution to get 25 gallons of 40% salinesolution, how many gallons of 10% and 60% solutions were mixed?

Exercise 7.2.78An investor earned triple the pro�ts of what she earned last year. If she made $500,000.48 total forboth years, how much did she earn in pro�ts each year?

Exercise 7.2.79 (Solution on p. 783.)

An investor who dabbles in real estate invested 1.1 million dollars into two land investments. Onthe �rst investment, Swan Peak, her return was a 110% increase on the money she invested. On thesecond investment, Riverside Community, she earned 50% over what she invested. If she earned $1million in pro�ts, how much did she invest in each of the land deals?

Exercise 7.2.80If an investor invests a total of $25,000 into two bonds, one that pays 3% simple interest, and theother that pays 2 7

8% interest, and the investor earns $737.50 annual interest, how much was investedin each account?

Exercise 7.2.81 (Solution on p. 783.)

If an investor invests $23,000 into two bonds, one that pays 4% in simple interest, and the otherpaying 2% simple interest, and the investor earns $710.00 annual interest, how much was investedin each account?

Exercise 7.2.82CDs cost $5.96 more than DVDs at All Bets Are O� Electronics. How much would 6 CDs and 2DVDs cost if 5 CDs and 2 DVDs cost $127.73?

Available for free at Connexions <http://legacy.cnx.org/content/col11759/1.2>

631

Exercise 7.2.83 (Solution on p. 783.)

A store clerk sold 60 pairs of sneakers. The high-tops sold for $98.99 and the low-tops sold for$129.99. If the receipts for the two types of sales totaled $6,404.40, how many of each type ofsneaker were sold?

Exercise 7.2.84A concert manager counted 350 ticket receipts the day after a concert. The price for a studentticket was $12.50, and the price for an adult ticket was $16.00. The register con�rms that $5,075was taken in. How many student tickets and adult tickets were sold?

Exercise 7.2.85 (Solution on p. 783.)

Admission into an amusement park for 4 children and 2 adults is $116.90. For 6 children and 3adults, the admission is $175.35. Assuming a di�erent price for children and adults, what is theprice of the child's ticket and the price of the adult ticket?

Available for free at Connexions <http://legacy.cnx.org/content/col11759/1.2>

632 CHAPTER 7. SYSTEMS OF EQUATIONS AND INEQUALITIES

7.3 Systems of Linear Equations: Three Variables6

Figure 7.14: (credit: �Elembis,� Wikimedia Commons)

John received an inheritance of $12,000 that he divided into three parts and invested in three ways: in amoney-market fund paying 3% annual interest; in municipal bonds paying 4% annual interest; and in mutualfunds paying 7% annual interest. John invested $4,000 more in municipal funds than in municipal bonds.He earned $670 in interest the �rst year. How much did John invest in each type of fund?

Understanding the correct approach to setting up problems such as this one makes �nding a solution amatter of following a pattern. We will solve this and similar problems involving three equations and threevariables in this section. Doing so uses similar techniques as those used to solve systems of two equations intwo variables. However, �nding solutions to systems of three equations requires a bit more organization anda touch of visual gymnastics.

6This content is available online at <http://legacy.cnx.org/content/m49419/1.4/>.

Available for free at Connexions <http://legacy.cnx.org/content/col11759/1.2>

633

7.3.1 Solving Systems of Three Equations in Three Variables

In order to solve systems of equations in three variables, known as three-by-three systems, the primary toolwe will be using is called Gaussian elimination, named after the proli�c German mathematician KarlFriedrich Gauss. While there is no de�nitive order in which operations are to be performed, there arespeci�c guidelines as to what type of moves can be made. We may number the equations to keep track ofthe steps we apply. The goal is to eliminate one variable at a time to achieve upper triangular form,the ideal form for a three-by-three system because it allows for straightforward back-substitution to �nda solution (x, y, z) , which we call an ordered triple. A system in upper triangular form looks like thefollowing:

Ax+By + Cz = D

Ey + Fz = G

Hz = K

(7.61)

The third equation can be solved for z, and then we back-substitute to �nd y andx.To write the system inupper triangular form, we can perform the following operations:

1. Interchange the order of any two equations.2. Multiply both sides of an equation by a nonzero constant.3. Add a nonzero multiple of one equation to another equation.

The solution set to a three-by-three system is an ordered triple {(x, y, z)}.Graphically, the ordered triplede�nes the point that is the intersection of three planes in space. You can visualize such an intersection byimagining any corner in a rectangular room. A corner is de�ned by three planes: two adjoining walls and the�oor (or ceiling). Any point where two walls and the �oor meet represents the intersection of three planes.

A General Note: Figure 7.15 and Figure 7.16 illustrate possible solution scenarios for three-by-three systems.

• Systems that have a single solution are those which, after elimination, result in a solution setconsisting of an ordered triple {(x, y, z)}.Graphically, the ordered triple de�nes a point that isthe intersection of three planes in space.• Systems that have an in�nite number of solutions are those which, after elimination, result inan expression that is always true, such as 0 = 0.Graphically, an in�nite number of solutionsrepresents a line or coincident plane that serves as the intersection of three planes in space.• Systems that have no solution are those that, after elimination, result in a statement that isa contradiction, such as 3 = 0.Graphically, a system with no solution is represented by threeplanes with no point in common.

Available for free at Connexions <http://legacy.cnx.org/content/col11759/1.2>

634 CHAPTER 7. SYSTEMS OF EQUATIONS AND INEQUALITIES

Figure 7.15: (a)Three planes intersect at a single point, representing a three-by-three system with asingle solution. (b) Three planes intersect in a line, representing a three-by-three system with in�nitesolutions.

Figure 7.16: All three �gures represent three-by-three systems with no solution. (a) The three planesintersect with each other, but not at a common point. (b) Two of the planes are parallel and intersectwith the third plane, but not with each other. (c) All three planes are parallel, so there is no point ofintersection.

Example 7.12Determining Whether an Ordered Triple Is a Solution to a SystemDetermine whether the ordered triple (3,−2, 1) is a solution to the system.

Available for free at Connexions <http://legacy.cnx.org/content/col11759/1.2>

635

x+ y + z = 2

6x− 4y + 5z = 31

5x+ 2y + 2z = 13

(7.62)

Solution

We will check each equation by substituting in the values of the ordered triple forx, y, and z.

x+ y + z = 2

(3) + (−2) + (1) = 2

True

6x− 4y + 5z = 31

6 (3)− 4 (−2) + 5 (1) = 31

18 + 8 + 5 = 31

True

5x+ 2y + 2z = 13

5 (3) + 2 (−2) + 2 (1) = 13

15− 4 + 2 = 13

True

Table 7.1

The ordered triple (3,−2, 1) is indeed a solution to the system.

How To: Given a linear system of three equations, solve for three unknowns.

1.Pick any pair of equations and solve for one variable.2.Pick another pair of equations and solve for the same variable.3.You have created a system of two equations in two unknowns. Solve the resulting two-by-twosystem.

4.Back-substitute known variables into any one of the original equations and solve for the missingvariable.

Example 7.13Solving a System of Three Equations in Three Variables by EliminationFind a solution to the following system:

x− 2y + 3z = 9 (1)

− x+ 3y − z = −6 (2)

2x− 5y + 5z = 17 (3)

(7.63)

Solution

There will always be several choices as to where to begin, but the most obvious �rst step here isto eliminatex by adding equations (1) and (2).

x− 2y + 3z = 9 (1)

− x+ 3y − z = −6 (2)

y + 2z = 3 (3)(7.64)

The second step is multiplying equation (1) by − 2 and adding the result to equation (3). Thesetwo steps will eliminate the variablex.

Available for free at Connexions <http://legacy.cnx.org/content/col11759/1.2>

636 CHAPTER 7. SYSTEMS OF EQUATIONS AND INEQUALITIES

−2x+ 4y − 6z = −18 (1) multiplied by − 2

2x− 5y + 5z = 17 (3)____________________________________

− y − z = −1 (5)

(7.65)

In equations (4) and (5), we have created a new two-by-two system. We can solve for z by addingthe two equations.

y + 2z = 3 (4)

−y − z = −1 (5)

z = 2 (6)(7.66)

Choosing one equation from each new system, we obtain the upper triangular form:

x− 2y + 3z = 9 (1)

y + 2z = 3 (4)

z = 2 (6)

(7.67)

Next, we back-substitute z = 2 into equation (4) and solve for y.

y + 2 (2) = 3

y + 4 = 3

y = −1

(7.68)

Finally, we can back-substitute z = 2 and y = −1 into equation (1). This will yield the solutionforx.

x− 2 (−1) + 3 (2) = 9

x+ 2 + 6 = 9

x = 1

(7.69)

The solution is the ordered triple (1,−1, 2) . See Figure 7.17.

Available for free at Connexions <http://legacy.cnx.org/content/col11759/1.2>

637

Figure 7.17

Example 7.14Solving a Real-World Problem Using a System of Three Equations in Three VariablesIn the problem posed at the beginning of the section, John invested his inheritance of $12,000 inthree di�erent funds: part in a money-market fund paying 3% interest annually; part in municipalbonds paying 4% annually; and the rest in mutual funds paying 7% annually. John invested $4,000more in mutual funds than he invested in municipal bonds. The total interest earned in one yearwas $670. How much did he invest in each type of fund?

SolutionTo solve this problem, we use all of the information given and set up three equations. First, weassign a variable to each of the three investment amounts:

x = amount invested in money-market fund

y = amount invested in municipal bonds

z = amount invested in mutual funds

(7.70)

The �rst equation indicates that the sum of the three principal amounts is $12,000.

x+ y + z = 12, 000 (7.71)

Available for free at Connexions <http://legacy.cnx.org/content/col11759/1.2>

638 CHAPTER 7. SYSTEMS OF EQUATIONS AND INEQUALITIES

We form the second equation according to the information that John invested $4,000 more inmutual funds than he invested in municipal bonds.

z = y + 4, 000 (7.72)

The third equation shows that the total amount of interest earned from each fund equals $670.

0.03x+ 0.04y + 0.07z = 670 (7.73)

Then, we write the three equations as a system.

x+ y + z = 12, 000

− y + z = 4, 000

0.03x+ 0.04y + 0.07z = 670

(7.74)

To make the calculations simpler, we can multiply the third equation by 100. Thus,

x+ y + z = 12, 000 (1)

− y + z = 4, 000 (2)

3x+ 4y + 7z = 67, 000 (3)

(7.75)

Step 1. Interchange equation (2) and equation (3) so that the two equations with three variableswill line up.

x+ y + z = 12, 000

3x+ 4y + 7z = 67, 000

− y + z = 4, 000

(7.76)

Step 2. Multiply equation (1) by − 3 and add to equation (2). Write the result as row 2.

x+ y + z = 12, 000

y + 4z = 31, 000

− y + z = 4, 000

(7.77)

Step 3. Add equation (2) to equation (3) and write the result as equation (3).

x+ y + z = 12, 000

y + 4z = 31, 000

5z = 35, 000

(7.78)

Step 4. Solve for z in equation (3). Back-substitute that value in equation (2) and solve for y.Then,back-substitute the values for z and y into equation (1) and solve forx.

Available for free at Connexions <http://legacy.cnx.org/content/col11759/1.2>

639

5z = 35, 000

z = 7, 000

y + 4 (7, 000) = 31, 000

y = 3, 000

x+ 3, 000 + 7, 000 = 12, 000

x = 2, 000

(7.79)

John invested $2,000 in a money-market fund, $3,000 in municipal bonds, and $7,000 in mutualfunds.

Try It:

Exercise 7.3.1 (Solution on p. 783.)

Solve the system of equations in three variables.

2x+ y − 2z = −1

3x− 3y − z = 5

x− 2y + 3z = 6

(7.80)

7.3.2 Identifying Inconsistent Systems of Equations Containing Three Variables

Just as with systems of equations in two variables, we may come across an inconsistent system of equationsin three variables, which means that it does not have a solution that satis�es all three equations. Theequations could represent three parallel planes, two parallel planes and one intersecting plane, or threeplanes that intersect the other two but not at the same location. The process of elimination will result in afalse statement, such as 3 = 7 or some other contradiction.

Example 7.15Solving an Inconsistent System of Three Equations in Three VariablesSolve the following system.

x− 3y + z = 4 (1)

− x+ 2y − 5z = 3 (2)

5x− 13y + 13z = 8 (3)

(7.81)

Solution

Available for free at Connexions <http://legacy.cnx.org/content/col11759/1.2>

640 CHAPTER 7. SYSTEMS OF EQUATIONS AND INEQUALITIES

Looking at the coe�cients ofx, we can see that we can eliminate x by adding equation (1) toequation (2).

x− 3y + z = 4 (1)

−x+ 2y − 5z = 3 (2)

− y − 4z = 7 (4)(7.82)

Next, we multiply equation (1) by − 5 and add it to equation (3).

−5x+ 15y − 5z = −20 (1) multiplied by − 5

5x− 13y + 13z = 8 (3)______________________________________

2y + 8z = −12 (5)

(7.83)

Then, we multiply equation (4) by 2 and add it to equation (5).

−2y − 8z = 14 (4) multiplied by 2

2y + 8z = −12 (5)_______________________________________

0 = 2

(7.84)

The �nal equation 0 = 2 is a contradiction, so we conclude that the system of equations in incon-sistent and, therefore, has no solution.

AnalysisIn this system, each plane intersects the other two, but not at the same location. Therefore, thesystem is inconsistent.

Try It:

Exercise 7.3.2 (Solution on p. 783.)

Solve the system of three equations in three variables.

x+ y + z = 2

y − 3z = 1

2x+ y + 5z = 0

(7.85)

7.3.3 Expressing the Solution of a System of Dependent Equations Containing

Three Variables

We know from working with systems of equations in two variables that a dependent system of equationshas an in�nite number of solutions. The same is true for dependent systems of equations in three variables.An in�nite number of solutions can result from several situations. The three planes could be the same, sothat a solution to one equation will be the solution to the other two equations. All three equations could

Available for free at Connexions <http://legacy.cnx.org/content/col11759/1.2>

641

be di�erent but they intersect on a line, which has in�nite solutions. Or two of the equations could be thesame and intersect the third on a line.

Example 7.16Finding the Solution to a Dependent System of EquationsFind the solution to the given system of three equations in three variables.

2x+ y − 3z = 0 (1)

4x+ 2y − 6z = 0 (2)

x− y + z = 0 (3)

(7.86)

Solution

First, we can multiply equation (1) by − 2 and add it to equation (2).

−4x− 2y + 6z = 0 equation (1) multiplied by − 2

4x+ 2y − 6z = 0 (2)____________________________________________

0 = 0

(7.87)

We do not need to proceed any further. The result we get is an identity, 0 = 0,which tells us thatthis system has an in�nite number of solutions. There are other ways to begin to solve this system,such as multiplying equation (3) by − 2,and adding it to equation (1). We then perform the samesteps as above and �nd the same result, 0 = 0.

When a system is dependent, we can �nd general expressions for the solutions. Adding equations(1) and (3), we have

2x+ y − 3z = 0

x− y + z = 0_____________

3x− 2z = 0

(7.88)

We then solve the resulting equation for z.

3x− 2z = 0

z = 32x

(7.89)

We back-substitute the expression for z into one of the equations and solve for y.

2x+ y − 3(

32x)

= 0

2x+ y − 92x = 0

y = 92x− 2x

y = 52x

(7.90)

So the general solution is(x, 5

2x,32x). In this solution,x can be any real number. The values

of y and z are dependent on the value selected forx.

Available for free at Connexions <http://legacy.cnx.org/content/col11759/1.2>

642 CHAPTER 7. SYSTEMS OF EQUATIONS AND INEQUALITIES

AnalysisAs shown in Figure 7.18, two of the planes are the same and they intersect the third plane on a line.The solution set is in�nite, as all points along the intersection line will satisfy all three equations.

Figure 7.18

Q&A: Does the generic solution to a dependent system always have to be written interms of x?

No, you can write the generic solution in terms of any of the variables, but it is common to writeit in terms of x and if neededx and y.

Try It:

Exercise 7.3.3 (Solution on p. 783.)

Solve the following system.

x+ y + z = 7

3x− 2y − z = 4

x+ 6y + 5z = 24

(7.91)

Available for free at Connexions <http://legacy.cnx.org/content/col11759/1.2>

643

Media: Access these online resources for additional instruction and practice with systems ofequations in three variables.

• Ex 1: System of Three Equations with Three Unknowns Using Elimination7

• Ex. 2: System of Three Equations with Three Unknowns Using Elimination8

7.3.4 Key Concepts

• A solution set is an ordered triple {(x, y, z)} that represents the intersection of three planes in space.SeeExample 7.12.

• A system of three equations in three variables can be solved by using a series of steps that forces avariable to be eliminated. The steps include interchanging the order of equations, multiplying bothsides of an equation by a nonzero constant, and adding a nonzero multiple of one equation to anotherequation. See Example 7.13.

• Systems of three equations in three variables are useful for solving many di�erent types of real-worldproblems. See Example 7.14.

• A system of equations in three variables is inconsistent if no solution exists. After performing elimina-tion operations, the result is a contradiction. See Example 7.15.

• Systems of equations in three variables that are inconsistent could result from three parallel planes,two parallel planes and one intersecting plane, or three planes that intersect the other two but not atthe same location.

• A system of equations in three variables is dependent if it has an in�nite number of solutions. Afterperforming elimination operations, the result is an identity. See Example 7.16.

• Systems of equations in three variables that are dependent could result from three identical planes,three planes intersecting at a line, or two identical planes that intersect the third on a line.

7.3.5 Section Exercises

7.3.5.1 Verbal

Exercise 7.3.4 (Solution on p. 783.)

Can a linear system of three equations have exactly two solutions? Explain why or why not

Exercise 7.3.5If a given ordered triple solves the system of equations, is that solution unique? If so, explain why.If not, give an example where it is not unique.

Exercise 7.3.6 (Solution on p. 783.)

If a given ordered triple does not solve the system of equations, is there no solution? If so, explainwhy. If not, give an example.

Exercise 7.3.7Using the method of addition, is there only one way to solve the system?

Exercise 7.3.8 (Solution on p. 783.)

Can you explain whether there can be only one method to solve a linear system of equations? Ifyes, give an example of such a system of equations. If not, explain why not.

7http://openstaxcollege.org/l/systhree8http://openstaxcollege.org/l/systhelim

Available for free at Connexions <http://legacy.cnx.org/content/col11759/1.2>

644 CHAPTER 7. SYSTEMS OF EQUATIONS AND INEQUALITIES

7.3.5.2 Algebraic

For the following exercises, determine whether the ordered triple given is the solution to the system ofequations.

Exercise 7.3.9

2x− 6y + 6z = −12

x+ 4y + 5z = −1

−x + 2y + 3z = −1

and (0, 1,−1)

Exercise 7.3.10 (Solution on p. 783.)

6x− y + 3z = 6

3x+ 5y + 2z = 0

x+ y = 0

and(3,−3,−5)

Exercise 7.3.11

6x− 7y + z = 2

−x− y + 3z = 4

2x+ y − z = 1

and (4, 2,−6)

Exercise 7.3.12 (Solution on p. 783.)

x− y = 0

x− z = 5

x− y + z = −1

and (4, 4,−1)

Exercise 7.3.13

−x− y + 2z = 3

5x+ 8y − 3z = 4

−x + 3y − 5z = −5

and (4, 1,−7)

For the following exercises, solve each system by substitution.

Exercise 7.3.14 (Solution on p. 783.)

3x− 4y + 2z = −15

2x+ 4y + z = 16

2x+ 3y + 5z = 20

Exercise 7.3.15

5x− 2y + 3z = 20

2x− 4y − 3z = −9

x+ 6y − 8z = 21

Exercise 7.3.16 (Solution on p. 783.)

5x+ 2y + 4z = 9

−3x+ 2y + z = 10

4x− 3y + 5z = −3

Available for free at Connexions <http://legacy.cnx.org/content/col11759/1.2>

645

Exercise 7.3.17

4x− 3y + 5z = 31

−x+ 2y + 4z = 20

x+ 5y − 2z = −29

Exercise 7.3.18 (Solution on p. 784.)

5x− 2y + 3z = 4

−4x+ 6y − 7z = −1

3x+ 2y − z = 4

Exercise 7.3.19

4x+ 6y + 9z = 0

−5x+ 2y − 6z = 3

7x− 4y + 3z = −3

For the following exercises, solve each system by Gaussian elimination.

Exercise 7.3.20 (Solution on p. 784.)

2x− y + 3z = 17

−5x+ 4y − 2z = −46

2y + 5z = −7

Exercise 7.3.21

5x− 6y + 3z = 50

− x+ 4y = 10

2x− z = 10

Exercise 7.3.22 (Solution on p. 784.)

2x+ 3y − 6z = 1

−4x− 6y + 12z = −2

x+ 2y + 5z = 10

Exercise 7.3.23

4x+ 6y − 2z = 8

6x+ 9y − 3z = 12

−2x− 3y + z = −4

Exercise 7.3.24 (Solution on p. 784.)

2x+ 3y − 4z = 5

−3x+ 2y + z = 11

−x+ 5y + 3z = 4

Exercise 7.3.25

10x+ 2y − 14z = 8

−x− 2y − 4z = −1

−12x− 6y + 6z = −12

Available for free at Connexions <http://legacy.cnx.org/content/col11759/1.2>

646 CHAPTER 7. SYSTEMS OF EQUATIONS AND INEQUALITIES

Exercise 7.3.26 (Solution on p. 784.)

x+ y + z = 14

2y + 3z = −14

−16y − 24z = −112

Exercise 7.3.27

5x− 3y + 4z = −1

−4x+ 2y − 3z = 0

−x + 5y + 7z = −11

Exercise 7.3.28 (Solution on p. 784.)

x+ y + z = 0

2x− y + 3z = 0

x− z = 0

Exercise 7.3.29

3x+ 2y − 5z = 6

5x− 4y + 3z = −12

4x+ 5y − 2z = 15

Exercise 7.3.30 (Solution on p. 784.)

x+ y + z = 0

2x− y + 3z = 0

x− z = 1

Exercise 7.3.31

3x− 12y − z = − 1

2

4x+ z = 3

− x+ 32y = 5

2

Exercise 7.3.32 (Solution on p. 784.)

6x− 5y + 6z = 3815x−

12y + 3

5z = 1

− 4x− 32y − z = −74

Exercise 7.3.3312x−

15y + 2

5z = − 1310

14x−

25y −

15z = − 7

20

− 12x−

34y −

12z = − 5

4

Exercise 7.3.34 (Solution on p. 784.)

− 13x−

12y −

14z = 3

4

− 12x−

14y −

12z = 2

− 14x−

34y −

12z = − 1

2

Available for free at Connexions <http://legacy.cnx.org/content/col11759/1.2>

647

Exercise 7.3.3512x−

14y + 3

4z = 014x−

110y + 2

5z = −218x+ 1

5y −18z = 2

Exercise 7.3.36 (Solution on p. 784.)45x−

78y + 1

2z = 1

− 45x−

34y + 1

3z = −8

− 25x−

78y + 1

2z = −5

Exercise 7.3.37

− 13x−

18y + 1

6z = − 43

− 23x−

78y + 1

3z = − 233

− 13x−

58y + 5

6z = 0

Exercise 7.3.38 (Solution on p. 784.)

− 14x−

54y + 5

2z = −5

− 12x−

53y + 5

4z = 5512

− 13x−

13y + 1

3z = 53

Exercise 7.3.39140x+ 1

60y + 180z = 1

100

− 12x−

13y −

14z = − 1

5

38x+ 3

12y + 316z = 3

20

Exercise 7.3.40 (Solution on p. 784.)

0.1x− 0.2y + 0.3z = 2

0.5x− 0.1y + 0.4z = 8

0.7x− 0.2y + 0.3z = 8

Exercise 7.3.41

0.2x+ 0.1y − 0.3z = 0.2

0.8x+ 0.4y − 1.2z = 0.1

1.6x+ 0.8y − 2.4z = 0.2

Exercise 7.3.42 (Solution on p. 784.)

1.1x+ 0.7y − 3.1z = −1.79

2.1x+ 0.5y − 1.6z = −0.13

0.5x+ 0.4y − 0.5z = −0.07

Exercise 7.3.43

0.5x− 0.5y + 0.5z = 10

0.2x− 0.2y + 0.2z = 4

0.1x− 0.1y + 0.1z = 2

Available for free at Connexions <http://legacy.cnx.org/content/col11759/1.2>

648 CHAPTER 7. SYSTEMS OF EQUATIONS AND INEQUALITIES

Exercise 7.3.44 (Solution on p. 784.)

0.1x+ 0.2y + 0.3z = 0.37

0.1x− 0.2y − 0.3z = −0.27

0.5x− 0.1y − 0.3z = −0.03

Exercise 7.3.45

0.5x− 0.5y − 0.3z = 0.13

0.4x− 0.1y − 0.3z = 0.11

0.2x− 0.8y − 0.9z = −0.32

Exercise 7.3.46 (Solution on p. 784.)

0.5x+ 0.2y − 0.3z = 1

0.4x− 0.6y + 0.7z = 0.8

0.3x− 0.1y − 0.9z = 0.6

Exercise 7.3.47

0.3x+ 0.3y + 0.5z = 0.6

0.4x+ 0.4y + 0.4z = 1.8

0.4x+ 0.2y + 0.1z = 1.6

Exercise 7.3.48 (Solution on p. 784.)

0.8x+ 0.8y + 0.8z = 2.4

0.3x− 0.5y + 0.2z = 0

0.1x+ 0.2y + 0.3z = 0.6

7.3.5.3 Extensions

For the following exercises, solve the system forx, y, and z.

Exercise 7.3.49

x+ y + z = 3x−1

2 + y−32 + z+1

2 = 0x−2

3 + y+43 + z−3

3 = 23

Exercise 7.3.50 (Solution on p. 784.)

5x− 3y − z+12 = 1

2

6x+ y−92 + 2z = −3

x+82 − 4y + z = 4

Exercise 7.3.51x+4

7 −y−1

6 + z+23 = 1

x−24 + y+1

8 −z+812 = 0

x+63 −

y+23 + z+4

2 = 3

Exercise 7.3.52 (Solution on p. 784.)x−3

6 + y+22 −

z−33 = 2

x+24 + y−5

2 + z+42 = 1

x+62 −

y−32 + z + 1 = 9

Available for free at Connexions <http://legacy.cnx.org/content/col11759/1.2>

649

Exercise 7.3.53x−1

3 + y+34 + z+2

6 = 1

4x+ 3y − 2z = 11

0.02x+ 0.015y − 0.01z = 0.065

7.3.5.4 Real-World Applications

Exercise 7.3.54 (Solution on p. 784.)

Three even numbers sum up to 108. The smaller is half the larger and the middle number is 34 the

larger. What are the three numbers?

Exercise 7.3.55Three numbers sum up to 147. The smallest number is half the middle number, which is half thelargest number. What are the three numbers?

Exercise 7.3.56 (Solution on p. 784.)

At a family reunion, there were only blood relatives, consisting of children, parents, and grand-parents, in attendance. There were 400 people total. There were twice as many parents as grand-parents, and 50 more children than parents. How many children, parents, and grandparents werein attendance?

Exercise 7.3.57An animal shelter has a total of 350 animals comprised of cats, dogs, and rabbits. If the numberof rabbits is 5 less than one-half the number of cats, and there are 20 more cats than dogs, howmany of each animal are at the shelter?

Exercise 7.3.58 (Solution on p. 784.)

Your roommate, Sarah, o�ered to buy groceries for you and your other roommate. The total billwas $82. She forgot to save the individual receipts but remembered that your groceries were $0.05cheaper than half of her groceries, and that your other roommate's groceries were $2.10 more thanyour groceries. How much was each of your share of the groceries?

Exercise 7.3.59Your roommate, John, o�ered to buy household supplies for you and your other roommate. Youlive near the border of three states, each of which has a di�erent sales tax. The total amount ofmoney spent was $100.75. Your supplies were bought with 5% tax, John's with 8% tax, and yourthird roommate's with 9% sales tax. The total amount of money spent without taxes is $93.50.If your supplies before tax were $1 more than half of what your third roommate's supplies werebefore tax, how much did each of you spend? Give your answer both with and without taxes.

Exercise 7.3.60 (Solution on p. 784.)

Three coworkers work for the same employer. Their jobs are warehouse manager, o�ce manager,and truck driver. The sum of the annual salaries of the warehouse manager and o�ce manager is$82,000. The o�ce manager makes $4,000 more than the truck driver annually. The annual salariesof the warehouse manager and the truck driver total $78,000. What is the annual salary of each ofthe co-workers?

Exercise 7.3.61At a carnival, $2,914.25 in receipts were taken at the end of the day. The cost of a child's ticketwas $20.50, an adult ticket was $29.75, and a senior citizen ticket was $15.25. There were twice asmany senior citizens as adults in attendance, and 20 more children than senior citizens. How manychildren, adult, and senior citizen tickets were sold?

Available for free at Connexions <http://legacy.cnx.org/content/col11759/1.2>

650 CHAPTER 7. SYSTEMS OF EQUATIONS AND INEQUALITIES

Exercise 7.3.62 (Solution on p. 784.)

A local band sells out for their concert. They sell all 1,175 tickets for a total purse of $28,112.50.The tickets were priced at $20 for student tickets, $22.50 for children, and $29 for adult tickets. Ifthe band sold twice as many adult as children tickets, how many of each type was sold?

Exercise 7.3.63In a bag, a child has 325 coins worth $19.50. There were three types of coins: pennies, nickels,and dimes. If the bag contained the same number of nickels as dimes, how many of each type ofcoin was in the bag?

Exercise 7.3.64 (Solution on p. 784.)

Last year, at Haven's Pond Car Dealership, for a particular model of BMW, Jeep, and Toyota,one could purchase all three cars for a total of $140,000. This year, due to in�ation, the same carswould cost $151,830. The cost of the BMW increased by 8%, the Jeep by 5%, and the Toyota by12%. If the price of last year's Jeep was $7,000 less than the price of last year's BMW, what wasthe price of each of the three cars last year?

Exercise 7.3.65A recent college graduate took advantage of his business education and invested in three investmentsimmediately after graduating. He invested $80,500 into three accounts, one that paid 4% simpleinterest, one that paid 3 1

8% simple interest, and one that paid 2 12%simple interest. He earned

$2,670 interest at the end of one year. If the amount of the money invested in the second accountwas four times the amount invested in the third account, how much was invested in each account?

Exercise 7.3.66 (Solution on p. 784.)

You inherit one million dollars. You invest it all in three accounts for one year. The �rst accountpays 3% compounded annually, the second account pays 4% compounded annually, and the thirdaccount pays 2% compounded annually. After one year, you earn $34,000 in interest. If you investfour times the money into the account that pays 3% compared to 2%, how much did you invest ineach account?

Exercise 7.3.67You inherit one hundred thousand dollars. You invest it all in three accounts for one year. The�rst account pays 4% compounded annually, the second account pays 3% compounded annually,and the third account pays 2% compounded annually. After one year, you earn $3,650 in interest.If you invest �ve times the money in the account that pays 4% compared to 3%, how much did youinvest in each account?

Exercise 7.3.68 (Solution on p. 785.)

The top three countries in oil consumption in a certain year are as follows: the United States, Japan,and China. In millions of barrels per day, the three top countries consumed 39.8% of the world'sconsumed oil. The United States consumed 0.7% more than four times China's consumption. TheUnited States consumed 5% more than triple Japan's consumption. What percent of the world oilconsumption did the United States, Japan, and China consume?9

Exercise 7.3.69The top three countries in oil production in the same year are Saudi Arabia, the United States,and Russia. In millions of barrels per day, the top three countries produced 31.4% of the world'sproduced oil. Saudi Arabia and the United States combined for 22.1% of the world's production,and Saudi Arabia produced 2% more oil than Russia. What percent of the world oil productiondid Saudi Arabia, the United States, and Russia produce?10

Exercise 7.3.70 (Solution on p. 785.)

The top three sources of oil imports for the United States in the same year were Saudi Arabia,Mexico, and Canada. The three top countries accounted for 47% of oil imports. The United States

9�Oil reserves, production and consumption in 2001,� accessed April 6, 2014, http://scaru�.com/politics/oil.html.10�Oil reserves, production and consumption in 2001,� accessed April 6, 2014, http://scaru�.com/politics/oil.html.

Available for free at Connexions <http://legacy.cnx.org/content/col11759/1.2>

651

imported 1.8% more from Saudi Arabia than they did from Mexico, and 1.7% more from SaudiArabia than they did from Canada. What percent of the United States oil imports were from thesethree countries?11

Exercise 7.3.71The top three oil producers in the United States in a certain year are the Gulf of Mexico, Texas,and Alaska. The three regions were responsible for 64% of the United States oil production. TheGulf of Mexico and Texas combined for 47% of oil production. Texas produced 3% more thanAlaska. What percent of United States oil production came from these regions?12

Exercise 7.3.72 (Solution on p. 785.)

At one time, in the United States, 398 species of animals were on the endangered species list. Thetop groups were mammals, birds, and �sh, which comprised 55% of the endangered species. Birdsaccounted for 0.7% more than �sh, and �sh accounted for 1.5% more than mammals. What percentof the endangered species came from mammals, birds, and �sh?

Exercise 7.3.73Meat consumption in the United States can be broken into three categories: red meat, poultry, and�sh. If �sh makes up 4% less than one-quarter of poultry consumption, and red meat consumptionis 18.2% higher than poultry consumption, what are the percentages of meat consumption?13

7.4 Systems of Nonlinear Equations and Inequalities: Two Variables14

Halley's Comet (Figure 7.19) orbits the sun about once every 75 years. Its path can be considered to be avery elongated ellipse. Other comets follow similar paths in space. These orbital paths can be studied usingsystems of equations. These systems, however, are di�erent from the ones we considered in the previoussection because the equations are not linear.

11�Oil reserves, production and consumption in 2001,� accessed April 6, 2014, http://scaru�.com/politics/oil.html.12�USA: The coming global oil crisis,� accessed April 6, 2014, http://www.oilcrisis.com/us/.13�The United States Meat Industry at a Glance,� accessed April 6, 2014, http://www.meatami.com/ht/d/sp/i/47465/pid/47465.14This content is available online at <http://legacy.cnx.org/content/m49431/1.5/>.

Available for free at Connexions <http://legacy.cnx.org/content/col11759/1.2>

652 CHAPTER 7. SYSTEMS OF EQUATIONS AND INEQUALITIES

Figure 7.19: Halley's Comet (credit: "NASA Blueshift"/Flickr)

In this section, we will consider the intersection of a parabola and a line, a circle and a line, and acircle and an ellipse. The methods for solving systems of nonlinear equations are similar to those for linearequations.

7.4.1 Solving a System of Nonlinear Equations Using Substitution

A system of nonlinear equations is a system of two or more equations in two or more variables containingat least one equation that is not linear. Recall that a linear equation can take the formAx+By+C = 0.Anyequation that cannot be written in this form in nonlinear. The substitution method we used for linearsystems is the same method we will use for nonlinear systems. We solve one equation for one variable andthen substitute the result into the second equation to solve for another variable, and so on. There is, however,a variation in the possible outcomes.

7.4.1.1 Intersection of a Parabola and a Line

There are three possible types of solutions for a system of nonlinear equations involving a parabola and aline.

Available for free at Connexions <http://legacy.cnx.org/content/col11759/1.2>

653

A General Note: Figure 7.20 illustrates possible solution sets for a system of equations involvinga parabola and a line.

• No solution. The line will never intersect the parabola.• One solution. The line is tangent to the parabola and intersects the parabola at exactly onepoint.• Two solutions. The line crosses on the inside of the parabola and intersects the parabola attwo points.

Figure 7.20

How To: Given a system of equations containing a line and a parabola, �nd thesolution.

1.Solve the linear equation for one of the variables.2.Substitute the expression obtained in step one into the parabola equation.3.Solve for the remaining variable.4.Check your solutions in both equations.

Example 7.17Solving a System of Nonlinear Equations Representing a Parabola and a LineSolve the system of equations.

x− y = −1

y = x2 + 1(7.92)

Solution

Solve the �rst equation forx and then substitute the resulting expression into the second equation.

Available for free at Connexions <http://legacy.cnx.org/content/col11759/1.2>

654 CHAPTER 7. SYSTEMS OF EQUATIONS AND INEQUALITIES

x− y = −1

x = y − 1 Solve for x.

y = x2 + 1

y = (y − 1)2 + 1 Substitute expression for x.

(7.93)

Expand the equation and set it equal to zero.

y = (y − 1)2

=(y2 − 2y + 1

)+ 1

= y2 − 2y + 2

0 = y2 − 3y + 2

= (y − 2) (y − 1)

(7.94)

Solving for y gives y = 2 and y = 1. Next, substitute each value for y into the �rst equation to solveforx.Always substitute the value into the linear equation to check for extraneous solutions.

x− y = −1

x− (2) = −1

x = 1

x− (1) = −1

x = 0

(7.95)

The solutions are (1, 2) and (0, 1) , which can be veri�ed by substituting these (x, y) values intoboth of the original equations. See Figure 7.21.

Available for free at Connexions <http://legacy.cnx.org/content/col11759/1.2>

655

Figure 7.21

Q&A: Could we have substituted values for y into the second equation to solve for x inExample 7.17?

Yes, but becausex is squared in the second equation this could give us extraneous solutions forx.

For y = 1

y = x2 + 1

y = x2 + 1

x2 = 0

x = ±√

0 = 0

(7.96)

This gives us the same value as in the solution.

For y = 2

Available for free at Connexions <http://legacy.cnx.org/content/col11759/1.2>

656 CHAPTER 7. SYSTEMS OF EQUATIONS AND INEQUALITIES

y = x2 + 1

2 = x2 + 1

x2 = 1

x = ±√

1 = ±1

(7.97)

Notice that − 1 is an extraneous solution.

Try It:

Exercise 7.4.1 (Solution on p. 785.)

Solve the given system of equations by substitution.

3x− y = −2

2x2 − y = 0(7.98)

7.4.1.2 Intersection of a Circle and a Line

Just as with a parabola and a line, there are three possible outcomes when solving a system of equationsrepresenting a circle and a line.

A General Note: Figure 7.22 illustrates possible solution sets for a system of equations involvinga circle and a line.

• No solution. The line does not intersect the circle.• One solution. The line is tangent to the circle and intersects the circle at exactly one point.• Two solutions. The line crosses the circle and intersects it at two points.

Figure 7.22

Available for free at Connexions <http://legacy.cnx.org/content/col11759/1.2>

657

How To: Given a system of equations containing a line and a circle, �nd the solution.

1.Solve the linear equation for one of the variables.2.Substitute the expression obtained in step one into the equation for the circle.3.Solve for the remaining variable.4.Check your solutions in both equations.

Example 7.18Finding the Intersection of a Circle and a Line by SubstitutionFind the intersection of the given circle and the given line by substitution.

x2 + y2 = 5

y = 3x− 5(7.99)

Solution

One of the equations has already been solved for y.We will substitute y = 3x− 5 into the equationfor the circle.

x2 + (3x− 5)2 = 5

x2 + 9x2 − 30x+ 25 = 5

10x2 − 30x+ 20 = 0

(7.100)

Now, we factor and solve forx.

10(x2 − 3x+ 2

)= 0

10 (x− 2) (x− 1) = 0

x = 2

x = 1

(7.101)

Substitute the two x-values into the original linear equation to solve for y.

y = 3 (2)− 5

= 1

y = 3 (1)− 5

= −2

(7.102)

The line intersects the circle at (2, 1) and (1,−2) ,which can be veri�ed by substitutingthese (x, y) values into both of the original equations. See Figure 7.23.

Available for free at Connexions <http://legacy.cnx.org/content/col11759/1.2>

658 CHAPTER 7. SYSTEMS OF EQUATIONS AND INEQUALITIES

Figure 7.23

Try It:

Exercise 7.4.2 (Solution on p. 785.)

Solve the system of nonlinear equations.

x2 + y2 = 10

x− 3y = −10(7.103)

7.4.2 Solving a System of Nonlinear Equations Using Elimination

We have seen that substitution is often the preferred method when a system of equations includes a linearequation and a nonlinear equation. However, when both equations in the system have like variables of

Available for free at Connexions <http://legacy.cnx.org/content/col11759/1.2>

659

the second degree, solving them using elimination by addition is often easier than substitution. Generally,elimination is a far simpler method when the system involves only two equations in two variables (a two-by-two system), rather than a three-by-three system, as there are fewer steps. As an example, we will investigatethe possible types of solutions when solving a system of equations representing a circle and an ellipse.

A General Note: Figure 7.24 illustrates possible solution sets for a system of equations involvinga circle and an ellipse.

• No solution. The circle and ellipse do not intersect. One shape is inside the other or the circleand the ellipse are a distance away from the other.• One solution. The circle and ellipse are tangent to each other, and intersect at exactly onepoint.• Two solutions. The circle and the ellipse intersect at two points.• Three solutions. The circle and the ellipse intersect at three points.• Four solutions. The circle and the ellipse intersect at four points.

Figure 7.24

Example 7.19Solving a System of Nonlinear Equations Representing a Circle and an EllipseSolve the system of nonlinear equations.

x2 + y2 = 26 (1)

3x2 + 25y2 = 100 (2)(7.104)

Solution

Let's begin by multiplying equation (1) by − 3, and adding it to equation (2).

(−3)(x2 + y2

)= (−3) (26)

− 3x2 − 3y2 = −78

3x2 + 25y2 = 100

22y2 = 22(7.105)

After we add the two equations together, we solve for y.

Available for free at Connexions <http://legacy.cnx.org/content/col11759/1.2>

660 CHAPTER 7. SYSTEMS OF EQUATIONS AND INEQUALITIES

y2 = 1

y = ±√

1 = ±1(7.106)

Substitute y = ±1 into one of the equations and solve forx.

x2 + (1)2 = 26

x2 + 1 = 26

x2 = 25

x = ±√

25 = ±5

x2 + (−1)2 = 26

x2 + 1 = 26

x2 = 25 = ±5

(7.107)

There are four solutions: (5, 1) , (−5, 1) , (5,−1) , and (−5,−1) . See Figure 7.25.

Figure 7.25

Available for free at Connexions <http://legacy.cnx.org/content/col11759/1.2>

661

Try It:

Exercise 7.4.3 (Solution on p. 785.)

Find the solution set for the given system of nonlinear equations.

4x2 + y2 = 13

x2 + y2 = 10(7.108)

7.4.3 Graphing a Nonlinear Inequality

All of the equations in the systems that we have encountered so far have involved equalities, but we mayalso encounter systems that involve inequalities. We have already learned to graph linear inequalities bygraphing the corresponding equation, and then shading the region represented by the inequality symbol.Now, we will follow similar steps to graph a nonlinear inequality so that we can learn to solve systems ofnonlinear inequalities. A nonlinear inequality is an inequality containing a nonlinear expression. Graphinga nonlinear inequality is much like graphing a linear inequality.

Recall that when the inequality is greater than, y > a,or less than, y < a, the graph is drawn with adashed line. When the inequality is greater than or equal to, y ≥ a, or less than or equal to, y ≤ a, thegraph is drawn with a solid line. The graphs will create regions in the plane, and we will test each regionfor a solution. If one point in the region works, the whole region works. That is the region we shade. SeeFigure 7.26.

Figure 7.26: (a) an example of y > a; (b) an example of y ≥ a; (c) an example of y < a; (d) an exampleof y ≤ a

Available for free at Connexions <http://legacy.cnx.org/content/col11759/1.2>

662 CHAPTER 7. SYSTEMS OF EQUATIONS AND INEQUALITIES

How To: Given an inequality bounded by a parabola, sketch a graph.

1.Graph the parabola as if it were an equation. This is the boundary for the region that is thesolution set.

2.If the boundary is included in the region (the operator is ≤ or ≥), the parabola is graphedas a solid line.

3.If the boundary is not included in the region (the operator is < or >), the parabola is graphedas a dashed line.

4.Test a point in one of the regions to determine whether it satis�es the inequality statement.If the statement is true, the solution set is the region including the point. If the statement isfalse, the solution set is the region on the other side of the boundary line.

5.Shade the region representing the solution set.

Example 7.20Graphing an Inequality for a ParabolaGraph the inequality y > x2 + 1.

Solution

First, graph the corresponding equation y = x2 + 1. Since y > x2 + 1 has a greater than symbol,we draw the graph with a dashed line. Then we choose points to test both inside and outside theparabola. Let's test the points(0, 2) and (2, 0) .One point is clearly inside the parabola and the other point is clearly outside.

y > x2 + 1

2 > (0)2 + 1

2 > 1 True

0 > (2)2 + 1

0 > 5 False

(7.109)

The graph is shown in Figure 7.27. We can see that the solution set consists of all points insidethe parabola, but not on the graph itself.

Available for free at Connexions <http://legacy.cnx.org/content/col11759/1.2>

663

Figure 7.27

7.4.4 Graphing a System of Nonlinear Inequalities

Now that we have learned to graph nonlinear inequalities, we can learn how to graph systems of nonlinearinequalities. A system of nonlinear inequalities is a system of two or more inequalities in two or morevariables containing at least one inequality that is not linear. Graphing a system of nonlinear inequalitiesis similar to graphing a system of linear inequalities. The di�erence is that our graph may result in moreshaded regions that represent a solution than we �nd in a system of linear inequalities. The solution to anonlinear system of inequalities is the region of the graph where the shaded regions of the graph of eachinequality overlap, or where the regions intersect, called the feasible region.

How To: Given a system of nonlinear inequalities, sketch a graph.

1.Find the intersection points by solving the corresponding system of nonlinear equations.2.Graph the nonlinear equations.3.Find the shaded regions of each inequality.4.Identify the feasible region as the intersection of the shaded regions of each inequality or theset of points common to each inequality.

Available for free at Connexions <http://legacy.cnx.org/content/col11759/1.2>

664 CHAPTER 7. SYSTEMS OF EQUATIONS AND INEQUALITIES

Example 7.21Graphing a System of InequalitiesGraph the given system of inequalities.

x2 − y ≤ 0

2x2 + y ≤ 12(7.110)

Solution

These two equations are clearly parabolas. We can �nd the points of intersection by the eliminationprocess: Add both equations and the variable ywill be eliminated. Then we solve forx.

x2 − y = 0

2x2 + y = 12____________

3x2 = 12

x2 = 4

x = ±2

(7.111)

Substitute the x-values into one of the equations and solve for y.

x2 − y = 0

(2)2 − y = 0

4− y = 0

y = 4

(−2)2 − y = 0

4− y = 0

y = 4

(7.112)

The two points of intersection are (2, 4) and (−2, 4) .Notice that the equations can be rewrittenas follows.

x2 − y ≤ 0

x2 ≤ yy ≥ x2

2x2 + y ≤ 12

y ≤ −2x2 + 12

(7.113)

Graph each inequality. See Figure 7.28. The feasible region is the region between the two equationsbounded by 2x2 + y ≤ 12 on the top andx2 − y ≤ 0 on the bottom.

Available for free at Connexions <http://legacy.cnx.org/content/col11759/1.2>

665

Figure 7.28

Try It:

Exercise 7.4.4 (Solution on p. 785.)

Graph the given system of inequalities.

y ≥ x2 − 1

x− y ≥ −1(7.114)

Media: Access these online resources for additional instruction and practice with nonlinearequations.

Available for free at Connexions <http://legacy.cnx.org/content/col11759/1.2>

666 CHAPTER 7. SYSTEMS OF EQUATIONS AND INEQUALITIES

• Solve a System of Nonlinear Equations Using Substitution15

• Solve a System of Nonlinear Equations Using Elimination16

7.4.5 Key Concepts

• There are three possible types of solutions to a system of equations representing a line and a parabola:(1) no solution, the line does not intersect the parabola; (2) one solution, the line is tangent to theparabola; and (3) two solutions, the line intersects the parabola in two points. See Example 7.17.

• There are three possible types of solutions to a system of equations representing a circle and a line: (1)no solution, the line does not intersect the circle; (2) one solution, the line is tangent to the parabola;(3) two solutions, the line intersects the circle in two points. See Example 7.18.

• There are �ve possible types of solutions to the system of nonlinear equations representing an ellipseand a circle:(1) no solution, the circle and the ellipse do not intersect; (2) one solution, the circle and the ellipseare tangent to each other; (3) two solutions, the circle and the ellipse intersect in two points; (4) threesolutions, the circle and ellipse intersect in three places; (5) four solutions, the circle and the ellipseintersect in four points. See Example 7.19.

• An inequality is graphed in much the same way as an equation, except for > or <, we draw a dashedline and shade the region containing the solution set. See Example 7.20.

• Inequalities are solved the same way as equalities, but solutions to systems of inequalities must satisfyboth inequalities. See Example 7.21.

7.4.6 Section Exercises

7.4.6.1 Verbal

Exercise 7.4.5 (Solution on p. 785.)

Explain whether a system of two nonlinear equations can have exactly two solutions. What aboutexactly three? If not, explain why not. If so, give an example of such a system, in graph form, andexplain why your choice gives two or three answers.

Exercise 7.4.6When graphing an inequality, explain why we only need to test one point to determine whether anentire region is the solution?

Exercise 7.4.7 (Solution on p. 786.)

When you graph a system of inequalities, will there always be a feasible region? If so, explain why.If not, give an example of a graph of inequalities that does not have a feasible region. Why does itnot have a feasible region?

Exercise 7.4.8If you graph a revenue and cost function, explain how to determine in what regions there is pro�t.

Exercise 7.4.9 (Solution on p. 786.)

If you perform your break-even analysis and there is more than one solution, explain how you woulddetermine which x-values are pro�t and which are not.

15http://openstaxcollege.org/l/nonlinsub16http://openstaxcollege.org/l/nonlinelim

Available for free at Connexions <http://legacy.cnx.org/content/col11759/1.2>

667

7.4.6.2 Algebraic

For the following exercises, solve the system of nonlinear equations using substitution.

Exercise 7.4.10

x+ y = 4

x2 + y2 = 9

Exercise 7.4.11 (Solution on p. 786.)

y = x− 3

x2 + y2 = 9

Exercise 7.4.12

y = x

x2 + y2 = 9

Exercise 7.4.13 (Solution on p. 786.)

y = −xx2 + y2 = 9

Exercise 7.4.14

x = 2

x2 − y2 = 9

For the following exercises, solve the system of nonlinear equations using elimination.

Exercise 7.4.15 (Solution on p. 786.)

4x2 − 9y2 = 36

4x2 + 9y2 = 36

Exercise 7.4.16

x2 + y2 = 25

x2 − y2 = 1

Exercise 7.4.17 (Solution on p. 786.)

2x2 + 4y2 = 4

2x2 − 4y2 = 25x− 10

Exercise 7.4.18

y2 − x2 = 9

3x2 + 2y2 = 8

Exercise 7.4.19 (Solution on p. 786.)

x2 + y2 + 116 = 2500

y = 2x2

For the following exercises, use any method to solve the system of nonlinear equations.

Exercise 7.4.20

−2x2 + y = −5

6x− y = 9

Available for free at Connexions <http://legacy.cnx.org/content/col11759/1.2>

668 CHAPTER 7. SYSTEMS OF EQUATIONS AND INEQUALITIES

Exercise 7.4.21 (Solution on p. 786.)

−x2 + y = 2

− x+ y = 2

Exercise 7.4.22

x2 + y2 = 1

y = 20x2 − 1

Exercise 7.4.23 (Solution on p. 786.)

x2 + y2 = 1

y = −x2

Exercise 7.4.24

2x3 − x2 = y

y = 12 − x

Exercise 7.4.25 (Solution on p. 786.)

9x2 + 25y2 = 225

(x− 6)2 + y2 = 1

Exercise 7.4.26

x4 − x2 = y

x2 + y = 0

Exercise 7.4.27 (Solution on p. 786.)

2x3 − x2 = y

x2 + y = 0

For the following exercises, use any method to solve the nonlinear system.

Exercise 7.4.28

x2 + y2 = 9

y = 3− x2

Exercise 7.4.29 (Solution on p. 786.)

x2 − y2 = 9

x = 3

Exercise 7.4.30

x2 − y2 = 9

y = 3

Exercise 7.4.31 (Solution on p. 786.)

x2 − y2 = 9

x− y = 0

Exercise 7.4.32

−x2 + y = 2

−4x+ y = −1

Available for free at Connexions <http://legacy.cnx.org/content/col11759/1.2>

669

Exercise 7.4.33 (Solution on p. 786.)

−x2 + y = 2

2y = −xExercise 7.4.34

x2 + y2 = 25

x2 − y2 = 36

Exercise 7.4.35 (Solution on p. 786.)

x2 + y2 = 1

y2 = x2

Exercise 7.4.36

16x2 − 9y2 + 144 = 0

y2 + x2 = 16

Exercise 7.4.37 (Solution on p. 786.)

3x2 − y2 = 12

(x− 1)2 + y2 = 1

Exercise 7.4.38

3x2 − y2 = 12

(x− 1)2 + y2 = 4

Exercise 7.4.39 (Solution on p. 786.)

3x2 − y2 = 12

x2 + y2 = 16

Exercise 7.4.40

x2 − y2 − 6x− 4y − 11 = 0

− x2 + y2 = 5

Exercise 7.4.41 (Solution on p. 786.)

x2 + y2 − 6y = 7

x2 + y = 1

Exercise 7.4.42

x2 + y2 = 6

xy = 1

7.4.6.3 Graphical

For the following exercises, graph the inequality.

Exercise 7.4.43 (Solution on p. 786.)

x2 + y < 9Exercise 7.4.44x2 + y2 < 4

For the following exercises, graph the system of inequalities. Label all points of intersection.

Available for free at Connexions <http://legacy.cnx.org/content/col11759/1.2>

670 CHAPTER 7. SYSTEMS OF EQUATIONS AND INEQUALITIES

Exercise 7.4.45 (Solution on p. 787.)

x2 + y < 1

y > 2x

Exercise 7.4.46

x2 + y < − 5

y > 5x+ 10

Exercise 7.4.47 (Solution on p. 788.)

x2 + y2 < 25

3x2 − y2 > 12

Exercise 7.4.48

x2 − y2 > − 4

x2 + y2 < 12

Exercise 7.4.49 (Solution on p. 789.)

x2 + 3y2 > 16

3x2 − y2 < 1

7.4.6.4 Extensions

For the following exercises, graph the inequality.

Exercise 7.4.50

y ≥ ex

y ≤ ln (x) + 5

Exercise 7.4.51 (Solution on p. 790.)

y ≤ −log (x)

y ≤ ex

For the following exercises, �nd the solutions to the nonlinear equations with two variables.

Exercise 7.4.524x2 + 1

y2 = 245x2 − 2

y2 + 4 = 0

Exercise 7.4.53 (Solution on p. 791.)6x2 − 1

y2 = 81x2 − 6

y2 = 18

Exercise 7.4.54

x2 − xy + y2 − 2 = 0

x+ 3y = 4

Exercise 7.4.55 (Solution on p. 791.)

x2 − xy − 2y2 − 6 = 0

x2 + y2 = 1

Available for free at Connexions <http://legacy.cnx.org/content/col11759/1.2>

671

Exercise 7.4.56

x2 + 4xy − 2y2 − 6 = 0

x = y + 2

7.4.6.5 Technology

For the following exercises, solve the system of inequalities. Use a calculator to graph the system to con�rmthe answer.

Exercise 7.4.57 (Solution on p. 791.)

xy < 1

y >√x

Exercise 7.4.58

x2 + y < 3

y > 2x

7.4.6.6 Real-World Applications

For the following exercises, construct a system of nonlinear equations to describe the given behavior, thensolve for the requested solutions.

Exercise 7.4.59 (Solution on p. 791.)

Two numbers add up to 300. One number is twice the square of the other number. What are thenumbers?

Exercise 7.4.60The squares of two numbers add to 360. The second number is half the value of the �rst numbersquared. What are the numbers?

Exercise 7.4.61 (Solution on p. 791.)

A laptop company has discovered their cost and revenue functions for each day:C (x) = 3x2−10x+200 andR (x) = −2x2 + 100x+ 50. If they want to make a pro�t, what is the range of laptops perday that they should produce? Round to the nearest number which would generate pro�t.

Exercise 7.4.62A cell phone company has the following cost and revenue functions:C (x) = 8x2 − 600x +21, 500 andR (x) = −3x2 + 480x.What is the range of cell phones they should produce each day sothere is pro�t? Round to the nearest number that generates pro�t.

7.5 Partial Fractions17

Earlier in this chapter, we studied systems of two equations in two variables, systems of three equations inthree variables, and nonlinear systems. Here we introduce another way that systems of equations can beutilized�the decomposition of rational expressions.

Fractions can be complicated; adding a variable in the denominator makes them even more so. Themethods studied in this section will help simplify the concept of a rational expression.

17This content is available online at <http://legacy.cnx.org/content/m49432/1.3/>.

Available for free at Connexions <http://legacy.cnx.org/content/col11759/1.2>

672 CHAPTER 7. SYSTEMS OF EQUATIONS AND INEQUALITIES

7.5.1 DecomposingP (x)Q(x)

Where Q(x) Has Only Nonrepeated Linear Factors

Recall the algebra regarding adding and subtracting rational expressions. These operations depend on �ndinga common denominator so that we can write the sum or di�erence as a single, simpli�ed rational expression.In this section, we will look at partial fraction decomposition, which is the undoing of the procedureto add or subtract rational expressions. In other words, it is a return from the single simpli�ed rationalexpression to the original expressions, called the partial fractions.

For example, suppose we add the following fractions:

2x− 3

+−1x+ 2

(7.115)

We would �rst need to �nd a common denominator, (x+ 2) (x− 3) .Next, we would write each expression with this common denominator and �nd the sum of the terms.

2x−3

(x+2x+2

)+ −1

x+2

(x−3x−3

)=

2x+4−x+3(x+2)(x−3) = x+7

x2−x−6

(7.116)

Partial fraction decomposition is the reverse of this procedure. We would start with the solution andrewrite (decompose) it as the sum of two fractions.

x+ 7x2 − x− 6

Simpli�ed sum

=2

x− 3+−1x+ 2

Partial fraction decomposition

(7.117)

We will investigate rational expressions with linear factors and quadratic factors in the denominator wherethe degree of the numerator is less than the degree of the denominator. Regardless of the type of expressionwe are decomposing, the �rst and most important thing to do is factor the denominator.

When the denominator of the simpli�ed expression contains distinct linear factors, it is likely that eachof the original rational expressions, which were added or subtracted, had one of the linear factors as thedenominator. In other words, using the example above, the factors ofx2 − x − 6 are (x− 3) (x+ 2) , thedenominators of the decomposed rational expression. So we will rewrite the simpli�ed form as the sum ofindividual fractions and use a variable for each numerator. Then, we will solve for each numerator using oneof several methods available for partial fraction decomposition.

A general note label: The partial fraction decomposition of P (x)Q(x) whenQ (x) has nonre-

peated linear factors and the degree ofP (x) is less than the degree ofQ (x) is

P (x)Q (x)

=A1

(a1x+ b1)+

A2

(a2x+ b2)+

A3

(a3x+ b3)+ · · ·+ An

(anx+ bn). (7.118)

How to feature: Given a rational expression with distinct linear factors in thedenominator, decompose it.

1.Use a variable for the original numerators, usuallyA,B, orC, depending on the number offactors, placing each variable over a single factor. For the purpose of this de�nition, weuseAn for each numerator

P (x)Q (x)

=A1

(a1x+ b1)+

A2

(a2x+ b2)+ · · ·+ An

(anx+ bn)(7.119)

Available for free at Connexions <http://legacy.cnx.org/content/col11759/1.2>

673

2.Multiply both sides of the equation by the common denominator to eliminate fractions.3.Expand the right side of the equation and collect like terms.4.Set coe�cients of like terms from the left side of the equation equal to those on the right sideto create a system of equations to solve for the numerators.

Example 7.22Decomposing a Rational Function with Distinct Linear FactorsDecompose the given rational expression with distinct linear factors.

3x(x+ 2) (x− 1)

(7.120)

Solution

We will separate the denominator factors and give each numerator a symbolic label, likeA,B ,orC.

3x(x+ 2) (x− 1)

=A

(x+ 2)+

B

(x− 1)(7.121)

Multiply both sides of the equation by the common denominator to eliminate the fractions:

(x+ 2) (x− 1)[

3x(x+ 2) (x− 1)

]= ) (x+ 2) (x− 1)

[A

) (x+ 2)

]+ (x+ 2) ) (x− 1)

[B

) (x− 1)

](7.122)

The resulting equation is

3x = A (x− 1) +B (x+ 2) (7.123)

Expand the right side of the equation and collect like terms.

3x = Ax−A+Bx+ 2B

3x = (A+B)x−A+ 2B(7.124)

Set up a system of equations associating corresponding coe�cients.

3 = A+B

0 = −A+ 2B(7.125)

Add the two equations and solve forB.

3 = A+B

0 = −A+ 2BΨ

3 = 0 + 3B

1 = B

(7.126)

SubstituteB = 1 into one of the original equations in the system.

3 = A+ 1

2 = A(7.127)

Thus, the partial fraction decomposition is

Available for free at Connexions <http://legacy.cnx.org/content/col11759/1.2>

674 CHAPTER 7. SYSTEMS OF EQUATIONS AND INEQUALITIES

3x(x+ 2) (x− 1)

=2

(x+ 2)+

1(x− 1)

(7.128)

Another method to use to solve forA orB is by considering the equation that resulted from elimi-nating the fractions and substituting a value forx that will make either the A- or B-term equal 0.If we letx = 1, theA− term becomes 0 and we can simply solve forB.

3x = A (x− 1) +B (x+ 2)

3 (1) = A [(1)− 1] +B [(1) + 2]

3 = 0 + 3B

1 = B

(7.129)

Next, either substituteB = 1 into the equation and solve forA, or make the B-term 0 bysubstitutingx = −2 into the equation.

3x = A (x− 1) +B (x+ 2)

3 (−2) = A [(−2)− 1] +B [(−2) + 2]

− 6 = −3A+ 0−6−3 = A

2 = A

(7.130)

We obtain the same values forA andB using either method, so the decompositions are the sameusing either method.

3x(x+ 2) (x− 1)

=2

(x+ 2)+

1(x− 1)

(7.131)

Although this method is not seen very often in textbooks, we present it here as an alternative thatmay make some partial fraction decompositions easier. It is known as the Heaviside method,named after Charles Heaviside, a pioneer in the study of electronics.

Try it feature:

Exercise 7.5.1 (Solution on p. 791.)

Find the partial fraction decomposition of the following expression.

x

(x− 3) (x− 2)(7.132)

Available for free at Connexions <http://legacy.cnx.org/content/col11759/1.2>

675

7.5.2 DecomposingP (x)Q(x)

Where Q(x) Has Repeated Linear Factors

Some fractions we may come across are special cases that we can decompose into partial fractions withrepeated linear factors. We must remember that we account for repeated factors by writing each factor inincreasing powers.

A general note label: The partial fraction decomposition of P (x)Q(x) , whenQ (x) has a repeated

linear factor occurringn times and the degree ofP (x) is less than the degree ofQ (x) , is

P (x)Q (x)

=A1

(ax+ b)+

A2

(ax+ b)2 +A3

(ax+ b)3 + · · ·+ An(ax+ b)n

(7.133)

Write the denominator powers in increasing order.

How to feature: Given a rational expression with repeated linear factors, decomposeit.

1.Use a variable likeA,B, orC for the numerators and account for increasing powers of thedenominators.

P (x)Q (x)

=A1

(ax+ b)+

A2

(ax+ b)2 + . . . +An

(ax+ b)n(7.134)

2.Multiply both sides of the equation by the common denominator to eliminate fractions.3.Expand the right side of the equation and collect like terms.4.Set coe�cients of like terms from the left side of the equation equal to those on the right sideto create a system of equations to solve for the numerators.

Example 7.23Decomposing with Repeated Linear FactorsDecompose the given rational expression with repeated linear factors.

−x2 + 2x+ 4x3 − 4x2 + 4x

(7.135)

Solution

The denominator factors arex(x− 2)2.To allow for the repeated factor of (x− 2) , the decompo-

sition will include three denominators:x, (x− 2) , and (x− 2)2.Thus,

−x2 + 2x+ 4x3 − 4x2 + 4x

=A

x+

B

(x− 2)+

C

(x− 2)2 (7.136)

Next, we multiply both sides by the common denominator.

x(x− 2)2[−x2+2x+4x(x−2)2

]=[Ax + B

(x−2) + C(x−2)2

]x(x− 2)2

− x2 + 2x+ 4 = A(x− 2)2 +Bx (x− 2) + Cx(7.137)

On the right side of the equation, we expand and collect like terms.

−x2 + 2x+ 4 = A(x2 − 4x+ 4

)+B

(x2 − 2x

)+ Cx

= Ax2 − 4Ax+ 4A+Bx2 − 2Bx+ Cx

= (A+B)x2 + (−4A− 2B + C)x+ 4A

(7.138)

Available for free at Connexions <http://legacy.cnx.org/content/col11759/1.2>

676 CHAPTER 7. SYSTEMS OF EQUATIONS AND INEQUALITIES

Next, we compare the coe�cients of both sides. This will give the system of equations in threevariables:

−x2 + 2x+ 4 = (A+B)x2 + (−4A− 2B + C)x+ 4A (7.139)

A+B = −1 (1)

−4A− 2B + C = 2 (2)

4A = 4 (3)

(7.140)

Solving forA, we have

4A = 4

A = 1(7.141)

SubstituteA = 1 into equation (1).

A+B = −1

(1) +B = −1

B = −2

(7.142)

Then, to solve forC, substitute the values forA andB into equation (2).

− 4A− 2B + C = 2

−4 (1)− 2 (−2) + C = 2

− 4 + 4 + C = 2

C = 2

(7.143)

Thus,

−x2 + 2x+ 4x3 − 4x2 + 4x

=1x− 2

(x− 2)+

2(x− 2)2 (7.144)

Try it feature:

Exercise 7.5.2 (Solution on p. 791.)

Find the partial fraction decomposition of the expression with repeated linear factors.

6x− 11(x− 1)2 (7.145)

Available for free at Connexions <http://legacy.cnx.org/content/col11759/1.2>

677

7.5.3 DecomposingP (x)Q(x)

, Where Q(x) Has a Nonrepeated Irreducible Quadratic

Factor

So far, we have performed partial fraction decomposition with expressions that have had linear factors in thedenominator, and we applied numeratorsA,B, orC representing constants. Now we will look at an examplewhere one of the factors in the denominator is a quadratic expression that does not factor. This is referredto as an irreducible quadratic factor. In cases like this, we use a linear numerator such asAx+B,Bx+C, etc.

A general note label: The partial fraction decomposition of P (x)Q(x) such thatQ (x) has a nonre-

peated irreducible quadratic factor and the degree ofP (x) is less than the degree ofQ (x) is writtenas

P (x)Q (x)

=A1x+B1

(a1x2 + b1x+ c1)+

A2x+B2

(a2x2 + b2x+ c2)+ · · ·+ Anx+Bn

(anx2 + bnx+ cn)(7.146)

The decomposition may contain more rational expressions if there are linear factors. Each linearfactor will have a di�erent constant numerator:A,B,C, and so on.

How to feature: Given a rational expression where the factors of the denominatorare distinct, irreducible quadratic factors, decompose it.

1.Use variables such asA,B, orC for the constant numerators over linear factors, and linearexpressions such asA1x + B1, A2x + B2, etc., for the numerators of each quadratic factor inthe denominator.

P (x)Q (x)

=A

ax+ b+

A1x+B1

(a1x2 + b1x+ c1)+

A2x+B2

(a2x2 + b2x+ c2)+ · · ·+ Anx+Bn

(anx2 + bnx+ cn)(7.147)

2.Multiply both sides of the equation by the common denominator to eliminate fractions.3.Expand the right side of the equation and collect like terms.4.Set coe�cients of like terms from the left side of the equation equal to those on the right sideto create a system of equations to solve for the numerators.

Example 7.24

Decomposing P (x)Q(x) When Q(x) Contains a Nonrepeated Irreducible Quadratic Factor

Find a partial fraction decomposition of the given expression.

8x2 + 12x− 20(x+ 3) (x2 + x+ 2)

(7.148)

Solution

We have one linear factor and one irreducible quadratic factor in the denominator, so one numeratorwill be a constant and the other numerator will be a linear expression. Thus,

8x2 + 12x− 20(x+ 3) (x2 + x+ 2)

=A

(x+ 3)+

Bx+ C

(x2 + x+ 2)(7.149)

We follow the same steps as in previous problems. First, clear the fractions by multiplying bothsides of the equation by the common denominator.

(x+ 3)(x2 + x+ 2

) [8x2+12x−20

(x+3)(x2+x+2)

]=[

A(x+3) + Bx+C

(x2+x+2)

](x+ 3)

(x2 + x+ 2

)8x2 + 12x− 20 = A

(x2 + x+ 2

)+ (Bx+ C) (x+ 3)

(7.150)

Available for free at Connexions <http://legacy.cnx.org/content/col11759/1.2>

678 CHAPTER 7. SYSTEMS OF EQUATIONS AND INEQUALITIES

Notice we could easily solve forA by choosing a value forx that will make theBx + C term equal0. Letx = −3 and substitute it into the equation.

8x2 + 12x− 20 = A(x2 + x+ 2

)+ (Bx+ C) (x+ 3)

8(−3)2 + 12 (−3)− 20 = A(

(−3)2 + (−3) + 2)

+ (B (−3) + C) ((−3) + 3)

16 = 8A

A = 2

(7.151)

Now that we know the value ofA, substitute it back into the equation. Then expand the right sideand collect like terms.

8x2 + 12x− 20 = 2(x2 + x+ 2

)+ (Bx+ C) (x+ 3)

8x2 + 12x− 20 = 2x2 + 2x+ 4 +Bx2 + 3B + Cx+ 3C

8x2 + 12x− 20 = (2 +B)x2 + (2 + 3B + C)x+ (4 + 3C)

(7.152)

Setting the coe�cients of terms on the right side equal to the coe�cients of terms on the left sidegives the system of equations.

2 +B = 8 (1)

2 + 3B + C = 12 (2)

4 + 3C = −20 (3)

(7.153)

Solve forB using equation (1) and solve forC using equation (3).

2 +B = 8 (1)

B = 6

4 + 3C = −20 (3)

3C = −24

C = −8

(7.154)

Thus, the partial fraction decomposition of the expression is

8x2 + 12x− 20(x+ 3) (x2 + x+ 2)

=2

(x+ 3)+

6x− 8(x2 + x+ 2)

(7.155)

QA feature: Could we have just set up a system of equations to solve Example 7.24?

Yes, we could have solved it by setting up a system of equations without solving forA �rst. Theexpansion on the right would be:

Available for free at Connexions <http://legacy.cnx.org/content/col11759/1.2>

679

8x2 + 12x− 20 = Ax2 +Ax+ 2A+Bx2 + 3B + Cx+ 3C

8x2 + 12x− 20 = (A+B)x2 + (A+ 3B + C)x+ (2A+ 3C)

(7.156)

So the system of equations would be:

A+B = 8

A+ 3B + C = 12

2A+ 3C = −20

(7.157)

Try it feature:

Exercise 7.5.3 (Solution on p. 791.)

Find the partial fraction decomposition of the expression with a nonrepeating irreduciblequadratic factor.

5x2 − 6x+ 7(x− 1) (x2 + 1)

(7.158)

7.5.4 DecomposingP (x)Q(x)

When Q(x) Has a Repeated Irreducible Quadratic Factor

Now that we can decompose a simpli�ed rational expression with an irreducible quadratic factor, wewill learn how to do partial fraction decomposition when the simpli�ed rational expression has repeatedirreducible quadratic factors. The decomposition will consist of partial fractions with linear numerators overeach irreducible quadratic factor represented in increasing powers.

A general note label: The partial fraction decomposition of P (x)Q(x) , whenQ (x) has a repeated

irreducible quadratic factor and the degree ofP (x) is less than the degree ofQ (x) , is

P (x)(ax2 + bx+ c)n

=A1x+B1

(ax2 + bx+ c)+

A2x+B2

(ax2 + bx+ c)2 +A3x+B3

(ax2 + bx+ c)3 + · · ·+ Anx+Bn(ax2 + bx+ c)n

(7.159)Write the denominators in increasing powers.

How to feature: Given a rational expression that has a repeated irreducible factor,decompose it.

1.Use variables likeA,B, orC for the constant numerators over linear factors, and linear expres-sions such asA1x + B1, A2x + B2, etc., for the numerators of each quadratic factor in thedenominator written in increasing powers, such as

P (x)Q (x)

=A

ax+ b+

A1x+B1

(ax2 + bx+ c)+

A2x+B2

(ax2 + bx+ c)2 + · · ·+ An +Bn(ax2 + bx+ c)n

(7.160)

2.Multiply both sides of the equation by the common denominator to eliminate fractions.3.Expand the right side of the equation and collect like terms.

Available for free at Connexions <http://legacy.cnx.org/content/col11759/1.2>

680 CHAPTER 7. SYSTEMS OF EQUATIONS AND INEQUALITIES

4.Set coe�cients of like terms from the left side of the equation equal to those on the right sideto create a system of equations to solve for the numerators.

Example 7.25Decomposing a Rational Function with a Repeated Irreducible Quadratic Factor inthe DenominatorDecompose the given expression that has a repeated irreducible factor in the denominator.

x4 + x3 + x2 − x+ 1x(x2 + 1)2 (7.161)

Solution

The factors of the denominator arex,(x2 + 1

), and

(x2 + 1

)2.Recall that, when a factor in the

denominator is a quadratic that includes at least two terms, the numerator must be of the linearformAx+B. So, let's begin the decomposition.

x4 + x3 + x2 − x+ 1x(x2 + 1)2 =

A

x+Bx+ C

(x2 + 1)+

Dx+ E

(x2 + 1)2 (7.162)

We eliminate the denominators by multiplying each term byx(x2 + 1

)2.Thus,

x4 + x3 + x2 − x+ 1 = A(x2 + 1

)2+ (Bx+ C) (x)

(x2 + 1

)+ (Dx+ E) (x) (7.163)

Expand the right side.

x4 + x3 + x2 − x+ 1 = A(x4 + 2x2 + 1

)+Bx4 +Bx2 + Cx3 + Cx+Dx2 + Ex

= Ax4 + 2Ax2 +A+Bx4 +Bx2 + Cx3 + Cx+Dx2 + Ex(7.164)

Now we will collect like terms.

x4 + x3 + x2 − x+ 1 = (A+B)x4 + (C)x3 + (2A+B +D)x2 + (C + E)x+A (7.165)

Set up the system of equations matching corresponding coe�cients on each side of the equal sign.

A+B = 1

C = 1

2A+B +D = 1

C + E = −1

A = 1

(7.166)

We can use substitution from this point. SubstituteA = 1 into the �rst equation.

1 +B = 1

B = 0(7.167)

SubstituteA = 1 andB = 0 into the third equation.

2 (1) + 0 +D = 1

D = −1(7.168)

Available for free at Connexions <http://legacy.cnx.org/content/col11759/1.2>

681

SubstituteC = 1 into the fourth equation.

1 + E = −1

E = −2(7.169)

Now we have solved for all of the unknowns on the right side of the equal sign. We haveA =1, B = 0, C = 1, D = −1, andE = −2.We can write the decomposition as follows:

x4 + x3 + x2 − x+ 1x(x2 + 1)2 =

1x

+1

(x2 + 1)− x+ 2

(x2 + 1)2 (7.170)

Try IT Feature label:

Exercise 7.5.4 (Solution on p. 791.)

Find the partial fraction decomposition of the expression with a repeated irreduciblequadratic factor.

x3 − 4x2 + 9x− 5(x2 − 2x+ 3)2 (7.171)

Media Feature Label: Access these online resources for additional instruction and practicewith partial fractions.

• Partial Fraction Decomposition18

• Partial Fraction Decomposition With Repeated Linear Factors19

• Partial Fraction Decomposition With Linear and Quadratic Factors20

7.5.5 Key Concepts

• Decompose P (x)Q(x) by writing the partial fractions as A

a1x+b1+ B

a2x+b2. Solve by clearing the fractions,

expanding the right side, collecting like terms, and setting corresponding coe�cients equal to eachother, then setting up and solving a system of equations. See Example 7.22.

• The decomposition of P (x)Q(x) with repeated linear factors must account for the factors of the denominator

in increasing powers. See Example 7.23.

• The decomposition of P (x)Q(x) with a nonrepeated irreducible quadratic factor needs a linear numerator

over the quadratic factor, as in Ax + Bx+C

(ax2+bx+c) . See Example 7.24.

18http://openstaxcollege.org/l/partdecomp19http://openstaxcollege.org/l/partdecomprlf20http://openstaxcollege.org/l/partdecomlqu

Available for free at Connexions <http://legacy.cnx.org/content/col11759/1.2>

682 CHAPTER 7. SYSTEMS OF EQUATIONS AND INEQUALITIES

• In the decomposition of P (x)Q(x) , whereQ (x) has a repeated irreducible quadratic factor, when the ir-

reducible quadratic factors are repeated, powers of the denominator factors must be represented inincreasing powers as

Ax+B

(ax2 + bx+ c)+

A2x+B2

(ax2 + bx+ c)2 + · · ·+ Anx+Bn(ax2 + bx+ c)n

. (7.172)

See Example 7.25.

7.5.6 Section Exercises

7.5.6.1 Verbal

Exercise 7.5.5 (Solution on p. 791.)

Can any quotient of polynomials be decomposed into at least two partial fractions? If so, explainwhy, and if not, give an example of such a fraction

Exercise 7.5.6Can you explain why a partial fraction decomposition is unique? (Hint: Think about it as a systemof equations.)

Exercise 7.5.7 (Solution on p. 791.)

Can you explain how to verify a partial fraction decomposition graphically?

Exercise 7.5.8You are unsure if you correctly decomposed the partial fraction correctly. Explain how you coulddouble-check your answer.

Exercise 7.5.9 (Solution on p. 791.)

Once you have a system of equations generated by the partial fraction decomposition, can youexplain another method to solve it? For example if you had 7x+13

3x2+8x+15 = Ax+1 + B

3x+5 , we eventuallysimplify to 7x+ 13 = A (3x+ 5) +B (x+ 1) .Explain how you could intelligently choose anx-valuethat will eliminate eitherA orB and solve forA andB.

7.5.6.2 Algebraic

For the following exercises, �nd the decomposition of the partial fraction for the nonrepeating linear factors.

Exercise 7.5.105x+16

x2+10x+24

Exercise 7.5.11 (Solution on p. 791.)3x−79

x2−5x−24

Exercise 7.5.12−x−24

x2−2x−24

Exercise 7.5.13 (Solution on p. 791.)10x+47

x2+7x+10

Exercise 7.5.14x

6x2+25x+25

Exercise 7.5.15 (Solution on p. 792.)32x−11

20x2−13x+2

Exercise 7.5.16x+1

x2+7x+10

Available for free at Connexions <http://legacy.cnx.org/content/col11759/1.2>

683

Exercise 7.5.17 (Solution on p. 792.)5xx2−9

Exercise 7.5.1810xx2−25

Exercise 7.5.19 (Solution on p. 792.)6xx2−4

Exercise 7.5.202x−3

x2−6x+5

Exercise 7.5.21 (Solution on p. 792.)4x−1

x2−x−6

Exercise 7.5.224x+3

x2+8x+15

Exercise 7.5.23 (Solution on p. 792.)3x−1

x2−5x+6

For the following exercises, �nd the decomposition of the partial fraction for the repeating linear factors.

Exercise 7.5.24−5x−19(x+4)2

Exercise 7.5.25 (Solution on p. 792.)x

(x−2)2

Exercise 7.5.267x+14(x+3)2

Exercise 7.5.27 (Solution on p. 792.)−24x−27(4x+5)2

Exercise 7.5.28−24x−27(6x−7)2

Exercise 7.5.29 (Solution on p. 792.)5−x

(x−7)2

Exercise 7.5.305x+14

2x2+12x+18

Exercise 7.5.31 (Solution on p. 792.)5x2+20x+82x(x+1)2

Exercise 7.5.324x2+55x+255x(3x+5)2

Exercise 7.5.33 (Solution on p. 792.)54x3+127x2+80x+16

2x2(3x+2)2

Exercise 7.5.34x3−5x2+12x+144x2(x2+12x+36)

For the following exercises, �nd the decomposition of the partial fraction for the irreducible nonrepeatingquadratic factor.

Exercise 7.5.35 (Solution on p. 792.)4x2+6x+11

(x+2)(x2+x+3)

Exercise 7.5.364x2+9x+23

(x−1)(x2+6x+11)

Available for free at Connexions <http://legacy.cnx.org/content/col11759/1.2>

684 CHAPTER 7. SYSTEMS OF EQUATIONS AND INEQUALITIES

Exercise 7.5.37 (Solution on p. 792.)−2x2+10x+4

(x−1)(x2+3x+8)

Exercise 7.5.38x2+3x+1

(x+1)(x2+5x−2)

Exercise 7.5.39 (Solution on p. 792.)4x2+17x−1

(x+3)(x2+6x+1)

Exercise 7.5.404x2

(x+5)(x2+7x−5)

Exercise 7.5.41 (Solution on p. 792.)4x2+5x+3x3−1

Exercise 7.5.42−5x2+18x−4

x3+8

Exercise 7.5.43 (Solution on p. 792.)3x2−7x+33x3+27

Exercise 7.5.44x2+2x+40x3−125

Exercise 7.5.45 (Solution on p. 792.)4x2+4x+12

8x3−27

Exercise 7.5.46−50x2+5x−3

125x3−1

Exercise 7.5.47 (Solution on p. 792.)−2x3−30x2+36x+216

x4+216x

For the following exercises, �nd the decomposition of the partial fraction for the irreducible repeatingquadratic factor.

Exercise 7.5.483x3+2x2+14x+15

(x2+4)2

Exercise 7.5.49 (Solution on p. 792.)x3+6x2+5x+9

(x2+1)2

Exercise 7.5.50x3−x2+x−1

(x2−3)2

Exercise 7.5.51 (Solution on p. 792.)x2+5x+5(x+2)2

Exercise 7.5.52x3+2x2+4x(x2+2x+9)2

Exercise 7.5.53 (Solution on p. 792.)x2+25

(x2+3x+25)2

Exercise 7.5.542x3+11x+7x+70

(2x2+x+14)2

Exercise 7.5.55 (Solution on p. 792.)5x+2

x(x2+4)2

Exercise 7.5.56x4+x3+8x2+6x+36

x(x2+6)2

Available for free at Connexions <http://legacy.cnx.org/content/col11759/1.2>

685

Exercise 7.5.57 (Solution on p. 792.)2x−9

(x2−x)2

Exercise 7.5.585x3−2x+1(x2+2x)2

7.5.6.3 Extensions

For the following exercises, �nd the partial fraction expansion.

Exercise 7.5.59 (Solution on p. 792.)x2+4

(x+1)3

Exercise 7.5.60x3−4x2+5x+4

(x−2)3

For the following exercises, perform the operation and then �nd the partial fraction decomposition.

Exercise 7.5.61 (Solution on p. 792.)7

x+8 + 5x−2 −

x−1x2−6x−16

Exercise 7.5.621

x−4 −3

x+6 −2x+7

x2+2x−24

Exercise 7.5.63 (Solution on p. 793.)2x

x2−16 −1−2x

x2+6x+8 −x−5x2−4x

Available for free at Connexions <http://legacy.cnx.org/content/col11759/1.2>

686 CHAPTER 7. SYSTEMS OF EQUATIONS AND INEQUALITIES

7.6 Matrices and Matrix Operations21

Figure 7.29: (credit: �SD Dirk,� Flickr)

Two club soccer teams, the Wildcats and the Mud Cats, are hoping to obtain new equipment for an upcomingseason. Table 7.2 shows the needs of both teams.

21This content is available online at <http://legacy.cnx.org/content/m49433/1.3/>.

Available for free at Connexions <http://legacy.cnx.org/content/col11759/1.2>

687

Wildcats Mud Cats

Goals 6 10

Balls 30 24

Jerseys 14 20

Table 7.2

A goal costs $300; a ball costs $10; and a jersey costs $30. How can we �nd the total cost for theequipment needed for each team? In this section, we discover a method in which the data in the soccerequipment table can be displayed and used for calculating other information. Then, we will be able tocalculate the cost of the equipment.

7.6.1 Finding the Sum and Di�erence of Two Matrices

To solve a problem like the one described for the soccer teams, we can use a matrix, which is a rectangulararray of numbers. A row in a matrix is a set of numbers that are aligned horizontally. A column in a matrixis a set of numbers that are aligned vertically. Each number is an entry, sometimes called an element, of thematrix. Matrices (plural) are enclosed in [ ] or ( ), and are usually named with capital letters. For example,three matrices namedA,B, andC are shown below.

A =

1 2

3 4

, B =

1 2 7

0 −5 6

7 8 2

, C =

−1

0

3

3

2

1

(7.173)

7.6.1.1 Describing Matrices

A matrix is often referred to by its size or dimensions: m × n indicatingm rows andn columns. Matrixentries are de�ned �rst by row and then by column. For example, to locate the entry in matrixA identi�edas aij , we look for the entry in row i, column j. In matrixA, shown below, the entry in row 2, column 3 is a23.

A =

a11 a12 a13

a21 a22 a23

a31 a32 a33

(7.174)

A square matrix is a matrix with dimensions n × n, meaning that it has the same number of rows ascolumns. The 3× 3matrix above is an example of a square matrix.

A row matrix is a matrix consisting of one row with dimensions 1 × n.[a11 a12 a13

](7.175)

A column matrix is a matrix consisting of one column with dimensionsm × 1.a11

a21

a31

(7.176)

Available for free at Connexions <http://legacy.cnx.org/content/col11759/1.2>

688 CHAPTER 7. SYSTEMS OF EQUATIONS AND INEQUALITIES

A matrix may be used to represent a system of equations. In these cases, the numbers represent thecoe�cients of the variables in the system. Matrices often make solving systems of equations easier becausethey are not encumbered with variables. We will investigate this idea further in the next section, but �rstwe will look at basic matrix operations.

A General Note: A matrix is a rectangular array of numbers that is usually named by acapital letter:A,B,C, and so on. Each entry in a matrix is referred to as aij ,such that i representsthe row and j represents the column. Matrices are often referred to by their dimensions:m ×n indicatingm rows andn columns.

Example 7.26Finding the Dimensions of the Given Matrix and Locating EntriesGiven matrixA :

a. What are the dimensions of matrixA?b. What are the entries at a31 and a22?

A =

2 1 0

2 4 7

3 1 −2

(7.177)

Solution

a. The dimensions are 3 × 3 because there are three rows and three columns.b. Entry a31 is the number at row 3, column 1, which is 3. The entry a22 is the number at row 2,

column 2, which is 4. Remember, the row comes �rst, then the column.

7.6.1.2 Adding and Subtracting Matrices

We use matrices to list data or to represent systems. Because the entries are numbers, we can performoperations on matrices. We add or subtract matrices by adding or subtracting corresponding entries.

In order to do this, the entries must correspond. Therefore, addition and subtraction of matrices isonly possible when the matrices have the same dimensions. We can add or subtract a 3 × 3 matrix andanother 3 × 3 matrix, but we cannot add or subtract a 2 × 3 matrix and a 3 × 3 matrix because someentries in one matrix will not have a corresponding entry in the other matrix.

A General Note: Given matricesA andB of like dimensions, addition and subtractionofA andB will produce matrixC ormatrixD of the same dimension.

A+B = C such that aij + bij = cij (7.178)

A−B = D such that aij − bij = dij (7.179)

Matrix addition is commutative.

Available for free at Connexions <http://legacy.cnx.org/content/col11759/1.2>

689

A+B = B +A (7.180)

It is also associative.

(A+B) + C = A+ (B + C) (7.181)

Example 7.27Finding the Sum of MatricesFind the sum ofA andB, given

A =

a b

c d

and B =

e f

g h

(7.182)

Solution

Add corresponding entries.

A+B =

a b

c d

+

e f

g h

=

a+ e b+ f

c+ g d+ h

(7.183)

Example 7.28Adding Matrix A and Matrix BFind the sum ofA andB.

A =

4 1

3 2

and B =

5 9

0 7

(7.184)

Solution

Add corresponding entries. Add the entry in row 1, column 1, a11, of matrixA to the entry in row1, column 1, b11,ofB.Continue the pattern until all entries have been added.

A+B =

4 1

3 2

+

5 9

0 7

=

4 + 5 1 + 9

3 + 0 2 + 7

=

9 10

3 9

(7.185)

Available for free at Connexions <http://legacy.cnx.org/content/col11759/1.2>

690 CHAPTER 7. SYSTEMS OF EQUATIONS AND INEQUALITIES

Example 7.29Finding the Di�erence of Two MatricesFind the di�erence ofA andB.

A =

−2 3

0 1

and B =

8 1

5 4

(7.186)

Solution

We subtract the corresponding entries of each matrix.

A−B =

−2 3

0 1

− 8 1

5 4

=

−2− 8 3− 1

0− 5 1− 4

=

−10 2

−5 −3

(7.187)

Example 7.30Finding the Sum and Di�erence of Two 3 x 3 MatricesGivenA andB :

a. Find the sum.b. Find the di�erence.

A =

2 −10 −2

14 12 10

4 −2 2

and B =

6 10 −2

0 −12 −4

−5 2 −2

(7.188)

Solution

a. Add the corresponding entries.

Available for free at Connexions <http://legacy.cnx.org/content/col11759/1.2>

691

A+B =

2 − 10 − 2

14 12 10

4 − 2 2

+

6 10 − 2

0 − 12 − 4

−5 2 − 2

=

2 + 6 − 10 + 10 − 2− 2

14 + 0 12− 12 10− 4

4− 5 − 2 + 2 2− 2

=

8 0 − 4

14 0 6

−1 0 0

(7.189)

b. Subtract the corresponding entries.

A−B =

2 −10 −2

14 12 10

4 −2 2

6 10 −2

0 −12 −4

−5 2 −2

=

2− 6 − 10− 10 − 2 + 2

14− 0 12 + 12 10 + 4

4 + 5 − 2− 2 2 + 2

=

−4 − 20 0

14 24 14

9 − 4 4

(7.190)

Try It:

Exercise 7.6.1 (Solution on p. 793.)

Add matrixA and matrixB.

A =

2 6

1 0

1 −3

and B =

3 −2

1 5

−4 3

(7.191)

Available for free at Connexions <http://legacy.cnx.org/content/col11759/1.2>

692 CHAPTER 7. SYSTEMS OF EQUATIONS AND INEQUALITIES

7.6.2 Finding Scalar Multiples of a Matrix

Besides adding and subtracting whole matrices, there are many situations in which we need to multiply amatrix by a constant called a scalar. Recall that a scalar is a real number quantity that has magnitude, butnot direction. For example, time, temperature, and distance are scalar quantities. The process of scalarmultiplication involves multiplying each entry in a matrix by a scalar. A scalar multiple is any entry ofa matrix that results from scalar multiplication.

Consider a real-world scenario in which a university needs to add to its inventory of computers, computertables, and chairs in two of the campus labs due to increased enrollment. They estimate that 15% moreequipment is needed in both labs. The school's current inventory is displayed in Table 7.3.

Lab A Lab B

Computers 15 27

Computer Tables 16 34

Chairs 16 34

Table 7.3

Converting the data to a matrix, we have

C2013 =

15

16

16

27

34

34

(7.192)

To calculate how much computer equipment will be needed, we multiply all entries in matrixC by 0.15.

(0.15)C2013 =

(0.15) 15

(0.15) 16

(0.15) 16

(0.15) 27

(0.15) 34

(0.15) 34

=

2.25

2.4

2.4

4.05

5.1

5.1

(7.193)

We must round up to the next integer, so the amount of new equipment needed is3

3

3

5

6

6

(7.194)

Adding the two matrices as shown below, we see the new inventory amounts.15

16

16

27

34

34

+

3

3

3

5

6

6

=

18

19

19

32

40

40

(7.195)

This means

C2014 =

18

19

19

32

40

40

(7.196)

Available for free at Connexions <http://legacy.cnx.org/content/col11759/1.2>

693

Thus, Lab A will have 18 computers, 19 computer tables, and 19 chairs; Lab B will have 32 computers, 40computer tables, and 40 chairs.

A General Note: Scalar multiplication involves �nding the product of a constant by each entryin the matrix. Given

A =

a11 a12

a21 a22

(7.197)

the scalar multiple cA is

cA = c

a11 a12

a21 a22

=

ca11 ca12

ca21 ca22

(7.198)

Scalar multiplication is distributive. For the matricesA,B,andC with scalars a and b,

a (A+B) = aA+ aB

(a+ b)A = aA+ bA

(7.199)

Example 7.31Multiplying the Matrix by a ScalarMultiply matrixA by the scalar 3.

A =

8 1

5 4

(7.200)

Solution

Multiply each entry inA by the scalar 3.

3A = 3

8 1

5 4

=

3 · 8 3 · 13 · 5 3 · 4

=

24 3

15 12

(7.201)

Available for free at Connexions <http://legacy.cnx.org/content/col11759/1.2>

694 CHAPTER 7. SYSTEMS OF EQUATIONS AND INEQUALITIES

Try It:

Exercise 7.6.2 (Solution on p. 793.)

Given matrixB, �nd − 2B where

B =

4 1

3 2

(7.202)

Example 7.32Finding the Sum of Scalar MultiplesFind the sum 3A+ 2B.

A =

1 −2 0

0 −1 2

4 3 −6

and B =

−1 2 1

0 −3 2

0 1 −4

(7.203)

Solution

First, �nd 3A, then 2B.

3A =

3 · 1 3 (−2) 3 · 03 · 0 3 (−1) 3 · 23 · 4 3 · 3 3 (−6)

=

3 − 6 0

0 − 3 6

12 9 − 18

(7.204)

2B =

2 (−1) 2 · 2 2 · 12 · 0 2 (−3) 2 · 22 · 0 2 · 1 2 (−4)

=

−2 4 2

0 −6 4

0 2 −8

(7.205)

Now, add 3A+ 2B.

Available for free at Connexions <http://legacy.cnx.org/content/col11759/1.2>

695

3A+ 2B =

3 −6 0

0 −3 6

12 9 −18

+

−2 4 2

0 −6 4

0 2 −8

=

3− 2 − 6 + 4 0 + 2

0 + 0 − 3− 6 6 + 4

12 + 0 9 + 2 −18− 8

=

1 − 2 2

0 − 9 10

12 11 −26

(7.206)

7.6.3 Finding the Product of Two Matrices

In addition to multiplying a matrix by a scalar, we can multiply two matrices. Finding the product of twomatrices is only possible when the inner dimensions are the same, meaning that the number of columnsof the �rst matrix is equal to the number of rows of the second matrix. IfA is an m × r matrix andB isan r × n matrix, then the product matrixAB is an m × n matrix. For example, the productAB is possiblebecause the number of columns inA is the same as the number of rows inB. If the inner dimensions do notmatch, the product is not de�ned.

We multiply entries ofAwith entries ofB according to a speci�c pattern as outlined below. The processof matrix multiplication becomes clearer when working a problem with real numbers.

To obtain the entries in row i ofAB, we multiply the entries in row i ofA by column j inB and add. Forexample, given matricesA andB, where the dimensions ofA are 2 × 3 and the dimensions ofB are 3 × 3, theproduct ofAB will be a 2 × 3matrix.

A =

a11 a12 a13

a21 a22 a23

and B =

b11 b12 b13

b21 b22 b23

b31 b32 b33

(7.207)

Multiply and add as follows to obtain the �rst entry of the product matrixAB.

Available for free at Connexions <http://legacy.cnx.org/content/col11759/1.2>

696 CHAPTER 7. SYSTEMS OF EQUATIONS AND INEQUALITIES

1. To obtain the entry in row 1, column 1 ofAB, multiply the �rst row inA by the �rst column inB,andadd.

[a11 a12 a13

b11

b21

b31

= a11 · b11 + a12 · b21 + a13 · b31 (7.208)

2. To obtain the entry in row 1, column 2 ofAB, multiply the �rst row ofA by the second column inB,andadd.

[a11 a12 a13

b12

b22

b32

= a11 · b12 + a12 · b22 + a13 · b32 (7.209)

3. To obtain the entry in row 1, column 3 ofAB, multiply the �rst row ofA by the third column inB,andadd.

[a11 a12 a13

b13

b23

b33

= a11 · b13 + a12 · b23 + a13 · b33 (7.210)

We proceed the same way to obtain the second row ofAB. In other words, row 2 ofA times column 1 ofB; row2 ofA times column 2 ofB; row 2 ofA times column 3 ofB.When complete, the product matrix will be

AB =

a11 · b11 + a12 · b21 + a13 · b31

a21 · b11 + a22 · b21 + a23 · b31

a11 · b12 + a12 · b22 + a13 · b32

a21 · b12 + a22 · b22 + a23 · b32

a11 · b13 + a12 · b23 + a13 · b33

a21 · b13 + a22 · b23 + a23 · b33

(7.211)

A General Note: For the matricesA,B, andC the following properties hold.

• Matrix multiplication is associative: (AB)C = A (BC) .

• Matrix multiplication is distributive: C (A+B) = CA+ CB,

(A+B)C = AC +BC.

Note that matrix multiplication is not commutative.

Example 7.33Multiplying Two MatricesMultiply matrixA and matrixB.

A =

1 2

3 4

and B =

5 6

7 8

(7.212)

Solution

Available for free at Connexions <http://legacy.cnx.org/content/col11759/1.2>

697

First, we check the dimensions of the matrices. MatrixA has dimensions 2 × 2 and matrixB hasdimensions 2 × 2.The inner dimensions are the same so we can perform the multiplication. Theproduct will have the dimensions 2 × 2.

We perform the operations outlined previously.

Example 7.34Multiplying Two MatricesGivenA andB :

a. FindAB.b. FindBA.

A =

−1 2 3

4 0 5

and B =

5

−4

2

−1

0

3

(7.213)

Solution

a. As the dimensions ofA are 2 × 3 and the dimensions ofB are 3 × 2, these matrices can bemultiplied together because the number of columns inAmatches the number of rows inB.Theresulting product will be a 2 × 2matrix, the number of rows inA by the number of columnsinB.

Available for free at Connexions <http://legacy.cnx.org/content/col11759/1.2>

698 CHAPTER 7. SYSTEMS OF EQUATIONS AND INEQUALITIES

AB =

−1 2 3

4 0 5

5 −1

−4 0

2 3

=

−1 (5) + 2 (−4) + 3 (2) − 1 (−1) + 2 (0) + 3 (3)

4 (5) + 0 (−4) + 5 (2) 4 (−1) + 0 (0) + 5 (3)

=

−7 10

30 11

(7.214)

b. The dimensions ofB are 3 × 2 and the dimensions ofA are 2 × 3.The inner dimensions matchso the product is de�ned and will be a 3 × 3matrix.

BA =

5 −1

−4 0

2 3

−1 2 3

4 0 5

=

5 (−1) +−1 (4) 5 (2) +−1 (0) 5 (3) +−1 (5)

−4 (−1) + 0 (4) − 4 (2) + 0 (0) − 4 (3) + 0 (5)

2 (−1) + 3 (4) 2 (2) + 3 (0) 2 (3) + 3 (5)

=

−9 10 10

4 −8 −12

10 4 21

(7.215)

AnalysisNotice that the productsAB andBA are not equal.

AB =

−7 10

30 11

6=−9 10 10

4 −8 −12

10 4 21

= BA (7.216)

This illustrates the fact that matrix multiplication is not commutative.

Q&A: Is it possible for AB to be de�ned but not BA?

Yes, consider a matrix A with dimension 3 × 4 and matrix B with dimension 4 × 2.For the productAB the inner dimensions are 4 and the product is de�ned, but for the product BA the innerdimensions are 2 and 3 so the product is unde�ned.

Example 7.35Using Matrices in Real-World ProblemsLet's return to the problem presented at the opening of this section. We have Table 7.4, representingthe equipment needs of two soccer teams.

Available for free at Connexions <http://legacy.cnx.org/content/col11759/1.2>

699

Wildcats Mud Cats

Goals 6 10

Balls 30 24

Jerseys 14 20

Table 7.4

We are also given the prices of the equipment, as shown in Table 7.5.

Goal $300

Ball $10

Jersey $30

Table 7.5

We will convert the data to matrices. Thus, the equipment need matrix is written as

E =

6

30

14

10

24

20

(7.217)

The cost matrix is written as

C =[

300 10 30]

(7.218)

We perform matrix multiplication to obtain costs for the equipment.

CE =[

300 10 30]·

6 10

30 24

14 20

=[

300 (6) + 10 (30) + 30 (14) 300 (10) + 10 (24) + 30 (20)]

=[

2, 520 3, 840]

(7.219)

The total cost for equipment for the Wildcats is $2,520, and the total cost for equipment for theMud Cats is $3,840.

How To: Given a matrix operation, evaluate using a calculator.

1.Save each matrix as a matrix variable [A] , [B] , [C] , ...2.Enter the operation into the calculator, calling up each matrix variable as needed.

Available for free at Connexions <http://legacy.cnx.org/content/col11759/1.2>

700 CHAPTER 7. SYSTEMS OF EQUATIONS AND INEQUALITIES

3.If the operation is de�ned, the calculator will present the solution matrix; if the operation isunde�ned, it will display an error message.

Example 7.36Using a Calculator to Perform Matrix OperationsFindAB − C given

A =

−15 25 32

41 −7 −28

10 34 −2

, B =

45 21 −37

−24 52 19

6 −48 −31

, and C =

−100 −89 −98

25 −56 74

−67 42 −75

. (7.220)

Solution

On the matrix page of the calculator, we enter matrixA above as the matrixvariable [A] ,matrixB above as the matrix variable [B] ,and matrixC above as the matrixvariable [C] .

On the home screen of the calculator, we type in the problem and call up each matrix variableas needed.

[A]× [B]− [C] (7.221)

The calculator gives us the following matrix.−983 − 462 136

1, 820 1, 897 − 856

−311 2, 032 413

(7.222)

Media: Access these online resources for additional instruction and practice with matrices andmatrix operations.

• Dimensions of a Matrix22

• Matrix Addition and Subtraction23

• Matrix Operations24

• Matrix Multiplication25

7.6.4 Key Concepts

• A matrix is a rectangular array of numbers. Entries are arranged in rows and columns.• The dimensions of a matrix refer to the number of rows and the number of columns. A 3 × 2matrix

has three rows and two columns. See Example 7.26.• We add and subtract matrices of equal dimensions by adding and subtracting corresponding entries of

each matrix. See Example 7.27, Example 7.28, Example 7.29, and Example 7.30.

22http://openstaxcollege.org/l/matrixdimen23http://openstaxcollege.org/l/matrixaddsub24http://openstaxcollege.org/l/matrixoper25http://openstaxcollege.org/l/matrixmult

Available for free at Connexions <http://legacy.cnx.org/content/col11759/1.2>

701

• Scalar multiplication involves multiplying each entry in a matrix by a constant. See Example 7.31.• Scalar multiplication is often required before addition or subtraction can occur. See Example 7.32.• Multiplying matrices is possible when inner dimensions are the same�the number of columns in the

�rst matrix must match the number of rows in the second.• The product of two matrices,A andB,is obtained by multiplying each entry in row 1 ofA by each entry

in column 1 ofB; then multiply each entry of row 1 ofA by each entry in columns 2 ofB, and so on.See Example 7.33 and Example 7.34.

• Many real-world problems can often be solved using matrices. See Example 7.35.• We can use a calculator to perform matrix operations after saving each matrix as a matrix variable.

See Example 7.36.

7.6.5 Section Exercises

7.6.5.1 Verbal

Exercise 7.6.3 (Solution on p. 793.)

Can we add any two matrices together? If so, explain why; if not, explain why not and give anexample of two matrices that cannot be added together.

Exercise 7.6.4Can we multiply any column matrix by any row matrix? Explain why or why not.

Exercise 7.6.5 (Solution on p. 793.)

Can both the productsAB andBA be de�ned? If so, explain how; if not, explain why.

Exercise 7.6.6Can any two matrices of the same size be multiplied? If so, explain why, and if not, explain whynot and give an example of two matrices of the same size that cannot be multiplied together.

Exercise 7.6.7 (Solution on p. 793.)

Does matrix multiplication commute? That is, doesAB = BA? If so, prove why it does. If not,explain why it does not.

7.6.5.2 Algebraic

For the following exercises, use the matrices below and perform the matrix addition or subtraction. Indicateif the operation is unde�ned.

A =

[1 3

0 7

], B =

[2 14

22 6

], C =

1 5

8 92

12 6

, D =

10 14

7 2

5 61

, E =

[6 12

14 5

], F =

0 9

78 17

15 4

(7.223)

Exercise 7.6.8A+B

Available for free at Connexions <http://legacy.cnx.org/content/col11759/1.2>

702 CHAPTER 7. SYSTEMS OF EQUATIONS AND INEQUALITIES

Exercise 7.6.9 (Solution on p. 793.)

C +D

Exercise 7.6.10A+ C

Exercise 7.6.11 (Solution on p. 793.)

B − EExercise 7.6.12C + F

Exercise 7.6.13 (Solution on p. 793.)

D −BFor the following exercises, use the matrices below to perform scalar multiplication.

A =

4 6

13 12

, B =

3 9

21 12

0 64

, C =

16 3 7 18

90 5 3 29

, D =

18 12 13

8 14 6

7 4 21

(7.224)

Exercise 7.6.145AExercise 7.6.15 (Solution on p. 793.)

3BExercise 7.6.16−2BExercise 7.6.17 (Solution on p. 793.)

−4CExercise 7.6.1812C

Exercise 7.6.19 (Solution on p. 793.)

100DFor the following exercises, use the matrices below to perform matrix multiplication.

A =

−1 5

3 2

, B =

3 6 4

−8 0 12

, C =

4 10

−2 6

5 9

, D =

2 −3 12

9 3 1

0 8 −10

(7.225)

Exercise 7.6.20AB

Exercise 7.6.21 (Solution on p. 794.)

BC

Exercise 7.6.22CA

Exercise 7.6.23 (Solution on p. 794.)

BD

Exercise 7.6.24DC

Available for free at Connexions <http://legacy.cnx.org/content/col11759/1.2>

703

Exercise 7.6.25 (Solution on p. 794.)

CB

For the following exercises, use the matrices below to perform the indicated operation if possible. If notpossible, explain why the operation cannot be performed.

A =

2 −5

6 7

, B =

−9 6

−4 2

, C =

0 9

7 1

, D =

−8 7 −5

4 3 2

0 9 2

, E =

4 5 3

7 −6 −5

1 0 9

(7.226)

Exercise 7.6.26A+B − CExercise 7.6.27 (Solution on p. 794.)

4A+ 5DExercise 7.6.282C +B

Exercise 7.6.29 (Solution on p. 794.)

3D + 4EExercise 7.6.30C − 0.5DExercise 7.6.31 (Solution on p. 794.)

100D − 10EFor the following exercises, use the matrices below to perform the indicated operation if possible. If notpossible, explain why the operation cannot be performed. (Hint:A2 = A ·A )

A =

−10 20

5 25

, B =

40 10

−20 30

, C =

−1 0

0 −1

1 0

(7.227)

Exercise 7.6.32AB

Exercise 7.6.33 (Solution on p. 794.)

BA

Exercise 7.6.34CA

Exercise 7.6.35 (Solution on p. 794.)

BC

Exercise 7.6.36A2

Exercise 7.6.37 (Solution on p. 794.)

B2

Exercise 7.6.38C2

Exercise 7.6.39 (Solution on p. 794.)

B2A2

Exercise 7.6.40A2B2

Available for free at Connexions <http://legacy.cnx.org/content/col11759/1.2>

704 CHAPTER 7. SYSTEMS OF EQUATIONS AND INEQUALITIES

Exercise 7.6.41 (Solution on p. 794.)

(AB)2

Exercise 7.6.42(BA)2

For the following exercises, use the matrices below to perform the indicated operation if possible. If notpossible, explain why the operation cannot be performed. (Hint:A2 = A ·A )

A =

1 0

2 3

, B =

−2 3 4

−1 1 −5

, C =

0.5 0.1

1 0.2

−0.5 0.3

, D =

1 0 −1

−6 7 5

4 2 1

(7.228)

Exercise 7.6.43 (Solution on p. 794.)

AB

Exercise 7.6.44BA

Exercise 7.6.45 (Solution on p. 795.)

BD

Exercise 7.6.46DC

Exercise 7.6.47 (Solution on p. 795.)

D2

Exercise 7.6.48A2

Exercise 7.6.49 (Solution on p. 795.)

D3

Exercise 7.6.50(AB)CExercise 7.6.51 (Solution on p. 795.)

A (BC)

7.6.5.3 Technology

For the following exercises, use the matrices below to perform the indicated operation if possible. If notpossible, explain why the operation cannot be performed. Use a calculator to verify your solution.

A =

−2 0 9

1 8 −3

0.5 4 5

, B =

0.5 3 0

−4 1 6

8 7 2

, C =

1 0 1

0 1 0

1 0 1

(7.229)

Exercise 7.6.52AB

Exercise 7.6.53 (Solution on p. 795.)

BA

Exercise 7.6.54CA

Available for free at Connexions <http://legacy.cnx.org/content/col11759/1.2>

705

Exercise 7.6.55 (Solution on p. 795.)

BC

Exercise 7.6.56ABC

7.6.5.4 Extensions

For the following exercises, use the matrix below to perform the indicated operation on the given matrix.

B =

1 0 0

0 0 1

0 1 0

(7.230)

Exercise 7.6.57 (Solution on p. 795.)

B2

Exercise 7.6.58B3

Exercise 7.6.59 (Solution on p. 795.)

B4

Exercise 7.6.60B5

Exercise 7.6.61 (Solution on p. 795.)

Using the above questions, �nd a formula forBn.Test the formula forB201 andB202, using a calcu-lator.

Available for free at Connexions <http://legacy.cnx.org/content/col11759/1.2>

706 CHAPTER 7. SYSTEMS OF EQUATIONS AND INEQUALITIES

7.7 Solving Systems with Gaussian Elimination26

Figure 7.30: German mathematician Carl Friedrich Gauss (1777�1855).

26This content is available online at <http://legacy.cnx.org/content/m49434/1.3/>.

Available for free at Connexions <http://legacy.cnx.org/content/col11759/1.2>

707

Carl Friedrich Gauss lived during the late 18th century and early 19th century, but he is still considered oneof the most proli�c mathematicians in history. His contributions to the science of mathematics and physicsspan �elds such as algebra, number theory, analysis, di�erential geometry, astronomy, and optics, amongothers. His discoveries regarding matrix theory changed the way mathematicians have worked for the lasttwo centuries.

We �rst encountered Gaussian elimination in Systems of Linear Equations: Two Variables (Section 7.3).In this section, we will revisit this technique for solving systems, this time using matrices.

7.7.1 Writing the Augmented Matrix of a System of Equations

A matrix can serve as a device for representing and solving a system of equations. To express a system inmatrix form, we extract the coe�cients of the variables and the constants, and these become the entries ofthe matrix. We use a vertical line to separate the coe�cient entries from the constants, essentially replacingthe equal signs. When a system is written in this form, we call it an augmented matrix.

For example, consider the following 2 × 2 system of equations.

3x+ 4y = 7

4x− 2y = 5(7.231)

We can write this system as an augmented matrix: 3 4

4 −2|

7

5

(7.232)

We can also write a matrix containing just the coe�cients. This is called the coe�cient matrix. 3 4

4 −2

(7.233)

A three-by-three system of equations such as

3x− y − z = 0

x+ y = 5

2x− 3z = 2

(7.234)

has a coe�cient matrix 3 −1 −1

1 1 0

2 0 −3

(7.235)

and is represented by the augmented matrix3 −1 −1

1 1 0

2 0 −3

|0

5

2

(7.236)

Notice that the matrix is written so that the variables line up in their own columns: x-terms go in the�rst column, y-terms in the second column, and z-terms in the third column. It is very important that each

Available for free at Connexions <http://legacy.cnx.org/content/col11759/1.2>

708 CHAPTER 7. SYSTEMS OF EQUATIONS AND INEQUALITIES

equation is written in standard form ax+ by + cz = d so that the variables line up. When there is a missingvariable term in an equation, the coe�cient is 0.

How To: Given a system of equations, write an augmented matrix.

1.Write the coe�cients of the x-terms as the numbers down the �rst column.2.Write the coe�cients of the y-terms as the numbers down the second column.3.If there are z-terms, write the coe�cients as the numbers down the third column.4.Draw a vertical line and write the constants to the right of the line.

Example 7.37Writing the Augmented Matrix for a System of EquationsWrite the augmented matrix for the given system of equations.

x+ 2y − z = 3

2x− y + 2z = 6

x− 3y + 3z = 4

(7.237)

Solution

The augmented matrix displays the coe�cients of the variables, and an additional column for theconstants.

1 2 −1

2 −1 2

1 −3 3

|3

6

4

(7.238)

Try It:

Exercise 7.7.1 (Solution on p. 796.)

Write the augmented matrix of the given system of equations.

4x− 3y = 11

3x+ 2y = 4(7.239)

7.7.2 Writing a System of Equations from an Augmented Matrix

We can use augmented matrices to help us solve systems of equations because they simplify operations whenthe systems are not encumbered by the variables. However, it is important to understand how to move backand forth between formats in order to make �nding solutions smoother and more intuitive. Here, we willuse the information in an augmented matrix to write the system of equations in standard form.

Example 7.38Writing a System of Equations from an Augmented Matrix FormFind the system of equations from the augmented matrix.

Available for free at Connexions <http://legacy.cnx.org/content/col11759/1.2>

709

1 −3 −5

2 −5 −4

−3 5 4

|−2

5

6

(7.240)

Solution

When the columns represent the variablesx, y, and z,1 −3 −5

2 −5 −4

−3 5 4

|−2

5

6

→x− 3y − 5z = −2

2x− 5y − 4z = 5

−3x+ 5y + 4z = 6

(7.241)

Try It:

Exercise 7.7.2 (Solution on p. 796.)

Write the system of equations from the augmented matrix.

1 −1 1

2 −1 3

0 1 1

|5

1

−9

(7.242)

7.7.3 Performing Row Operations on a Matrix

Now that we can write systems of equations in augmented matrix form, we will examine the various rowoperations that can be performed on a matrix, such as addition, multiplication by a constant, and inter-changing rows.

Performing row operations on a matrix is the method we use for solving a system of equations. In orderto solve the system of equations, we want to convert the matrix to row-echelon form, in which there areones down the main diagonal from the upper left corner to the lower right corner, and zeros in everyposition below the main diagonal as shown.

Row-echelon form1 a b

0 1 d

0 0 1

(7.243)

We use row operations corresponding to equation operations to obtain a new matrix that is row-equivalentin a simpler form. Here are the guidelines to obtaining row-echelon form.

1. In any nonzero row, the �rst nonzero number is a 1. It is called a leading 1.2. Any all-zero rows are placed at the bottom on the matrix.

Available for free at Connexions <http://legacy.cnx.org/content/col11759/1.2>

710 CHAPTER 7. SYSTEMS OF EQUATIONS AND INEQUALITIES

3. Any leading 1 is below and to the right of a previous leading 1.4. Any column containing a leading 1 has zeros in all other positions in the column.

To solve a system of equations we can perform the following row operations to convert the coe�cientmatrix to row-echelon form and do back-substitution to �nd the solution.

1. Interchange rows. (Notation:Ri ↔ Rj)2. Multiply a row by a constant. (Notation: cRi)3. Add the product of a row multiplied by a constant to another row. (Notation:Ri + cRj

Each of the row operations corresponds to the operations we have already learned to solve systems ofequations in three variables. With these operations, there are some key moves that will quickly achieve thegoal of writing a matrix in row-echelon form. To obtain a matrix in row-echelon form for �nding solutions,we use Gaussian elimination, a method that uses row operations to obtain a 1 as the �rst entry so that row1 can be used to convert the remaining rows.

A General Note: The Gaussian elimination method refers to a strategy used to obtain therow-echelon form of a matrix. The goal is to write matrixAwith the number 1 as the entry downthe main diagonal and have all zeros below.

A =

a11 a12 a13

a21 a22 a23

a31 a32 a33

After Gaussian elimination→ A =

1 b12 b13

0 1 b23

0 0 1

(7.244)

The �rst step of the Gaussian strategy includes obtaining a 1 as the �rst entry, so that row 1 maybe used to alter the rows below.

How To: Given an augmented matrix, perform row operations to achieve row-echelonform.

1.The �rst equation should have a leading coe�cient of 1. Interchange rows or multiply by aconstant, if necessary.

2.Use row operations to obtain zeros down the �rst column below the �rst entry of 1.3.Use row operations to obtain a 1 in row 2, column 2.4.Use row operations to obtain zeros down column 2, below the entry of 1.5.Use row operations to obtain a 1 in row 3, column 3.6.Continue this process for all rows until there is a 1 in every entry down the main diagonal andthere are only zeros below.

7.If any rows contain all zeros, place them at the bottom.

Example 7.39Solving a 2× 2 System by Gaussian EliminationSolve the given system by Gaussian elimination.

2x+ 3y = 6

x− y = 12

(7.245)

Solution

First, we write this as an augmented matrix.

Available for free at Connexions <http://legacy.cnx.org/content/col11759/1.2>

711

2 3

1 −1|

612

(7.246)

We want a 1 in row 1, column 1. This can be accomplished by interchanging row 1 and row 2.

R1 ↔ R2 →

1 −1

2 3|

12

6

(7.247)

We now have a 1 as the �rst entry in row 1, column 1. Now let's obtain a 0 in row 2, column 1.This can be accomplished by multiplying row 1 by − 2,and then adding the result to row 2.

−2R1 +R2 = R2 →

1 −1

0 5|

12

5

(7.248)

We only have one more step, to multiply row 2 by 15 .

15R2 = R2 →

1 −1

0 1|

12

1

(7.249)

Use back-substitution. The second row of the matrix represents y = 1.Back-substitute y = 1 intothe �rst equation.

x− (1) = 12

x = 32

(7.250)

The solution is the point(

32 , 1).

Try It:

Exercise 7.7.3 (Solution on p. 796.)

Solve the given system by Gaussian elimination.

4x+ 3y = 11

x− 3y = −1(7.251)

Example 7.40Using Gaussian Elimination to Solve a System of EquationsUse Gaussian elimination to solve the given 2 × 2 system of equations.

2x+ y = 1

4x+ 2y = 6(7.252)

Solution

Available for free at Connexions <http://legacy.cnx.org/content/col11759/1.2>

712 CHAPTER 7. SYSTEMS OF EQUATIONS AND INEQUALITIES

Write the system as an augmented matrix. 2 1

4 2|

1

6

(7.253)

Obtain a 1 in row 1, column 1. This can be accomplished by multiplying the �rst row by 12 .

12R1 = R1 →

1 12

4 2|

12

6

(7.254)

Next, we want a 0 in row 2, column 1. Multiply row 1 by − 4 and add row 1 to row 2.

−4R1 +R2 = R2 →

1 12

0 0|

12

4

(7.255)

The second row represents the equation 0 = 4.Therefore, the system is inconsistent and has nosolution.

Example 7.41Solving a Dependent SystemSolve the system of equations.

3x+ 4y = 12

6x+ 8y = 24(7.256)

Solution

Perform row operations on the augmented matrix to try and achieve row-echelon form.

A =

3 4

6 8|

12

24

(7.257)

− 12R2 +R1 = R1 →

0 0

6 8|

0

24

R1 ↔ R2 →

6 8

0 0|

24

0

(7.258)

The matrix ends up with all zeros in the last row: 0y = 0.Thus, there are an in�nite number ofsolutions and the system is classi�ed as dependent. To �nd the generic solution, return to one ofthe original equations and solve for y.

Available for free at Connexions <http://legacy.cnx.org/content/col11759/1.2>

713

3x+ 4y = 12

4y = 12− 3x

y = 3− 34x

(7.259)

So the solution to this system is(x, 3− 3

4x).

Example 7.42Performing Row Operations on a 3×3 Augmented Matrix to Obtain Row-EchelonFormPerform row operations on the given matrix to obtain row-echelon form.

1 −3 4

2 −5 6

−3 3 4

|3

6

6

(7.260)

Solution

The �rst row already has a 1 in row 1, column 1. The next step is to multiply row 1 by − 2 andadd it to row 2. Then replace row 2 with the result.

−2R1 +R2 = R2 →

1 −3 4

0 1 −2

−3 3 4

|3

0

6

(7.261)

Next, obtain a zero in row 3, column 1.

3R1 +R3 = R3 →

1 −3 4

0 1 −2

0 −6 16

|3

0

15

(7.262)

Next, obtain a zero in row 3, column 2.

6R2 +R3 = R3 →

1 −3 4

0 1 −2

0 0 4

|3

0

15

(7.263)

The last step is to obtain a 1 in row 3, column 3.

12R3 = R3 →

1 −3 4

0 1 −2

0 0 1

|3

−6212

(7.264)

Available for free at Connexions <http://legacy.cnx.org/content/col11759/1.2>

714 CHAPTER 7. SYSTEMS OF EQUATIONS AND INEQUALITIES

Try It:

Exercise 7.7.4 (Solution on p. 796.)

Write the system of equations in row-echelon form.

x− 2y + 3z = 9

− x+ 3y = −4

2x− 5y + 5z = 17

(7.265)

7.7.4 Solving a System of Linear Equations Using Matrices

We have seen how to write a system of equations with an augmented matrix, and then how to use rowoperations and back-substitution to obtain row-echelon form. Now, we will take row-echelon form a stepfarther to solve a 3 by 3 system of linear equations. The general idea is to eliminate all but one variableusing row operations and then back-substitute to solve for the other variables.

Example 7.43Solving a System of Linear Equations Using MatricesSolve the system of linear equations using matrices.

x − y + z = 8

2x + 3y − z = −2

3x − 2y − 9z = 9

(7.266)

Solution

First, we write the augmented matrix.1 −1 1

2 3 −1

3 −2 −9

|8

−2

9

(7.267)

Next, we perform row operations to obtain row-echelon form.

−2R1 + R2 = R2 →

1 −1 1

0 5 −3

3 −2 −9

|8

−18

9

−3R1 + R3 = R3 →

1 −1 1

0 5 −3

0 1 −12

|8

−18

−15

(7.268)

The easiest way to obtain a 1 in row 2 of column 1 is to interchangeR2 andR3.

Available for free at Connexions <http://legacy.cnx.org/content/col11759/1.2>

715

InterchangeR2 andR3 →

1 −1 1 8

0 1 −12 −15

0 5 −3 −18

(7.269)

Then

−5R2 + R3 = R3 →

1 −1 1

0 1 −12

0 0 57

|8

−15

57

− 157

R3 = R3 →

1 −1 1

0 1 −12

0 0 1

|8

−15

1

(7.270)

The last matrix represents the equivalent system.

x− y + z = 8

y − 12z = −15

z = 1

(7.271)

Using back-substitution, we obtain the solution as (4,−3, 1) .

Example 7.44Solving a Dependent System of Linear Equations Using MatricesSolve the following system of linear equations using matrices.

−x− 2y + z = −1

2x+ 3y = 2

y − 2z = 0

(7.272)

Solution

Write the augmented matrix. −1 −2 1

2 3 0

0 1 −2

|−1

2

0

(7.273)

First, multiply row 1 by − 1 to get a 1 in row 1, column 1. Then, perform row operations toobtain row-echelon form.

−R1 →

1 2 −1 1

2 3 0 2

0 1 −2 0

(7.274)

Available for free at Connexions <http://legacy.cnx.org/content/col11759/1.2>

716 CHAPTER 7. SYSTEMS OF EQUATIONS AND INEQUALITIES

R2 ↔ R3 →

1 2 −1

0 1 −2

2 3 0

|1

0

2

(7.275)

−2R1 +R3 = R3 →

1 2 −1

0 1 −2

0 −1 2

|1

0

0

(7.276)

R2 +R3 = R3 →

1 2 −1

0 1 −2

0 0 0

|2

1

0

(7.277)

The last matrix represents the following system.

x+ 2y − z = 1

y − 2z = 0

0 = 0

(7.278)

We see by the identity 0 = 0 that this is a dependent system with an in�nite number of solutions.We then �nd the generic solution. By solving the second equation for y and substituting it into the�rst equation we can solve for z in terms ofx.

x+ 2y − z = 1

y = 2z

x+ 2 (2z)− z = 1

x+ 3z = 1

z = 1−x3

(7.279)

Now we substitute the expression for z into the second equation to solve for y in terms ofx.

y − 2z = 0

z = 1−x3

y − 2(

1−x3

)= 0

y = 2−2x3

(7.280)

The generic solution is(x, 2−2x

3 , 1−x3

).

Available for free at Connexions <http://legacy.cnx.org/content/col11759/1.2>

717

Try It:

Exercise 7.7.5 (Solution on p. 796.)

Solve the system using matrices.

x+ 4y − z = 4

2x+ 5y + 8z = 15

x+ 3y − 3z = 1

(7.281)

Q&A: Can any system of linear equations be solved by Gaussian elimination?

Yes, a system of linear equations of any size can be solved by Gaussian elimination.

How To: Given a system of equations, solve with matrices using a calculator.

1.Save the augmented matrix as a matrix variable [A] , [B] , [C] , . . . .2.Use the ref( function in the calculator, calling up each matrix variable as needed.

Example 7.45Solving Systems of Equations with Matrices Using a CalculatorSolve the system of equations.

5x+ 3y + 9z = −1

−2x+ 3y − z = −2

−x− 4y + 5z = 1

(7.282)

Solution

Write the augmented matrix for the system of equations.5 3 9

−2 3 −1

−1 −4 5

|5

−2

−1

(7.283)

On the matrix page of the calculator, enter the augmented matrix above as the matrix variable [A] .

[A] =

5 3 9 −1

−2 3 −1 −2

−1 −4 5 1

(7.284)

Use the ref( function in the calculator, calling up the matrix variable [A] .

ref ([A]) (7.285)

Evaluate.

Available for free at Connexions <http://legacy.cnx.org/content/col11759/1.2>

718 CHAPTER 7. SYSTEMS OF EQUATIONS AND INEQUALITIES

1 3

595

15

0 1 1321 − 4

7

0 0 1 − 24187

→x+ 3

5y + 95z = − 1

5

y + 1321z = − 4

7

z = − 24187

(7.286)

Using back-substitution, the solution is(

61187 ,−

92187 ,−

24187

).

Example 7.46Applying 2 × 2 Matrices to FinanceCarolyn invests a total of $12,000 in two municipal bonds, one paying 10.5% interest and the otherpaying 12% interest. The annual interest earned on the two investments last year was $1,335. Howmuch was invested at each rate?

SolutionWe have a system of two equations in two variables. Letx = the amount invested at 10.5% interest,and y = the amount invested at 12% interest.

x+ y = 12, 000

0.105x+ 0.12y = 1, 335(7.287)

As a matrix, we have 1 1

0.105 0.12|

12, 000

1, 335

(7.288)

Multiply row 1 by − 0.105 and add the result to row 2. 1 1

0 0.015|

12, 000

75

(7.289)

Then,

0.015y = 75

y = 5, 000(7.290)

So 12, 000− 5, 000 = 7, 000.Thus, $5,000 was invested at 12% interest and $7,000 at 10.5% interest.

Example 7.47Applying 3 × 3 Matrices to FinanceAva invests a total of $10,000 in three accounts, one paying 5% interest, another paying 8% interest,and the third paying 9% interest. The annual interest earned on the three investments last year was$770. The amount invested at 9% was twice the amount invested at 5%. How much was investedat each rate?

Available for free at Connexions <http://legacy.cnx.org/content/col11759/1.2>

719

SolutionWe have a system of three equations in three variables. Letx be the amount invested at 5% interest,let y be the amount invested at 8% interest, and let z be the amount invested at 9% interest. Thus,

x+ y + z = 10, 000

0.05x+ 0.08y + 0.09z = 770

2x− z = 0

(7.291)

As a matrix, we have 1 1 1

0.05 0.08 0.09

2 0 −1

|10, 000

770

0

(7.292)

Now, we perform Gaussian elimination to achieve row-echelon form.

−0.05R1 +R2 = R2 →

1 1 1

0 0.03 0.04

2 0 −1

|10, 000

270

0

− 2R1 +R3 = R3 →

1 1 1

0 0.03 0.04

0 −2 −3

|10, 000

270

−20, 000

1

0.03R2 = R2 →

0 1 1

0 1 43

0 −2 −3

|10, 000

9, 000

−20, 000

2R2 +R3 = R3 →

1 1 1

0 1 43

0 0 − 13

|10, 000

9, 000

−2, 000

(7.293)

The third row tells us − 13z = −2, 000; thus z = 6, 000.

The second row tells us y + 43z = 9, 000. Substituting z = 6, 000,we get

y + 43 (6, 000) = 9, 000

y + 8, 000 = 9, 000

y = 1, 000

(7.294)

The �rst row tells usx+ y + z = 10, 000. Substituting y = 1, 000 and z = 6, 000,we get

x+ 1, 000 + 6, 000 = 10, 000

x = 3, 000(7.295)

The answer is $3,000 invested at 5% interest, $1,000 invested at 8%, and $6,000 invested at 9%interest.

Available for free at Connexions <http://legacy.cnx.org/content/col11759/1.2>

720 CHAPTER 7. SYSTEMS OF EQUATIONS AND INEQUALITIES

Try It:

Exercise 7.7.6 (Solution on p. 796.)

A small shoe company took out a loan of $1,500,000 to expand their inventory. Part ofthe money was borrowed at 7%, part was borrowed at 8%, and part was borrowed at 10%.The amount borrowed at 10% was four times the amount borrowed at 7%, and the annualinterest on all three loans was $130,500. Use matrices to �nd the amount borrowed ateach rate.

Media: Access these online resources for additional instruction and practice with solving systemsof linear equations using Gaussian elimination.

• Solve a System of Two Equations Using an Augmented Matrix27

• Solve a System of Three Equations Using an Augmented Matrix28

• Augmented Matrices on the Calculator29

7.7.5 Key Concepts

• An augmented matrix is one that contains the coe�cients and constants of a system of equations. SeeExample 7.37.

• A matrix augmented with the constant column can be represented as the original system of equations.See Example 7.38.

• Row operations include multiplying a row by a constant, adding one row to another row, and inter-changing rows.

• We can use Gaussian elimination to solve a system of equations. See Example 7.39, Example 7.40, andExample 7.41.

• Row operations are performed on matrices to obtain row-echelon form. See Example 7.42.• To solve a system of equations, write it in augmented matrix form. Perform row operations to obtain

row-echelon form. Back-substitute to �nd the solutions. See Example 7.43 and Example 7.44.• A calculator can be used to solve systems of equations using matrices. See Example 7.45.• Many real-world problems can be solved using augmented matrices. See Example 7.46 and Exam-

ple 7.47.

7.7.6 Section Exercises

7.7.6.1 Verbal

Exercise 7.7.7 (Solution on p. 796.)

Can any system of linear equations be written as an augmented matrix? Explain why or why not.Explain how to write that augmented matrix.

Exercise 7.7.8Can any matrix be written as a system of linear equations? Explain why or why not. Explain howto write that system of equations.

27http://openstaxcollege.org/l/system2augmat28http://openstaxcollege.org/l/system3augmat29http://openstaxcollege.org/l/augmatcalc

Available for free at Connexions <http://legacy.cnx.org/content/col11759/1.2>

721

Exercise 7.7.9 (Solution on p. 796.)

Is there only one correct method of using row operations on a matrix? Try to explain two di�erent

row operations possible to solve the augmented matrix

9 3

1 −2|

0

6

.Exercise 7.7.10Can a matrix whose entry is 0 on the diagonal be solved? Explain why or why not. What wouldyou do to remedy the situation?

Exercise 7.7.11 (Solution on p. 796.)

Can a matrix that has 0 entries for an entire row have one solution? Explain why or why not.

7.7.6.2 Algebraic

For the following exercises, write the augmented matrix for the linear system.

Exercise 7.7.12

8x− 37y = 8

2x+ 12y = 3

Exercise 7.7.13 (Solution on p. 796.)

16y = 4

9x− y = 2

Exercise 7.7.14

3x+ 2y + 10z = 3

−6x+ 2y + 5z = 13

4x+ z = 18

Exercise 7.7.15 (Solution on p. 796.)

x+ 5y + 8z = 19

12x+ 3y = 4

3x+ 4y + 9z = −7

Exercise 7.7.16

6x+ 12y + 16z = 4

19x− 5y + 3z = −9

x+ 2y = −8

For the following exercises, write the linear system from the augmented matrix.

Exercise 7.7.17 (Solution on p. 796.) −2 5

6 −18|

5

26

Exercise 7.7.18 3 4

10 17|

10

439

Available for free at Connexions <http://legacy.cnx.org/content/col11759/1.2>

722 CHAPTER 7. SYSTEMS OF EQUATIONS AND INEQUALITIES

Exercise 7.7.19 (Solution on p. 797.)3 2 0

−1 −9 4

8 5 7

|3

−1

8

Exercise 7.7.20

8 29 1

−1 7 5

0 0 3

|43

38

10

Exercise 7.7.21 (Solution on p. 797.)

4 5 −2

0 1 58

8 7 −3

|12

2

−5

For the following exercises, solve the system by Gaussian elimination.

Exercise 7.7.22 1 0

0 0|

3

0

Exercise 7.7.23 (Solution on p. 797.) 1 0

1 0|

1

2

Exercise 7.7.24 1 2

4 5|

3

6

Exercise 7.7.25 (Solution on p. 797.) −1 2

4 −5|−3

6

Exercise 7.7.26 −2 0

0 2|

1

−1

Exercise 7.7.27 (Solution on p. 797.)

2x− 3y = −9

5x+ 4y = 58

Exercise 7.7.28

6x+ 2y = −4

3x+ 4y = −17

Exercise 7.7.29 (Solution on p. 797.)

2x+ 3y = 12

4x+ y = 14

Available for free at Connexions <http://legacy.cnx.org/content/col11759/1.2>

723

Exercise 7.7.30

−4x− 3y = −2

3x− 5y = −13

Exercise 7.7.31 (Solution on p. 797.)

−5x+ 8y = 3

10x+ 6y = 5

Exercise 7.7.32

3x+ 4y = 12

−6x− 8y = −24

Exercise 7.7.33 (Solution on p. 797.)

−60x+ 45y = 12

20x− 15y = −4

Exercise 7.7.34

11x+ 10y = 43

15x+ 20y = 65

Exercise 7.7.35 (Solution on p. 797.)

2x− y = 2

3x+ 2y = 17

Exercise 7.7.36

−1.06x− 2.25y = 5.51

−5.03x− 1.08y = 5.40

Exercise 7.7.37 (Solution on p. 797.)34x−

35y = 4

14x+ 2

3y = 1

Exercise 7.7.3814x−

23y = −1

12x+ 1

3y = 3

Exercise 7.7.39 (Solution on p. 797.)1 0 0

0 1 1

0 0 1

|31

45

87

Exercise 7.7.40

1 0 1

1 1 0

0 1 1

|50

20

−90

Available for free at Connexions <http://legacy.cnx.org/content/col11759/1.2>

724 CHAPTER 7. SYSTEMS OF EQUATIONS AND INEQUALITIES

Exercise 7.7.41 (Solution on p. 797.)1 2 3

0 5 6

0 0 8

|4

7

9

Exercise 7.7.42−0.1 0.3 −0.1

−0.4 0.2 0.1

0.6 0.1 0.7

|0.2

0.8

−0.8

Exercise 7.7.43 (Solution on p. 797.)

− 2x+ 3y − 2z = 3

4x+ 2y − z = 9

4x− 8y + 2z = −6

Exercise 7.7.44

x+ y − 4z = −4

5x− 3y − 2z = 0

2x+ 6y + 7z = 30

Exercise 7.7.45 (Solution on p. 797.)

2x+ 3y + 2z = 1

− 4x− 6y − 4z = −2

10x+ 15y + 10z = 5

Exercise 7.7.46

x+ 2y − z = 1

−x− 2y + 2z = −2

3x+ 6y − 3z = 5

Exercise 7.7.47 (Solution on p. 797.)

x+ 2y − z = 1

−x− 2y + 2z = −2

3x+ 6y − 3z = 3

Exercise 7.7.48

x+ y = 2

x+ z = 1

−y − z = −3

Exercise 7.7.49 (Solution on p. 797.)

x+ y + z = 100

x+ 2z = 125

−y + 2z = 25

Available for free at Connexions <http://legacy.cnx.org/content/col11759/1.2>

725

Exercise 7.7.5014x−

23z = − 1

2

15x+ 1

3y = 47

15y −

13z = 2

9

Exercise 7.7.51 (Solution on p. 797.)

− 12x+ 1

2y + 17z = − 53

14

12x−

12y + 1

4z = 314x+ 1

5y + 13z = 23

15

Exercise 7.7.52

− 12x−

13y + 1

4z = − 296

15x+ 1

6y −17z = 431

210

− 18x+ 1

9y + 110z = − 49

45

7.7.6.3 Extensions

For the following exercises, use Gaussian elimination to solve the system.

Exercise 7.7.53 (Solution on p. 797.)x−1

7 + y−28 + z−3

4 = 0

x+ y + z = 6x+2

3 + 2y + z−33 = 5

Exercise 7.7.54x−1

4 −y+1

4 + 3z = −1x+5

2 + y+74 − z = 4

x+ y − z−22 = 1

Exercise 7.7.55 (Solution on p. 797.)x−3

4 −y−1

3 + 2z = −1x+5

2 + y+52 + z+5

2 = 8

x+ y + z = 1

Exercise 7.7.56x−310 + y+3

2 − 2z = 3x+5

4 −y−1

8 + z = 32

x−14 + y+4

2 + 3z = 32

Exercise 7.7.57 (Solution on p. 797.)x−3

4 −y−1

3 + 2z = −1x+5

2 + y+52 + z+5

2 = 7

x+ y + z = 1

Available for free at Connexions <http://legacy.cnx.org/content/col11759/1.2>

726 CHAPTER 7. SYSTEMS OF EQUATIONS AND INEQUALITIES

7.7.6.4 Real-World Applications

For the following exercises, set up the augmented matrix that describes the situation, and solve for thedesired solution.

Exercise 7.7.58Every day, a cupcake store sells 5,000 cupcakes in chocolate and vanilla �avors. If the chocolate�avor is 3 times as popular as the vanilla �avor, how many of each cupcake sell per day?

Exercise 7.7.59 (Solution on p. 797.)

At a competing cupcake store, $4,520 worth of cupcakes are sold daily. The chocolate cupcakescost $2.25 and the red velvet cupcakes cost $1.75. If the total number of cupcakes sold per day is2,200, how many of each �avor are sold each day?

Exercise 7.7.60You invested $10,000 into two accounts: one that has simple 3% interest, the other with 2.5%interest. If your total interest payment after one year was $283.50, how much was in each accountafter the year passed?

Exercise 7.7.61 (Solution on p. 798.)

You invested $2,300 into account 1, and $2,700 into account 2. If the total amount of interest afterone year is $254, and account 2 has 1.5 times the interest rate of account 1, what are the interestrates? Assume simple interest rates.

Exercise 7.7.62Bikes'R'Us manufactures bikes, which sell for $250. It costs the manufacturer $180 per bike, plusa startup fee of $3,500. After how many bikes sold will the manufacturer break even?

Exercise 7.7.63 (Solution on p. 798.)

A major appliance store is considering purchasing vacuums from a small manufacturer. The storewould be able to purchase the vacuums for $86 each, with a delivery fee of $9,200, regardless ofhow many vacuums are sold. If the store needs to start seeing a pro�t after 230 units are sold, howmuch should they charge for the vacuums?

Exercise 7.7.64The three most popular ice cream �avors are chocolate, strawberry, and vanilla, comprising 83% ofthe �avors sold at an ice cream shop. If vanilla sells 1% more than twice strawberry, and chocolatesells 11% more than vanilla, how much of the total ice cream consumption are the vanilla, chocolate,and strawberry �avors?

Exercise 7.7.65 (Solution on p. 798.)

At an ice cream shop, three �avors are increasing in demand. Last year, banana, pumpkin, androcky road ice cream made up 12% of total ice cream sales. This year, the same three ice creamsmade up 16.9% of ice cream sales. The rocky road sales doubled, the banana sales increased by50%, and the pumpkin sales increased by 20%. If the rocky road ice cream had one less percentof sales than the banana ice cream, �nd out the percentage of ice cream sales each individual icecream made last year.

Exercise 7.7.66A bag of mixed nuts contains cashews, pistachios, and almonds. There are 1,000 total nuts in thebag, and there are 100 less almonds than pistachios. The cashews weigh 3 g, pistachios weigh 4 g,and almonds weigh 5 g. If the bag weighs 3.7 kg, �nd out how many of each type of nut is in thebag.

Exercise 7.7.67 (Solution on p. 798.)

A bag of mixed nuts contains cashews, pistachios, and almonds. Originally there were 900 nuts inthe bag. 30% of the almonds, 20% of the cashews, and 10% of the pistachios were eaten, and nowthere are 770 nuts left in the bag. Originally, there were 100 more cashews than almonds. Figureout how many of each type of nut was in the bag to begin with.

Available for free at Connexions <http://legacy.cnx.org/content/col11759/1.2>

727

7.8 Solving Systems with Inverses30

Nancy plans to invest $10,500 into two di�erent bonds to spread out her risk. The �rst bond has an annualreturn of 10%, and the second bond has an annual return of 6%. In order to receive an 8.5% return from thetwo bonds, how much should Nancy invest in each bond? What is the best method to solve this problem?

There are several ways we can solve this problem. As we have seen in previous sections, systems ofequations and matrices are useful in solving real-world problems involving �nance. After studying thissection, we will have the tools to solve the bond problem using the inverse of a matrix.

7.8.1 Finding the Inverse of a Matrix

We know that the multiplicative inverse of a real number a is a−1, and aa−1 = a−1a =(

1a

)a = 1.For

example, 2−1 = 12 and

(12

)2 = 1.The multiplicative inverse of a matrix is similar in concept, ex-

cept that the product of matrixA and its inverseA−1 equals the identity matrix. The identity matrix isa square matrix containing ones down the main diagonal and zeros everywhere else. We identify identitymatrices by In wheren represents the dimension of the matrix. (7.296) and (7.297) are the identity matricesfor a 2× 2matrix and a 3× 3matrix, respectively.

I2 =

1 0

0 1

(7.296)

I3 =

1 0 0

0 1 0

0 0 1

(7.297)

The identity matrix acts as a 1 in matrix algebra. For example,AI = IA = A.A matrix that has a multiplicative inverse has the properties

AA−1 = I

A−1A = I(7.298)

A matrix that has a multiplicative inverse is called an invertible matrix. Only a square matrix may havea multiplicative inverse, as the reversibility,AA−1 = A−1A = I, is a requirement. Not all square matriceshave an inverse, but ifA is invertible, thenA−1 is unique. We will look at two methods for �nding the inverseof a 2 × 2matrix and a third method that can be used on both 2 × 2 and3 × 3matrices.

A General Note: The identity matrix, In, is a square matrix containing ones down the maindiagonal and zeros everywhere else.

I2 =

1 0

0 1

I3 =

1 0 0

0 1 0

0 0 1

2 × 2 3 × 3

(7.299)

30This content is available online at <http://legacy.cnx.org/content/m49435/1.4/>.

Available for free at Connexions <http://legacy.cnx.org/content/col11759/1.2>

728 CHAPTER 7. SYSTEMS OF EQUATIONS AND INEQUALITIES

IfA is ann × n matrix andB is ann × n matrix such thatAB = BA = In, thenB = A−1, themultiplicative inverse of a matrixA.

Example 7.48Showing That the Identity Matrix Acts as a 1Given matrix A, show thatAI = IA = A.

A =

3 4

−2 5

(7.300)

Solution

Use matrix multiplication to show that the product ofA and the identity is equal to the productof the identity and A.

AI =

3 4

−2 5

1 0

0 1

=

3 · 1 + 4 · 0 3 · 0 + 4 · 1−2 · 1 + 5 · 0 −2 · 0 + 5 · 1

=

3 4

−2 5

(7.301)

AI =

1 0

0 1

3 4

−2 5

=

1 · 3 + 0 · (−2) 1 · 4 + 0 · 50 · 3 + 1 · (−2) 0 · 4 + 1 · 5

=

3 4

−2 5

(7.302)

How To: Given two matrices, show that one is the multiplicative inverse of the other.

1.Given matrixA of ordern × n and matrixB of ordern × nmultiplyAB.2.IfAB = I, then �nd the productBA. IfBA = I, thenB = A−1 andA = B−1.

Example 7.49Showing That Matrix A Is the Multiplicative Inverse of Matrix BShow that the given matrices are multiplicative inverses of each other.

A =

1 5

−2 −9

, B =

−9 −5

2 1

(7.303)

Solution

MultiplyAB andBA. If both products equal the identity, then the two matrices are inverses of eachother.

AB =

1 5

−2 −9

· −9 −5

2 1

=

1 (−9) + 5 (2) 1 (−5) + 5 (1)

−2 (−9)− 9 (2) −2 (−5)− 9 (1)

=

1 0

0 1

(7.304)

Available for free at Connexions <http://legacy.cnx.org/content/col11759/1.2>

729

BA =

−9 −5

2 1

· 1 5

−2 −9

=

−9 (1)− 5 (−2) −9 (5)− 5 (−9)

2 (1) + 1 (−2) 2 (−5) + 1 (−9)

=

1 0

0 1

(7.305)

A andBare inverses of each other.

Try It:

Exercise 7.8.1 (Solution on p. 798.)

Show that the following two matrices are inverses of each other.

A =

1 4

−1 −3

, B =

−3 −4

1 1

(7.306)

7.8.1.1 Finding the Multiplicative Inverse Using Matrix Multiplication

We can now determine whether two matrices are inverses, but how would we �nd the inverse of a givenmatrix? Since we know that the product of a matrix and its inverse is the identity matrix, we can �nd theinverse of a matrix by setting up an equation using matrix multiplication.

Example 7.50Finding the Multiplicative Inverse Using Matrix MultiplicationUse matrix multiplication to �nd the inverse of the given matrix.

A =

1 −2

2 −3

(7.307)

Solution

For this method, we multiplyA by a matrix containing unknown constants and set it equal to theidentity. 1 −2

2 −3

a b

c d

=

1 0

0 1

(7.308)

Find the product of the two matrices on the left side of the equal sign.

Available for free at Connexions <http://legacy.cnx.org/content/col11759/1.2>

730 CHAPTER 7. SYSTEMS OF EQUATIONS AND INEQUALITIES

1 −2

2 −3

a b

c d

=

1a− 2c 1b− 2d

2a− 3c 2b− 3d

(7.309)

Next, set up a system of equations with the entry in row 1, column 1 of the new matrix equal tothe �rst entry of the identity, 1. Set the entry in row 2, column 1 of the new matrix equal to thecorresponding entry of the identity, which is 0.

1a− 2c = 1 R1

2a− 3c = 0 R2

(7.310)

Using row operations, multiply and add as follows: (−2)R1 + R2 → R2.Add the equations, andsolve for c.

1a− 2c = 1

0 + 1c = −2

c = −2

(7.311)

Back-substitute to solve for a.

a− 2 (−2) = 1

a+ 4 = 1

a = −3

(7.312)

Write another system of equations setting the entry in row 1, column 2 of the new matrix equalto the corresponding entry of the identity, 0. Set the entry in row 2, column 2 equal to thecorresponding entry of the identity.

1b− 2d = 0 R1

2b− 3d = 1 R2

(7.313)

Using row operations, multiply and add as follows: (−2)R1 +R2 = R2.Add the two equations andsolve for d.

1b− 2d = 00+1d=1

d=1(7.314)

Once more, back-substitute and solve for b.

b− 2 (1) = 0

b− 2 = 0

b = 2

(7.315)

A−1 =

−3 2

−2 1

(7.316)

Available for free at Connexions <http://legacy.cnx.org/content/col11759/1.2>

731

7.8.1.2 Finding the Multiplicative Inverse by Augmenting with the Identity

Another way to �nd the multiplicative inverse is by augmenting with the identity. When matrixA istransformed into I, the augmented matrix I transforms intoA−1.

For example, given

A =

2 1

5 3

(7.317)

augmentAwith the identity 2 1

5 3|

1 0

0 1

(7.318)

Perform row operations with the goal of turningA into the identity.

1. Switch row 1 and row 2. 5 3

2 1|

0 1

1 0

(7.319)

2. Multiply row 2 by − 2 and add to row 1. 1 1

2 1|−2 1

1 0

(7.320)

3. Multiply row 1 by − 2 and add to row 2. 1 1

0 −1|−2 1

5 −2

(7.321)

4. Add row 2 to row 1. 1 0

0 −1|

3 −1

5 −2

(7.322)

5. Multiply row 2 by − 1. 1 0

0 1|

3 −1

−5 2

(7.323)

Available for free at Connexions <http://legacy.cnx.org/content/col11759/1.2>

732 CHAPTER 7. SYSTEMS OF EQUATIONS AND INEQUALITIES

The matrix we have found isA−1.

A−1 =

3 −1

−5 2

(7.324)

7.8.1.3 Finding the Multiplicative Inverse of 2×2 Matrices Using a Formula

When we need to �nd the multiplicative inverse of a 2 × 2matrix, we can use a special formula insteadof using matrix multiplication or augmenting with the identity.

IfA is a 2× 2matrix, such as

A =

a b

c d

(7.325)

the multiplicative inverse ofA is given by the formula

A−1 =1

ad− bc

d −b−c a

(7.326)

where ad− bc 6= 0. If ad− bc = 0, thenA has no inverse.

Example 7.51Using the Formula to Find the Multiplicative Inverse of Matrix AUse the formula to �nd the multiplicative inverse of

A =

1 −2

2 −3

(7.327)

Solution

Using the formula, we have

A−1 = 1(1)(−3)−(−2)(2)

−3 2

−2 1

= 1−3+4

−3 2

−2 1

=

−3 2

−2 1

(7.328)

AnalysisWe can check that our formula works by using one of the other methods to calculate the inverse.Let's augmentAwith the identity. 1 −2

2 −3|

1 0

0 1

(7.329)

Perform row operations with the goal of turningA into the identity.

Available for free at Connexions <http://legacy.cnx.org/content/col11759/1.2>

733

1. Multiply row 1 by − 2 and add to row 2. 1 −2

0 1|

1 0

−2 1

(7.330)

2. Multiply row 1 by 2 and add to row 1. 1 0

0 1|−3 2

−2 1

(7.331)

So, we have veri�ed our original solution.

A−1 =

−3 2

−2 1

(7.332)

Try It:

Exercise 7.8.2 (Solution on p. 798.)

Use the formula to �nd the inverse of matrixA.Verify your answer by augmenting withthe identity matrix.

A =

1 −1

2 3

(7.333)

Example 7.52Finding the Inverse of the Matrix, If It ExistsFind the inverse, if it exists, of the given matrix.

A =

3 6

1 2

(7.334)

Solution

We will use the method of augmenting with the identity. 3 6

1 3|

1 0

0 1

(7.335)

Available for free at Connexions <http://legacy.cnx.org/content/col11759/1.2>

734 CHAPTER 7. SYSTEMS OF EQUATIONS AND INEQUALITIES

1. Switch row 1 and row 2. 1 3

3 6|

0 1

1 0

(7.336)

2. Multiply row 1 by −3 and add it to row 2. 1 2

0 0|

1 0

−3 1

(7.337)

3. There is nothing further we can do. The zeros in row 2 indicate that this matrix has noinverse.

7.8.1.4 Finding the Multiplicative Inverse of 3×3 Matrices

Unfortunately, we do not have a formula similar to the one for a 2×2matrix to �nd the inverse of a 3×3matrix.Instead, we will augment the original matrix with the identity matrix and use row operations to obtainthe inverse.

Given a 3 × 3 matrix

A =

2 3 1

3 3 1

2 4 1

(7.338)

augmentAwith the identity matrix

A|I =

2 3 1

3 3 1

2 4 1

|1 0 0

0 1 0

0 0 1

(7.339)

To begin, we write the augmented matrix with the identity on the right andA on the left. Performingelementary row operations so that the identity matrix appears on the left, we will obtain the inversematrix on the right. We will �nd the inverse of this matrix in the next example.

How To: Given a 3 × 3matrix, �nd the inverse

1.Write the original matrix augmented with the identity matrix on the right.2.Use elementary row operations so that the identity appears on the left.3.What is obtained on the right is the inverse of the original matrix.4.Use matrix multiplication to show thatAA−1 = I andA−1A = I.

Example 7.53Finding the Inverse of a 3 × 3 MatrixGiven the 3 × 3matrixA, �nd the inverse.

Available for free at Connexions <http://legacy.cnx.org/content/col11759/1.2>

735

A =

2 3 1

3 3 1

2 4 1

(7.340)

Solution

AugmentAwith the identity matrix, and then begin row operations until the identity matrixreplacesA.The matrix on the right will be the inverse ofA.

2 3 1

3 3 1

2 4 1

|1 0 0

0 1 0

0 0 1

Interchange R2 and R1→

3 3 1

2 3 1

2 4 1

|0 1 0

1 0 0

0 0 1

(7.341)

−R2 +R1 = R1 →

1 0 0

2 3 1

2 4 1

|−1 1 0

1 0 0

0 0 1

(7.342)

−R2 +R3 = R3 →

1 0 0

2 3 1

0 1 0

|−1 1 0

1 0 0

−1 0 1

(7.343)

R3 ↔ R2 →

1 0 0

0 1 0

2 3 1

|−1 1 0

−1 0 1

1 0 0

(7.344)

−2R1 +R3 = R3 →

1 0 0

0 1 0

0 3 1

|−1 1 0

−1 0 1

3 −2 0

(7.345)

−3R2 +R3 = R3 →

1 0 0

0 1 0

0 0 1

|−1 1 0

−1 0 1

6 −2 −3

(7.346)

Thus,

A−1 = B =

−1 1 0

−1 0 1

6 −2 −3

(7.347)

Available for free at Connexions <http://legacy.cnx.org/content/col11759/1.2>

736 CHAPTER 7. SYSTEMS OF EQUATIONS AND INEQUALITIES

Analysis

To prove thatB = A−1, let's multiply the two matrices together to see if the product equals theidentity, ifAA−1 = I andA−1A = I.

AA−1 =

2 3 1

3 3 1

2 4 1

−1 1 0

−1 0 1

6 −2 −3

=

2 (−1) + 3 (−1) + 1 (6) 2 (1) + 3 (0) + 1 (−2) 2 (0) + 3 (1) + 1 (−3)

3 (−1) + 3 (−1) + 1 (6) 3 (1) + 3 (0) + 1 (−2) 3 (0) + 3 (1) + 1 (−3)

2 (−1) + 4 (−1) + 1 (6) 2 (1) + 4 (0) + 1 (−2) 2 (0) + 4 (1) + 1 (−3)

=

1 0 0

0 1 0

0 0 1

(7.348)

A−1A =

−1 1 0

−1 0 1

6 −2 −3

2 3 1

3 3 1

2 4 1

=

−1 (2) + 1 (3) + 0 (2) − 1 (3) + 1 (3) + 0 (4) − 1 (1) + 1 (1) + 0 (1)

−1 (2) + 0 (3) + 1 (2) − 1 (3) + 0 (3) + 1 (4) − 1 (1) + 0 (1) + 1 (1)

6 (2) +−2 (3) +−3 (2) 6 (3) +−2 (3) +−3 (4) 6 (1) +−2 (1) +−3 (1)

=

1 0 0

0 1 0

0 0 1

(7.349)

Try It:

Exercise 7.8.3 (Solution on p. 798.)

Find the inverse of the 3× 3matrix.

Available for free at Connexions <http://legacy.cnx.org/content/col11759/1.2>

737

A =

2 −17 11

−1 11 −7

0 3 −2

(7.350)

7.8.2 Solving a System of Linear Equations Using the Inverse of a Matrix

Solving a system of linear equations using the inverse of a matrix requires the de�nition of two newmatrices:X is the matrix representing the variables of the system, andB is the matrix representing theconstants. Using matrix multiplication, we may de�ne a system of equations with the same number ofequations as variables as

AX = B (7.351)

To solve a system of linear equations using an inverse matrix, letA be the coe�cient matrix, letX be thevariable matrix, and letB be the constant matrix. Thus, we want to solve a systemAX = B.For example,look at the following system of equations.

a1x+ b1y = c1

a2x+ b2y = c2(7.352)

From this system, the coe�cient matrix is

A =

a1 b1

a2 b2

(7.353)

The variable matrix is

X =

x

y

(7.354)

And the constant matrix is

B =

c1

c2

(7.355)

ThenAX = B looks like a1 b1

a2 b2

x

y

=

c1

c2

(7.356)

Recall the discussion earlier in this section regarding multiplying a real number by its inverse,(2−1)

2 =(12

)2 = 1.To solve a single linear equation ax = b forx, we would simply multiply both sides of the equation

by the multiplicative inverse (reciprocal) of a.Thus,

Available for free at Connexions <http://legacy.cnx.org/content/col11759/1.2>

738 CHAPTER 7. SYSTEMS OF EQUATIONS AND INEQUALITIES

ax = b(1a

)ax =

(1a

)b(

a−1)ax =

(a−1

)b[(

a−1)a]x =

(a−1

)b

1x =(a−1

)b

x =(a−1

)b

(7.357)

The only di�erence between a solving a linear equation and a system of equations written in matrix formis that �nding the inverse of a matrix is more complicated, and matrix multiplication is a longer process.However, the goal is the same�to isolate the variable.

We will investigate this idea in detail, but it is helpful to begin with a 2 × 2 system and then move on toa 3 × 3 system.

A General Note: Given a system of equations, write the coe�cient matrixA, the variablematrixX, and the constant matrixB.Then

AX = B (7.358)

Multiply both sides by the inverse ofA to obtain the solution.

(A−1

)AX =

(A−1

)B[(

A−1)A]X =

(A−1

)B

IX =(A−1

)B

X =(A−1

)B

(7.359)

Q&A: If the coe�cient matrix does not have an inverse, does that mean the systemhas no solution?

No, if the coe�cient matrix is not invertible, the system could be inconsistent and have no solution,or be dependent and have in�nitely many solutions.

Example 7.54Solving a 2 × 2 System Using the Inverse of a MatrixSolve the given system of equations using the inverse of a matrix.

3x+ 8y = 5

4x+ 11y = 7(7.360)

Solution

Write the system in terms of a coe�cient matrix, a variable matrix, and a constant matrix.

A =

3 8

4 11

, X =

x

y

, B =

5

7

(7.361)

Then

Available for free at Connexions <http://legacy.cnx.org/content/col11759/1.2>

739

3 8

4 11

x

y

=

5

7

(7.362)

First, we need to calculateA−1.Using the formula to calculate the inverse of a 2 by 2 matrix, wehave:

A−1 = 1ad−bc

d −b−c a

= 1

3(11)−8(4)

11 −8

−4 3

= 1

1

11 −8

−4 3

(7.363)

So,

A−1 =

11 −8

−4 3

(7.364)

Now we are ready to solve. Multiply both sides of the equation byA−1.

(A−1

)AX =

(A−1

)B 11 −8

−4 3

3 8

4 11

x

y

=

11 −8

−4 3

5

7

1 0

0 1

x

y

=

11 (5) + (−8) 7

−4 (5) + 3 (7)

x

y

=

−1

1

(7.365)

The solution is (−1, 1) .

Q&A: Can we solve forX by �nding the productBA−1?

No, recall that matrix multiplication is not commutative, soA−1B 6= BA−1.Consider our steps forsolving the matrix equation.

(A−1

)AX =

(A−1

)B[(

A−1)A]X =

(A−1

)B

IX =(A−1

)B

X =(A−1

)B

(7.366)

Available for free at Connexions <http://legacy.cnx.org/content/col11759/1.2>

740 CHAPTER 7. SYSTEMS OF EQUATIONS AND INEQUALITIES

Notice in the �rst step we multiplied both sides of the equation byA−1, but theA−1 was to theleft ofA on the left side and to the left ofB on the right side. Because matrix multiplication is notcommutative, order matters.

Example 7.55Solving a 3 × 3 System Using the Inverse of a MatrixSolve the following system using the inverse of a matrix.

5x+ 15y + 56z = 35

−4x− 11y − 41z = −26

−x− 3y − 11z = −7

(7.367)

Solution

Write the equationAX = B.5 15 56

−4 −11 −41

−1 −3 −11

x

y

z

=

35

−26

−7

(7.368)

First, we will �nd the inverse ofA by augmenting with the identity.5 15 56

−4 −11 −41

−1 −3 −11

|1 0 0

0 1 0

0 0 1

(7.369)

Multiply row 1 by 15 .

1 3 565

−4 −11 −41

−1 −3 −11

|

15 0 0

0 1 0

0 0 1

(7.370)

Multiply row 1 by 4 and add to row 2.1 3 56

5

0 1 195

−1 −3 −11

|

15 0 045 1 0

0 0 1

(7.371)

Add row 1 to row 3. 1 3 56

5

0 1 195

0 0 15

|

15 0 045 1 015 0 1

(7.372)

Multiply row 2 by −3 and add to row 1.

Available for free at Connexions <http://legacy.cnx.org/content/col11759/1.2>

741

1 0 − 1

5

0 1 195

0 0 15

|− 11

5 −3 045 1 015 0 1

(7.373)

Multiply row 3 by 5. 1 0 − 1

5

0 1 195

0 0 1

|− 11

5 −3 045 1 0

1 0 5

(7.374)

Multiply row 3 by 15 and add to row 1.

1 0 0

0 1 195

0 0 1

|−2 −3 145 1 0

1 0 5

(7.375)

Multiply row 3 by − 195 and add to row 2.

1 0 0

0 1 0

0 0 1

|−2 −3 1

−3 1 −19

1 0 5

(7.376)

So,

A−1 =

−2 −3 1

−3 1 −19

1 0 5

(7.377)

Multiply both sides of the equation byA−1.We wantA−1AX = A−1B :−2 −3 1

−3 1 −19

1 0 5

5 15 56

−4 −11 −41

−1 −3 −11

x

y

z

=

−2 −3 1

−3 1 −19

1 0 5

35

−26

−7

(7.378)

Thus,

A−1B =

−70 + 78− 7

−105− 26 + 133

35 + 0− 35

=

1

2

0

(7.379)

The solution is (1, 2, 0) .

Available for free at Connexions <http://legacy.cnx.org/content/col11759/1.2>

742 CHAPTER 7. SYSTEMS OF EQUATIONS AND INEQUALITIES

Try It:

Exercise 7.8.4 (Solution on p. 798.)

Solve the system using the inverse of the coe�cient matrix.

2x− 17y + 11z = 0

− x+ 11y − 7z = 8

3y − 2z = −2

(7.380)

How To: Given a system of equations, solve with matrix inverses using a calculator.

1.Save the coe�cient matrix and the constant matrix as matrix variables [A] and [B] .2.Enter the multiplication into the calculator, calling up each matrix variable as needed.3.If the coe�cient matrix is invertible, the calculator will present the solution matrix; if thecoe�cient matrix is not invertible, the calculator will present an error message.

Example 7.56Using a Calculator to Solve a System of Equations with Matrix InversesSolve the system of equations with matrix inverses using a calculator

2x+ 3y + z = 32

3x+ 3y + z = −27

2x+ 4y + z = −2

(7.381)

Solution

On the matrix page of the calculator, enter the coe�cient matrix as the matrix variable [A] , andenter the constant matrix as the matrix variable [B] .

[A] =

2 3 1

3 3 1

2 4 1

, [B] =

32

−27

−2

(7.382)

On the home screen of the calculator, type in the multiplication to solve forX, calling up eachmatrix variable as needed.

[A]−1 × [B] (7.383)

Evaluate the expression. −59

−34

252

(7.384)

Available for free at Connexions <http://legacy.cnx.org/content/col11759/1.2>

743

Media: Access these online resources for additional instruction and practice with solving systemswith inverses.

• The Identity Matrix31

• Determining Inverse Matrices32

• Using a Matrix Equation to Solve a System of Equations33

7.8.3 Key Equations

Identity matrix for a2 × 2matrix I2 =

1 0

0 1

Identity matrix for a3 × 3matrix I3 =

1 0 0

0 1 0

0 0 1

Multiplicative inverse of a2 × 2matrix A−1 = 1

ad−bc

d −b−c a

, where ad− bc 6= 0

Table 7.6

7.8.4 Key Concepts

• An identity matrix has the propertyAI = IA = A. See Example 7.48.• An invertible matrix has the propertyAA−1 = A−1A = I. See Example 7.49.• Use matrix multiplication and the identity to �nd the inverse of a 2× 2matrix. See Example 7.50.• The multiplicative inverse can be found using a formula. See Example 7.51.• Another method of �nding the inverse is by augmenting with the identity. See Example 7.52.• We can augment a 3×3matrix with the identity on the right and use row operations to turn the original

matrix into the identity, and the matrix on the right becomes the inverse. See Example 7.53.• Write the system of equations asAX = B, and multiply both sides by the inverse ofA : A−1AX =

A−1B. See Example 7.54 and Example 7.55.• We can also use a calculator to solve a system of equations with matrix inverses. See Example 7.56.

7.8.5 Section Exercises

7.8.5.1 Verbal

Exercise 7.8.5 (Solution on p. 798.)

In a previous section, we showed that matrix multiplication is not commutative, that is,AB 6= BA inmost cases. Can you explain why matrix multiplication is commutative for matrix inverses, thatis,A−1A = AA−1?Exercise 7.8.6Does every 2 × 2matrix have an inverse? Explain why or why not. Explain what condition isnecessary for an inverse to exist.

31http://openstaxcollege.org/l/identmatrix32http://openstaxcollege.org/l/inversematrix33http://openstaxcollege.org/l/matrixsystem

Available for free at Connexions <http://legacy.cnx.org/content/col11759/1.2>

744 CHAPTER 7. SYSTEMS OF EQUATIONS AND INEQUALITIES

Exercise 7.8.7 (Solution on p. 798.)

Can you explain whether a 2× 2matrix with an entire row of zeros can have an inverse?

Exercise 7.8.8Can a matrix with an entire column of zeros have an inverse? Explain why or why not.

Exercise 7.8.9 (Solution on p. 798.)

Can a matrix with zeros on the diagonal have an inverse? If so, �nd an example. If not, prove whynot. For simplicity, assume a 2× 2matrix.

7.8.5.2 Algebraic

In the following exercises, show that matrixA is the inverse of matrixB.

Exercise 7.8.10

A =

1 0

−1 1

, B =

1 0

1 1

Exercise 7.8.11 (Solution on p. 798.)

A =

1 2

3 4

, B =

−2 132 − 1

2

Exercise 7.8.12

A =

4 5

7 0

, B =

0 17

15 − 4

35

Exercise 7.8.13 (Solution on p. 799.)

A =

−2 12

3 −1

, B =

−2 −1

−6 −4

Exercise 7.8.14

A =

1 0 1

0 1 −1

0 1 1

, B = 12

2 1 −1

0 1 1

0 −1 1

Exercise 7.8.15 (Solution on p. 799.)

A =

1 2 3

4 0 2

1 6 9

, B = 14

6 0 −2

17 −3 −5

−12 2 4

Exercise 7.8.16

A =

3 8 2

1 1 1

5 6 12

, B = 136

−6 84 −6

7 −26 1

−1 −22 5

For the following exercises, �nd the multiplicative inverse of each matrix, if it exists.

Exercise 7.8.17 (Solution on p. 799.) 3 −2

1 9

Available for free at Connexions <http://legacy.cnx.org/content/col11759/1.2>

745

Exercise 7.8.18 −2 2

3 1

Exercise 7.8.19 (Solution on p. 799.) −3 7

9 2

Exercise 7.8.20 −4 −3

−5 8

Exercise 7.8.21 (Solution on p. 799.) 1 1

2 2

Exercise 7.8.22 0 1

1 0

Exercise 7.8.23 (Solution on p. 799.) 0.5 1.5

1 −0.5

Exercise 7.8.24

1 0 6

−2 1 7

3 0 2

Exercise 7.8.25 (Solution on p. 799.)

0 1 −3

4 1 0

1 0 5

Exercise 7.8.26

1 2 −1

−3 4 1

−2 −4 −5

Exercise 7.8.27 (Solution on p. 799.)

1 9 −3

2 5 6

4 −2 7

Exercise 7.8.28

1 −2 3

−4 8 −12

1 4 2

Available for free at Connexions <http://legacy.cnx.org/content/col11759/1.2>

746 CHAPTER 7. SYSTEMS OF EQUATIONS AND INEQUALITIES

Exercise 7.8.29 (Solution on p. 799.)12

12

12

13

14

15

16

17

18

Exercise 7.8.30

1 2 3

4 5 6

7 8 9

For the following exercises, solve the system using the inverse of a 2 × 2matrix.

Exercise 7.8.31 (Solution on p. 799.)

5x− 6y = −61

4x+ 3y = −2

Exercise 7.8.32

8x+ 4y = −100

3x− 4y = 1

Exercise 7.8.33 (Solution on p. 799.)

3x− 2y = 6

−x+ 5y = −2

Exercise 7.8.34

5x− 4y = −5

4x+ y = 2.3

Exercise 7.8.35 (Solution on p. 799.)

−3x− 4y = 9

12x+ 4y = −6

Exercise 7.8.36

−2x+ 3y = 310

− x+ 5y = 12

Exercise 7.8.37 (Solution on p. 799.)85x−

45y = 2

5

− 85x+ 1

5y = 710

Exercise 7.8.3812x+ 1

5y = − 14

12x−

35y = − 9

4

For the following exercises, solve a system using the inverse of a 3× 3 matrix.

Exercise 7.8.39 (Solution on p. 799.)

3x− 2y + 5z = 21

5x+ 4y = 37

x− 2y − 5z = 5

Available for free at Connexions <http://legacy.cnx.org/content/col11759/1.2>

747

Exercise 7.8.40

4x+ 4y + 4z = 40

2x− 3y + 4z = −12

− x+ 3y + 4z = 9

Exercise 7.8.41 (Solution on p. 800.)

6x− 5y − z = 31

− x+ 2y + z = −6

3x+ 3y + 2z = 13

Exercise 7.8.42

6x− 5y + 2z = −4

2x+ 5y − z = 12

2x+ 5y + z = 12

Exercise 7.8.43 (Solution on p. 800.)

4x− 2y + 3z = −12

2x+ 2y − 9z = 33

6y − 4z = 1

Exercise 7.8.44110x−

15y + 4z = −41

2

15x− 20y + 2

5z = −101310x+ 4y − 3

10z = 23

Exercise 7.8.45 (Solution on p. 800.)12x−

15y + 1

5z = 31100

− 34x−

14y + 1

2z = 740

− 45x−

12y + 3

2z = 14

Exercise 7.8.46

0.1x+ 0.2y + 0.3z = −1.4

0.1x− 0.2y + 0.3z = 0.6

0.4y + 0.9z = −2

7.8.5.3 Technology

For the following exercises, use a calculator to solve the system of equations with matrix inverses.

Exercise 7.8.47 (Solution on p. 800.)

2x− y = −3

−x+ 2y = 2.3

Exercise 7.8.48

− 12x−

32y = − 43

20

52x+ 11

5 y = 314

Available for free at Connexions <http://legacy.cnx.org/content/col11759/1.2>

748 CHAPTER 7. SYSTEMS OF EQUATIONS AND INEQUALITIES

Exercise 7.8.49 (Solution on p. 800.)

12.3x− 2y − 2.5z = 2

36.9x+ 7y − 7.5z = −7

8y − 5z = −10

Exercise 7.8.50

0.5x− 3y + 6z = −0.8

0.7x− 2y = −0.06

0.5x+ 4y + 5z = 0

7.8.5.4 Extensions

For the following exercises, �nd the inverse of the given matrix.

Exercise 7.8.51 (Solution on p. 800.)1 0 1 0

0 1 0 1

0 1 1 0

0 0 1 1

Exercise 7.8.52−1 0 2 5

0 0 0 2

0 2 −1 0

1 −3 0 1

Exercise 7.8.53 (Solution on p. 800.)

1 −2 3 0

0 1 0 2

1 4 −2 3

−5 0 1 1

Exercise 7.8.54

1 2 0 2 3

0 2 1 0 0

0 0 3 0 1

0 2 0 0 1

0 0 1 2 0

Available for free at Connexions <http://legacy.cnx.org/content/col11759/1.2>

749

Exercise 7.8.55 (Solution on p. 800.)

1 0 0 0 0 0

0 1 0 0 0 0

0 0 1 0 0 0

0 0 0 1 0 0

0 0 0 0 1 0

1 1 1 1 1 1

7.8.5.5 Real-World Applications

For the following exercises, write a system of equations that represents the situation. Then, solve the systemusing the inverse of a matrix.

Exercise 7.8.562,400 tickets were sold for a basketball game. If the prices for �oor 1 and �oor 2 were di�erent, andthe total amount of money brought in is $64,000, how much was the price of each ticket?

Exercise 7.8.57 (Solution on p. 800.)

In the previous exercise, if you were told there were 400 more tickets sold for �oor 2 than �oor 1,how much was the price of each ticket?

Exercise 7.8.58A food drive collected two di�erent types of canned goods, green beans and kidney beans. Thetotal number of collected cans was 350 and the total weight of all donated food was 348 lb, 12 oz. Ifthe green bean cans weigh 2 oz less than the kidney bean cans, how many of each can was donated?

Exercise 7.8.59 (Solution on p. 800.)

Students were asked to bring their favorite fruit to class. 95% of the fruits consisted of banana,apple, and oranges. If oranges were twice as popular as bananas, and apples were 5% less popularthan bananas, what are the percentages of each individual fruit?

Exercise 7.8.60A sorority held a bake sale to raise money and sold brownies and chocolate chip cookies. Theypriced the brownies at $1 and the chocolate chip cookies at $0.75. They raised $700 and sold 850items. How many brownies and how many cookies were sold?

Exercise 7.8.61 (Solution on p. 800.)

A clothing store needs to order new inventory. It has three di�erent types of hats for sale: strawhats, beanies, and cowboy hats. The straw hat is priced at $13.99, the beanie at $7.99, and thecowboy hat at $14.49. If 100 hats were sold this past quarter, $1,119 was taken in by sales, andthe amount of beanies sold was 10 more than cowboy hats, how many of each should the clothingstore order to replace those already sold?

Exercise 7.8.62Anna, Ashley, and Andrea weigh a combined 370 lb. If Andrea weighs 20 lb more than Ashley, andAnna weighs 1.5 times as much as Ashley, how much does each girl weigh?

Exercise 7.8.63 (Solution on p. 800.)

Three roommates shared a package of 12 ice cream bars, but no one remembers who ate how many.If Tom ate twice as many ice cream bars as Joe, and Albert ate three less than Tom, how many icecream bars did each roommate eat?

Exercise 7.8.64A farmer constructed a chicken coop out of chicken wire, wood, and plywood. The chicken wirecost $2 per square foot, the wood $10 per square foot, and the plywood $5 per square foot. The

Available for free at Connexions <http://legacy.cnx.org/content/col11759/1.2>

750 CHAPTER 7. SYSTEMS OF EQUATIONS AND INEQUALITIES

farmer spent a total of $51, and the total amount of materials used was 14 ft2.He used 3 ft2 morechicken wire than plywood. How much of each material in did the farmer use?

Exercise 7.8.65 (Solution on p. 800.)

Jay has lemon, orange, and pomegranate trees in his backyard. An orange weighs 8 oz, a lemon 5oz, and a pomegranate 11 oz. Jay picked 142 pieces of fruit weighing a total of 70 lb, 10 oz. Hepicked 15.5 times more oranges than pomegranates. How many of each fruit did Jay pick?

7.9 Solving Systems with Cramer's Rule34

We have learned how to solve systems of equations in two variables and three variables, and by multiplemethods: substitution, addition, Gaussian elimination, using the inverse of a matrix, and graphing. Some ofthese methods are easier to apply than others and are more appropriate in certain situations. In this section,we will study two more strategies for solving systems of equations.

7.9.1 Evaluating the Determinant of a 2×2 Matrix

A determinant is a real number that can be very useful in mathematics because it has multiple applications,such as calculating area, volume, and other quantities. Here, we will use determinants to reveal whether amatrix is invertible by using the entries of a square matrix to determine whether there is a solution to thesystem of equations. Perhaps one of the more interesting applications, however, is their use in cryptography.Secure signals or messages are sometimes sent encoded in a matrix. The data can only be decrypted with aninvertible matrix and the determinant. For our purposes, we focus on the determinant as an indicationof the invertibility of the matrix. Calculating the determinant of a matrix involves following the speci�cpatterns that are outlined in this section.

: The determinant of a 2 × 2matrix, given

A =

a b

c d

(7.385)

is de�ned as

Notice the change in notation. There are several ways to indicate the determinant,including det (A) and replacing the brackets in a matrix with straight lines, |A|.

Example 7.57Finding the Determinant of a 2 × 2 MatrixFind the determinant of the given matrix.

A =

5 2

−6 3

(7.386)

34This content is available online at <http://legacy.cnx.org/content/m49436/1.4/>.

Available for free at Connexions <http://legacy.cnx.org/content/col11759/1.2>

751

Solution

det (A) = |5 2

−6 3|

= 5 (3)− (−6) (2)

= 27

(7.387)

7.9.2 Using Cramer's Rule to Solve a System of Two Equations in Two Variables

We will now introduce a �nal method for solving systems of equations that uses determinants. Known asCramer's Rule, this technique dates back to the middle of the 18th century and is named for its innovator,the Swiss mathematician Gabriel Cramer (1704-1752), who introduced it in 1750 in Introduction à l'Analysedes lignes Courbes algébriques. Cramer's Rule is a viable and e�cient method for �nding solutions to systemswith an arbitrary number of unknowns, provided that we have the same number of equations as unknowns.

Cramer's Rule will give us the unique solution to a system of equations, if it exists. However, if thesystem has no solution or an in�nite number of solutions, this will be indicated by a determinant of zero.To �nd out if the system is inconsistent or dependent, another method, such as elimination, will have to beused.

To understand Cramer's Rule, let's look closely at how we solve systems of linear equations using basicrow operations. Consider a system of two equations in two variables.

a1x+ b1y = c1 (1)

a2x+ b2y = c2 (2)(7.388)

We eliminate one variable using row operations and solve for the other. Say that we wish to solve forx. Ifequation (2) is multiplied by the opposite of the coe�cient of y in equation (1), equation (1) is multiplied bythe coe�cient of y in equation (2), and we add the two equations, the variable ywill be eliminated.

b2a1x+ b2b1y = b2c1 Multiply R1 by b2

−b1a2x− b1b2y = −b1c2 Multiply R2 by− b1________________________________________________________

b2a1x− b1a2x = b2c1 − b1c2

(7.389)

Available for free at Connexions <http://legacy.cnx.org/content/col11759/1.2>

752 CHAPTER 7. SYSTEMS OF EQUATIONS AND INEQUALITIES

Now, solve forx.

b2a1x− b1a2x = b2c1 − b1c2x (b2a1 − b1a2) = b2c1 − b1c2

x = b2c1−b1c2b2a1−b1a2

=

2664 c1 b1

c2 b2

37752664 a1 b1

a2 b2

3775

(7.390)

Similarly, to solve for y,we will eliminatex.

a2a1x+ a2b1y = a2c1 Multiply R1 by a2

−a1a2x− a1b2y = −a1c2 Multiply R2 by− a1

________________________________________________________

a2b1y − a1b2y = a2c1 − a1c2

(7.391)

Solving for y gives

a2b1y − a1b2y = a2c1 − a1c2

y (a2b1 − a1b2) = a2c1 − a1c2

y = a2c1−a1c2a2b1−a1b2

= a1c2−a2c1a1b2−a2b1

=

|a1 c1

a2 c2|

|a1 b1

a2 b2|

(7.392)

Notice that the denominator for bothx and y is the determinant of the coe�cient matrix.We can use these formulas to solve forx and y, but Cramer's Rule also introduces new notation:

� D :determinant of the coe�cient matrix� Dx :determinant of the numerator in the solution ofx

x =Dx

D(7.393)

� Dy :determinant of the numerator in the solution of y

y =Dy

D(7.394)

The key to Cramer's Rule is replacing the variable column of interest with the constant column and calcu-lating the determinants. We can then expressx and y as a quotient of two determinants.

: Cramer's Rule is a method that uses determinants to solve systems of equations that have thesame number of equations as variables.

Consider a system of two linear equations in two variables.

Available for free at Connexions <http://legacy.cnx.org/content/col11759/1.2>

753

a1x+ b1y = c1

a2x+ b2y = c2(7.395)

The solution using Cramer's Rule is given as

x =Dx

D=

|c1 b1

c2 b2|

|a1 b1

a2 b2|

, D 6= 0; y =Dy

D=

|a1 c1

a2 c2|

|a1 b1

a2 b2|

, D 6= 0. (7.396)

If we are solving forx, thex column is replaced with the constant column. If we are solvingfor y, the y column is replaced with the constant column.

Example 7.58Using Cramer's Rule to Solve a 2 × 2 SystemSolve the following 2 × 2 system using Cramer's Rule.

12x+ 3y = 15

2x− 3y = 13(7.397)

Solution

Solve forx.

x =Dx

D=

|15 3

13 −3|

|12 3

2 −3|

=−45− 39−36− 6

=−84−42

= 2 (7.398)

Solve for y.

y =Dy

D=

|12 15

2 13|

|12 3

2 −3|

=156− 30−36− 6

= −12642

= −3 (7.399)

The solution is (2,−3) .

try it feature:

Exercise 7.9.1 (Solution on p. 800.)

Use Cramer's Rule to solve the 2 × 2 system of equations.

Available for free at Connexions <http://legacy.cnx.org/content/col11759/1.2>

754 CHAPTER 7. SYSTEMS OF EQUATIONS AND INEQUALITIES

x+ 2y = −11

−2x+ y = −13(7.400)

7.9.3 Evaluating the Determinant of a 3 × 3 Matrix

Finding the determinant of a 2×2 matrix is straightforward, but �nding the determinant of a 3×3 matrixis more complicated. One method is to augment the 3×3 matrix with a repetition of the �rst two columns,giving a 3×5 matrix. Then we calculate the sum of the products of entries down each of the three diagonals(upper left to lower right), and subtract the products of entries up each of the three diagonals (lower left toupper right). This is more easily understood with a visual and an example.

Find the determinant of the 3×3 matrix.

A =

a1 b1 c1

a2 b2 c2

a3 b3 c3

(7.401)

1. AugmentAwith the �rst two columns.

det (A) = |a1 b1 c1

a2 b2 c2

a3 b3 c3

|a1

a2

a3

b1

b2

b3

| (7.402)

2. From upper left to lower right: Multiply the entries down the �rst diagonal. Add the result to theproduct of entries down the second diagonal. Add this result to the product of the entries down thethird diagonal.

3. From lower left to upper right: Subtract the product of entries up the �rst diagonal. From this resultsubtract the product of entries up the second diagonal. From this result, subtract the product of entriesup the third diagonal.

The algebra is as follows:

|A| = a1b2c3 + b1c2a3 + c1a2b3 − a3b2c1 − b3c2a1 − c3a2b1 (7.403)

Example 7.59Finding the Determinant of a 3 × 3 MatrixFind the determinant of the 3 × 3 matrix given

Available for free at Connexions <http://legacy.cnx.org/content/col11759/1.2>

755

A =

0 2 1

3 −1 1

4 0 1

(7.404)

Solution

Augment the matrix with the �rst two columns and then follow the formula. Thus,

|A| = |0 2 1

3 −1 1

4 0 1

|0

3

4

2

−1

0

|

= 0 (−1) (1) + 2 (1) (4) + 1 (3) (0)− 4 (−1) (1)− 0 (1) (0)− 1 (3) (2)

= 0 + 8 + 0 + 4− 0− 6

= 6

(7.405)

try it feature:

Exercise 7.9.2 (Solution on p. 800.)

Find the determinant of the 3 × 3 matrix.

det (A) = |1 −3 7

1 1 1

1 −2 3

| (7.406)

qa feature: Can we use the same method to �nd the determinant of a larger matrix?

No, this method only works for 2 × 2 and 3 × 3matrices. For larger matrices it is best to use agraphing utility or computer software.

7.9.4 Using Cramer's Rule to Solve a System of Three Equations in Three Vari-

ables

Now that we can �nd the determinant of a 3 × 3 matrix, we can apply Cramer's Rule to solve a systemof three equations in three variables. Cramer's Rule is straightforward, following a pattern consistentwith Cramer's Rule for 2 × 2 matrices. As the order of the matrix increases to 3 × 3, however, there aremany more calculations required.

When we calculate the determinant to be zero, Cramer's Rule gives no indication as to whether thesystem has no solution or an in�nite number of solutions. To �nd out, we have to perform elimination onthe system.

Consider a 3 × 3 system of equations.

Available for free at Connexions <http://legacy.cnx.org/content/col11759/1.2>

756 CHAPTER 7. SYSTEMS OF EQUATIONS AND INEQUALITIES

x =Dx

D, y =

Dy

D, z =

Dz

D,D 6= 0 (7.407)

where

If we are writing the determinantDx,we replace thex column with the constant column. If we arewriting the determinantDy,we replace the y column with the constant column. If we are writing thedeterminantDz,we replace the z column with the constant column. Always check the answer.

Example 7.60Solving a 3 × 3 System Using Cramer's RuleFind the solution to the given 3 × 3 system using Cramer's Rule.

x+ y − z = 6

3x− 2y + z = −5

x+ 3y − 2z = 14

(7.408)

Solution

Use Cramer's Rule.

D = |1 1 −1

3 −2 1

1 3 −2

|, Dx = |6 1 −1

−5 −2 1

14 3 −2

|, Dy = |1 6 −1

3 −5 1

1 14 −2

|, Dz = |1 1 6

3 −2 −5

1 3 14

| (7.409)

Then,

x = Dx

D = −3−3 = 1

y = Dy

D = −9−3 = 3

z = Dz

D = 6−3 = −2

(7.410)

The solution is(1, 3,−2) .

Available for free at Connexions <http://legacy.cnx.org/content/col11759/1.2>

757

try it feature:

Exercise 7.9.3 (Solution on p. 800.)

Use Cramer's Rule to solve the 3 × 3 matrix.

x− 3y + 7z = 13

x+ y + z = 1

x− 2y + 3z = 4

(7.411)

Example 7.61Using Cramer's Rule to Solve an Inconsistent SystemSolve the system of equations using Cramer's Rule.

3x− 2y = 4 (1)

6x− 4y = 0 (2)(7.412)

Solution

We begin by �nding the determinantsD,Dx, and Dy.

D = |3 −2

6 −4| = 3 (−4)− 6 (−2) = 0 (7.413)

We know that a determinant of zero means that either the system has no solution or it has anin�nite number of solutions. To see which one, we use the process of elimination. Our goal is toeliminate one of the variables.

1. Multiply equation (1) by − 2.2. Add the result to equation (2) .

−6x+ 4y = −8

6x− 4y = 0_______________

0 = − 8

(7.414)

We obtain the equation 0 = −8, which is false. Therefore, the system has no solution. Graphingthe system reveals two parallel lines. See Figure 7.31.

Available for free at Connexions <http://legacy.cnx.org/content/col11759/1.2>

758 CHAPTER 7. SYSTEMS OF EQUATIONS AND INEQUALITIES

Figure 7.31

Example 7.62Use Cramer's Rule to Solve a Dependent SystemSolve the system with an in�nite number of solutions.

x− 2y + 3z = 0 (1)

3x+ y − 2z = 0 (2)

2x− 4y + 6z = 0 (3)

(7.415)

Solution

Available for free at Connexions <http://legacy.cnx.org/content/col11759/1.2>

759

Let's �nd the determinant �rst. Set up a matrix augmented by the �rst two columns.

|1 −2 3

3 1 −2

2 −4 6

|1 −2

3 1

2 −4

| (7.416)

Then,

1 (1) (6) + (−2) (−2) (2) + 3 (3) (−4)− 2 (1) (3)− (−4) (−2) (1)− 6 (3) (−2) = 0 (7.417)

As the determinant equals zero, there is either no solution or an in�nite number of solutions. Wehave to perform elimination to �nd out.

1. Multiply equation (1) by − 2 and add the result to equation (3):

−2x+ 4y − 6x = 0

2x− 4y + 6z = 0

0 = 0(7.418)

2. Obtaining an answer of 0 = 0, a statement that is always true, means that the system has anin�nite number of solutions. Graphing the system, we can see that two of the planes are thesame and they both intersect the third plane on a line. See Figure 7.32.

Figure 7.32

Available for free at Connexions <http://legacy.cnx.org/content/col11759/1.2>

760 CHAPTER 7. SYSTEMS OF EQUATIONS AND INEQUALITIES

7.9.5 Understanding Properties of Determinants

There are many properties of determinants. Listed here are some properties that may be helpful incalculating the determinant of a matrix.

a general note label:

1.If the matrix is in upper triangular form, the determinant equals the product of entries downthe main diagonal.

2.When two rows are interchanged, the determinant changes sign.3.If either two rows or two columns are identical, the determinant equals zero.4.If a matrix contains either a row of zeros or a column of zeros, the determinant equals zero.5.The determinant of an inverse matrixA−1 is the reciprocal of the determinant of the matrixA.6.If any row or column is multiplied by a constant, the determinant is multiplied by the samefactor.

Example 7.63Illustrating Properties of DeterminantsIllustrate each of the properties of determinants.

SolutionProperty 1 states that if the matrix is in upper triangular form, the determinant is the product ofthe entries down the main diagonal.

A =

1 2 3

0 2 1

0 0 −1

(7.419)

AugmentAwith the �rst two columns.

A =

1 2 3

0 2 1

0 0 −1

|1

0

0

2

2

0

(7.420)

Then

det (A) = 1 (2) (−1) + 2 (1) (0) + 3 (0) (0)− 0 (2) (3)− 0 (1) (1) + 1 (0) (2)

= −2(7.421)

Property 2 states that interchanging rows changes the sign. Given

A =

−1 5

4 −3

, det (A) = (−1) (−3)− (4) (5) = 3− 20 = −17

B =

4 −3

−1 5

, det (B) = (4) (5)− (−1) (−3) = 20− 3 = 17

(7.422)

Property 3 states that if two rows or two columns are identical, the determinant equals zero.

Available for free at Connexions <http://legacy.cnx.org/content/col11759/1.2>

761

A =

1 2 2

2 2 2

−1 2 2

|1

2

−1

2

2

2

det (A) = 1 (2) (2) + 2 (2) (−1) + 2 (2) (2) + 1 (2) (2)− 2 (2) (1)− 2 (2) (2)

= 4− 4 + 8 + 4− 4− 8 = 0

(7.423)

Property 4 states that if a row or column equals zero, the determinant equals zero. Thus,

A =

1 2

0 0

, det (A) = 1 (0)− 2 (0) = 0 (7.424)

Property 5 states that the determinant of an inverse matrixA−1 is the reciprocal of thedeterminantA.Thus,

A =

1 2

3 4

,det (A) = 1 (4)− 3 (2) = −2

A−1 =

−2 132 − 1

2

,det(A−1

)= −2

(− 1

2

)−(

32

)(1) = − 1

2

(7.425)

Property 6 states that if any row or column of a matrix is multiplied by a constant, the determinantis multiplied by the same factor. Thus,

A =

1 2

3 4

,det (A) = 1 (4)− 2 (3) = −2

B =

2 (1) 2 (2)

3 4

,det (B) = 2 (4)− 3 (4) = −4

(7.426)

Example 7.64Using Cramer's Rule and Determinant Properties to Solve a SystemFind the solution to the given 3 × 3 system.

2x+ 4y + 4z = 2 (1)

3x+ 7y + 7z = −5 (2)

x+ 2y + 2z = 4 (3)

(7.427)

Solution

Using Cramer's Rule, we have

Available for free at Connexions <http://legacy.cnx.org/content/col11759/1.2>

762 CHAPTER 7. SYSTEMS OF EQUATIONS AND INEQUALITIES

D = |2 4 4

3 7 7

1 2 2

| (7.428)

Notice that the second and third columns are identical. According to Property 3, the determinantwill be zero, so there is either no solution or an in�nite number of solutions. We have to performelimination to �nd out.

1. Multiply equation (3) by �2 and add the result to equation (1).

−2x− 4y − 4x = −8

2x+ 4y + 4z = 2

0 = −6(7.429)

Obtaining a statement that is a contradiction means that the system has no solution.

media feature label: Access these online resources for additional instruction and practice withCramer's Rule.

• Solve a System of Two Equations Using Cramer's Rule35

• Solve a Systems of Three Equations using Cramer's Rule36

7.9.6 Key Concepts

• The determinant for

a b

c d

is ad− bc. See Example 7.57.

• Cramer's Rule replaces a variable column with the constant column. Solutions arex = Dx

D , y = Dy

D . SeeExample 7.58.

• To �nd the determinant of a 3×3 matrix, augment with the �rst two columns. Add the three diagonalentries (upper left to lower right) and subtract the three diagonal entries (lower left to upper right).See Example 7.59.

• To solve a system of three equations in three variables using Cramer's Rule, replace a variable columnwith the constant column for each desired solution:x = Dx

D , y = Dy

D , z = Dz

D . See Example 7.60.• Cramer's Rule is also useful for �nding the solution of a system of equations with no solution or in�nite

solutions. See Example 7.61 and Example 7.62.• Certain properties of determinants are useful for solving problems. For example:

· If the matrix is in upper triangular form, the determinant equals the product of entries down themain diagonal.

· When two rows are interchanged, the determinant changes sign.· If either two rows or two columns are identical, the determinant equals zero.· If a matrix contains either a row of zeros or a column of zeros, the determinant equals zero.· The determinant of an inverse matrixA−1 is the reciprocal of the determinant of the matrixA.

35http://openstaxcollege.org/l/system2cramer36http://openstaxcollege.org/l/system3cramer

Available for free at Connexions <http://legacy.cnx.org/content/col11759/1.2>

763

· If any row or column is multiplied by a constant, the determinant is multiplied by the same factor.See Example 7.63 and Example 7.64.

7.9.7 Section Exercises

7.9.7.1 Verbal

Exercise 7.9.4 (Solution on p. 800.)

Explain why we can always evaluate the determinant of a square matrix.

Exercise 7.9.5Examining Cramer's Rule, explain why there is no unique solution to the system when the deter-minant of your matrix is 0. For simplicity, use a 2 × 2matrix.Exercise 7.9.6 (Solution on p. 801.)

Explain what it means in terms of an inverse for a matrix to have a 0 determinant.

Exercise 7.9.7The determinant of 2 × 2matrixA is 3. If you switch the rows and multiply the �rst row by 6 andthe second row by 2, explain how to �nd the determinant and provide the answer.

7.9.7.2 Algebraic

For the following exercises, �nd the determinant.

Exercise 7.9.8 (Solution on p. 801.)

|1 2

3 4|

Exercise 7.9.9

|−1 2

3 −4|

Exercise 7.9.10 (Solution on p. 801.)

|2 −5

−1 6|

Exercise 7.9.11

|−8 4

−1 5|

Exercise 7.9.12 (Solution on p. 801.)

|1 0

3 −4|

Exercise 7.9.13

|10 20

0 −10|

Exercise 7.9.14 (Solution on p. 801.)

|10 0.2

5 0.1|

Available for free at Connexions <http://legacy.cnx.org/content/col11759/1.2>

764 CHAPTER 7. SYSTEMS OF EQUATIONS AND INEQUALITIES

Exercise 7.9.15

|6 −3

8 4|

Exercise 7.9.16 (Solution on p. 801.)

|−2 −3

3.1 4, 000|

Exercise 7.9.17

|−1.1 0.6

7.2 −0.5|

Exercise 7.9.18 (Solution on p. 801.)

|−1 0 0

0 1 0

0 0 −3

|

Exercise 7.9.19

|−1 4 0

0 2 3

0 0 −3

|

Exercise 7.9.20 (Solution on p. 801.)

|1 0 1

0 1 0

1 0 0

|

Exercise 7.9.21

|2 −3 1

3 −4 1

−5 6 1

|

Exercise 7.9.22 (Solution on p. 801.)

|−2 1 4

−4 2 −8

2 −8 −3

|

Exercise 7.9.23

|6 −1 2

−4 −3 5

1 9 −1

|

Exercise 7.9.24 (Solution on p. 801.)

|5 1 −1

2 3 1

3 −6 −3

|

Available for free at Connexions <http://legacy.cnx.org/content/col11759/1.2>

765

Exercise 7.9.25

|1.1 2 −1

−4 0 0

4.1 −0.4 2.5

|

Exercise 7.9.26 (Solution on p. 801.)

|2 −1.6 3.1

1.1 3 −8

−9.3 0 2

|

Exercise 7.9.27

|− 1

213

14

15 − 1

617

0 0 18

|

For the following exercises, solve the system of linear equations using Cramer's Rule.

Exercise 7.9.28 (Solution on p. 801.)

2x− 3y = −1

4x+ 5y = 9

Exercise 7.9.29

5x− 4y = 2

−4x+ 7y = 6

Exercise 7.9.30 (Solution on p. 801.)

6x− 3y = 2

−8x+ 9y = −1

Exercise 7.9.31

2x+ 6y = 12

5x− 2y = 13

Exercise 7.9.32 (Solution on p. 801.)

4x+ 3y = 23

2x− y = −1

Exercise 7.9.33

10x− 6y = 2

−5x+ 8y = −1

Exercise 7.9.34 (Solution on p. 801.)

4x− 3y = −3

2x+ 6y = −4

Exercise 7.9.35

4x− 5y = 7

−3x+ 9y = 0

Exercise 7.9.36 (Solution on p. 801.)

4x+ 10y = 180

−3x− 5y = −105

Available for free at Connexions <http://legacy.cnx.org/content/col11759/1.2>

766 CHAPTER 7. SYSTEMS OF EQUATIONS AND INEQUALITIES

Exercise 7.9.37

8x− 2y = −3

−4x+ 6y = 4

For the following exercises, solve the system of linear equations using Cramer's Rule.

Exercise 7.9.38 (Solution on p. 801.)

x+ 2y − 4z = −1

7x+ 3y + 5z = 26

−2x− 6y + 7z = −6

Exercise 7.9.39

−5x+ 2y − 4z = −47

4x− 3y − z = −94

3x− 3y + 2z = 94

Exercise 7.9.40 (Solution on p. 801.)

4x+ 5y − z = −7

−2x− 9y + 2z = 8

5y + 7z = 21

Exercise 7.9.41

4x− 3y + 4z = 10

5x− 2z = −2

3x+ 2y − 5z = −9

Exercise 7.9.42 (Solution on p. 801.)

4x− 2y + 3z = 6

− 6x+ y = −2

2x+ 7y + 8z = 24

Exercise 7.9.43

5x+ 2y − z = 1

−7x− 8y + 3z = 1.5

6x− 12y + z = 7

Exercise 7.9.44 (Solution on p. 801.)

13x− 17y + 16z = 73

−11x+ 15y + 17z = 61

46x+ 10y − 30z = −18

Exercise 7.9.45

−4x− 3y − 8z = −7

2x− 9y + 5z = 0.5

5x− 6y − 5z = −2

Available for free at Connexions <http://legacy.cnx.org/content/col11759/1.2>

767

Exercise 7.9.46 (Solution on p. 801.)

4x− 6y + 8z = 10

−2x+ 3y − 4z = −5

x+ y + z = 1

Exercise 7.9.47

4x− 6y + 8z = 10

−2x+ 3y − 4z = −5

12x+ 18y − 24z = −30

7.9.7.3 Technology

For the following exercises, use the determinant function on a graphing utility.

Exercise 7.9.48 (Solution on p. 801.)

|

1 0 8 9

0 2 1 0

1 0 3 0

0 2 4 3

|

Exercise 7.9.49

|

1 0 2 1

0 −9 1 3

3 0 −2 −1

0 1 1 −2

|

Exercise 7.9.50 (Solution on p. 801.)

|

12 1 7 4

0 12 100 5

0 0 2 2, 000

0 0 0 2

|

Exercise 7.9.51

|

1 0 0 0

2 3 0 0

4 5 6 0

7 8 9 0

|

7.9.7.4 Real-World Applications

For the following exercises, create a system of linear equations to describe the behavior. Then, calculate thedeterminant. Will there be a unique solution? If so, �nd the unique solution.

Exercise 7.9.52 (Solution on p. 801.)

Two numbers add up to 56. One number is 20 less than the other.

Available for free at Connexions <http://legacy.cnx.org/content/col11759/1.2>

768 CHAPTER 7. SYSTEMS OF EQUATIONS AND INEQUALITIES

Exercise 7.9.53Two numbers add up to 104. If you add two times the �rst number plus two times the secondnumber, your total is 208

Exercise 7.9.54 (Solution on p. 801.)

Three numbers add up to 106. The �rst number is 3 less than the second number. The thirdnumber is 4 more than the �rst number.

Exercise 7.9.55Three numbers add to 216. The sum of the �rst two numbers is 112. The third number is 8 lessthan the �rst two numbers combined.

For the following exercises, create a system of linear equations to describe the behavior. Then, solve thesystem for all solutions using Cramer's Rule.

Exercise 7.9.56 (Solution on p. 802.)

You invest $10,000 into two accounts, which receive 8% interest and 5% interest. At the end of ayear, you had $10,710 in your combined accounts. How much was invested in each account?

Exercise 7.9.57You invest $80,000 into two accounts, $22,000 in one account, and $58,000 in the other account.At the end of one year, assuming simple interest, you have earned $2,470 in interest. The secondaccount receives half a percent less than twice the interest on the �rst account. What are theinterest rates for your accounts?

Exercise 7.9.58 (Solution on p. 802.)

A movie theater needs to know how many adult tickets and children tickets were sold out of the1,200 total tickets. If children's tickets are $5.95, adult tickets are $11.15, and the total amount ofrevenue was $12,756, how many children's tickets and adult tickets were sold?

Exercise 7.9.59A concert venue sells single tickets for $40 each and couple's tickets for $65. If the total revenuewas $18,090 and the 321 tickets were sold, how many single tickets and how many couple's ticketswere sold?

Exercise 7.9.60 (Solution on p. 802.)

You decide to paint your kitchen green. You create the color of paint by mixing yellow and bluepaints. You cannot remember how many gallons of each color went into your mix, but you knowthere were 10 gal total. Additionally, you kept your receipt, and know the total amount spent was$29.50. If each gallon of yellow costs $2.59, and each gallon of blue costs $3.19, how many gallonsof each color go into your green mix?

Exercise 7.9.61You sold two types of scarves at a farmers' market and would like to know which one was morepopular. The total number of scarves sold was 56, the yellow scarf cost $10, and the purple scarfcost $11. If you had total revenue of $583, how many yellow scarves and how many purple scarveswere sold?

Exercise 7.9.62 (Solution on p. 802.)

Your garden produced two types of tomatoes, one green and one red. The red weigh 10 oz, andthe green weigh 4 oz. You have 30 tomatoes, and a total weight of 13 lb, 14 oz. How many of eachtype of tomato do you have?

Exercise 7.9.63At a market, the three most popular vegetables make up 53% of vegetable sales. Corn has 4% highersales than broccoli, which has 5% more sales than onions. What percentage does each vegetablehave in the market share?

Available for free at Connexions <http://legacy.cnx.org/content/col11759/1.2>

769

Exercise 7.9.64 (Solution on p. 802.)

At the same market, the three most popular fruits make up 37% of the total fruit sold. Strawberriessell twice as much as oranges, and kiwis sell one more percentage point than oranges. For eachfruit, �nd the percentage of total fruit sold.

Exercise 7.9.65Three bands performed at a concert venue. The �rst band charged $15 per ticket, the second bandcharged $45 per ticket, and the �nal band charged $22 per ticket. There were 510 tickets sold, for atotal of $12,700. If the �rst band had 40 more audience members than the second band, how manytickets were sold for each band?

Exercise 7.9.66 (Solution on p. 802.)

A movie theatre sold tickets to three movies. The tickets to the �rst movie were $5, the tickets tothe second movie were $11, and the third movie was $12. 100 tickets were sold to the �rst movie.The total number of tickets sold was 642, for a total revenue of $6,774. How many tickets for eachmovie were sold?

Exercise 7.9.67Men aged 20�29, 30�39, and 40�49 made up 78% of the population at a prison last year. This year,the same age groups made up 82.08% of the population. The 20�29 age group increased by 20%,the 30�39 age group increased by 2%, and the 40�49 age group decreased to 3

4 of their previouspopulation. Originally, the 30�39 age group had 2% more prisoners than the 20�29 age group.Determine the prison population percentage for each age group last year.

Exercise 7.9.68 (Solution on p. 802.)

At a women's prison down the road, the total number of inmates aged 20�49 totaled 5,525. Thisyear, the 20�29 age group increased by 10%, the 30�39 age group decreased by 20%, and the 40�49age group doubled. There are now 6,040 prisoners. Originally, there were 500 more in the 30�39age group than the 20�29 age group. Determine the prison population for each age group last year.

For the following exercises, use this scenario: A health-conscious company decides to make a trail mix outof almonds, dried cranberries, and chocolate-covered cashews. The nutritional information for these items isshown in Table 7.7.

Almonds (10) Cranberries (10) Cashews (10)

Fat (g) 6 0.02 7

Protein (g) 2 0 3.5

Carbohydrates(g) 3 8 5.5

Table 7.7

Exercise 7.9.69For the special �low-carb�trail mix, there are 1,000 pieces of mix. The total number of carbohydratesis 425 g, and the total amount of fat is 570.2 g. If there are 200 more pieces of cashews thancranberries, how many of each item is in the trail mix?

Exercise 7.9.70 (Solution on p. 802.)

For the �hiking� mix, there are 1,000 pieces in the mix, containing 390.8 g of fat, and 165 g ofprotein. If there is the same amount of almonds as cashews, how many of each item is in the trailmix?

Exercise 7.9.71For the �energy-booster� mix, there are 1,000 pieces in the mix, containing 145 g of protein and 625g of carbohydrates. If the number of almonds and cashews summed together is equivalent to theamount of cranberries, how many of each item is in the trail mix?

Available for free at Connexions <http://legacy.cnx.org/content/col11759/1.2>

770 CHAPTER 7. SYSTEMS OF EQUATIONS AND INEQUALITIES

7.9.8 Review Exercises

7.9.8.1 Systems of Linear Equations: Two Variables (Section 7.2)

For the following exercises, determine whether the ordered pair is a solution to the system of equations.

Exercise 7.9.72 (Solution on p. 802.)

3x− y = 4

x+ 4y = −3and (−1, 1)

Exercise 7.9.73

6x− 2y = 24

−3x+ 3y = 18and (9, 15)

For the following exercises, use substitution to solve the system of equations.

Exercise 7.9.74 (Solution on p. 802.)

10x+ 5y = −5

3x− 2y = −12

Exercise 7.9.7547x+ 1

5y = 4370

56x−

13y = − 2

3

Exercise 7.9.76 (Solution on p. 802.)

5x+ 6y = 14

4x+ 8y = 8

For the following exercises, use addition to solve the system of equations.

Exercise 7.9.77

3x+ 2y = −7

2x+ 4y = 6

Exercise 7.9.78 (Solution on p. 802.)

3x+ 4y = 2

9x+ 12y = 3

Exercise 7.9.79

8x+ 4y = 2

6x− 5y = 0.7

For the following exercises, write a system of equations to solve each problem. Solve the system of equations.

Exercise 7.9.80 (Solution on p. 802.)

A factory has a cost of productionC (x) = 150x+15,000 and a revenue functionR (x) = 200x.Whatis the break-even point?

Exercise 7.9.81 (Solution on p. 802.)

A performer chargesC (x) = 50x+ 10,000, wherex is the total number of attendees at a show. Thevenue charges $75 per ticket. After how many people buy tickets does the venue break even, andwhat is the value of the total tickets sold at that point?

Available for free at Connexions <http://legacy.cnx.org/content/col11759/1.2>

771

7.9.8.2 Systems of Linear Equations: Three Variables (Section 7.3)

For the following exercises, solve the system of three equations using substitution or addition.

Exercise 7.9.82 (Solution on p. 802.)

0.5x− 0.5y = 10

− 0.2y + 0.2x = 4

0.1x+ 0.1z = 2

Exercise 7.9.83

5x+ 3y − z = 5

3x− 2y + 4z = 13

4x+ 3y + 5z = 22

Exercise 7.9.84 (Solution on p. 802.)

x+ y + z = 1

2x+ 2y + 2z = 1

3x+ 3y = 2

Exercise 7.9.85

2x− 3y + z = −1

x+ y + z = −4

4x+ 2y − 3z = 33

Exercise 7.9.86 (Solution on p. 802.)

3x+ 2y − z = −10

x− y + 2z = 7

−x+ 3y + z = −2

Exercise 7.9.87

3x+ 4z = −11

x− 2y = 5

4y − z = −10

Exercise 7.9.88 (Solution on p. 802.)

2x− 3y + z = 0

2x+ 4y − 3z = 0

6x− 2y − z = 0

Exercise 7.9.89

6x− 4y − 2z = 2

3x+ 2y − 5z = 4

6y − 7z = 5

For the following exercises, write a system of equations to solve each problem. Solve the system of equations.

Exercise 7.9.90 (Solution on p. 802.)

Three odd numbers sum up to 61. The smaller is one-third the larger and the middle number is 16less than the larger. What are the three numbers?

Available for free at Connexions <http://legacy.cnx.org/content/col11759/1.2>

772 CHAPTER 7. SYSTEMS OF EQUATIONS AND INEQUALITIES

Exercise 7.9.91A local theatre sells out for their show. They sell all 500 tickets for a total purse of $8,070.00. Thetickets were priced at $15 for students, $12 for children, and $18 for adults. If the band sold threetimes as many adult tickets as children's tickets, how many of each type was sold?

7.9.8.3 Systems of Nonlinear Equations and Inequalities: Two Variables (Section 7.4)

For the following exercises, solve the system of nonlinear equations.

Exercise 7.9.92 (Solution on p. 802.)

y = x2 − 7

y = 5x− 13

Exercise 7.9.93

y = x2 − 4

y = 5x+ 10

Exercise 7.9.94 (Solution on p. 802.)

x2 + y2 = 16

y = x− 8

Exercise 7.9.95

x2 + y2 = 25

y = x2 + 5

Exercise 7.9.96 (Solution on p. 802.)

x2 + y2 = 4

y − x2 = 3

For the following exercises, graph the inequality.

Exercise 7.9.97y > x2 − 1Exercise 7.9.98 (Solution on p. 802.)14x

2 + y2 < 4For the following exercises, graph the system of inequalities.

Exercise 7.9.99

x2 + y2 + 2x < 3

y > − x2 − 3

Exercise 7.9.100 (Solution on p. 803.)

x2 − 2x+ y2 − 4x < 4

y < − x+ 4

Exercise 7.9.101

x2 + y2 < 1

y2 < x

Available for free at Connexions <http://legacy.cnx.org/content/col11759/1.2>

773

7.9.8.4 Partial Fractions (Section 7.5)

For the following exercises, decompose into partial fractions.

Exercise 7.9.102 (Solution on p. 803.)−2x+6x2+3x+2

Exercise 7.9.10310x+2

4x2+4x+1

Exercise 7.9.104 (Solution on p. 804.)7x+20

x2+10x+25

Exercise 7.9.105x−18

x2−12x+36

Exercise 7.9.106 (Solution on p. 804.)−x2+36x+70x3−125

Exercise 7.9.107−5x2+6x−2x3+27

Exercise 7.9.108 (Solution on p. 804.)x3−4x2+3x+11

(x2−2)2

Exercise 7.9.1094x4−2x3+22x2−6x+48

x(x2+4)2

7.9.8.5 Matrices and Matrix Operations (Section 7.6)

For the following exercises, perform the requested operations on the given matrices.

A =

[4 −2

1 3

], B =

[6 7 −3

11 −2 4

], C =

6 7

11 −2

14 0

, D =

1 −4 9

10 5 −7

2 8 5

, E =

7 −14 3

2 −1 3

0 1 9

(7.430)

Exercise 7.9.110 (Solution on p. 804.)

−4AExercise 7.9.11110D − 6EExercise 7.9.112 (Solution on p. 804.)

B + C

Exercise 7.9.113AB

Exercise 7.9.114 (Solution on p. 804.)

BA

Exercise 7.9.115BC

Available for free at Connexions <http://legacy.cnx.org/content/col11759/1.2>

774 CHAPTER 7. SYSTEMS OF EQUATIONS AND INEQUALITIES

Exercise 7.9.116 (Solution on p. 804.)

CB

Exercise 7.9.117DE

Exercise 7.9.118 (Solution on p. 804.)

ED

Exercise 7.9.119EC

Exercise 7.9.120 (Solution on p. 804.)

CE

Exercise 7.9.121A3

7.9.8.6 Solving Systems with Gaussian Elimination (Section 7.7)

For the following exercises, write the system of linear equations from the augmented matrix. Indicate whetherthere will be a unique solution.

Exercise 7.9.122 (Solution on p. 804.)1 0 −3

0 1 2

0 0 0

|7

−5

0

Exercise 7.9.123

1 0 5

0 1 −2

0 0 0

|−9

4

3

For the following exercises, write the augmented matrix from the system of linear equations.

Exercise 7.9.124 (Solution on p. 804.)

−2x+ 2y + z = 7

2x− 8y + 5z = 0

19x− 10y + 22z = 3

Exercise 7.9.125

4x+ 2y − 3z = 14

−12x+ 3y + z = 100

9x− 6y + 2z = 31

Exercise 7.9.126 (Solution on p. 804.)

x+ 3z = 12

−x+ 4y = 0

y + 2z = −7

For the following exercises, solve the system of linear equations using Gaussian elimination.

Available for free at Connexions <http://legacy.cnx.org/content/col11759/1.2>

775

Exercise 7.9.127

3x− 4y = −7

−6x+ 8y = 14

Exercise 7.9.128 (Solution on p. 804.)

3x− 4y = 1

−6x+ 8y = 6

Exercise 7.9.129

−1.1x− 2.3y = 6.2

−5.2x− 4.1y = 4.3

Exercise 7.9.130 (Solution on p. 804.)

2x+ 3y + 2z = 1

−4x− 6y − 4z = −2

10x+ 15y + 10z = 0

Exercise 7.9.131

−x+ 2y − 4z = 8

3y + 8z = −4

−7x+ y + 2z = 1

7.9.8.7 Solving Systems with Inverses (Section 7.8)

For the following exercises, �nd the inverse of the matrix.

Exercise 7.9.132 (Solution on p. 804.) −0.2 1.4

1.2 −0.4

Exercise 7.9.133 1

2 − 12

− 14

34

Exercise 7.9.134 (Solution on p. 804.)

12 9 −6

−1 3 2

−4 −3 2

Exercise 7.9.135

2 1 3

1 2 3

3 2 1

For the following exercises, �nd the solutions by computing the inverse of the matrix.

Exercise 7.9.136 (Solution on p. 805.)

0.3x− 0.1y = −10

−0.1x+ 0.3y = 14

Available for free at Connexions <http://legacy.cnx.org/content/col11759/1.2>

776 CHAPTER 7. SYSTEMS OF EQUATIONS AND INEQUALITIES

Exercise 7.9.137

0.4x− 0.2y = −0.6

−0.1x+ 0.05y = 0.3

Exercise 7.9.138 (Solution on p. 805.)

4x+ 3y − 3z = −4.3

5x− 4y − z = −6.1

x+ z = −0.7

Exercise 7.9.139

−2x− 3y + 2z = 3

−x+ 2y + 4z = −5

−2y + 5z = −3

For the following exercises, write a system of equations to solve each problem. Solve the system of equations.

Exercise 7.9.140 (Solution on p. 805.)

Students were asked to bring their favorite fruit to class. 90% of the fruits consisted of banana,apple, and oranges. If oranges were half as popular as bananas and apples were 5% more popularthan bananas, what are the percentages of each individual fruit?

Exercise 7.9.141A sorority held a bake sale to raise money and sold brownies and chocolate chip cookies. Theypriced the brownies at $2 and the chocolate chip cookies at $1. They raised $250 and sold 175items. How many brownies and how many cookies were sold?

7.9.8.8 Solving Systems with Cramer's Rule (Section 7.9)

For the following exercises, �nd the determinant.

Exercise 7.9.142 (Solution on p. 805.)

|100 0

0 0|

Exercise 7.9.143

|0.2 −0.6

0.7 −1.1|

Exercise 7.9.144 (Solution on p. 805.)

|−1 4 3

0 2 3

0 0 −3

|

Exercise 7.9.145

|

√2 0 0

0√

2 0

0 0√

2

|

For the following exercises, use Cramer's Rule to solve the linear systems of equations.

Available for free at Connexions <http://legacy.cnx.org/content/col11759/1.2>

777

Exercise 7.9.146 (Solution on p. 805.)

4x− 2y = 23

−5x− 10y = −35

Exercise 7.9.147

0.2x− 0.1y = 0

−0.3x+ 0.3y = 2.5

Exercise 7.9.148 (Solution on p. 805.)

−0.5x+ 0.1y = 0.3

−0.25x+ 0.05y = 0.15

Exercise 7.9.149

x+ 6y + 3z = 4

2x+ y + 2z = 3

3x− 2y + z = 0

Exercise 7.9.150 (Solution on p. 805.)

4x− 3y + 5z = − 52

7x− 9y − 3z = 32

x− 5y − 5z = 52

Exercise 7.9.151310x−

15y −

310z = − 1

50

110x−

110y −

12z = − 9

50

25x−

12y −

35z = − 1

5

7.9.9 Practice Test

Is the following ordered pair a solution to the system of equations?

Exercise 7.9.152 (Solution on p. 805.)

−5x− y = 12

x+ 4y = 9

with (−3, 3)

For the following exercises, solve the systems of linear and nonlinear equations using substitution or elimi-nation. Indicate if no solution exists.

Exercise 7.9.15312x−

13y = 4

32x− y = 0

Exercise 7.9.154 (Solution on p. 805.)

− 12x− 4y = 4

2x+ 16y = 2

Available for free at Connexions <http://legacy.cnx.org/content/col11759/1.2>

778 CHAPTER 7. SYSTEMS OF EQUATIONS AND INEQUALITIES

Exercise 7.9.155

5x− y = 1

−10x+ 2y = −2

Exercise 7.9.156 (Solution on p. 805.)

4x− 6y − 2z = 110

x− 7y + 5z = − 14

3x+ 6y − 9z = 65

Exercise 7.9.157

x+ z = 20

x+ y + z = 20

x+ 2y + z = 10

Exercise 7.9.158 (Solution on p. 805.)

5x− 4y − 3z = 0

2x+ y + 2z = 0

x− 6y − 7z = 0

Exercise 7.9.159

y = x2 + 2x− 3

y = x− 1

Exercise 7.9.160 (Solution on p. 805.)

y2 + x2 = 25

y2 − 2x2 = 1

For the following exercises, graph the following inequalities.

Exercise 7.9.161y < x2 + 9Exercise 7.9.162 (Solution on p. 805.)

x2 + y2 > 4

y < x2 + 1

For the following exercises, write the partial fraction decomposition.

Exercise 7.9.163−8x−30

x2+10x+25

Exercise 7.9.164 (Solution on p. 806.)13x+2

(3x+1)2

Exercise 7.9.165x4−x3+2x−1x(x2+1)2

For the following exercises, perform the given matrix operations.

Exercise 7.9.166 (Solution on p. 806.)

5

4 9

−2 3

+ 12

−6 12

4 −8

Available for free at Connexions <http://legacy.cnx.org/content/col11759/1.2>

779

Exercise 7.9.1671 4 −7

−2 9 5

12 0 −4

3 −4

1 3

5 10

Exercise 7.9.168 (Solution on p. 806.) 1

213

14

15

−1

Exercise 7.9.169

det|0 0

400 4,000|

Exercise 7.9.170 (Solution on p. 806.)

det|

12 − 1

2 0

− 12 0 1

2

0 12 0

|

Exercise 7.9.171If det (A) = −6, what would be the determinant if you switched rows 1 and 3, multiplied the secondrow by 12, and took the inverse?

Exercise 7.9.172 (Solution on p. 806.)

Rewrite the system of linear equations as an augmented matrix.

14x− 2y + 13z = 140

−2x+ 3y − 6z = −1

x− 5y + 12z = 11

(7.431)

Exercise 7.9.173Rewrite the augmented matrix as a system of linear equations.

1 0 3

−2 4 9

−6 1 2

|12

−5

8

(7.432)

For the following exercises, use Gaussian elimination to solve the systems of equations.

Exercise 7.9.174 (Solution on p. 806.)

x− 6y = 4

2x− 12y = 0

Exercise 7.9.175

2x+ y + z = −3

x− 2y + 3z = 6

x− y − z = 6

For the following exercises, use the inverse of a matrix to solve the systems of equations.

Available for free at Connexions <http://legacy.cnx.org/content/col11759/1.2>

780 CHAPTER 7. SYSTEMS OF EQUATIONS AND INEQUALITIES

Exercise 7.9.176 (Solution on p. 806.)

4x− 5y = −50

−x+ 2y = 80

Exercise 7.9.1771

100x−3

100y + 120z = −49

3100x−

7100y −

1100z = 13

9100x−

9100y −

9100z = 99

For the following exercises, use Cramer's Rule to solve the systems of equations.

Exercise 7.9.178 (Solution on p. 806.)

200x− 300y = 2

400x+ 715y = 4

Exercise 7.9.179

0.1x+ 0.1y − 0.1z = −1.2

0.1x− 0.2y + 0.4z = −1.2

0.5x− 0.3y + 0.8z = −5.9

For the following exercises, solve using a system of linear equations.

Exercise 7.9.180 (Solution on p. 807.)

A factory producing cell phones has the following cost and revenue functions:C (x) = x2 + 75x +2,688 andR (x) = x2 +160x.What is the range of cell phones they should produce each day so thereis pro�t? Round to the nearest number that generates pro�t.

Exercise 7.9.181A small fair charges $1.50 for students, $1 for children, and $2 for adults. In one day, three timesas many children as adults attended. A total of 800 tickets were sold for a total revenue of $1,050.How many of each type of ticket was sold?

Available for free at Connexions <http://legacy.cnx.org/content/col11759/1.2>

781

Solutions to Exercises in Chapter 7

Solution to Exercise 7.2.1 (p. 603)Not a solution.Solution to Exercise 7.2.2 (p. 605)The solution to the system is the ordered pair (−5, 3) .

So-lution to Exercise 7.2.3 (p. 607)(−2,−5)Solution to Exercise 7.2.4 (p. 611)(−6,−2)Solution to Exercise 7.2.5 (p. 614)

(10,−4)

Solution to Exercise 7.2.6 (p. 616)No solution. It is an inconsistent system.Solution to Exercise 7.2.7 (p. 618)The system is dependent so there are in�nite solutions of the form (x, 2x+ 5) .Solution to Exercise 7.2.8 (p. 623)700 children, 950 adultsSolution to Exercise 7.2.9 (p. 624)No, you can either have zero, one, or in�nitely many. Examine graphs.

Available for free at Connexions <http://legacy.cnx.org/content/col11759/1.2>

782 CHAPTER 7. SYSTEMS OF EQUATIONS AND INEQUALITIES

Solution to Exercise 7.2.11 (p. 624)This means there is no realistic break-even point. By the time the company produces one unit they arealready making pro�t.Solution to Exercise 7.2.13 (p. 624)You can solve by substitution (isolatingx or y ), graphically, or by addition.Solution to Exercise 7.2.15 (p. 624)YesSolution to Exercise 7.2.17 (p. 625)YesSolution to Exercise 7.2.19 (p. 625)(−1, 2)Solution to Exercise 7.2.21 (p. 625)(−3, 1)Solution to Exercise 7.2.23 (p. 625)(− 3

5 , 0)

Solution to Exercise 7.2.25 (p. 625)No solutions exist.Solution to Exercise 7.2.27 (p. 626)(

725 ,

1325

)Solution to Exercise 7.2.29 (p. 626)(6,−6)Solution to Exercise 7.2.31 (p. 626)(− 1

2 ,110

)Solution to Exercise 7.2.33 (p. 626)No solutions exist.Solution to Exercise 7.2.35 (p. 626)(− 1

5 ,23

)Solution to Exercise 7.2.37 (p. 626)(x, x+3

2

)Solution to Exercise 7.2.39 (p. 627)(−4, 4)Solution to Exercise 7.2.41 (p. 627)(

12 ,

18

)Solution to Exercise 7.2.43 (p. 627)(

16 , 0)

Solution to Exercise 7.2.45 (p. 627)(x, 2 (7x− 6))Solution to Exercise 7.2.47 (p. 627)(− 5

6 ,43

)Solution to Exercise 7.2.49 (p. 628)Consistent with one solutionSolution to Exercise 7.2.51 (p. 628)Consistent with one solutionSolution to Exercise 7.2.53 (p. 628)Dependent with in�nitely many solutionsSolution to Exercise 7.2.55 (p. 628)(−3.08, 4.91)Solution to Exercise 7.2.57 (p. 628)(−1.52, 2.29)Solution to Exercise 7.2.59 (p. 629)(A+B

2 , A−B2

)Available for free at Connexions <http://legacy.cnx.org/content/col11759/1.2>

783

Solution to Exercise 7.2.61 (p. 629)(−1A−B ,

AA−B

)Solution to Exercise 7.2.63 (p. 629)(CE−BFBD−AE ,

AF−CDBD−AE

)Solution to Exercise 7.2.65 (p. 629)They never turn a pro�t.Solution to Exercise 7.2.67 (p. 629)(1, 250, 100, 000)Solution to Exercise 7.2.69 (p. 629)The numbers are 7.5 and 20.5.Solution to Exercise 7.2.71 (p. 630)24,000Solution to Exercise 7.2.73 (p. 630)790 sophomores, 805 freshmanSolution to Exercise 7.2.75 (p. 630)56 men, 74 womenSolution to Exercise 7.2.77 (p. 630)10 gallons of 10% solution, 15 gallons of 60% solutionSolution to Exercise 7.2.79 (p. 630)Swan Peak: $750,000, Riverside: $350,000Solution to Exercise 7.2.81 (p. 630)$12,500 in the �rst account, $10,500 in the second account.Solution to Exercise 7.2.83 (p. 631)High-tops: 45, Low-tops: 15Solution to Exercise 7.2.85 (p. 631)In�nitely many solutions. We need more information.Solution to Exercise 7.3.1 (p. 639)(1,−1, 1)Solution to Exercise 7.3.2 (p. 640)No solution.Solution to Exercise 7.3.3 (p. 642)In�nite number of solutions of the form (x, 4x− 11,−5x+ 18) .Solution to Exercise 7.3.4 (p. 643)No, there can be only one, zero, or in�nitely many solutions.Solution to Exercise 7.3.6 (p. 643)Not necessarily. There could be zero, one, or in�nitely many solutions. For example, (0, 0, 0) is not asolution to the system below, but that does not mean that it has no solution.

2x+ 3y − 6z = 1

−4x− 6y + 12z = −2

x+ 2y + 5z = 10Solution to Exercise 7.3.8 (p. 643)Every system of equations can be solved graphically, by substitution, and by addition. However, systemsof three equations become very complex to solve graphically so other methods are usually preferable.Solution to Exercise 7.3.10 (p. 644)NoSolution to Exercise 7.3.12 (p. 644)YesSolution to Exercise 7.3.14 (p. 644)(−1, 4, 2)

Available for free at Connexions <http://legacy.cnx.org/content/col11759/1.2>

784 CHAPTER 7. SYSTEMS OF EQUATIONS AND INEQUALITIES

Solution to Exercise 7.3.16 (p. 644)(− 85

107 ,312107 ,

191107

)Solution to Exercise 7.3.18 (p. 645)(

1, 12 , 0)

Solution to Exercise 7.3.20 (p. 645)(4,−6, 1)Solution to Exercise 7.3.22 (p. 645)(x, 1

27 (65− 16x) , x+2827

)Solution to Exercise 7.3.24 (p. 645)(− 45

13 ,1713 ,−2

)Solution to Exercise 7.3.26 (p. 645)No solutions existSolution to Exercise 7.3.28 (p. 646)(0, 0, 0)Solution to Exercise 7.3.30 (p. 646)(

47 ,−

17 ,−

37

)Solution to Exercise 7.3.32 (p. 646)(7, 20, 16)Solution to Exercise 7.3.34 (p. 646)(−6, 2, 1)Solution to Exercise 7.3.36 (p. 647)(5, 12, 15)Solution to Exercise 7.3.38 (p. 647)(−5,−5,−5)Solution to Exercise 7.3.40 (p. 647)(10, 10, 10)Solution to Exercise 7.3.42 (p. 647)(

12 ,

15 ,

45

)Solution to Exercise 7.3.44 (p. 647)(

12 ,

25 ,

45

)Solution to Exercise 7.3.46 (p. 648)(2, 0, 0)Solution to Exercise 7.3.48 (p. 648)(1, 1, 1)Solution to Exercise 7.3.50 (p. 648)(

128557 ,

23557 ,

28557

)Solution to Exercise 7.3.52 (p. 648)(6,−1, 0)Solution to Exercise 7.3.54 (p. 649)24, 36, 48Solution to Exercise 7.3.56 (p. 649)70 grandparents, 140 parents, 190 childrenSolution to Exercise 7.3.58 (p. 649)Your share was $19.95, Sarah's share was $40, and your other roommate's share was $22.05.Solution to Exercise 7.3.60 (p. 649)There are in�nitely many solutions; we need more informationSolution to Exercise 7.3.62 (p. 650)500 students, 225 children, and 450 adultsSolution to Exercise 7.3.64 (p. 650)The BMW was $49,636, the Jeep was $42,636, and the Toyota was $47,727.

Available for free at Connexions <http://legacy.cnx.org/content/col11759/1.2>

785

Solution to Exercise 7.3.66 (p. 650)$400,000 in the account that pays 3% interest, $500,000 in the account that pays 4% interest, and $100,000in the account that pays 2% interest.Solution to Exercise 7.3.68 (p. 650)The United States consumed 26.3%, Japan 7.1%, and China 6.4% of the world's oil.Solution to Exercise 7.3.70 (p. 650)Saudi Arabia imported 16.8%, Canada imported 15.1%, and Mexico 15.0%Solution to Exercise 7.3.72 (p. 651)Birds were 19.3%, �sh were 18.6%, and mammals were 17.1% of endangered speciesSolution to Exercise 7.4.1 (p. 656)(− 1

2 ,12

)and (2, 8)

Solution to Exercise 7.4.2 (p. 658)(−1, 3)Solution to Exercise 7.4.3 (p. 661){(1, 3) , (1,−3) , (−1, 3) , (−1,−3)}Solution to Exercise 7.4.4 (p. 665)Shade the area bounded by the two curves, above the quadratic and below the line.

So-

Available for free at Connexions <http://legacy.cnx.org/content/col11759/1.2>

786 CHAPTER 7. SYSTEMS OF EQUATIONS AND INEQUALITIES

lution to Exercise 7.4.5 (p. 666)A nonlinear system could be representative of two circles that overlap and intersect in two locations, hencetwo solutions. A nonlinear system could be representative of a parabola and a circle, where the vertex ofthe parabola meets the circle and the branches also intersect the circle, hence three solutions.Solution to Exercise 7.4.7 (p. 666)No. There does not need to be a feasible region. Consider a system that is bounded by two parallel lines.One inequality represents the region above the upper line; the other represents the region below the lowerline. In this case, no points in the plane are located in both regions; hence there is no feasible region.Solution to Exercise 7.4.9 (p. 666)Choose any number between each solution and plug intoC (x) andR (x) . IfC (x) < R (x) , then there ispro�t.Solution to Exercise 7.4.11 (p. 667)(0,−3) , (3, 0)Solution to Exercise 7.4.13 (p. 667)(− 3√

22 , 3

√2

2

),(

3√

22 ,− 3

√2

2

)Solution to Exercise 7.4.15 (p. 667)(−3, 0) , (3, 0)Solution to Exercise 7.4.17 (p. 667)(

14 ,−

√628

),(

14 ,√

628

)Solution to Exercise 7.4.19 (p. 667)(−√

3984 , 199

4

),(√

3984 , 199

4

)Solution to Exercise 7.4.21 (p. 668)(0, 2) , (1, 3)Solution to Exercise 7.4.23 (p. 668)(−√

12

(√5− 1

), 1

2

(1−√

5))

,

(√12

(√5− 1

), 1

2

(1−√

5))

Solution to Exercise 7.4.25 (p. 668)(5, 0)Solution to Exercise 7.4.27 (p. 668)(0, 0)Solution to Exercise 7.4.29 (p. 668)(3, 0)Solution to Exercise 7.4.31 (p. 668)No Solutions ExistSolution to Exercise 7.4.33 (p. 668)No Solutions ExistSolution to Exercise 7.4.35 (p. 669)(−√

22 ,−

√2

2

),(−√

22 ,√

22

),(√

22 ,−

√2

2

),(√

22 ,√

22

)Solution to Exercise 7.4.37 (p. 669)(2, 0)Solution to Exercise 7.4.39 (p. 669)(−√

7,−3),(−√

7, 3),(√

7,−3),(√

7, 3)

Solution to Exercise 7.4.41 (p. 669)(−√

12

(√73− 5

), 1

2

(7−√

73))

,

(√12

(√73− 5

), 1

2

(7−√

73))

Available for free at Connexions <http://legacy.cnx.org/content/col11759/1.2>

787

Solution to Exercise 7.4.43 (p. 669)

Solu-

Available for free at Connexions <http://legacy.cnx.org/content/col11759/1.2>

788 CHAPTER 7. SYSTEMS OF EQUATIONS AND INEQUALITIES

tion to Exercise 7.4.45 (p. 669)

Solu-

Available for free at Connexions <http://legacy.cnx.org/content/col11759/1.2>

789

tion to Exercise 7.4.47 (p. 670)

Solu-

Available for free at Connexions <http://legacy.cnx.org/content/col11759/1.2>

790 CHAPTER 7. SYSTEMS OF EQUATIONS AND INEQUALITIES

tion to Exercise 7.4.49 (p. 670)

Solu-

Available for free at Connexions <http://legacy.cnx.org/content/col11759/1.2>

791

tion to Exercise 7.4.51 (p. 670)

Solu-tion to Exercise 7.4.53 (p. 670)(−2√

70383 ,−2

√3529

),(−2√

70383 , 2

√3529

),(

2√

70383 ,−2

√3529

),(

2√

70383 , 2

√3529

)Solution to Exercise 7.4.55 (p. 670)No Solution ExistsSolution to Exercise 7.4.57 (p. 671)x = 0, y > 0 and0 < x < 1,

√x < y < 1

xSolution to Exercise 7.4.59 (p. 671)12, 288Solution to Exercise 7.4.61 (p. 671)2�20 computersSolution to Exercise 7.5.1 (p. 674)

3x−3 −

2x−2

Solution to Exercise 7.5.2 (p. 676)6

x−1 −5

(x−1)2

Solution to Exercise 7.5.3 (p. 679)3

x−1 + 2x−4x2+1

Solution to Exercise 7.5.4 (p. 681)x−2

x2−2x+3 + 2x+1(x2−2x+3)2

Solution to Exercise 7.5.5 (p. 682)No, a quotient of polynomials can only be decomposed if the denominator can be factored. Forexample, 1

x2+1 cannot be decomposed because the denominator cannot be factored.Solution to Exercise 7.5.7 (p. 682)Graph both sides and ensure they are equal.Solution to Exercise 7.5.9 (p. 682)If we choosex = −1, then the B-term disappears, letting us immediately know thatA = 3.We could alter-natively plug inx = − 5

3 , giving us a B-value of − 2.Solution to Exercise 7.5.11 (p. 682)

8x+3 −

5x−8

Available for free at Connexions <http://legacy.cnx.org/content/col11759/1.2>

792 CHAPTER 7. SYSTEMS OF EQUATIONS AND INEQUALITIES

Solution to Exercise 7.5.13 (p. 682)1

x+5 + 9x+2

Solution to Exercise 7.5.15 (p. 682)3

5x−2 + 44x−1

Solution to Exercise 7.5.17 (p. 682)5

2(x+3) + 52(x−3)

Solution to Exercise 7.5.19 (p. 683)3

x+2 + 3x−2

Solution to Exercise 7.5.21 (p. 683)9

5(x+2) + 115(x−3)

Solution to Exercise 7.5.23 (p. 683)8

x−3 −5

x−2Solution to Exercise 7.5.25 (p. 683)

1x−2 + 2

(x−2)2

Solution to Exercise 7.5.27 (p. 683)− 6

4x+5 + 3(4x+5)2

Solution to Exercise 7.5.29 (p. 683)− 1x−7 −

2(x−7)2

Solution to Exercise 7.5.31 (p. 683)4x −

32(x+1) + 7

2(x+1)2

Solution to Exercise 7.5.33 (p. 683)4x + 2

x2 − 33x+2 + 7

2(3x+2)2

Solution to Exercise 7.5.35 (p. 683)x+1

x2+x+3 + 3x+2

Solution to Exercise 7.5.37 (p. 683)4−3x

x2+3x+8 + 1x−1

Solution to Exercise 7.5.39 (p. 684)2x−1

x2+6x+1 + 2x+3

Solution to Exercise 7.5.41 (p. 684)1

x2+x+1 + 4x−1

Solution to Exercise 7.5.43 (p. 684)2

x2−3x+9 + 3x+3

Solution to Exercise 7.5.45 (p. 684)− 1

4x2+6x+9 + 12x−3

Solution to Exercise 7.5.47 (p. 684)1x + 1

x+6 −4x

x2−6x+36Solution to Exercise 7.5.49 (p. 684)x+6x2+1 + 4x+3

(x2+1)2

Solution to Exercise 7.5.51 (p. 684)x+1x+2 + 2x+3

(x+2)2

Solution to Exercise 7.5.53 (p. 684)1

x2+3x+25 −3x

(x2+3x+25)2

Solution to Exercise 7.5.55 (p. 684)1

8x −x

8(x2+4) + 10−x2(x2+4)2

Solution to Exercise 7.5.57 (p. 684)− 16

x −9x2 + 16

x−1 −7

(x−1)2

Solution to Exercise 7.5.59 (p. 685)1

x+1 −2

(x+1)2+ 5

(x+1)3

Available for free at Connexions <http://legacy.cnx.org/content/col11759/1.2>

793

Solution to Exercise 7.5.61 (p. 685)5

x−2 −3

10(x+2) + 7x+8 −

710(x−8)

Solution to Exercise 7.5.63 (p. 685)− 5

4x −5

2(x+2) + 112(x+4) + 5

4(x+4)

Solution to Exercise 7.6.1 (p. 691)

A+B =

2

1

1

6

0

− 3

+

3

1

−4

− 2

5

3

=

2 + 3

1 + 1

1 + (−4)

6 + (−2)

0 + 5

−3 + 3

=

5

2

−3

4

5

0

(7.433)

Solution to Exercise 7.6.2 (p. 694)

−2B =

−8 −2

−6 −4

Solution to Exercise 7.6.3 (p. 701)No, they must have the same dimensions. An example would include two matrices of di�erent dimensions.One cannot add the following two matrices because the �rst is a 2 × 2matrix and the second is a 2 ×

3matrix.

1 2

3 4

+

6 5 4

3 2 1

has no sum.

Solution to Exercise 7.6.5 (p. 701)Yes, if the dimensions ofA arem× n and the dimensions ofB aren×m, both products will be de�ned.Solution to Exercise 7.6.7 (p. 701)Not necessarily. To �ndAB, we multiply the �rst row ofA by the �rst column ofB to get the �rst entryofAB.To �ndBA, we multiply the �rst row ofB by the �rst column ofA to get the �rst entry ofBA.Thus,if those are unequal, then the matrix multiplication does not commute.Solution to Exercise 7.6.9 (p. 701)

11 19

15 94

17 67

Solution to Exercise 7.6.11 (p. 702) −4 2

8 1

Solution to Exercise 7.6.13 (p. 702)Undidenti�ed; dimensions do not matchSolution to Exercise 7.6.15 (p. 702)

9 27

63 36

0 192

Solution to Exercise 7.6.17 (p. 702) −64 −12 −28 −72

−360 −20 −12 −116

Available for free at Connexions <http://legacy.cnx.org/content/col11759/1.2>

794 CHAPTER 7. SYSTEMS OF EQUATIONS AND INEQUALITIES

Solution to Exercise 7.6.19 (p. 702)1, 800 1, 200 1, 300

800 1, 400 600

700 400 2, 100

Solution to Exercise 7.6.21 (p. 702) 20 102

28 28

Solution to Exercise 7.6.23 (p. 702) 60 41 2

−16 120 −216

Solution to Exercise 7.6.25 (p. 702)−68 24 136

−54 −12 64

−57 30 128

Solution to Exercise 7.6.27 (p. 703)Unde�ned; dimensions do not match.Solution to Exercise 7.6.29 (p. 703)−8 41 −3

40 −15 −14

4 27 42

Solution to Exercise 7.6.31 (p. 703)−840 650 −530

330 360 250

−10 900 110

Solution to Exercise 7.6.33 (p. 703) −350 1, 050

350 350

Solution to Exercise 7.6.35 (p. 703)Unde�ned; inner dimensions do not match.Solution to Exercise 7.6.37 (p. 703) 1, 400 700

−1, 400 700

Solution to Exercise 7.6.39 (p. 703) 332, 500 927, 500

−227, 500 87, 500

Solution to Exercise 7.6.41 (p. 704) 490, 000 0

0 490, 000

Available for free at Connexions <http://legacy.cnx.org/content/col11759/1.2>

795

Solution to Exercise 7.6.43 (p. 704) −2 3 4

−7 9 −7

Solution to Exercise 7.6.45 (p. 704) −4 29 21

−27 −3 1

Solution to Exercise 7.6.47 (p. 704)−3 −2 −2

−28 59 46

−4 16 7

Solution to Exercise 7.6.49 (p. 704)

1 −18 −9

−198 505 369

−72 126 91

Solution to Exercise 7.6.51 (p. 704) 0 1.6

9 −1

Solution to Exercise 7.6.53 (p. 704)

2 24 −4.5

12 32 −9

−8 64 61

Solution to Exercise 7.6.55 (p. 704)

0.5 3 0.5

2 1 2

10 7 10

Solution to Exercise 7.6.57 (p. 705)

1 0 0

0 1 0

0 0 1

Solution to Exercise 7.6.59 (p. 705)

1 0 0

0 1 0

0 0 1

Available for free at Connexions <http://legacy.cnx.org/content/col11759/1.2>

796 CHAPTER 7. SYSTEMS OF EQUATIONS AND INEQUALITIES

Solution to Exercise 7.6.61 (p. 705)

Bn = {

1 0 0

0 1 0

0 0 1

, n even,

1 0 0

0 0 1

0 1 0

, n odd.

Solution to Exercise 7.7.1 (p. 708) 4 −3

3 2|

11

4

Solution to Exercise 7.7.2 (p. 709)

x − y + z = 5

2x − y + 3z = 1

y + z = −9Solution to Exercise 7.7.3 (p. 711)(2, 1)Solution to Exercise 7.7.4 (p. 714)

1 − 52

52

0 1 5

0 0 1

|

172

9

2

Solution to Exercise 7.7.5 (p. 717)(1, 1, 1)Solution to Exercise 7.7.6 (p. 720)$150,000 at 7%, $750,000 at 8%, $600,000 at 10%Solution to Exercise 7.7.7 (p. 720)Yes. For each row, the coe�cients of the variables are written across the corresponding row, and a verticalbar is placed; then the constants are placed to the right of the vertical bar.Solution to Exercise 7.7.9 (p. 721)No, there are numerous correct methods of using row operations on a matrix. Two possible ways are thefollowing: (1) Interchange rows 1 and 2. ThenR2 = R2 − 9R1. (2)R2 = R1 − 9R2.Then divide row 1 by 9.Solution to Exercise 7.7.11 (p. 721)No. A matrix with 0 entries for an entire row would have either zero or in�nitely many solutions.Solution to Exercise 7.7.13 (p. 721) 0 16

9 −1|

4

2

Solution to Exercise 7.7.15 (p. 721)

1 5 8

12 3 0

3 4 9

|16

4

−7

Available for free at Connexions <http://legacy.cnx.org/content/col11759/1.2>

797

Solution to Exercise 7.7.17 (p. 721)

−2x+ 5y = 5

6x− 18y = 26Solution to Exercise 7.7.19 (p. 721)

3x+ 2y = 13

−x− 9y + 4z = 53

8x+ 5y + 7z = 80Solution to Exercise 7.7.21 (p. 722)

4x+ 5y − 2z = 12

y + 58z = 2

8x+ 7y − 3z = −5Solution to Exercise 7.7.23 (p. 722)No solutionsSolution to Exercise 7.7.25 (p. 722)(−1,−2)Solution to Exercise 7.7.27 (p. 722)(6, 7)Solution to Exercise 7.7.29 (p. 722)(3, 2)Solution to Exercise 7.7.31 (p. 723)(

15 ,

12

)Solution to Exercise 7.7.33 (p. 723)(x, 4

15 (5x+ 1))

Solution to Exercise 7.7.35 (p. 723)(3, 4)Solution to Exercise 7.7.37 (p. 723)(

19639 ,−

513

)Solution to Exercise 7.7.39 (p. 723)(31,−42, 87)Solution to Exercise 7.7.41 (p. 723)(

2140 ,

120 ,

98

)Solution to Exercise 7.7.43 (p. 724)(

1813 ,

1513 ,−

1513

)Solution to Exercise 7.7.45 (p. 724)(x, y, 1

2 (1− 2x− 3y))

Solution to Exercise 7.7.47 (p. 724)(x,−x2 ,−1

)Solution to Exercise 7.7.49 (p. 724)(125,−25, 0)Solution to Exercise 7.7.51 (p. 725)(8, 1,−2)Solution to Exercise 7.7.53 (p. 725)(1, 2, 3)Solution to Exercise 7.7.55 (p. 725)(x, 31

28 −3x4 ,

128 (−7x− 3)

)Solution to Exercise 7.7.57 (p. 725)No solutions exist.

Available for free at Connexions <http://legacy.cnx.org/content/col11759/1.2>

798 CHAPTER 7. SYSTEMS OF EQUATIONS AND INEQUALITIES

Solution to Exercise 7.7.59 (p. 726)860 red velvet, 1,340 chocolateSolution to Exercise 7.7.61 (p. 726)4% for account 1, 6% for account 2Solution to Exercise 7.7.63 (p. 726)$126Solution to Exercise 7.7.65 (p. 726)Banana was 3%, pumpkin was 7%, and rocky road was 2%Solution to Exercise 7.7.67 (p. 726)100 almonds, 200 cashews, 600 pistachiosSolution to Exercise 7.8.1 (p. 729)

AB =

[1 4

−1 −3

] [−3 −4

1 1

]=

[1 (−3) + 4 (1) 1 (−4) + 4 (1)

−1 (−3) +−3 (1) −1 (−4) +−3 (1)

]=

[1 0

0 1

]

BA =

[−3 −4

1 1

] [1 4

−1 −3

]=

[−3 (1) +−4 (−1) −3 (4) +−4 (−3)

1 (1) + 1 (−1) 1 (4) + 1 (−3)

]=

[1 0

0 1

](7.434)

Solution to Exercise 7.8.2 (p. 733)

A−1 =

35

15

− 25

15

Solution to Exercise 7.8.3 (p. 736)

A−1 =

1 1 2

2 4 −3

3 6 −5

Solution to Exercise 7.8.4 (p. 742)

X =

4

38

58

Solution to Exercise 7.8.5 (p. 743)IfA−1 is the inverse ofA, thenAA−1 = I, the identity matrix. SinceA is also the inverse ofA−1, A−1A =I.You can also check by proving this for a 2× 2matrix.Solution to Exercise 7.8.7 (p. 744)No, because ad and bc are both 0, so ad− bc = 0, which requires us to divide by 0 in the formula.Solution to Exercise 7.8.9 (p. 744)

Yes. Consider the matrix

0 1

1 0

.The inverse is found with the following calculation:A−1 =

10(0)−1(1)

0 −1

−1 0

=

0 1

1 0

.

Available for free at Connexions <http://legacy.cnx.org/content/col11759/1.2>

799

Solution to Exercise 7.8.11 (p. 744)

AB = BA =

1 0

0 1

= I

Solution to Exercise 7.8.13 (p. 744)

AB = BA =

1 0

0 1

= I

Solution to Exercise 7.8.15 (p. 744)

AB = BA =

1 0 0

0 1 0

0 0 1

= I

Solution to Exercise 7.8.17 (p. 744)

129

9 2

−1 3

Solution to Exercise 7.8.19 (p. 745)

169

−2 7

9 3

Solution to Exercise 7.8.21 (p. 745)There is no inverseSolution to Exercise 7.8.23 (p. 745)

47

0.5 1.5

1 −0.5

Solution to Exercise 7.8.25 (p. 745)

117

−5 5 −3

20 −3 12

1 −1 4

Solution to Exercise 7.8.27 (p. 745)

1209

47 −57 69

10 19 −12

−24 38 −13

Solution to Exercise 7.8.29 (p. 745)

18 60 −168

−56 −140 448

40 80 −280

Solution to Exercise 7.8.31 (p. 746)(−5, 6)Solution to Exercise 7.8.33 (p. 746)(2, 0)Solution to Exercise 7.8.35 (p. 746)(

13 ,−

52

)Solution to Exercise 7.8.37 (p. 746)(− 2

3 ,−116

)Available for free at Connexions <http://legacy.cnx.org/content/col11759/1.2>

800 CHAPTER 7. SYSTEMS OF EQUATIONS AND INEQUALITIES

Solution to Exercise 7.8.39 (p. 746)(7, 1

2 ,15

)Solution to Exercise 7.8.41 (p. 747)(5, 0,−1)Solution to Exercise 7.8.43 (p. 747)

134 (−35,−97,−154)Solution to Exercise 7.8.45 (p. 747)

1690 (65,−1136,−229)Solution to Exercise 7.8.47 (p. 747)(− 37

30 ,815

)Solution to Exercise 7.8.49 (p. 747)(

10123 ,−1, 2

5

)Solution to Exercise 7.8.51 (p. 748)

12

2 1 −1 −1

0 1 1 −1

0 −1 1 1

0 1 −1 1

Solution to Exercise 7.8.53 (p. 748)

139

3 2 1 −7

18 −53 32 10

24 −36 21 9

−9 46 −16 −5

Solution to Exercise 7.8.55 (p. 748)

1 0 0 0 0 0

0 1 0 0 0 0

0 0 1 0 0 0

0 0 0 1 0 0

0 0 0 0 1 0

−1 −1 −1 −1 −1 1

Solution to Exercise 7.8.57 (p. 749)In�nite solutions.Solution to Exercise 7.8.59 (p. 749)50% oranges, 25% bananas, 20% applesSolution to Exercise 7.8.61 (p. 749)10 straw hats, 50 beanies, 40 cowboy hatsSolution to Exercise 7.8.63 (p. 749)Tom ate 6, Joe ate 3, and Albert ate 3.Solution to Exercise 7.8.65 (p. 750)124 oranges, 10 lemons, 8 pomegranatesSolution to Exercise 7.9.1 (p. 753)(3,−7)Solution to Exercise 7.9.2 (p. 755)−10Solution to Exercise 7.9.3 (p. 757)(−2, 3

5 ,125

)

Available for free at Connexions <http://legacy.cnx.org/content/col11759/1.2>

801

Solution to Exercise 7.9.4 (p. 763)A determinant is the sum and products of the entries in the matrix, so you can always evaluate thatproduct�even if it does end up being 0.Solution to Exercise 7.9.6 (p. 763)The inverse does not exist.Solution to Exercise 7.9.8 (p. 763)−2Solution to Exercise 7.9.10 (p. 763)7Solution to Exercise 7.9.12 (p. 763)−4Solution to Exercise 7.9.14 (p. 763)0Solution to Exercise 7.9.16 (p. 764)−7, 990.7Solution to Exercise 7.9.18 (p. 764)3Solution to Exercise 7.9.20 (p. 764)−1Solution to Exercise 7.9.22 (p. 764)224Solution to Exercise 7.9.24 (p. 764)15Solution to Exercise 7.9.26 (p. 765)−17.03Solution to Exercise 7.9.28 (p. 765)(1, 1)Solution to Exercise 7.9.30 (p. 765)(

12 ,

13

)Solution to Exercise 7.9.32 (p. 765)(2, 5)Solution to Exercise 7.9.34 (p. 765)(−1,− 1

3

)Solution to Exercise 7.9.36 (p. 765)(15, 12)Solution to Exercise 7.9.38 (p. 766)(1, 3, 2)Solution to Exercise 7.9.40 (p. 766)(−1, 0, 3)Solution to Exercise 7.9.42 (p. 766)(

12 , 1, 2

)Solution to Exercise 7.9.44 (p. 766)(2, 1, 4)Solution to Exercise 7.9.46 (p. 766)In�nite solutionsSolution to Exercise 7.9.48 (p. 767)24Solution to Exercise 7.9.50 (p. 767)1Solution to Exercise 7.9.52 (p. 767)Yes; 18, 38

Available for free at Connexions <http://legacy.cnx.org/content/col11759/1.2>

802 CHAPTER 7. SYSTEMS OF EQUATIONS AND INEQUALITIES

Solution to Exercise 7.9.54 (p. 768)Yes; 33, 36, 37Solution to Exercise 7.9.56 (p. 768)$7,000 in �rst account, $3,000 in second account.Solution to Exercise 7.9.58 (p. 768)120 children, 1,080 adultSolution to Exercise 7.9.60 (p. 768)4 gal yellow, 6 gal blueSolution to Exercise 7.9.62 (p. 768)13 green tomatoes, 17 red tomatoesSolution to Exercise 7.9.64 (p. 769)Strawberries 18%, oranges 9%, kiwi 10%Solution to Exercise 7.9.66 (p. 769)100 for movie 1, 230 for movie 2, 312 for movie 3Solution to Exercise 7.9.68 (p. 769)20�29: 2,100, 30�39: 2,600, 40�49: 825Solution to Exercise 7.9.70 (p. 769)300 almonds, 400 cranberries, 300 cashewsSolution to Exercise 7.9.72 (p. 770)NoSolution to Exercise 7.9.74 (p. 770)(−2, 3)Solution to Exercise 7.9.76 (p. 770)(4,−1)Solution to Exercise 7.9.78 (p. 770)No solutions exist.Solution to Exercise 7.9.80 (p. 770)(300, 60, 000)Solution to Exercise 7.9.81 (p. 770)(400, 30, 000)Solution to Exercise 7.9.82 (p. 771)(10,−10, 10)Solution to Exercise 7.9.84 (p. 771)No solutions exist.Solution to Exercise 7.9.86 (p. 771)(−1,−2, 3)Solution to Exercise 7.9.88 (p. 771)(x, 8x

5 ,14x5

)Solution to Exercise 7.9.90 (p. 771)11, 17, 33Solution to Exercise 7.9.92 (p. 772)(2,−3) , (3, 2)Solution to Exercise 7.9.94 (p. 772)No solutionSolution to Exercise 7.9.96 (p. 772)No solution

Available for free at Connexions <http://legacy.cnx.org/content/col11759/1.2>

803

Solution to Exercise 7.9.98 (p. 772)

Solu-tion to Exercise 7.9.100 (p. 772)

Solu-

Available for free at Connexions <http://legacy.cnx.org/content/col11759/1.2>

804 CHAPTER 7. SYSTEMS OF EQUATIONS AND INEQUALITIES

tion to Exercise 7.9.102 (p. 773)2

x+2 ,−4x+1

Solution to Exercise 7.9.104 (p. 773)7

x+5 ,−15

(x+5)2

Solution to Exercise 7.9.106 (p. 773)3

x−5 ,−4x+1

x2+5x+25Solution to Exercise 7.9.108 (p. 773)x−4

(x2−2) ,5x+3

(x2−2)2

Solution to Exercise 7.9.110 (p. 773) −16 8

−4 −12

Solution to Exercise 7.9.112 (p. 773)unde�ned; dimensions do not matchSolution to Exercise 7.9.114 (p. 773)unde�ned; inner dimensions do not matchSolution to Exercise 7.9.116 (p. 774)

113 28 10

44 81 −41

84 98 −42

Solution to Exercise 7.9.118 (p. 774)−127 −74 176

−2 11 40

28 77 38

Solution to Exercise 7.9.120 (p. 774)unde�ned; inner dimensions do not matchSolution to Exercise 7.9.122 (p. 774)

x− 3z = 7

y + 2z = −5with in�nite solutions

Solution to Exercise 7.9.124 (p. 774)−2 2 1

2 −8 5

19 −10 22

|7

0

3

Solution to Exercise 7.9.126 (p. 774)

1 0 3

−1 4 0

0 1 2

|12

0

−7

Solution to Exercise 7.9.128 (p. 775)No solutions exist.Solution to Exercise 7.9.130 (p. 775)No solutions exist.Solution to Exercise 7.9.132 (p. 775)

18

2 7

6 1

Available for free at Connexions <http://legacy.cnx.org/content/col11759/1.2>

805

Solution to Exercise 7.9.134 (p. 775)No inverse exists.Solution to Exercise 7.9.136 (p. 775)(−20, 40)Solution to Exercise 7.9.138 (p. 776)(−1, 0.2, 0.3)Solution to Exercise 7.9.140 (p. 776)17% oranges, 34% bananas, 39% applesSolution to Exercise 7.9.142 (p. 776)0Solution to Exercise 7.9.144 (p. 776)6Solution to Exercise 7.9.146 (p. 776)(

6, 12

)Solution to Exercise 7.9.148 (p. 777)(x, 5x + 3)Solution to Exercise 7.9.150 (p. 777)(

0, 0,− 12

)Solution to Exercise 7.9.152 (p. 777)YesSolution to Exercise 7.9.154 (p. 777)No solutions exist.Solution to Exercise 7.9.156 (p. 778)

120 (10, 5, 4)Solution to Exercise 7.9.158 (p. 778)(x, 16x

5 −13x5

)Solution to Exercise 7.9.160 (p. 778)(−2√

2,−√

17),(−2√

2,√

17),(2√

2,−√

17),(2√

2,√

17)

Available for free at Connexions <http://legacy.cnx.org/content/col11759/1.2>

806 CHAPTER 7. SYSTEMS OF EQUATIONS AND INEQUALITIES

Solution to Exercise 7.9.162 (p. 778)

Solu-tion to Exercise 7.9.164 (p. 778)

53x+1 −

2x+3(3x+1)2

Solution to Exercise 7.9.166 (p. 778) 17 51

−8 11

Solution to Exercise 7.9.168 (p. 779) 12 −20

−15 30

Solution to Exercise 7.9.170 (p. 779)− 1

8Solution to Exercise 7.9.172 (p. 779)

14 −2 13

−2 3 −6

1 −5 12

|140

−1

11

Solution to Exercise 7.9.174 (p. 779)No solutions exist.Solution to Exercise 7.9.176 (p. 779)(100, 90)

Available for free at Connexions <http://legacy.cnx.org/content/col11759/1.2>

807

Solution to Exercise 7.9.178 (p. 780)(1

100 , 0)

Solution to Exercise 7.9.180 (p. 780)32 or more cell phones per day

Available for free at Connexions <http://legacy.cnx.org/content/col11759/1.2>

808 CHAPTER 7. SYSTEMS OF EQUATIONS AND INEQUALITIES

Available for free at Connexions <http://legacy.cnx.org/content/col11759/1.2>

Chapter 8

Analytic Geometry

8.1 Introduction to Analytic Geometry1

Figure 8.1: (a) Greek philosopher Aristotle (384�322 BCE) (b) German mathematician and astronomerJohannes Kepler (1571�1630)

1This content is available online at <http://legacy.cnx.org/content/m49437/1.2/>.

Available for free at Connexions <http://legacy.cnx.org/content/col11759/1.2>

809

810 CHAPTER 8. ANALYTIC GEOMETRY

The Greek mathematician Menaechmus (c. 380�c. 320 BCE) is generally credited with discovering theshapes formed by the intersection of a plane and a right circular cone. Depending on how he tilted theplane when it intersected the cone, he formed di�erent shapes at the intersection�beautiful shapes withnear-perfect symmetry.

It was also said that Aristotle may have had an intuitive understanding of these shapes, as he observedthe orbit of the planet to be circular. He presumed that the planets moved in circular orbits around Earth,and for nearly 2000 years this was the commonly held belief.

It was not until the Renaissance movement that Johannes Kepler noticed that the orbits of the planetwere not circular in nature. His published law of planetary motion in the 1600s changed our view of thesolar system forever. He claimed that the sun was at one end of the orbits, and the planets revolved aroundthe sun in an oval-shaped path.

In this chapter, we will investigate the two-dimensional �gures that are formed when a right circular coneis intersected by a plane. We will begin by studying each of three �gures created in this manner. We willdevelop de�ning equations for each �gure and then learn how to use these equations to solve a variety ofproblems.

8.2 The Ellipse2

Figure 8.2: The National Statuary Hall in Washington, D.C. (credit: Greg Palmer, Flickr)

2This content is available online at <http://legacy.cnx.org/content/m49438/1.3/>.

Available for free at Connexions <http://legacy.cnx.org/content/col11759/1.2>

811

Can you imagine standing at one end of a large room and still being able to hear a whisper from a personstanding at the other end? The National Statuary Hall in Washington, D.C., shown in Figure 8.2, is sucha room.3 It is an oval-shaped room called a whispering chamber because the shape makes it possible forsound to travel along the walls. In this section, we will investigate the shape of this room and its real-worldapplications, including how far apart two people in Statuary Hall can stand and still hear each other whisper.

8.2.1 Writing Equations of Ellipses in Standard Form

A conic section, or conic, is a shape resulting from intersecting a right circular cone with a plane. The angleat which the plane intersects the cone determines the shape, as shown in Figure 8.3.

Figure 8.3

Conic sections can also be described by a set of points in the coordinate plane. Later in this chapter,we will see that the graph of any quadratic equation in two variables is a conic section. The signs of theequations and the coe�cients of the variable terms determine the shape. This section focuses on the fourvariations of the standard form of the equation for the ellipse. An ellipse is the set of all points (x, y) in aplane such that the sum of their distances from two �xed points is a constant. Each �xed point is called afocus (plural: foci).

We can draw an ellipse using a piece of cardboard, two thumbtacks, a pencil, and string. Place thethumbtacks in the cardboard to form the foci of the ellipse. Cut a piece of string longer than the distancebetween the two thumbtacks (the length of the string represents the constant in the de�nition). Tack eachend of the string to the cardboard, and trace a curve with a pencil held taut against the string. The resultis an ellipse. See Figure 8.4.

3Architect of the Capitol. http://www.aoc.gov. Accessed April 15, 2014.

Available for free at Connexions <http://legacy.cnx.org/content/col11759/1.2>

812 CHAPTER 8. ANALYTIC GEOMETRY

Figure 8.4

Every ellipse has two axes of symmetry. The longer axis is called the major axis, and the shorter axis iscalled the minor axis. Each endpoint of the major axis is the vertex of the ellipse (plural: vertices), andeach endpoint of the minor axis is a co-vertex of the ellipse. The center of an ellipse is the midpoint of

Available for free at Connexions <http://legacy.cnx.org/content/col11759/1.2>

813

both the major and minor axes. The axes are perpendicular at the center. The foci always lie on the majoraxis, and the sum of the distances from the foci to any point on the ellipse (the constant sum) is greaterthan the distance between the foci. See Figure 8.5.

Figure 8.5

In this section, we restrict ellipses to those that are positioned vertically or horizontally in the coordinateplane. That is, the axes will either lie on or be parallel to the x- and y-axes. Later in the chapter, we willsee ellipses that are rotated in the coordinate plane.

To work with horizontal and vertical ellipses in the coordinate plane, we consider two cases: those thatare centered at the origin and those that are centered at a point other than the origin. First we will learnto derive the equations of ellipses, and then we will learn how to write the equations of ellipses in standardform. Later we will use what we learn to draw the graphs.

8.2.1.1 Deriving the Equation of an Ellipse Centered at the Origin

To derive the equation of an ellipse centered at the origin, we begin with the foci (−c, 0) and (c, 0) .Theellipse is the set of all points (x, y) such that the sum of the distances from (x, y) to the foci is constant, asshown in Figure 8.6.

Available for free at Connexions <http://legacy.cnx.org/content/col11759/1.2>

814 CHAPTER 8. ANALYTIC GEOMETRY

Figure 8.6

If (a, 0) is a vertex of the ellipse, the distance from (−c, 0) to (a, 0) is a − (−c) = a + c.The distancefrom (c, 0) to (a, 0) is a− c. The sum of the distances from the foci to the vertex is

(a+ c) + (a− c) = 2a (8.1)

If (x, y) is a point on the ellipse, then we can de�ne the following variables:

d1 = the distance from (−c, 0) to (x, y)

d2 = the distance from (c, 0) to (x, y)(8.2)

By the de�nition of an ellipse, d1 + d2 is constant for any point (x, y) on the ellipse. We know that the sumof these distances is 2a for the vertex (a, 0) . It follows that d1 + d2 = 2a for any point on the ellipse. We willbegin the derivation by applying the distance formula. The rest of the derivation is algebraic.

Available for free at Connexions <http://legacy.cnx.org/content/col11759/1.2>

815

d1 + d2 =√

(x− (−c))2 + (y − 0)2 +√

(x− c)2 + (y − 0)2 = 2a Distance formula√(x + c)2 + y2 +

√(x− c)2 + y2 = 2a Simplify expressions.√

(x + c)2 + y2 = 2a−√

(x− c)2 + y2 Move radical to opposite side.

(x + c)2 + y2 =

[2a−

√(x− c)2 + y2

]2

Square both sides.

x2 + 2cx + c2 + y2 = 4a2 − 4a√

(x− c)2 + y2 + (x− c)2 + y2 Expand the squares.

x2 + 2cx + c2 + y2 = 4a2 − 4a√

(x− c)2 + y2 + x2 − 2cx + c2 + y2 Expand remaining squares.

2cx = 4a2 − 4a√

(x− c)2 + y2 − 2cx Combine like terms.

4cx− 4a2 = −4a√

(x− c)2 + y2 Isolate the radical.

cx− a2 = −a√

(x− c)2 + y2 Divide by 4.

[cx− a2]2

= a2

[√(x− c)2 + y2

]2

Square both sides.

c2x2 − 2a2cx + a4 = a2 (x2 − 2cx + c2 + y2) Expand the squares.

c2x2 − 2a2cx + a4 = a2x2 − 2a2cx + a2c2 + a2y2 Distribute a2.

a2x2 − c2x2 + a2y2 = a4 − a2c2 Rewrite.

x2 (a2 − c2) + a2y2 = a2 (a2 − c2) Factor common terms.

x2b2 + a2y2 = a2b2 Set b2 = a2 − c2.x2b2

a2b2+ a2y2

a2b2= a2b2

a2b2Divide both sides by a2b2.

x2

a2 + y2

b2= 1 Simplify.

(8.3)

Thus, the standard equation of an ellipse is x2

a2 + y2

b2 = 1.This equation de�nes an ellipse centered at theorigin. If a > b,the ellipse is stretched further in the horizontal direction, and if b > a, the ellipse is stretchedfurther in the vertical direction.

8.2.1.2 Writing Equations of Ellipses Centered at the Origin in Standard Form

Standard forms of equations tell us about key features of graphs. Take a moment to recall some of thestandard forms of equations we've worked with in the past: linear, quadratic, cubic, exponential, logarithmic,and so on. By learning to interpret standard forms of equations, we are bridging the relationship betweenalgebraic and geometric representations of mathematical phenomena.

The key features of the ellipse are its center, vertices, co-vertices, foci, and lengths and positions ofthe major and minor axes. Just as with other equations, we can identify all of these features just bylooking at the standard form of the equation. There are four variations of the standard form of the ellipse.These variations are categorized �rst by the location of the center (the origin or not the origin), and thenby the position (horizontal or vertical). Each is presented along with a description of how the parts of theequation relate to the graph. Interpreting these parts allows us to form a mental picture of the ellipse.

A General Note: The standard form of the equation of an ellipse with center (0, 0) and major

Available for free at Connexions <http://legacy.cnx.org/content/col11759/1.2>

816 CHAPTER 8. ANALYTIC GEOMETRY

axis on the x-axis is

x2

a2+y2

b2= 1 (8.4)

where

• a > b• the length of the major axis is 2a• the coordinates of the vertices are (±a, 0)• the length of the minor axis is 2b• the coordinates of the co-vertices are (0,±b)• the coordinates of the foci are (±c, 0), where c2 = a2 − b2. See Figure 8.7a

The standard form of the equation of an ellipse with center (0, 0) and major axis on the y-axis is

x2

b2+y2

a2= 1 (8.5)

where

• a > b• the length of the major axis is 2a• the coordinates of the vertices are (0,±a)• the length of the minor axis is 2b• the coordinates of the co-vertices are (±b, 0)• the coordinates of the foci are (0,±c), where c2 = a2 − b2. See Figure 8.7b

Note that the vertices, co-vertices, and foci are related by the equation c2 = a2 − b2.When we aregiven the coordinates of the foci and vertices of an ellipse, we can use this relationship to �nd theequation of the ellipse in standard form.

Available for free at Connexions <http://legacy.cnx.org/content/col11759/1.2>

817

Figure 8.7: (a) Horizontal ellipse with center (0, 0) (b) Vertical ellipse with center (0, 0)

How To: Given the vertices and foci of an ellipse centered at the origin, write itsequation in standard form.

1.Determine whether the major axis lies on the x- or y-axis.

a.If the given coordinates of the vertices and foci have theform (±a, 0) and (±c, 0) respectively, then the major axis is the x-axis. Use the

standard form x2

a2 + y2

b2 = 1.b.If the given coordinates of the vertices and foci have theform (0,±a) and (±c, 0) ,respectively, then the major axis is the y-axis. Use the

standard form x2

b2 + y2

a2 = 1.2.Use the equation c2 = a2 − b2, along with the given coordinates of the vertices and foci, tosolve for b2.

3.Substitute the values for a2 and b2 into the standard form of the equation determined in Step1.

Example 8.1Writing the Equation of an Ellipse Centered at the Origin in Standard FormWhat is the standard form equation of the ellipse that has vertices (±8, 0) and foci (±5, 0)?

SolutionThe foci are on the x-axis, so the major axis is the x-axis. Thus, the equation will have the form

x2

a2+y2

b2= 1 (8.6)

The vertices are (±8, 0) ,so a = 8 and a2 = 64.The foci are (±5, 0) ,so c = 5 and c2 = 25.

Available for free at Connexions <http://legacy.cnx.org/content/col11759/1.2>

818 CHAPTER 8. ANALYTIC GEOMETRY

We know that the vertices and foci are related by the equation c2 = a2 − b2. Solving for b2, wehave:

c2 = a2 − b2

25 = 64− b2 Substitute for c2 and a2.

b2 = 39 Solve for b2.

(8.7)

Now we need only substitute a2 = 64 and b2 = 39 into the standard form of the equation. The

equation of the ellipse is x2

64 + y2

39 = 1.

Try It:

Exercise 8.2.1 (Solution on p. 942.)

What is the standard form equation of the ellipse that has vertices (0,±4) andfoci

(0,±√

15)?

Q&A: Can we write the equation of an ellipse centered at the origin given coordinatesof just one focus and vertex?

Yes. Ellipses are symmetrical, so the coordinates of the vertices of an ellipse centered aroundthe origin will always have the form (±a, 0) or (0, ± a) . Similarly, the coordinates of the foci willalways have the form (±c, 0) or (0, ± c) .Knowing this, we can use a and c from the given points,along with the equation c2 = a2 − b2,to �nd b2.

8.2.1.3 Writing Equations of Ellipses Not Centered at the Origin

Like the graphs of other equations, the graph of an ellipse can be translated. If an ellipse is translatedh unitshorizontally and k units vertically, the center of the ellipse will be (h, k) .This translation results in thestandard form of the equation we saw previously, withx replaced by (x− h) and y replaced by (y − k) .

A General Note: The standard form of the equation of an ellipse with center (h, k) andmajoraxis parallel to the x-axis is

(x− h)2

a2+

(y − k)2

b2= 1 (8.8)

where

• a > b• the length of the major axis is 2a• the coordinates of the vertices are (h± a, k)• the length of the minor axis is 2b• the coordinates of the co-vertices are (h, k ± b)• the coordinates of the foci are (h± c, k) ,where c2 = a2 − b2. See Figure 8.8a

Available for free at Connexions <http://legacy.cnx.org/content/col11759/1.2>

819

The standard form of the equation of an ellipse with center (h, k) and major axis parallel to they-axis is

(x− h)2

b2+

(y − k)2

a2= 1 (8.9)

where

• a > b• the length of the major axis is 2a• the coordinates of the vertices are (h, k ± a)• the length of the minor axis is 2b• the coordinates of the co-vertices are (h± b, k)• the coordinates of the foci are (h, k ± c) , where c2 = a2 − b2. See Figure 8.8b

Just as with ellipses centered at the origin, ellipses that are centered at a point (h, k) have vertices,co-vertices, and foci that are related by the equation c2 = a2−b2.We can use this relationship alongwith the midpoint and distance formulas to �nd the equation of the ellipse in standard form whenthe vertices and foci are given.

Figure 8.8: (a) Horizontal ellipse with center (h, k) (b) Vertical ellipse with center (h, k)

How To: Given the vertices and foci of an ellipse not centered at the origin, write itsequation in standard form.

1.Determine whether the major axis is parallel to the x- or y-axis.

a.If the y-coordinates of the given vertices and foci are the same, then the major axis is

parallel to the x-axis. Use the standard form (x−h)2

a2 + (y−k)2

b2 = 1.

Available for free at Connexions <http://legacy.cnx.org/content/col11759/1.2>

820 CHAPTER 8. ANALYTIC GEOMETRY

b.If the x-coordinates of the given vertices and foci are the same, then the major axis is

parallel to the y-axis. Use the standard form (x−h)2

b2 + (y−k)2

a2 = 1.2.Identify the center of the ellipse (h, k) using the midpoint formula and the given coordinatesfor the vertices.

3.Find a2 by solving for the length of the major axis, 2a, which is the distance between the givenvertices.

4.Find c2 usingh and k, found in Step 2, along with the given coordinates for the foci.5.Solve for b2 using the equation c2 = a2 − b2.6.Substitute the values forh, k, a2, and b2 into the standard form of the equation determined inStep 1.

Example 8.2Writing the Equation of an Ellipse Centered at a Point Other Than the OriginWhat is the standard form equation of the ellipse that has vertices (−2,−8) and (−2, 2)

and foci (−2,−7) and (−2, 1)?

SolutionThe x-coordinates of the vertices and foci are the same, so the major axis is parallel to the y-axis.Thus, the equation of the ellipse will have the form

(x− h)2

b2+

(y − k)2

a2= 1 (8.10)

First, we identify the center, (h, k) .The center is halfway between thevertices, (−2,−8) and (−2, 2) .Applying the midpoint formula, we have:

(h, k) =(−2+(−2)

2 , −8+22

)= (−2,−3)

(8.11)

Next, we �nd a2.The length of the major axis, 2a, is bounded by the vertices. We solve for a by�nding the distance between the y-coordinates of the vertices.

2a = 2− (−8)

2a = 10

a = 5

(8.12)

So a2 = 25.Now we �nd c2.The foci are given by (h, k ± c) . So, (h, k − c) = (−2,−7) and (h, k + c) =

(−2, 1) .We substitute k = −3 using either of these points to solve for c.

k + c = 1

−3 + c = 1

c = 4

(8.13)

So c2 = 16.Next, we solve for b2 using the equation c2 = a2 − b2.

c2 = a2 − b2

16 = 25− b2

b2 = 9

(8.14)

Available for free at Connexions <http://legacy.cnx.org/content/col11759/1.2>

821

Finally, we substitute the values found forh, k, a2, and b2 into the standard form equation for anellipse:

(x+ 2)2

9+

(y + 3)2

25= 1 (8.15)

Try It:

Exercise 8.2.2 (Solution on p. 942.)

What is the standard form equation of the ellipse that has vertices (−3, 3) and (5, 3) andfoci

(1− 2

√3, 3)and

(1 + 2

√3, 3)?

8.2.2 Graphing Ellipses Centered at the Origin

Just as we can write the equation for an ellipse given its graph, we can graph an ellipse given its equation.

To graph ellipses centered at the origin, we use the standard form x2

a2 + y2

b2 = 1, a > b for horizontal ellipses

and x2

b2 + y2

a2 = 1, a > b for vertical ellipses.

How To: Given the standard form of an equation for an ellipse centered at (0, 0) ,sketch the graph.

1.Use the standard forms of the equations of an ellipse to determine the major axis, vertices,co-vertices, and foci.

a.If the equation is in the form x2

a2 + y2

b2 = 1, where a > b, then

• the major axis is the x-axis• the coordinates of the vertices are (±a, 0)• the coordinates of the co-vertices are (0,±b)• the coordinates of the foci are (±c, 0)

b.If the equation is in the form x2

b2 + y2

a2 = 1,where a > b, then

• the major axis is the y-axis• the coordinates of the vertices are (0,±a)• the coordinates of the co-vertices are (±b, 0)• the coordinates of the foci are (0,±c)

2.Solve for c using the equation c2 = a2 − b2.3.Plot the center, vertices, co-vertices, and foci in the coordinate plane, and draw a smoothcurve to form the ellipse.

Example 8.3Graphing an Ellipse Centered at the Origin

Graph the ellipse given by the equation, x2

9 + y2

25 = 1. Identify and label the center, vertices, co-vertices, and foci.

SolutionFirst, we determine the position of the major axis. Because 25 > 9,the major axis is on the y-axis.

Therefore, the equation is in the form x2

b2 + y2

a2 = 1,where b2 = 9 and a2 = 25. It follows that:

Available for free at Connexions <http://legacy.cnx.org/content/col11759/1.2>

822 CHAPTER 8. ANALYTIC GEOMETRY

• the center of the ellipse is (0, 0)• the coordinates of the vertices are (0,±a) =

(0,±√

25)

= (0,±5)• the coordinates of the co-vertices are (±b, 0) =

(±√

9, 0)

= (±3, 0)• the coordinates of the foci are (0,±c) , where c2 = a2 − b2 Solving for c, we have:

c = ±√a2 − b2

= ±√

25− 9

= ±√

16

= ±4

(8.16)

Therefore, the coordinates of the foci are (0,±4) .Next, we plot and label the center, vertices, co-vertices, and foci, and draw a smooth curve to

form the ellipse. See Figure 8.9.

Figure 8.9

Available for free at Connexions <http://legacy.cnx.org/content/col11759/1.2>

823

Try It:

Exercise 8.2.3 (Solution on p. 942.)

Graph the ellipse given by the equation x2

36 + y2

4 = 1. Identify and label the center, vertices,co-vertices, and foci.

Example 8.4Graphing an Ellipse Centered at the Origin from an Equation Not in Standard FormGraph the ellipse given by the equation 4x2 + 25y2 = 100.Rewrite the equation in standard form.Then identify and label the center, vertices, co-vertices, and foci.

SolutionFirst, use algebra to rewrite the equation in standard form.

4x2 + 25y2 = 1004x2

100 + 25y2

100 = 100100

x2

25 + y2

4 = 1

(8.17)

Next, we determine the position of the major axis. Because 25 > 4, the major axis is on the x-axis.Therefore, the equation is in the form x2

a2 + y2

b2 = 1, where a2 = 25 and b2 = 4. It follows that:

• the center of the ellipse is (0, 0)• the coordinates of the vertices are (±a, 0) =

(±√

25, 0)

= (±5, 0)• the coordinates of the co-vertices are (0,±b) =

(0,±√

4)

= (0,±2)• the coordinates of the foci are (±c, 0) , where c2 = a2 − b2. Solving for c, we have:

c = ±√a2 − b2

= ±√

25− 4

= ±√

21

(8.18)

Therefore the coordinates of the foci are(±√

21, 0).

Next, we plot and label the center, vertices, co-vertices, and foci, and draw a smooth curve toform the ellipse.

Available for free at Connexions <http://legacy.cnx.org/content/col11759/1.2>

824 CHAPTER 8. ANALYTIC GEOMETRY

Figure 8.10

Try It:

Exercise 8.2.4 (Solution on p. 942.)

Graph the ellipse given by the equation 49x2 + 16y2 = 784.Rewrite the equation in stan-dard form. Then identify and label the center, vertices, co-vertices, and foci.

8.2.3 Graphing Ellipses Not Centered at the Origin

When an ellipse is not centered at the origin, we can still use the standard forms to �nd the key features of

the graph. When the ellipse is centered at some point, (h, k) ,we use the standard forms (x−h)2

a2 + (y−k)2

b2 =

1, a > b for horizontal ellipses and (x−h)2

b2 + (y−k)2

a2 = 1, a > b for vertical ellipses. From these standardequations, we can easily determine the center, vertices, co-vertices, foci, and positions of the major andminor axes.

How To: Given the standard form of an equation for an ellipse centered at (h, k) ,sketch the graph.

1.Use the standard forms of the equations of an ellipse to determine the center, position of themajor axis, vertices, co-vertices, and foci.

a.If the equation is in the form (x−h)2

a2 + (y−k)2

b2 = 1, where a > b, then

• the center is (h, k)• the major axis is parallel to the x-axis

Available for free at Connexions <http://legacy.cnx.org/content/col11759/1.2>

825

• the coordinates of the vertices are (h± a, k)• the coordinates of the co-vertices are (h, k ± b)• the coordinates of the foci are (h± c, k)

b.If the equation is in the form (x−h)2

b2 + (y−k)2

a2 = 1, where a > b, then

• the center is (h, k)• the major axis is parallel to the y-axis• the coordinates of the vertices are (h, k ± a)• the coordinates of the co-vertices are (h± b, k)• the coordinates of the foci are (h, k ± c)

2.Solve for c using the equation c2 = a2 − b2.3.Plot the center, vertices, co-vertices, and foci in the coordinate plane, and draw a smoothcurve to form the ellipse.

Example 8.5Graphing an Ellipse Centered at (h, k)

Graph the ellipse given by the equation, (x+2)2

4 + (y−5)2

9 = 1. Identify and label the center, vertices,co-vertices, and foci.

SolutionFirst, we determine the position of the major axis. Because 9 > 4, the major axis is parallel tothe y-axis. Therefore, the equation is in the form (x−h)2

b2 + (y−k)2

a2 = 1, where b2 = 4 and a2 = 9. Itfollows that:

• the center of the ellipse is (h, k) = (−2, 5)• the coordinates of the vertices are (h, k ± a) =

(−2, 5±

√9)

= (−2, 5± 3) ,or (−2, 2) and (−2, 8)

• the coordinates of the co-vertices are (h± b, k) =(−2±

√4, 5)

= (−2± 2, 5) ,or (−4, 5) and (0, 5)

• the coordinates of the foci are (h, k ± c) , where c2 = a2 − b2. Solving for c,we have:

c = ±√a2 − b2

= ±√

9− 4

= ±√

5

(8.19)

Therefore, the coordinates of the foci are(−2, 5−

√5)and

(−2, 5+

√5).

Next, we plot and label the center, vertices, co-vertices, and foci, and draw a smooth curve toform the ellipse.

Available for free at Connexions <http://legacy.cnx.org/content/col11759/1.2>

826 CHAPTER 8. ANALYTIC GEOMETRY

Figure 8.11

Try It:

Exercise 8.2.5 (Solution on p. 943.)

Graph the ellipse given by the equation (x−4)2

36 + (y−2)2

20 = 1. Identify and label the center,vertices, co-vertices, and foci.

How To: Given the general form of an equation for an ellipse centered at (h, k),express the equation in standard form.

Available for free at Connexions <http://legacy.cnx.org/content/col11759/1.2>

827

1.Recognize that an ellipse described by an equation in the form ax2 + by2 + cx+ dy + e = 0 isin general form.

2.Rearrange the equation by grouping terms that contain the same variable. Move the constantterm to the opposite side of the equation.

3.Factor out the coe�cients of thex2 and y2 terms in preparation for completing the square.4.Complete the square for each variable to rewrite the equation in the form of the sum ofmultiples of two binomials squared set equal to a constant,m1(x− h)2 + m2(y − k)2 = m3,wherem1,m2, andm3 are constants.

5.Divide both sides of the equation by the constant term to express the equation in standardform.

Example 8.6Graphing an Ellipse Centered at (h, k) by First Writing It in Standard FormGraph the ellipse given by the equation 4x2 + 9y2 − 40x + 36y + 100 = 0. Identify and label thecenter, vertices, co-vertices, and foci.

SolutionWe must begin by rewriting the equation in standard form.

4x2 + 9y2 − 40x+ 36y + 100 = 0 (8.20)

Group terms that contain the same variable, and move the constant to the opposite side of theequation.

(4x2 − 40x

)+(9y2 + 36y

)= −100 (8.21)

Factor out the coe�cients of the squared terms.

4(x2 − 10x

)+ 9

(y2 + 4y

)= −100 (8.22)

Complete the square twice. Remember to balance the equation by adding the same constants toeach side.

4(x2 − 10x+ 25

)+ 9

(y2 + 4y + 4

)= −100 + 100 + 36 (8.23)

Rewrite as perfect squares.

4(x− 5)2 + 9(y + 2)2 = 36 (8.24)

Divide both sides by the constant term to place the equation in standard form.

(x− 5)2

9+

(y + 2)2

4= 1 (8.25)

Now that the equation is in standard form, we can determine the position of the major axis.Because 9 > 4, the major axis is parallel to the x-axis. Therefore, the equation is in the

form (x−h)2

a2 + (y−k)2

b2 = 1, where a2 = 9 and b2 = 4. It follows that:

• the center of the ellipse is (h, k) = (5,−2)• the coordinates of the vertices are (h± a, k) =

(5±√

9,−2)

=(5± 3,−2) , or (2,−2) and (8,−2)

• the coordinates of the co-vertices are (h, k ± b) =(5,−2±

√4)

=(5,−2± 2) , or (5,−4) and (5, 0)

• the coordinates of the foci are (h± c, k) , where c2 = a2 − b2. Solving for c, we have:

Available for free at Connexions <http://legacy.cnx.org/content/col11759/1.2>

828 CHAPTER 8. ANALYTIC GEOMETRY

c = ±√a2 − b2

= ±√

9− 4

= ±√

5

(8.26)

Therefore, the coordinates of the foci are(5−√

5,−2)and

(5+√

5,−2).

Next we plot and label the center, vertices, co-vertices, and foci, and draw a smooth curve toform the ellipse as shown in Figure 8.12.

Figure 8.12

Try It:

Exercise 8.2.6 (Solution on p. 944.)

Express the equation of the ellipse given in standard form. Identify the center, vertices,co-vertices, and foci of the ellipse.

Available for free at Connexions <http://legacy.cnx.org/content/col11759/1.2>

829

4x2 + y2 − 24x+ 2y + 21 = 0 (8.27)

8.2.4 Solving Applied Problems Involving Ellipses

Many real-world situations can be represented by ellipses, including orbits of planets, satellites, moons andcomets, and shapes of boat keels, rudders, and some airplane wings. A medical device called a lithotripter useselliptical re�ectors to break up kidney stones by generating sound waves. Some buildings, called whisperingchambers, are designed with elliptical domes so that a person whispering at one focus can easily be heard bysomeone standing at the other focus. This occurs because of the acoustic properties of an ellipse. When asound wave originates at one focus of a whispering chamber, the sound wave will be re�ected o� the ellipticaldome and back to the other focus. See Figure 8.13. In the whisper chamber at the Museum of Science andIndustry in Chicago, two people standing at the foci�about 43 feet apart�can hear each other whisper.

Figure 8.13: Sound waves are re�ected between foci in an elliptical room, called a whispering chamber.

Example 8.7Locating the Foci of a Whispering ChamberThe Statuary Hall in the Capitol Building in Washington, D.C. is a whispering chamber. Its

Available for free at Connexions <http://legacy.cnx.org/content/col11759/1.2>

830 CHAPTER 8. ANALYTIC GEOMETRY

dimensions are 46 feet wide by 96 feet long as shown in Figure 8.14.

a. What is the standard form of the equation of the ellipse representing the outline of the room?Hint: assume a horizontal ellipse, and let the center of the room be the point (0, 0) .

b. If two senators standing at the foci of this room can hear each other whisper, how far apartare the senators? Round to the nearest foot.

Figure 8.14

Solution

a. We are assuming a horizontal ellipse with center (0, 0) , so we need to �nd an equation of the

form x2

a2 + y2

b2 = 1, where a > b.We know that the length of the major axis, 2a, is longer thanthe length of the minor axis, 2b. So the length of the room, 96, is represented by the majoraxis, and the width of the room, 46, is represented by the minor axis.

• Solving for a, we have 2a = 96, so a = 48, and a2 = 2304.• Solving for b, we have 2b = 46, so b = 23, and b2 = 529.

Therefore, the equation of the ellipse is x2

2304 + y2

529 = 1.b. To �nd the distance between the senators, we must �nd the distance between the

foci, (±c, 0) , where c2 = a2 − b2. Solving for c,we have:

Available for free at Connexions <http://legacy.cnx.org/content/col11759/1.2>

831

c2 = a2 − b2

c2 = 2304− 529 Substitute using the values found in part (a).

c = ±√

2304− 529 Take the square root of both sides.

c = ±√

1775 Subtract.

c ≈ ±42 Round to the nearest foot.

(8.28)

The points (±42, 0) represent the foci. Thus, the distance between the senators is 2 (42) =84 feet.

Try It:

Exercise 8.2.7 (Solution on p. 944.)

Suppose a whispering chamber is 480 feet long and 320 feet wide.

a.What is the standard form of the equation of the ellipse representing the room? Hint:assume a horizontal ellipse, and let the center of the room be the point (0, 0) .

b.If two people are standing at the foci of this room and can hear each other whisper,how far apart are the people? Round to the nearest foot.

Media: Access these online resources for additional instruction and practice with ellipses.

• Conic Sections: The Ellipse4• Graph an Ellipse with Center at the Origin5

• Graph an Ellipse with Center Not at the Origin6

8.2.5 Key Equations

Horizontal ellipse, center at origin x2

a2 + y2

b2 = 1, a > b

Vertical ellipse, center at origin x2

b2 + y2

a2 = 1, a > b

Horizontal ellipse, center (h, k) (x−h)2

a2 + (y−k)2

b2 = 1, a > b

Vertical ellipse, center (h, k) (x−h)2

b2 + (y−k)2

a2 = 1, a > b

Table 8.1

4http://openstaxcollege.org/l/conicellipse5http://openstaxcollege.org/l/grphellorigin6http://openstaxcollege.org/l/grphellnot

Available for free at Connexions <http://legacy.cnx.org/content/col11759/1.2>

832 CHAPTER 8. ANALYTIC GEOMETRY

8.2.6 Key Concepts

• An ellipse is the set of all points (x, y) in a plane such that the sum of their distances from two �xedpoints is a constant. Each �xed point is called a focus (plural: foci).

• When given the coordinates of the foci and vertices of an ellipse, we can write the equation of theellipse in standard form. See Example 8.1 and Example 8.2.

• When given an equation for an ellipse centered at the origin in standard form, we can identify itsvertices, co-vertices, foci, and the lengths and positions of the major and minor axes in order to graphthe ellipse. See Example 8.3 and Example 8.4.

• When given the equation for an ellipse centered at some point other than the origin, we can identifyits key features and graph the ellipse. See Example 8.5 and Example 8.6.

• Real-world situations can be modeled using the standard equations of ellipses and then evaluated to�nd key features, such as lengths of axes and distance between foci. See Example 8.7.

8.2.7 Section Exercises

8.2.7.1 Verbal

Exercise 8.2.8 (Solution on p. 944.)

De�ne an ellipse in terms of its foci.

Exercise 8.2.9Where must the foci of an ellipse lie?

Exercise 8.2.10 (Solution on p. 944.)

What special case of the ellipse do we have when the major and minor axis are of the same length?

Exercise 8.2.11For the special case mentioned above, what would be true about the foci of that ellipse?

Exercise 8.2.12 (Solution on p. 944.)

What can be said about the symmetry of the graph of an ellipse with center at the origin and focialong the y-axis?

8.2.7.2 Algebraic

For the following exercises, determine whether the given equations represent ellipses. If yes, write in standardform.

Exercise 8.2.132x2 + y = 4Exercise 8.2.14 (Solution on p. 944.)

4x2 + 9y2 = 36Exercise 8.2.154x2 − y2 = 4Exercise 8.2.16 (Solution on p. 944.)

4x2 + 9y2 = 1Exercise 8.2.174x2 − 8x+ 9y2 − 72y + 112 = 0

For the following exercises, write the equation of an ellipse in standard form, and identify the end points ofthe major and minor axes as well as the foci.

Exercise 8.2.18 (Solution on p. 944.)x2

4 + y2

49 = 1

Available for free at Connexions <http://legacy.cnx.org/content/col11759/1.2>

833

Exercise 8.2.19x2

100 + y2

64 = 1Exercise 8.2.20 (Solution on p. 945.)

x2 + 9y2 = 1Exercise 8.2.214x2 + 16y2 = 1Exercise 8.2.22 (Solution on p. 945.)(x−2)2

49 + (y−4)2

25 = 1Exercise 8.2.23(x−2)2

81 + (y+1)2

16 = 1Exercise 8.2.24 (Solution on p. 945.)(x+5)2

4 + (y−7)2

9 = 1Exercise 8.2.25(x−7)2

49 + (y−7)2

49 = 1Exercise 8.2.26 (Solution on p. 945.)

4x2 − 8x+ 9y2 − 72y + 112 = 0Exercise 8.2.279x2 − 54x+ 9y2 − 54y + 81 = 0Exercise 8.2.28 (Solution on p. 945.)

4x2 − 24x+ 36y2 − 360y + 864 = 0Exercise 8.2.294x2 + 24x+ 16y2 − 128y + 228 = 0Exercise 8.2.30 (Solution on p. 945.)

4x2 + 40x+ 25y2 − 100y + 100 = 0Exercise 8.2.31x2 + 2x+ 100y2 − 1000y + 2401 = 0Exercise 8.2.32 (Solution on p. 945.)

4x2 + 24x+ 25y2 + 200y + 336 = 0Exercise 8.2.339x2 + 72x+ 16y2 + 16y + 4 = 0

For the following exercises, �nd the foci for the given ellipses.

Exercise 8.2.34 (Solution on p. 945.)(x+3)2

25 + (y+1)2

36 = 1Exercise 8.2.35(x+1)2

100 + (y−2)2

4 = 1Exercise 8.2.36 (Solution on p. 945.)

x2 + y2 = 1Exercise 8.2.37x2 + 4y2 + 4x+ 8y = 1Exercise 8.2.38 (Solution on p. 945.)

10x2 + y2 + 200x = 0

Available for free at Connexions <http://legacy.cnx.org/content/col11759/1.2>

834 CHAPTER 8. ANALYTIC GEOMETRY

8.2.7.3 Graphical

For the following exercises, graph the given ellipses, noting center, vertices, and foci.

Exercise 8.2.39x2

25 + y2

36 = 1Exercise 8.2.40 (Solution on p. 945.)x2

16 + y2

9 = 1Exercise 8.2.414x2 + 9y2 = 1Exercise 8.2.42 (Solution on p. 946.)

81x2 + 49y2 = 1Exercise 8.2.43(x−2)2

64 + (y−4)2

16 = 1Exercise 8.2.44 (Solution on p. 946.)(x+3)2

9 + (y−3)2

9 = 1Exercise 8.2.45x2

2 + (y+1)2

5 = 1Exercise 8.2.46 (Solution on p. 947.)

4x2 − 8x+ 16y2 − 32y − 44 = 0Exercise 8.2.47x2 − 8x+ 25y2 − 100y + 91 = 0Exercise 8.2.48 (Solution on p. 947.)

x2 + 8x+ 4y2 − 40y + 112 = 0Exercise 8.2.4964x2 + 128x+ 9y2 − 72y − 368 = 0Exercise 8.2.50 (Solution on p. 948.)

16x2 + 64x+ 4y2 − 8y + 4 = 0Exercise 8.2.51100x2 + 1000x+ y2 − 10y + 2425 = 0Exercise 8.2.52 (Solution on p. 948.)

4x2 + 16x+ 4y2 + 16y + 16 = 0For the following exercises, use the given information about the graph of each ellipse to determine its equation.

Exercise 8.2.53Center at the origin, symmetric with respect to the x- and y-axes, focus at (4, 0) , and point ongraph (0, 3) .Exercise 8.2.54 (Solution on p. 949.)

Center at the origin, symmetric with respect to the x- and y-axes, focus at (0,−2) , and point ongraph (5, 0) .Exercise 8.2.55Center at the origin, symmetric with respect to the x- and y-axes, focus at (3, 0) , and major axisis twice as long as minor axis.

Exercise 8.2.56 (Solution on p. 949.)

Center (4, 2); vertex (9, 2); one focus:(4 + 2

√6, 2).

Exercise 8.2.57Center (3, 5); vertex (3, 11); one focus:

(3, 5+4

√2)

Available for free at Connexions <http://legacy.cnx.org/content/col11759/1.2>

835

Exercise 8.2.58 (Solution on p. 949.)

Center (−3, 4); vertex (1, 4); one focus:(−3 + 2

√3, 4)

For the following exercises, given the graph of the ellipse, determine its equation.Exercise 8.2.59

Available for free at Connexions <http://legacy.cnx.org/content/col11759/1.2>

836 CHAPTER 8. ANALYTIC GEOMETRY

Exercise 8.2.60 (Solution on p. 949.)

Exercise 8.2.61

Available for free at Connexions <http://legacy.cnx.org/content/col11759/1.2>

837

Exercise 8.2.62 (Solution on p. 949.)

Exercise 8.2.63

Available for free at Connexions <http://legacy.cnx.org/content/col11759/1.2>

838 CHAPTER 8. ANALYTIC GEOMETRY

8.2.7.4 Extensions

For the following exercises, �nd the area of the ellipse. The area of an ellipse is given by the formulaArea =a · b · π.

Exercise 8.2.64 (Solution on p. 949.)(x−3)2

9 + (y−3)2

16 = 1Exercise 8.2.65(x+6)2

16 + (y−6)2

36 = 1Exercise 8.2.66 (Solution on p. 949.)(x+1)2

4 + (y−2)2

5 = 1Exercise 8.2.674x2 − 8x+ 9y2 − 72y + 112 = 0Exercise 8.2.68 (Solution on p. 949.)

9x2 − 54x+ 9y2 − 54y + 81 = 0

8.2.7.5 Real-World Applications

Exercise 8.2.69Find the equation of the ellipse that will just �t inside a box that is 8 units wide and 4 units high.

Exercise 8.2.70 (Solution on p. 949.)

Find the equation of the ellipse that will just �t inside a box that is four times as wide as it is high.Express in terms ofh, the height.

Exercise 8.2.71An arch has the shape of a semi-ellipse (the top half of an ellipse). The arch has a height of 8feet and a span of 20 feet. Find an equation for the ellipse, and use that to �nd the height to thenearest 0.01 foot of the arch at a distance of 4 feet from the center.

Exercise 8.2.72 (Solution on p. 949.)

An arch has the shape of a semi-ellipse. The arch has a height of 12 feet and a span of 40 feet. Findan equation for the ellipse, and use that to �nd the distance from the center to a point at whichthe height is 6 feet. Round to the nearest hundredth.

Exercise 8.2.73A bridge is to be built in the shape of a semi-elliptical arch and is to have a span of 120 feet. Theheight of the arch at a distance of 40 feet from the center is to be 8 feet. Find the height of thearch at its center.

Exercise 8.2.74 (Solution on p. 949.)

A person in a whispering gallery standing at one focus of the ellipse can whisper and be heard by aperson standing at the other focus because all the sound waves that reach the ceiling are re�ectedto the other person. If a whispering gallery has a length of 120 feet, and the foci are located 30 feetfrom the center, �nd the height of the ceiling at the center.

Exercise 8.2.75A person is standing 8 feet from the nearest wall in a whispering gallery. If that person is at onefocus, and the other focus is 80 feet away, what is the length and height at the center of the gallery?

Available for free at Connexions <http://legacy.cnx.org/content/col11759/1.2>

839

8.3 The Hyperbola7

What do paths of comets, supersonic booms, ancient Grecian pillars, and natural draft cooling towers havein common? They can all be modeled by the same type of conic. For instance, when something movesfaster than the speed of sound, a shock wave in the form of a cone is created. A portion of a conic is formedwhen the wave intersects the ground, resulting in a sonic boom. See Figure 8.15.

Figure 8.15: A shock wave intersecting the ground forms a portion of a conic and results in a sonicboom.

Most people are familiar with the sonic boom created by supersonic aircraft, but humans were breakingthe sound barrier long before the �rst supersonic �ight. The crack of a whip occurs because the tip isexceeding the speed of sound. The bullets shot from many �rearms also break the sound barrier, althoughthe bang of the gun usually supersedes the sound of the sonic boom.

8.3.1 Locating the Vertices and Foci of a Hyperbola

In analytic geometry, a hyperbola is a conic section formed by intersecting a right circular cone with aplane at an angle such that both halves of the cone are intersected. This intersection produces two separateunbounded curves that are mirror images of each other. See Figure 8.16.

7This content is available online at <http://legacy.cnx.org/content/m49439/1.3/>.

Available for free at Connexions <http://legacy.cnx.org/content/col11759/1.2>

840 CHAPTER 8. ANALYTIC GEOMETRY

Figure 8.16: A hyperbola

Like the ellipse, the hyperbola can also be de�ned as a set of points in the coordinate plane. A hyperbolais the set of all points (x, y) in a plane such that the di�erence of the distances between (x, y) and the fociis a positive constant.

Notice that the de�nition of a hyperbola is very similar to that of an ellipse. The distinction is that thehyperbola is de�ned in terms of the di�erence of two distances, whereas the ellipse is de�ned in terms of thesum of two distances.

As with the ellipse, every hyperbola has two axes of symmetry. The transverse axis is a line segmentthat passes through the center of the hyperbola and has vertices as its endpoints. The foci lie on the line thatcontains the transverse axis. The conjugate axis is perpendicular to the transverse axis and has the co-vertices as its endpoints. The center of a hyperbola is the midpoint of both the transverse and conjugateaxes, where they intersect. Every hyperbola also has two asymptotes that pass through its center. As a

Available for free at Connexions <http://legacy.cnx.org/content/col11759/1.2>

841

hyperbola recedes from the center, its branches approach these asymptotes. The central rectangle of thehyperbola is centered at the origin with sides that pass through each vertex and co-vertex; it is a useful toolfor graphing the hyperbola and its asymptotes. To sketch the asymptotes of the hyperbola, simply sketchand extend the diagonals of the central rectangle. See Figure 8.17.

Figure 8.17: Key features of the hyperbola

In this section, we will limit our discussion to hyperbolas that are positioned vertically or horizontallyin the coordinate plane; the axes will either lie on or be parallel to the x- and y-axes. We will consider twocases: those that are centered at the origin, and those that are centered at a point other than the origin.

8.3.1.1 Deriving the Equation of an Ellipse Centered at the Origin

Let (−c, 0) and (c, 0) be the foci of a hyperbola centered at the origin. The hyperbola is the set of allpoints (x, y) such that the di�erence of the distances from (x, y) to the foci is constant. See Figure 8.18.

Available for free at Connexions <http://legacy.cnx.org/content/col11759/1.2>

842 CHAPTER 8. ANALYTIC GEOMETRY

Figure 8.18

If (a, 0) is a vertex of the hyperbola, the distance from (−c, 0) to (a, 0) is a− (−c) = a+ c.The distancefrom (c, 0) to (a, 0) is c− a.The sum of the distances from the foci to the vertex is

(a+ c)− (c− a) = 2a (8.29)

If (x, y) is a point on the hyperbola, we can de�ne the following variables:

d2 = the distance from (−c, 0) to (x, y)

d1 = the distance from (c, 0) to (x, y)(8.30)

By de�nition of a hyperbola, d2 − d1 is constant for any point (x, y) on the hyperbola. We know thatthe di�erence of these distances is 2a for the vertex (a, 0) . It follows that d2 − d1 = 2a for any point on thehyperbola. As with the derivation of the equation of an ellipse, we will begin by applying the distanceformula. The rest of the derivation is algebraic. Compare this derivation with the one from the previoussection for ellipses.

Available for free at Connexions <http://legacy.cnx.org/content/col11759/1.2>

843

d2 − d1 =√

(x− (−c))2 + (y − 0)2 −√

(x− c)2 + (y − 0)2 = 2a Distance Formula√(x + c)2 + y2 −

√(x− c)2 + y2 = 2a Simplify expressions.√

(x + c)2 + y2 = 2a +√

(x− c)2 + y2 Move radical to opposite side.

(x + c)2 + y2 =

(2a +

√(x− c)2 + y2

)2

Square both sides.

x2 + 2cx + c2 + y2 = 4a2 + 4a√

(x− c)2 + y2 + (x− c)2 + y2 Expand the squares.

x2 + 2cx + c2 + y2 = 4a2 + 4a√

(x− c)2 + y2 + x2 − 2cx + c2 + y2 Expand remaining square.

2cx = 4a2 + 4a√

(x− c)2 + y2 − 2cx Combine like terms.

4cx− 4a2 = 4a√

(x− c)2 + y2 Isolate the radical.

cx− a2 = a√

(x− c)2 + y2 Divide by 4.

(cx− a2)2

= a2

[√(x− c)2 + y2

]2

Square both sides.

c2x2 − 2a2cx + a4 = a2 (x2 − 2cx + c2 + y2) Expand the squares.

c2x2 − 2a2cx + a4 = a2x2 − 2a2cx + a2c2 + a2y2 Distribute a2.

a4 + c2x2 = a2x2 + a2c2 + a2y2 Combine like terms.

c2x2 − a2x2 − a2y2 = a2c2 − a4 Rearrange terms.

x2 (c2 − a2)− a2y2 = a2 (c2 − a2) Factor common terms.

x2b2 − a2y2 = a2b2 Set b2 = c2 − a2.x2b2

a2b2− a2y2

a2b2= a2b2

a2b2Divide both sides by a2b2

x2

a2 − y2

b2= 1

(8.31)

This equation de�nes a hyperbola centered at the origin with vertices (±a, 0) and co-vertices (0± b) .

A General Note: The standard form of the equation of a hyperbola with center (0, 0) andtransverse axis on the x-axis is

x2

a2− y2

b2= 1 (8.32)

where

• the length of the transverse axis is 2a• the coordinates of the vertices are (±a, 0)• the length of the conjugate axis is 2b• the coordinates of the co-vertices are (0,±b)• the distance between the foci is 2c, where c2 = a2 + b2

• the coordinates of the foci are (±c, 0)• the equations of the asymptotes are y = ± b

ax

Available for free at Connexions <http://legacy.cnx.org/content/col11759/1.2>

844 CHAPTER 8. ANALYTIC GEOMETRY

See Figure 8.19a.

The standard form of the equation of a hyperbola with center (0, 0) and transverse axis on they-axis is

y2

a2− x2

b2= 1 (8.33)

where

• the length of the transverse axis is 2a• the coordinates of the vertices are (0,±a)• the length of the conjugate axis is 2b• the coordinates of the co-vertices are (±b, 0)• the distance between the foci is 2c, where c2 = a2 + b2

• the coordinates of the foci are (0,±c)• the equations of the asymptotes are y = ±abx

See Figure 8.19b.

Note that the vertices, co-vertices, and foci are related by the equation c2 = a2 + b2.When we aregiven the equation of a hyperbola, we can use this relationship to identify its vertices and foci.

Figure 8.19: (a) Horizontal hyperbola with center (0, 0) (b) Vertical hyperbola with center (0, 0)

How To: Given the equation of a hyperbola in standard form, locate its vertices andfoci.

1.Determine whether the transverse axis lies on the x- or y-axis. Notice that a2 is always underthe variable with the positive coe�cient. So, if you set the other variable equal to zero, youcan easily �nd the intercepts. In the case where the hyperbola is centered at the origin, theintercepts coincide with the vertices.

Available for free at Connexions <http://legacy.cnx.org/content/col11759/1.2>

845

a.If the equation has the form x2

a2 − y2

b2 = 1, then the transverse axis lies on the x-axis. Thevertices are located at (±a, 0) , and the foci are located at (±c, 0) .

b.If the equation has the form y2

a2 − x2

b2 = 1, then the transverse axis lies on the y-axis. Thevertices are located at (0,±a) , and the foci are located at (0,±c) .

2.Solve for a using the equation a =√a2.

3.Solve for c using the equation c =√a2 + b2.

Example 8.8Locating a Hyperbola's Vertices and Foci

Identify the vertices and foci of the hyperbola with equation y2

49 −x2

32 = 1.

SolutionThe equation has the form y2

a2 − x2

b2 = 1, so the transverse axis lies on the y-axis. The hyperbola iscentered at the origin, so the vertices serve as the y-intercepts of the graph. To �nd the vertices,setx = 0, and solve for y.

1 = y2

49 −x2

32

1 = y2

49 −02

32

1 = y2

49

y2 = 49

y = ±√

49 = ±7

(8.34)

The foci are located at (0,±c) . Solving for c,

c =√a2 + b2 =

√49 + 32 =

√81 = 9 (8.35)

Therefore, the vertices are located at (0,±7) , and the foci are located at (0, 9) .

Try It:

Exercise 8.3.1 (Solution on p. 949.)

Identify the vertices and foci of the hyperbola with equation x2

9 −y2

25 = 1.

8.3.2 Writing Equations of Hyperbolas in Standard Form

Just as with ellipses, writing the equation for a hyperbola in standard form allows us to calculate the keyfeatures: its center, vertices, co-vertices, foci, asymptotes, and the lengths and positions of the transverseand conjugate axes. Conversely, an equation for a hyperbola can be found given its key features. We beginby �nding standard equations for hyperbolas centered at the origin. Then we will turn our attention to�nding standard equations for hyperbolas centered at some point other than the origin.

Available for free at Connexions <http://legacy.cnx.org/content/col11759/1.2>

846 CHAPTER 8. ANALYTIC GEOMETRY

8.3.2.1 Hyperbolas Centered at the Origin

Reviewing the standard forms given for hyperbolas centered at (0, 0) ,we see that the vertices, co-vertices, andfoci are related by the equation c2 = a2+b2.Note that this equation can also be rewritten as b2 = c2−a2.Thisrelationship is used to write the equation for a hyperbola when given the coordinates of its foci and vertices.

How To: Given the vertices and foci of a hyperbola centered at (0, 0) , write itsequation in standard form.

1.Determine whether the transverse axis lies on the x- or y-axis.

a.If the given coordinates of the vertices and foci have theform (±a, 0) and (±c, 0) , respectively, then the transverse axis is the x-axis. Use

the standard form x2

a2 − y2

b2 = 1.b.If the given coordinates of the vertices and foci have theform (0,±a) and (0,±c) , respectively, then the transverse axis is the y-axis. Use

the standard form y2

a2 − x2

b2 = 1.2.Find b2 using the equation b2 = c2 − a2.3.Substitute the values for a2 and b2 into the standard form of the equation determined in Step1.

Example 8.9Finding the Equation of a Hyperbola Centered at (0,0) Given its Foci and VerticesWhat is the standard form equation of the hyperbola that has vertices (±6, 0) and

foci(±2√

10, 0)?

SolutionThe vertices and foci are on the x-axis. Thus, the equation for the hyperbola will have the

form x2

a2 − y2

b2 = 1.The vertices are (±6, 0) , so a = 6 and a2 = 36.The foci are

(±2√

10, 0), so c = 2

√10 and c2 = 40.

Solving for b2, we have

b2 = c2 − a2

b2 = 40− 36 Substitute for c2 and a2.

b2 = 4 Subtract.

(8.36)

Finally, we substitute a2 = 36 and b2 = 4 into the standard form of the equation, x2

a2 − y2

b2 = 1. Theequation of the hyperbola is x

2

36 −y2

4 = 1, as shown in Figure 8.20.

Available for free at Connexions <http://legacy.cnx.org/content/col11759/1.2>

847

Figure 8.20

Try It:

Exercise 8.3.2 (Solution on p. 949.)

What is the standard form equation of the hyperbola that has vertices (0,±2) andfoci

(0,±2

√5)?

8.3.2.2 Hyperbolas Not Centered at the Origin

Like the graphs for other equations, the graph of a hyperbola can be translated. If a hyperbola istranslatedh units horizontally and k units vertically, the center of the hyperbola will be (h, k) .This transla-tion results in the standard form of the equation we saw previously, withx replaced by (x− h) and y replacedby (y − k) .

A General Note: The standard form of the equation of a hyperbola with center (h, k) andtransverse axis parallel to the x-axis is

(x− h)2

a2− (y − k)2

b2= 1 (8.37)

where

Available for free at Connexions <http://legacy.cnx.org/content/col11759/1.2>

848 CHAPTER 8. ANALYTIC GEOMETRY

• the length of the transverse axis is 2a• the coordinates of the vertices are (h± a, k)• the length of the conjugate axis is 2b• the coordinates of the co-vertices are (h, k ± b)• the distance between the foci is 2c, where c2 = a2 + b2

• the coordinates of the foci are (h± c, k)

The asymptotes of the hyperbola coincide with the diagonals of the central rectangle. The length ofthe rectangle is 2a and its width is 2b.The slopes of the diagonals are ± b

a ,and each diagonal passesthrough the center (h, k) .Using the point-slope formula, it is simple to show that the equationsof the asymptotes are y = ± b

a (x− h) + k. See Figure 8.21a

The standard form of the equation of a hyperbola with center (h, k) and transverse axis parallel tothe y-axis is

(y − k)2

a2− (x− h)2

b2= 1 (8.38)

where

�the length of the transverse axis is 2a�the coordinates of the vertices are (h, k ± a)�the length of the conjugate axis is 2b�the coordinates of the co-vertices are (h± b, k)�the distance between the foci is 2c, where c2 = a2 + b2

�the coordinates of the foci are (h, k ± c)

Using the reasoning above, the equations of the asymptotes are y = ±ab (x− h)+k. See Figure 8.21b.

Figure 8.21: (a) Horizontal hyperbola with center (h, k) (b) Vertical hyperbola with center (h, k)

Available for free at Connexions <http://legacy.cnx.org/content/col11759/1.2>

849

Like hyperbolas centered at the origin, hyperbolas centered at a point (h, k) have vertices, co-vertices, andfoci that are related by the equation c2 = a2 + b2.We can use this relationship along with the midpoint anddistance formulas to �nd the standard equation of a hyperbola when the vertices and foci are given.

How To: Given the vertices and foci of a hyperbola centered at (h, k) ,write its equationin standard form.

1.Determine whether the transverse axis is parallel to the x- or y-axis.

a.If the y-coordinates of the given vertices and foci are the same, then the transverse axis

is parallel to the x-axis. Use the standard form (x−h)2

a2 − (y−k)2

b2 = 1.b.If the x-coordinates of the given vertices and foci are the same, then the transverse axis

is parallel to the y-axis. Use the standard form (y−k)2

a2 − (x−h)2

b2 = 1.2.Identify the center of the hyperbola, (h, k) ,using the midpoint formula and the given coordi-nates for the vertices.

3.Find a2 by solving for the length of the transverse axis, 2a, which is the distance between thegiven vertices.

4.Find c2 usingh and k found in Step 2 along with the given coordinates for the foci.5.Solve for b2 using the equation b2 = c2 − a2.6.Substitute the values forh, k, a2, and b2 into the standard form of the equation determined inStep 1.

Example 8.10Finding the Equation of a Hyperbola Centered at (h, k) Given its Foci and VerticesWhat is the standard form equation of the hyperbola that has vertices at(0,−2)and(6,−2)andfoci at(−2,−2)and(8,−2)?

SolutionThe y-coordinates of the vertices and foci are the same, so the transverse axis is parallel to thex-axis. Thus, the equation of the hyperbola will have the form

(x− h)2

a2− (y − k)2

b2= 1 (8.39)

First, we identify the center, (h, k) .The center is halfway between thevertices (0,−2) and (6,−2) .Applying the midpoint formula, we have

(h, k) =(

0 + 62

,−2 + (−2)

2

)= (3,−2) (8.40)

Next, we �nd a2.The length of the transverse axis, 2a,is bounded by the vertices. So, we can�nd a2 by �nding the distance between the x-coordinates of the vertices.

2a = |0− 6|2a = 6

a = 3

a2 = 9

(8.41)

Now we need to �nd c2.The coordinates of the foci are (h± c, k) . So (h− c, k) =(−2,−2) and(h+ c, k) = (8,−2) .We can use the x-coordinate from either of these points to solvefor c.Using the point(8,−2) , and substitutingh = 3,

Available for free at Connexions <http://legacy.cnx.org/content/col11759/1.2>

850 CHAPTER 8. ANALYTIC GEOMETRY

h+ c = 8

3 + c = 8

c = 5

c2 = 25

(8.42)

Next, solve for b2 using the equation b2 = c2 − a2 :

b2 = c2 − a2

= 25− 9

= 16

(8.43)

Finally, substitute the values found forh, k, a2,and b2 into the standard form of the equation.

(x− 3)2

9− (y + 2)2

16= 1 (8.44)

Try It:

Exercise 8.3.3 (Solution on p. 949.)

What is the standard form equation of the hyperbola that hasvertices (1,−2) and (1, 8) and foci (1,−10) and (1, 16)?

8.3.3 Graphing Hyperbolas Centered at the Origin

When we have an equation in standard form for a hyperbola centered at the origin, we can interpret itsparts to identify the key features of its graph: the center, vertices, co-vertices, asymptotes, foci, and lengthsand positions of the transverse and conjugate axes. To graph hyperbolas centered at the origin, we use

the standard form x2

a2 − y2

b2 = 1 for horizontal hyperbolas and the standard form y2

a2 − x2

b2 = 1 for verticalhyperbolas.

How To: Given a standard form equation for a hyperbola centered at (0, 0) , sketchthe graph.

1.Determine which of the standard forms applies to the given equation.2.Use the standard form identi�ed in Step 1 to determine the position of the transverse axis;coordinates for the vertices, co-vertices, and foci; and the equations for the asymptotes.

a.If the equation is in the form x2

a2 − y2

b2 = 1, then• the transverse axis is on the x-axis• the coordinates of the vertices are (±a, 0)• the coordinates of the co-vertices are (0,±b)• the coordinates of the foci are (±c, 0)• the equations of the asymptotes are y = ± b

ax

b.If the equation is in the form y2

a2 − x2

b2 = 1, then

Available for free at Connexions <http://legacy.cnx.org/content/col11759/1.2>

851

• the transverse axis is on the y-axis• the coordinates of the vertices are (0,±a)• the coordinates of the co-vertices are (±b, 0)• the coordinates of the foci are (0,±c)• the equations of the asymptotes are y = ±abx

3.Solve for the coordinates of the foci using the equation c = ±√a2 + b2.

4.Plot the vertices, co-vertices, foci, and asymptotes in the coordinate plane, and draw a smoothcurve to form the hyperbola.

Example 8.11Graphing a Hyperbola Centered at (0, 0) Given an Equation in Standard Form

Graph the hyperbola given by the equation y2

64−x2

36 = 1. Identify and label the vertices, co-vertices,foci, and asymptotes.

SolutionThe standard form that applies to the given equation is y

2

a2 − x2

b2 = 1. Thus, the transverse axis ison the y-axis

The coordinates of the vertices are (0,±a) =(0,±√

64)

= (0,±8)The coordinates of the co-vertices are (±b, 0) =

(±√

36, 0)

= (±6, 0)The coordinates of the foci are (0,±c) , where c = ±

√a2 + b2. Solving for c, we have

c = ±√a2 + b2 = ±

√64 + 36 = ±

√100 = ±10 (8.45)

Therefore, the coordinates of the foci are (0,±10)The equations of the asymptotes are y = ±abx = ± 8

6x = ± 43x

Plot and label the vertices and co-vertices, and then sketch the central rectangle. Sides of therectangle are parallel to the axes and pass through the vertices and co-vertices. Sketch and extendthe diagonals of the central rectangle to show the asymptotes. The central rectangle and asymptotesprovide the framework needed to sketch an accurate graph of the hyperbola. Label the foci andasymptotes, and draw a smooth curve to form the hyperbola, as shown in Figure 8.22.

Available for free at Connexions <http://legacy.cnx.org/content/col11759/1.2>

852 CHAPTER 8. ANALYTIC GEOMETRY

Figure 8.22

Try It:

Exercise 8.3.4 (Solution on p. 949.)

Graph the hyperbola given by the equation x2

144 −y2

81 = 1. Identify and label the vertices,co-vertices, foci, and asymptotes.

8.3.4 Graphing Hyperbolas Not Centered at the Origin

Graphing hyperbolas centered at a point (h, k)other than the origin is similar to graphing ellipses centered

at a point other than the origin. We use the standard forms (x−h)2

a2 − (y−k)2

b2 = 1 for horizontal hyperbolas,

and (y−k)2

a2 − (x−h)2

b2 = 1 for vertical hyperbolas. From these standard form equations we can easily calculate

Available for free at Connexions <http://legacy.cnx.org/content/col11759/1.2>

853

and plot key features of the graph: the coordinates of its center, vertices, co-vertices, and foci; the equationsof its asymptotes; and the positions of the transverse and conjugate axes.

How To: Given a general form for a hyperbola centered at (h, k) , sketch the graph.

1.Convert the general form to that standard form. Determine which of the standard formsapplies to the given equation.

2.Use the standard form identi�ed in Step 1 to determine the position of the transverse axis;coordinates for the center, vertices, co-vertices, foci; and equations for the asymptotes.

a.If the equation is in the form (x−h)2

a2 − (y−k)2

b2 = 1, then• the transverse axis is parallel to the x-axis• the center is (h, k)• the coordinates of the vertices are (h± a, k)• the coordinates of the co-vertices are (h, k ± b)• the coordinates of the foci are (h± c, k)• the equations of the asymptotes are y = ± b

a (x− h) + k

b.If the equation is in the form (y−k)2

a2 − (x−h)2

b2 = 1, then• the transverse axis is parallel to the y-axis• the center is (h, k)• the coordinates of the vertices are (h, k ± a)• the coordinates of the co-vertices are (h± b, k)• the coordinates of the foci are (h, k ± c)• the equations of the asymptotes are y = ±ab (x− h) + k

3.Solve for the coordinates of the foci using the equation c = ±√a2 + b2.

4.Plot the center, vertices, co-vertices, foci, and asymptotes in the coordinate plane and draw asmooth curve to form the hyperbola.

Example 8.12Graphing a Hyperbola Centered at (h, k) Given an Equation in General FormGraph the hyperbola given by the equation 9x2 − 4y2 − 36x − 40y − 388 = 0. Identify and labelthe center, vertices, co-vertices, foci, and asymptotes.

SolutionStart by expressing the equation in standard form. Group terms that contain the same variable,and move the constant to the opposite side of the equation.

(9x2 − 36x

)−(4y2 + 40y

)= 388 (8.46)

Factor the leading coe�cient of each expression.

9(x2 − 4x

)− 4

(y2 + 10y

)= 388 (8.47)

Complete the square twice. Remember to balance the equation by adding the same constants toeach side.

9(x2 − 4x+ 4

)− 4

(y2 + 10y + 25

)= 388 + 36− 100 (8.48)

Rewrite as perfect squares.

9(x− 2)2 − 4(y + 5)2 = 324 (8.49)

Divide both sides by the constant term to place the equation in standard form.

Available for free at Connexions <http://legacy.cnx.org/content/col11759/1.2>

854 CHAPTER 8. ANALYTIC GEOMETRY

(x− 2)2

36− (y + 5)2

81= 1 (8.50)

The standard form that applies to the given equation is (x−h)2

a2 − (y−k)2

b2 = 1, where a2 = 36 and b2 =81,or a = 6 and b = 9.Thus, the transverse axis is parallel to the x-axis. It follows that:

• the center of the ellipse is (h, k) = (2,−5)• the coordinates of the vertices are (h± a, k) = (2± 6,−5) , or (−4,−5) and (8,−5)• the coordinates of the co-vertices are (h, k ± b) = (2,−5± 9) , or (2,−14) and (2, 4)• the coordinates of the foci are (h± c, k) , where c = ±

√a2 + b2. Solving for c,we have

c = ±√

36 + 81 = ±√

117 = ±3√

13 (8.51)

Therefore, the coordinates of the foci are(2− 3

√13,−5

)and

(2 + 3

√13,−5

).

The equations of the asymptotes are y = ± ba (x− h) + k = ± 3

2 (x− 2)− 5.Next, we plot and label the center, vertices, co-vertices, foci, and asymptotes and draw smooth

curves to form the hyperbola, as shown in Figure 8.23.

Available for free at Connexions <http://legacy.cnx.org/content/col11759/1.2>

855

Figure 8.23

Try It:

Exercise 8.3.5 (Solution on p. 950.)

Graph the hyperbola given by the standard form of an equation (y+4)2

100 − (x−3)2

64 =1. Identify and label the center, vertices, co-vertices, foci, and asymptotes.

Available for free at Connexions <http://legacy.cnx.org/content/col11759/1.2>

856 CHAPTER 8. ANALYTIC GEOMETRY

8.3.5 Solving Applied Problems Involving Hyperbolas

As we discussed at the beginning of this section, hyperbolas have real-world applications in many �elds,such as astronomy, physics, engineering, and architecture. The design e�ciency of hyperbolic cooling towersis particularly interesting. Cooling towers are used to transfer waste heat to the atmosphere and are oftentouted for their ability to generate power e�ciently. Because of their hyperbolic form, these structures areable to withstand extreme winds while requiring less material than any other forms of their size and strength.See Figure 8.24. For example, a 500-foot tower can be made of a reinforced concrete shell only 6 or 8 incheswide!

Figure 8.24: Cooling towers at the Drax power station in North Yorkshire, United Kingdom (credit:Les Haines, Flickr)

The �rst hyperbolic towers were designed in 1914 and were 35 meters high. Today, the tallest coolingtowers are in France, standing a remarkable 170 meters tall. In Example 8.13 we will use the design layoutof a cooling tower to �nd a hyperbolic equation that models its sides.

Available for free at Connexions <http://legacy.cnx.org/content/col11759/1.2>

857

Example 8.13Solving Applied Problems Involving HyperbolasThe design layout of a cooling tower is shown in Figure 8.25. The tower stands 179.6 meters tall.The diameter of the top is 72 meters. At their closest, the sides of the tower are 60 meters apart.

Figure 8.25: Project design for a natural draft cooling tower

Find the equation of the hyperbola that models the sides of the cooling tower. Assume thatthe center of the hyperbola�indicated by the intersection of dashed perpendicular lines in the�gure�is the origin of the coordinate plane. Round �nal values to four decimal places.

SolutionWe are assuming the center of the tower is at the origin, so we can use the standard form of a

horizontal hyperbola centered at the origin: x2

a2 − y2

b2 = 1,where the branches of the hyperbola formthe sides of the cooling tower. We must �nd the values of a2 and b2to complete the model.

First, we �nd a2.Recall that the length of the transverse axis of a hyperbola is 2a.This lengthis represented by the distance where the sides are closest, which is given as 65.3 meters. So, 2a =

Available for free at Connexions <http://legacy.cnx.org/content/col11759/1.2>

858 CHAPTER 8. ANALYTIC GEOMETRY

60.Therefore, a = 30 and a2 = 900.To solve for b2,we need to substitute forx and y in our equation using a known point. To do this,

we can use the dimensions of the tower to �nd some point (x, y) that lies on the hyperbola. Wewill use the top right corner of the tower to represent that point. Since the y-axis bisects the tower,our x-value can be represented by the radius of the top, or 36 meters. The y-value is representedby the distance from the origin to the top, which is given as 79.6 meters. Therefore,

x2

a2 − y2

b2 = 1 Standard form of horizontal hyperbola.

b2 = y2

x2

a2−1Isolate b2

= (79.6)2

(36)2900 −1

Substitute for a2, x, and y

≈ 14400.3636 Round to four decimal places

(8.52)

The sides of the tower can be modeled by the hyperbolic equation

x2

900− y2

14400.3636= 1, or

x2

302 −y2

120.00152 = 1 (8.53)

Try It:

Exercise 8.3.6 (Solution on p. 951.)

A design for a cooling tower project is shown in Figure 8.26. Find the equation of thehyperbola that models the sides of the cooling tower. Assume that the center of thehyperbola�indicated by the intersection of dashed perpendicular lines in the �gure�isthe origin of the coordinate plane. Round �nal values to four decimal places.

Available for free at Connexions <http://legacy.cnx.org/content/col11759/1.2>

859

Figure 8.26

Media: Access these online resources for additional instruction and practice with hyperbolas.

• Conic Sections: The Hyperbola Part 1 of 28• Conic Sections: The Hyperbola Part 2 of 29• Graph a Hyperbola with Center at Origin10

• Graph a Hyperbola with Center not at Origin11

8http://openstaxcollege.org/l/hyperbola19http://openstaxcollege.org/l/hyperbola2

10http://openstaxcollege.org/l/hyperbolaorigin11http://openstaxcollege.org/l/hbnotorigin

Available for free at Connexions <http://legacy.cnx.org/content/col11759/1.2>

860 CHAPTER 8. ANALYTIC GEOMETRY

8.3.6 Key Equations

Hyperbola, center at origin, transverse axis on x-axis x2

a2 − y2

b2 = 1

Hyperbola, center at origin, transverse axis on y-axis y2

a2 − x2

b2 = 1

Hyperbola, center at (h, k) ,transverse axis parallel to x-axis (x−h)2

a2 − (y−k)2

b2 = 1

Hyperbola, center at (h, k) ,transverse axis parallel to y-axis (y−k)2

a2 − (x−h)2

b2 = 1

Table 8.2

8.3.7 Key Concepts

• A hyperbola is the set of all points (x, y) in a plane such that the di�erence of the distancesbetween (x, y) and the foci is a positive constant.

• The standard form of a hyperbola can be used to locate its vertices and foci. See Example 8.8.• When given the coordinates of the foci and vertices of a hyperbola, we can write the equation of the

hyperbola in standard form. See Example 8.9 and Example 8.10.• When given an equation for a hyperbola, we can identify its vertices, co-vertices, foci, asymptotes,

and lengths and positions of the transverse and conjugate axes in order to graph the hyperbola. SeeExample 8.11 and Example 8.12.

• Real-world situations can be modeled using the standard equations of hyperbolas. For instance, giventhe dimensions of a natural draft cooling tower, we can �nd a hyperbolic equation that models itssides. See Example 8.13.

8.3.8 Section Exercises

8.3.8.1 Verbal

Exercise 8.3.7 (Solution on p. 951.)

De�ne a hyperbola in terms of its foci.

Exercise 8.3.8What can we conclude about a hyperbola if its asymptotes intersect at the origin?

Exercise 8.3.9 (Solution on p. 951.)

What must be true of the foci of a hyperbola?

Exercise 8.3.10If the transverse axis of a hyperbola is vertical, what do we know about the graph?

Exercise 8.3.11 (Solution on p. 951.)

Where must the center of hyperbola be relative to its foci?

8.3.8.2 Algebraic

For the following exercises, determine whether the following equations represent hyperbolas. If so, write instandard form.

Exercise 8.3.123y2 + 2x = 6Exercise 8.3.13 (Solution on p. 951.)x2

36 −y2

9 = 1

Available for free at Connexions <http://legacy.cnx.org/content/col11759/1.2>

861

Exercise 8.3.145y2 + 4x2 = 6xExercise 8.3.15 (Solution on p. 951.)

25x2 − 16y2 = 400Exercise 8.3.16−9x2 + 18x+ y2 + 4y − 14 = 0

For the following exercises, write the equation for the hyperbola in standard form if it is not already, andidentify the vertices and foci, and write equations of asymptotes.

Exercise 8.3.17 (Solution on p. 952.)x2

25 −y2

36 = 1Exercise 8.3.18x2

100 −y2

9 = 1Exercise 8.3.19 (Solution on p. 952.)y2

4 −x2

81 = 1Exercise 8.3.209y2 − 4x2 = 1Exercise 8.3.21 (Solution on p. 952.)(x−1)2

9 − (y−2)2

16 = 1Exercise 8.3.22(y−6)2

36 − (x+1)2

16 = 1Exercise 8.3.23 (Solution on p. 952.)(x−2)2

49 − (y+7)2

49 = 1Exercise 8.3.244x2 − 8x− 9y2 − 72y + 112 = 0Exercise 8.3.25 (Solution on p. 952.)

−9x2 − 54x+ 9y2 − 54y + 81 = 0Exercise 8.3.264x2 − 24x− 36y2 − 360y + 864 = 0Exercise 8.3.27 (Solution on p. 952.)

−4x2 + 24x+ 16y2 − 128y + 156 = 0Exercise 8.3.28−4x2 + 40x+ 25y2 − 100y + 100 = 0Exercise 8.3.29 (Solution on p. 952.)

x2 + 2x− 100y2 − 1000y + 2401 = 0Exercise 8.3.30−9x2 + 72x+ 16y2 + 16y + 4 = 0Exercise 8.3.31 (Solution on p. 952.)

4x2 + 24x− 25y2 + 200y − 464 = 0For the following exercises, �nd the equations of the asymptotes for each hyperbola.

Exercise 8.3.32y2

32 − x2

32 = 1Exercise 8.3.33 (Solution on p. 952.)(x−3)2

52 − (y+4)2

22 = 1Exercise 8.3.34(y−3)2

32 − (x+5)2

62 = 1

Available for free at Connexions <http://legacy.cnx.org/content/col11759/1.2>

862 CHAPTER 8. ANALYTIC GEOMETRY

Exercise 8.3.35 (Solution on p. 952.)

9x2 − 18x− 16y2 + 32y − 151 = 0Exercise 8.3.3616y2 + 96y − 4x2 + 16x+ 112 = 0

8.3.8.3 Graphical

For the following exercises, sketch a graph of the hyperbola, labeling vertices and foci.

Exercise 8.3.37 (Solution on p. 952.)x2

49 −y2

16 = 1Exercise 8.3.38x2

64 −y2

4 = 1Exercise 8.3.39 (Solution on p. 952.)y2

9 −x2

25 = 1Exercise 8.3.4081x2 − 9y2 = 1Exercise 8.3.41 (Solution on p. 953.)(y+5)2

9 − (x−4)2

25 = 1Exercise 8.3.42(x−2)2

8 − (y+3)2

27 = 1Exercise 8.3.43 (Solution on p. 953.)(y−3)2

9 − (x−3)2

9 = 1Exercise 8.3.44−4x2 − 8x+ 16y2 − 32y − 52 = 0Exercise 8.3.45 (Solution on p. 954.)

x2 − 8x− 25y2 − 100y − 109 = 0Exercise 8.3.46−x2 + 8x+ 4y2 − 40y + 88 = 0Exercise 8.3.47 (Solution on p. 954.)

64x2 + 128x− 9y2 − 72y − 656 = 0Exercise 8.3.4816x2 + 64x− 4y2 − 8y − 4 = 0Exercise 8.3.49 (Solution on p. 955.)

−100x2 + 1000x+ y2 − 10y − 2575 = 0Exercise 8.3.504x2 + 16x− 4y2 + 16y + 16 = 0

For the following exercises, given information about the graph of the hyperbola, �nd its equation.

Exercise 8.3.51 (Solution on p. 955.)

Vertices at (3, 0) and (−3, 0) and one focus at (5, 0) .Exercise 8.3.52Vertices at (0, 6) and (0,−6) and one focus at (0,−8) .Exercise 8.3.53 (Solution on p. 956.)

Vertices at (1, 1) and (11, 1) and one focus at (12, 1) .Exercise 8.3.54Center: (0, 0) ;vertex: (0,−13) ;one focus:

(0,√

313).

Available for free at Connexions <http://legacy.cnx.org/content/col11759/1.2>

863

Exercise 8.3.55 (Solution on p. 956.)

Center: (4, 2) ;vertex: (9, 2) ;one focus:(4 +√

26, 2).

Exercise 8.3.56Center: (3, 5) ; vertex: (3, 11) ; one focus:

(3, 5 + 2

√10).

For the following exercises, given the graph of the hyperbola, �nd its equation.Exercise 8.3.57 (Solution on p. 956.)

Available for free at Connexions <http://legacy.cnx.org/content/col11759/1.2>

864 CHAPTER 8. ANALYTIC GEOMETRY

Exercise 8.3.58

Available for free at Connexions <http://legacy.cnx.org/content/col11759/1.2>

865

Exercise 8.3.59 (Solution on p. 956.)

Available for free at Connexions <http://legacy.cnx.org/content/col11759/1.2>

866 CHAPTER 8. ANALYTIC GEOMETRY

Exercise 8.3.60

Available for free at Connexions <http://legacy.cnx.org/content/col11759/1.2>

867

Exercise 8.3.61 (Solution on p. 956.)

8.3.8.4 Extensions

For the following exercises, express the equation for the hyperbola as two functions, with y as a functionofx.Express as simply as possible. Use a graphing calculator to sketch the graph of the two functions onthe same axes.

Exercise 8.3.62x2

4 −y2

9 = 1Exercise 8.3.63 (Solution on p. 956.)y2

9 −x2

1 = 1Exercise 8.3.64(x−2)2

16 − (y+3)2

25 = 1Exercise 8.3.65 (Solution on p. 956.)

−4x2 − 16x+ y2 − 2y − 19 = 0Exercise 8.3.664x2 − 24x− y2 − 4y + 16 = 0

Available for free at Connexions <http://legacy.cnx.org/content/col11759/1.2>

868 CHAPTER 8. ANALYTIC GEOMETRY

8.3.8.5 Real-World Applications

For the following exercises, a hedge is to be constructed in the shape of a hyperbola near a fountain at thecenter of the yard. Find the equation of the hyperbola and sketch the graph.

Exercise 8.3.67 (Solution on p. 957.)

The hedge will follow the asymptotes y = x and y = −x, and its closest distance to the centerfountain is 5 yards.

Exercise 8.3.68The hedge will follow the asymptotes y = 2x and y = −2x, and its closest distance to the centerfountain is 6 yards.

Exercise 8.3.69 (Solution on p. 958.)

The hedge will follow the asymptotes y = 12x and y = − 1

2x, and its closest distance to the centerfountain is 10 yards.

Exercise 8.3.70The hedge will follow the asymptotes y = 2

3x and y = − 23x, and its closest distance to the center

fountain is 12 yards.

Exercise 8.3.71 (Solution on p. 958.)

The hedge will follow the asymptotes y = 34x and y = − 3

4x, and its closest distance to the centerfountain is 20 yards.

For the following exercises, assume an object enters our solar system and we want to graph its path on acoordinate system with the sun at the origin and the x-axis as the axis of symmetry for the object's path.Give the equation of the �ight path of each object using the given information.

Exercise 8.3.72The object enters along a path approximated by the line y = x − 2 and passes within 1 au (astro-nomical unit) of the sun at its closest approach, so that the sun is one focus of the hyperbola. Itthen departs the solar system along a path approximated by the line y = −x+ 2.Exercise 8.3.73 (Solution on p. 959.)

The object enters along a path approximated by the line y = 2x − 2 and passes within 0.5 au ofthe sun at its closest approach, so the sun is one focus of the hyperbola. It then departs the solarsystem along a path approximated by the line y = −2x+ 2.Exercise 8.3.74The object enters along a path approximated by the line y = 0.5x + 2 and passes within 1 au ofthe sun at its closest approach, so the sun is one focus of the hyperbola. It then departs the solarsystem along a path approximated by the line y = −0.5x− 2.Exercise 8.3.75 (Solution on p. 959.)

The object enters along a path approximated by the line y = 13x− 1 and passes within 1 au of the

sun at its closest approach, so the sun is one focus of the hyperbola. It then departs the solarsystem along a path approximated by the line y = − 1

3x+ 1.Exercise 8.3.76The object It enters along a path approximated by the line y = 3x − 9 and passes within 1 au ofthe sun at its closest approach, so the sun is one focus of the hyperbola. It then departs the solarsystem along a path approximated by the line y = −3x+ 9.

Available for free at Connexions <http://legacy.cnx.org/content/col11759/1.2>

869

8.4 The Parabola12

Figure 8.27: The Olympic torch concludes its journey around the world when it is used to light theOlympic cauldron during the opening ceremony. (credit: Ken Hackman, U.S. Air Force)

Did you know that the Olympic torch is lit several months before the start of the games? The ceremonialmethod for lighting the �ame is the same as in ancient times. The ceremony takes place at the Temple ofHera in Olympia, Greece, and is rooted in Greek mythology, paying tribute to Prometheus, who stole �refrom Zeus to give to all humans. One of eleven acting priestesses places the torch at the focus of a parabolicmirror (see Figure 8.27), which focuses light rays from the sun to ignite the �ame.

Parabolic mirrors (or re�ectors) are able to capture energy and focus it to a single point. The advantages ofthis property are evidenced by the vast list of parabolic objects we use every day: satellite dishes, suspensionbridges, telescopes, microphones, spotlights, and car headlights, to name a few. Parabolic re�ectors are alsoused in alternative energy devices, such as solar cookers and water heaters, because they are inexpensive tomanufacture and need little maintenance. In this section we will explore the parabola and its uses, includinglow-cost, energy-e�cient solar designs.

12This content is available online at <http://legacy.cnx.org/content/m49440/1.3/>.

Available for free at Connexions <http://legacy.cnx.org/content/col11759/1.2>

870 CHAPTER 8. ANALYTIC GEOMETRY

8.4.1 Graphing Parabolas with Vertices at the Origin

In The Ellipse (Section 8.2), we saw that an ellipse is formed when a plane cuts through a right circular cone.If the plane is parallel to the edge of the cone, an unbounded curve is formed. This curve is a parabola.See Figure 8.28.

Figure 8.28: Parabola

Like the ellipse and hyperbola, the parabola can also be de�ned by a set of points in the coordinateplane. A parabola is the set of all points (x, y) in a plane that are the same distance from a �xed line, calledthe directrix, and a �xed point (the focus) not on the directrix.

In Quadratic Functions13, we learned about a parabola's vertex and axis of symmetry. Now we extend thediscussion to include other key features of the parabola. See Figure 8.29. Notice that the axis of symmetry

13"Power Functions and Polynomial Functions" <http://legacy.cnx.org/content/m49346/latest/>

Available for free at Connexions <http://legacy.cnx.org/content/col11759/1.2>

871

passes through the focus and vertex and is perpendicular to the directrix. The vertex is the midpoint betweenthe directrix and the focus.

The line segment that passes through the focus and is parallel to the directrix is called the latus rectum.The endpoints of the latus rectum lie on the curve. By de�nition, the distance d from the focus to anypointP on the parabola is equal to the distance fromP to the directrix.

Figure 8.29: Key features of the parabola

To work with parabolas in the coordinate plane, we consider two cases: those with a vertex at theorigin and those with a vertex at a point other than the origin. We begin with the former.

Available for free at Connexions <http://legacy.cnx.org/content/col11759/1.2>

872 CHAPTER 8. ANALYTIC GEOMETRY

Figure 8.30

Let (x, y) be a point on the parabola with vertex (0, 0) , focus (0, p) ,and directrix y = −p as shown inFigure 8.30. The distance d from point (x, y) to point (x,−p) on the directrix is the di�erence of the y-values: d = y+p.The distance from the focus (0, p) to the point (x, y) is also equal to d and can be expressedusing the distance formula.

d =√

(x− 0)2 + (y − p)2

=√x2 + (y − p)2

(8.54)

Set the two expressions for d equal to each other and solve for y to derive the equation of the parabola. Wedo this because the distance from (x, y) to (0, p) equals the distance from (x, y) to (x, −p) .√

x2 + (y − p)2 = y + p (8.55)

We then square both sides of the equation, expand the squared terms, and simplify by combining like terms.

x2 + (y − p)2 = (y + p)2

x2 + y2 − 2py + p2 = y2 + 2py + p2

x2 − 2py = 2py

x2 = 4py

(8.56)

Available for free at Connexions <http://legacy.cnx.org/content/col11759/1.2>

873

The equations of parabolas with vertex (0, 0) are y2 = 4pxwhen the x-axis is the axis of symmetry andx2 =4pywhen the y-axis is the axis of symmetry. These standard forms are given below, along with their generalgraphs and key features.

A General Note: Table 8.3 and Figure 8.31 summarize the standard features of parabolas witha vertex at the origin.

Axis of Symmetry Equation Focus Directrix Endpoints of Latus Rectum

x-axis y2 = 4px (p, 0) x = −p (p, ± 2p)

y-axis x2 = 4py (0, p) y = −p (±2p, p)

Table 8.3

Figure 8.31: (a) When p > 0 and the axis of symmetry is the x-axis, the parabola opens right. (b)When p < 0 and the axis of symmetry is the x-axis, the parabola opens left. (c) When p < 0 and theaxis of symmetry is the y-axis, the parabola opens up. (d) When p < 0 and the axis of symmetry is they-axis, the parabola opens down.

Available for free at Connexions <http://legacy.cnx.org/content/col11759/1.2>

874 CHAPTER 8. ANALYTIC GEOMETRY

The key features of a parabola are its vertex, axis of symmetry, focus, directrix, and latus rectum. SeeFigure 8.31. When given a standard equation for a parabola centered at the origin, we can easily identifythe key features to graph the parabola.

A line is said to be tangent to a curve if it intersects the curve at exactly one point. If we sketch linestangent to the parabola at the endpoints of the latus rectum, these lines intersect on the axis of symmetry,as shown in Figure 8.32.

Figure 8.32

Available for free at Connexions <http://legacy.cnx.org/content/col11759/1.2>

875

How To: Given a standard form equation for a parabola centered at (0, 0), sketchthe graph.

1.Determine which of the standard forms applies to the given equation: y2 = 4px orx2 = 4py.2.Use the standard form identi�ed in Step 1 to determine the axis of symmetry, focus, equationof the directrix, and endpoints of the latus rectum.

a.If the equation is in the form y2 = 4px, then• the axis of symmetry is the x-axis, y = 0• set 4p equal to the coe�cient of x in the given equation to solve for p. If p > 0, theparabola opens right. If p < 0, the parabola opens left.• use p to �nd the coordinates of the focus, (p, 0)• use p to �nd the equation of the directrix,x = −p• use p to �nd the endpoints of the latus rectum, (p,±2p) .Alternately, substitutex =p into the original equation.

b.If the equation is in the formx2 = 4py,then• the axis of symmetry is the y-axis,x = 0• set 4p equal to the coe�cient of y in the given equation to solve for p. If p > 0, theparabola opens up. If p < 0, the parabola opens down.• use p to �nd the coordinates of the focus, (0, p)• use p to �nd equation of the directrix, y = −p• use p to �nd the endpoints of the latus rectum, (±2p, p)

3.Plot the focus, directrix, and latus rectum, and draw a smooth curve to form the parabola.

Example 8.14Graphing a Parabola with Vertex (0, 0) and the x-axis as the Axis of SymmetryGraph y2 = 24x. Identify and label the focus, directrix, and endpoints of the latus rectum.

SolutionThe standard form that applies to the given equation is y2 = 4px.Thus, the axis of symmetry isthe x-axis. It follows that:

• 24 = 4p, so p = 6. Since p > 0, the parabola opens right• the coordinates of the focus are (p, 0) = (6, 0)• the equation of the directrix isx = −p = −6• the endpoints of the latus rectum have the same x-coordinate at the focus. To �nd the

endpoints, substitutex = 6 into the original equation: (6,±12)

Next we plot the focus, directrix, and latus rectum, and draw a smooth curve to form theparabola. Figure 8.33

Available for free at Connexions <http://legacy.cnx.org/content/col11759/1.2>

876 CHAPTER 8. ANALYTIC GEOMETRY

Figure 8.33

Try It:

Exercise 8.4.1 (Solution on p. 959.)

Graph y2 = −16x. Identify and label the focus, directrix, and endpoints of the latus rec-tum.

Example 8.15Graphing a Parabola with Vertex (0, 0) and the y-axis as the Axis of SymmetryGraphx2 = −6y. Identify and label the focus, directrix, and endpoints of the latus rectum.

SolutionThe standard form that applies to the given equation isx2 = 4py.Thus, the axis of symmetry isthe y-axis. It follows that:

• −6 = 4p,so p = − 32 . Since p < 0, the parabola opens down.

Available for free at Connexions <http://legacy.cnx.org/content/col11759/1.2>

877

• the coordinates of the focus are (0, p) =(0,− 3

2

)• the equation of the directrix is y = −p = 3

2• the endpoints of the latus rectum can be found by substituting y = 3

2 into the originalequation,

(±3,− 3

2

)Next we plot the focus, directrix, and latus rectum, and draw a smooth curve to form the

parabola.

Figure 8.34

Try It:

Exercise 8.4.2 (Solution on p. 960.)

Graphx2 = 8y. Identify and label the focus, directrix, and endpoints of the latus rectum.

8.4.2 Writing Equations of Parabolas in Standard Form

In the previous examples, we used the standard form equation of a parabola to calculate the locations of itskey features. We can also use the calculations in reverse to write an equation for a parabola when given itskey features.

Available for free at Connexions <http://legacy.cnx.org/content/col11759/1.2>

878 CHAPTER 8. ANALYTIC GEOMETRY

How To: Given its focus and directrix, write the equation for a parabola in standardform.

1.Determine whether the axis of symmetry is the x- or y-axis.

a.If the given coordinates of the focus have the form (p, 0) , then the axis of symmetry isthe x-axis. Use the standard form y2 = 4px.

b.If the given coordinates of the focus have the form (0, p) , then the axis of symmetry isthe y-axis. Use the standard formx2 = 4py.

2.Multiply 4p.3.Substitute the value from Step 2 into the equation determined in Step 1.

Example 8.16Writing the Equation of a Parabola in Standard Form Given its Focus and DirectrixWhat is the equation for the parabola with focus

(− 1

2 , 0)and directrixx = 1

2?

SolutionThe focus has the form (p, 0) , so the equation will have the form y2 = 4px.

• Multiplying 4p, we have 4p = 4(− 1

2

)= −2.

• Substituting for 4p, we have y2 = 4px = −2x.

Therefore, the equation for the parabola is y2 = −2x.

Try It:

Exercise 8.4.3 (Solution on p. 961.)

What is the equation for the parabola with focus(0, 7

2

)and directrix y = − 7

2?

8.4.3 Graphing Parabolas with Vertices Not at the Origin

Like other graphs we've worked with, the graph of a parabola can be translated. If a parabola istranslatedh units horizontally and k units vertically, the vertex will be (h, k) .This translation results in thestandard form of the equation we saw previously withx replaced by (x− h) and y replaced by (y − k) .

To graph parabolas with a vertex (h, k) other than the origin, we use the standard form (y − k)2 =4p (x− h) for parabolas that have an axis of symmetry parallel to the x-axis, and (x− h)2 = 4p (y − k) forparabolas that have an axis of symmetry parallel to the y-axis. These standard forms are given below, alongwith their general graphs and key features.

A General Note: Table 8.4 and Figure 8.35 summarize the standard features of parabolas witha vertex at a point (h, k) .

Axis of Sym-metry

Equation Focus Directrix Endpoints ofLatus Rectum

continued on next page

Available for free at Connexions <http://legacy.cnx.org/content/col11759/1.2>

879

y = k (y − k)2 =4p (x− h)

(h+ p, k) x = h− p (h+ p, k ± 2p)

x = h (x− h)2 =4p (y − k)

(h, k + p) y = k − p (h± 2p, k + p)

Table 8.4

Figure 8.35: (a) When p > 0, the parabola opens right. (b) When p < 0, the parabola opens left. (c)When p > 0,the parabola opens up. (d) When p < 0, the parabola opens down.

How To: Given a standard form equation for a parabola centered at (h, k), sketchthe graph.

Available for free at Connexions <http://legacy.cnx.org/content/col11759/1.2>

880 CHAPTER 8. ANALYTIC GEOMETRY

1.Determine which of the standard forms applies to the given equation: (y − k)2 =4p (x− h) or (x− h)2 = 4p (y − k) .

2.Use the standard form identi�ed in Step 1 to determine the vertex, axis of symmetry, focus,equation of the directrix, and endpoints of the latus rectum.

a.If the equation is in the form (y − k)2 = 4p (x− h) ,then:• use the given equation to identifyh and k for the vertex, (h, k)• use the value of k to determine the axis of symmetry, y = k• set 4p equal to the coe�cient of (x− h) in the given equation to solve for p. If p > 0,theparabola opens right. If p < 0, the parabola opens left.• useh, k, and p to �nd the coordinates of the focus, (h+ p, k)• useh and p to �nd the equation of the directrix,x = h− p• useh, k, and p to �nd the endpoints of the latus rectum, (h+ p, k ± 2p)

b.If the equation is in the form (x− h)2 = 4p (y − k) ,then:• use the given equation to identifyh and k for the vertex, (h, k)• use the value ofh to determine the axis of symmetry,x = h• set 4p equal to the coe�cient of (y − k) in the given equation to solve for p. If p > 0,the parabola opens up. If p < 0, the parabola opens down.• useh, k, and p to �nd the coordinates of the focus, (h, k + p)• use k and p to �nd the equation of the directrix, y = k − p• useh, k, and p to �nd the endpoints of the latus rectum, (h± 2p, k + p)

3.Plot the vertex, axis of symmetry, focus, directrix, and latus rectum, and draw a smooth curveto form the parabola.

Example 8.17Graphing a Parabola with Vertex (h, k) and Axis of Symmetry Parallel to the x-axis

Graph (y − 1)2 = −16 (x+ 3) . Identify and label the vertex, axis of symmetry, focus, direc-trix, and endpoints of the latus rectum.

SolutionThe standard form that applies to the given equation is (y − k)2 = 4p (x− h) .Thus, the axis ofsymmetry is parallel to the x-axis. It follows that:

• the vertex is (h, k) = (−3, 1)• the axis of symmetry is y = k = 1• −16 = 4p,so p = −4. Since p < 0, the parabola opens left.• the coordinates of the focus are (h+ p, k) = (−3 + (−4) , 1) = (−7, 1)• the equation of the directrix isx = h− p = −3− (−4) = 1• the endpoints of the latus rectum are (h+ p, k ± 2p) = (−3 + (−4) , 1± 2 (−4)) ,

or (−7,−7) and (−7, 9)

Next we plot the vertex, axis of symmetry, focus, directrix, and latus rectum, and draw a smoothcurve to form the parabola. See Figure 8.36.

Available for free at Connexions <http://legacy.cnx.org/content/col11759/1.2>

881

Figure 8.36

Try It:

Exercise 8.4.4 (Solution on p. 961.)

Graph (y + 1)2 = 4 (x− 8) . Identify and label the vertex, axis of symmetry, focus, direc-trix, and endpoints of the latus rectum.

Available for free at Connexions <http://legacy.cnx.org/content/col11759/1.2>

882 CHAPTER 8. ANALYTIC GEOMETRY

Example 8.18Graphing a Parabola from an Equation Given in General FormGraphx2 − 8x − 28y − 208 = 0. Identify and label the vertex, axis of symmetry, focus, directrix,and endpoints of the latus rectum.

SolutionStart by writing the equation of the parabola in standard form. The standard form that appliesto the given equation is (x− h)2 = 4p (y − k) .Thus, the axis of symmetry is parallel to the y-axis.To express the equation of the parabola in this form, we begin by isolating the terms that containthe variablex in order to complete the square.

x2 − 8x− 28y − 208 = 0

x2 − 8x = 28y + 208

x2 − 8x+ 16 = 28y + 208 + 16

(x− 4)2 = 28y + 224

(x− 4)2 = 28 (y + 8)

(x− 4)2 = 4 · 7 · (y + 8)

(8.57)

It follows that:

• the vertex is (h, k) = (4,−8)• the axis of symmetry isx = h = 4• since p = 7, p > 0 and so the parabola opens up• the coordinates of the focus are (h, k + p) = (4,−8 + 7) = (4,−1)• the equation of the directrix is y = k − p = −8− 7 = −15• the endpoints of the latus rectum are (h± 2p, k + p) = (4± 2 (7) ,−8 + 7) ,

or (−10,−1) and (18,−1)

Next we plot the vertex, axis of symmetry, focus, directrix, and latus rectum, and draw a smoothcurve to form the parabola. See Figure 8.37.

Available for free at Connexions <http://legacy.cnx.org/content/col11759/1.2>

883

Figure 8.37

Try It:

Exercise 8.4.5 (Solution on p. 962.)

Graph (x+ 2)2 = −20 (y − 3) . Identify and label the vertex, axis of symmetry, focus,directrix, and endpoints of the latus rectum.

8.4.4 Solving Applied Problems Involving Parabolas

As we mentioned at the beginning of the section, parabolas are used to design many objects we use everyday, such as telescopes, suspension bridges, microphones, and radar equipment. Parabolic mirrors, such asthe one used to light the Olympic torch, have a very unique re�ecting property. When rays of light parallelto the parabola's axis of symmetry are directed toward any surface of the mirror, the light is re�ecteddirectly to the focus. See Figure 8.38. This is why the Olympic torch is ignited when it is held at the focusof the parabolic mirror.

Available for free at Connexions <http://legacy.cnx.org/content/col11759/1.2>

884 CHAPTER 8. ANALYTIC GEOMETRY

Figure 8.38: Re�ecting property of parabolas

Parabolic mirrors have the ability to focus the sun's energy to a single point, raising the temperaturehundreds of degrees in a matter of seconds. Thus, parabolic mirrors are featured in many low-cost, energye�cient solar products, such as solar cookers, solar heaters, and even travel-sized �re starters.

Example 8.19Solving Applied Problems Involving ParabolasA cross-section of a design for a travel-sized solar �re starter is shown in Figure 8.39. The sun'srays re�ect o� the parabolic mirror toward an object attached to the igniter. Because the igniteris located at the focus of the parabola, the re�ected rays cause the object to burn in just seconds.

a. Find the equation of the parabola that models the �re starter. Assume that the vertex of theparabolic mirror is the origin of the coordinate plane.

b. Use the equation found in part (a) to �nd the depth of the �re starter.

Available for free at Connexions <http://legacy.cnx.org/content/col11759/1.2>

885

Figure 8.39: Cross-section of a travel-sized solar �re starter

Solution

a. The vertex of the dish is the origin of the coordinate plane, so the parabola will take thestandard formx2 = 4py,where p > 0. The igniter, which is the focus, is 1.7 inches above thevertex of the dish. Thus we have p = 1.7.

x2 = 4py Standard form of upward-facing parabola with vertex (0,0)

x2 = 4 (1.7) y Substitute 1.7 for p.

x2 = 6.8y Multiply.

(8.58)

b. The dish extends 4.52 = 2.25 inches on either side of the origin. We can substitute 2.25 forx in

the equation from part (a) to �nd the depth of the dish.

x2 = 6.8y Equation found in part (a).

(2.25)2 = 6.8y Substitute 2.25 for x.

y ≈ 0.74 Solve for y.

(8.59)

The dish is about 0.74 inches deep.

Try It:

Exercise 8.4.6 (Solution on p. 963.)

Balcony-sized solar cookers have been designed for families living in India. The top of adish has a diameter of 1600 mm. The sun's rays re�ect o� the parabolic mirror towardthe �cooker,� which is placed 320 mm from the base.

Available for free at Connexions <http://legacy.cnx.org/content/col11759/1.2>

886 CHAPTER 8. ANALYTIC GEOMETRY

a.Find an equation that models a cross-section of the solar cooker. Assume that thevertex of the parabolic mirror is the origin of the coordinate plane, and that theparabola opens to the right (i.e., has the x-axis as its axis of symmetry).

b.Use the equation found in part (a) to �nd the depth of the cooker.

Media: Access these online resources for additional instruction and practice with parabolas.

• Conic Sections: The Parabola Part 1 of 214• Conic Sections: The Parabola Part 2 of 215• Parabola with Vertical Axis16

• Parabola with Horizontal Axis17

8.4.5 Key Equations

Parabola, vertex at origin, axis of symmetry on x-axis y2 = 4px

Parabola, vertex at origin, axis of symmetry on y-axis x2 = 4py

Parabola, vertex at (h, k) ,axis of symmetry on x-axis (y − k)2 = 4p (x− h)

Parabola, vertex at (h, k) ,axis of symmetry on y-axis (x− h)2 = 4p (y − k)

Table 8.5

8.4.6 Key Concepts

• A parabola is the set of all points (x, y) in a plane that are the same distance from a �xed line, calledthe directrix, and a �xed point (the focus) not on the directrix.

• The standard form of a parabola with vertex (0, 0) and the x-axis as its axis of symmetry can be usedto graph the parabola. If p > 0, the parabola opens right. If p < 0, the parabola opens left. SeeExample 8.14.

• The standard form of a parabola with vertex (0, 0) and the y-axis as its axis of symmetry can be usedto graph the parabola. If p > 0, the parabola opens up. If p < 0, the parabola opens down. SeeExample 8.15.

• When given the focus and directrix of a parabola, we can write its equation in standard form. SeeExample 8.16.

• The standard form of a parabola with vertex (h, k) and axis of symmetry parallel to the x-axis can beused to graph the parabola. If p > 0, the parabola opens right. If p < 0, the parabola opens left. SeeExample 8.17.

• The standard form of a parabola with vertex (h, k) and axis of symmetry parallel to the y-axis can beused to graph the parabola. If p > 0, the parabola opens up. If p < 0, the parabola opens down. SeeExample 8.18.

• Real-world situations can be modeled using the standard equations of parabolas. For instance, giventhe diameter and focus of a cross-section of a parabolic re�ector, we can �nd an equation that modelsits sides. See Example 8.19.

14http://openstaxcollege.org/l/parabola115http://openstaxcollege.org/l/parabola216http://openstaxcollege.org/l/parabolavertcal17http://openstaxcollege.org/l/parabolahoriz

Available for free at Connexions <http://legacy.cnx.org/content/col11759/1.2>

887

8.4.7 Section Exercises

8.4.7.1 Verbal

Exercise 8.4.7 (Solution on p. 963.)

De�ne a parabola in terms of its focus and directrix.

Exercise 8.4.8If the equation of a parabola is written in standard form and p is positive and the directrix is avertical line, then what can we conclude about its graph?

Exercise 8.4.9 (Solution on p. 963.)

If the equation of a parabola is written in standard form and p is negative and the directrix is ahorizontal line, then what can we conclude about its graph?

Exercise 8.4.10What is the e�ect on the graph of a parabola if its equation in standard form has increasing valuesof p?

Exercise 8.4.11 (Solution on p. 963.)

As the graph of a parabola becomes wider, what will happen to the distance between the focus anddirectrix?

8.4.7.2 Algebraic

For the following exercises, determine whether the given equation is a parabola. If so, rewrite the equationin standard form.

Exercise 8.4.12y2 = 4− x2

Exercise 8.4.13 (Solution on p. 963.)

y = 4x2

Exercise 8.4.143x2 − 6y2 = 12Exercise 8.4.15 (Solution on p. 963.)

(y − 3)2 = 8 (x− 2)Exercise 8.4.16y2 + 12x− 6y − 51 = 0

For the following exercises, rewrite the given equation in standard form, and then determine the vertex (V ) ,focus (F ) , and directrix (d) of the parabola.

Exercise 8.4.17 (Solution on p. 963.)

x = 8y2

Exercise 8.4.18y = 1

4x2

Exercise 8.4.19 (Solution on p. 963.)

y = −4x2

Exercise 8.4.20x = 1

8y2

Exercise 8.4.21 (Solution on p. 963.)

x = 36y2

Exercise 8.4.22x = 1

36y2

Available for free at Connexions <http://legacy.cnx.org/content/col11759/1.2>

888 CHAPTER 8. ANALYTIC GEOMETRY

Exercise 8.4.23 (Solution on p. 963.)

(x− 1)2 = 4 (y − 1)Exercise 8.4.24(y − 2)2 = 4

5 (x+ 4)Exercise 8.4.25 (Solution on p. 963.)

(y − 4)2 = 2 (x+ 3)Exercise 8.4.26(x+ 1)2 = 2 (y + 4)Exercise 8.4.27 (Solution on p. 963.)

(x+ 4)2 = 24 (y + 1)Exercise 8.4.28(y + 4)2 = 16 (x+ 4)Exercise 8.4.29 (Solution on p. 963.)

y2 + 12x− 6y + 21 = 0Exercise 8.4.30x2 − 4x− 24y + 28 = 0Exercise 8.4.31 (Solution on p. 963.)

5x2 − 50x− 4y + 113 = 0Exercise 8.4.32y2 − 24x+ 4y − 68 = 0Exercise 8.4.33 (Solution on p. 964.)

x2 − 4x+ 2y − 6 = 0Exercise 8.4.34y2 − 6y + 12x− 3 = 0Exercise 8.4.35 (Solution on p. 964.)

3y2 − 4x− 6y + 23 = 0Exercise 8.4.36x2 + 4x+ 8y − 4 = 0

8.4.7.3 Graphical

For the following exercises, graph the parabola, labeling the focus and the directrix.

Exercise 8.4.37 (Solution on p. 964.)

x = 18y

2

Exercise 8.4.38y = 36x2

Exercise 8.4.39 (Solution on p. 964.)

y = 136x

2

Exercise 8.4.40y = −9x2

Exercise 8.4.41 (Solution on p. 965.)

(y − 2)2 = − 43 (x+ 2)

Exercise 8.4.42−5(x+ 5)2 = 4 (y + 5)

Available for free at Connexions <http://legacy.cnx.org/content/col11759/1.2>

889

Exercise 8.4.43 (Solution on p. 965.)

−6(y + 5)2 = 4 (x− 4)Exercise 8.4.44y2 − 6y − 8x+ 1 = 0Exercise 8.4.45 (Solution on p. 966.)

x2 + 8x+ 4y + 20 = 0Exercise 8.4.463x2 + 30x− 4y + 95 = 0Exercise 8.4.47 (Solution on p. 967.)

y2 − 8x+ 10y + 9 = 0Exercise 8.4.48x2 + 4x+ 2y + 2 = 0Exercise 8.4.49 (Solution on p. 968.)

y2 + 2y − 12x+ 61 = 0Exercise 8.4.50−2x2 + 8x− 4y − 24 = 0

For the following exercises, �nd the equation of the parabola given information about its graph.

Exercise 8.4.51 (Solution on p. 969.)

Vertex is (0, 0) ;directrix is y = 4, focus is (0,−4) .Exercise 8.4.52Vertex is (0, 0) ; directrix isx = 4, focus is (−4, 0) .Exercise 8.4.53 (Solution on p. 969.)

Vertex is (2, 2) ; directrix isx = 2−√

2, focus is(2 +√

2, 2).

Exercise 8.4.54Vertex is (−2, 3) ; directrix isx = − 7

2 , focus is(− 1

2 , 3).

Exercise 8.4.55 (Solution on p. 969.)

Vertex is(√

2,−√

3)

; directrix isx = 2√

2, focus is(0,−√

3).

Exercise 8.4.56Vertex is (1, 2) ; directrix is y = 11

3 , focus is(1, 1

3

).

For the following exercises, determine the equation for the parabola from its graph.

Available for free at Connexions <http://legacy.cnx.org/content/col11759/1.2>

890 CHAPTER 8. ANALYTIC GEOMETRY

Exercise 8.4.57 (Solution on p. 969.)

Available for free at Connexions <http://legacy.cnx.org/content/col11759/1.2>

891

Exercise 8.4.58

Available for free at Connexions <http://legacy.cnx.org/content/col11759/1.2>

892 CHAPTER 8. ANALYTIC GEOMETRY

Exercise 8.4.59 (Solution on p. 969.)

Available for free at Connexions <http://legacy.cnx.org/content/col11759/1.2>

893

Exercise 8.4.60

Available for free at Connexions <http://legacy.cnx.org/content/col11759/1.2>

894 CHAPTER 8. ANALYTIC GEOMETRY

Exercise 8.4.61 (Solution on p. 969.)

8.4.7.4 Extensions

For the following exercises, the vertex and endpoints of the latus rectum of a parabola are given. Find theequation.

Exercise 8.4.62V (0, 0) , Endpoints (2, 1) , (−2, 1)Exercise 8.4.63 (Solution on p. 969.)

V (0, 0) , Endpoints (−2, 4) , (−2,−4)Exercise 8.4.64V (1, 2) , Endpoints (−5, 5) , (7, 5)Exercise 8.4.65 (Solution on p. 969.)

V (−3,−1) ,Endpoints (0, 5) , (0,−7)

Available for free at Connexions <http://legacy.cnx.org/content/col11759/1.2>

895

Exercise 8.4.66V (4,−3) , Endpoints

(5,− 7

2

),(3,− 7

2

)

8.4.7.5 Real-World Applications

Exercise 8.4.67 (Solution on p. 969.)

The mirror in an automobile headlight has a parabolic cross-section with the light bulb at the focus.On a schematic, the equation of the parabola is given asx2 = 4y.At what coordinates should youplace the light bulb?

Exercise 8.4.68If we want to construct the mirror from the previous exercise such that the focus is locatedat (0, 0.25) , what should the equation of the parabola be?

Exercise 8.4.69 (Solution on p. 970.)

A satellite dish is shaped like a paraboloid of revolution. This means that it can be formed byrotating a parabola around its axis of symmetry. The receiver is to be located at the focus. Ifthe dish is 12 feet across at its opening and 4 feet deep at its center, where should the receiver beplaced?

Exercise 8.4.70Consider the satellite dish from the previous exercise. If the dish is 8 feet across at the openingand 2 feet deep, where should we place the receiver?

Exercise 8.4.71 (Solution on p. 970.)

A searchlight is shaped like a paraboloid of revolution. A light source is located 1 foot from thebase along the axis of symmetry. If the opening of the searchlight is 3 feet across, �nd the depth.

Exercise 8.4.72If the searchlight from the previous exercise has the light source located 6 inches from the basealong the axis of symmetry and the opening is 4 feet, �nd the depth.

Exercise 8.4.73 (Solution on p. 970.)

An arch is in the shape of a parabola. It has a span of 100 feet and a maximum height of 20 feet.Find the equation of the parabola, and determine the height of the arch 40 feet from the center.

Exercise 8.4.74If the arch from the previous exercise has a span of 160 feet and a maximum height of 40 feet, �ndthe equation of the parabola, and determine the distance from the center at which the height is 20feet.

Exercise 8.4.75 (Solution on p. 970.)

An object is projected so as to follow a parabolic path given by y = −x2 + 96x,wherex is thehorizontal distance traveled in feet and y is the height. Determine the maximum height the objectreaches.

Exercise 8.4.76For the object from the previous exercise, assume the path followed is given by y = −0.5x2 + 80x.Determine how far along the horizontal the object traveled to reach maximum height.

Available for free at Connexions <http://legacy.cnx.org/content/col11759/1.2>

896 CHAPTER 8. ANALYTIC GEOMETRY

8.5 Rotation of Axes18

As we have seen, conic sections are formed when a plane intersects two right circular cones aligned tip totip and extending in�nitely far in opposite directions, which we also call a cone. The way in which we slicethe cone will determine the type of conic section formed at the intersection. A circle is formed by slicing acone with a plane perpendicular to the axis of symmetry of the cone. An ellipse is formed by slicing a singlecone with a slanted plane not perpendicular to the axis of symmetry. A parabola is formed by slicing theplane through the top or bottom of the double-cone, whereas a hyperbola is formed when the plane slicesboth the top and bottom of the cone. See Figure 8.40.

Figure 8.40: The nondegenerate conic sections

Ellipses, circles, hyperbolas, and parabolas are sometimes called the nondegenerate conic sections,in contrast to the degenerate conic sections, which are shown in Figure 8.41. A degenerate conic resultswhen a plane intersects the double cone and passes through the apex. Depending on the angle of the plane,three types of degenerate conic sections are possible: a point, a line, or two intersecting lines.

18This content is available online at <http://legacy.cnx.org/content/m49441/1.3/>.

Available for free at Connexions <http://legacy.cnx.org/content/col11759/1.2>

897

Figure 8.41: Degenerate conic sections

8.5.1 Identifying Nondegenerate Conics in General Form

In previous sections of this chapter, we have focused on the standard form equations for nondegenerate conicsections. In this section, we will shift our focus to the general form equation, which can be used for anyconic. The general form is set equal to zero, and the terms and coe�cients are given in a particular order,as shown below.

Ax2 +Bxy + Cy2 +Dx+ Ey + F = 0 (8.60)

whereA,B, and C are not all zero. We can use the values of the coe�cients to identify which type conic isrepresented by a given equation.

You may notice that the general form equation has anxy term that we have not seen in any of thestandard form equations. As we will discuss later, thexy term rotates the conic whenever B is not equal tozero.

Available for free at Connexions <http://legacy.cnx.org/content/col11759/1.2>

898 CHAPTER 8. ANALYTIC GEOMETRY

Conic Sections Example

ellipse 4x2 + 9y2 = 1

circle 4x2 + 4y2 = 1

hyperbola 4x2 − 9y2 = 1

parabola 4x2 = 9y or 4y2 = 9x

one line 4x+ 9y = 1

intersecting lines (x− 4) (y + 4) = 0

parallel lines (x− 4) (x− 9) = 0

a point 4x2 + 4y2 = 0

no graph 4x2 + 4y2 = − 1

Table 8.6

A General Note: A nondegenerate conic section has the general form

Ax2 +Bxy + Cy2 +Dx+ Ey + F = 0 (8.61)

whereA,B, andC are not all zero.

Table 8.7 summarizes the di�erent conic sections whereB = 0, and A and C are nonzero realnumbers. This indicates that the conic has not been rotated.

ellipseAx2 + Cy2 +Dx+Ey + F =0, A 6=C and AC > 0

circle Ax2 + Cy2 +Dx+Ey + F =0, A = C

hyperbolaAx2 − Cy2 +Dx+Ey + F =0 or − Ax2 +Cy2 + Dx +Ey + F =0,whereA andC arepositive

continued on next

page

Available for free at Connexions <http://legacy.cnx.org/content/col11759/1.2>

899

parabolaAx2+Dx+Ey+F = 0 or Cy2 +Dx+Ey+F = 0

Table 8.7

How To: Given the equation of a conic, identify the type of conic.

1.Rewrite the equation in the general form, Ax2 +Bxy + Cy2 +Dx+ Ey + F = 0.2.Identify the values ofA andC from the general form.

a.IfA andC are nonzero, have the same sign, and are not equal to each other, then the graphis an ellipse.

b.IfA andC are equal and nonzero and have the same sign, then the graph is a circle.c.IfA andC are nonzero and have opposite signs, then the graph is a hyperbola.d.If eitherA orC is zero, then the graph is a parabola.

Example 8.20Identifying a Conic from Its General FormIdentify the graph of each of the following nondegenerate conic sections.

a. 4x2 − 9y2 + 36x+ 36y − 125 = 0b. 9y2 + 16x+ 36y − 10 = 0c. 3x2 + 3y2 − 2x− 6y − 4 = 0d. −25x2 − 4y2 + 100x+ 16y + 20 = 0

Solution

a. Rewriting the general form, we have

A = 4 andC = −9, so we observe thatA andC have opposite signs. The graph of this equationis a hyperbola.

b. Rewriting the general form, we have

A = 0 andC = 9. We can determine that the equation is a parabola, sinceA is zero.c. Rewriting the general form, we have

Available for free at Connexions <http://legacy.cnx.org/content/col11759/1.2>

900 CHAPTER 8. ANALYTIC GEOMETRY

A = 3 andC = 3.BecauseA = C, the graph of this equation is a circle.d. Rewriting the general form, we have

A = −25 andC = −4.BecauseAC > 0 andA 6= C, the graph of this equation is an ellipse.

Try It:

Exercise 8.5.1 (Solution on p. 970.)

Identify the graph of each of the following nondegenerate conic sections.

a.16y2 − x2 + x− 4y − 9 = 0b.16x2 + 4y2 + 16x+ 49y − 81 = 0

8.5.1.1 Finding a New Representation of the Given Equation after Rotating through a GivenAngle

Until now, we have looked at equations of conic sections without anxy term, which aligns the graphs withthe x- and y-axes. When we add anxy term, we are rotating the conic about the origin. If the x- andy-axes are rotated through an angle, say θ,then every point on the plane may be thought of as having tworepresentations: (x, y) on the Cartesian plane with the original x-axis and y-axis, and (x′, y′) on the newplane de�ned by the new, rotated axes, called the x'-axis and y'-axis. See Figure 8.42.

Available for free at Connexions <http://legacy.cnx.org/content/col11759/1.2>

901

Figure 8.42: The graph of the rotated ellipsex2 + y2 −−xy −−15 = 0

We will �nd the relationships betweenx and y on the Cartesian plane withx′ and y′ on the new rotatedplane. See Figure 8.43.

Available for free at Connexions <http://legacy.cnx.org/content/col11759/1.2>

902 CHAPTER 8. ANALYTIC GEOMETRY

Figure 8.43: The Cartesian plane with x- and y-axes and the resulting x′− and y ′−axes formed by arotation by an angle θ.

The original coordinate x- and y-axes have unit vectors i and j .The rotated coordinate axes have unitvectors i′ and j′.The angle θ is known as the angle of rotation. See Figure 8.44. We may write the new unitvectors in terms of the original ones.

i′ = cos θi+ sin θj

j′ = −sin θi+ cos θj(8.62)

Available for free at Connexions <http://legacy.cnx.org/content/col11759/1.2>

903

Figure 8.44: Relationship between the old and new coordinate planes.

Consider a vector u in the new coordinate plane. It may be represented in terms of its coordinate axes.

u = x′i′ + y′j′

u = x′ (i cos θ + j sin θ) + y′ (−i sin θ + j cos θ) Substitute.

u = ix' cos θ + jx' sin θ − iy' sin θ + jy' cos θ Distribute.

u = ix' cos θ − iy' sin θ + jx' sin θ + jy' cos θ Apply commutative property.

u = (x' cos θ − y' sin θ) i+ (x' sin θ + y' cos θ) j Factor by grouping.

(8.63)

Becauseu = x′i′ + y′j′, we have representations ofx and y in terms of the new coordinate system.

x = x′cos θ − y′sin θ

and

y = x′sin θ + y′cos θ

(8.64)

Available for free at Connexions <http://legacy.cnx.org/content/col11759/1.2>

904 CHAPTER 8. ANALYTIC GEOMETRY

A General Note: If a point (x, y) on the Cartesian plane is represented on a new coordinateplane where the axes of rotation are formed by rotating an angle θ from the positive x-axis, thenthe coordinates of the point with respect to the new axes are (x′, y′) .We can use the followingequations of rotation to de�ne the relationship between (x, y) and (x′, y′) :

x = x′cos θ − y′sin θ (8.65)

and

y = x′sin θ + y′cos θ (8.66)

How To: Given the equation of a conic, �nd a new representation after rotatingthrough an angle.

1.Findx and ywherex = x′cos θ − y′sin θ and y = x′sin θ + y′cos θ.2.Substitute the expression forx and y into in the given equation, then simplify.3.Write the equations withx′ and y′ in standard form.

Example 8.21Finding a New Representation of an Equation after Rotating through a Given AngleFind a new representation of the equation 2x2 − xy+ 2y2 − 30 = 0 after rotating through an angleof θ = 45 ◦.

SolutionFindx and y,wherex = x′cos θ − y′sin θ and y = x′sin θ + y′cos θ.

Because θ = 45 ◦,

x = x′cos (45 ◦)− y′sin (45 ◦)

x = x′(

1√2

)− y′

(1√2

)x = x′−y′√

2

(8.67)

and

y = x′sin (45 ◦) + y′cos (45 ◦)

y = x′(

1√2

)+ y′

(1√2

)y = x′+y′√

2

(8.68)

Substitutex = x′cosθ − y′sinθ and y = x′sin θ + y′cos θ into 2x2 − xy + 2y2 − 30 = 0.

2(x′ − y′√

2

)2

−(x′ − y′√

2

)(x′ + y′√

2

)+ 2(x′ + y′√

2

)2

− 30 = 0 (8.69)

Simplify.

Available for free at Connexions <http://legacy.cnx.org/content/col11759/1.2>

905

)2 (x′−y′)(x′−y′)

)2− (x′−y′)(x′+y′)

2+ )2 (x′+y′)(x′+y′)

)2− 30 = 0 FOIL method

x′2)− 2x′y′+ y′2 − (x′2−y′2)

2+ x′2) + 2x′y′ + y′2 − 30 = 0 Combine like terms.

2x′2 + 2y′2 − (x′2−y′2)2

= 30 Combine like terms.

2

(2x′2 + 2y′2 − (x′2−y′2)

2

)= 2 (30) Multiply both sides by 2.

4x′2 + 4y′2 − (x′2 − y′2) = 60 Simplify.

4x′2 + 4y′2 − x′2 + y′2 = 60 Distribute.

3x′2

60+ 5y′2

60= 60

60Set equal to 1.

(8.70)

Write the equations withx′ and y′ in the standard form.

x′2

20+y′

2

12= 1 (8.71)

This equation is an ellipse. Figure 8.45 shows the graph.

Available for free at Connexions <http://legacy.cnx.org/content/col11759/1.2>

906 CHAPTER 8. ANALYTIC GEOMETRY

Figure 8.45

8.5.2 Writing Equations of Rotated Conics in Standard Form

Now that we can �nd the standard form of a conic when we are given an angle of rotation, we will learn howto transform the equation of a conic given in the formAx2 +Bxy+Cy2 +Dx+Ey+F = 0 into standard formby rotating the axes. To do so, we will rewrite the general form as an equation in thex′ and y′ coordinatesystem without thex′y′ term, by rotating the axes by a measure of θ that satis�es

cot (2θ) =A− CB

(8.72)

Available for free at Connexions <http://legacy.cnx.org/content/col11759/1.2>

907

We have learned already that any conic may be represented by the second degree equation

Ax2 +Bxy + Cy2 +Dx+ Ey + F = 0 (8.73)

whereA,B,andC are not all zero. However, ifB 6= 0, then we have anxy term that prevents us from rewritingthe equation in standard form. To eliminate it, we can rotate the axes by an acute angle θwhere cot (2θ) =A−CB .

• If cot (2θ) > 0, then 2θ is in the �rst quadrant, and θ is between (0 ◦, 45 ◦) .• If cot (2θ) < 0, then 2θ is in the second quadrant, and θ is between (45 ◦, 90 ◦) .• IfA = C, then θ = 45 ◦.

How To: Given an equation for a conic in the x′y′ system, rewrite the equation withoutthe x′y′ term in terms of x′ and y′,where the x′ and y′ axes are rotations of the standardaxes by θ degrees.

1.Find cot (2θ) .2.Find sin θ and cos θ.3.Substitute sin θ and cos θ intox = x′cos θ − y′sin θ and y = x′sin θ + y′cos θ.4.Substitute the expression forx and y into in the given equation, and then simplify.5.Write the equations withx′ and y′ in the standard form with respect to the rotated axes.

Example 8.22Rewriting an Equation with respect to the x′ and y′ axes without the x′y′ TermRewrite the equation 8x2 − 12xy + 17y2 = 20 in thex′y′ system without anx′y′ term.

SolutionFirst, we �nd cot (2θ) . See Figure 8.46.

8x2 − 12xy + 17y2 = 20⇒ A = 8, B = −12 andC = 17

cot (2θ) = A−CB = 8−17

−12

cot (2θ) = −9−12 = 3

4

(8.74)

Available for free at Connexions <http://legacy.cnx.org/content/col11759/1.2>

908 CHAPTER 8. ANALYTIC GEOMETRY

Figure 8.46

cot (2θ) =34

=adjacent

opposite(8.75)

So the hypotenuse is

32 + 42 = h2

9 + 16 = h2

25 = h2

h = 5

(8.76)

Next, we �nd sin θ and cos θ.

sin θ =√

1−cos(2θ)2 =

√1− 3

52 =

√55−

35

2 =√

5−35 ·

12 =

√210 =

√15

sin θ = 1√5

cos θ =√

1+cos(2θ)2 =

√1+ 3

52 =

√55 + 3

52 =

√5+3

5 ·12 =

√810 =

√45

cos θ = 2√5

(8.77)

Available for free at Connexions <http://legacy.cnx.org/content/col11759/1.2>

909

Substitute the values of sin θ and cos θ intox = x′cos θ − y′sin θ and y = x′sin θ + y′cos θ.

x = x′cos θ − y′sin θ

x = x′(

2√5

)− y′

(1√5

)x = 2x′−y′√

5

(8.78)

and

y = x′sin θ + y′cos θ

y = x′(

1√5

)+ y′

(2√5

)y = x′+2y′√

5

(8.79)

Substitute the expressions forx and y into in the given equation, and then simplify.

8(

2x′−y′√5

)2

− 12(

2x′−y′√5

)(x′+2y′√

5

)+ 17

(x′+2y′√

5

)2

= 20

8(

(2x′−y′)(2x′−y′)5

)− 12

((2x′−y′)(x′+2y′)

5

)+ 17

((x′+2y′)(x′+2y′)

5

)= 20

8 (4x′2 − 4x′y′ + y′2)− 12 (2x′2 + 3x′y′ − 2y′2) + 17 (x′2 + 4x′y′ + 4y′2) = 100

32x′2 − 32x′y′ + 8y′2 − 24x′2 − 36x′y′ + 24y′2 + 17x′2 + 68x′y′ + 68y′2 = 100

25x′2 + 100y′2 = 10025100

x′2 + 100100

y′2 = 100100

(8.80)

Write the equations withx′ and y′ in the standard form with respect to the new coordinatesystem.

x′2

4+y′

2

1= 1 (8.81)

Figure 8.47 shows the graph of the ellipse.

Available for free at Connexions <http://legacy.cnx.org/content/col11759/1.2>

910 CHAPTER 8. ANALYTIC GEOMETRY

Figure 8.47

Try It:

Exercise 8.5.2 (Solution on p. 970.)

Rewrite the 13x2 − 6√

3xy + 7y2 = 16 in thex′y′ system without thex′y′ term.

Example 8.23Graphing an Equation That Has No x′y′ TermsGraph the following equation relative to thex′y′ system:

x2 + 12xy − 4y2 = 30 (8.82)

Solution

First, we �nd cot (2θ) .

x2 + 12xy − 4y2 = 20⇒ A = 1, B = 12, and C = −4 (8.83)

cot (2θ) = A−CB

cot (2θ) = 1−(−4)12

cot (2θ) = 512

(8.84)

Because cot (2θ) = 512 , we can draw a reference triangle as in Figure 8.48.

Available for free at Connexions <http://legacy.cnx.org/content/col11759/1.2>

911

Figure 8.48

Available for free at Connexions <http://legacy.cnx.org/content/col11759/1.2>

912 CHAPTER 8. ANALYTIC GEOMETRY

cot (2θ) =512

=adjacent

opposite(8.85)

Thus, the hypotenuse is

52 + 122 = h2

25 + 144 = h2

169 = h2

h = 13

(8.86)

Next, we �nd sin θ and cos θ.We will use half-angle identities.

sin θ =√

1−cos(2θ)2 =

√1− 5

132 =

√1313−

513

2 =√

813 ·

12 = 2√

13

cos θ =√

1+cos(2θ)2 =

√1+ 5

132 =

√1313 + 5

132 =

√1813 ·

12 = 3√

13

(8.87)

Now we �ndx and y.

x = x′cos θ − y′sin θ

x = x′(

3√13

)− y′

(2√13

)x = 3x′−2y′√

13

(8.88)

and

y = x′sin θ + y′cos θ

y = x′(

2√13

)+ y′

(3√13

)y = 2x′+3y′√

13

(8.89)

Now we substitutex = 3x′−2y′√13

and y = 2x′+3y′√13

intox2 + 12xy − 4y2 = 30.

(3x′−2y′√

13

)2

+ 12(

3x′−2y′√13

)(2x′+3y′√

13

)− 4(

2x′+3y′√13

)2

= 30(113

) [(3x′ − 2y′)2 + 12 (3x′ − 2y′) (2x′ + 3y′)− 4(2x′ + 3y′)2] = 30 Factor.(

113

)[9x′2 − 12x′y′ + 4y′2 + 12 (6x′2 + 5x′y′ − 6y′2)− 4 (4x′2 + 12x′y′ + 9y′2)] = 30 Multiply.(

113

)[9x′2 − 12x′y′ + 4y′2 + 72x′2 + 60x′y′ − 72y′2 − 16x′2 − 48x′y′ − 36y′2] = 30 Distribute.(

113

)[65x′2 − 104y′2] = 30 Combine like terms.

65x′2 − 104y′2 = 390 Multiply.x′2

6− 4y′2

15= 1 Divide by 390.

(8.90)

Available for free at Connexions <http://legacy.cnx.org/content/col11759/1.2>

913

Figure 8.49 shows the graph of the hyperbola x′2

6 −4y′2

15 = 1.

Figure 8.49

8.5.3 Identifying Conics without Rotating Axes

Now we have come full circle. How do we identify the type of conic described by an equation? What happenswhen the axes are rotated? Recall, the general form of a conic is

Ax2 +Bxy + Cy2 +Dx+ Ey + F = 0 (8.91)

Available for free at Connexions <http://legacy.cnx.org/content/col11759/1.2>

914 CHAPTER 8. ANALYTIC GEOMETRY

If we apply the rotation formulas to this equation we get the form

A′x′2 +B′x′y′ + C ′y′

2 +D′x′ + E′y′ + F ′ = 0 (8.92)

It may be shown thatB2 − 4AC = B′2 − 4A′C ′.The expression does not vary after rotation, so we call

the expression invariant. The discriminant,B2 − 4AC, is invariant and remains unchanged after rotation.Because the discriminant remains unchanged, observing the discriminant enables us to identify the conicsection.

A General Note: If the equationAx2 +Bxy+Cy2 +Dx+Ey+F = 0 is transformed by rotatingaxes into the equationA′x′

2 +B′x′y′+C ′y′2 +D′x′+E′y′+F ′ = 0, thenB2−4AC = B′

2−4A′C ′.

The equationAx2 + Bxy + Cy2 +Dx+ Ey + F = 0 is an ellipse, a parabola, or a hyperbola, or adegenerate case of one of these.

If the discriminant,B2 − 4AC,is

• < 0, the conic section is an ellipse• = 0, the conic section is a parabola• > 0, the conic section is a hyperbola

Example 8.24Identifying the Conic without Rotating AxesIdentify the conic for each of the following without rotating axes.

a. 5x2 + 2√

3xy + 2y2 − 5 = 0b. 5x2 + 2

√3xy + 12y2 − 5 = 0

Solution

a. Let's begin by determiningA,B, andC.

5︸︷︷︸A

x2 + 2√

3︸︷︷︸B

xy + 2︸︷︷︸C

y2 − 5 = 0 (8.93)

Now, we �nd the discriminant.

B2 − 4AC =(2√

3)2 − 4 (5) (2)

= 4 (3)− 40

= 12− 40

= −28 < 0

(8.94)

Therefore, 5x2 + 2√

3xy + 2y2 − 5 = 0 represents an ellipse.b. Again, let's begin by determiningA,B, andC.

5︸︷︷︸A

x2 + 2√

3︸︷︷︸B

xy + 12︸︷︷︸C

y2 − 5 = 0 (8.95)

Now, we �nd the discriminant.

B2 − 4AC =(2√

3)2 − 4 (5) (12)

= 4 (3)− 240

= 12− 240

= −228 < 0

(8.96)

Available for free at Connexions <http://legacy.cnx.org/content/col11759/1.2>

915

Therefore, 5x2 + 2√

3xy + 12y2 − 5 = 0 represents an ellipse.

Try It:

Exercise 8.5.3 (Solution on p. 970.)

Identify the conic for each of the following without rotating axes.

a.x2 − 9xy + 3y2 − 12 = 0b.10x2 − 9xy + 4y2 − 4 = 0

Media: Access this online resource for additional instruction and practice with conic sections androtation of axes.

• Introduction to Conic Sections19

8.5.4 Key Equations

General Form equation of a conic section Ax2 +Bxy + Cy2 +Dx+ Ey + F = 0

Rotation of a conic sectionx = x′cos θ − y′sin θ

y = x′sin θ + y′cos θ

Angle of rotation θ,where cot (2θ) = A−CB

Table 8.8

8.5.5 Key Concepts

• Four basic shapes can result from the intersection of a plane with a pair of right circular cones connectedtail to tail. They include an ellipse, a circle, a hyperbola, and a parabola.

• A nondegenerate conic section has the general formAx2 + Bxy + Cy2 + Dx + Ey + F =0whereA,B andC are not all zero. The values ofA,B, andC determine the type of conic. See Ex-ample 8.20.

• Equations of conic sections with anxy term have been rotated about the origin. See Example 8.21.• The general form can be transformed into an equation in thex′ and y′ coordinate system without

thex′y′ term. See Example 8.22 and Example 8.23.• An expression is described as invariant if it remains unchanged after rotating. Because the discriminant

is invariant, observing it enables us to identify the conic section. See Example 8.24.

19http://openstaxcollege.org/l/introconic

Available for free at Connexions <http://legacy.cnx.org/content/col11759/1.2>

916 CHAPTER 8. ANALYTIC GEOMETRY

8.5.6 Section Exercises

8.5.6.1 Verbal

Exercise 8.5.4 (Solution on p. 970.)

What e�ect does thexy term have on the graph of a conic section?

Exercise 8.5.5If the equation of a conic section is written in the formAx2 +By2 +Cx+Dy+E = 0 andAB = 0,what can we conclude?

Exercise 8.5.6 (Solution on p. 970.)

If the equation of a conic section is written in the formAx2 + Bxy + Cy2 + Dx + Ey + F =0,andB2 − 4AC > 0, what can we conclude?

Exercise 8.5.7Given the equation ax2 + 4x+ 3y2 − 12 = 0, what can we conclude if a > 0?Exercise 8.5.8 (Solution on p. 970.)

For the equationAx2+Bxy+Cy2+Dx+Ey+F = 0, the value of θ that satis�es cot (2θ) = A−CB gives

us what information?

8.5.6.2 Algebraic

For the following exercises, determine which conic section is represented based on the given equation.

Exercise 8.5.99x2 + 4y2 + 72x+ 36y − 500 = 0Exercise 8.5.10 (Solution on p. 970.)

x2 − 10x+ 4y − 10 = 0Exercise 8.5.112x2 − 2y2 + 4x− 6y − 2 = 0Exercise 8.5.12 (Solution on p. 970.)

4x2 − y2 + 8x− 1 = 0Exercise 8.5.134y2 − 5x+ 9y + 1 = 0Exercise 8.5.14 (Solution on p. 970.)

2x2 + 3y2 − 8x− 12y + 2 = 0Exercise 8.5.154x2 + 9xy + 4y2 − 36y − 125 = 0Exercise 8.5.16 (Solution on p. 970.)

3x2 + 6xy + 3y2 − 36y − 125 = 0Exercise 8.5.17−3x2 + 3

√3xy − 4y2 + 9 = 0

Exercise 8.5.18 (Solution on p. 970.)

2x2 + 4√

3xy + 6y2 − 6x− 3 = 0Exercise 8.5.19−x2 + 4

√2xy + 2y2 − 2y + 1 = 0

Exercise 8.5.20 (Solution on p. 970.)

8x2 + 4√

2xy + 4y2 − 10x+ 1 = 0For the following exercises, �nd a new representation of the given equation after rotating through the givenangle.

Available for free at Connexions <http://legacy.cnx.org/content/col11759/1.2>

917

Exercise 8.5.213x2 + xy + 3y2 − 5 = 0, θ = 45 ◦

Exercise 8.5.22 (Solution on p. 970.)

4x2 − xy + 4y2 − 2 = 0, θ = 45 ◦

Exercise 8.5.232x2 + 8xy − 1 = 0, θ = 30 ◦

Exercise 8.5.24 (Solution on p. 970.)

−2x2 + 8xy + 1 = 0, θ = 45 ◦

Exercise 8.5.254x2 +

√2xy + 4y2 + y + 2 = 0, θ = 45 ◦

For the following exercises, determine the angle θ that will eliminate thexy term and write the correspondingequation without thexy term.

Exercise 8.5.26 (Solution on p. 970.)

x2 + 3√

3xy + 4y2 + y − 2 = 0Exercise 8.5.274x2 + 2

√3xy + 6y2 + y − 2 = 0

Exercise 8.5.28 (Solution on p. 970.)

9x2 − 3√

3xy + 6y2 + 4y − 3 = 0Exercise 8.5.29−3x2 −

√3xy − 2y2 − x = 0

Exercise 8.5.30 (Solution on p. 970.)

16x2 + 24xy + 9y2 + 6x− 6y + 2 = 0Exercise 8.5.31x2 + 4xy + 4y2 + 3x− 2 = 0Exercise 8.5.32 (Solution on p. 970.)

x2 + 4xy + y2 − 2x+ 1 = 0Exercise 8.5.334x2 − 2

√3xy + 6y2 − 1 = 0

8.5.6.3 Graphical

For the following exercises, rotate through the given angle based on the given equation. Give the newequation and graph the original and rotated equation.

Exercise 8.5.34 (Solution on p. 970.)

y = −x2, θ = −45◦

Exercise 8.5.35x = y2, θ = 45◦

Exercise 8.5.36 (Solution on p. 971.)x2

4 + y2

1 = 1, θ = 45◦

Exercise 8.5.37y2

16 + x2

9 = 1, θ = 45◦

Exercise 8.5.38 (Solution on p. 971.)

y2 − x2 = 1, θ = 45◦

Exercise 8.5.39y = x2

2 , θ = 30◦

Available for free at Connexions <http://legacy.cnx.org/content/col11759/1.2>

918 CHAPTER 8. ANALYTIC GEOMETRY

Exercise 8.5.40 (Solution on p. 972.)

x = (y − 1)2, θ = 30◦

Exercise 8.5.41x2

9 + y2

4 = 1, θ = 30◦

For the following exercises, graph the equation relative to thex′y′ system in which the equation hasnox′y′ term.

Exercise 8.5.42 (Solution on p. 972.)

xy = 9Exercise 8.5.43x2 + 10xy + y2 − 6 = 0Exercise 8.5.44 (Solution on p. 973.)

x2 − 10xy + y2 − 24 = 0Exercise 8.5.454x2 − 3

√3xy + y2 − 22 = 0

Exercise 8.5.46 (Solution on p. 974.)

6x2 + 2√

3xy + 4y2 − 21 = 0Exercise 8.5.4711x2 + 10

√3xy + y2 − 64 = 0

Exercise 8.5.48 (Solution on p. 975.)

21x2 + 2√

3xy + 19y2 − 18 = 0Exercise 8.5.4916x2 + 24xy + 9y2 − 130x+ 90y = 0Exercise 8.5.50 (Solution on p. 976.)

16x2 + 24xy + 9y2 − 60x+ 80y = 0Exercise 8.5.5113x2 − 6

√3xy + 7y2 − 16 = 0

Exercise 8.5.52 (Solution on p. 976.)

4x2 − 4xy + y2 − 8√

5x− 16√

5y = 0For the following exercises, determine the angle of rotation in order to eliminate thexy term. Then graphthe new set of axes.

Exercise 8.5.536x2 − 5

√3xy + y2 + 10x− 12y = 0

Exercise 8.5.54 (Solution on p. 977.)

6x2 − 5xy + 6y2 + 20x− y = 0Exercise 8.5.556x2 − 8

√3xy + 14y2 + 10x− 3y = 0

Exercise 8.5.56 (Solution on p. 977.)

4x2 + 6√

3xy + 10y2 + 20x− 40y = 0Exercise 8.5.578x2 + 3xy + 4y2 + 2x− 4 = 0Exercise 8.5.58 (Solution on p. 978.)

16x2 + 24xy + 9y2 + 20x− 44y = 0For the following exercises, determine the value of k based on the given equation.

Exercise 8.5.59Given 4x2 + kxy + 16y2 + 8x+ 24y − 48 = 0, �nd k for the graph to be a parabola.

Available for free at Connexions <http://legacy.cnx.org/content/col11759/1.2>

919

Exercise 8.5.60 (Solution on p. 978.)

Given 2x2 + kxy + 12y2 + 10x− 16y + 28 = 0, �nd k for the graph to be an ellipse.

Exercise 8.5.61Given 3x2 + kxy + 4y2 − 6x+ 20y + 128 = 0, �nd k for the graph to be a hyperbola.

Exercise 8.5.62 (Solution on p. 978.)

Given kx2 + 8xy + 8y2 − 12x+ 16y + 18 = 0, �nd k for the graph to be a parabola.

Exercise 8.5.63Given 6x2 + 12xy + ky2 + 16x+ 10y + 4 = 0, �nd k for the graph to be an ellipse.

8.6 Conic Sections in Polar Coordinates20

Figure 8.50: Planets orbiting the sun follow elliptical paths. (credit: NASA Blueshift, Flickr)

Most of us are familiar with orbital motion, such as the motion of a planet around the sun or an electronaround an atomic nucleus. Within the planetary system, orbits of planets, asteroids, and comets arounda larger celestial body are often elliptical. Comets, however, may take on a parabolic or hyperbolic orbitinstead. And, in reality, the characteristics of the planets' orbits may vary over time. Each orbit is tied tothe location of the celestial body being orbited and the distance and direction of the planet or other objectfrom that body. As a result, we tend to use polar coordinates to represent these orbits.

In an elliptical orbit, the periapsis is the point at which the two objects are closest, and the apoapsis isthe point at which they are farthest apart. Generally, the velocity of the orbiting body tends to increase as itapproaches the periapsis and decrease as it approaches the apoapsis. Some objects reach an escape velocity,which results in an in�nite orbit. These bodies exhibit either a parabolic or a hyperbolic orbit about a body;the orbiting body breaks free of the celestial body's gravitational pull and �res o� into space. Each of theseorbits can be modeled by a conic section in the polar coordinate system.

20This content is available online at <http://legacy.cnx.org/content/m49442/1.4/>.

Available for free at Connexions <http://legacy.cnx.org/content/col11759/1.2>

920 CHAPTER 8. ANALYTIC GEOMETRY

8.6.1 Identifying a Conic in Polar Form

Any conic may be determined by three characteristics: a single focus, a �xed line called the directrix,and the ratio of the distances of each to a point on the graph. Consider the parabolax = 2 + y2 shown inFigure 8.51.

Figure 8.51

In The Parabola (Section 8.4), we learned how a parabola is de�ned by the focus (a �xed point) and thedirectrix (a �xed line). In this section, we will learn how to de�ne any conic in the polar coordinate systemin terms of a �xed point, the focusP (r, θ) at the pole, and a line, the directrix, which is perpendicular tothe polar axis.

IfF is a �xed point, the focus, andD is a �xed line, the directrix, then we can let e be a �xed positivenumber, called the eccentricity, which we can de�ne as the ratio of the distances from a point on the graphto the focus and the point on the graph to the directrix. Then the set of all pointsP such that e = PF

PD is aconic. In other words, we can de�ne a conic as the set of all pointsP with the property that the ratio of thedistance fromP toF to the distance fromP toD is equal to the constant e.

For a conic with eccentricity e,

• if 0 ≤ e < 1, the conic is an ellipse• if e = 1, the conic is a parabola• if e > 1, the conic is an hyperbola

Available for free at Connexions <http://legacy.cnx.org/content/col11759/1.2>

921

With this de�nition, we may now de�ne a conic in terms of the directrix,x = ±p, the eccentricity e, and theangle θ. Thus, each conic may be written as a polar equation, an equation written in terms of r and θ.

A General Note: For a conic with a focus at the origin, if the directrix isx = ±p, where p isa positive real number, and the eccentricity is a positive real number e, the conic has a polarequation

r =ep

1± e cos θ(8.97)

For a conic with a focus at the origin, if the directrix is y = ±p, where p is a positive real number,and the eccentricity is a positive real number e, the conic has a polar equation

r =ep

1± e sin θ(8.98)

How To: Given the polar equation for a conic, identify the type of conic, the directrix,and the eccentricity.

1.Multiply the numerator and denominator by the reciprocal of the constant in the denominatorto rewrite the equation in standard form.

2.Identify the eccentricity e as the coe�cient of the trigonometric function in the denominator.3.Compare ewith 1 to determine the shape of the conic.4.Determine the directrix asx = p if cosine is in the denominator and y = p if sine is in thedenominator. Set ep equal to the numerator in standard form to solve forx or y.

Example 8.25Identifying a Conic Given the Polar FormFor each of the following equations, identify the conic with focus at the origin, the directrix, andthe eccentricity.

a. r = 63+2 sin θ

b. r = 124+5 cos θ

c. r = 72−2 sin θ

SolutionFor each of the three conics, we will rewrite the equation in standard form. Standard form has a 1 asthe constant in the denominator. Therefore, in all three parts, the �rst step will be to multiply thenumerator and denominator by the reciprocal of the constant of the original equation, 1

c , where c isthat constant.

a. Multiply the numerator and denominator by 13 .

r =6

3 + 2sin θ·(

13

)(13

) =6(

13

)3(

13

)+ 2

(13

)sin θ

=2

1 + 23 sin θ

(8.99)

Becausesin θ is in the denominator, the directrix is y = p.Comparing to standard form, notethat e = 2

3 .Therefore, from the numerator,

2 = ep

2 = 23p(

32

)2 =

(32

)23p

3 = p

(8.100)

Available for free at Connexions <http://legacy.cnx.org/content/col11759/1.2>

922 CHAPTER 8. ANALYTIC GEOMETRY

Since e < 1, the conic is an ellipse. The eccentricity is e = 23and the directrix is y = 3.

b. Multiply the numerator and denominator by 14 .

r = 124+5 cos θ ·

( 14 )

( 14 )

r =12( 1

4 )4( 1

4 )+5( 14 )cos θ

r = 31+ 5

4 cos θ

(8.101)

Because cos θ is in the denominator, the directrix isx = p.Comparing to standard form, e =54 .Therefore, from the numerator,

3 = ep

3 = 54p(

45

)3 =

(45

)54p

125 = p

(8.102)

Since e > 1, the conic is a hyperbola. The eccentricity is e = 54 and the directrix isx = 12

5 =2.4.

c. Multiply the numerator and denominator by 12 .

r = 72−2 sin θ ·

( 12 )

( 12 )

r =7( 1

2 )2( 1

2 )−2( 12 ) sin θ

r =72

1−sin θ

(8.103)

Because sine is in the denominator, the directrix is y = −p.Comparing to standard form, e =1.Therefore, from the numerator,

72 = ep

72 = (1) p72 = p

(8.104)

Because e = 1, the conic is a parabola. The eccentricity is e = 1 and the directrix is y =− 7

2 = −3.5.

Try It:

Exercise 8.6.1 (Solution on p. 978.)

Identify the conic with focus at the origin, the directrix, and the eccentricity for r =2

3−cos θ .

Available for free at Connexions <http://legacy.cnx.org/content/col11759/1.2>

923

8.6.2 Graphing the Polar Equations of Conics

When graphing in Cartesian coordinates, each conic section has a unique equation. This is not the casewhen graphing in polar coordinates. We must use the eccentricity of a conic section to determine which typeof curve to graph, and then determine its speci�c characteristics. The �rst step is to rewrite the conic instandard form as we have done in the previous example. In other words, we need to rewrite the equation sothat the denominator begins with 1. This enables us to determine e and, therefore, the shape of the curve.The next step is to substitute values for θ and solve for r to plot a few key points. Setting θ equal to 0, π2 , π,and 3π

2 provides the vertices so we can create a rough sketch of the graph.

Example 8.26Graphing a Parabola in Polar FormGraph r = 5

3+3 cos θ .

SolutionFirst, we rewrite the conic in standard form by multiplying the numerator and denominator bythe reciprocal of 3, which is 1

3 .

r = 53+3 cos θ =

5( 13 )

3( 13 )+3( 1

3 )cos θ

r =53

1+cos θ

(8.105)

Because e = 1,we will graph a parabola with a focus at the origin. The function has a cos θ, andthere is an addition sign in the denominator, so the directrix isx = p.

53 = ep

53 = (1) p53 = p

(8.106)

The directrix isx = 53 .

Plotting a few key points as in Table 8.9 will enable us to see the vertices. See Figure 8.52.

A B C D

θ 0 π2 π 3π

2

r = 53+3 cos θ

56 ≈ 0.83 5

3 ≈ 1.67 unde�ned 53 ≈ 1.67

Table 8.9

Available for free at Connexions <http://legacy.cnx.org/content/col11759/1.2>

924 CHAPTER 8. ANALYTIC GEOMETRY

Figure 8.52

AnalysisWe can check our result with a graphing utility. See Figure 8.53.

Available for free at Connexions <http://legacy.cnx.org/content/col11759/1.2>

925

Figure 8.53

Example 8.27Graphing a Hyperbola in Polar FormGraph r = 8

2−3 sin θ .

SolutionFirst, we rewrite the conic in standard form by multiplying the numerator and denominator bythe reciprocal of 2, which is 1

2 .

r = 82−3sin θ =

8( 12 )

2( 12 )−3( 1

2 )sin θ

r = 41− 3

2 sin θ

(8.107)

Available for free at Connexions <http://legacy.cnx.org/content/col11759/1.2>

926 CHAPTER 8. ANALYTIC GEOMETRY

Because e = 32 , e > 1, so we will graph a hyperbola with a focus at the origin. The function has

a sin θ term and there is a subtraction sign in the denominator, so the directrix is y = −p.

4 = ep

4 =(

32

)p

4(

23

)= p

83 = p

(8.108)

The directrix is y = − 83 .

Plotting a few key points as in Table 8.10 will enable us to see the vertices. See Figure 8.54.

A B C D

θ 0 π2 π 3π

2

r=8 2−3sin θ 4 −8 4 85 = 1.6

Table 8.10

Available for free at Connexions <http://legacy.cnx.org/content/col11759/1.2>

927

Figure 8.54

Example 8.28Graphing an Ellipse in Polar FormGraph r = 10

5−4 cos θ .

SolutionFirst, we rewrite the conic in standard form by multiplying the numerator and denominator bythe reciprocal of 5, which is 1

5 .

r = 105−4cos θ =

10( 15 )

5( 15 )−4( 1

5 )cos θ

r = 21− 4

5 cos θ

(8.109)

Available for free at Connexions <http://legacy.cnx.org/content/col11759/1.2>

928 CHAPTER 8. ANALYTIC GEOMETRY

Because e = 45 , e < 1, so we will graph an ellipse with a focus at the origin. The function has

a cos θ, and there is a subtraction sign in the denominator, so the directrix isx = −p.

2 = ep

2 =(

45

)p

2(

54

)= p

52 = p

(8.110)

The directrix isx = − 52 .

Plotting a few key points as in Table 8.11 will enable us to see the vertices. See Figure 8.55.

A B C D

θ 0 π2 π 3π

2

r = 105−4 cos θ 10 2 10

9 ≈ 1.1 2

Table 8.11

Available for free at Connexions <http://legacy.cnx.org/content/col11759/1.2>

929

Figure 8.55

AnalysisWe can check our result using a graphing utility. See Figure 8.56.

Available for free at Connexions <http://legacy.cnx.org/content/col11759/1.2>

930 CHAPTER 8. ANALYTIC GEOMETRY

Figure 8.56: r = 105−4 cos θ

graphed on a viewing windowof [−− 3, 12, 1] by [−− 4, 4, 1] , θmin = 0and θmax = 2π.

Try It:

Exercise 8.6.2 (Solution on p. 978.)

Graph r = 24−cos θ .

8.6.3 De�ning Conics in Terms of a Focus and a Directrix

So far we have been using polar equations of conics to describe and graph the curve. Now we will work inreverse; we will use information about the origin, eccentricity, and directrix to determine the polar equation.

Available for free at Connexions <http://legacy.cnx.org/content/col11759/1.2>

931

How To: Given the focus, eccentricity, and directrix of a conic, determine the polarequation.

1.Determine whether the directrix is horizontal or vertical. If the directrix is given in terms of y,we use the general polar form in terms of sine. If the directrix is given in terms ofx, we usethe general polar form in terms of cosine.

2.Determine the sign in the denominator. If p < 0, use subtraction. If p > 0, use addition.3.Write the coe�cient of the trigonometric function as the given eccentricity.4.Write the absolute value of p in the numerator, and simplify the equation.

Example 8.29Finding the Polar Form of a Vertical Conic Given a Focus at the Origin and the

Eccentricity and DirectrixFind the polar form of the conic given a focus at the origin, e = 3 and directrix y = −2.

SolutionThe directrix is y = −p, so we know the trigonometric function in the denominator is sine.

Because y = −2,−− 2 < 0, so we know there is a subtraction sign in the denominator. We usethe standard form of

r =ep

1− e sin θ(8.111)

and e = 3 and | − 2| = 2 = p.Therefore,

r = (3)(2)1−3 sin θ

r = 61−3 sin θ

(8.112)

Example 8.30Finding the Polar Form of a Horizontal Conic Given a Focus at the Origin and the

Eccentricity and DirectrixFind the polar form of a conic given a focus at the origin, e = 3

5 , and directrixx = 4.

SolutionBecause the directrix isx = p,we know the function in the denominator is cosine. Becausex =

4, 4 > 0,so we know there is an addition sign in the denominator. We use the standard form of

r =ep

1 + e cos θ(8.113)

and e = 35 and |4| = 4 = p.

Therefore,

Available for free at Connexions <http://legacy.cnx.org/content/col11759/1.2>

932 CHAPTER 8. ANALYTIC GEOMETRY

r = ( 35 )(4)

1+ 35 cos θ

r =125

1+ 35 cos θ

r =125

1( 55 )+ 3

5 cos θ

r =125

55 + 3

5 cos θ

r = 125 ·

55+3 cos θ

r = 125+3 cos θ

(8.114)

Try It:

Exercise 8.6.3 (Solution on p. 979.)

Find the polar form of the conic given a focus at the origin, e = 1, and directrixx = −1.

Example 8.31Converting a Conic in Polar Form to Rectangular FormConvert the conic r = 1

5−5sin θ to rectangular form.

SolutionWe will rearrange the formula to use the identities r =

√x2 + y2, x = r cos θ, and y = r sin θ.

r = 15−5 sin θ

r · (5− 5 sin θ) = 15−5 sin θ · (5− 5 sin θ) Eliminate the fraction.

5r − 5r sin θ = 1 Distribute.

5r = 1 + 5r sin θ Isolate 5r.

25r2 = (1 + 5r sin θ)2Square both sides.

25(x2 + y2

)= (1 + 5y)2

Substitute r =√x2 + y2 and y = r sin θ.

25x2 + 25y2 = 1 + 10y + 25y2 Distribute and use FOIL.

25x2 − 10y = 1 Rearrange terms and set equal to 1.

(8.115)

Try It:

Exercise 8.6.4 (Solution on p. 979.)

Convert the conic r = 21+2 cos θ to rectangular form.

Available for free at Connexions <http://legacy.cnx.org/content/col11759/1.2>

933

Media: Access these online resources for additional instruction and practice with conics in polarcoordinates.

• Polar Equations of Conic Sections21• Graphing Polar Equations of Conics - 122• Graphing Polar Equations of Conics - 223

Visit this website24 for additional practice questions from Learningpod.

8.6.4 Key Concepts

• Any conic may be determined by a single focus, the corresponding eccentricity, and the directrix. Wecan also de�ne a conic in terms of a �xed point, the focusP (r, θ) at the pole, and a line, the directrix,which is perpendicular to the polar axis.

• A conic is the set of all points e = PFPD , where eccentricity e is a positive real number. Each conic may

be written in terms of its polar equation. See Example 8.25.• The polar equations of conics can be graphed. See Example 8.26, Example 8.27, and Example 8.28.• Conics can be de�ned in terms of a focus, a directrix, and eccentricity. See Example 8.29 and Exam-

ple 8.30.• We can use the identities r =

√x2 + y2, x = r cos θ,and y = r sin θ to convert the equation for a conic

from polar to rectangular form. See Example 8.31.

8.6.5 Section Exercises

8.6.5.1 Verbal

Exercise 8.6.5 (Solution on p. 979.)

Explain how eccentricity determines which conic section is given.

Exercise 8.6.6If a conic section is written as a polar equation, what must be true of the denominator?

Exercise 8.6.7 (Solution on p. 979.)

If a conic section is written as a polar equation, and the denominator involves sin θ,what conclusioncan be drawn about the directrix?

Exercise 8.6.8If the directrix of a conic section is perpendicular to the polar axis, what do we know about theequation of the graph?

Exercise 8.6.9 (Solution on p. 979.)

What do we know about the focus/foci of a conic section if it is written as a polar equation?

8.6.5.2 Algebraic

For the following exercises, identify the conic with a focus at the origin, and then give the directrix andeccentricity.

Exercise 8.6.10r = 6

1−2 cos θ

21http://openstaxcollege.org/l/determineconic22http://openstaxcollege.org/l/graphconic123http://openstaxcollege.org/l/graphconic224http://openstaxcollege.org/l/PreCalcLPC10

Available for free at Connexions <http://legacy.cnx.org/content/col11759/1.2>

934 CHAPTER 8. ANALYTIC GEOMETRY

Exercise 8.6.11 (Solution on p. 979.)

r = 34−4 sin θ

Exercise 8.6.12r = 8

4−3 cos θ

Exercise 8.6.13 (Solution on p. 979.)

r = 51+2 sin θ

Exercise 8.6.14r = 16

4+3 cos θ

Exercise 8.6.15 (Solution on p. 979.)

r = 310+10 cos θ

Exercise 8.6.16r = 2

1−cos θ

Exercise 8.6.17 (Solution on p. 979.)

r = 47+2 cos θ

Exercise 8.6.18r (1− cos θ) = 3Exercise 8.6.19 (Solution on p. 980.)

r (3 + 5sin θ) = 11Exercise 8.6.20r (4− 5sin θ) = 1Exercise 8.6.21 (Solution on p. 980.)

r (7 + 8cos θ) = 7For the following exercises, convert the polar equation of a conic section to a rectangular equation.

Exercise 8.6.22r = 4

1+3 sin θ

Exercise 8.6.23 (Solution on p. 980.)

r = 25−3 sin θ

Exercise 8.6.24r = 8

3−2 cos θ

Exercise 8.6.25 (Solution on p. 980.)

r = 32+5 cos θ

Exercise 8.6.26r = 4

2+2 sin θ

Exercise 8.6.27 (Solution on p. 980.)

r = 38−8 cos θ

Exercise 8.6.28r = 2

6+7 cos θ

Exercise 8.6.29 (Solution on p. 980.)

r = 55−11 sin θ

Exercise 8.6.30r (5 + 2 cos θ) = 6Exercise 8.6.31 (Solution on p. 980.)

r (2− cos θ) = 1Exercise 8.6.32r (2.5− 2.5 sin θ) = 5

Available for free at Connexions <http://legacy.cnx.org/content/col11759/1.2>

935

Exercise 8.6.33 (Solution on p. 980.)

r = 6sec θ−2+3 sec θ

Exercise 8.6.34r = 6csc θ

3+2 csc θ

For the following exercises, graph the given conic section. If it is a parabola, label the vertex, focus, anddirectrix. If it is an ellipse, label the vertices and foci. If it is a hyperbola, label the vertices and foci.

Exercise 8.6.35 (Solution on p. 980.)

r = 52+cos θ

Exercise 8.6.36r = 2

3+3 sin θ

Exercise 8.6.37 (Solution on p. 980.)

r = 105−4 sin θ

Exercise 8.6.38r = 3

1+2 cos θ

Exercise 8.6.39 (Solution on p. 981.)

r = 84−5 cos θ

Exercise 8.6.40r = 3

4−4 cos θ

Exercise 8.6.41 (Solution on p. 982.)

r = 21−sin θ

Exercise 8.6.42r = 6

3+2 sin θ

Exercise 8.6.43 (Solution on p. 982.)

r (1 + cos θ) = 5Exercise 8.6.44r (3− 4sin θ) = 9Exercise 8.6.45 (Solution on p. 983.)

r (3− 2sin θ) = 6Exercise 8.6.46r (6− 4cos θ) = 5

For the following exercises, �nd the polar equation of the conic with focus at the origin and the giveneccentricity and directrix.

Exercise 8.6.47 (Solution on p. 984.)

Directrix:x = 4; e = 15

Exercise 8.6.48Directrix:x = −4; e = 5Exercise 8.6.49 (Solution on p. 984.)

Directrix:y = 2; e = 2Exercise 8.6.50Directrix: y = −2; e = 1

2

Exercise 8.6.51 (Solution on p. 984.)

Directrix:x = 1; e = 1Exercise 8.6.52Directrix:x = −1; e = 1Exercise 8.6.53 (Solution on p. 984.)

Directrix: x = − 14 ; e = 7

2

Available for free at Connexions <http://legacy.cnx.org/content/col11759/1.2>

936 CHAPTER 8. ANALYTIC GEOMETRY

Exercise 8.6.54Directrix:y = 2

5 ; e = 72

Exercise 8.6.55 (Solution on p. 984.)

Directrix: y = 4; e = 32

Exercise 8.6.56Directrix:x = −2; e = 8

3

Exercise 8.6.57 (Solution on p. 984.)

Directrix:x = −5; e = 34

Exercise 8.6.58Directrix:y = 2; e = 2.5Exercise 8.6.59 (Solution on p. 984.)

Directrix:x = −3; e = 13

8.6.5.3 Extensions

Recall from Rotation of Axes (Section 8.5) that equations of conics with anxy term have rotated graphs.For the following exercises, express each equation in polar form with r as a function of θ.

Exercise 8.6.60xy = 2Exercise 8.6.61 (Solution on p. 984.)

x2 + xy + y2 = 4Exercise 8.6.622x2 + 4xy + 2y2 = 9Exercise 8.6.63 (Solution on p. 984.)

16x2 + 24xy + 9y2 = 4Exercise 8.6.642xy + y = 1

8.6.6 Chapter Review Exercises

8.6.6.1 The Ellipse (Section 8.2)

For the following exercises, write the equation of the ellipse in standard form. Then identify the center,vertices, and foci.

Exercise 8.6.65 (Solution on p. 984.)x2

25 + y2

64 = 1Exercise 8.6.66(x−2)2

100 + (y+3)2

36 = 1Exercise 8.6.67 (Solution on p. 984.)

9x2 + y2 + 54x− 4y + 76 = 0Exercise 8.6.689x2 + 36y2 − 36x+ 72y + 36 = 0

For the following exercises, graph the ellipse, noting center, vertices, and foci.

Exercise 8.6.69 (Solution on p. 984.)x2

36 + y2

9 = 1

Available for free at Connexions <http://legacy.cnx.org/content/col11759/1.2>

937

Exercise 8.6.70(x−4)2

25 + (y+3)2

49 = 1Exercise 8.6.71 (Solution on p. 985.)

4x2 + y2 + 16x+ 4y − 44 = 0Exercise 8.6.722x2 + 3y2 − 20x+ 12y + 38 = 0

For the following exercises, use the given information to �nd the equation for the ellipse.

Exercise 8.6.73 (Solution on p. 985.)

Center at (0, 0) ,focus at (3, 0) ,vertex at (−5, 0)Exercise 8.6.74Center at (2,−2) ,vertex at (7,−2) ,focus at (4,−2)Exercise 8.6.75 (Solution on p. 985.)

A whispering gallery is to be constructed such that the foci are located 35 feet from the center. Ifthe length of the gallery is to be 100 feet, what should the height of the ceiling be?

8.6.6.2 The Hyperbola (Section 8.3)

For the following exercises, write the equation of the hyperbola in standard form. Then give the center,vertices, and foci.

Exercise 8.6.76x2

81 −y2

9 = 1Exercise 8.6.77 (Solution on p. 985.)(y+1)2

16 − (x−4)2

36 = 1Exercise 8.6.789y2 − 4x2 + 54y − 16x+ 29 = 0Exercise 8.6.79 (Solution on p. 986.)

3x2 − y2 − 12x− 6y − 9 = 0For the following exercises, graph the hyperbola, labeling vertices and foci.

Exercise 8.6.80x2

9 −y2

16 = 1Exercise 8.6.81 (Solution on p. 986.)(y−1)2

49 − (x+1)2

4 = 1Exercise 8.6.82x2 − 4y2 + 6x+ 32y − 91 = 0Exercise 8.6.83 (Solution on p. 986.)

2y2 − x2 − 12y − 6 = 0For the following exercises, �nd the equation of the hyperbola.

Exercise 8.6.84Center at (0, 0) ,vertex at (0, 4) ,focus at (0,−6)Exercise 8.6.85 (Solution on p. 987.)

Foci at (3, 7) and (7, 7) ,vertex at (6, 7)

Available for free at Connexions <http://legacy.cnx.org/content/col11759/1.2>

938 CHAPTER 8. ANALYTIC GEOMETRY

8.6.6.3 The Parabola

For the following exercises, write the equation of the parabola in standard form. Then give the vertex, focus,and directrix.

Exercise 8.6.86y2 = 12xExercise 8.6.87 (Solution on p. 987.)

(x+ 2)2 = 12 (y − 1)

Exercise 8.6.88y2 − 6y − 6x− 3 = 0Exercise 8.6.89 (Solution on p. 987.)

x2 + 10x− y + 23 = 0For the following exercises, graph the parabola, labeling vertex, focus, and directrix.

Exercise 8.6.90x2 + 4y = 0Exercise 8.6.91 (Solution on p. 987.)

(y − 1)2 = 12 (x+ 3)

Exercise 8.6.92x2 − 8x− 10y + 46 = 0Exercise 8.6.93 (Solution on p. 988.)

2y2 + 12y + 6x+ 15 = 0For the following exercises, write the equation of the parabola using the given information.

Exercise 8.6.94Focus at (−4, 0) ; directrix isx = 4Exercise 8.6.95 (Solution on p. 989.)

Focus at(2, 9

8

); directrix is y = 7

8

Exercise 8.6.96A cable TV receiving dish is the shape of a paraboloid of revolution. Find the location of thereceiver, which is placed at the focus, if the dish is 5 feet across at its opening and 1.5 feet deep.

8.6.6.4 Rotation of Axes (Section 8.5)

For the following exercises, determine which of the conic sections is represented.

Exercise 8.6.97 (Solution on p. 989.)

16x2 + 24xy + 9y2 + 24x− 60y − 60 = 0Exercise 8.6.984x2 + 14xy + 5y2 + 18x− 6y + 30 = 0Exercise 8.6.99 (Solution on p. 989.)

4x2 + xy + 2y2 + 8x− 26y + 9 = 0For the following exercises, determine the angle θ that will eliminate thexy term, and write the correspondingequation without thexy term.

Exercise 8.6.100x2 + 4xy − 2y2 − 6 = 0Exercise 8.6.101 (Solution on p. 989.)

x2 − xy + y2 − 6 = 0

Available for free at Connexions <http://legacy.cnx.org/content/col11759/1.2>

939

For the following exercises, graph the equation relative to thex′y′ system in which the equation hasnox′y′ term.

Exercise 8.6.1029x2 − 24xy + 16y2 − 80x− 60y + 100 = 0Exercise 8.6.103 (Solution on p. 989.)

x2 − xy + y2 − 2 = 0Exercise 8.6.1046x2 + 24xy − y2 − 12x+ 26y + 11 = 0

8.6.6.5 Conic Sections in Polar Coordinates (Section 8.6)

For the following exercises, given the polar equation of the conic with focus at the origin, identify theeccentricity and directrix.

Exercise 8.6.105 (Solution on p. 990.)

r = 101−5 cos θ

Exercise 8.6.106r = 6

3+2 cos θ

Exercise 8.6.107 (Solution on p. 990.)

r = 14+3 sin θ

Exercise 8.6.108r = 3

5−5 sin θ

For the following exercises, graph the conic given in polar form. If it is a parabola, label the vertex, focus,and directrix. If it is an ellipse or a hyperbola, label the vertices and foci.

Exercise 8.6.109 (Solution on p. 990.)

r = 31−sin θ

Exercise 8.6.110r = 8

4+3 sin θ

Exercise 8.6.111 (Solution on p. 991.)

r = 104+5 cos θ

Exercise 8.6.112r = 9

3−6 cos θ

For the following exercises, given information about the graph of a conic with focus at the origin, �nd theequation in polar form.

Exercise 8.6.113 (Solution on p. 991.)

Directrix isx = 3 and eccentricity e = 1Exercise 8.6.114Directrix is y = −2 and eccentricity e = 4

Available for free at Connexions <http://legacy.cnx.org/content/col11759/1.2>

940 CHAPTER 8. ANALYTIC GEOMETRY

8.6.7 Practice Test

For the following exercises, write the equation in standard form and state the center, vertices, and foci.

Exercise 8.6.115 (Solution on p. 991.)x2

9 + y2

4 = 1Exercise 8.6.1169y2 + 16x2 − 36y + 32x− 92 = 0

For the following exercises, sketch the graph, identifying the center, vertices, and foci.

Exercise 8.6.117 (Solution on p. 991.)(x−3)2

64 + (y−2)2

36 = 1Exercise 8.6.1182x2 + y2 + 8x− 6y − 7 = 0Exercise 8.6.119 (Solution on p. 992.)

Write the standard form equation of an ellipse with a center at (1, 2) ,vertex at (7, 2) ,and focusat (4, 2) .Exercise 8.6.120A whispering gallery is to be constructed with a length of 150 feet. If the foci are to be located 20feet away from the wall, how high should the ceiling be?

For the following exercises, write the equation of the hyperbola in standard form, and give the center, vertices,foci, and asymptotes.

Exercise 8.6.121 (Solution on p. 992.)x2

49 −y2

81 = 1Exercise 8.6.12216y2 − 9x2 + 128y + 112 = 0

For the following exercises, graph the hyperbola, noting its center, vertices, and foci. State the equations ofthe asymptotes.

Exercise 8.6.123 (Solution on p. 992.)(x−3)2

25 − (y+3)2

1 = 1Exercise 8.6.124y2 − x2 + 4y − 4x− 18 = 0Exercise 8.6.125 (Solution on p. 992.)

Write the standard form equation of a hyperbola with foci at (1, 0) and (1, 6) ,and a vertex at (1, 2) .For the following exercises, write the equation of the parabola in standard form, and give the vertex, focus,and equation of the directrix.

Exercise 8.6.126y2 + 10x = 0Exercise 8.6.127 (Solution on p. 992.)

3x2 − 12x− y + 11 = 0For the following exercises, graph the parabola, labeling the vertex, focus, and directrix.

Exercise 8.6.128(x− 1)2 = −4 (y + 3)Exercise 8.6.129 (Solution on p. 992.)

y2 + 8x− 8y + 40 = 0Exercise 8.6.130Write the equation of a parabola with a focus at (2, 3) and directrix y = −1.

Available for free at Connexions <http://legacy.cnx.org/content/col11759/1.2>

941

Exercise 8.6.131 (Solution on p. 993.)

A searchlight is shaped like a paraboloid of revolution. If the light source is located 1.5 feet fromthe base along the axis of symmetry, and the depth of the searchlight is 3 feet, what should thewidth of the opening be?

For the following exercises, determine which conic section is represented by the given equation, and thendetermine the angle θ that will eliminate thexy term.

Exercise 8.6.1323x2 − 2xy + 3y2 = 4Exercise 8.6.133 (Solution on p. 993.)

x2 + 4xy + 4y2 + 6x− 8y = 0For the following exercises, rewrite in thex′y′ system without thex′y′ term, and graph the rotated graph.

Exercise 8.6.13411x2 + 10

√3xy + y2 = 4

Exercise 8.6.135 (Solution on p. 993.)

16x2 + 24xy + 9y2 − 125x = 0For the following exercises, identify the conic with focus at the origin, and then give the directrix andeccentricity.

Exercise 8.6.136r = 3

2−sin θ

Exercise 8.6.137 (Solution on p. 994.)

r = 54+6 cos θ

For the following exercises, graph the given conic section. If it is a parabola, label vertex, focus, and directrix.If it is an ellipse or a hyperbola, label vertices and foci.

Exercise 8.6.138r = 12

4−8 sin θ

Exercise 8.6.139 (Solution on p. 994.)

r = 24+4 sin θ

Exercise 8.6.140Find a polar equation of the conic with focus at the origin, eccentricity of e = 2,and directrix:x = 3.

Available for free at Connexions <http://legacy.cnx.org/content/col11759/1.2>

942 CHAPTER 8. ANALYTIC GEOMETRY

Solutions to Exercises in Chapter 8

Solution to Exercise 8.2.1 (p. 818)

x2 + y2

16 = 1Solution to Exercise 8.2.2 (p. 821)(x−1)2

16 + (y−3)2

4 = 1Solution to Exercise 8.2.3 (p. 823)center: (0, 0) ; vertices: (±6, 0) ; co-vertices: (0,±2) ; foci:

(±4√

2, 0)

So-lution to Exercise 8.2.4 (p. 824)

Standard form: x2

16 + y2

49 = 1; center: (0, 0) ; vertices: (0,±7) ; co-vertices: (±4, 0) ; foci:(0,±√

33)

Available for free at Connexions <http://legacy.cnx.org/content/col11759/1.2>

943

So-lution to Exercise 8.2.5 (p. 826)Center: (4, 2) ; vertices: (−2, 2) and (10, 2) ; co-vertices:

(4, 2− 2

√5)and

(4, 2 + 2

√5)

; foci: (0, 2) and (8, 2)

Available for free at Connexions <http://legacy.cnx.org/content/col11759/1.2>

944 CHAPTER 8. ANALYTIC GEOMETRY

So-lution to Exercise 8.2.6 (p. 828)(x−3)2

4 + (y+1)2

16 = 1; center: (3,−1) ; vertices: (3,−5) and (3, 3) ; co-vertices: (1,−1) and (5,−1) ; foci:(3,−1− 2

√3)and

(3,−1+2

√3)

Solution to Exercise 8.2.7 (p. 831)

a. x2

57,600 + y2

25,600 = 1b. The people are standing 358 feet apart.

Solution to Exercise 8.2.8 (p. 832)An ellipse is the set of all points in the plane the sum of whose distances from two �xed points, called thefoci, is a constant.Solution to Exercise 8.2.10 (p. 832)This special case would be a circle.Solution to Exercise 8.2.12 (p. 832)It is symmetric about the x-axis, y-axis, and the origin.Solution to Exercise 8.2.14 (p. 832)

yes; x2

32 + y2

22 = 1Solution to Exercise 8.2.16 (p. 832)

yes; x2

( 12 )2 + y2

( 13 )2 = 1

Available for free at Connexions <http://legacy.cnx.org/content/col11759/1.2>

945

Solution to Exercise 8.2.18 (p. 832)x2

22 + y2

72 = 1; Endpoints of major axis (0, 7) and (0,−7) .Endpoints of minor axis (2, 0) and (−2, 0) .Fociat(0, 3√

5),(0,−3

√5).

Solution to Exercise 8.2.20 (p. 833)x2

(1)2+ y2

( 13 )2 = 1; Endpoints of major axis (1, 0) and (−1, 0) .Endpoints of minor axis

(0, 1

3

),(0,− 1

3

).Foci

at(

2√

23 , 0

),(− 2√

23 , 0

).

Solution to Exercise 8.2.22 (p. 833)(x−2)2

72 + (y−4)2

52 = 1; Endpoints of major axis (9, 4) , (−5, 4) .Endpoints of minor axis (2, 9) , (2,−1) .Fociat(2 + 2

√6, 4),(2− 2

√6, 4).

Solution to Exercise 8.2.24 (p. 833)(x+5)2

22 + (y−7)2

32 = 1; Endpoints of major axis (−5, 10) , (−5, 4) .Endpoints of minor axis (−3, 7) , (−7, 7) .Fociat(−5, 7 +

√5),(−5, 7−

√5).

Solution to Exercise 8.2.26 (p. 833)(x−1)2

32 + (y−4)2

22 = 1; Endpoints of major axis (4, 4) , (−2, 4) .Endpoints of minor axis (1, 6) , (1, 2) .Fociat(1 +√

5, 4),(1−√

5, 4).

Solution to Exercise 8.2.28 (p. 833)(x−3)2

(3√

2)2 + (y−5)2

(√

2)2 = 1; Endpoints of major axis(3 + 3

√2, 5),(3− 3

√2, 5). Endpoints of minor

axis(3, 5 +

√2),(3, 5−

√2). Foci at (7, 5) , (−1, 5) .

Solution to Exercise 8.2.30 (p. 833)(x+5)2

(5)2+ (y−2)2

(2)2= 1; Endpoints of major axis (0, 2) , (−10, 2) .Endpoints of minor axis (−5, 4) , (−5, 0) .Foci

at(−5 +

√21, 2

),(−5−

√21, 2

).

Solution to Exercise 8.2.32 (p. 833)(x+3)2

(5)2+ (y+4)2

(2)2= 1; Endpoints of major axis (2,−4) , (−8,−4) .Endpoints of minor

axis (−3,−2) , (−3,−6) .Foci at(−3 +

√21,−4

),(−3−

√21,−4

).

Solution to Exercise 8.2.34 (p. 833)Foci

(−3,−1 +

√11),(−3,−1−

√11)

Solution to Exercise 8.2.36 (p. 833)Focus (0, 0)Solution to Exercise 8.2.38 (p. 833)Foci (−10, 30) , (−10,−30)Solution to Exercise 8.2.40 (p. 834)Center (0, 0) , Vertices (4, 0) , (−4, 0) , (0, 3) , (0,−3) , Foci

(√7, 0),(−√

7, 0)

Available for free at Connexions <http://legacy.cnx.org/content/col11759/1.2>

946 CHAPTER 8. ANALYTIC GEOMETRY

Solution to Exercise 8.2.42 (p. 834)

Center (0, 0) , Vertices(

19 , 0),(− 1

9 , 0),(0, 1

7

),(0,− 1

7

), Foci

(0, 4√

263

),(

0,− 4√

263

)

Solution to Exercise 8.2.44 (p. 834)Center (−3, 3) , Vertices (0, 3) , (−6, 3) , (−3, 0) , (−3, 6) , Focus (−3, 3)

Note that this ellipse is a circle. The circle has only one focus, which coincides with the center.

Available for free at Connexions <http://legacy.cnx.org/content/col11759/1.2>

947

Solution to Exercise 8.2.46 (p. 834)Center (1, 1) , Vertices (5, 1) , (−3, 1) , (1, 3) , (1,−1) , Foci

(1, 1 + 4

√3),(1, 1− 4

√3)

Available for free at Connexions <http://legacy.cnx.org/content/col11759/1.2>

948 CHAPTER 8. ANALYTIC GEOMETRY

Solution to Exercise 8.2.48 (p. 834)Center (−4, 5) , Vertices (−2, 5) , (−6, 4) , (−4, 6) , (−4, 4) , Foci

(−4 +

√3, 5),(−4−

√3, 5)

Solution to Exercise 8.2.50 (p. 834)Center (−2, 1) , Vertices (0, 1) , (−4, 1) , (−2, 5) , (−2,−3) , Foci

(−2, 1 + 2

√3),(−2, 1− 2

√3)

Solution to Exercise 8.2.52 (p. 834)Center (−2,−2) , Vertices (0,−2) , (−4,−2) , (−2, 0) , (−2,−4) , Focus (−2,−2)

Available for free at Connexions <http://legacy.cnx.org/content/col11759/1.2>

949

Solution to Exercise 8.2.54 (p. 834)x2

25 + y2

29 = 1Solution to Exercise 8.2.56 (p. 834)(x−4)2

25 + (y−2)2

1 = 1Solution to Exercise 8.2.58 (p. 834)(x+3)2

16 + (y−4)2

4 = 1Solution to Exercise 8.2.60 (p. 836)x2

81 + y2

9 = 1Solution to Exercise 8.2.62 (p. 837)(x+2)2

4 + (y−2)2

9 = 1Solution to Exercise 8.2.64 (p. 838)Area = 12π square unitsSolution to Exercise 8.2.66 (p. 838)Area = 2

√5π square units

Solution to Exercise 8.2.68 (p. 838)Area = 9π square unitsSolution to Exercise 8.2.70 (p. 838)x2

4h2 + y2

14h

2 = 1Solution to Exercise 8.2.72 (p. 838)x2

400 + y2

144 = 1. Distance = 17.32 feetSolution to Exercise 8.2.74 (p. 838)Approximately 51.96 feetSolution to Exercise 8.3.1 (p. 845)Vertices: (±3, 0) ; Foci:

(±√

34, 0)

Solution to Exercise 8.3.2 (p. 847)y2

4 −x2

16 = 1Solution to Exercise 8.3.3 (p. 850)(y−3)2

25 + (x−1)2

144 = 1Solution to Exercise 8.3.4 (p. 852)vertices: (±12, 0) ; co-vertices: (0,±9) ; foci: (±15, 0) ; asymptotes: y = ± 3

4x;

Available for free at Connexions <http://legacy.cnx.org/content/col11759/1.2>

950 CHAPTER 8. ANALYTIC GEOMETRY

So-lution to Exercise 8.3.5 (p. 855)center: (3,−4) ; vertices: (3,−14) and (3, 6) ; co-vertices: (−5,−4) ; and (11,−4) ; foci:

(3,−4− 2

√41)and

(3,−4 + 2

√41)

; asymptotes: y =± 5

4 (x− 3)− 4

Available for free at Connexions <http://legacy.cnx.org/content/col11759/1.2>

951

Solution to Exercise 8.3.6 (p. 858)

The sides of the tower can be modeled by the hyperbolic equation. x2

400 −y2

3600 = 1or x2

202 − y2

602 = 1.Solution to Exercise 8.3.7 (p. 860)A hyperbola is the set of points in a plane the di�erence of whose distances from two �xed points (foci) is apositive constant.Solution to Exercise 8.3.9 (p. 860)The foci must lie on the transverse axis and be in the interior of the hyperbola.Solution to Exercise 8.3.11 (p. 860)The center must be the midpoint of the line segment joining the foci.Solution to Exercise 8.3.13 (p. 860)

yes x2

62 − y2

32 = 1

Available for free at Connexions <http://legacy.cnx.org/content/col11759/1.2>

952 CHAPTER 8. ANALYTIC GEOMETRY

Solution to Exercise 8.3.15 (p. 861)

yes x2

42 − y2

52 = 1Solution to Exercise 8.3.17 (p. 861)x2

52 − y2

62 = 1; vertices: (5, 0) , (−5, 0) ; foci:(√

61, 0),(−√

61, 0)

; asymptotes: y = 65x, y = − 6

5xSolution to Exercise 8.3.19 (p. 861)y2

22 − x2

92 = 1; vertices: (0, 2) , (0,−2) ; foci:(0,√

85),(0,−√

85)

; asymptotes: y = 29x, y = − 2

9xSolution to Exercise 8.3.21 (p. 861)(x−1)2

32 − (y−2)2

42 = 1; vertices: (4, 2) , (−2, 2) ; foci: (6, 2) , (−4, 2) ; asymptotes: y = 43 (x− 1) + 2, y =

− 43 (x− 1) + 2

Solution to Exercise 8.3.23 (p. 861)(x−2)2

72 − (y+7)2

72 = 1; vertices: (9,−7) , (−5,−7) ; foci:(2 + 7

√2,−7

),(2− 7

√2,−7

); asymptotes: y = x −

9, y = −x− 5Solution to Exercise 8.3.25 (p. 861)(x+3)2

32 − (y−3)2

32 = 1; vertices: (0, 3) , (−6, 3) ; foci:(−3 + 3

√2, 1),(−3− 3

√2, 1)

; asymptotes: y = x+ 6, y =−xSolution to Exercise 8.3.27 (p. 861)(y−4)2

22 − (x−3)2

42 = 1; vertices: (3, 6) , (3, 2) ; foci:(3, 4 + 2

√5),(3, 4− 2

√5)

; asymptotes: y = 12 (x− 3) +

4, y = − 12 (x− 3) + 4

Solution to Exercise 8.3.29 (p. 861)(y+5)2

72 − (x+1)2

702 = 1; vertices: (−1, 2) , (−1,−12) ; foci:(−1,−5 + 7

√101),(−1,−5− 7

√101)

; asymptotes: y =110 (x+ 1)− 5, y = − 1

10 (x+ 1)− 5Solution to Exercise 8.3.31 (p. 861)(x+3)2

52 − (y−4)2

22 = 1; vertices: (2, 4) , (−8, 4) ; foci:(−3 +

√29, 4

),(−3−

√29, 4

); asymptotes: y =

25 (x+ 3) + 4, y = − 2

5 (x+ 3) + 4Solution to Exercise 8.3.33 (p. 861)y = 2

5 (x− 3)− 4, y = − 25 (x− 3)− 4

Solution to Exercise 8.3.35 (p. 862)y = 3

4 (x− 1) + 1, y = − 34 (x− 1) + 1

Solution to Exercise 8.3.37 (p. 862)

Available for free at Connexions <http://legacy.cnx.org/content/col11759/1.2>

953

Solution to Exercise 8.3.39 (p. 862)

Solution to Exercise 8.3.41 (p. 862)

Available for free at Connexions <http://legacy.cnx.org/content/col11759/1.2>

954 CHAPTER 8. ANALYTIC GEOMETRY

Solution to Exercise 8.3.43 (p. 862)

Solution to Exercise 8.3.45 (p. 862)

Available for free at Connexions <http://legacy.cnx.org/content/col11759/1.2>

955

Solution to Exercise 8.3.47 (p. 862)

Solution to Exercise 8.3.49 (p. 862)

Available for free at Connexions <http://legacy.cnx.org/content/col11759/1.2>

956 CHAPTER 8. ANALYTIC GEOMETRY

Solution to Exercise 8.3.51 (p. 862)x2

9 −y2

16 = 1Solution to Exercise 8.3.53 (p. 862)(x−6)2

25 − (y−1)2

11 = 1Solution to Exercise 8.3.55 (p. 863)(x−4)2

25 − (y−2)2

1 = 1Solution to Exercise 8.3.57 (p. 863)y2

16 −x2

25 = 1Solution to Exercise 8.3.59 (p. 865)y2

9 −(x+1)2

9 = 1Solution to Exercise 8.3.61 (p. 867)(x+3)2

25 − (y+3)2

25 = 1Solution to Exercise 8.3.63 (p. 867)y (x) = 3

√x2 + 1, y (x) = −3

√x2 + 1

Solution to Exercise 8.3.65 (p. 867)y (x) = 1 + 2

√x2 + 4x+ 5, y (x) = 1− 2

√x2 + 4x+ 5

Available for free at Connexions <http://legacy.cnx.org/content/col11759/1.2>

957

Solution to Exercise 8.3.67 (p. 868)x2

25 −y2

25 = 1

Available for free at Connexions <http://legacy.cnx.org/content/col11759/1.2>

958 CHAPTER 8. ANALYTIC GEOMETRY

So-lution to Exercise 8.3.69 (p. 868)x2

100 −y2

25 = 1

Available for free at Connexions <http://legacy.cnx.org/content/col11759/1.2>

959

Solution to Exercise 8.3.71 (p. 868)x2

400 −y2

225 = 1

Solution to Exercise 8.3.73 (p. 868)(x−1)2

0.25 −y2

0.75 = 1Solution to Exercise 8.3.75 (p. 868)(x−3)2

4 − y2

5 = 1Solution to Exercise 8.4.1 (p. 876)Focus: (−4, 0) ; Directrix:x = 4; Endpoints of the latus rectum: (−4,±8)

Available for free at Connexions <http://legacy.cnx.org/content/col11759/1.2>

960 CHAPTER 8. ANALYTIC GEOMETRY

So-lution to Exercise 8.4.2 (p. 877)Focus: (0, 2) ; Directrix: y = −2; Endpoints of the latus rectum: (±4, 2) .

Available for free at Connexions <http://legacy.cnx.org/content/col11759/1.2>

961

Solution to Exercise 8.4.3 (p. 878)x2 = 14y.Solution to Exercise 8.4.4 (p. 881)Vertex: (8,−1) ; Axis of symmetry: y = −1; Focus: (9,−1) ; Directrix:x = 7; Endpoints of the latus

rectum: (9,−3) and (9, 1) .

Available for free at Connexions <http://legacy.cnx.org/content/col11759/1.2>

962 CHAPTER 8. ANALYTIC GEOMETRY

Solution to Exercise 8.4.5 (p. 883)Vertex: (−2, 3) ; Axis of symmetry:x = −2; Focus: (−2,−2) ; Directrix: y = 8; Endpoints of the latusrectum: (−12,−2) and (8,−2) .

Available for free at Connexions <http://legacy.cnx.org/content/col11759/1.2>

963

Solution to Exercise 8.4.6 (p. 885)

a. y2 = 1280xb. The depth of the cooker is 500 mm

Solution to Exercise 8.4.7 (p. 887)A parabola is the set of points in the plane that lie equidistant from a �xed point, the focus, and a �xedline, the directrix.Solution to Exercise 8.4.9 (p. 887)The graph will open down.Solution to Exercise 8.4.11 (p. 887)The distance between the focus and directrix will increase.Solution to Exercise 8.4.13 (p. 887)yes y = 4 (1)x2

Solution to Exercise 8.4.15 (p. 887)

yes (y − 3)2 = 4 (2) (x− 2)Solution to Exercise 8.4.17 (p. 887)y2 = 1

8x, V : (0, 0) ;F :(

132 , 0

); d : x = − 1

32Solution to Exercise 8.4.19 (p. 887)x2 = − 1

4y, V : (0, 0) ;F :(0,− 1

16

); d : y = 1

16Solution to Exercise 8.4.21 (p. 887)y2 = 1

36x, V : (0, 0) ;F :(

1144 , 0

); d : x = − 1

144Solution to Exercise 8.4.23 (p. 887)

(x− 1)2 = 4 (y − 1) , V : (1, 1) ;F : (1, 2) ; d : y = 0Solution to Exercise 8.4.25 (p. 888)

(y − 4)2 = 2 (x+ 3) , V : (−3, 4) ;F :(− 5

2 , 4)

; d : x = − 72

Solution to Exercise 8.4.27 (p. 888)

(x+ 4)2 = 24 (y + 1) , V : (−4,−1) ;F : (−4, 5) ; d : y = −7Solution to Exercise 8.4.29 (p. 888)

(y − 3)2 = −12 (x+ 1) , V : (−1, 3) ;F : (−4, 3) ; d : x = 2

Available for free at Connexions <http://legacy.cnx.org/content/col11759/1.2>

964 CHAPTER 8. ANALYTIC GEOMETRY

Solution to Exercise 8.4.31 (p. 888)

(x− 5)2 = 45 (y + 3) , V : (5,−3) ;F :

(5,− 14

5

); d : y = − 16

5Solution to Exercise 8.4.33 (p. 888)

(x− 2)2 = −2 (y − 5) , V : (2, 5) ;F :(2, 9

2

); d : y = 11

2Solution to Exercise 8.4.35 (p. 888)

(y − 1)2 = 43 (x− 5) , V : (5, 1) ;F :

(163 , 1

); d : x = 14

3Solution to Exercise 8.4.37 (p. 888)

Available for free at Connexions <http://legacy.cnx.org/content/col11759/1.2>

965

Solution to Exercise 8.4.39 (p. 888)

Solution to Exercise 8.4.41 (p. 888)

Available for free at Connexions <http://legacy.cnx.org/content/col11759/1.2>

966 CHAPTER 8. ANALYTIC GEOMETRY

Solution to Exercise 8.4.43 (p. 888)

Available for free at Connexions <http://legacy.cnx.org/content/col11759/1.2>

967

Solution to Exercise 8.4.45 (p. 889)

Available for free at Connexions <http://legacy.cnx.org/content/col11759/1.2>

968 CHAPTER 8. ANALYTIC GEOMETRY

Solution to Exercise 8.4.47 (p. 889)

Available for free at Connexions <http://legacy.cnx.org/content/col11759/1.2>

969

Solution to Exercise 8.4.49 (p. 889)

Solution to Exercise 8.4.51 (p. 889)x2 = −16ySolution to Exercise 8.4.53 (p. 889)

(y − 2)2 = 4√

2 (x− 2)Solution to Exercise 8.4.55 (p. 889)(y +√

3)2

= −4√

2(x−√

2)

Solution to Exercise 8.4.57 (p. 889)x2 = ySolution to Exercise 8.4.59 (p. 892)

(y − 2)2 = 14 (x+ 2)

Solution to Exercise 8.4.61 (p. 894)(y −√

3)2

= 4√

5(x+√

2)

Solution to Exercise 8.4.63 (p. 894)y2 = −8xSolution to Exercise 8.4.65 (p. 894)

(y + 1)2 = 12 (x+ 3)

Available for free at Connexions <http://legacy.cnx.org/content/col11759/1.2>

970 CHAPTER 8. ANALYTIC GEOMETRY

Solution to Exercise 8.4.67 (p. 895)(0, 1)Solution to Exercise 8.4.69 (p. 895)At the point 2.25 feet above the vertex.Solution to Exercise 8.4.71 (p. 895)0.5625 feetSolution to Exercise 8.4.73 (p. 895)x2 = −125 (y − 20) , height is 7.2 feetSolution to Exercise 8.4.75 (p. 895)2304 feetSolution to Exercise 8.5.1 (p. 900)

a. hyperbolab. ellipse

Solution to Exercise 8.5.2 (p. 910)x′2

4 + y′2

1 = 1Solution to Exercise 8.5.3 (p. 915)

a. hyperbolab. ellipse

Solution to Exercise 8.5.4 (p. 916)Thexy term causes a rotation of the graph to occur.Solution to Exercise 8.5.6 (p. 916)The conic section is a hyperbola.Solution to Exercise 8.5.8 (p. 916)It gives the angle of rotation of the axes in order to eliminate thexy term.Solution to Exercise 8.5.10 (p. 916)AB = 0, parabolaSolution to Exercise 8.5.12 (p. 916)AB = −4 < 0, hyperbolaSolution to Exercise 8.5.14 (p. 916)AB = 6 > 0, ellipseSolution to Exercise 8.5.16 (p. 916)B2 − 4AC = 0, parabolaSolution to Exercise 8.5.18 (p. 916)B2 − 4AC = 0, parabolaSolution to Exercise 8.5.20 (p. 916)B2 − 4AC = −96 < 0, ellipseSolution to Exercise 8.5.22 (p. 917)

7x′2 + 9y′2 − 4 = 0Solution to Exercise 8.5.24 (p. 917)

3x′2 + 2x′y′ − 5y′2 + 1 = 0Solution to Exercise 8.5.26 (p. 917)

θ = 60◦, 11x′2 − y′2 +√

3x′ + y′ − 4 = 0Solution to Exercise 8.5.28 (p. 917)

θ = 150◦, 21x′2 + 9y′2 + 4x′ − 4√

3y′ − 6 = 0Solution to Exercise 8.5.30 (p. 917)

θ ≈ 36.9◦, 125x′2 + 6x′ − 42y′ + 10 = 0Solution to Exercise 8.5.32 (p. 917)

θ = 45◦, 3x′2 − y′2 −√

2x′ +√

2y′ + 1 = 0

Available for free at Connexions <http://legacy.cnx.org/content/col11759/1.2>

971

Solution to Exercise 8.5.34 (p. 917)√2

2 (x′ + y′) = 12 (x′ − y′)2

Solution to Exercise 8.5.36 (p. 917)(x′−y′)2

8 + (x′+y′)2

2 = 1

Solution to Exercise 8.5.38 (p. 917)(x′+y′)2

2 − (x′−y′)2

2 = 1

Available for free at Connexions <http://legacy.cnx.org/content/col11759/1.2>

972 CHAPTER 8. ANALYTIC GEOMETRY

Solution to Exercise 8.5.40 (p. 917)√

32 x′ − 1

2y′ =

(12x′ +

√3

2 y′ − 1

)2

Available for free at Connexions <http://legacy.cnx.org/content/col11759/1.2>

973

Solution to Exercise 8.5.42 (p. 918)

Available for free at Connexions <http://legacy.cnx.org/content/col11759/1.2>

974 CHAPTER 8. ANALYTIC GEOMETRY

Solution to Exercise 8.5.44 (p. 918)

Available for free at Connexions <http://legacy.cnx.org/content/col11759/1.2>

975

Solution to Exercise 8.5.46 (p. 918)

Available for free at Connexions <http://legacy.cnx.org/content/col11759/1.2>

976 CHAPTER 8. ANALYTIC GEOMETRY

Solution to Exercise 8.5.48 (p. 918)

Solution to Exercise 8.5.50 (p. 918)

Available for free at Connexions <http://legacy.cnx.org/content/col11759/1.2>

977

Solution to Exercise 8.5.52 (p. 918)

Solution to Exercise 8.5.54 (p. 918)θ = 45◦

Available for free at Connexions <http://legacy.cnx.org/content/col11759/1.2>

978 CHAPTER 8. ANALYTIC GEOMETRY

Solution to Exercise 8.5.56 (p. 918)θ = 60◦

Solution to Exercise 8.5.58 (p. 918)θ ≈ 36.9◦

Solution to Exercise 8.5.60 (p. 919)−4√

6 < k < 4√

6Solution to Exercise 8.5.62 (p. 919)k = 2Solution to Exercise 8.6.1 (p. 922)ellipse; e = 1

3 ; x = −2

Available for free at Connexions <http://legacy.cnx.org/content/col11759/1.2>

979

Solution to Exercise 8.6.2 (p. 930)

Solu-tion to Exercise 8.6.3 (p. 932)r = 1

1−cosθSolution to Exercise 8.6.4 (p. 932)4− 8x+ 3x2 − y2 = 0Solution to Exercise 8.6.5 (p. 933)If eccentricity is less than 1, it is an ellipse. If eccentricity is equal to 1, it is a parabola. If eccentricity isgreater than 1, it is a hyperbola.Solution to Exercise 8.6.7 (p. 933)The directrix will be parallel to the polar axis.Solution to Exercise 8.6.9 (p. 933)One of the foci will be located at the origin.Solution to Exercise 8.6.11 (p. 933)Parabola with e = 1 and directrix 3

4 units below the pole.Solution to Exercise 8.6.13 (p. 934)Hyperbola with e = 2 and directrix 5

2 units above the pole.Solution to Exercise 8.6.15 (p. 934)Parabola with e = 1 and directrix 3

10 units to the right of the pole.

Available for free at Connexions <http://legacy.cnx.org/content/col11759/1.2>

980 CHAPTER 8. ANALYTIC GEOMETRY

Solution to Exercise 8.6.17 (p. 934)Ellipse with e = 2

7 and directrix 2 units to the right of the pole.Solution to Exercise 8.6.19 (p. 934)Hyperbola with e = 5

3 and directrix 115 units above the pole.

Solution to Exercise 8.6.21 (p. 934)Hyperbola with e = 8

7 and directrix 78 units to the right of the pole.

Solution to Exercise 8.6.23 (p. 934)25x2 + 16y2 − 12y − 4 = 0Solution to Exercise 8.6.25 (p. 934)21x2 − 4y2 − 30x+ 9 = 0Solution to Exercise 8.6.27 (p. 934)64y2 = 48x+ 9Solution to Exercise 8.6.29 (p. 934)96y2 − 25x2 + 110y + 25 = 0Solution to Exercise 8.6.31 (p. 934)3x2 + 4y2 − 2x− 1 = 0Solution to Exercise 8.6.33 (p. 934)5x2 + 9y2 − 24x− 36 = 0Solution to Exercise 8.6.35 (p. 935)

Available for free at Connexions <http://legacy.cnx.org/content/col11759/1.2>

981

Solution to Exercise 8.6.37 (p. 935)

Available for free at Connexions <http://legacy.cnx.org/content/col11759/1.2>

982 CHAPTER 8. ANALYTIC GEOMETRY

Solution to Exercise 8.6.39 (p. 935)

Solution to Exercise 8.6.41 (p. 935)

Available for free at Connexions <http://legacy.cnx.org/content/col11759/1.2>

983

Solution to Exercise 8.6.43 (p. 935)

Available for free at Connexions <http://legacy.cnx.org/content/col11759/1.2>

984 CHAPTER 8. ANALYTIC GEOMETRY

Solution to Exercise 8.6.45 (p. 935)

Solution to Exercise 8.6.47 (p. 935)r = 4

5+cosθSolution to Exercise 8.6.49 (p. 935)r = 4

1+2sinθSolution to Exercise 8.6.51 (p. 935)r = 1

1+cosθSolution to Exercise 8.6.53 (p. 935)r = 7

8−28cosθSolution to Exercise 8.6.55 (p. 936)r = 12

2+3sinθSolution to Exercise 8.6.57 (p. 936)r = 15

4−3cosθSolution to Exercise 8.6.59 (p. 936)r = 3

3−3cosθSolution to Exercise 8.6.61 (p. 936)r = ± 2√

1+sinθcosθ

Solution to Exercise 8.6.63 (p. 936)r = ± 2

4cosθ+3sinθSolution to Exercise 8.6.65 (p. 936)x2

52 + y2

82 = 1; center: (0, 0) ; vertices: (5, 0) , (−5, 0) , (0, 8) , (0,−8) ; foci:(0,√

39),(0,−√

39)

Solution to Exercise 8.6.67 (p. 936)(x+3)2

12 + (y−2)2

32 = 1 (−3, 2) ; (−2, 2) , (−4, 2) , (−3, 5) , (−3,−1) ;(−3, 2 + 2

√2),(−3, 2− 2

√2)

Solution to Exercise 8.6.69 (p. 936)center: (0, 0) ; vertices: (6, 0) , (−6, 0) , (0, 3) , (0,−3) ; foci:

(3√

3, 0),(−3√

3, 0)

Available for free at Connexions <http://legacy.cnx.org/content/col11759/1.2>

985

Solution to Exercise 8.6.71 (p. 937)center: (−2,−2) ; vertices: (2,−2) , (−6,−2) , (−2, 6) , (−2,−10) ; foci:

(−2,−2 + 4

√3,),(−2,−2− 4

√3)

Solution to Exercise 8.6.73 (p. 937)x2

25 + y2

16 = 1Solution to Exercise 8.6.75 (p. 937)Approximately 35.71 feet

Available for free at Connexions <http://legacy.cnx.org/content/col11759/1.2>

986 CHAPTER 8. ANALYTIC GEOMETRY

Solution to Exercise 8.6.77 (p. 937)(y+1)2

42 − (x−4)2

62 = 1; center: (4,−1) ; vertices: (4, 3) , (4,−5) ; foci:(4,−1 + 2

√13),(4,−1− 2

√13)

Solution to Exercise 8.6.79 (p. 937)(x−2)2

22 − (y+3)2

(2√

3)2 = 1; center: (2,−3) ; vertices: (4,−3) , (0,−3) ; foci: (6,−3) , (−2,−3)

Solution to Exercise 8.6.81 (p. 937)

Solution to Exercise 8.6.83 (p. 937)

Available for free at Connexions <http://legacy.cnx.org/content/col11759/1.2>

987

Solution to Exercise 8.6.85 (p. 937)(x−5)2

1 − (y−7)2

3 = 1Solution to Exercise 8.6.87 (p. 938)

(x+ 2)2 = 12 (y − 1) ; vertex: (−2, 1) ; focus:

(−2, 9

8

); directrix: y = 7

8Solution to Exercise 8.6.89 (p. 938)

(x+ 5)2 = (y + 2) ; vertex: (−5,−2) ; focus:(−5,− 7

4

); directrix: y = − 9

4Solution to Exercise 8.6.91 (p. 938)

Available for free at Connexions <http://legacy.cnx.org/content/col11759/1.2>

988 CHAPTER 8. ANALYTIC GEOMETRY

Solution to Exercise 8.6.93 (p. 938)

Available for free at Connexions <http://legacy.cnx.org/content/col11759/1.2>

989

Solution to Exercise 8.6.95 (p. 938)

(x− 2)2 =(

12

)(y − 1)

Solution to Exercise 8.6.97 (p. 938)B2 − 4AC = 0, parabolaSolution to Exercise 8.6.99 (p. 938)B2 − 4AC = −31 < 0, ellipseSolution to Exercise 8.6.101 (p. 938)

θ = 45◦, x′2 + 3y′2 − 12 = 0Solution to Exercise 8.6.103 (p. 939)θ = 45◦

Available for free at Connexions <http://legacy.cnx.org/content/col11759/1.2>

990 CHAPTER 8. ANALYTIC GEOMETRY

Solution to Exercise 8.6.105 (p. 939)Hyperbola with e = 5 and directrix 2 units to the left of the pole.Solution to Exercise 8.6.107 (p. 939)Ellipse with e = 3

4 and directrix 13 unit above the pole.

Solution to Exercise 8.6.109 (p. 939)

Available for free at Connexions <http://legacy.cnx.org/content/col11759/1.2>

991

Solution to Exercise 8.6.111 (p. 939)

Solution to Exercise 8.6.113 (p. 939)r = 3

1+cos θSolution to Exercise 8.6.115 (p. 940)x2

32 + y2

22 = 1; center: (0, 0) ; vertices: (3, 0) , (−− 3, 0) , (0, 2) , (0,−2) ; foci:(√

5, 0),(−√

5, 0)

Solution to Exercise 8.6.117 (p. 940)center: (3, 2) ; vertices: (11, 2) , (−5, 2) , (3, 8) , (3,−4) ; foci:

(3 + 2

√7, 2),(3− 2

√7, 2)

Available for free at Connexions <http://legacy.cnx.org/content/col11759/1.2>

992 CHAPTER 8. ANALYTIC GEOMETRY

Solution to Exercise 8.6.119 (p. 940)(x−1)2

36 + (y−2)2

27 = 1Solution to Exercise 8.6.121 (p. 940)x2

72 − y2

92 = 1; center: (0, 0) ; vertices (7, 0) , (−7, 0) ; foci:(√

130, 0),(−√

130, 0)

; asymptotes: y = ± 97x

Solution to Exercise 8.6.123 (p. 940)center: (3,−3) ; vertices: (8,−3) , (−2,−3) ;foci:

(3 +√

26,−3),(3−√

26,−3)

; asymptotes: y =± 1

5 (x− 3)− 3

Solution to Exercise 8.6.125 (p. 940)(y−3)2

1 − (x−1)2

8 = 1Solution to Exercise 8.6.127 (p. 940)

(x− 2)2 = 13 (y + 1) ; vertex: (2,−1) ; focus:

(2,− 11

12

); directrix: y = − 13

12

Available for free at Connexions <http://legacy.cnx.org/content/col11759/1.2>

993

Solution to Exercise 8.6.129 (p. 940)

Solution to Exercise 8.6.131 (p. 940)Approximately 8.49 feetSolution to Exercise 8.6.133 (p. 941)parabola; θ ≈ 63.4◦

Solution to Exercise 8.6.135 (p. 941)

x′2 − 4x′ + 3y′ = 0

Available for free at Connexions <http://legacy.cnx.org/content/col11759/1.2>

994 CHAPTER 8. ANALYTIC GEOMETRY

Solution to Exercise 8.6.137 (p. 941)Hyperbola with e = 3

2 , and directrix 56 units to the right of the pole.

Available for free at Connexions <http://legacy.cnx.org/content/col11759/1.2>

995

Solution to Exercise 8.6.139 (p. 941)

Available for free at Connexions <http://legacy.cnx.org/content/col11759/1.2>

996 CHAPTER 8. ANALYTIC GEOMETRY

Available for free at Connexions <http://legacy.cnx.org/content/col11759/1.2>

Chapter 9

Sequences, Probability, and Counting

Theory

9.1 Introduction to Sequences, Probability and Counting Theory1

Figure 9.1: (credit: Robert S. Donovan, Flickr.)

1This content is available online at <http://legacy.cnx.org/content/m49443/1.3/>.

Available for free at Connexions <http://legacy.cnx.org/content/col11759/1.2>

997

998 CHAPTER 9. SEQUENCES, PROBABILITY, AND COUNTING THEORY

A lottery winner has some big decisions to make regarding what to do with the winnings. Buy a villa inSaint Barthélemy? A luxury convertible? A cruise around the world?

The likelihood of winning the lottery is slim, but we all love to fantasize about what we could buy withthe winnings. One of the �rst things a lottery winner has to decide is whether to take the winnings in theform of a lump sum or as a series of regular payments, called an annuity, over the next 30 years or so.

This decision is often based on many factors, such as tax implications, interest rates, and investmentstrategies. There are also personal reasons to consider when making the choice, and one can make manyarguments for either decision. However, most lottery winners opt for the lump sum.

In this chapter, we will explore the mathematics behind situations such as these. We will take an in-depth look at annuities. We will also look at the branch of mathematics that would allow us to calculatethe number of ways to choose lottery numbers and the probability of winning.

9.2 Sequences and Their Notations2

A video game company launches an exciting new advertising campaign. They predict the number of onlinevisits to their website, or hits, will double each day. The model they are using shows 2 hits the �rst day, 4hits the second day, 8 hits the third day, and so on. See Table 9.1.

Day 1 2 3 4 5 . . .

Hits 2 4 8 16 32 . . .

Table 9.1

If their model continues, how many hits will there be at the end of the month? To answer this question,we'll �rst need to know how to determine a list of numbers written in a speci�c order. In this section, wewill explore these kinds of ordered lists.

9.2.1 Writing the Terms of a Sequence De�ned by an Explicit Formula

One way to describe an ordered list of numbers is as a sequence. A sequence is a function whose domainis a subset of the counting numbers. The sequence established by the number of hits on the website is

{2, 4, 8, 16, 32, . . . }. (9.1)

The ellipsis (. . .) indicates that the sequence continues inde�nitely. Each number in the sequence is calleda term. The �rst �ve terms of this sequence are 2, 4, 8, 16, and 32.

Listing all of the terms for a sequence can be cumbersome. For example, �nding the number of hits onthe website at the end of the month would require listing out as many as 31 terms. A more e�cient way todetermine a speci�c term is by writing a formula to de�ne the sequence.

One type of formula is an explicit formula, which de�nes the terms of a sequence using their positionin the sequence. Explicit formulas are helpful if we want to �nd a speci�c term of a sequence without �ndingall of the previous terms. We can use the formula to �nd the nth term of the sequence, where n is anypositive number. In our example, each number in the sequence is double the previous number, so we canuse powers of 2 to write a formula for the nth term.

2This content is available online at <http://legacy.cnx.org/content/m49444/1.4/>.

Available for free at Connexions <http://legacy.cnx.org/content/col11759/1.2>

999

The �rst term of the sequence is 21 = 2, the second term is 22 = 4, the third term is 23 = 8, and so on.The nth term of the sequence can be found by raising 2 to the nth power. An explicit formula for a sequenceis named by a lower case letter a, b, c... with the subscript n. The explicit formula for this sequence is

an = 2n. (9.2)

Now that we have a formula for the nth term of the sequence, we can answer the question posed at thebeginning of this section. We were asked to �nd the number of hits at the end of the month, which we willtake to be 31 days. To �nd the number of hits on the last day of the month, we need to �nd the 31st termof the sequence. We will substitute 31 for n in the formula.

a31 = 231

= 2,147,483,648(9.3)

If the doubling trend continues, the company will get 2,147,483,648 hits on the last day of the month. Thatis over 2.1 billion hits! The huge number is probably a little unrealistic because it does not take consumerinterest and competition into account. It does, however, give the company a starting point from which toconsider business decisions.

Another way to represent the sequence is by using a table. The �rst �ve terms of the sequence and thenth term of the sequence are shown in Table 9.2.

n 1 2 3 4 5 n

nth term of the sequence, an 2 4 8 16 32 2n

Table 9.2

Graphing provides a visual representation of the sequence as a set of distinct points. We can see fromthe graph in Figure 9.2 that the number of hits is rising at an exponential rate. This particular sequenceforms an exponential function.

Available for free at Connexions <http://legacy.cnx.org/content/col11759/1.2>

1000 CHAPTER 9. SEQUENCES, PROBABILITY, AND COUNTING THEORY

Figure 9.2

Lastly, we can write this particular sequence as

{2, 4, 8, 16, 32, . . . , 2n, . . . }. (9.4)

A sequence that continues inde�nitely is called an in�nite sequence. The domain of an in�nite sequenceis the set of counting numbers. If we consider only the �rst 10 terms of the sequence, we could write

{2, 4, 8, 16, 32, . . . , 2n, . . . , 1024}. (9.5)

This sequence is called a �nite sequence because it does not continue inde�nitely.

A General Note: A sequence is a function whose domain is the set of positive integers.A �nite sequence is a sequence whose domain consists of only the �rst n positive integers. Thenumbers in a sequence are called terms. The variable a with a number subscript is used to representthe terms in a sequence and to indicate the position of the term in the sequence.

Available for free at Connexions <http://legacy.cnx.org/content/col11759/1.2>

1001

a1, a2, a3, . . . , an, . . . (9.6)

We call a1 the �rst term of the sequence, a2 the second term of the sequence, a3 the third termof the sequence, and so on. The term an is called the nth term of the sequence, or the generalterm of the sequence. An explicit formula de�nes the nth term of a sequence using the positionof the term. A sequence that continues inde�nitely is an in�nite sequence.

Q&A: Does a sequence always have to begin with a1?

No. In certain problems, it may be useful to de�ne the initial term as a0instead of a1. In theseproblems, the domain of the function includes 0.

How To: Given an explicit formula, write the �rstn terms of a sequence.

1.Substitute each value of n into the formula. Begin with n = 1 to �nd the �rst term, a1.2.To �nd the second term, a2, use n = 2.3.Continue in the same manner until you have identi�ed all n terms.

Example 9.1Writing the Terms of a Sequence De�ned by an Explicit FormulaWrite the �rst �ve terms of the sequence de�ned by the explicit formula an = −3n+ 8.

SolutionSubstitute n = 1 into the formula. Repeat with values 2 through 5 for n.

n = 1 a1 = −3 (1) + 8 = 5

n = 2 a2 = −3 (2) + 8 = 2

n = 3 a3 = −3 (3) + 8 = −1

n = 4 a4 = −3 (4) + 8 = −4

n = 5 a5 = −3 (5) + 8 = −7

(9.7)

The �rst �ve terms are {5, 2, − 1, − 4, − 7}.AnalysisThe sequence values can be listed in a table. A table, such as Table 9.3, is a convenient way toinput the function into a graphing utility.

n 1 2 3 4 5

an 5 2 �1 �4 �7

Table 9.3

A graph can be made from this table of values. From the graph in Figure 9.3, we can seethat this sequence represents a linear function, but notice the graph is not continuous because thedomain is over the positive integers only.

Available for free at Connexions <http://legacy.cnx.org/content/col11759/1.2>

1002 CHAPTER 9. SEQUENCES, PROBABILITY, AND COUNTING THEORY

Figure 9.3

Available for free at Connexions <http://legacy.cnx.org/content/col11759/1.2>

1003

Try It:

Exercise 9.2.1 (Solution on p. 1120.)

Write the �rst �ve terms of the sequence de�ned by the explicit formula tn = 5n− 4.

9.2.1.1 Investigating Alternating Sequences

Sometimes sequences have terms that are alternate. In fact, the terms may actually alternate in sign. Thesteps to �nding terms of the sequence are the same as if the signs did not alternate. However, the resultingterms will not show increase or decrease as n increases. Let's take a look at the following sequence.

{2,−4, 6,−8} (9.8)

Notice the �rst term is greater than the second term, the second term is less than the third term, and thethird term is greater than the fourth term. This trend continues forever. Do not rearrange the terms innumerical order to interpret the sequence.

How To: Given an explicit formula with alternating terms, write the �rst n terms ofa sequence.

1.Substitute each value of n into the formula. Begin with n = 1 to �nd the �rst term, a1. Thesign of the term is given by the (−1)n in the explicit formula.

2.To �nd the second term, a2, usen = 2.3.Continue in the same manner until you have identi�ed all n terms.

Example 9.2Writing the Terms of an Alternating Sequence De�ned by an Explicit FormulaWrite the �rst �ve terms of the sequence.

an =(−1)nn2

n+ 1(9.9)

Solution

Substitute n = 1, n = 2, and so on in the formula.

n = 1 a1 = (−1)122

1+1 = − 12

n = 2 a2 = (−1)222

2+1 = 43

n = 3 a3 = (−1)332

3+1 = − 94

n = 4 a4 = (−1)442

4+1 = 165

n = 5 a5 = (−1)552

5+1 = − 256

(9.10)

The �rst �ve terms are {− 12 ,

43 ,−

94 ,

165 ,−

256 }.

AnalysisThe graph of this function, shown in Figure 9.4, looks di�erent from the ones we have seenpreviously in this section because the terms of the sequence alternate between positive and negativevalues.

Available for free at Connexions <http://legacy.cnx.org/content/col11759/1.2>

1004 CHAPTER 9. SEQUENCES, PROBABILITY, AND COUNTING THEORY

Figure 9.4

Q&A: In Example 9.2, does the (�1) to the power of n account for the oscillations ofsigns?

Yes, the power might be n, n + 1, n − 1, and so on, but any odd powers will result in a negativeterm, and any even power will result in a positive term.

Available for free at Connexions <http://legacy.cnx.org/content/col11759/1.2>

1005

Try It:

Exercise 9.2.2 (Solution on p. 1120.)

Write the �rst �ve terms of the sequence:

an =4n

(−2)n(9.11)

9.2.1.2 Investigating Piecewise Explicit Formulas

We've learned that sequences are functions whose domain is over the positive integers. This is true for othertypes of functions, including some piecewise functions. Recall that a piecewise function is a functionde�ned by multiple subsections. A di�erent formula might represent each individual subsection.

How To: Given an explicit formula for a piecewise function, write the �rst n terms ofa sequence

1.Identify the formula to which n = 1 applies.2.To �nd the �rst term, a1, usen = 1 in the appropriate formula.3.Identify the formula to whichn = 2 applies.4.To �nd the second term, a2, usen = 2 in the appropriate formula.5.Continue in the same manner until you have identi�ed alln terms.

Example 9.3Writing the Terms of a Sequence De�ned by a Piecewise Explicit FormulaWrite the �rst six terms of the sequence.

an = {n2 if n is not divisible by 3

n3 if n is divisible by 3

(9.12)

Solution

Substituten = 1, n = 2, and so on in the appropriate formula. Use n2 when n is not a multiple of3. Use n

3 when n is a multiple of 3.

a1 = 12 = 1 1 is not a multiple of 3. Use n2.

a2 = 22 = 4 2 is not a multiple of 3. Use n2.

a3 = 33 = 1 3 is a multiple of 3. Use n

3 .

a4 = 42 = 16 4 is not a multiple of 3. Use n2.

a5 = 52 = 25 5 is not a multiple of 3. Use n2.

a6 = 63 = 2 6 is a multiple of 3. Use n

3 .

(9.13)

The �rst six terms are {1, 4, 1, 16, 25, 2}.AnalysisEvery third point on the graph shown in Figure 9.5 stands out from the two nearby points. Thisoccurs because the sequence was de�ned by a piecewise function.

Available for free at Connexions <http://legacy.cnx.org/content/col11759/1.2>

1006 CHAPTER 9. SEQUENCES, PROBABILITY, AND COUNTING THEORY

Figure 9.5

Try It:

Exercise 9.2.3 (Solution on p. 1120.)

Write the �rst six terms of the sequence.

an = {2n3 if n is odd

5n2 if n is even

(9.14)

9.2.1.3 Finding an Explicit Formula

Thus far, we have been given the explicit formula and asked to �nd a number of terms of the sequence.Sometimes, the explicit formula for thenth term of a sequence is not given. Instead, we are given severalterms from the sequence. When this happens, we can work in reverse to �nd an explicit formula from the �rstfew terms of a sequence. The key to �nding an explicit formula is to look for a pattern in the terms. Keep inmind that the pattern may involve alternating terms, formulas for numerators, formulas for denominators,exponents, or bases.

Available for free at Connexions <http://legacy.cnx.org/content/col11759/1.2>

1007

How To: Given the �rst few terms of a sequence, �nd an explicit formula for thesequence.

1.Look for a pattern among the terms.2.If the terms are fractions, look for a separate pattern among the numerators and denominators.3.Look for a pattern among the signs of the terms.4.Write a formula for an in terms of n. Test your formula for n = 1, n = 2, and n = 3.

Example 9.4Writing an Explicit Formula for the nth Term of a SequenceWrite an explicit formula for the nth term of each sequence.

a. {− 211 ,

313 ,−

415 ,

517 ,−

619 , . . . }

b. {− 225 ,−

2125 ,−

2625 ,−

23,125 ,−

215,625 , . . . }

c. {e4,e5,e6,e7,e8, . . . }

SolutionLook for the pattern in each sequence.

a. The terms alternate between positive and negative. We can use (−1)n to make the termsalternate. The numerator can be represented by n+ 1. The denominator can be represented

by 2n+ 9. an = (−1)n(n+1)2n+9

b. The terms are all negative.

So we know that the fraction is negative, the numerator is 2, and the denominator can berepresented by 5n+1.

an = − 25n+1

(9.15)

c. The terms are powers of e. For n = 1, the �rst term is e4 so the exponent must be n+ 3.

an = en+3 (9.16)

Try It:

Exercise 9.2.4 (Solution on p. 1120.)

Write an explicit formula for the nth term of the sequence.

{9, − 81, 729, − 6,561, 59,049, . . .} (9.17)

Available for free at Connexions <http://legacy.cnx.org/content/col11759/1.2>

1008 CHAPTER 9. SEQUENCES, PROBABILITY, AND COUNTING THEORY

Try It:

Exercise 9.2.5 (Solution on p. 1120.)

Write an explicit formula for thenth term of the sequence.

{−34,−9

8,−27

12,−81

16,−243

20, ...} (9.18)

Try It:

Exercise 9.2.6 (Solution on p. 1120.)

Write an explicit formula for thenth term of the sequence.

{ 1e2,

1e, 1, e, e2, ...} (9.19)

9.2.2 Writing the Terms of a Sequence De�ned by a Recursive Formula

Sequences occur naturally in the growth patterns of nautilus shells, pinecones, tree branches, and many othernatural structures. We may see the sequence in the leaf or branch arrangement, the number of petals of a�ower, or the pattern of the chambers in a nautilus shell. Their growth follows the Fibonacci sequence, afamous sequence in which each term can be found by adding the preceding two terms. The numbers in thesequence are 1, 1, 2, 3, 5, 8, 13, 21, 34,. . .. Other examples from the natural world that exhibit the Fibonaccisequence are the Calla Lily, which has just one petal, the Black-Eyed Susan with 13 petals, and di�erentvarieties of daisies that may have 21 or 34 petals.

Each term of the Fibonacci sequence depends on the terms that come before it. The Fibonacci sequencecannot easily be written using an explicit formula. Instead, we describe the sequence using a recursiveformula, a formula that de�nes the terms of a sequence using previous terms.

A recursive formula always has two parts: the value of an initial term (or terms), and an equation de�ningan in terms of preceding terms. For example, suppose we know the following:

a1 = 3

an = 2an−1 − 1, for n ≥ 2(9.20)

We can �nd the subsequent terms of the sequence using the �rst term.

a1 = 3

a2 = 2a1 − 1 = 2 (3)− 1 = 5

a3 = 2a2 − 1 = 2 (5)− 1 = 9

a4 = 2a3 − 1 = 2 (9)− 1 = 17

(9.21)

So the �rst four terms of the sequence are {3, 5, 9, 17}.The recursive formula for the Fibonacci sequence states the �rst two terms and de�nes each successive

term as the sum of the preceding two terms.

Available for free at Connexions <http://legacy.cnx.org/content/col11759/1.2>

1009

a1 = 1

a2 = 1

an = an−1 + an−2, for n ≥ 3

(9.22)

To �nd the tenth term of the sequence, for example, we would need to add the eighth and ninth terms. Wewere told previously that the eighth and ninth terms are 21 and 34, so

a10 = a9 + a8 = 34 + 21 = 55 (9.23)

A General Note: A recursive formula is a formula that de�nes each term of a sequence usingpreceding term(s). Recursive formulas must always state the initial term, or terms, of the sequence.

Q&A: Must the �rst two terms always be given in a recursive formula?

No. The Fibonacci sequence de�nes each term using the two preceding terms, but many recursiveformulas de�ne each term using only one preceding term. These sequences need only the �rst termto be de�ned.

How To: Given a recursive formula with only the �rst term provided, write the �rstn terms of a sequence.

1.Identify the initial term, a1, which is given as part of the formula. This is the �rst term.2.To �nd the second term, a2, substitute the initial term into the formula for an−1. Solve.3.To �nd the third term, a3, substitute the second term into the formula. Solve.4.Repeat until you have solved for the nth term.

Example 9.5Writing the Terms of a Sequence De�ned by a Recursive FormulaWrite the �rst �ve terms of the sequence de�ned by the recursive formula.

a1 = 9

an = 3an−1 − 20, for n ≥ 2

(9.24)

Solution

The �rst term is given in the formula. For each subsequent term, we replace an−1 with the valueof the preceding term.

n = 1 a1 = 9

n = 2 a2 = 3a1 − 20 = 3 (9)− 20 = 27− 20 = 7

n = 3 a3 = 3a2 − 20 = 3 (7)− 20 = 21− 20 = 1

n = 4 a4 = 3a3 − 20 = 3 (1)− 20 = 3− 20 = −17

n = 5 a5 = 3a4 − 20 = 3 (−17)− 20 = −51− 20 = −71

(9.25)

The �rst �ve terms are {9, 7, 1, −−17, −−71}. See Figure 9.6.

Available for free at Connexions <http://legacy.cnx.org/content/col11759/1.2>

1010 CHAPTER 9. SEQUENCES, PROBABILITY, AND COUNTING THEORY

Figure 9.6

Try It:

Exercise 9.2.7 (Solution on p. 1120.)

Write the �rst �ve terms of the sequence de�ned by the recursive formula.

a1 = 2

an = 2an−1 + 1, for n ≥ 2(9.26)

How To: Given a recursive formula with two initial terms, write the �rst n terms ofa sequence.

1.Identify the initial term, a1, which is given as part of the formula.2.Identify the second term, a2, which is given as part of the formula.3.To �nd the third term, substitute the initial term and the second term into the formula.Evaluate.

4.Repeat until you have evaluated the nth term.

Example 9.6Writing the Terms of a Sequence De�ned by a Recursive FormulaWrite the �rst six terms of the sequence de�ned by the recursive formula.

Available for free at Connexions <http://legacy.cnx.org/content/col11759/1.2>

1011

a1 = 1

a2 = 2

an = 3an−1 + 4an−2, for n ≥ 3

(9.27)

Solution

The �rst two terms are given. For each subsequent term, we replace an−1 and an−2 with the valuesof the two preceding terms.

n = 3 a3 = 3a2 + 4a1 = 3 (2) + 4 (1) = 10

n = 4 a4 = 3a3 + 4a2 = 3 (10) + 4 (2) = 38

n = 5 a5 = 3a4 + 4a3 = 3 (38) + 4 (10) = 154

n = 6 a6 = 3a5 + 4a4 = 3 (154) + 4 (38) = 614

(9.28)

The �rst six terms are {1,2,10,38,154,614}. See Figure 9.7.

Figure 9.7

Available for free at Connexions <http://legacy.cnx.org/content/col11759/1.2>

1012 CHAPTER 9. SEQUENCES, PROBABILITY, AND COUNTING THEORY

Try It:

Exercise 9.2.8 (Solution on p. 1120.)

Write the �rst 8 terms of the sequence de�ned by the recursive formula.

a1 = 0

a2 = 1

a3 = 1

an = an−1an−2

+ an−3, for n ≥ 4

(9.29)

9.2.3 Using Factorial Notation

The formulas for some sequences include products of consecutive positive integers. n factorial, written asn!, is the product of the positive integers from 1 to n. For example,

4! = 4 · 3 · 2 · 1 = 24

5! = 5 · 4 · 3 · 2 · 1 = 120(9.30)

An example of formula containing a factorial is an = (n+ 1)!. The sixth term of the sequence can be foundby substituting 6 for n.

a6 = (6 + 1)! = 7! = 7 · 6 · 5 · 4 · 3 · 2 · 1 = 5040 (9.31)

The factorial of any whole number n is n (n− 1)! We can therefore also think of 5! as 5 · 4!.

A General Note: n factorial is a mathematical operation that can be de�ned using a recursiveformula. The factorial ofn, denotedn!, is de�ned for a positive integer n as:

0! = 1

1! = 1

n! = n (n− 1) (n− 2) · · · (2) (1) , for n ≥ 2

(9.32)

The special case 0! is de�ned as 0! = 1.

Q&A: Can factorials always be found using a calculator?

No. Factorials get large very quickly�faster than even exponential functions! When the outputgets too large for the calculator, it will not be able to calculate the factorial.

Example 9.7Writing the Terms of a Sequence Using FactorialsWrite the �rst �ve terms of the sequence de�ned by the explicit formula an = 5n

(n+2)! .

SolutionSubstituten = 1, n = 2, and so on in the formula.

Available for free at Connexions <http://legacy.cnx.org/content/col11759/1.2>

1013

n = 1 a1 = 5(1)(1+2)! = 5

3! = 53·2·1 = 5

6

n = 2 a2 = 5(2)(2+2)! = 10

4! = 104·3·2·1 = 5

12

n = 3 a3 = 5(3)(3+2)! = 15

5! = 155·4·3·2·1 = 1

8

n = 4 a4 = 5(4)(4+2)! = 20

6! = 206·5·4·3·2·1 = 1

36

n = 5 a5 = 5(5)(5+2)! = 25

7! = 257·6·5·4·3·2·1 = 5

1,008

(9.33)

The �rst �ve terms are { 56 ,

512 ,

18 ,

136 ,

51,008}.

AnalysisFigure 9.8 shows the graph of the sequence. Notice that, since factorials grow very quickly, thepresence of the factorial term in the denominator results in the denominator becoming much largerthan the numerator as n increases. This means the quotient gets smaller and, as the plot of theterms shows, the terms are decreasing and nearing zero.

Figure 9.8

Try It:

Exercise 9.2.9 (Solution on p. 1120.)

Write the �rst �ve terms of the sequence de�ned by the explicit formula an = (n+1)!2n .

Available for free at Connexions <http://legacy.cnx.org/content/col11759/1.2>

1014 CHAPTER 9. SEQUENCES, PROBABILITY, AND COUNTING THEORY

Media: Access this online resource for additional instruction and practice with sequences.

• Finding Terms in a Sequence3

9.2.4 Key Equations

Formula for a factorial

0! = 1

1! = 1

n! = n (n− 1) (n− 2) · · · (2) (1) , for n ≥ 2

Table 9.4

9.2.5 Key Concepts

• A sequence is a list of numbers, called terms, written in a speci�c order.• Explicit formulas de�ne each term of a sequence using the position of the term. See Example 9.1,

Example 9.2, and Example 9.3.• An explicit formula for thenth term of a sequence can be written by analyzing the pattern of several

terms. See Example 9.4.• Recursive formulas de�ne each term of a sequence using previous terms.• Recursive formulas must state the initial term, or terms, of a sequence.• A set of terms can be written by using a recursive formula. See Example 9.5 and Example 9.6.• A factorial is a mathematical operation that can be de�ned recursively.• The factorial ofn is the product of all integers from 1 ton See Example 9.7.

9.2.6 Section Exercises

9.2.6.1 Verbal

Exercise 9.2.10 (Solution on p. 1120.)

Discuss the meaning of a sequence. If a �nite sequence is de�ned by a formula, what is its domain?What about an in�nite sequence?

Exercise 9.2.11Describe three ways that a sequence can be de�ned.

Exercise 9.2.12 (Solution on p. 1120.)

Is the ordered set of even numbers an in�nite sequence? What about the ordered set of oddnumbers? Explain why or why not.

Exercise 9.2.13What happens to the terms an of a sequence when there is a negative factor in the formula thatis raised to a power that includes n? What is the term used to describe this phenomenon?

Exercise 9.2.14 (Solution on p. 1120.)

What is a factorial, and how is it denoted? Use an example to illustrate how factorial notationcan be bene�cial.

3http://openstaxcollege.org/l/�ndingterms

Available for free at Connexions <http://legacy.cnx.org/content/col11759/1.2>

1015

9.2.6.2 Algebraic

For the following exercises, write the �rst four terms of the sequence.

Exercise 9.2.15an = 2n − 2Exercise 9.2.16 (Solution on p. 1120.)

an = − 16n+1

Exercise 9.2.17an = −(−5)n−1

Exercise 9.2.18 (Solution on p. 1120.)

an = 2n

n3

Exercise 9.2.19an = 2n+1

n3

Exercise 9.2.20 (Solution on p. 1120.)

an = 1.25 · (−4)n−1

Exercise 9.2.21an = −4 · (−6)n−1

Exercise 9.2.22 (Solution on p. 1120.)

an = n2

2n+1

Exercise 9.2.23an = (−10)n + 1Exercise 9.2.24 (Solution on p. 1120.)

an = −(

4·(−5)n−1

5

)For the following exercises, write the �rst eight terms of the piecewise sequence.

Exercise 9.2.25

an = {(−2)n − 2 if n is even

(3)n−1if n is odd

Exercise 9.2.26 (Solution on p. 1120.)

an = {n2

2n+1 if n ≤ 5

n2 − 5 if n >5

Exercise 9.2.27

an = {(2n+ 1)2

if n is divisible by 4

2n if n is not divisible by 4

Exercise 9.2.28 (Solution on p. 1120.)

an = {−0.6 · 5n−1 if n is prime or 1

2.5 · (−2)n−1if n is composite

Exercise 9.2.29

an = {4(n2 − 2

)if n ≤ 3 or n > 6

n2−24 if 3 < n ≤ 6

For the following exercises, write an explicit formula for each sequence.

Exercise 9.2.30 (Solution on p. 1120.)

4, 7, 12, 19, 28, . . .

Available for free at Connexions <http://legacy.cnx.org/content/col11759/1.2>

1016 CHAPTER 9. SEQUENCES, PROBABILITY, AND COUNTING THEORY

Exercise 9.2.31−4, 2,−10, 14,−34, . . .Exercise 9.2.32 (Solution on p. 1120.)

1, 1, 43 , 2,

165 , . . .

Exercise 9.2.330, 1−e1

1+e2 ,1−e21+e3 ,

1−e31+e4 ,

1−e41+e5 , . . .

Exercise 9.2.34 (Solution on p. 1120.)

1,− 12 ,

14 ,−

18 ,

116 , . . .

For the following exercises, write the �rst �ve terms of the sequence.

Exercise 9.2.35a1 = 9, an = an−1 + n

Exercise 9.2.36 (Solution on p. 1120.)

a1 = 3, an = (−3) an−1

Exercise 9.2.37a1 = −4, an = an−1+2n

an−1−1

Exercise 9.2.38 (Solution on p. 1121.)

a1 = −1, an = (−3)n−1

an−1−2

Exercise 9.2.39a1 = −30, an = (2 + an−1)

(12

)nFor the following exercises, write the �rst eight terms of the sequence.

Exercise 9.2.40 (Solution on p. 1121.)

a1 = 124 , a2 = 1, an = (2an−2) (3an−1)

Exercise 9.2.41a1 = −1, a2 = 5, an = an−2 (3− an−1)Exercise 9.2.42 (Solution on p. 1121.)

a1 = 2, a2 = 10, an = 2(an−1+2)an−2

For the following exercises, write a recursive formula for each sequence.

Exercise 9.2.43−2.5,−5,−10,−20,−40, . . .Exercise 9.2.44 (Solution on p. 1121.)

−8,−6,−3, 1, 6, . . .Exercise 9.2.452, 4, 12, 48, 240, . . .Exercise 9.2.46 (Solution on p. 1121.)

35, 38, 41, 44, 47, . . .Exercise 9.2.4715, 3, 3

5 ,325 ,

3125 , · · ·

For the following exercises, evaluate the factorial.

Exercise 9.2.48 (Solution on p. 1121.)

6!Exercise 9.2.49(

126

)!

Exercise 9.2.50 (Solution on p. 1121.)12!6!

Available for free at Connexions <http://legacy.cnx.org/content/col11759/1.2>

1017

Exercise 9.2.51100!99!

For the following exercises, write the �rst four terms of the sequence.

Exercise 9.2.52 (Solution on p. 1121.)

an = n!n2

Exercise 9.2.53an = 3·n!

4·n!

Exercise 9.2.54 (Solution on p. 1121.)

an = n!n2−n−1

Exercise 9.2.55an = 100·n

n(n−1)!

9.2.6.3 Graphical

For the following exercises, graph the �rst �ve terms of the indicated sequence

Exercise 9.2.56 (Solution on p. 1121.)

an = (−1)n

n + n

Exercise 9.2.57

an = {4+n2n if n is even

3 + n if n is odd

Exercise 9.2.58 (Solution on p. 1121.)

a1 = 2, an = (−an−1 + 1)2

Exercise 9.2.59an = 1, an = an−1 + 8Exercise 9.2.60 (Solution on p. 1122.)

an = (n+1)!(n−1)!

For the following exercises, write an explicit formula for the sequence using the �rst �ve points shown onthe graph.

Available for free at Connexions <http://legacy.cnx.org/content/col11759/1.2>

1018 CHAPTER 9. SEQUENCES, PROBABILITY, AND COUNTING THEORY

Exercise 9.2.61

Exercise 9.2.62 (Solution on p. 1122.)

Available for free at Connexions <http://legacy.cnx.org/content/col11759/1.2>

1019

Exercise 9.2.63

For the following exercises, write a recursive formula for the sequence using the �rst �ve points shown onthe graph.

Available for free at Connexions <http://legacy.cnx.org/content/col11759/1.2>

1020 CHAPTER 9. SEQUENCES, PROBABILITY, AND COUNTING THEORY

Exercise 9.2.64 (Solution on p. 1122.)

Available for free at Connexions <http://legacy.cnx.org/content/col11759/1.2>

1021

Exercise 9.2.65

9.2.6.4 Technology

Follow these steps to evaluate a sequence de�ned recursively using a graphing calculator:

• On the home screen, key in the value for the initial term a1 and press [ENTER].• Enter the recursive formula by keying in all numerical values given in the formula, along with the key

strokes [2ND] ANS for the previous term an−1.Press [ENTER].• Continue pressing [ENTER] to calculate the values for each successive term.

For the following exercises, use the steps above to �nd the indicated term or terms for the sequence.

Exercise 9.2.66 (Solution on p. 1122.)

Find the �rst �ve terms of the sequence a1 = 87111 , an = 4

3an−1 + 1237 . Use the >Frac feature to

give fractional results.

Exercise 9.2.67Find the 15th term of the sequence a1 = 625, an = 0.8an−1 + 18.Exercise 9.2.68 (Solution on p. 1122.)

Find the �rst �ve terms of the sequence a1 = 2, an = 2[(an−1)−1] + 1.Exercise 9.2.69Find the �rst ten terms of the sequence a1 = 8, an = (an−1+1)!

an−1! .

Exercise 9.2.70 (Solution on p. 1122.)

Find the tenth term of the sequence a1 = 2, an = nan−1

Follow these steps to evaluate a �nite sequence de�ned by an explicit formula. Using a TI-84, do the following.

Available for free at Connexions <http://legacy.cnx.org/content/col11759/1.2>

1022 CHAPTER 9. SEQUENCES, PROBABILITY, AND COUNTING THEORY

• In the home screen, press [2ND] LIST.• Scroll over to OPS and choose �seq(� from the dropdown list. Press [ENTER].• In the line headed �Expr:� type in the explicit formula, using the [X,T, θ, n] button forn• In the line headed �Variable:� type in the variable used on the previous step.• In the line headed �start:� key in the value ofn that begins the sequence.• In the line headed �end:� key in the value ofn that ends the sequence.• Press [ENTER] 3 times to return to the home screen. You will see the sequence syntax on the screen.

Press [ENTER] to see the list of terms for the �nite sequence de�ned. Use the right arrow key toscroll through the list of terms.

Using a TI-83, do the following.

• In the home screen, press [2ND] LIST.• Scroll over to OPS and choose �seq(� from the dropdown list. Press [ENTER].• Enter the items in the order �Expr� , �Variable� , �start� , �end� separated by commas. See the

instructions above for the description of each item.• Press [ENTER] to see the list of terms for the �nite sequence de�ned. Use the right arrow key to

scroll through the list of terms.

For the following exercises, use the steps above to �nd the indicated terms for the sequence. Round to thenearest thousandth when necessary.

Exercise 9.2.71List the �rst �ve terms of the sequence an = − 28

9 n+ 53 .

Exercise 9.2.72 (Solution on p. 1122.)

List the �rst six terms of the sequence an = n3−3.5n2+ 4.1n−1.52.4n .

Exercise 9.2.73List the �rst �ve terms of the sequence an = 15n·(−2)n−1

47

Exercise 9.2.74 (Solution on p. 1122.)

List the �rst four terms of the sequence an = 5.7n + 0.275 (n− 1)!Exercise 9.2.75List the �rst six terms of the sequence an = n!

n .

9.2.6.5 Extensions

Exercise 9.2.76 (Solution on p. 1122.)

Consider the sequence de�ned by an = −6− 8n. Is an = −421 a term in the sequence? Verify theresult.

Exercise 9.2.77What term in the sequence an = n2+4n+4

2(n+2) has the value 41? Verify the result.

Exercise 9.2.78 (Solution on p. 1123.)

Find a recursive formula for the sequence 1, 0, − 1, − 1, 0, 1, 1, 0, − 1, − 1, 0, 1, 1, ... .(Hint: �nd a pattern for an based on the �rst two terms.)

Exercise 9.2.79Calculate the �rst eight terms of the sequences an = (n+2)!

(n−1)! and bn = n3 + 3n2 + 2n, and then

make a conjecture about the relationship between these two sequences.

Exercise 9.2.80 (Solution on p. 1123.)

Prove the conjecture made in the preceding exercise.

Available for free at Connexions <http://legacy.cnx.org/content/col11759/1.2>

1023

9.3 Arithmetic Sequences4

Companies often make large purchases, such as computers and vehicles, for business use. The book-valueof these supplies decreases each year for tax purposes. This decrease in value is called depreciation. Onemethod of calculating depreciation is straight-line depreciation, in which the value of the asset decreases bythe same amount each year.

As an example, consider a woman who starts a small contracting business. She purchases a new truckfor $25,000. After �ve years, she estimates that she will be able to sell the truck for $8,000. The loss invalue of the truck will therefore be $17,000, which is $3,400 per year for �ve years. The truck will be worth$21,600 after the �rst year; $18,200 after two years; $14,800 after three years; $11,400 after four years; and$8,000 at the end of �ve years. In this section, we will consider speci�c kinds of sequences that will allow usto calculate depreciation, such as the truck's value.

9.3.1 Finding Common Di�erences

The values of the truck in the example are said to form an arithmetic sequence because they changeby a constant amount each year. Each term increases or decreases by the same constant value called thecommon di�erence of the sequence. For this sequence, the common di�erence is �3,400.

The sequence below is another example of an arithmetic sequence. In this case, the constant di�erenceis 3. You can choose any term of the sequence, and add 3 to �nd the subsequent term.

A General Note: An arithmetic sequence is a sequence that has the property that thedi�erence between any two consecutive terms is a constant. This constant is called the commondi�erence. If a1 is the �rst term of an arithmetic sequence and d is the common di�erence, thesequence will be:

{an} = {a1, a1 + d, a1 + 2d, a1 + 3d, ...} (9.34)

Example 9.8Finding Common Di�erencesIs each sequence arithmetic? If so, �nd the common di�erence.

a. {1, 2, 4, 8, 16, ...}b. {−3, 1, 5, 9, 13, ...}

SolutionSubtract each term from the subsequent term to determine whether a common di�erence exists.

4This content is available online at <http://legacy.cnx.org/content/m49445/1.7/>.

Available for free at Connexions <http://legacy.cnx.org/content/col11759/1.2>

1024 CHAPTER 9. SEQUENCES, PROBABILITY, AND COUNTING THEORY

a. The sequence is not arithmetic because there is no common di�erence.

b. The sequence is arithmetic because there is a common di�erence. The common di�erence is4.

AnalysisThe graph of each of these sequences is shown in Figure 9.9. We can see from the graphs that,although both sequences show growth, a is not linear whereas b is linear. Arithmetic sequenceshave a constant rate of change so their graphs will always be points on a line.

Figure 9.9

Q&A: If we are told that a sequence is arithmetic, do we have to subtract every termfrom the following term to �nd the common di�erence?

No. If we know that the sequence is arithmetic, we can choose any one term in the sequence, andsubtract it from the subsequent term to �nd the common di�erence.

Try It:

Exercise 9.3.1 (Solution on p. 1123.)

Is the given sequence arithmetic? If so, �nd the common di�erence.

{18, 16, 14, 12, 10, . . . } (9.35)

Available for free at Connexions <http://legacy.cnx.org/content/col11759/1.2>

1025

Try It:

Exercise 9.3.2 (Solution on p. 1123.)

Is the given sequence arithmetic? If so, �nd the common di�erence.

{1, 3, 6, 10, 15, . . . } (9.36)

9.3.2 Writing Terms of Arithmetic Sequences

Now that we can recognize an arithmetic sequence, we will �nd the terms if we are given the �rst term andthe common di�erence. The terms can be found by beginning with the �rst term and adding the commondi�erence repeatedly. In addition, any term can also be found by plugging in the values of n and d intoformula below.

an = a1 + (n− 1) d (9.37)

How To: Given the �rst term and the common di�erence of an arithmetic sequence,�nd the �rst several terms.

1.Add the common di�erence to the �rst term to �nd the second term.2.Add the common di�erence to the second term to �nd the third term.3.Continue until all of the desired terms are identi�ed.4.Write the terms separated by commas within brackets.

Example 9.9Writing Terms of Arithmetic SequencesWrite the �rst �ve terms of the arithmetic sequence with a1 = 17 and d = −3.

SolutionAdding − 3 is the same as subtracting 3. Beginning with the �rst term, subtract 3 from each termto �nd the next term.

The �rst �ve terms are {17, 14, 11, 8, 5}AnalysisAs expected, the graph of the sequence consists of points on a line as shown in Figure 9.10.

Available for free at Connexions <http://legacy.cnx.org/content/col11759/1.2>

1026 CHAPTER 9. SEQUENCES, PROBABILITY, AND COUNTING THEORY

Figure 9.10

Try It:

Exercise 9.3.3 (Solution on p. 1123.)

List the �rst �ve terms of the arithmetic sequence with a1 = 1 and d = 5.

How To: Given any the �rst term and any other term in an arithmetic sequence, �nda given term.

1.Substitute the values given for a1, an, n into the formula an = a1 + (n− 1) d to solve for d.2.Find a given term by substituting the appropriate values for a1, n, and d into the formulaan =a1 + (n− 1) d.

Example 9.10Writing Terms of Arithmetic SequencesGiven a1 = 8 and a4 = 14, �nd a5.

SolutionThe sequence can be written in terms of the initial term 8 and the common di�erence d.

{8, 8 + d, 8 + 2d, 8 + 3d} (9.38)

We know the fourth term equals 14; we know the fourth term has the form a1 + 3d = 8 + 3d.We can �nd the common di�erence d.

Available for free at Connexions <http://legacy.cnx.org/content/col11759/1.2>

1027

an = a1 + (n− 1) d

a4 = a1 + 3d

a4 = 8 + 3d Write the fourth term of the sequence in terms of a1 and d.

14 = 8 + 3d Substitute 14 for a4.

d = 2 Solve for the common di�erence.

(9.39)

Find the �fth term by adding the common di�erence to the fourth term.

a5 = a4 + 2 = 16 (9.40)

AnalysisNotice that the common di�erence is added to the �rst term once to �nd the second term, twiceto �nd the third term, three times to �nd the fourth term, and so on. The tenth term couldbe found by adding the common di�erence to the �rst term nine times or by using the equationan = a1 + (n− 1) d.

Try It:

Exercise 9.3.4 (Solution on p. 1123.)

Given a3 = 7 and a5 = 17, �nd a2.

9.3.3 Using Recursive Formulas for Arithmetic Sequences

Some arithmetic sequences are de�ned in terms of the previous term using a recursive formula. Theformula provides an algebraic rule for determining the terms of the sequence. A recursive formula allows usto �nd any term of an arithmetic sequence using a function of the preceding term. Each term is the sum ofthe previous term and the common di�erence. For example, if the common di�erence is 5, then each termis the previous term plus 5. As with any recursive formula, the �rst term must be given.

an = an−1 + d n ≥ 2 (9.41)

A General Note: The recursive formula for an arithmetic sequence with common di�erence dis:

an = an−1 + d n ≥ 2 (9.42)

How To: Given an arithmetic sequence, write its recursive formula.

1.Subtract any term from the subsequent term to �nd the common di�erence.2.State the initial term and substitute the common di�erence into the recursive formula forarithmetic sequences.

Available for free at Connexions <http://legacy.cnx.org/content/col11759/1.2>

1028 CHAPTER 9. SEQUENCES, PROBABILITY, AND COUNTING THEORY

Example 9.11Writing a Recursive Formula for an Arithmetic SequenceWrite a recursive formula for the arithmetic sequence.

{−18, − 7, 4, 15, 26, . . .} (9.43)

Solution

The �rst term is given as −18. The common di�erence can be found by subtracting the �rst termfrom the second term.

d = −7− (−18) = 11 (9.44)

Substitute the initial term and the common di�erence into the recursive formula for arithmeticsequences.

a1 = −18

an = an−1 + 11, for n ≥ 2(9.45)

AnalysisWe see that the common di�erence is the slope of the line formed when we graph the terms ofthe sequence, as shown in Figure 9.11. The growth pattern of the sequence shows the constantdi�erence of 11 units.

Figure 9.11

Available for free at Connexions <http://legacy.cnx.org/content/col11759/1.2>

1029

Q&A: Do we have to subtract the �rst term from the second term to �nd the commondi�erence?

No. We can subtract any term in the sequence from the subsequent term. It is, however, mostcommon to subtract the �rst term from the second term because it is often the easiest method of�nding the common di�erence.

Try It:

Exercise 9.3.5 (Solution on p. 1123.)

Write a recursive formula for the arithmetic sequence.

{25, 37, 49, 61, . . .} (9.46)

9.3.4 Using Explicit Formulas for Arithmetic Sequences

We can think of an arithmetic sequence as a function on the domain of the natural numbers; it is a linearfunction because it has a constant rate of change. The common di�erence is the constant rate of change,or the slope of the function. We can construct the linear function if we know the slope and the verticalintercept.

an = a1 + d (n− 1) (9.47)

To �nd the y-intercept of the function, we can subtract the common di�erence from the �rst term of thesequence. Consider the following sequence.

The common di�erence is −50, so the sequence represents a linear function with a slope of −50. To �ndthe y-intercept, we subtract −50 from200 : 200− (−50) = 200 + 50 = 250. You can also �nd the y-interceptby graphing the function and determining where a line that connects the points would intersect the verticalaxis. The graph is shown in Figure 9.12.

Available for free at Connexions <http://legacy.cnx.org/content/col11759/1.2>

1030 CHAPTER 9. SEQUENCES, PROBABILITY, AND COUNTING THEORY

Figure 9.12

Recall the slope-intercept form of a line is y = mx+ b.When dealing with sequences, we use an in placeof y and n in place of x. If we know the slope and vertical intercept of the function, we can substitute themfor m and b in the slope-intercept form of a line. Substituting − 50 for the slope and 250 for the verticalintercept, we get the following equation:

an = −50n+ 250 (9.48)

We do not need to �nd the vertical intercept to write an explicit formula for an arithmetic sequence.Another explicit formula for this sequence is an = 200− 50 (n− 1), which simpli�es to an = −50n+ 250.

A General Note: An explicit formula for the nth term of an arithmetic sequence is given by

an = a1 + d (n− 1) (9.49)

How To: Given the �rst several terms for an arithmetic sequence, write an explicitformula.

1.Find the common di�erence, a2 − a1.2.Substitute the common di�erence and the �rst term into an = a1 + d (n− 1) .

Example 9.12Writing the nth Term Explicit Formula for an Arithmetic SequenceWrite an explicit formula for the arithmetic sequence.

{2, 12, 22, 32, 42, . . .} (9.50)

Solution

The common di�erence can be found by subtracting the �rst term from the second term.

d = a2 − a1

= 12− 2

= 10

(9.51)

Available for free at Connexions <http://legacy.cnx.org/content/col11759/1.2>

1031

The common di�erence is 10. Substitute the common di�erence and the �rst term of the sequenceinto the formula and simplify.

an = 2 + 10 (n− 1)

an = 10n− 8(9.52)

AnalysisThe graph of this sequence, represented in Figure 9.13, shows a slope of 10 and a vertical interceptof −8.

Figure 9.13

Try It:

Exercise 9.3.6 (Solution on p. 1123.)

Write an explicit formula for the following arithmetic sequence.

{50, 47, 44, 41, . . . } (9.53)

Available for free at Connexions <http://legacy.cnx.org/content/col11759/1.2>

1032 CHAPTER 9. SEQUENCES, PROBABILITY, AND COUNTING THEORY

9.3.4.1 Finding the Number of Terms in a Finite Arithmetic Sequence

Explicit formulas can be used to determine the number of terms in a �nite arithmetic sequence. We need to�nd the common di�erence, and then determine how many times the common di�erence must be added tothe �rst term to obtain the �nal term of the sequence.

How To: Given the �rst three terms and the last term of a �nite arithmetic sequence,�nd the total number of terms.

1.Find the common di�erence d.2.Substitute the common di�erence and the �rst term into an = a1 + d (n−−1) .3.Substitute the last term for an and solve for n.

Example 9.13Finding the Number of Terms in a Finite Arithmetic SequenceFind the number of terms in the �nite arithmetic sequence.

{8, 1, −−6, ..., −−41} (9.54)

Solution

The common di�erence can be found by subtracting the �rst term from the second term.

1− 8 = −7 (9.55)

The common di�erence is−7. Substitute the common di�erence and the initial term of the sequenceinto the nth term formula and simplify.

an = a1 + d (n− 1)

an = 8 +−7 (n− 1)

an = 15− 7n

(9.56)

Substitute −41 for an and solve for n

−41 = 15− 7n

8 = n(9.57)

There are eight terms in the sequence.

Try It:

Exercise 9.3.7 (Solution on p. 1123.)

Find the number of terms in the �nite arithmetic sequence.

{6, 11, 16, ..., 56} (9.58)

Available for free at Connexions <http://legacy.cnx.org/content/col11759/1.2>

1033

9.3.4.2 Solving Application Problems with Arithmetic Sequences

In many application problems, it often makes sense to use an initial term of a0 instead of a1. In theseproblems, we alter the explicit formula slightly to account for the di�erence in initial terms. We use thefollowing formula:

an = a0 + dn (9.59)

Example 9.14Solving Application Problems with Arithmetic SequencesA �ve-year old child receives an allowance of $1 each week. His parents promise him an annualincrease of $2 per week.

a. Write a formula for the child's weekly allowance in a given year.b. What will the child's allowance be when he is 16 years old?

Solution

a. The situation can be modeled by an arithmetic sequence with an initial term of 1 and acommon di�erence of 2.Let A be the amount of the allowance and n be the number of years after age 5. Using thealtered explicit formula for an arithmetic sequence we get:

An = 1 + 2n (9.60)

b. We can �nd the number of years since age 5 by subtracting.

16− 5 = 11 (9.61)

We are looking for the child's allowance after 11 years. Substitute 11 into the formula to �ndthe child's allowance at age 16.

A11 = 1 + 2 (11) = 23 (9.62)

The child's allowance at age 16 will be $23 per week.

Try It:

Exercise 9.3.8 (Solution on p. 1123.)

A woman decides to go for a 10-minute run every day this week and plans to increasethe time of her daily run by 4 minutes each week. Write a formula for the time of her runafter n weeks. How long will her daily run be 8 weeks from today?

Media: Access this online resource for additional instruction and practice with arithmetic se-quences.

• Arithmetic Sequences5

5http://openstaxcollege.org/l/arithmeticseq

Available for free at Connexions <http://legacy.cnx.org/content/col11759/1.2>

1034 CHAPTER 9. SEQUENCES, PROBABILITY, AND COUNTING THEORY

9.3.5 Key Equations

recursive formula for nth term of an arithmetic sequence an = an−1 + dn ≥ 2

explicit formula for nth term of an arithmetic sequence an = a1 + d (n− 1)

Table 9.5

9.3.6 Key Concepts

• An arithmetic sequence is a sequence where the di�erence between any two consecutive terms is aconstant.

• The constant between two consecutive terms is called the common di�erence.• The common di�erence is the number added to any one term of an arithmetic sequence that generates

the subsequent term. See Example 9.8.• The terms of an arithmetic sequence can be found by beginning with the initial term and adding the

common di�erence repeatedly. See Example 9.9 and Example 9.10.• A recursive formula for an arithmetic sequence with common di�erence d is given by an = an−1 +d, n ≥

2. See Example 9.11.• As with any recursive formula, the initial term of the sequence must be given.• An explicit formula for an arithmetic sequence with common di�erence d is given by an = a1+d (n− 1) .

See Example 9.12.• An explicit formula can be used to �nd the number of terms in a sequence. See Example 9.13.• In application problems, we sometimes alter the explicit formula slightly to an = a0 + dn. See Exam-

ple 9.14.

9.3.7 Section Exercises

9.3.7.1 Verbal

Exercise 9.3.9 (Solution on p. 1123.)

What is an arithmetic sequence?

Exercise 9.3.10How is the common di�erence of an arithmetic sequence found?

Exercise 9.3.11 (Solution on p. 1123.)

How do we determine whether a sequence is arithmetic?

Exercise 9.3.12What are the main di�erences between using a recursive formula and using an explicit formula todescribe an arithmetic sequence?

Exercise 9.3.13 (Solution on p. 1123.)

Describe how linear functions and arithmetic sequences are similar. How are they di�erent?

9.3.7.2 Algebraic

For the following exercises, �nd the common di�erence for the arithmetic sequence provided.

Exercise 9.3.14{5, 11, 17, 23, 29, ...}

Available for free at Connexions <http://legacy.cnx.org/content/col11759/1.2>

1035

Exercise 9.3.15 (Solution on p. 1123.)

{0, 12 , 1,

32 , 2, ...}

For the following exercises, determine whether the sequence is arithmetic. If so �nd the common di�erence.

Exercise 9.3.16{11.4, 9.3, 7.2, 5.1, 3, ...}

Exercise 9.3.17 (Solution on p. 1123.)

{4, 16, 64, 256, 1024, ...}For the following exercises, write the �rst �ve terms of the arithmetic sequence given the �rst term andcommon di�erence.

Exercise 9.3.18a1 = −25, d = −9Exercise 9.3.19 (Solution on p. 1123.)

a1 = 0, d = 23

For the following exercises, write the �rst �ve terms of the arithmetic series given two terms.

Exercise 9.3.20a1 = 17, a7 = −31Exercise 9.3.21 (Solution on p. 1123.)

a13 = −60, a33 = −160For the following exercises, �nd the speci�ed term for the arithmetic sequence given the �rst term andcommon di�erence.

Exercise 9.3.22First term is 3, common di�erence is 4, �nd the 5th term.

Exercise 9.3.23 (Solution on p. 1123.)

First term is 4, common di�erence is 5, �nd the 4th term.

Exercise 9.3.24First term is 5, common di�erence is 6, �nd the 8th term.

Exercise 9.3.25 (Solution on p. 1123.)

First term is 6, common di�erence is 7, �nd the 6th term.

Exercise 9.3.26First term is 7, common di�erence is 8, �nd the 7th term.

For the following exercises, �nd the �rst term given two terms from an arithmetic sequence.

Exercise 9.3.27 (Solution on p. 1123.)

Find the �rst term or a1 of an arithmetic sequence if a6 = 12 and a14 = 28.Exercise 9.3.28Find the �rst term or a1 of an arithmetic sequence if a7 = 21 and a15 = 42.Exercise 9.3.29 (Solution on p. 1123.)

Find the �rst term or a1 of an arithmetic sequence if a8 = 40 and a23 = 115.Exercise 9.3.30Find the �rst term or a1 of an arithmetic sequence if a9 = 54 and a17 = 102.Exercise 9.3.31 (Solution on p. 1123.)

Find the �rst term or a1 of an arithmetic sequence if a11 = 11 and a21 = 16.For the following exercises, �nd the speci�ed term given two terms from an arithmetic sequence.

Exercise 9.3.32a1 = 33 and a7 = −15. Find a4.

Available for free at Connexions <http://legacy.cnx.org/content/col11759/1.2>

1036 CHAPTER 9. SEQUENCES, PROBABILITY, AND COUNTING THEORY

Exercise 9.3.33 (Solution on p. 1124.)

a3 = −17.1 and a10 = −15.7. Finda21.

For the following exercises, use the recursive formula to write the �rst �ve terms of the arithmetic sequence.

Exercise 9.3.34a1 = 39; an = an−1 − 3Exercise 9.3.35 (Solution on p. 1124.)

a1 = −19; an = an−1 − 1.4For the following exercises, write a recursive formula for each arithmetic sequence.

Exercise 9.3.36an = {40, 60, 80, ...}Exercise 9.3.37 (Solution on p. 1124.)

an = {17, 26, 35, ...}Exercise 9.3.38an = {−1, 2, 5, ...}Exercise 9.3.39 (Solution on p. 1124.)

an = {12, 17, 22, ...}Exercise 9.3.40an = {−15,−7, 1, ...}Exercise 9.3.41 (Solution on p. 1124.)

an = {8.9, 10.3, 11.7, ...}Exercise 9.3.42an = {−0.52,−1.02,−1.52, ...}Exercise 9.3.43 (Solution on p. 1124.)

an = { 15 ,

920 ,

710 , ...}

Exercise 9.3.44an = {− 1

2 ,−54 ,−2, ...}

Exercise 9.3.45 (Solution on p. 1124.)

an = { 16 ,−

1112 ,−2, ...}

For the following exercises, write a recursive formula for the given arithmetic sequence, and then �nd thespeci�ed term.

Exercise 9.3.46an = {7, 4, 1, ...}; Find the 17th term.

Exercise 9.3.47 (Solution on p. 1124.)

an = {4, 11, 18, ...}; Find the 14th term.

Exercise 9.3.48an = {2, 6, 10, ...}; Find the 12th term.

For the following exercises, use the explicit formula to write the �rst �ve terms of the arithmetic sequence.

Exercise 9.3.49 (Solution on p. 1124.)

an = 24− 4nExercise 9.3.50an = 1

2n−12

For the following exercises, write an explicit formula for each arithmetic sequence.

Exercise 9.3.51 (Solution on p. 1124.)

an = {3, 5, 7, ...}

Available for free at Connexions <http://legacy.cnx.org/content/col11759/1.2>

1037

Exercise 9.3.52an = {32, 24, 16, ...}Exercise 9.3.53 (Solution on p. 1124.)

an = {−5, 95, 195, ...}Exercise 9.3.54an = {−17, − 217, − 417,...}Exercise 9.3.55 (Solution on p. 1124.)

an = {1.8, 3.6, 5.4, ...}Exercise 9.3.56an = {−18.1,−16.2,−14.3, ...}Exercise 9.3.57 (Solution on p. 1124.)

an = {15.8, 18.5, 21.2, ...}Exercise 9.3.58an = { 1

3 ,−43 ,−3, ...}

Exercise 9.3.59 (Solution on p. 1124.)

an = {0, 13 ,

23 , ...}

Exercise 9.3.60an = {−5,− 10

3 ,−53 , . . . }

For the following exercises, �nd the number of terms in the given �nite arithmetic sequence.

Exercise 9.3.61 (Solution on p. 1124.)

an = {3,− 4,− 11, ...,− 60}Exercise 9.3.62an = {1.2, 1.4, 1.6, ..., 3.8}Exercise 9.3.63 (Solution on p. 1124.)

an = { 12 , 2,

72 , ..., 8}

9.3.7.3 Graphical

For the following exercises, determine whether the graph shown represents an arithmetic sequence.

Available for free at Connexions <http://legacy.cnx.org/content/col11759/1.2>

1038 CHAPTER 9. SEQUENCES, PROBABILITY, AND COUNTING THEORY

Exercise 9.3.64

Available for free at Connexions <http://legacy.cnx.org/content/col11759/1.2>

1039

Exercise 9.3.65 (Solution on p. 1124.)

For the following exercises, use the information provided to graph the �rst 5 terms of the arithmetic sequence.

Available for free at Connexions <http://legacy.cnx.org/content/col11759/1.2>

1040 CHAPTER 9. SEQUENCES, PROBABILITY, AND COUNTING THEORY

Exercise 9.3.66a1 = 0, d = 4Exercise 9.3.67 (Solution on p. 1124.)

a1 = 9; an = an−1 − 10Exercise 9.3.68an = −12 + 5n

9.3.7.4 Technology

For the following exercises, follow the steps to work with the arithmetic sequence an = 3n − 2 using agraphing calculator:

� Press [MODE]

- Select SEQ in the fourth line- Select DOT in the �fth line- Press [ENTER]

� Press [Y=]

- nMin is the �rst counting number for the sequence. Set nMin = 1- u (n) is the pattern for the sequence. Set u (n) = 3n− 2- u (nMin) is the �rst number in the sequence. Set u (nMin) = 1

� Press [2ND] then [WINDOW] to go to TBLSET

- SetTblStart = 1- Set∆Tbl = 1- Set Indpnt: Auto and Depend: Auto

� Press [2ND] then [GRAPH] to go to the TABLE

Exercise 9.3.69 (Solution on p. 1125.)

What are the �rst seven terms shown in the column with the heading u (n) ?Exercise 9.3.70Use the scroll-down arrow to scroll ton = 50. What value is given for u (n) ?Exercise 9.3.71 (Solution on p. 1125.)

Press [WINDOW]. SetnMin = 1, nMax = 5, xMin = 0, xMax = 6, yMin = −1, and yMax =14.Then press [GRAPH]. Graph the sequence as it appears on the graphing calculator.

For the following exercises, follow the steps given above to work with the arithmetic sequence an = 12n+ 5

using a graphing calculator.

Exercise 9.3.72What are the �rst seven terms shown in the column with the headingu (n) in the TABLE feature?

Exercise 9.3.73 (Solution on p. 1126.)

Graph the sequence as it appears on the graphing calculator. Be sure to adjust the WINDOWsettings as needed.

Available for free at Connexions <http://legacy.cnx.org/content/col11759/1.2>

1041

9.3.7.5 Extensions

Exercise 9.3.74Give two examples of arithmetic sequences whose 4th terms are 9.Exercise 9.3.75 (Solution on p. 1127.)

Give two examples of arithmetic sequences whose 10th terms are 206.Exercise 9.3.76Find the 5th term of the arithmetic sequence {9b, 5b, b, . . . }.Exercise 9.3.77 (Solution on p. 1127.)

Find the 11th term of the arithmetic sequence {3a− 2b, a+ 2b,−a+ 6b . . . }.Exercise 9.3.78At which term does the sequence {5.4, 14.5, 23.6, ...} exceed 151?

Exercise 9.3.79 (Solution on p. 1127.)

At which term does the sequence { 173 ,

316 ,

143 , ...} begin to have negative values?

Exercise 9.3.80For which terms does the �nite arithmetic sequence { 5

2 ,198 ,

94 , ...,

18} have integer values?

Exercise 9.3.81 (Solution on p. 1127.)

Write an arithmetic sequence using a recursive formula. Show the �rst 4 terms, and then �nd the31st term.

Exercise 9.3.82Write an arithmetic sequence using an explicit formula. Show the �rst 4 terms, and then �nd the28th term.

9.4 Geometric Sequences6

Many jobs o�er an annual cost-of-living increase to keep salaries consistent with in�ation. Suppose, forexample, a recent college graduate �nds a position as a sales manager earning an annual salary of $26,000.He is promised a 2% cost of living increase each year. His annual salary in any given year can be found bymultiplying his salary from the previous year by 102%. His salary will be $26,520 after one year; $27,050.40after two years; $27,591.41 after three years; and so on. When a salary increases by a constant rate eachyear, the salary grows by a constant factor. In this section, we will review sequences that grow in this way.

9.4.1 Finding Common Ratios

The yearly salary values described form a geometric sequence because they change by a constant factoreach year. Each term of a geometric sequence increases or decreases by a constant factor called the commonratio. The sequence below is an example of a geometric sequence because each term increases by a constantfactor of 6. Multiplying any term of the sequence by the common ratio 6 generates the subsequent term.

6This content is available online at <http://legacy.cnx.org/content/m49446/1.6/>.

Available for free at Connexions <http://legacy.cnx.org/content/col11759/1.2>

1042 CHAPTER 9. SEQUENCES, PROBABILITY, AND COUNTING THEORY

A General Note: A geometric sequence is one in which any term divided by the previousterm is a constant. This constant is called the common ratio of the sequence. The common ratiocan be found by dividing any term in the sequence by the previous term. If a1 is the initial termof a geometric sequence and r is the common ratio, the sequence will be

{a1, a1r, a1r2, a1r

3, ...}. (9.63)

How To: Given a set of numbers, determine if they represent a geometric sequence.

1.Divide each term by the previous term.2.Compare the quotients. If they are the same, a common ratio exists and the sequence isgeometric.

Example 9.15Finding Common RatiosIs the sequence geometric? If so, �nd the common ratio.

a. 1, 2, 4, 8, 16, ...b. 48, 12, 4, 2, ...

SolutionDivide each term by the previous term to determine whether a common ratio exists.

a. 21 = 2 4

2 = 2 84 = 2 16

8 = 2 The sequence is geometric because there is a

common ratio. The common ratio is 2.b. 12

48 = 14

412 = 1

324 = 1

2The sequence is not geometric because there is not a

common ratio.

AnalysisThe graph of each sequence is shown in Figure 9.14. It seems from the graphs that both (a) and(b) appear have the form of the graph of an exponential function in this viewing window. However,we know that (a) is geometric and so this interpretation holds, but (b) is not.

Figure 9.14

Available for free at Connexions <http://legacy.cnx.org/content/col11759/1.2>

1043

Q&A: If you are told that a sequence is geometric, do you have to divide every termby the previous term to �nd the common ratio?

No. If you know that the sequence is geometric, you can choose any one term in the sequence anddivide it by the previous term to �nd the common ratio.

Try It:

Exercise 9.4.1 (Solution on p. 1127.)

Is the sequence geometric? If so, �nd the common ratio.

5, 10, 15, 20, ... (9.64)

Try It:

Exercise 9.4.2 (Solution on p. 1127.)

Is the sequence geometric? If so, �nd the common ratio.

100, 20, 4,45, ... (9.65)

9.4.2 Writing Terms of Geometric Sequences

Now that we can identify a geometric sequence, we will learn how to �nd the terms of a geometric sequenceif we are given the �rst term and the common ratio. The terms of a geometric sequence can be found bybeginning with the �rst term and multiplying by the common ratio repeatedly. For instance, if the �rstterm of a geometric sequence is a1 = −2 and the common ratio is r = 4, we can �nd subsequent terms bymultiplying −2 · 4 to get −8 then multiplying the result −8 · 4 to get −32 and so on.

a1 = −2

a2 = (−2 · 4) = −8

a3 = (−8 · 4) = −32

a4 = (−32 · 4)− 128

(9.66)

The �rst four terms are{− − 2, −−8, −−32, −−128}.

How To: Given the �rst term and the common factor, �nd the �rst four terms of ageometric sequence.

1.Multiply the initial term, a1, by the common ratio to �nd the next term, a2.2.Repeat the process, using an = a2 to �nd a3 and then a3 to �nd a4, until all four terms havebeen identi�ed.

3.Write the terms separated by commons within brackets.

Available for free at Connexions <http://legacy.cnx.org/content/col11759/1.2>

1044 CHAPTER 9. SEQUENCES, PROBABILITY, AND COUNTING THEORY

Example 9.16Writing the Terms of a Geometric SequenceList the �rst four terms of the geometric sequence with a1 = 5 and r = −− 2.

SolutionMultiply a1 by −2 to �nd a2. Repeat the process, using a2 to �nd a3, and so on.

a1 = 5

a2 = −2a1 = −10

a3 = −2a2 = 20

a4 = −2a3 = −40

(9.67)

The �rst four terms are {5,−− 10, 20,−− 40}.

Try It:

Exercise 9.4.3 (Solution on p. 1127.)

List the �rst �ve terms of the geometric sequence with a1 = 18 and r = 13 .

9.4.3 Using Recursive Formulas for Geometric Sequences

A recursive formula allows us to �nd any term of a geometric sequence by using the previous term. Eachterm is the product of the common ratio and the previous term. For example, suppose the common ratio is9. Then each term is nine times the previous term. As with any recursive formula, the initial term must begiven.

A General Note: The recursive formula for a geometric sequence with common ratio r and�rst term a1 is

an = ran−1, n ≥ 2 (9.68)

How To: Given the �rst several terms of a geometric sequence, write its recursiveformula.

1.State the initial term.2.Find the common ratio by dividing any term by the preceding term.3.Substitute the common ratio into the recursive formula for a geometric sequence.

Example 9.17Using Recursive Formulas for Geometric SequencesWrite a recursive formula for the following geometric sequence.

{6, 9, 13.5, 20.25, ...} (9.69)

Solution

The �rst term is given as 6. The common ratio can be found by dividing the second term by the�rst term.

Available for free at Connexions <http://legacy.cnx.org/content/col11759/1.2>

1045

r =96

= 1.5 (9.70)

Substitute the common ratio into the recursive formula for geometric sequences and de�ne a1.

an = ran−1

an = 1.5an−1 for n ≥ 2

a1 = 6

(9.71)

AnalysisThe sequence of data points follows an exponential pattern. The common ratio is also the base ofan exponential function as shown in Figure 9.15

Figure 9.15

A General Note: Do we have to divide the second term by the �rst term to �nd thecommon ratio?

No. We can divide any term in the sequence by the previous term. It is, however, most commonto divide the second term by the �rst term because it is often the easiest method of �nding thecommon ratio.

Try It:

Exercise 9.4.4 (Solution on p. 1127.)

Write a recursive formula for the following geometric sequence.

Available for free at Connexions <http://legacy.cnx.org/content/col11759/1.2>

1046 CHAPTER 9. SEQUENCES, PROBABILITY, AND COUNTING THEORY

{2, 43,

89,

1627, ...} (9.72)

9.4.4 Using Explicit Formulas for Geometric Sequences

Because a geometric sequence is an exponential function whose domain is the set of positive integers, andthe common ratio is the base of the function, we can write explicit formulas that allow us to �nd particularterms.

an = a1rn−1 (9.73)

Let's take a look at the sequence {18, 36, 72, 144, 288, ...}. This is a geometric sequence with a commonratio of 2 and an exponential function with a base of 2. An explicit formula for this sequence is

an = 18 · 2n−1 (9.74)

The graph of the sequence is shown in Figure 9.16.

Available for free at Connexions <http://legacy.cnx.org/content/col11759/1.2>

1047

Figure 9.16

A General Note: The nth term of a geometric sequence is given by the explicit formula:

an = a1rn−1 (9.75)

Example 9.18Writing Terms of Geometric Sequences Using the Explicit FormulaGiven a geometric sequence with a1 = 3 and a4 = 24, �nd a2.

SolutionThe sequence can be written in terms of the initial term and the common ratio r.

3, 3r, 3r2, 3r3, ... (9.76)

Available for free at Connexions <http://legacy.cnx.org/content/col11759/1.2>

1048 CHAPTER 9. SEQUENCES, PROBABILITY, AND COUNTING THEORY

Find the common ratio using the given fourth term.

an = a1rn−1

a4 = 3r3 Write the fourth term of sequence in terms of α1 and r

24 = 3r3 Substitute 24 for a4

8 = r3 Divide

r = 2 Solve for the common ratio

(9.77)

Find the second term by multiplying the �rst term by the common ratio.

a2 = 2a1

= 2 (3)

= 6

(9.78)

AnalysisThe common ratio is multiplied by the �rst term once to �nd the second term, twice to �nd thethird term, three times to �nd the fourth term, and so on. The tenth term could be found bymultiplying the �rst term by the common ratio nine times or by multiplying by the common ratioraised to the ninth power.

Try It:

Exercise 9.4.5 (Solution on p. 1128.)

Given a geometric sequence with a2 = 4 and a3 = 32, �nd a6.

Example 9.19Writing an Explicit Formula for the nth Term of a Geometric SequenceWrite an explicit formula for the nth term of the following geometric sequence.

{2, 10, 50, 250, ...} (9.79)

Solution

The �rst term is 2. The common ratio can be found by dividing the second term by the �rst term.

102

= 5 (9.80)

The common ratio is 5. Substitute the common ratio and the �rst term of the sequence into theformula.

an = a1r(n−1)

an = 2 · 5n−1(9.81)

The graph of this sequence in Figure 9.17 shows an exponential pattern.

Available for free at Connexions <http://legacy.cnx.org/content/col11759/1.2>

1049

Figure 9.17

Try It:

Exercise 9.4.6 (Solution on p. 1128.)

Write an explicit formula for the following geometric sequence.

{− − 1, 3, −−9, 27, ...} (9.82)

9.4.5 Solving Application Problems with Geometric Sequences

In real-world scenarios involving arithmetic sequences, we may need to use an initial term of a0 instead ofa1. In these problems, we can alter the explicit formula slightly by using the following formula:

an = a0rn (9.83)

Example 9.20Solving Application Problems with Geometric SequencesIn 2013, the number of students in a small school is 284. It is estimated that the student populationwill increase by 4% each year.

Available for free at Connexions <http://legacy.cnx.org/content/col11759/1.2>

1050 CHAPTER 9. SEQUENCES, PROBABILITY, AND COUNTING THEORY

a. Write a formula for the student population.b. Estimate the student population in 2020.

Solution

a. The situation can be modeled by a geometric sequence with an initial term of 284. Thestudent population will be 104% of the prior year, so the common ratio is 1.04.Let P be the student population and n be the number of years after 2013. Using the explicitformula for a geometric sequence we get

Pn = 284 · 1.04n (9.84)

b. We can �nd the number of years since 2013 by subtracting.

2020− 2013 = 7 (9.85)

We are looking for the population after 7 years. We can substitute 7 for n to estimate thepopulation in 2020.

P7 = 284 · 1.047 ≈ 374 (9.86)

The student population will be about 374 in 2020.

Try It:

Exercise 9.4.7 (Solution on p. 1128.)

A business starts a new website. Initially the number of hits is 293 due to the curiosityfactor. The business estimates the number of hits will increase by 2.6% per week.

a.Write a formula for the number of hits.b.Estimate the number of hits in 5 weeks.

Media: Access these online resources for additional instruction and practice with geometricsequences.

• Geometric Sequences7• Determine the Type of Sequence8• Find the Formula for a Sequence9

9.4.6 Key Equations

recursive formula for nth term of a geometric sequence an = ran−1, n ≥ 2

explicit formula fornth term of a geometric sequence an = a1rn−1

Table 9.6

7http://openstaxcollege.org/l/geometricseq8http://openstaxcollege.org/l/sequencetype9http://openstaxcollege.org/l/sequenceformula

Available for free at Connexions <http://legacy.cnx.org/content/col11759/1.2>

1051

9.4.7 Key Concepts

• A geometric sequence is a sequence in which the ratio between any two consecutive terms is a constant.• The constant ratio between two consecutive terms is called the common ratio.• The common ratio can be found by dividing any term in the sequence by the previous term. See

Example 9.15.• The terms of a geometric sequence can be found by beginning with the �rst term and multiplying by

the common ratio repeatedly. See Example 9.16 and Example 9.18.• A recursive formula for a geometric sequence with common ratio r is given by an = ran−−1 for n ≥ 2.• As with any recursive formula, the initial term of the sequence must be given. See Example 9.17.• An explicit formula for a geometric sequence with common ratio r is given by an = a1r

n−−1. SeeExample 9.19.

• In application problems, we sometimes alter the explicit formula slightly to an = a0rn. See Exam-

ple 9.20.

9.4.8 Section Exercises

9.4.8.1 Verbal

Exercise 9.4.8 (Solution on p. 1128.)

What is a geometric sequence?

Exercise 9.4.9How is the common ratio of a geometric sequence found?

Exercise 9.4.10 (Solution on p. 1128.)

What is the procedure for determining whether a sequence is geometric?

Exercise 9.4.11What is the di�erence between an arithmetic sequence and a geometric sequence?

Exercise 9.4.12 (Solution on p. 1128.)

Describe how exponential functions and geometric sequences are similar. How are they di�erent?

9.4.8.2 Algebraic

For the following exercises, �nd the common ratio for the geometric sequence.

Exercise 9.4.131, 3, 9, 27, 81, ...Exercise 9.4.14 (Solution on p. 1128.)

−0.125, 0.25,−0.5, 1,−2, ...Exercise 9.4.15−2,− 1

2 ,−18 ,−

132 ,−

1128 , ...

For the following exercises, determine whether the sequence is geometric. If so, �nd the common ratio.

Exercise 9.4.16 (Solution on p. 1128.)

−6,−12,−24,−48,−96, ...Exercise 9.4.175, 5.2, 5.4, 5.6, 5.8, ...Exercise 9.4.18 (Solution on p. 1128.)

−1, 12 ,−

14 ,

18 ,−

116 , ...

Exercise 9.4.196, 8, 11, 15, 20, ...

Available for free at Connexions <http://legacy.cnx.org/content/col11759/1.2>

1052 CHAPTER 9. SEQUENCES, PROBABILITY, AND COUNTING THEORY

Exercise 9.4.20 (Solution on p. 1128.)

0.8, 4, 20, 100, 500, ...For the following exercises, write the �rst �ve terms of the geometric sequence, given the �rst term andcommon ratio.

Exercise 9.4.21

a1 = 8, r = 0.3

Exercise 9.4.22 (Solution on p. 1128.)

a1 = 5, r = 15

For the following exercises, write the �rst �ve terms of the geometric sequence, given any two terms.

Exercise 9.4.23

a7 = 64, a10 = 512

Exercise 9.4.24 (Solution on p. 1128.)

a6 = 25, a8 = 6.25

For the following exercises, �nd the speci�ed term for the geometric sequence, given the �rst term andcommon ratio.

Exercise 9.4.25The �rst term is 2, and the common ratio is 3. Find the 5th term.

Exercise 9.4.26 (Solution on p. 1128.)

The �rst term is 16 and the common ratio is − 13 . Find the 4th term.

For the following exercises, �nd the speci�ed term for the geometric sequence, given the �rst four terms.

Exercise 9.4.27an = {−1, 2,−4, 8, ...}. Find a12.

Exercise 9.4.28 (Solution on p. 1128.)

an = {−2, 23 ,−

29 ,

227 , ...}. Find a7.

For the following exercises, write the �rst �ve terms of the geometric sequence.

Exercise 9.4.29

a1 = −486, an = − 13an−1

Exercise 9.4.30 (Solution on p. 1128.)

a1 = 7, an = 0.2an−1

For the following exercises, write a recursive formula for each geometric sequence.

Exercise 9.4.31an = {−1, 5,−25, 125, ...}Exercise 9.4.32 (Solution on p. 1128.)

an = {−32,−16,−8,−4, ...}Exercise 9.4.33an = {14, 56, 224, 896, ...}Exercise 9.4.34 (Solution on p. 1128.)

an = {10,−3, 0.9,−0.27, ...}Exercise 9.4.35an = {0.61, 1.83, 5.49, 16.47, ...}Exercise 9.4.36 (Solution on p. 1128.)

an = { 35 ,

110 ,

160 ,

1360 , ...}

Exercise 9.4.37an = {−2, 4

3 ,−89 ,

1627 , ...}

Available for free at Connexions <http://legacy.cnx.org/content/col11759/1.2>

1053

Exercise 9.4.38 (Solution on p. 1128.)

an = { 1512 ,−

1128 ,

132 ,−

18 , ...}

For the following exercises, write the �rst �ve terms of the geometric sequence.

Exercise 9.4.39an = −4 · 5n−1

Exercise 9.4.40 (Solution on p. 1128.)

an = 12 ·(− 1

2

)n−1

For the following exercises, write an explicit formula for each geometric sequence.

Exercise 9.4.41an = {−2,−4,−8,−16, ...}Exercise 9.4.42 (Solution on p. 1128.)

an = {1, 3, 9, 27, ...}Exercise 9.4.43an = {−4,−12,−36,−108, ...}Exercise 9.4.44 (Solution on p. 1128.)

an = {0.8,−4, 20,−100, ...}Exercise 9.4.45an = {−1.25,−5,−20,−80, ...}Exercise 9.4.46 (Solution on p. 1128.)

an = {−1,− 45 ,−

1625 ,−

64125 , ...}

Exercise 9.4.47an = {2, 1

3 ,118 ,

1108 , ...}

Exercise 9.4.48 (Solution on p. 1129.)

an = {3,−1, 13 ,−

19 , ...}

For the following exercises, �nd the speci�ed term for the geometric sequence given.

Exercise 9.4.49Let a1 = 4, an = −3an−1. Find a8.

Exercise 9.4.50 (Solution on p. 1129.)

Let an = −(− 1

3

)n−1. Find a12.

For the following exercises, �nd the number of terms in the given �nite geometric sequence.

Exercise 9.4.51an = {−1, 3,−9, ..., 2187}Exercise 9.4.52 (Solution on p. 1129.)

an = {2, 1, 12 , ...,

11024}

Available for free at Connexions <http://legacy.cnx.org/content/col11759/1.2>

1054 CHAPTER 9. SEQUENCES, PROBABILITY, AND COUNTING THEORY

9.4.8.3 Graphical

For the following exercises, determine whether the graph shown represents a geometric sequence.Exercise 9.4.53

Available for free at Connexions <http://legacy.cnx.org/content/col11759/1.2>

1055

Exercise 9.4.54 (Solution on p. 1129.)

For the following exercises, use the information provided to graph the �rst �ve terms of the geometricsequence.

Available for free at Connexions <http://legacy.cnx.org/content/col11759/1.2>

1056 CHAPTER 9. SEQUENCES, PROBABILITY, AND COUNTING THEORY

Exercise 9.4.55

a1 = 1, r = 12

Exercise 9.4.56 (Solution on p. 1129.)

a1 = 3, an = 2an−1

Exercise 9.4.57an = 27 · 0.3n−1

9.4.8.4 Extensions

Exercise 9.4.58 (Solution on p. 1129.)

Use recursive formulas to give two examples of geometric sequences whose 3rd terms are 200.Exercise 9.4.59Use explicit formulas to give two examples of geometric sequences whose 7th terms are 1024.Exercise 9.4.60 (Solution on p. 1129.)

Find the 5th term of the geometric sequence {b, 4b, 16b, ...}.Exercise 9.4.61Find the 7th term of the geometric sequence {64a (−b) , 32a (−3b) , 16a (−9b) , ...}.Exercise 9.4.62 (Solution on p. 1129.)

At which term does the sequence {10, 12, 14.4, 17.28, ...} exceed 100?Exercise 9.4.63At which term does the sequence { 1

2187 ,1

729 ,1

243 ,181 ...} begin to have integer values?

Exercise 9.4.64 (Solution on p. 1129.)

For which term does the geometric sequence an

= −36(

23

)n−1�rst have a non-integer value?

Exercise 9.4.65Use the recursive formula to write a geometric sequence whose common ratio is an integer. Showthe �rst four terms, and then �nd the 10th term.

Exercise 9.4.66 (Solution on p. 1129.)

Use the explicit formula to write a geometric sequence whose common ratio is a decimal numberbetween 0 and 1. Show the �rst 4 terms, and then �nd the 8th term.

Exercise 9.4.67Is it possible for a sequence to be both arithmetic and geometric? If so, give an example.

9.5 Series and Their Notations10

A couple decides to start a college fund for their daughter. They plan to invest $50 in the fund each month.The fund pays 6% annual interest, compounded monthly. How much money will they have saved when theirdaughter is ready to start college in 6 years? In this section, we will learn how to answer this question. Todo so, we need to consider the amount of money invested and the amount of interest earned.

10This content is available online at <http://legacy.cnx.org/content/m49447/1.6/>.

Available for free at Connexions <http://legacy.cnx.org/content/col11759/1.2>

1057

9.5.1 Using Summation Notation

To �nd the total amount of money in the college fund and the sum of the amounts deposited, we need to addthe amounts deposited each month and the amounts earned monthly. The sum of the terms of a sequence iscalled a series. Consider, for example, the following series.

3 + 7 + 11 + 15 + 19 + ... (9.87)

The nth partial sum of a series is the sum of a �nite number of consecutive terms beginning with the �rstterm. The notation Sn represents the partial sum.

S1 = 3

S2 = 3 + 7 = 10

S3 = 3 + 7 + 11 = 21

S4 = 3 + 7 + 11 + 15 = 36

(9.88)

Summation notation is used to represent series. Summation notation is often known as sigma notationbecause it uses the Greek capital letter sigma, Σ, to represent the sum. Summation notation includes anexplicit formula and speci�es the �rst and last terms in the series. An explicit formula for each term of theseries is given to the right of the sigma. A variable called the index of summation is written below thesigma. The index of summation is set equal to the lower limit of summation, which is the number used togenerate the �rst term in the series. The number above the sigma, called the upper limit of summation,is the number used to generate the last term in a series.

Ifwe interpret the given notation, we see that it asks us to �nd the sum of the terms in the series ak = 2k fork = 1 through k = 5. We can begin by substituting the terms for k and listing out the terms of this series.

a1 = 2 (1) = 2

a2 = 2 (2) = 4

a3 = 2 (3) = 6

a4 = 2 (4) = 8

a5 = 2 (5) = 10

(9.89)

We can �nd the sum of the series by adding the terms:

5∑k=1

2k = 2 + 4 + 6 + 8 + 10 = 30 (9.90)

A General Note: The sum of the �rstnterms of a series can be expressed in summationnotation as follows:

n∑k=1

ak (9.91)

Available for free at Connexions <http://legacy.cnx.org/content/col11759/1.2>

1058 CHAPTER 9. SEQUENCES, PROBABILITY, AND COUNTING THEORY

This notation tells us to �nd the sum of ak from k = 1 to k = n.

k is called the index of summation, 1 is the lower limit of summation, and n is the upperlimit of summation.

Q&A: Does the lower limit of summation have to be 1?

No. The lower limit of summation can be any number, but 1 is frequently used. We will look atexamples with lower limits of summation other than 1.

How To: Given summation notation for a series, evaluate the value.

1.Identify the lower limit of summation.2.Identify the upper limit of summation.3.Substitute each value of k from the lower limit to the upper limit into the formula.4.Add to �nd the sum.

Example 9.21Using Summation NotationEvaluate

∑7k=3 k

2.

SolutionAccording to the notation, the lower limit of summation is 3 and the upper limit is 7. So weneed to �nd the sum of k2 from k = 3 to k = 7. We �nd the terms of the series by substitutingk = 3,4,5,6, and 7 into the function k2. We add the terms to �nd the sum.

∑7k=3 k

2 = 32 + 42 + 52 + 62 + 72

= 9 + 16 + 25 + 36 + 49

= 135

(9.92)

Try It:

Exercise 9.5.1 (Solution on p. 1129.)

Evaluate∑5k=2 (3k −−1) .

9.5.2 Using the Formula for Arithmetic Series

Just as we studied special types of sequences, we will look at special types of series. Recall that an arith-metic sequence is a sequence in which the di�erence between any two consecutive terms is the commondi�erence,d. The sum of the terms of an arithmetic sequence is called an arithmetic series. We can writethe sum of the �rst n terms of an arithmetic series as:

Sn = a1 + (a1 + d) + (a1 + 2d) + ...+ (an −−d) + an. (9.93)

We can also reverse the order of the terms and write the sum as

Sn = an + (an −−d) + (an −−2d) + ...+ (a1 + d) + a1. (9.94)

Available for free at Connexions <http://legacy.cnx.org/content/col11759/1.2>

1059

If we add these two expressions for the sum of the �rst nterms of an arithmetic series, we can derive aformula for the sum of the �rst n terms of any arithmetic series.

Sn = a1 + (a1 + d) + (a1 + 2d) + ...+ (an −−d) + an

+ Sn = an + (an −−d) + (an −−2d) + ...+ (a1 + d) + a1

2Sn = (a1 + an) + (a1 + an) + ...+ (a1 + an)(9.95)

Because there are n terms in the series, we can simplify this sum to

2Sn = n (a1 + an) . (9.96)

We divide by 2 to �nd the formula for the sum of the �rst n terms of an arithmetic series.

Sn =n (a1 + an)

2(9.97)

A General Note: An arithmetic series is the sum of the terms of an arithmetic sequence.The formula for the sum of the �rst n terms of an arithmetic sequence is

Sn =n (a1 + an)

2(9.98)

How To: Given terms of an arithmetic series, �nd the sum of the �rst n terms.

1.Identify a1 and an.2.Determine n.3.Substitute values for a1, an, and n into the formula Sn = n(a1+an)

2 .4.Simplify to �nd Sn.

Example 9.22Finding the First n Terms of an Arithmetic SeriesFind the sum of each arithmetic series.

a. 5 + 8 + 11 + 14 + 17 + 20 + 23 + 26 + 29 + 32b. 20 + 15 + 10 +. . .+ −50c.∑12k=1 3k − 8

Solution

a. We are given a1 = 5 and an = 32.Count the number of terms in the sequence to �nd n = 10.Substitute values for a1, an , and n into the formula and simplify.

Sn = n(a1+an)2

S10 = 10(5+32)2 = 185

(9.99)

Available for free at Connexions <http://legacy.cnx.org/content/col11759/1.2>

1060 CHAPTER 9. SEQUENCES, PROBABILITY, AND COUNTING THEORY

b. We are given a1 = 20 and an = −50.Use the formula for the general term of an arithmetic sequence to �nd n.

an = a1 + (n− 1) d

−50 = 20 + (n− 1) (−5)

−70 = (n− 1) (−5)

14 = n− 1

15 = n

(9.100)

Substitute values for a1, an,n into the formula and simplify.

Sn = n(a1+an)2

S15 = 15(20−50)2 = −225

(9.101)

c. To �nd a1, substitute k = 1 into the given explicit formula.

ak = 3k − 8

a1 = 3 (1)− 8 = −5(9.102)

We are given that n = 12. To �nd a12, substitute k = 12 into the given explicit formula.

ak = 3k − 8

a12 = 3 (12)− 8 = 28(9.103)

Substitute values for a1, an, and n into the formula and simplify.

Sn = n(a1+an)2

S12 = 12(−5+28)2 = 138

(9.104)

Use the formula to �nd the sum of each arithmetic series.

Try It:

Exercise 9.5.2 (Solution on p. 1129.)

1.4 + 1.6 + 1.8 + 2.0 + 2.2 + 2.4 + 2.6 + 2.8 + 3.0 + 3.2 + 3.4

Try It:

Exercise 9.5.3 (Solution on p. 1129.)

13 + 21 + 29 + . . .+ 69

Try It:

Exercise 9.5.4 (Solution on p. 1129.)∑10k=1 5− 6k

Available for free at Connexions <http://legacy.cnx.org/content/col11759/1.2>

1061

Example 9.23Solving Application Problems with Arithmetic SeriesOn the Sunday after a minor surgery, a woman is able to walk a half-mile. Each Sunday, she walksan additional quarter-mile. After 8 weeks, what will be the total number of miles she has walked?

SolutionThis problem can be modeled by an arithmetic series with a1 = 1

2 and d = 14 .We are looking for the

total number of miles walked after 8 weeks, so we know that n = 8, and we are looking forS8.To�nd a8, we can use the explicit formula for an arithmetic sequence.

an = a1 + d (n− 1)

a8 = 12 + 1

4 (8− 1) = 94

(9.105)

We can now use the formula for arithmetic series.

Sn = n(a1+an)2

S8 =8( 1

2 + 94 )

2 = 11(9.106)

She will have walked a total of 11 miles.

Try It:

Exercise 9.5.5 (Solution on p. 1130.)

A man earns $100 in the �rst week of June. Each week, he earns $12.50 more than theprevious week. After 12 weeks, how much has he earned?

9.5.3 Using the Formula for Geometric Series

Just as the sum of the terms of an arithmetic sequence is called an arithmetic series, the sum of the termsin a geometric sequence is called a geometric series. Recall that a geometric sequence is a sequence inwhich the ratio of any two consecutive terms is the common ratio, r.We can write the sum of the �rst nterms of a geometric series as

Sn = a1 + ra1 + r2a1 + ...+ rn−−1a1. (9.107)

Just as with arithmetic series, we can do some algebraic manipulation to derive a formula for the sum ofthe �rstn terms of a geometric series. We will begin by multiplying both sides of the equation by r.

rSn = ra1 + r2a1 + r3a1 + ...+ rna1 (9.108)

Next, we subtract this equation from the original equation.

Sn = a1 + ra1 + r2a1 + ...+ rn−−1a1

−rSn = −(ra1 + r2a1 + r3a1 + ...+ rna1

)(1−r)Sn=a1−rna1

(9.109)

Available for free at Connexions <http://legacy.cnx.org/content/col11759/1.2>

1062 CHAPTER 9. SEQUENCES, PROBABILITY, AND COUNTING THEORY

Notice that when we subtract, all but the �rst term of the top equation and the last term of the bottomequation cancel out. To obtain a formula for Sn, divide both sides by (1− r) .

Sn =a1 (1− rn)

1− rr 6= 1 (9.110)

A General Note: A geometric series is the sum of the terms in a geometric sequence. Theformula for the sum of the �rstn terms of a geometric sequence is represented as

Sn =a1 (1− rn)

1− rr 6= 1 (9.111)

How To: Given a geometric series, �nd the sum of the �rst n terms.

1.Identify a1, r, andn.

2.Substitute values for a1, r, and n into the formula Sn = a1(1−−rn)1−−r .

3.Simplify to �nd Sn.

Example 9.24Finding the First n Terms of a Geometric SeriesUse the formula to �nd the indicated partial sum of each geometric series.

a. S11for the series 8 + -4 + 2 + . . .

b.k=1∑

63 · 2k

Solution

a. a1 = 8, and we are given that n = 11.We can �nd r by dividing the second term of the series by the �rst.

r =−48

= −12

(9.112)

Substitute values for a1, r, and n into the formula and simplify.

Sn = a1(1−rn)1−r

S11 =8“

1−(− 12 )11

”1−(− 1

2 ) ≈ 5.336(9.113)

b. Find a1 by substituting k = 1 into the given explicit formula.

a1 = 3 · 21 = 6 (9.114)

We can see from the given explicit formula that r = 2. The upper limit of summation is 6,so n = 6.Substitute values for a1, r, and n into the formula, and simplify.

Sn = a1(1−rn)1−r

S6 =6(1−26)

1−2 = 378(9.115)

Available for free at Connexions <http://legacy.cnx.org/content/col11759/1.2>

1063

Use the formula to �nd the indicated partial sum of each geometric series.

Try It:

Exercise 9.5.6 (Solution on p. 1130.)

S20 for the series 1,000 + 500 + 250 + . . .

Try It:

Exercise 9.5.7 (Solution on p. 1130.)∑8k=1 3k

Example 9.25Solving an Application Problem with a Geometric SeriesAt a new job, an employee's starting salary is $26,750. He receives a 1.6% annual raise. Find histotal earnings at the end of 5 years.

SolutionThe problem can be represented by a geometric series with a1 = 26, 750;n = 5; and r = 1.016.Substitute values for a1, r, and n into the formula and simplify to �nd the total amount earned atthe end of 5 years.

Sn = a1(1−rn)1−r

S5 =26,750(1−1.0165)

1−1.016 ≈ 138,099.03(9.116)

He will have earned a total of $138,099.03 by the end of 5 years.

Try It:

Exercise 9.5.8 (Solution on p. 1130.)

At a new job, an employee's starting salary is $32,100. She receives a 2% annual raise.How much will she have earned by the end of 8 years?

9.5.4 Using the Formula for the Sum of an In�nite Geometric Series

Thus far, we have looked only at �nite series. Sometimes, however, we are interested in the sum of the termsof an in�nite sequence rather than the sum of only the �rst nterms. An in�nite series is the sum of theterms of an in�nite sequence. An example of an in�nite series is 2 + 4 + 6 + 8 + ...

This series can also be written in summation notation as∑∞k=1 2k, where the upper limit of summation

is in�nity. Because the terms are not tending to zero, the sum of the series increases without bound as weadd more terms. Therefore, the sum of this in�nite series is not de�ned. When the sum is not a real number,we say the series diverges.

Available for free at Connexions <http://legacy.cnx.org/content/col11759/1.2>

1064 CHAPTER 9. SEQUENCES, PROBABILITY, AND COUNTING THEORY

9.5.4.1 Determining Whether the Sum of an In�nite Geometric Series is De�ned

If the terms of an in�nite geometric series approach 0, the sum of an in�nite geometric series can bede�ned. The terms in this series approach 0:

1 + 0.2 + 0.04 + 0.008 + 0.0016 + ... (9.117)

The common ratio r = 0.2. Asn gets very large, the values of rn get very small and approach 0. Eachsuccessive term a�ects the sum less than the preceding term. As each succeeding term gets closer to 0, thesum of the terms approaches a �nite value. The terms of any in�nite geometric series with −1 < r < 1approach 0; the sum of a geometric series is de�ned when −1 < r < 1.

A General Note: The sum of an in�nite series is de�ned if the series is geometric and −1 <r < 1.

How To: Given the �rst several terms of an in�nite series, determine if the sum ofthe series exists.

1.Find the ratio of the second term to the �rst term.2.Find the ratio of the third term to the second term.3.Continue this process to ensure the ratio of a term to the preceding term is constant through-out. If so, the series is geometric.

4.If a common ratio, r, was found in step 3, check to see if −1 < r < 1. If so, the sum is de�ned.If not, the sum is not de�ned.

Example 9.26Determining Whether the Sum of an In�nite Series is De�nedDetermine whether the sum of each in�nite series is de�ned.

a. 12 + 8 + 4 + . . .b. 3

4 + 12 + 1

3 + ...

c.∑∞k=1 27 ·

(13

)kd.∑∞k=1 5k

Solution

a. The ratio of the second term to the �rst is 23 , which is not the same as the ratio of the third

term to the second, 12 . The series is not geometric.

b. The ratio of the second term to the �rst is the same as the ratio of the third term to thesecond. The series is geometric with a common ratio of 2

3 . The sum of the in�nite series isde�ned.

c. The given formula is exponential with a base of 13 ; the series is geometric with a common

ratio of 13 . The sum of the in�nite series is de�ned.

d. The given formula is not exponential; the series is not geometric because the terms are in-creasing, and so cannot yield a �nite sum.

Determine whether the sum of the in�nite series is de�ned.

Try It:

Exercise 9.5.9 (Solution on p. 1130.)

13 + 1

2 + 34 + 9

8 + ...

Available for free at Connexions <http://legacy.cnx.org/content/col11759/1.2>

1065

Try It:

Exercise 9.5.10 (Solution on p. 1130.)

24 + (−12) + 6 + (−3) + ...

Try It:

Exercise 9.5.11 (Solution on p. 1130.)∑∞k=1 15 · (−− 0.3)k

9.5.4.2 Finding Sums of In�nite Series

When the sum of an in�nite geometric series exists, we can calculate the sum. The formula for the sum ofan in�nite series is related to the formula for the sum of the �rst nterms of a geometric series.

Sn =a1 (1− rn)

1− r(9.118)

We will examine an in�nite series with r = 12 . What happens to rn as n increases?

(12

)2 = 14(

12

)3 = 18(

12

)4 = 116

(9.119)

The value of rn decreases rapidly. What happens for greater values of n?

(12

)10 = 11,024(

12

)20 = 11,048,576(

12

)30 = 11,073,741,824

(9.120)

As n gets very large, rn gets very small. We say that, as n increases without bound, rnapproaches 0. As rn

approaches 0,1− rn approaches 1. When this happens, the numerator approaches a1. This give us a formulafor the sum of an in�nite geometric series.

A General Note: The formula for the sum of an in�nite geometric series with −1 < r < 1 is

S =a1

1− r(9.121)

How To: Given an in�nite geometric series, �nd its sum.

1.Identifya1and r.2.Con�rm that −− 1 < r < 1.3.Substitute values for a1 and r into the formula, S = a1

1−r .4.Simplify to �ndS.

Example 9.27Finding the Sum of an In�nite Geometric SeriesFind the sum, if it exists, for the following:

a. 10 + 9 + 8 + 7 + . . .

Available for free at Connexions <http://legacy.cnx.org/content/col11759/1.2>

1066 CHAPTER 9. SEQUENCES, PROBABILITY, AND COUNTING THEORY

b. 248.6 + 99.44 + 39.776 + . . .c.∑∞k=1 4,374 ·

(−− 1

3

)k−−1

d.∑∞k=1

19 ·(

43

)kSolution

a. There is not a constant ratio; the series is not geometric.b. There is a constant ratio; the series is geometric. a1 = 248.6andr = 99.44

248.6 = 0.4, so the sumexists. Substitute a1 = 248.6 and r = 0.4 into the formula and simplify to �nd the sum:

S = a11−r

S = 248.61−0.4 = 414.3

(9.122)

c. The formula is exponential, so the series is geometric with r = −− 13 . Finda1 by substituting

k = 1 into the given explicit formula:

a1 = 4,374 ·(−− 1

3

)1−−1

= 4,374 (9.123)

Substitute a1 = 4,374 and r = − 13 into the formula, and simplify to �nd the sum:

S = a11−r

S = 4,374

1−(− 13 ) = 3,280.5

(9.124)

d. The formula is exponential, so the series is geometric, but r > 1.The sum does not exist.

Example 9.28Finding an Equivalent Fraction for a Repeating DecimalFind an equivalent fraction for the repeating decimal 0.3.

SolutionWe notice the repeating decimal 0.3 = 0.333... so we can rewrite the repeating decimal as a sumof terms.

0.3 = 0.3 + 0.03 + 0.003 + ... (9.125)

Looking for a pattern, we rewrite the sum, noticing that we see the �rst term multiplied to 0.1 inthe second term, and the second term multiplied to 0.1 in the third term.

0.3 = 0.3 + (0.1) (0.3)︸︷︷︸First Term

+ (0.1) (0.1) (0.3)︸ ︷︷ ︸Second Term

. (9.126)

Notice the pattern; we multiply each consecutive term by a common ratio of 0.1 starting with the�rst term of 0.3. So, substituting into our formula for an in�nite geometric sum, we have

Sn =a1

1− r=

0.31− 0.1

=0.30.9

=13. (9.127)

Available for free at Connexions <http://legacy.cnx.org/content/col11759/1.2>

1067

Find the sum, if it exists.

Try It:

Exercise 9.5.12 (Solution on p. 1130.)

2 + 23 + 2

9 + ...

Try It:

Exercise 9.5.13 (Solution on p. 1130.)∑∞k=1 0.76k + 1

Try It:

Exercise 9.5.14 (Solution on p. 1130.)∑∞k=1

(− 3

8

)k

9.5.5 Solving Annuity Problems

At the beginning of the section, we looked at a problem in which a couple invested a set amount of moneyeach month into a college fund for six years. An annuity is an investment in which the purchaser makes asequence of periodic, equal payments. To �nd the amount of an annuity, we need to �nd the sum of all thepayments and the interest earned. In the example, the couple invests $50 each month. This is the value ofthe initial deposit. The account paid 6% annual interest, compounded monthly. To �nd the interest rateper payment period, we need to divide the 6% annual percentage interest (APR) rate by 12. So the monthlyinterest rate is 0.5%. We can multiply the amount in the account each month by 100.5% to �nd the valueof the account after interest has been added.

We can �nd the value of the annuity right after the last deposit by using a geometric series with a1 = 50and r = 100.5% = 1.005. After the �rst deposit, the value of the annuity will be $50. Let us see if we candetermine the amount in the college fund and the interest earned.

We can �nd the value of the annuity after n deposits using the formula for the sum of the �rst n terms of ageometric series. In 6 years, there are 72 months, so n = 72.We can substitute a1 = 50, r = 1.005, and n =72 into the formula, and simplify to �nd the value of the annuity after 6 years.

S72 =50(1− 1.00572

)1− 1.005

≈ 4,320.44 (9.128)

After the last deposit, the couple will have a total of $4,320.44 in the account. Notice, the couple made72 payments of $50 each for a total of 72(50) = $3,600. This means that because of the annuity, the coupleearned $720.44 interest in their college fund.

How To: Given an initial deposit and an interest rate, �nd the value of an annuity.

1.Determine a1, the value of the initial deposit.2.Determinen, the number of deposits.3.Determine r.

a.Divide the annual interest rate by the number of times per year that interest is com-pounded.

b.Add 1 to this amount to �nd r.

Available for free at Connexions <http://legacy.cnx.org/content/col11759/1.2>

1068 CHAPTER 9. SEQUENCES, PROBABILITY, AND COUNTING THEORY

4.Substitute values for a1, r, andn into the formula for the sum of the �rst n terms of a geometric

series,Sn = a1(1−−rn)1−−r .

5.Simplify to �nd Sn, the value of the annuity after n deposits.

Example 9.29Solving an Annuity ProblemA deposit of $100 is placed into a college fund at the beginning of every month for 10 years. Thefund earns 9% annual interest, compounded monthly, and paid at the end of the month. How muchis in the account right after the last deposit?

SolutionThe value of the initial deposit is $100, so a1 = 100.A total of 120 monthly deposits are made inthe 10 years, son = 120. To �nd r, divide the annual interest rate by 12 to �nd the monthly interestrate and add 1 to represent the new monthly deposit.

r = 1 +0.0912

= 1.0075 (9.129)

Substitute a1 = 100, r = 1.0075, andn = 120 into the formula for the sum of the �rst n terms of ageometric series, and simplify to �nd the value of the annuity.

S120 =100

(1− 1.0075120

)1− 1.0075

≈ 19,351.43 (9.130)

So the account has $19,351.43 after the last deposit is made.

Try It:

Exercise 9.5.15 (Solution on p. 1130.)

At the beginning of each month, $200 is deposited into a retirement fund. The fund earns6% annual interest, compounded monthly, and paid into the account at the end of themonth. How much is in the account if deposits are made for 10 years?

Media: Access these online resources for additional instruction and practice with series.

• Arithmetic Series11• Geometric Series12• Summation Notation13

9.5.6 Key Equations

sum of the �rstn terms of an arithmetic series Sn = n(a1+an)2

sum of the �rstn terms of a geometric series Sn = a1(1−rn)1−r · r 6= 1

sum of an in�nite geometric series with −−1 < r < 1 Sn = a11−r · r 6= 1

Table 9.7

11http://openstaxcollege.org/l/arithmeticser12http://openstaxcollege.org/l/geometricser13http://openstaxcollege.org/l/sumnotation

Available for free at Connexions <http://legacy.cnx.org/content/col11759/1.2>

1069

9.5.7 Key Concepts

• The sum of the terms in a sequence is called a series.• A common notation for series is called summation notation, which uses the Greek letter sigma to

represent the sum. See Example 9.21.• The sum of the terms in an arithmetic sequence is called an arithmetic series.• The sum of the �rstnterms of an arithmetic series can be found using a formula. See Example 9.22

and Example 9.23.• The sum of the terms in a geometric sequence is called a geometric series.• The sum of the �rstnterms of a geometric series can be found using a formula. See Example 9.24 and

Example 9.25.• The sum of an in�nite series exists if the series is geometric with −− 1 < r < 1.• If the sum of an in�nite series exists, it can be found using a formula. SeeExample 9.26, Example 9.27,

and .• An annuity is an account into which the investor makes a series of regularly scheduled payments. The

value of an annuity can be found using geometric series. See .

9.5.8 Section Exercises

9.5.8.1 Verbal

Exercise 9.5.16 (Solution on p. 1130.)

What is an nth partial sum?

Exercise 9.5.17What is the di�erence between an arithmetic sequence and an arithmetic series?

Exercise 9.5.18 (Solution on p. 1130.)

What is a geometric series?

Exercise 9.5.19How is �nding the sum of an in�nite geometric series di�erent from �nding the nth partial sum?

Exercise 9.5.20 (Solution on p. 1130.)

What is an annuity?

9.5.8.2 Algebraic

For the following exercises, express each description of a sum using summation notation.

Exercise 9.5.21The sum of terms m2 + 3m from m = 1 to m = 5Exercise 9.5.22 (Solution on p. 1130.)

The sum from of n = 0 to n = 4 of 5nExercise 9.5.23The sum of 6k − 5 from k = −2 to k = 1Exercise 9.5.24 (Solution on p. 1130.)

The sum that results from adding the number 4 �ve times

For the following exercises, express each arithmetic sum using summation notation.

Exercise 9.5.255 + 10 + 15 + 20 + 25 + 30 + 35 + 40 + 45 + 50Exercise 9.5.26 (Solution on p. 1130.)

10 + 18 + 26 + · · ·+ 162

Available for free at Connexions <http://legacy.cnx.org/content/col11759/1.2>

1070 CHAPTER 9. SEQUENCES, PROBABILITY, AND COUNTING THEORY

Exercise 9.5.2712 + 1 + 3

2 + 2 + · · ·+ 4For the following exercises, use the formula for the sum of the �rst n terms of each arithmetic sequence.

Exercise 9.5.28 (Solution on p. 1130.)32 + 2 + 5

2 + 3 + 72

Exercise 9.5.2919 + 25 + 31 + · · ·+ 73Exercise 9.5.30 (Solution on p. 1130.)

3.2 + 3.4 + 3.6 + · · ·+ 5.6For the following exercises, express each geometric sum using summation notation.

Exercise 9.5.311 + 3 + 9 + 27 + 81 + 243 + 729 + 2187Exercise 9.5.32 (Solution on p. 1130.)

8 + 4 + 2 + · · ·+ 0.125Exercise 9.5.33− 1

6 + 112 −

124 + · · ·+ 1

768

For the following exercises, use the formula for the sum of the �rst n terms of each geometric sequence, andthen state the indicated sum.

Exercise 9.5.34 (Solution on p. 1130.)

9 + 3 + 1 + 13 + 1

9

Exercise 9.5.35∑9n=1 5 · 2n−1

Exercise 9.5.36 (Solution on p. 1130.)∑11a=1 64 · 0.2a−1

For the following exercises, determine whether the in�nite series has a sum. If so, write the formula for thesum. If not, state the reason.

Exercise 9.5.3712 + 18 + 24 + 30 + ...

Exercise 9.5.38 (Solution on p. 1130.)

2 + 1.6 + 1.28 + 1.024 + ...

Exercise 9.5.39∑∞m=1 4m−1

Exercise 9.5.40 (Solution on p. 1130.)k=1∑∞−(− 1

2

)k−1

9.5.8.3 Graphical

For the following exercises, use the following scenario. Javier makes monthly deposits into a savings account.He opened the account with an initial deposit of $50. Each month thereafter he increased the previousdeposit amount by $20.

Exercise 9.5.41Graph the arithmetic sequence showing one year of Javier's deposits.

Exercise 9.5.42 (Solution on p. 1131.)

Graph the arithmetic series showing the monthly sums of one year of Javier's deposits.

For the following exercises, use the geometric series∑∞k=1

(12

)k.

Available for free at Connexions <http://legacy.cnx.org/content/col11759/1.2>

1071

Exercise 9.5.43Graph the �rst 7 partial sums of the series.

Exercise 9.5.44 (Solution on p. 1131.)

What number does Sn seem to be approaching in the graph? Find the sum to explain why thismakes sense.

9.5.8.4 Numeric

For the following exercises, �nd the indicated sum.

Exercise 9.5.45∑14a=1 a

Exercise 9.5.46 (Solution on p. 1131.)∑6n=1 n (n− 2)

Exercise 9.5.47∑17k=1 k

2

Exercise 9.5.48 (Solution on p. 1131.)∑7k=1 2k

For the following exercises, use the formula for the sum of the �rst n terms of an arithmetic series to �ndthe sum.

Exercise 9.5.49−1.7 +−0.4 + 0.9 + 2.2 + 3.5 + 4.8Exercise 9.5.50 (Solution on p. 1131.)

6 + 152 + 9 + 21

2 + 12 + 272 + 15

Exercise 9.5.51−1 + 3 + 7 + ...+ 31Exercise 9.5.52 (Solution on p. 1131.)∑11

k=1

(k2 −

12

)For the following exercises, use the formula for the sum of the �rst n terms of a geometric series to �nd thepartial sum.

Exercise 9.5.53S6 for the series −2− 10− 50− 250...Exercise 9.5.54 (Solution on p. 1131.)

S7 for the series 0.4− 2 + 10− 50...Exercise 9.5.55∑9

k=1 2k−1

Exercise 9.5.56 (Solution on p. 1131.)∑10n=1−2 ·

(12

)n−1

For the following exercises, �nd the sum of the in�nite geometric series.

Exercise 9.5.574 + 2 + 1 + 1

2 ...

Exercise 9.5.58 (Solution on p. 1131.)

−1− 14 −

116 −

164 ...

Exercise 9.5.59k=1∑∞

3 ·(

14

)k−1

Available for free at Connexions <http://legacy.cnx.org/content/col11759/1.2>

1072 CHAPTER 9. SEQUENCES, PROBABILITY, AND COUNTING THEORY

Exercise 9.5.60 (Solution on p. 1131.)∑∞n=1 4.6 · 0.5n−1

For the following exercises, determine the value of the annuity for the indicated monthly deposit amount,the number of deposits, and the interest rate.

Exercise 9.5.61Deposit amount: $50; total deposits: 60; interest rate: 5%, compounded monthly

Exercise 9.5.62 (Solution on p. 1131.)

Deposit amount: $150; total deposits: 24; interest rate: 3%, compounded monthly

Exercise 9.5.63Deposit amount: $450; total deposits: 60; interest rate: 4.5%, compounded quarterly

Exercise 9.5.64 (Solution on p. 1131.)

Deposit amount: $100; total deposits: 120; interest rate: 10%, compounded semi-annually

9.5.8.5 Extensions

Exercise 9.5.65The sum of terms 50− k2 from k = x through 7 is 115. What is x?

Exercise 9.5.66 (Solution on p. 1132.)

Write an explicit formula foraksuch that∑6k=0 ak = 189. Assume this is an arithmetic series.

Exercise 9.5.67Find the smallest value of n such that

∑nk=1 (3k −−5) > 100.

Exercise 9.5.68 (Solution on p. 1132.)

How many terms must be added before the series −1− 3− 5− 7.... has a sum less than −75?Exercise 9.5.69Write 0.65 as an in�nite geometric series using summation notation. Then use the formula for�nding the sum of an in�nite geometric series to convert 0.65 to a fraction.

Exercise 9.5.70 (Solution on p. 1132.)

The sum of an in�nite geometric series is �ve times the value of the �rst term. What is the commonratio of the series?

Exercise 9.5.71To get the best loan rates available, the Riches want to save enough money to place 20% downon a $160,000 home. They plan to make monthly deposits of $125 in an investment account thato�ers 8.5% annual interest compounded semi-annually. Will the Riches have enough for a 20%down payment after �ve years of saving? How much money will they have saved?

Exercise 9.5.72 (Solution on p. 1132.)

Karl has two years to save $10, 000 to buy a used car when he graduates. To the nearest dollar,what would his monthly deposits need to be if he invests in an account o�ering a 4.2% annualinterest rate that compounds monthly?

Available for free at Connexions <http://legacy.cnx.org/content/col11759/1.2>

1073

9.5.8.6 Real-World Applications

Exercise 9.5.73Keisha devised a week-long study plan to prepare for �nals. On the �rst day, she plans to studyfor 1 hour, and each successive day she will increase her study time by 30minutes. How many hourswill Keisha have studied after one week?

Exercise 9.5.74 (Solution on p. 1132.)

A boulder rolled down a mountain, traveling 6 feet in the �rst second. Each successive second, itsdistance increased by 8 feet. How far did the boulder travel after 10 seconds?

Exercise 9.5.75A scientist places 50 cells in a petri dish. Every hour, the population increases by 1.5%. Whatwill the cell count be after 1 day?

Exercise 9.5.76 (Solution on p. 1132.)

A pendulum travels a distance of 3 feet on its �rst swing. On each successive swing, it travels 34

the distance of the previous swing. What is the total distance traveled by the pendulum when itstops swinging?

Exercise 9.5.77Rachael deposits $1,500 into a retirement fund each year. The fund earns 8.2% annual interest,compounded monthly. If she opened her account when she was 19 years old, how much will shehave by the time she is 55? How much of that amount will be interest earned?

9.6 Counting Principles14

A new company sells customizable cases for tablets and smartphones. Each case comes in a variety of colorsand can be personalized for an additional fee with images or a monogram. A customer can choose not topersonalize or could choose to have one, two, or three images or a monogram. The customer can choose theorder of the images and the letters in the monogram. The company is working with an agency to developa marketing campaign with a focus on the huge number of options they o�er. Counting the possibilities ischallenging!

We encounter a wide variety of counting problems every day. There is a branch of mathematics devotedto the study of counting problems such as this one. Other applications of counting include secure passwords,horse racing outcomes, and college scheduling choices. We will examine this type of mathematics in thissection.

9.6.1 Using the Addition Principle

The company that sells customizable cases o�ers cases for tablets and smartphones. There are 3 supportedtablet models and 5 supported smartphone models. The Addition Principle tells us that we can add thenumber of tablet options to the number of smartphone options to �nd the total number of options. By theAddition Principle, there are 8 total options, as we can see in Figure 9.18.

14This content is available online at <http://legacy.cnx.org/content/m49448/1.5/>.

Available for free at Connexions <http://legacy.cnx.org/content/col11759/1.2>

1074 CHAPTER 9. SEQUENCES, PROBABILITY, AND COUNTING THEORY

Figure 9.18

A General Note: According to the Addition Principle, if one event can occur in m ways anda second event with no common outcomes can occur in n ways, then the �rst or second event canoccur in m+ n ways.

Example 9.30Using the Addition PrincipleThere are 2 vegetarian entrée options and 5 meat entrée options on a dinner menu. What is thetotal number of entrée options?

SolutionWe can add the number of vegetarian options to the number of meat options to �nd the totalnumber of entrée options.

There are 7 total options.

Available for free at Connexions <http://legacy.cnx.org/content/col11759/1.2>

1075

Try It:

Exercise 9.6.1 (Solution on p. 1132.)

A student is shopping for a new computer. He is deciding among 3 desktop computersand 4 laptop computers. What is the total number of computer options?

9.6.2 Using the Multiplication Principle

The Multiplication Principle applies when we are making more than one selection. Suppose we arechoosing an appetizer, an entrée, and a dessert. If there are 2 appetizer options, 3 entrée options, and 2dessert options on a �xed-price dinner menu, there are a total of 12 possible choices of one each as shown inthe tree diagram in Figure 9.19.

Figure 9.19

The possible choices are:

1. soup, chicken, cake2. soup, chicken, pudding3. soup, �sh, cake4. soup, �sh, pudding5. soup, steak, cake6. soup, steak, pudding7. salad, chicken, cake8. salad, chicken, pudding9. salad, �sh, cake10. salad, �sh, pudding11. salad, steak, cake12. salad, steak, pudding We can also �nd the total number of possible dinners by multiplying.

Available for free at Connexions <http://legacy.cnx.org/content/col11759/1.2>

1076 CHAPTER 9. SEQUENCES, PROBABILITY, AND COUNTING THEORY

We could also conclude that there are 12 possible dinner choices simply by applying the MultiplicationPrinciple.

# of appetizer options × # of entree options × # of dessert options

2 × 3 × 2 = 12(9.131)

A General Note: According to the Multiplication Principle, if one event can occur in mways and a second event can occur in n ways after the �rst event has occurred, then the two eventscan occur in m× n ways. This is also known as the Fundamental Counting Principle.

Example 9.31Using the Multiplication PrincipleDiane packed 2 skirts, 4 blouses, and a sweater for her business trip. She will need to choose askirt and a blouse for each out�t and decide whether to wear the sweater. Use the MultiplicationPrinciple to �nd the total number of possible out�ts.

SolutionTo �nd the total number of out�ts, �nd the product of the number of skirt options, the number ofblouse options, and the number of sweater options.

Figure 9.20

There are 16 possible out�ts.

Try It:

Exercise 9.6.2 (Solution on p. 1132.)

A restaurant o�ers a breakfast special that includes a breakfast sandwich, a side dish,and a beverage. There are 3 types of breakfast sandwiches, 4 side dish options, and 5beverage choices. Find the total number of possible breakfast specials.

9.6.3 Finding the Number of Permutations of n Distinct Objects

The Multiplication Principle can be used to solve a variety of problem types. One type of problem involvesplacing objects in order. We arrange letters into words and digits into numbers, line up for photographs,decorate rooms, and more. An ordering of objects is called a permutation.

Available for free at Connexions <http://legacy.cnx.org/content/col11759/1.2>

1077

9.6.3.1 Finding the Number of Permutations of n Distinct Objects Using the MultiplicationPrinciple

To solve permutation problems, it is often helpful to draw line segments for each option. That enables us todetermine the number of each option so we can multiply. For instance, suppose we have four paintings, andwe want to �nd the number of ways we can hang three of the paintings in order on the wall. We can drawthree lines to represent the three places on the wall.

There are four options for the �rst place, so we write a 4 on the �rst line.

After the �rst place has been �lled, there are three options for the second place so we write a 3 on thesecond line.

After the second place has been �lled, there are two options for the third place so we write a 2 on thethird line. Finally, we �nd the product.

There are 24 possible permutations of the paintings.

How To: Givenndistinct options, determine how many permutations there are.

1.Determine how many options there are for the �rst situation.2.Determine how many options are left for the second situation.3.Continue until all of the spots are �lled.4.Multiply the numbers together.

Example 9.32Finding the Number of Permutations Using the Multiplication PrincipleAt a swimming competition, nine swimmers compete in a race.

a. How many ways can they place �rst, second, and third?b. How many ways can they place �rst, second, and third if a swimmer named Ariel wins �rst

place? (Assume there is only one contestant named Ariel.)c. How many ways can all nine swimmers line up for a photo?

Solution

a. Draw lines for each place.

There are 9 options for �rst place. Once someone has won �rst place, there are 8 remainingoptions for second place. Once �rst and second place have been won, there are 7 remainingoptions for third place.

Available for free at Connexions <http://legacy.cnx.org/content/col11759/1.2>

1078 CHAPTER 9. SEQUENCES, PROBABILITY, AND COUNTING THEORY

Multiply to �nd that there are 504 ways for the swimmers to place.b. Draw lines for describing each place.

We know Ariel must win �rst place, so there is only 1 option for �rst place. There are 8remaining options for second place, and then 7 remaining options for third place.

Multiply to �nd that there are 56 ways for the swimmers to place if Ariel wins �rst.c. Draw lines for describing each place in the photo.

There are 9 choices for the �rst spot, then 8 for the second, 7 for the third, 6 for the fourth,and so on until only 1 person remains for the last spot.

There are 362,880 possible permutations for the swimmers to line up.

AnalysisNote that in part c, we found there were 9! ways for 9 people to line up. The number ofpermutations ofn distinct objects can always be found byn!.

A family of �ve is having portraits taken. Use the Multiplication Principle to �nd the following.

Try It:

Exercise 9.6.3 (Solution on p. 1132.)

How many ways can the family line up for the portrait?

Try It:

Exercise 9.6.4 (Solution on p. 1132.)

How many ways can the photographer line up 3 family members?

Try It:

Exercise 9.6.5 (Solution on p. 1132.)

How many ways can the family line up for the portrait if the parents are required to standon each end?

Available for free at Connexions <http://legacy.cnx.org/content/col11759/1.2>

1079

9.6.3.2 Finding the Number of Permutations of n Distinct Objects Using a Formula

For some permutation problems, it is inconvenient to use the Multiplication Principle because there are somany numbers to multiply. Fortunately, we can solve these problems using a formula. Before we learn theformula, let's look at two common notations for permutations. If we have a set ofn objects and we wantto choose r objects from the set in order, we writeP (n, r) .Another way to write this is nPr, a notationcommonly seen on computers and calculators. To calculateP (n, r) , we begin by �ndingn!, the number ofways to line up all n objects. We then divide by (n− r)! to cancel out the (n− r) items that we do notwish to line up.

Let's see how this works with a simple example. Imagine a club of six people. They need to elect apresident, a vice president, and a treasurer. Six people can be elected president, any one of the �ve remainingpeople can be elected vice president, and any of the remaining four people could be elected treasurer. Thenumber of ways this may be done is 6× 5× 4 = 120. Using factorials, we get the same result.

6!3!

=6 · 5 · 4 · 3!

3!= 6 · 5 · 4 = 120 (9.132)

There are 120 ways to select 3 o�cers in order from a club with 6 members. We refer to this as a permutationof 6 taken 3 at a time. The general formula is as follows.

P (n, r) =n!

(n− r)!(9.133)

Note that the formula stills works if we are choosing alln objects and placing them in order. In that casewe would be dividing by (n− n)! or 0!, which we said earlier is equal to 1. So the number of permutationsofn objects takenn at a time is n!

1 or justn!.

A General Note: Givenn distinct objects, the number of ways to select r objects from the setin order is

P (n, r) =n!

(n− r)!(9.134)

How To: Given a word problem, evaluate the possible permutations.

1.Identifyn from the given information.2.Identify r from the given information.3.Replacen and r in the formula with the given values.4.Evaluate.

Example 9.33Finding the Number of Permutations Using the FormulaA professor is creating an exam of 9 questions from a test bank of 12 questions. How many wayscan she select and arrange the questions?

SolutionSubstituten = 12 and r = 9 into the permutation formula and simplify.

P (n, r) = n!(n−r)!

P (12, 9) = 12!(12−9)! = 12!

3! = 79,833,600(9.135)

There are 79,833,600 possible permutations of exam questions!

Available for free at Connexions <http://legacy.cnx.org/content/col11759/1.2>

1080 CHAPTER 9. SEQUENCES, PROBABILITY, AND COUNTING THEORY

AnalysisWe can also use a calculator to �nd permutations. For this problem, we would enter 15, pressthe nPr function, enter 12, and then press the equal sign. The nPr function may be located underthe MATH menu with probability commands.

Q&A: Could we have solved Example 9.33 using the Multiplication Principle?

Yes. We could have multiplied 15 · 14 · 13 · 12 · 11 · 10 · 9 · 8 · 7 · 6 · 5 · 4 to �nd the same answer.

A play has a cast of 7 actors preparing to make their curtain call. Use the permutation formula to �nd thefollowing.

Try It:

Exercise 9.6.6 (Solution on p. 1132.)

How many ways can the 7 actors line up?

Try It:

Exercise 9.6.7 (Solution on p. 1132.)

How many ways can 5 of the 7 actors be chosen to line up?

9.6.4 Find the Number of Combinations Using the Formula

So far, we have looked at problems asking us to put objects in order. There are many problems in whichwe want to select a few objects from a group of objects, but we do not care about the order. Whenwe are selecting objects and the order does not matter, we are dealing with combinations. A selectionof r objects from a set ofn objects where the order does not matter can be written asC (n, r) . Just as withpermutations, C (n, r) can also be written as nCr. In this case, the general formula is as follows.

C (n, r) =n!

r! (n− r)!(9.136)

An earlier problem considered choosing 3 of 4 possible paintings to hang on a wall. We found that therewere 24 ways to select 3 of the 4 paintings in order. But what if we did not care about the order? We wouldexpect a smaller number because selecting paintings 1, 2, 3 would be the same as selecting paintings 2, 3, 1.To �nd the number of ways to select 3 of the 4 paintings, disregarding the order of the paintings, divide thenumber of permutations by the number of ways to order 3 paintings. There are 3! = 3 ·2 ·1 = 6 ways to order3 paintings. There are 24

6 , or 4 ways to select 3 of the 4 paintings. This number makes sense because everytime we are selecting 3 paintings, we are not selecting 1 painting. There are 4 paintings we could choose notto select, so there are 4 ways to select 3 of the 4 paintings.

A General Note: Givenn distinct objects, the number of ways to select r objects from the setis

C (n, r) =n!

r! (n− r)!(9.137)

How To: Given a number of options, determine the possible number of combinations.

Available for free at Connexions <http://legacy.cnx.org/content/col11759/1.2>

1081

1.Identifyn from the given information.2.Identify r from the given information.3.Replacen and r in the formula with the given values.4.Evaluate.

Example 9.34Finding the Number of Combinations Using the FormulaA fast food restaurant o�ers �ve side dish options. Your meal comes with two side dishes.

a. How many ways can you select your side dishes?b. How many ways can you select 3 side dishes?

Solution

a. We want to choose 2 side dishes from 5 options.

C (5, 2) =5!

2! (5− 2)!= 10 (9.138)

b. We want to choose 3 side dishes from 5 options.

C (5, 3) =5!

3! (5− 3)!= 10 (9.139)

AnalysisWe can also use a graphing calculator to �nd combinations. Enter 5, then press nCr, enter 3,and then press the equal sign. The nCr, function may be located under the MATH menu withprobability commands.

Q&A: Is it a coincidence that parts (a) and (b) in Example 9.34 have the same answers?

No. When we choose r objects from n objects, we are not choosing (n−−r) objects.Therefore,C (n, r) = C (n, n−−r) .

Try It:

Exercise 9.6.8 (Solution on p. 1132.)

An ice cream shop o�ers 10 �avors of ice cream. How many ways are there to choose 3�avors for a banana split?

9.6.5 Finding the Number of Subsets of a Set

We have looked only at combination problems in which we chose exactly r objects. In some problems, wewant to consider choosing every possible number of objects. Consider, for example, a pizza restaurant thato�ers 5 toppings. Any number of toppings can be ordered. How many di�erent pizzas are possible?

To answer this question, we need to consider pizzas with any number of toppings. There is C (5, 0) = 1way to order a pizza with no toppings. There are C (5, 1) = 5 ways to order a pizza with exactly one topping.If we continue this process, we get

Available for free at Connexions <http://legacy.cnx.org/content/col11759/1.2>

1082 CHAPTER 9. SEQUENCES, PROBABILITY, AND COUNTING THEORY

C (5, 0) + C (5, 1) + C (5, 2) + C (5, 3) + C (5, 4) + C (5, 5) = 32 (9.140)

There are 32 possible pizzas. This result is equal to 25.We are presented with a sequence of choices. For each of the n objects we have two choices: include it in

the subset or not. So for the whole subset we have made n choices, each with two options. So there are atotal of 2 · 2 · 2 · . . . · 2 possible resulting subsets, all the way from the empty subset, which we obtain whenwe say �no� each time, to the original set itself, which we obtain when we say �yes� each time.

A General Note: A set containing n distinct objects has 2n subsets.

Example 9.35Finding the Number of Subsets of a SetA restaurant o�ers butter, cheese, chives, and sour cream as toppings for a baked potato. Howmany di�erent ways are there to order a potato?

SolutionWe are looking for the number of subsets of a set with 4 objects. Substitute n = 4 into the formula.

2n = 24

= 16(9.141)

There are 16 possible ways to order a potato.

Try It:

Exercise 9.6.9 (Solution on p. 1132.)

A sundae bar at a wedding has 6 toppings to choose from. Any number of toppings canbe chosen. How many di�erent sundaes are possible?

9.6.6 Finding the Number of Permutations of n Non-Distinct Objects

We have studied permutations where all of the objects involved were distinct. What happens if some of theobjects are indistinguishable? For example, suppose there is a sheet of 12 stickers. If all of the stickers weredistinct, there would be 12! ways to order the stickers. However, 4 of the stickers are identical stars, and 3are identical moons. Because all of the objects are not distinct, many of the 12! permutations we countedare duplicates. The general formula for this situation is as follows.

n!r1!r2! . . . rk!

(9.142)

In this example, we need to divide by the number of ways to order the 4 stars and the ways to order the 3moons to �nd the number of unique permutations of the stickers. There are 4! ways to order the stars and3! ways to order the moon.

12!4!3!

= 3,326,400 (9.143)

There are 3,326,400 ways to order the sheet of stickers.

Available for free at Connexions <http://legacy.cnx.org/content/col11759/1.2>

1083

A General Note: If there are n elements in a set and r1 are alike, r2 are alike, r3 are alike, andso on through rk, the number of permutations can be found by

n!r1!r2! . . . rk!

(9.144)

Example 9.36Finding the Number of Permutations of n Non-Distinct ObjectsFind the number of rearrangements of the letters in the word DISTINCT.

SolutionThere are 8 letters. Both I and T are repeated 2 times. Substituten = 8, r1 = 2, and r2 = 2 intothe formula.

8!2!2!

= 10,080 (9.145)

There are 10,080 arrangements.

Try It:

Exercise 9.6.10 (Solution on p. 1132.)

Find the number of rearrangements of the letters in the word CARRIER.

Media: Access these online resources for additional instruction and practice with combinationsand permutations.

• Combinations15• Permutations16

9.6.7 Key Equations

number of permutations ofn distinct objects taken r at a time P(n, r) = n!(n−r)!

number of combinations ofn distinct objects taken r at a time C(n, r) = n!r!(n−r)!

number of permutations ofn non-distinct objects n! r1!r2!...rk!

Table 9.8

15http://openstaxcollege.org/l/combinations16http://openstaxcollege.org/l/permutations

Available for free at Connexions <http://legacy.cnx.org/content/col11759/1.2>

1084 CHAPTER 9. SEQUENCES, PROBABILITY, AND COUNTING THEORY

9.6.8 Key Concepts

• If one event can occur in m ways and a second event with no common outcomes can occur in nways,then the �rst or second event can occur in m+ n ways. See Example 9.30.

• If one event can occur in m ways and a second event can occur in nways after the �rst event hasoccurred, then the two events can occur in m× n ways. See Example 9.31.

• A permutation is an ordering of n objects.• If we have a set of n objects and we want to choose r objects from the set in order, we write P (n, r) .• Permutation problems can be solved using the Multiplication Principle or the formula for P (n, r) .See

Example 9.32 and Example 9.33.• A selection of objects where the order does not matter is a combination.• Given ndistinct objects, the number of ways to select r objects from the set is C (n, r) and can be

found using a formula. See Example 9.34.• A set containing n distinct objects has 2n subsets. See Example 9.35.• For counting problems involving non-distinct objects, we need to divide to avoid counting duplicate

permutations. See Example 9.36.

9.6.9 Section Exercises

9.6.9.1 Verbal

For the following exercises, assume that there are n ways an event A can happen, m ways an event B canhappen, and that A and B are non-overlapping.

Exercise 9.6.11 (Solution on p. 1132.)

Use the Addition Principle of counting to explain how many ways event A or B can occur.

Exercise 9.6.12Use the Multiplication Principle of counting to explain how many ways eventA and B can occur.

Answer the following questions.

Exercise 9.6.13 (Solution on p. 1132.)

When given two separate events, how do we know whether to apply the Addition Principle orthe Multiplication Principle when calculating possible outcomes? What conjunctions may help todetermine which operations to use?

Exercise 9.6.14Describe how the permutation ofn objects di�ers from the permutation of choosing r objects froma set of n objects. Include how each is calculated.

Exercise 9.6.15 (Solution on p. 1132.)

What is the term for the arrangement that selects r objects from a set of n objects when theorder of the r objects is not important? What is the formula for calculating the number of possibleoutcomes for this type of arrangement?

9.6.9.2 Numeric

For the following exercises, determine whether to use the Addition Principle or the Multiplication Principle.Then perform the calculations.

Exercise 9.6.16Let the set A = {−5,−3,−1, 2, 3, 4, 5, 6}. How many ways are there to choose a negative or aneven number from A?

Available for free at Connexions <http://legacy.cnx.org/content/col11759/1.2>

1085

Exercise 9.6.17 (Solution on p. 1132.)

Let the set B = {−23,−16,−7,−2, 20, 36, 48, 72}. How many ways are there to choose a positiveor an odd number from A?Exercise 9.6.18How many ways are there to pick a red ace or a club from a standard card playing deck?

Exercise 9.6.19 (Solution on p. 1132.)

How many ways are there to pick a paint color from 5 shades of green, 4 shades of blue, or 7 shadesof yellow?

Exercise 9.6.20How many outcomes are possible from tossing a pair of coins?

Exercise 9.6.21 (Solution on p. 1132.)

How many outcomes are possible from tossing a coin and rolling a 6-sided die?

Exercise 9.6.22How many two-letter strings�the �rst letter fromA and the second letter fromB − −−can beformed from the setsA = {b, c, d} andB = {a, e, i, o, u}?Exercise 9.6.23 (Solution on p. 1132.)

How many ways are there to construct a string of 3 digits if numbers can be repeated?

Exercise 9.6.24How many ways are there to construct a string of 3 digits if numbers cannot be repeated?

For the following exercises, compute the value of the expression.

Exercise 9.6.25 (Solution on p. 1132.)

P (5, 2)Exercise 9.6.26P (8, 4)Exercise 9.6.27 (Solution on p. 1133.)

P (3, 3)Exercise 9.6.28P (9, 6)Exercise 9.6.29 (Solution on p. 1133.)

P (11, 5)Exercise 9.6.30C (8, 5)Exercise 9.6.31 (Solution on p. 1133.)

C (12, 4)Exercise 9.6.32C (26, 3)Exercise 9.6.33 (Solution on p. 1133.)

C (7, 6)Exercise 9.6.34C (10, 3)

For the following exercises, �nd the number of subsets in each given set.

Exercise 9.6.35 (Solution on p. 1133.)

{1, 2, 3, 4, 5, 6, 7, 8, 9, 10}Exercise 9.6.36{a, b, c, . . . , z}

Available for free at Connexions <http://legacy.cnx.org/content/col11759/1.2>

1086 CHAPTER 9. SEQUENCES, PROBABILITY, AND COUNTING THEORY

Exercise 9.6.37 (Solution on p. 1133.)

A set containing 5 distinct numbers, 4 distinct letters, and 3 distinct symbols

Exercise 9.6.38The set of even numbers from 2 to 28

Exercise 9.6.39 (Solution on p. 1133.)

The set of two-digit numbers between 1 and 100 containing the digit 0

For the following exercises, �nd the distinct number of arrangements.

Exercise 9.6.40The letters in the word �juggernaut�

Exercise 9.6.41 (Solution on p. 1133.)

The letters in the word �academia�

Exercise 9.6.42The letters in the word �academia� that begin and end in �a�

Exercise 9.6.43 (Solution on p. 1133.)

The symbols in the string #,#,#,@,@,$,$,$,%,%,%,%

Exercise 9.6.44The symbols in the string #,#,#,@,@,$,$,$,%,%,%,% that begin and end with �%�

9.6.9.3 Extensions

Exercise 9.6.45 (Solution on p. 1133.)

The set,S consists of 900,000,000whole numbers, each being the same number of digits long. Howmany digits long is a number fromS? (Hint: use the fact that a whole number cannot start withthe digit 0.)

Exercise 9.6.46The number of 5-element subsets from a set containingn elements is equal to the number of 6-element subsets from the same set. What is the value of n? (Hint: the order in which the elementsfor the subsets are chosen is not important.)

Exercise 9.6.47 (Solution on p. 1133.)

Can C (n, r) ever equal P (n, r)? Explain.

Exercise 9.6.48Suppose a set A has 2,048 subsets. How many distinct objects are contained in A?Exercise 9.6.49 (Solution on p. 1133.)

How many arrangements can be made from the letters of the word �mountains� if all the vowelsmust form a string?

9.6.9.4 Real-World Applications

Exercise 9.6.50A family consisting of 2 parents and 3 children is to pose for a picture with 2 family members inthe front and 3 in the back.

a. How many arrangements are possible with no restrictions?b. How many arrangements are possible if the parents must sit in the front?c. How many arrangements are possible if the parents must be next to each other?

Available for free at Connexions <http://legacy.cnx.org/content/col11759/1.2>

1087

Exercise 9.6.51 (Solution on p. 1133.)

A cell phone company o�ers 6 di�erent voice packages and 8 di�erent data packages. Of those, 3packages include both voice and data. How many ways are there to choose either voice or data,but not both?

Exercise 9.6.52In horse racing, a �trifecta� occurs when a bettor wins by selecting the �rst three �nishers in theexact order (1st place, 2nd place, and 3rd place). How many di�erent trifectas are possible if thereare 14 horses in a race?

Exercise 9.6.53 (Solution on p. 1133.)

A wholesale T-shirt company o�ers sizes small, medium, large, and extra-large in organic or non-organic cotton and colors white, black, gray, blue, and red. How many di�erent T-shirts are thereto choose from?

Exercise 9.6.54Hector wants to place billboard advertisements throughout the county for his new business. Howmany ways can Hector choose 15 neighborhoods to advertise in if there are 30 neighborhoods inthe county?

Exercise 9.6.55 (Solution on p. 1133.)

An art store has 4 brands of paint pens in 12 di�erent colors and 3 types of ink. How many paintpens are there to choose from?

Exercise 9.6.56How many ways can a committee of 3 freshmen and 4 juniors be formed from a group of 8 freshmenand 11 juniors?Exercise 9.6.57 (Solution on p. 1133.)

How many ways can a baseball coach arrange the order of 9 batters if there are 15 players on theteam?

Exercise 9.6.58A conductor needs 5 cellists and 5 violinists to play at a diplomatic event. To do this, he ranksthe orchestra's 10 cellists and 16 violinists in order of musical pro�ciency. What is the ratio of thetotal cellist rankings possible to the total violinist rankings possible?

Exercise 9.6.59 (Solution on p. 1133.)

A motorcycle shop has 10 choppers, 6 bobbers, and 5 café racers�di�erent types of vintagemotorcycles. How many ways can the shop choose 3 choppers, 5 bobbers, and 2 café racers for aweekend showcase?

Exercise 9.6.60A skateboard shop stocks 10 types of board decks, 3 types of trucks, and 4 types of wheels. Howmany di�erent skateboards can be constructed?

Exercise 9.6.61 (Solution on p. 1133.)

Just-For-Kicks Sneaker Company o�ers an online customizing service. How many ways are thereto design a custom pair of Just-For-Kicks sneakers if a customer can choose from a basic shoe upto 11 customizable options?

Exercise 9.6.62A car wash o�ers the following optional services to the basic wash: clear coat wax, triple foampolish, undercarriage wash, rust inhibitor, wheel brightener, air freshener, and interior shampoo.How many washes are possible if any number of options can be added to the basic wash?

Exercise 9.6.63 (Solution on p. 1133.)

Susan bought 20 plants to arrange along the border of her garden. How many distinct arrangementscan she make if the plants are comprised of 6 tulips, 6 roses, and 8 daisies?

Available for free at Connexions <http://legacy.cnx.org/content/col11759/1.2>

1088 CHAPTER 9. SEQUENCES, PROBABILITY, AND COUNTING THEORY

Exercise 9.6.64How many unique ways can a string of Christmas lights be arranged from 9 red, 10 green, 6 white,and 12 gold color bulbs?

9.7 Binomial Theorem17

A polynomial with two terms is called a binomial. We have already learned to multiply binomials and toraise binomials to powers, but raising a binomial to a high power can be tedious and time-consuming. Inthis section, we will discuss a shortcut that will allow us to �nd (x+ y)n without multiplying the binomialby itself n times.

9.7.1 Identifying Binomial Coe�cients

In Counting Principles (Section 9.6), we studied combinations. In the shortcut to �nding (x+ y)n, we willneed to use combinations to �nd the coe�cients that will appear in the expansion of the binomial. In this

case, we use the notation

n

r

instead of C (n, r) , but it can be calculated in the same way. So

n

r

= C (n, r) =n!

r! (n− r)!(9.146)

The combination

n

r

is called a binomial coe�cient. An example of a binomial coe�cient

is

5

2

= C (5, 2) = 10.

A General Note: If n and rare integers greater than or equal to 0 with n ≥ r, then thebinomial coe�cient is

n

r

= C (n, r) =n!

r! (n− r)!(9.147)

Q&A: Is a binomial coe�cient always a whole number?

Yes. Just as the number of combinations must always be a whole number, a binomial coe�cientwill always be a whole number.

Example 9.37Finding Binomial Coe�cientsFind each binomial coe�cient.

a.

5

3

17This content is available online at <http://legacy.cnx.org/content/m49449/1.7/>.

Available for free at Connexions <http://legacy.cnx.org/content/col11759/1.2>

1089

b.

9

2

c.

9

7

SolutionUse the formula to calculate each binomial coe�cient. You can also use the nCr function on yourcalculator. n

r

= C (n, r) =n!

r! (n− r)!(9.148)

a.

5

3

= 5!3!(5−3)! = 5·4·3!

3!2! = 10

b.

9

2

= 9!2!(9−2)! = 9·8·7!

2!7! = 36

c.

9

7

= 9!7!(9−7)! = 9·8·7!

7!2! = 36

AnalysisNotice that we obtained the same result for parts (b) and (c). If you look closely at the solutionfor these two parts, you will see that you end up with the same two factorials in the denominator,but the order is reversed, just as with combinations. n

r

=

n

n− r

(9.149)

Try It: Find each binomial coe�cient.

Exercise 9.7.1 (Solution on p. 1133.)

a.

7

3

b.

11

4

Available for free at Connexions <http://legacy.cnx.org/content/col11759/1.2>

1090 CHAPTER 9. SEQUENCES, PROBABILITY, AND COUNTING THEORY

9.7.2 Using the Binomial Theorem

When we expand (x+ y)n by multiplying, the result is called a binomial expansion, and it includes

binomial coe�cients. If we wanted to expand (x+ y)52, we might multiply (x+ y) by itself �fty-two times.

This could take hours! If we examine some simple binomial expansions, we can �nd patterns that will leadus to a shortcut for �nding more complicated binomial expansions.

(x+ y)2 = x2 + 2xy + y2

(x+ y)3 = x3 + 3x2y + 3xy2 + y3

(x+ y)4 = x4 + 4x3y + 6x2y2 + 4xy3 + y4

(9.150)

First, let's examine the exponents. With each successive term, the exponent for x decreases and the exponentfor y increases. The sum of the two exponents is n for each term.

Next, let's examine the coe�cients. Notice that the coe�cients increase and then decrease in a symmet-rical pattern. The coe�cients follow a pattern: n

0

,

n

1

,

n

2

, ...,

n

n

. (9.151)

These patterns lead us to the Binomial Theorem, which can be used to expand any binomial.

(x+ y)n =∑nk=0

n

k

xn−kyk

= xn +

n

1

xn−1y +

n

2

xn−2y2 + ...+

n

n− 1

xyn−1 + yn

(9.152)

Another way to see the coe�cients is to examine the expansion of a binomial in general form,x + y, tosuccessive powers 1, 2, 3, and 4.

(x+ y)1 = x+ y

(x+ y)2 = x2 + 2xy + y2

(x+ y)3 = x3 + 3x2y + 3xy2 + y3

(x+ y)4 = x4 + 4x3y + 6x2y2 + 4xy3 + y4

(9.153)

Can you guess the next expansion for the binomial (x+ y)5?

Available for free at Connexions <http://legacy.cnx.org/content/col11759/1.2>

1091

Figure 9.21

See Figure 9.21, which illustrates the following:

• There are n+ 1 terms in the expansion of (x+ y)n.• The degree (or sum of the exponents) for each term is n.• The powers on x begin with n and decrease to 0.• The powers on y begin with 0 and increase to n.• The coe�cients are symmetric.

To determine the expansion on (x+ y)5, we see n = 5, thus, there will be 5+1 = 6 terms. Each term has a

combined degree of 5. In descending order for powers of x, the pattern is as follows:

• Introduce x5, and then for each successive term reduce the exponent on x by 1 until x0 = 1 is reached.• Introduce y0 = 1, and then increase the exponent on y by 1 until y5 is reached.

x5, x4y, x3y2, x2y3, xy4, y5 (9.154)

The next expansion would be

(x+ y)5 = x5 + 5x4y + 10x3y2 + 10x2y3 + 5xy4 + y5. (9.155)

But where do those coe�cients come from? The binomial coe�cients are symmetric. We can see thesecoe�cients in an array known as Pascal's Triangle, shown in Figure 9.22.

Available for free at Connexions <http://legacy.cnx.org/content/col11759/1.2>

1092 CHAPTER 9. SEQUENCES, PROBABILITY, AND COUNTING THEORY

Figure 9.22

To generate Pascal's Triangle, we start by writing a 1. In the row below, row 2, we write two 1's. In the3rd row, �ank the ends of the rows with 1's, and add 1 + 1 to �nd the middle number, 2. In the nth row,�ank the ends of the row with 1's. Each element in the triangle is the sum of the two elements immediatelyabove it.

To see the connection between Pascal's Triangle and binomial coe�cients, let us revisit the expansion ofthe binomials in general form.

A General Note: The Binomial Theorem is a formula that can be used to expand anybinomial.

(x+ y)n =∑nk=0

n

k

xn−kyk

= xn +

n

1

xn−1y +

n

2

xn−2y2 + ...+

n

n− 1

xyn−1 + yn

(9.156)

How To: Given a binomial, write it in expanded form.

1.Determine the value of naccording to the exponent.2.Evaluate the k = 0 through k = n using the Binomial Theorem formula.

Available for free at Connexions <http://legacy.cnx.org/content/col11759/1.2>

1093

3.Simplify.

Example 9.38Expanding a BinomialWrite in expanded form.

a. (x+ y)5

b. (3x− y)4

Solution

a. Substitute n = 5 into the formula. Evaluate the k = 0 through k = 5 terms. Simplify.

(x + y)5 =

(5

0

)x5y0 +

(5

1

)x4y1 +

(5

2

)x3y2 +

(5

3

)x2y3 +

(5

4

)x1y4 +

(5

5

)x0y5

(x + y)5 = x5 + 5x4y + 10x3y2 + 10x2y3 + 5xy4 + y5

(9.157)

b. Substitute n = 4 into the formula. Evaluate the k = 0 through k = 4 terms. Notice that 3xis in the place that was occupied by x and that −− y is in the place that was occupied by y.So we substitute them. Simplify.

(3x− y)4 =

(4

0

)(3x)4(−y)0 +

(4

1

)(3x)3(−y)1 +

(4

2

)(3x)2(−y)2 +

(4

3

)(3x)1(−y)3 +

(4

4

)(3x)0(−y)4

(3x− y)4 = 81x4 − 108x3y + 54x2y2 − 12xy3 + y4

(9.158)

AnalysisNotice the alternating signs in part b. This happens because (−y) raised to odd powers is negative,but (−y) raised to even powers is positive. This will occur whenever the binomial contains asubtraction sign.

Try It:

Exercise 9.7.2 (Solution on p. 1133.)

Write in expanded form.

a.(x− y)5

b.(2x+ 5y)3

Available for free at Connexions <http://legacy.cnx.org/content/col11759/1.2>

1094 CHAPTER 9. SEQUENCES, PROBABILITY, AND COUNTING THEORY

9.7.3 Using the Binomial Theorem to Find a Single Term

Expanding a binomial with a high exponent such as (x+ 2y)16can be a lengthy process.

Sometimes we are interested only in a certain term of a binomial expansion. We do not need to fullyexpand a binomial to �nd a single speci�c term.

Note the pattern of coe�cients in the expansion of (x+ y)5.

(x+ y)5 = x5 +

5

1

x4y +

5

2

x3y2 +

5

3

x2y3 +

5

4

xy4 + y5 (9.159)

The second term is

5

1

x4y.The third term is

5

2

x3y2.We can generalize this result.

n

r

xn−ryr (9.160)

A General Note: The (r + 1) th term of the binomial expansion of (x+ y)n is: n

r

xn−ryr (9.161)

How To: Given a binomial, write a speci�c term without fully expanding.

1.Determine the value of n according to the exponent.2.Determine (r + 1) .3.Determine r.4.Replace r in the formula for the (r + 1) th term of the binomial expansion.

Example 9.39Writing a Given Term of a Binomial ExpansionFind the tenth term of (x+ 2y)16

without fully expanding the binomial.

SolutionBecause we are looking for the tenth term, r + 1 = 10, we will use r = 9 in our calculations. n

r

xn−ryr (9.162)

16

9

x16−9(2y)9 = 5,857,280x7y9 (9.163)

Available for free at Connexions <http://legacy.cnx.org/content/col11759/1.2>

1095

Try It:

Exercise 9.7.3 (Solution on p. 1134.)

Find the sixth term of (3x− y)9without fully expanding the binomial.

Media: Access these online resources for additional instruction and practice with binomial ex-pansion.

• The Binomial Theorem18

• Binomial Theorem Example19

9.7.4 Key Equations

Binomial Theorem (x+ y)n =∑nk−0

n

k

xn−kyk

(r + 1) th term of a binomial expansion

n

r

xn−ryr

Table 9.9

9.7.5 Key Concepts

n

r

is called a binomial coe�cient and is equal to C (n, r) . See Example 9.37.

• The Binomial Theorem allows us to expand binomials without multiplying. See Example 9.38.• We can �nd a given term of a binomial expansion without fully expanding the binomial. See Exam-

ple 9.39.

9.7.6 Section Exercises

9.7.6.1 Verbal

Exercise 9.7.4 (Solution on p. 1134.)

What is a binomial coe�cient, and how it is calculated?

Exercise 9.7.5What role do binomial coe�cients play in a binomial expansion? Are they restricted to any typeof number?

Exercise 9.7.6 (Solution on p. 1134.)

What is the Binomial Theorem and what is its use?

Exercise 9.7.7When is it an advantage to use the Binomial Theorem? Explain.

18http://openstaxcollege.org/l/binomialtheorem19http://openstaxcollege.org/l/btexample

Available for free at Connexions <http://legacy.cnx.org/content/col11759/1.2>

1096 CHAPTER 9. SEQUENCES, PROBABILITY, AND COUNTING THEORY

9.7.6.2 Algebraic

For the following exercises, evaluate the binomial coe�cient.

Exercise 9.7.8 (Solution on p. 1134.) 6

2

Exercise 9.7.9 5

3

Exercise 9.7.10 (Solution on p. 1134.) 7

4

Exercise 9.7.11 9

7

Exercise 9.7.12 (Solution on p. 1134.) 10

9

Exercise 9.7.13 25

11

Exercise 9.7.14 (Solution on p. 1134.) 17

6

Exercise 9.7.15 200

199

For the following exercises, use the Binomial Theorem to expand each binomial.

Exercise 9.7.16 (Solution on p. 1134.)

(4a− b)3

Exercise 9.7.17(5a+ 2)3

Exercise 9.7.18 (Solution on p. 1134.)

(3a+ 2b)3

Exercise 9.7.19(2x+ 3y)4

Exercise 9.7.20 (Solution on p. 1134.)

(4x+ 2y)5

Exercise 9.7.21(3x− 2y)4

Available for free at Connexions <http://legacy.cnx.org/content/col11759/1.2>

1097

Exercise 9.7.22 (Solution on p. 1134.)

(4x− 3y)5

Exercise 9.7.23(1x + 3y

)5Exercise 9.7.24 (Solution on p. 1134.)(x−1 + 2y−1

)4Exercise 9.7.25(√

x−√y)5

For the following exercises, use the Binomial Theorem to write the �rst three terms of each binomial.

Exercise 9.7.26 (Solution on p. 1134.)

(a+ b)17

Exercise 9.7.27(x− 1)18

Exercise 9.7.28 (Solution on p. 1134.)

(a− 2b)15

Exercise 9.7.29(x− 2y)8

Exercise 9.7.30 (Solution on p. 1134.)

(3a+ b)20

Exercise 9.7.31(2a+ 4b)7

Exercise 9.7.32 (Solution on p. 1134.)(x3 −√y

)8For the following exercises, �nd the indicated term of each binomial without fully expanding the binomial.

Exercise 9.7.33The fourth term of (2x− 3y)4

Exercise 9.7.34 (Solution on p. 1134.)

The fourth term of (3x− 2y)5

Exercise 9.7.35The third term of (6x− 3y)7

Exercise 9.7.36 (Solution on p. 1134.)

The eighth term of (7 + 5y)14

Exercise 9.7.37The seventh term of (a+ b)11

Exercise 9.7.38 (Solution on p. 1134.)

The �fth term of (x− y)7

Exercise 9.7.39The tenth term of (x− 1)12

Exercise 9.7.40 (Solution on p. 1134.)

The ninth term of(a− 3b2

)11

Exercise 9.7.41The fourth term of

(x3 − 1

2

)10

Exercise 9.7.42 (Solution on p. 1134.)

The eighth term of(y2 + 2

x

)9

Available for free at Connexions <http://legacy.cnx.org/content/col11759/1.2>

1098 CHAPTER 9. SEQUENCES, PROBABILITY, AND COUNTING THEORY

9.7.6.3 Graphical

For the following exercises, use the Binomial Theorem to expand the binomial f (x) = (x+ 3)4. Then �nd

and graph each indicated sum on one set of axes.

Exercise 9.7.43Find and graph f1 (x) , such that f1 (x) is the �rst term of the expansion.

Exercise 9.7.44 (Solution on p. 1134.)

Find and graph f2 (x) , such that f2 (x) is the sum of the �rst two terms of the expansion.

Exercise 9.7.45Find and graph f3 (x) , such that f3 (x) is the sum of the �rst three terms of the expansion.

Exercise 9.7.46 (Solution on p. 1135.)

Find and graph f4 (x) , such that f4 (x) is the sum of the �rst four terms of the expansion.

Exercise 9.7.47Find and graph f5 (x) , such that f5 (x) is the sum of the �rst �ve terms of the expansion.

9.7.6.4 Extensions

Exercise 9.7.48 (Solution on p. 1136.)

In the expansion of (5x+ 3y)n, each term has the form

n

k

an−−kbk, where k successively takes

on the value 0, 1, 2, ..., n.If

n

k

=

7

2

, what is the corresponding term?

Exercise 9.7.49In the expansion of (a+ b)n, the coe�cient of an−kbk is the same as the coe�cient of which otherterm?

Exercise 9.7.50 (Solution on p. 1136.)

Consider the expansion of (x+ b)40.What is the exponent of b in the kth term?

Exercise 9.7.51 (Solution on p. 1136.)

Find

n

k − 1

+

n

k

and write the answer as a binomial coe�cient in the

form

n

k

.Prove it. Hint: Use the fact that, for any integer p, such that p ≥ 1, p! = p (p− 1)!.

Exercise 9.7.52 (Solution on p. 1137.)

Which expression cannot be expanded using the Binomial Theorem? Explain.

•(x2 − 2x+ 1

)• (√a+ 4

√a− 5)8

•(x3 + 2y2 − z

)5•(

3x2 −√

2y3)12

Available for free at Connexions <http://legacy.cnx.org/content/col11759/1.2>

1099

9.8 Probability20

Figure 9.23: An example of a �spaghetti model,� which can be used to predict possible paths of atropical storm.21

Residents of the Southeastern United States are all too familiar with charts, known as spaghetti models,such as the one in Figure 9.23. They combine a collection of weather data to predict the most likely pathof a hurricane. Each colored line represents one possible path. The group of squiggly lines can begin toresemble strands of spaghetti, hence the name. In this section, we will investigate methods for making thesetypes of predictions.

9.8.1 Constructing Probability Models

Suppose we roll a six-sided number cube. Rolling a number cube is an example of an experiment, oran activity with an observable result. The numbers on the cube are possible results, or outcomes, of thisexperiment. The set of all possible outcomes of an experiment is called the sample space of the experiment.The sample space for this experiment is {1, 2, 3, 4, 5, 6}.An event is any subset of a sample space.

The likelihood of an event is known as probability. The probability of an event p is a number that alwayssatis�es 0 ≤ p ≤ 1, where 0 indicates an impossible event and 1 indicates a certain event. A probabilitymodel is a mathematical description of an experiment listing all possible outcomes and their associatedprobabilities. For instance, if there is a 1% chance of winning a ra�e and a 99% chance of losing the ra�e,a probability model would look much like Table 9.10.

20This content is available online at <http://legacy.cnx.org/content/m49450/1.4/>.

Available for free at Connexions <http://legacy.cnx.org/content/col11759/1.2>

1100 CHAPTER 9. SEQUENCES, PROBABILITY, AND COUNTING THEORY

Outcome Probability

Winning the ra�e 1%

Losing the ra�e 99%

Table 9.10

The sum of the probabilities listed in a probability model must equal 1, or 100%.

How To: Given a probability event where each event is equally likely, construct aprobability model.

1.Identify every outcome.2.Determine the total number of possible outcomes.3.Compare each outcome to the total number of possible outcomes.

Example 9.40Constructing a Probability ModelConstruct a probability model for rolling a single, fair die, with the event being the number shownon the die.

SolutionBegin by making a list of all possible outcomes for the experiment. The possible outcomes are thenumbers that can be rolled: 1, 2, 3, 4, 5, and 6. There are six possible outcomes that make up thesample space.

Assign probabilities to each outcome in the sample space by determining a ratio of the outcometo the number of possible outcomes. There is one of each of the six numbers on the cube, and thereis no reason to think that any particular face is more likely to show up than any other one, so theprobability of rolling any number is 1

6 .

Outcome Roll of 1 Roll of 2 Roll of 3 Roll of 4 Roll of 5 Roll of 6

Probability 16

16

16

16

16

16

Table 9.11

Q&A: Do probabilities always have to be expressed as fractions?

No. Probabilities can be expressed as fractions, decimals, or percents. Probability must always bea number between 0 and 1, inclusive of 0 and 1.

Try It:

Exercise 9.8.1 (Solution on p. 1137.)

Construct a probability model for tossing a fair coin.

Available for free at Connexions <http://legacy.cnx.org/content/col11759/1.2>

1101

9.8.2 Computing Probabilities of Equally Likely Outcomes

LetS be a sample space for an experiment. When investigating probability, an event is any subset ofS.Whenthe outcomes of an experiment are all equally likely, we can �nd the probability of an event by dividing thenumber of outcomes in the event by the total number of outcomes inS. Suppose a number cube is rolled,and we are interested in �nding the probability of the event �rolling a number less than or equal to 4.� Thereare 4 possible outcomes in the event and 6 possible outcomes inS, so the probability of the event is 4

6 = 23 .

A General Note: The probability of an event E in an experiment with sample space S withequally likely outcomes is given by

P (E) =number of elements in E

number of elements in S=n (E)n (S)

(9.164)

E is a subset of S, so it is always true that 0 ≤ P (E) ≤ 1.

Example 9.41Computing the Probability of an Event with Equally Likely OutcomesA number cube is rolled. Find the probability of rolling an odd number.

SolutionThe event �rolling an odd number� contains three outcomes. There are 6 equally likely outcomesin the sample space. Divide to �nd the probability of the event.

P (E) =36

=12

(9.165)

Try It:

Exercise 9.8.2 (Solution on p. 1137.)

A number cube is rolled. Find the probability of rolling a number greater than 2.

9.8.3 Computing the Probability of the Union of Two Events

We are often interested in �nding the probability that one of multiple events occurs. Suppose we are playing acard game, and we will win if the next card drawn is either a heart or a king. We would be interested in �ndingthe probability of the next card being a heart or a king. The union of two eventsE and F,written E∪F, isthe event that occurs if either or both events occur.

P (E ∪ F ) = P (E) + P (F )− P (E ∩ F ) (9.166)

Suppose the spinner in Figure 9.24 is spun. We want to �nd the probability of spinning orange or spinninga b.

Available for free at Connexions <http://legacy.cnx.org/content/col11759/1.2>

1102 CHAPTER 9. SEQUENCES, PROBABILITY, AND COUNTING THEORY

Figure 9.24

There are a total of 6 sections, and 3 of them are orange. So the probability of spinning orange is 36 =

12 .There are a total of 6 sections, and 2 of them have a b. So the probability of spinning a b is 2

6 = 13 . If we

added these two probabilities, we would be counting the sector that is both orange and a b twice. To �nd theprobability of spinning an orange or a b, we need to subtract the probability that the sector is both orangeand has a b.

12

+13− 1

6=

23

(9.167)

The probability of spinning orange or a b is 23 .

A General Note: The probability of the union of two events E and F (written E ∪ F ) equalsthe sum of the probability of E and the probability of F minus the probability of E and F occurringtogether (which is called the intersection of E and F and is written as E ∩ F ).

P (E ∪ F ) = P (E) + P (F )− P (E ∩ F ) (9.168)

Example 9.42Computing the Probability of the Union of Two EventsA card is drawn from a standard deck. Find the probability of drawing a heart or a 7.

Available for free at Connexions <http://legacy.cnx.org/content/col11759/1.2>

1103

SolutionA standard deck contains an equal number of hearts, diamonds, clubs, and spades. So the prob-ability of drawing a heart is 1

4 .There are four 7s in a standard deck, and there are a total of 52cards. So the probability of drawing a 7 is 1

13 .The only card in the deck that is both a heart and a 7 is the 7 of hearts, so the probability of

drawing both a heart and a 7 is 152 . SubstituteP (H) = 1

4 , P (7) = 113 , and P (H ∩ 7) = 1

52 into theformula.

P (E∪F ) = P (E) + P (F )− P (E∩F )

= 14 + 1

13 −152

= 413

(9.169)

The probability of drawing a heart or a 7 is 413 .

Try It:

Exercise 9.8.3 (Solution on p. 1137.)

A card is drawn from a standard deck. Find the probability of drawing a red card or anace.

9.8.4 Computing the Probability of Mutually Exclusive Events

Suppose the spinner in Figure 9.24 is spun again, but this time we are interested in the probability of spinningan orange or a d.There are no sectors that are both orange and contain a d, so these two events have nooutcomes in common. Events are said to be mutually exclusive events when they have no outcomes incommon. Because there is no overlap, there is nothing to subtract, so the general formula is

P (E ∪ F ) = P (E) + P (F ) (9.170)

Notice that with mutually exclusive events, the intersection ofE andF is the empty set. The probability ofspinning an orange is 3

6 = 12 and the probability of spinning a d is 1

6 .We can �nd the probability of spinningan orange or a d simply by adding the two probabilities.

P (E∪F ) = P (E) + P (F )

= 12 + 1

6

= 23

(9.171)

The probability of spinning an orange or a d is 23 .

A General Note: The probability of the union of two mutually exclusive eventsE andF is givenby

P (E ∪ F ) = P (E) + P (F ) (9.172)

How To: Given a set of events, compute the probability of the union of mutuallyexclusive events.

Available for free at Connexions <http://legacy.cnx.org/content/col11759/1.2>

1104 CHAPTER 9. SEQUENCES, PROBABILITY, AND COUNTING THEORY

1.Determine the total number of outcomes for the �rst event.2.Find the probability of the �rst event.3.Determine the total number of outcomes for the second event.4.Find the probability of the second event.5.Add the probabilities.

Example 9.43Computing the Probability of the Union of Mutually Exclusive EventsA card is drawn from a standard deck. Find the probability of drawing a heart or a spade.

SolutionThe events �drawing a heart� and �drawing a spade� are mutually exclusive because they cannotoccur at the same time. The probability of drawing a heart is 1

4 , and the probability of drawing aspade is also 1

4 , so the probability of drawing a heart or a spade is

14

+14

=12

(9.173)

Try It:

Exercise 9.8.4 (Solution on p. 1137.)

A card is drawn from a standard deck. Find the probability of drawing an ace or a king.

9.8.5 Using the Complement Rule to Compute Probabilities

We have discussed how to calculate the probability that an event will happen. Sometimes, we are interestedin �nding the probability that an event will not happen. The complement of an eventE, denotedE′, isthe set of outcomes in the sample space that are not inE.For example, suppose we are interested in theprobability that a horse will lose a race. If eventW is the horse winning the race, then the complement ofeventW is the horse losing the race.

To �nd the probability that the horse loses the race, we need to use the fact that the sum of all probabilitiesin a probability model must be 1.

P (E′) = 1− P (E) (9.174)

The probability of the horse winning added to the probability of the horse losing must be equal to 1.Therefore, if the probability of the horse winning the race is 1

9 , the probability of the horse losing the race issimply

1− 19

=89

(9.175)

A General Note: The probability that the complement of an event will occur is given by

P (E′) = 1− P (E) (9.176)

Available for free at Connexions <http://legacy.cnx.org/content/col11759/1.2>

1105

Example 9.44Using the Complement Rule to Calculate ProbabilitiesTwo six-sided number cubes are rolled.

a. Find the probability that the sum of the numbers rolled is less than or equal to 3.b. Find the probability that the sum of the numbers rolled is greater than 3.

SolutionThe �rst step is to identify the sample space, which consists of all the possible outcomes. Thereare two number cubes, and each number cube has six possible outcomes. Using the MultiplicationPrinciple, we �nd that there are 6×6, or 36 total possible outcomes. So, for example, 1-1 representsa 1 rolled on each number cube.

1-1 1-2 1-3 1-4 1-5 1-6

2-1 2-2 2-3 2-4 2-5 2-6

3-1 3-2 3-3 3-4 3-5 3-6

4-1 4-2 4-3 4-4 4-5 4-6

5-1 5-2 5-3 5-4 5-5 5-6

6-1 6-2 6-3 6-4 6-5 6-6

Table 9.12

a. We need to count the number of ways to roll a sum of 3 or less. These would include thefollowing outcomes: 1-1, 1-2, and 2-1. So there are only three ways to roll a sum of 3 or less.The probability is

336

=112

(9.177)

b. Rather than listing all the possibilities, we can use the Complement Rule. Because we havealready found the probability of the complement of this event, we can simply subtract thatprobability from 1 to �nd the probability that the sum of the numbers rolled is greater than3.

P (E′) = 1− P (E)

= 1− 112

= 1112

(9.178)

Try It:

Exercise 9.8.5 (Solution on p. 1137.)

Two number cubes are rolled. Use the Complement Rule to �nd the probability that thesum is less than 10.

Available for free at Connexions <http://legacy.cnx.org/content/col11759/1.2>

1106 CHAPTER 9. SEQUENCES, PROBABILITY, AND COUNTING THEORY

9.8.6 Computing Probability Using Counting Theory

Many interesting probability problems involve counting principles, permutations, and combinations. In theseproblems, we will use permutations and combinations to �nd the number of elements in events and samplespaces. These problems can be complicated, but they can be made easier by breaking them down into smallercounting problems.

Assume, for example, that a store has 8 cellular phones and that 3 of those are defective. We mightwant to �nd the probability that a couple purchasing 2 phones receives 2 phones that are not defective. Tosolve this problem, we need to calculate all of the ways to select 2 phones that are not defective as well asall of the ways to select 2 phones. There are 5 phones that are not defective, so there areC (5, 2) ways toselect 2 phones that are not defective. There are 8 phones, so there areC (8, 2) ways to select 2 phones. Theprobability of selecting 2 phones that are not defective is:

ways to select 2 phones that are not defectiveways to select 2 phones = C(5,2)

C(8,2)

= 1028

= 514

(9.179)

Example 9.45Computing Probability Using Counting TheoryA child randomly selects 5 toys from a bin containing 3 bunnies, 5 dogs, and 6 bears.

a. Find the probability that only bears are chosen.b. Find the probability that 2 bears and 3 dogs are chosen.c. Find the probability that at least 2 dogs are chosen.

Solution

a. We need to count the number of ways to choose only bears and the total number of possibleways to select 5 toys. There are 6 bears, so there areC (6, 5) ways to choose 5 bears. Thereare 14 toys, so there areC (14, 5) ways to choose any 5 toys.

C (6,5)C (14,5)

=6

2,002=

31,001

(9.180)

b. We need to count the number of ways to choose 2 bears and 3 dogs and the total number ofpossible ways to select 5 toys. There are 6 bears, so there areC (6, 2) ways to choose 2 bears.There are 5 dogs, so there areC (5, 3) ways to choose 3 dogs. Since we are choosing bothbears and dogs at the same time, we will use the Multiplication Principle. There areC (6, 2) ·C (5, 3) ways to choose 2 bears and 3 dogs. We can use this result to �nd the probability.

C (6,2)C (5,3)C (14,5)

=15 · 102,002

=75

1,001(9.181)

c. It is often easiest to solve �at least� problems using the Complement Rule. We will begin by�nding the probability that fewer than 2 dogs are chosen. If less than 2 dogs are chosen, theneither no dogs could be chosen, or 1 dog could be chosen. When no dogs are chosen, all 5toys come from the 9 toys that are not dogs. There areC (9, 5) ways to choose toys from the9 toys that are not dogs. Since there are 14 toys, there areC (14, 5) ways to choose the 5 toysfrom all of the toys.

C (9,5)C (14,5)

=63

1,001(9.182)

Available for free at Connexions <http://legacy.cnx.org/content/col11759/1.2>

1107

If there is 1 dog chosen, then 4 toys must come from the 9 toys that are not dogs, and 1 mustcome from the 5 dogs. Since we are choosing both dogs and other toys at the same time, wewill use the Multiplication Principle. There areC (5, 1) · C (9, 4) ways to choose 1 dog and 1other toy.

C (5,1)C (9,4)C (14,5)

=5 · 1262,002

=315

1,001(9.183)

Because these events would not occur together and are therefore mutually exclusive, we addthe probabilities to �nd the probability that fewer than 2 dogs are chosen.

631,001

+315

1,001=

3781,001

(9.184)

We then subtract that probability from 1 to �nd the probability that at least 2 dogs arechosen.

1− 3781,001

=623

1,001(9.185)

Try It:

Exercise 9.8.6 (Solution on p. 1137.)

A child randomly selects 3 gumballs from a container holding 4 purple gumballs, 8 yellowgumballs, and 2 green gumballs.

a.Find the probability that all 3 gumballs selected are purple.b.Find the probability that no yellow gumballs are selected.c.Find the probability that at least 1 yellow gumball is selected.

Media: Access these online resources for additional instruction and practice with probability.

• Introduction to Probability22

• Determining Probability23

Visit this website24 for additional practice questions from Learningpod.

9.8.7 Key Equations

probability of an event with equally likely outcomes P(E) = n(E)n(S)

probability of the union of two events P(E ∪ F ) = P (E) + P (F )− P (E ∩ F )

probability of the union of mutually exclusive events P(E ∪ F ) = P (E) + P (F )

probability of the complement of an event P(E') = 1− P (E)

Table 9.13

22http://openstaxcollege.org/l/introprob23http://openstaxcollege.org/l/determineprob24http://openstaxcollege.org/l/PreCalcLPC11

Available for free at Connexions <http://legacy.cnx.org/content/col11759/1.2>

1108 CHAPTER 9. SEQUENCES, PROBABILITY, AND COUNTING THEORY

9.8.8 Key Concepts

• Probability is always a number between 0 and 1, where 0 means an event is impossible and 1 meansan event is certain.

• The probabilities in a probability model must sum to 1. See Example 9.40.• When the outcomes of an experiment are all equally likely, we can �nd the probability of an event by

dividing the number of outcomes in the event by the total number of outcomes in the sample space forthe experiment. See Example 9.41.

• To �nd the probability of the union of two events, we add the probabilities of the two events andsubtract the probability that both events occur simultaneously. See Example 9.42.

• To �nd the probability of the union of two mutually exclusive events, we add the probabilities of eachof the events. See Example 9.43.

• The probability of the complement of an event is the di�erence between 1 and the probability that theevent occurs. See Example 9.44.

• In some probability problems, we need to use permutations and combinations to �nd the number ofelements in events and sample spaces. See Example 9.45.

9.8.9 Section Exercises

9.8.9.1 Verbal

Exercise 9.8.7 (Solution on p. 1137.)

What term is used to express the likelihood of an event occurring? Are there restrictions on itsvalues? If so, what are they? If not, explain.

Exercise 9.8.8What is a sample space?

Exercise 9.8.9 (Solution on p. 1137.)

What is an experiment?

Exercise 9.8.10What is the di�erence between events and outcomes? Give an example of both using the samplespace of tossing a coin 50 times.

Exercise 9.8.11 (Solution on p. 1138.)

The union of two sets is de�ned as a set of elements that are present in at least one of the sets.How is this similar to the de�nition used for the union of two events from a probability model?How is it di�erent?

9.8.9.2 Numeric

For the following exercises, use the spinner shown in Figure 9.25 to �nd the probabilities indicated.

Available for free at Connexions <http://legacy.cnx.org/content/col11759/1.2>

1109

Figure 9.25

Exercise 9.8.12Landing on red

Exercise 9.8.13 (Solution on p. 1138.)

Landing on a vowel

Exercise 9.8.14Not landing on blue

Exercise 9.8.15 (Solution on p. 1138.)

Landing on purple or a vowel

Exercise 9.8.16Landing on blue or a vowel

Exercise 9.8.17 (Solution on p. 1138.)

Landing on green or blue

Exercise 9.8.18Landing on yellow or a consonant

Exercise 9.8.19 (Solution on p. 1138.)

Not landing on yellow or a consonant

For the following exercises, two coins are tossed.

Available for free at Connexions <http://legacy.cnx.org/content/col11759/1.2>

1110 CHAPTER 9. SEQUENCES, PROBABILITY, AND COUNTING THEORY

Exercise 9.8.20What is the sample space?

Exercise 9.8.21 (Solution on p. 1138.)

Find the probability of tossing two heads.

Exercise 9.8.22Find the probability of tossing exactly one tail.

Exercise 9.8.23 (Solution on p. 1138.)

Find the probability of tossing at least one tail.

For the following exercises, four coins are tossed.

Exercise 9.8.24What is the sample space?

Exercise 9.8.25 (Solution on p. 1138.)

Find the probability of tossing exactly two heads.

Exercise 9.8.26Find the probability of tossing exactly three heads.

Exercise 9.8.27 (Solution on p. 1138.)

Find the probability of tossing four heads or four tails.

Exercise 9.8.28Find the probability of tossing all tails.

Exercise 9.8.29 (Solution on p. 1138.)

Find the probability of tossing not all tails.

Exercise 9.8.30Find the probability of tossing exactly two heads or at least two tails.

Exercise 9.8.31 (Solution on p. 1138.)

Find the probability of tossing either two heads or three heads.

For the following exercises, one card is drawn from a standard deck of 52 cards. Find the probability ofdrawing the following:

Exercise 9.8.32A club

Exercise 9.8.33 (Solution on p. 1138.)

A two

Exercise 9.8.34Six or seven

Exercise 9.8.35 (Solution on p. 1138.)

Red six

Exercise 9.8.36An ace or a diamond

Exercise 9.8.37 (Solution on p. 1138.)

A non-ace

Exercise 9.8.38A heart or a non-jack

For the following exercises, two dice are rolled, and the results are summed.

Exercise 9.8.39 (Solution on p. 1138.)

Construct a table showing the sample space of outcomes and sums.

Exercise 9.8.40Find the probability of rolling a sum of 3.

Available for free at Connexions <http://legacy.cnx.org/content/col11759/1.2>

1111

Exercise 9.8.41 (Solution on p. 1139.)

Find the probability of rolling at least one four or a sum of 8.Exercise 9.8.42Find the probability of rolling an odd sum less than 9.Exercise 9.8.43 (Solution on p. 1139.)

Find the probability of rolling a sum greater than or equal to 15.Exercise 9.8.44Find the probability of rolling a sum less than 15.Exercise 9.8.45 (Solution on p. 1139.)

Find the probability of rolling a sum less than 6 or greater than 9.Exercise 9.8.46Find the probability of rolling a sum between 6 and 9, inclusive.Exercise 9.8.47 (Solution on p. 1139.)

Find the probability of rolling a sum of 5 or 6.Exercise 9.8.48Find the probability of rolling any sum other than 5 or 6.

For the following exercises, a coin is tossed, and a card is pulled from a standard deck. Find the probabilityof the following:

Exercise 9.8.49 (Solution on p. 1139.)

A head on the coin or a club

Exercise 9.8.50A tail on the coin or red ace

Exercise 9.8.51 (Solution on p. 1139.)

A head on the coin or a face card

Exercise 9.8.52No aces

For the following exercises, use this scenario: a bag of M&Mscontains 12 blue, 6 brown, 10 orange, 8 yellow, 8 red, and 4 green M&Ms. Reaching into the bag, a per-son grabs 5 M&Ms.

Exercise 9.8.53 (Solution on p. 1139.)

What is the probability of getting all blue M&Ms?

Exercise 9.8.54What is the probability of getting 4 blue M&Ms?

Exercise 9.8.55 (Solution on p. 1139.)

What is the probability of getting 3 blue M&Ms?

Exercise 9.8.56What is the probability of getting no brown M&Ms?

Available for free at Connexions <http://legacy.cnx.org/content/col11759/1.2>

1112 CHAPTER 9. SEQUENCES, PROBABILITY, AND COUNTING THEORY

9.8.9.3 Extensions

Use the following scenario for the exercises that follow: In the game of Keno, a player starts byselecting 20 numbers from the numbers 1 to 80.After the player makes his selections, 20winning numbersare randomly selected from numbers 1 to 80.A win occurs if the player has correctly selected 3, 4, or 5 ofthe 20winning numbers. (Round all answers to the nearest hundredth of a percent.)

Exercise 9.8.57 (Solution on p. 1139.)

What is the percent chance that a player selects exactly 3 winning numbers?

Exercise 9.8.58What is the percent chance that a player selects exactly 4 winning numbers?

Exercise 9.8.59 (Solution on p. 1139.)

What is the percent chance that a player selects all 5 winning numbers?

Exercise 9.8.60What is the percent chance of winning?

Exercise 9.8.61 (Solution on p. 1139.)

How much less is a player's chance of selecting 3 winning numbers than the chance of selectingeither 4 or 5 winning numbers?

9.8.9.4 Real-World Applications

Use this data for the exercises that follow: In 2013, there were roughly 317 million citizens in the UnitedStates, and about 40 million were elderly (aged 65 and over).25

Exercise 9.8.62If you meet a U.S. citizen, what is the percent chance that the person is elderly? (Round to thenearest tenth of a percent.)

Exercise 9.8.63 (Solution on p. 1139.)

If you meet �ve U.S. citizens, what is the percent chance that exactly one is elderly? (Round tothe nearest tenth of a percent.)

Exercise 9.8.64If you meet �ve U.S. citizens, what is the percent chance that three are elderly? (Round to thenearest tenth of a percent.)

Exercise 9.8.65 (Solution on p. 1139.)

If you meet �ve U.S. citizens, what is the percent chance that four are elderly? (Round to thenearest thousandth of a percent.)

Exercise 9.8.66It is predicted that by 2030, one in �ve U.S. citizens will be elderly. How much greater will thechances of meeting an elderly person be at that time? What policy changes do you foresee if thesestatistics hold true?

25United States Census Bureau. http://www.census.gov

Available for free at Connexions <http://legacy.cnx.org/content/col11759/1.2>

1113

9.8.10 Chapter Review Exercises

9.8.10.1 Sequences and Their Notation (Section 9.2)

Exercise 9.8.67 (Solution on p. 1139.)

Write the �rst four terms of the sequence de�ned by the recursive formula a1 = 2, an = an−1 + n.

Exercise 9.8.68Evaluate 6!

(5−3)!3! .

Exercise 9.8.69 (Solution on p. 1139.)

Write the �rst four terms of the sequence de�ned by the explicit formula an = 10n + 3.Exercise 9.8.70Write the �rst four terms of the sequence de�ned by the explicit formula an = n!

n(n+1) .

9.8.10.2 Arithmetic Sequences (Section 9.3)

Exercise 9.8.71 (Solution on p. 1139.)

Is the sequence 47 ,

4721 ,

8221 ,

397 , ... arithmetic? If so, �nd the common di�erence.

Exercise 9.8.72Is the sequence 2, 4, 8, 16, ... arithmetic? If so, �nd the common di�erence.

Exercise 9.8.73 (Solution on p. 1139.)

An arithmetic sequence has the �rst term a1 = 18 and common di�erence d = −8.What are the�rst �ve terms?

Exercise 9.8.74An arithmetic sequence has terms a3 = 11.7 and a8 = −14.6. What is the �rst term?

Exercise 9.8.75 (Solution on p. 1139.)

Write a recursive formula for the arithmetic sequence −20,− 10, 0,10,. . .Exercise 9.8.76Write a recursive formula for the arithmetic sequence 0, − 1

2 , − 1, − 32 , . . . , and then �nd the

31st term.

Exercise 9.8.77 (Solution on p. 1139.)

Write an explicit formula for the arithmetic sequence 78 ,

2924 ,

3724 ,

158 , . . .

Exercise 9.8.78How many terms are in the �nite arithmetic sequence 12, 20, 28, . . . , 172?

9.8.10.3 Geometric Sequences (Section 9.4)

Exercise 9.8.79 (Solution on p. 1139.)

Find the common ratio for the geometric sequence 2.5, 5, 10, 20, . . .Exercise 9.8.80Is the sequence 4, 16, 28, 40, . . . geometric? If so �nd the common ratio. If not, explain why.

Exercise 9.8.81 (Solution on p. 1139.)

A geometric sequence has terms a7 = 16,384 and a9 = 262,144 . What are the �rst �ve terms?

Exercise 9.8.82A geometric sequence has the �rst term a1=− 3 and common ratio r = 1

2 .What is the 8th term?

Exercise 9.8.83 (Solution on p. 1139.)

What are the �rst �ve terms of the geometric sequence a1 = 3, an = 4 · an−1?

Available for free at Connexions <http://legacy.cnx.org/content/col11759/1.2>

1114 CHAPTER 9. SEQUENCES, PROBABILITY, AND COUNTING THEORY

Exercise 9.8.84Write a recursive formula for the geometric sequence 1, 1

3 ,19 ,

127 , . . .

Exercise 9.8.85 (Solution on p. 1139.)

Write an explicit formula for the geometric sequence − 15 , −

115 , −

145 , −

1135 , . . .

Exercise 9.8.86How many terms are in the �nite geometric sequence −5, − 5

3 , −59 , . . . , −

559,049?

9.8.10.4 Series and Their Notation (Section 9.5)

Exercise 9.8.87 (Solution on p. 1139.)

Use summation notation to write the sum of terms 12m+ 5 from m = 0 to m = 5.

Exercise 9.8.88Use summation notation to write the sum that results from adding the number 13 twenty times.

Exercise 9.8.89 (Solution on p. 1139.)

Use the formula for the sum of the �rst n terms of an arithmetic series to �nd the sum of the �rsteleven terms of the arithmetic series 2.5, 4, 5.5, . . . .

Exercise 9.8.90A ladder has 15 tapered rungs, the lengths of which increase by a common di�erence. The �rstrung is 5 inches long, and the last rung is 20 inches long. What is the sum of the lengths of therungs?

Exercise 9.8.91 (Solution on p. 1140.)

Use the formula for the sum of the �rst n terms of a geometric series to �nd S9 for the series12, 6, 3, 3

2 , . . .

Exercise 9.8.92The fees for the �rst three years of a hunting club membership are given in Table 9.14. If feescontinue to rise at the same rate, how much will the total cost be for the �rst ten years of mem-bership?

Year Membership Fees

1 $1500

2 $1950

3 $2535

Table 9.14

Exercise 9.8.93 (Solution on p. 1140.)

Find the sum of the in�nite geometric series∑∞k=1 45 ·

(− 1

3

)k−1.

Exercise 9.8.94A ball has a bounce-back ratio of 3

5 the height of the previous bounce. Write a series representingthe total distance traveled by the ball, assuming it was initially dropped from a height of 5 feet.What is the total distance? (Hint: the total distance the ball travels on each bounce is the sum ofthe heights of the rise and the fall.)

Exercise 9.8.95 (Solution on p. 1140.)

Alejandro deposits $80 of his monthly earnings into an annuity that earns 6.25% annual interest,compounded monthly. How much money will he have saved after 5 years?

Available for free at Connexions <http://legacy.cnx.org/content/col11759/1.2>

1115

Exercise 9.8.96The twins Sarah and Scott both opened retirement accounts on their 21st birthday. Sarah deposits$4,800.00 each year, earning 5.5% annual interest, compounded monthly. Scott deposits $3,600.00each year, earning 8.5% annual interest, compounded monthly. Which twin will earn the mostinterest by the time they are 55 years old? How much more?

9.8.10.5 Counting Principles (Section 9.6)

Exercise 9.8.97 (Solution on p. 1140.)

How many ways are there to choose a number from the set {−10, − 6, 4, 10, 12, 18, 24, 32} that isdivisible by either 4 or 6?Exercise 9.8.98In a group of 20 musicians, 12 play piano, 7 play trumpet, and 2 play both piano and trumpet.How many musicians play either piano or trumpet?

Exercise 9.8.99 (Solution on p. 1140.)

How many ways are there to construct a 4-digit code if numbers can be repeated?

Exercise 9.8.100A palette of water color paints has 3 shades of green, 3 shades of blue, 2 shades of red, 2 shadesof yellow, and 1 shade of black. How many ways are there to choose one shade of each color?

Exercise 9.8.101 (Solution on p. 1140.)

Calculate P (18, 4) .Exercise 9.8.102In a group of 5 freshman, 10 sophomores, 3 juniors, and 2 seniors, how many ways can a president,vice president, and treasurer be elected?

Exercise 9.8.103 (Solution on p. 1140.)

Calculate C (15, 6) .Exercise 9.8.104A co�ee shop has 7 Guatemalan roasts, 4 Cuban roasts, and 10 Costa Rican roasts. How manyways can the shop choose 2 Guatemalan, 2 Cuban, and 3 Costa Rican roasts for a co�ee tastingevent?

Exercise 9.8.105 (Solution on p. 1140.)

How many subsets does the set {1, 3, 5, . . . , 99} have?Exercise 9.8.106A day spa charges a basic day rate that includes use of a sauna, pool, and showers. For an extracharge, guests can choose from the following additional services: massage, body scrub, manicure,pedicure, facial, and straight-razor shave. How many ways are there to order additional services atthe day spa?

Exercise 9.8.107 (Solution on p. 1140.)

How many distinct ways can the word DEADWOOD be arranged?

Exercise 9.8.108How many distinct rearrangements of the letters of the word DEADWOOD are there if the ar-rangement must begin and end with the letter D?

Available for free at Connexions <http://legacy.cnx.org/content/col11759/1.2>

1116 CHAPTER 9. SEQUENCES, PROBABILITY, AND COUNTING THEORY

9.8.10.6 Binomial Theorem (Section 9.7)

Exercise 9.8.109 (Solution on p. 1140.)

Evaluate the binomial coe�cient

23

8

.

Exercise 9.8.110Use the Binomial Theorem to expand

(3x+ 1

2y)6.

Exercise 9.8.111 (Solution on p. 1140.)

Use the Binomial Theorem to write the �rst three terms of (2a+ b)17.

Exercise 9.8.112Find the fourth term of

(3a2 − 2b

)11without fully expanding the binomial.

9.8.10.7 Probability (Section 9.8)

For the following exercises, assume two die are rolled.

Exercise 9.8.113 (Solution on p. 1140.)

Construct a table showing the sample space.

Exercise 9.8.114What is the probability that a roll includes a 2?Exercise 9.8.115 (Solution on p. 1140.)

What is the probability of rolling a pair?

Exercise 9.8.116What is the probability that a roll includes a 2 or results in a pair?

Exercise 9.8.117 (Solution on p. 1140.)

What is the probability that a roll doesn't include a 2 or result in a pair?

Exercise 9.8.118What is the probability of rolling a 5 or a 6?

Exercise 9.8.119 (Solution on p. 1140.)

What is the probability that a roll includes neither a 5 nor a 6?

For the following exercises, use the following data: An elementary school survey found that 350 of the 500students preferred soda to milk. Suppose 8 children from the school are attending a birthday party. (Showcalculations and round to the nearest tenth of a percent.)

Exercise 9.8.120What is the percent chance that all the children attending the party prefer soda?

Exercise 9.8.121 (Solution on p. 1140.)

What is the percent chance that at least one of the children attending the party prefers milk?

Exercise 9.8.122What is the percent chance that exactly 3 of the children attending the party prefer soda?

Exercise 9.8.123 (Solution on p. 1140.)

What is the percent chance that exactly 3 of the children attending the party prefer milk?

Available for free at Connexions <http://legacy.cnx.org/content/col11759/1.2>

1117

9.8.11 Practice Test

Exercise 9.8.124 (Solution on p. 1140.)

Write the �rst four terms of the sequence de�ned by the recursive formula a = − − 14, an =2+an−−1

2 .

Exercise 9.8.125Write the �rst four terms of the sequence de�ned by the explicit formula an = n2−−n−−1

n! .

Exercise 9.8.126 (Solution on p. 1141.)

Is the sequence 0.3, 1.2, 2.1, 3, . . . arithmetic? If so �nd the common di�erence.

Exercise 9.8.127An arithmetic sequence has the �rst term a1 = −4 and common di�erence d = −− 4

3 .What is the6th term?

Exercise 9.8.128 (Solution on p. 1141.)

Write a recursive formula for the arithmetic sequence −2, − 72 , − 5, − 13

2 , . . . and then �nd the22nd term.

Exercise 9.8.129Write an explicit formula for the arithmetic sequence 15.6, 15, 14.4, 13.8, . . . and then �nd the32nd term.

Exercise 9.8.130 (Solution on p. 1141.)

Is the sequence − 2,− 1,− 12 ,−

14 , . . . geometric? If so �nd the common ratio. If not, explain why.

Exercise 9.8.131What is the 11th term of the geometric sequence − 1.5,−3,−6,−12, . . .?Exercise 9.8.132 (Solution on p. 1141.)

Write a recursive formula for the geometric sequence 1, − 12 ,

14 , −

18 , . . .

Exercise 9.8.133Write an explicit formula for the geometric sequence 4, − 4

3 ,49 , −

427 , . . .

Exercise 9.8.134 (Solution on p. 1141.)

Use summation notation to write the sum of terms 3k2 − 56k from k = −3 to k = 15.

Exercise 9.8.135A community baseball stadium has 10 seats in the �rst row, 13 seats in the second row, 16 seatsin the third row, and so on. There are 56 rows in all. What is the seating capacity of the stadium?

Exercise 9.8.136 (Solution on p. 1141.)

Use the formula for the sum of the �rst n terms of a geometric series to �nd∑7k=1−0.2 · (−5)k−1

.

Exercise 9.8.137Find the sum of the in�nite geometric series

∑∞k=1

13 ·(− 1

5

)k−1.

Exercise 9.8.138 (Solution on p. 1141.)

Rachael deposits $3,600 into a retirement fund each year. The fund earns 7.5% annual interest,compounded monthly. If she opened her account when she was 20 years old, how much will shehave by the time she's 55? How much of that amount was interest earned?

Exercise 9.8.139In a competition of 50 professional ballroom dancers, 22 compete in the fox-trot competition, 18compete in the tango competition, and 6 compete in both the fox-trot and tango competitions.How many dancers compete in the fox-trot or tango competitions?

Exercise 9.8.140 (Solution on p. 1141.)

A buyer of a new sedan can custom order the car by choosing from 5 di�erent exterior colors,3 di�erent interior colors, 2 sound systems, 3 motor designs, and either manual or automatictransmission. How many choices does the buyer have?

Available for free at Connexions <http://legacy.cnx.org/content/col11759/1.2>

1118 CHAPTER 9. SEQUENCES, PROBABILITY, AND COUNTING THEORY

Exercise 9.8.141To allocate annual bonuses, a manager must choose his top four employees and rank them �rst tofourth. In how many ways can he create the �Top-Four� list out of the 32 employees?

Exercise 9.8.142 (Solution on p. 1141.)

A rock group needs to choose 3 songs to play at the annual Battle of the Bands. How many wayscan they choose their set if have 15 songs to pick from?

Exercise 9.8.143A self-serve frozen yogurt shop has 8 candy toppings and 4 fruit toppings to choose from. Howmany ways are there to top a frozen yogurt?

Exercise 9.8.144 (Solution on p. 1141.)

How many distinct ways can the word EVANESCENCE be arranged if the anagram must endwith the letter E?

Exercise 9.8.145Use the Binomial Theorem to expand

(32x−

12y)5.

Exercise 9.8.146 (Solution on p. 1141.)

Find the seventh term of(x2 − 1

2

)13without fully expanding the binomial.

For the following exercises, use the spinner in Figure 9.26.

Figure 9.26

Available for free at Connexions <http://legacy.cnx.org/content/col11759/1.2>

1119

Exercise 9.8.147Construct a probability model showing each possible outcome and its associated probability. (Usethe �rst letter for colors.)

Exercise 9.8.148 (Solution on p. 1141.)

What is the probability of landing on an odd number?

Exercise 9.8.149What is the probability of landing on blue?

Exercise 9.8.150 (Solution on p. 1141.)

What is the probability of landing on blue or an odd number?

Exercise 9.8.151What is the probability of landing on anything other than blue or an odd number?

Exercise 9.8.152 (Solution on p. 1141.)

A bowl of candy holds 16 peppermint, 14 butterscotch, and 10 strawberry �avored candies. Supposea person grabs a handful of 7 candies. What is the percent chance that exactly 3 are butterscotch?(Show calculations and round to the nearest tenth of a percent.)

Available for free at Connexions <http://legacy.cnx.org/content/col11759/1.2>

1120 CHAPTER 9. SEQUENCES, PROBABILITY, AND COUNTING THEORY

Solutions to Exercises in Chapter 9

Solution to Exercise 9.2.1 (p. 1003)The �rst �ve terms are {1, 6, 11, 16, 21}.Solution to Exercise 9.2.2 (p. 1005)The �rst �ve terms are {−2, 2, − 3

2 , 1, − 58}.

Solution to Exercise 9.2.3 (p. 1006)The �rst six terms are {2, 5, 54, 10, 250, 15}.Solution to Exercise 9.2.4 (p. 1007)

an = (−1)n+19n

Solution to Exercise 9.2.5 (p. 1008)an = − 3n

4nSolution to Exercise 9.2.6 (p. 1008)an = en−3

Solution to Exercise 9.2.7 (p. 1010){2, 5, 11, 23, 47}Solution to Exercise 9.2.8 (p. 1012){0, 1, 1, 1, 2, 3, 5

2 ,176 }.

Solution to Exercise 9.2.9 (p. 1013)The �rst �ve terms are {1, 3

2 , 4, 15, 72}.Solution to Exercise 9.2.10 (p. 1014)A sequence is an ordered list of numbers that can be either �nite or in�nite in number. When a �nitesequence is de�ned by a formula, its domain is a subset of the non-negative integers. When an in�nitesequence is de�ned by a formula, its domain is all positive or all non-negative integers.Solution to Exercise 9.2.12 (p. 1014)Yes, both sets go on inde�nitely, so they are both in�nite sequences.Solution to Exercise 9.2.14 (p. 1014)A factorial is the product of a positive integer and all the positive integers below it. An exclamation pointis used to indicate the operation. Answers may vary. An example of the bene�t of using factorial notation iswhen indicating the product It is much easier to write than it is to write out 13 ·12 ·11 ·10 ·9 ·8 ·7 ·6 ·5 ·4 ·3 ·2 ·1.Solution to Exercise 9.2.16 (p. 1015)First four terms: −8, − 16

3 , − 4, − 165

Solution to Exercise 9.2.18 (p. 1015)First four terms: 2, 1

2 ,827 ,

14 .

Solution to Exercise 9.2.20 (p. 1015)First four terms: 1.25, − 5, 20, − 80.Solution to Exercise 9.2.22 (p. 1015)First four terms: 1

3 ,45 ,

97 ,

169 .

Solution to Exercise 9.2.24 (p. 1015)First four terms: − 4

5 , 4, − 20, 100Solution to Exercise 9.2.26 (p. 1015)

13 ,

45 ,

97 ,

169 ,

2511 , 31, 44, 59

Solution to Exercise 9.2.28 (p. 1015)−0.6,−3,−15,−20,−375,−80,−9375,−320Solution to Exercise 9.2.30 (p. 1015)an = n2 + 3Solution to Exercise 9.2.32 (p. 1016)

an = 2n

2n or 2n−1

nSolution to Exercise 9.2.34 (p. 1016)

an =(− 1

2

)n−1

Available for free at Connexions <http://legacy.cnx.org/content/col11759/1.2>

1121

Solution to Exercise 9.2.36 (p. 1016)First �ve terms: 3, − 9, 27, − 81, 243Solution to Exercise 9.2.38 (p. 1016)First �ve terms: −1, 1, − 9, 27

11 ,8915

Solution to Exercise 9.2.40 (p. 1016)124 , 1,

14 ,

32 ,

94 ,

814 ,

21878 , 531,441

16Solution to Exercise 9.2.42 (p. 1016)2, 10, 12, 14

5 ,45 , 2, 10, 12

Solution to Exercise 9.2.44 (p. 1016)a1 = −8, an = an−1 + nSolution to Exercise 9.2.46 (p. 1016)a1 = 35, an = an−1 + 3Solution to Exercise 9.2.48 (p. 1016)720Solution to Exercise 9.2.50 (p. 1016)665, 280Solution to Exercise 9.2.52 (p. 1017)First four terms: 1, 1

2 ,23 ,

32

Solution to Exercise 9.2.54 (p. 1017)First four terms: −1, 2, 6

5 ,2411

Solution to Exercise 9.2.56 (p. 1017)

Solu-

Available for free at Connexions <http://legacy.cnx.org/content/col11759/1.2>

1122 CHAPTER 9. SEQUENCES, PROBABILITY, AND COUNTING THEORY

tion to Exercise 9.2.58 (p. 1017)

Solu-tion to Exercise 9.2.60 (p. 1017)

Solu-tion to Exercise 9.2.62 (p. 1018)an = 2n−2

Solution to Exercise 9.2.64 (p. 1019)a1 = 6, an = 2an−1 − 5Solution to Exercise 9.2.66 (p. 1021)First �ve terms: 29

37 ,152111 ,

716333 ,

3188999 ,

137242997

Solution to Exercise 9.2.68 (p. 1021)First �ve terms: 2, 3, 5, 17, 65537Solution to Exercise 9.2.70 (p. 1021)a10 = 7, 257, 600Solution to Exercise 9.2.72 (p. 1022)First six terms: 0.042, 0.146, 0.875, 2.385, 4.708Solution to Exercise 9.2.74 (p. 1022)First four terms: 5.975, 32.765, 185.743, 1057.25, 6023.521Solution to Exercise 9.2.76 (p. 1022)If an = −421 is a term in the sequence, then solving the equation −421 = −6 − 8n for n will yield a

Available for free at Connexions <http://legacy.cnx.org/content/col11759/1.2>

1123

non-negative integer. However, if − 421 = −6 − 8n, then n = 51.875 so an = −421 is not a term in thesequence.Solution to Exercise 9.2.78 (p. 1022)a1 = 1, a2 = 0, an = an−1 − an−2

Solution to Exercise 9.2.80 (p. 1022)(n+2)!(n−1)! = (n+2)·(n+1)·(n)·(n−1)·...·3·2·1

(n−1)·...·3·2·1 = n (n+ 1) (n+ 2) = n3 + 3n2 + 2nSolution to Exercise 9.3.1 (p. 1024)The sequence is arithmetic. The common di�erence is −− 2.Solution to Exercise 9.3.2 (p. 1025)The sequence is not arithmetic because 3− 1 6= 6− 3.Solution to Exercise 9.3.3 (p. 1026){1, 6, 11, 16, 21}Solution to Exercise 9.3.4 (p. 1027)a2 = 2Solution to Exercise 9.3.5 (p. 1029)

a1 = 25

an = an−1 + 12, for n ≥ 2Solution to Exercise 9.3.6 (p. 1031)an = 53− 3nSolution to Exercise 9.3.7 (p. 1032)There are 11 terms in the sequence.Solution to Exercise 9.3.8 (p. 1033)The formula is Tn = 10 + 4n, and it will take her 42 minutes.Solution to Exercise 9.3.9 (p. 1034)A sequence where each successive term of the sequence increases (or decreases) by a constant value.Solution to Exercise 9.3.11 (p. 1034)We �nd whether the di�erence between all consecutive terms is the same. This is the same as saying thatthe sequence has a common di�erence.Solution to Exercise 9.3.13 (p. 1034)Both arithmetic sequences and linear functions have a constant rate of change. They are di�erent becausetheir domains are not the same; linear functions are de�ned for all real numbers, and arithmetic sequencesare de�ned for natural numbers or a subset of the natural numbers.Solution to Exercise 9.3.15 (p. 1034)The common di�erence is 1

2Solution to Exercise 9.3.17 (p. 1035)The sequence is not arithmetic because 16− 4 6= 64− 16.Solution to Exercise 9.3.19 (p. 1035)

0,23,

43, 2,

83

Solution to Exercise 9.3.21 (p. 1035)0,−5,−10,−15,−20Solution to Exercise 9.3.23 (p. 1035)a4 = 19Solution to Exercise 9.3.25 (p. 1035)a6 = 41Solution to Exercise 9.3.27 (p. 1035)a1 = 2Solution to Exercise 9.3.29 (p. 1035)a1 = 5

Available for free at Connexions <http://legacy.cnx.org/content/col11759/1.2>

1124 CHAPTER 9. SEQUENCES, PROBABILITY, AND COUNTING THEORY

Solution to Exercise 9.3.31 (p. 1035)a1 = 6Solution to Exercise 9.3.33 (p. 1035)a21 = −13.5Solution to Exercise 9.3.35 (p. 1036)−19,−20.4,−21.8,−23.2,−24.6Solution to Exercise 9.3.37 (p. 1036)

a1 = 17; an = an−1 + 9 n ≥ 2Solution to Exercise 9.3.39 (p. 1036)

a1 = 12; an = an−1 + 5 n ≥ 2Solution to Exercise 9.3.41 (p. 1036)

a1 = 8.9; an = an−1 + 1.4 n ≥ 2Solution to Exercise 9.3.43 (p. 1036)

a1 = 15 ; an = an−1 + 1

4 n ≥ 2Solution to Exercise 9.3.45 (p. 1036)

1 = 16 ; an = an−1 − 13

12 n ≥ 2Solution to Exercise 9.3.47 (p. 1036)a1 = 4; an = an−1 + 7; a14 = 95Solution to Exercise 9.3.49 (p. 1036)First �ve terms: 20, 16, 12, 8, 4.Solution to Exercise 9.3.51 (p. 1036)an = 1 + 2nSolution to Exercise 9.3.53 (p. 1037)an = −105 + 100nSolution to Exercise 9.3.55 (p. 1037)an = 1.8nSolution to Exercise 9.3.57 (p. 1037)an = 13.1 + 2.7nSolution to Exercise 9.3.59 (p. 1037)an = 1

3n−13

Solution to Exercise 9.3.61 (p. 1037)There are 10 terms in the sequence.Solution to Exercise 9.3.63 (p. 1037)There are 6 terms in the sequence.Solution to Exercise 9.3.65 (p. 1039)The graph does not represent an arithmetic sequence.

Available for free at Connexions <http://legacy.cnx.org/content/col11759/1.2>

1125

Solution to Exercise 9.3.67 (p. 1040)

Solu-tion to Exercise 9.3.69 (p. 1040)1, 4, 7, 10, 13, 16, 19

Available for free at Connexions <http://legacy.cnx.org/content/col11759/1.2>

1126 CHAPTER 9. SEQUENCES, PROBABILITY, AND COUNTING THEORY

Solution to Exercise 9.3.71 (p. 1040)

Solu-

Available for free at Connexions <http://legacy.cnx.org/content/col11759/1.2>

1127

tion to Exercise 9.3.73 (p. 1040)

Solu-tion to Exercise 9.3.75 (p. 1041)Answers will vary. Examples: an = 20.6nandan = 2 + 20.4n.Solution to Exercise 9.3.77 (p. 1041)a11 = −17a+ 38bSolution to Exercise 9.3.79 (p. 1041)The sequence begins to have negative values at the 13th term, a13 = − 1

3Solution to Exercise 9.3.81 (p. 1041)Answers will vary. Check to see that the sequence is arithmetic. Example: Recursive formula: a1 = 3, an =an−1 − 3. First 4 terms: 3, 0,−3,−6 a31 = −87Solution to Exercise 9.4.1 (p. 1043)The sequence is not geometric because 10

5 6=1510 .

Solution to Exercise 9.4.2 (p. 1043)The sequence is geometric. The common ratio is 1

5 .Solution to Exercise 9.4.3 (p. 1044)

{18, 6, 2,23,

29}

Solution to Exercise 9.4.4 (p. 1045)

a1 = 2

an = 23an−1 for n ≥ 2

Available for free at Connexions <http://legacy.cnx.org/content/col11759/1.2>

1128 CHAPTER 9. SEQUENCES, PROBABILITY, AND COUNTING THEORY

Solution to Exercise 9.4.5 (p. 1048)a6 = 16, 384Solution to Exercise 9.4.6 (p. 1049)

an = −(−3)n−1

Solution to Exercise 9.4.7 (p. 1050)

a. Pn = 293 · 1.026an

b. The number of hits will be about 333.

Solution to Exercise 9.4.8 (p. 1051)A sequence in which the ratio between any two consecutive terms is constant.Solution to Exercise 9.4.10 (p. 1051)Divide each term in a sequence by the preceding term. If the resulting quotients are equal, then the sequenceis geometric.Solution to Exercise 9.4.12 (p. 1051)Both geometric sequences and exponential functions have a constant ratio. However, their domains are notthe same. Exponential functions are de�ned for all real numbers, and geometric sequences are de�ned onlyfor positive integers. Another di�erence is that the base of a geometric sequence (the common ratio) can benegative, but the base of an exponential function must be positive.Solution to Exercise 9.4.14 (p. 1051)The common ratio is −2Solution to Exercise 9.4.16 (p. 1051)The sequence is geometric. The common ratio is 2.Solution to Exercise 9.4.18 (p. 1051)The sequence is geometric. The common ratio is − 1

2 .Solution to Exercise 9.4.20 (p. 1052)The sequence is geometric. The common ratio is 5.Solution to Exercise 9.4.22 (p. 1052)5, 1, 1

5 ,125 ,

1125

Solution to Exercise 9.4.24 (p. 1052)800, 400, 200, 100, 50Solution to Exercise 9.4.26 (p. 1052)a4 = − 16

27Solution to Exercise 9.4.28 (p. 1052)a7 = − 2

729Solution to Exercise 9.4.30 (p. 1052)7, 1.4, 0.28, 0.056, 0.0112Solution to Exercise 9.4.32 (p. 1052)

a1 = −32, an = 12an−1

Solution to Exercise 9.4.34 (p. 1052)

a1 = 10, an = −0.3an−1

Solution to Exercise 9.4.36 (p. 1052)

a1 = 35 , an = 1

6an−1

Solution to Exercise 9.4.38 (p. 1052)a1 = 1

512 , an = −4an−1

Solution to Exercise 9.4.40 (p. 1053)12,−6, 3,− 3

2 ,34

Solution to Exercise 9.4.42 (p. 1053)an = 3n−1

Solution to Exercise 9.4.44 (p. 1053)

an = 0.8 · (−5)n−1

Available for free at Connexions <http://legacy.cnx.org/content/col11759/1.2>

1129

Solution to Exercise 9.4.46 (p. 1053)

an = −(

45

)n−1

Solution to Exercise 9.4.48 (p. 1053)

an = 3 ·(− 1

3

)n−1

Solution to Exercise 9.4.50 (p. 1053)a12 = 1

177,147

Solution to Exercise 9.4.52 (p. 1053)There are 12 terms in the sequence.Solution to Exercise 9.4.54 (p. 1055)The graph does not represent a geometric sequence.Solution to Exercise 9.4.56 (p. 1056)

Solu-tion to Exercise 9.4.58 (p. 1056)

Answers will vary. Examples: a1 = 800, an = 0.5an−1

and a1 = 12.5, an = 4an−1

Solution to Exercise 9.4.60 (p. 1056)a5 = 256bSolution to Exercise 9.4.62 (p. 1056)The sequence exceeds 100 at the 14th term, a14 ≈ 107.Solution to Exercise 9.4.64 (p. 1056)a4 = − 32

3 is the �rst non-integer valueSolution to Exercise 9.4.66 (p. 1056)Answers will vary. Example: Explicit formula with a decimal common ratio: an = 400 · 0.5n−1; First 4terms: 400, 200, 100, 50; a8 = 3.125Solution to Exercise 9.5.1 (p. 1058)38Solution to Exercise 9.5.2 (p. 1060)26.4Solution to Exercise 9.5.3 (p. 1060)328

Available for free at Connexions <http://legacy.cnx.org/content/col11759/1.2>

1130 CHAPTER 9. SEQUENCES, PROBABILITY, AND COUNTING THEORY

Solution to Exercise 9.5.4 (p. 1060)−280Solution to Exercise 9.5.5 (p. 1061)$2,025Solution to Exercise 9.5.6 (p. 1063)≈ 2, 000.00Solution to Exercise 9.5.7 (p. 1063)9,840Solution to Exercise 9.5.8 (p. 1063)$275,513.31Solution to Exercise 9.5.9 (p. 1064)The sum is de�ned. It is geometric.Solution to Exercise 9.5.10 (p. 1065)The sum of the in�nite series is de�ned.Solution to Exercise 9.5.11 (p. 1065)The sum of the in�nite series is de�ned.Solution to Exercise 9.5.12 (p. 1067)3Solution to Exercise 9.5.13 (p. 1067)The series is not geometric.Solution to Exercise 9.5.14 (p. 1067)− 3

11Solution to Exercise 9.5.15 (p. 1068)$92,408.18Solution to Exercise 9.5.16 (p. 1069)An nth partial sum is the sum of the �rst n terms of a sequence.Solution to Exercise 9.5.18 (p. 1069)A geometric series is the sum of the terms in a geometric sequence.Solution to Exercise 9.5.20 (p. 1069)An annuity is a series of regular equal payments that earn a constant compounded interest.Solution to Exercise 9.5.22 (p. 1069)∑4n=0 5n

Solution to Exercise 9.5.24 (p. 1069)∑5k=1 4

Solution to Exercise 9.5.26 (p. 1069)∑20k=1 8k + 2

Solution to Exercise 9.5.28 (p. 1070)

S5 =5( 3

2 + 72 )

2Solution to Exercise 9.5.30 (p. 1070)

S13 = 13(3.2+5.6)2

Solution to Exercise 9.5.32 (p. 1070)∑7k=1 8 · 0.5k−1

Solution to Exercise 9.5.34 (p. 1070)

S5 =9“

1−( 13 )5”

1− 13

= 1219 ≈ 13.44

Solution to Exercise 9.5.36 (p. 1070)

S11 =64(1−0.211)

1−0.2 = 781,249,9849,765,625 ≈ 80

Solution to Exercise 9.5.38 (p. 1070)The series is de�ned. S = 2

1−0.8

Available for free at Connexions <http://legacy.cnx.org/content/col11759/1.2>

1131

Solution to Exercise 9.5.40 (p. 1070)The series is de�ned. S = −1

1−(− 12 )

Solution to Exercise 9.5.42 (p. 1070)

Solu-tion to Exercise 9.5.44 (p. 1071)

Sample answer: The graph of Sn seems to be approaching 1. This makes sense because∑∞k=1

(12

)kis a

de�ned in�nite geometric series with S =12

1−−( 12 ) = 1.

Solution to Exercise 9.5.46 (p. 1071)49Solution to Exercise 9.5.48 (p. 1071)254Solution to Exercise 9.5.50 (p. 1071)S7 = 147

2Solution to Exercise 9.5.52 (p. 1071)S11 = 55

2Solution to Exercise 9.5.54 (p. 1071)S7 = 5208.4Solution to Exercise 9.5.56 (p. 1071)S10 = − 1023

256Solution to Exercise 9.5.58 (p. 1071)S = − 4

3Solution to Exercise 9.5.60 (p. 1071)S = 9.2Solution to Exercise 9.5.62 (p. 1072)$3,705.42

Available for free at Connexions <http://legacy.cnx.org/content/col11759/1.2>

1132 CHAPTER 9. SEQUENCES, PROBABILITY, AND COUNTING THEORY

Solution to Exercise 9.5.64 (p. 1072)$695,823.97Solution to Exercise 9.5.66 (p. 1072)ak = 30− kSolution to Exercise 9.5.68 (p. 1072)9 termsSolution to Exercise 9.5.70 (p. 1072)r = 4

5Solution to Exercise 9.5.72 (p. 1072)$400 per monthSolution to Exercise 9.5.74 (p. 1073)420 feetSolution to Exercise 9.5.76 (p. 1073)12 feetSolution to Exercise 9.6.1 (p. 1075)7Solution to Exercise 9.6.2 (p. 1076)There are 60 possible breakfast specials.Solution to Exercise 9.6.3 (p. 1078)120Solution to Exercise 9.6.4 (p. 1078)60Solution to Exercise 9.6.5 (p. 1078)12Solution to Exercise 9.6.6 (p. 1080)P (7, 7) = 5, 040Solution to Exercise 9.6.7 (p. 1080)P (7, 5) = 2, 520Solution to Exercise 9.6.8 (p. 1081)C (10, 3) = 120Solution to Exercise 9.6.9 (p. 1082)64 sundaesSolution to Exercise 9.6.10 (p. 1083)840Solution to Exercise 9.6.11 (p. 1084)There arem+ nways for either eventA or eventB to occur.Solution to Exercise 9.6.13 (p. 1084)The addition principle is applied when determining the total possible of outcomes of either event occurring.The multiplication principle is applied when determining the total possible outcomes of both events occurring.The word �or� usually implies an addition problem. The word �and� usually implies a multiplication problem.Solution to Exercise 9.6.15 (p. 1084)A combination;C (n, r) = n!

(n−r)!r!Solution to Exercise 9.6.17 (p. 1084)4 + 2 = 6Solution to Exercise 9.6.19 (p. 1085)5 + 4 + 7 = 16Solution to Exercise 9.6.21 (p. 1085)2× 6 = 12Solution to Exercise 9.6.23 (p. 1085)103 = 1000

Available for free at Connexions <http://legacy.cnx.org/content/col11759/1.2>

1133

Solution to Exercise 9.6.25 (p. 1085)P (5, 2) = 20Solution to Exercise 9.6.27 (p. 1085)P (3, 3) = 6Solution to Exercise 9.6.29 (p. 1085)P (11, 5) = 55, 440Solution to Exercise 9.6.31 (p. 1085)C (12, 4) = 495Solution to Exercise 9.6.33 (p. 1085)C (7, 6) = 7Solution to Exercise 9.6.35 (p. 1085)210 = 1024Solution to Exercise 9.6.37 (p. 1085)212 = 4096Solution to Exercise 9.6.39 (p. 1086)29 = 512Solution to Exercise 9.6.41 (p. 1086)8!3! = 6720Solution to Exercise 9.6.43 (p. 1086)

12!3!2!3!4!Solution to Exercise 9.6.45 (p. 1086)9Solution to Exercise 9.6.47 (p. 1086)Yes, for the trivial cases r = 0 and r = 1. If r = 0, then C (n, r) = P (n, r) = 1. If r = 1, then r =1,C (n, r) = P (n, r) = n.Solution to Exercise 9.6.49 (p. 1086)6!2! × 4! = 8640Solution to Exercise 9.6.51 (p. 1087)6− 3 + 8− 3 = 8Solution to Exercise 9.6.53 (p. 1087)4× 2× 5 = 40Solution to Exercise 9.6.55 (p. 1087)4× 12× 3 = 144Solution to Exercise 9.6.57 (p. 1087)P (15, 9) = 1, 816, 214, 400Solution to Exercise 9.6.59 (p. 1087)C (10, 3)× C (6, 5)× C (5, 2) = 7, 200Solution to Exercise 9.6.61 (p. 1087)211 = 2048Solution to Exercise 9.6.63 (p. 1087)

20!6!6!8! = 116, 396, 280Solution to Exercise 9.7.1 (p. 1089)

a. 35b. 330

Solution to Exercise 9.7.2 (p. 1093)

a. x5 − 5x4y + 10x3y2 − 10x2y3 + 5xy4 − y5

b. 8x3 + 60x2y + 150xy2 + 125y3

Available for free at Connexions <http://legacy.cnx.org/content/col11759/1.2>

1134 CHAPTER 9. SEQUENCES, PROBABILITY, AND COUNTING THEORY

Solution to Exercise 9.7.3 (p. 1095)− 10, 206x4y5

Solution to Exercise 9.7.4 (p. 1095)

A binomial coe�cient is an alternative way of denoting the combination C (n, r) . It is de�ned as

n

r

=

C (n, r) = n!r!(n−r)! .

Solution to Exercise 9.7.6 (p. 1095)

The Binomial Theorem is de�ned as (x+ y)n =∑nk=0

n

k

xn−kyk and can be used to expand any

binomial.Solution to Exercise 9.7.8 (p. 1096)15Solution to Exercise 9.7.10 (p. 1096)35Solution to Exercise 9.7.12 (p. 1096)10Solution to Exercise 9.7.14 (p. 1096)12,376Solution to Exercise 9.7.16 (p. 1096)64a3 − 48a2b+ 12ab2 − b3Solution to Exercise 9.7.18 (p. 1096)27a3 + 54a2b+ 36ab2 + 8b3

Solution to Exercise 9.7.20 (p. 1096)1024x5 + 2560x4y + 2560x3y2 + 1280x2y3 + 320xy4 + 32y5

Solution to Exercise 9.7.22 (p. 1096)1024x5 − 3840x4y + 5760x3y2 − 4320x2y3 + 1620xy4 − 243y5

Solution to Exercise 9.7.24 (p. 1097)1x4 + 8

x3y + 24x2y2 + 32

xy3 + 16y4

Solution to Exercise 9.7.26 (p. 1097)a17 + 17a16b+ 136a15b2

Solution to Exercise 9.7.28 (p. 1097)a15 − 30a14b+ 420a13b2

Solution to Exercise 9.7.30 (p. 1097)3, 486, 784, 401a20 + 23, 245, 229, 340a19b+ 73, 609, 892, 910a18b2

Solution to Exercise 9.7.32 (p. 1097)x24 − 8x21√y + 28x18ySolution to Exercise 9.7.34 (p. 1097)−720x2y3

Solution to Exercise 9.7.36 (p. 1097)220, 812, 466, 875, 000y7

Solution to Exercise 9.7.38 (p. 1097)35x3y4

Solution to Exercise 9.7.40 (p. 1097)1, 082, 565a3b16

Solution to Exercise 9.7.42 (p. 1097)1152y2

x7

Solution to Exercise 9.7.44 (p. 1098)f2 (x) = x4 + 12x3

Available for free at Connexions <http://legacy.cnx.org/content/col11759/1.2>

1135

So-lution to Exercise 9.7.46 (p. 1098)f4 (x) = x4 + 12x3 + 54x2 + 108x

Available for free at Connexions <http://legacy.cnx.org/content/col11759/1.2>

1136 CHAPTER 9. SEQUENCES, PROBABILITY, AND COUNTING THEORY

So-lution to Exercise 9.7.48 (p. 1098)590, 625x5y2

Solution to Exercise 9.7.50 (p. 1098)k − 1

Available for free at Connexions <http://legacy.cnx.org/content/col11759/1.2>

1137

Solution to Exercise 9.7.51 (p. 1098) n

k − 1

+

n

k

=

n+ 1

k

; Proof: n

k − 1

+

n

k

= n!

k!(n−k)! + n!(k−1)!(n−(k−1))!

= n!k!(n−k)! + n!

(k−1)!(n−k+1)!

= (n−k+1)n!(n−k+1)k!(n−k)! + kn!

k(k−1)!(n−k+1)!

= (n−k+1)n!+kn!k!(n−k+1)!

= (n+1)n!k!((n+1)−k)!

= (n+1)!k!((n+1)−k)!

=

n+ 1

k

Solution to Exercise 9.7.52 (p. 1098)

The expression(x3 + 2y2 − z

)5cannot be expanded using the Binomial Theorem because it cannot be rewrit-

ten as a binomial.Solution to Exercise 9.8.1 (p. 1100)

Outcome Probability

Roll of 1

Roll of 2

Roll of 3

Roll of 4

Roll of 5

Roll of 6

Table 9.15

Solution to Exercise 9.8.2 (p. 1101)23Solution to Exercise 9.8.3 (p. 1103)

713Solution to Exercise 9.8.4 (p. 1104)

213Solution to Exercise 9.8.5 (p. 1105)56Solution to Exercise 9.8.6 (p. 1107)

a. 191 ; b. 5

91 ; c. 8691

Solution to Exercise 9.8.7 (p. 1108)probability; The probability of an event is restricted to values between 0 and 1, inclusive of 0 and 1.

Available for free at Connexions <http://legacy.cnx.org/content/col11759/1.2>

1138 CHAPTER 9. SEQUENCES, PROBABILITY, AND COUNTING THEORY

Solution to Exercise 9.8.9 (p. 1108)An experiment is an activity with an observable result.Solution to Exercise 9.8.11 (p. 1108)The probability of the union of two events occurring is a number that describes the likelihood that atleast one of the events from a probability model occurs. In both a union of setsA and B and a union ofeventsA and B, the union includes eitherA or B or both. The di�erence is that a union of sets results inanother set, while the union of events is a probability, so it is always a numerical value between 0 and 1.Solution to Exercise 9.8.13 (p. 1109)12 .Solution to Exercise 9.8.15 (p. 1109)58 .Solution to Exercise 9.8.17 (p. 1109)12 .Solution to Exercise 9.8.19 (p. 1109)38 .Solution to Exercise 9.8.21 (p. 1110)14 .Solution to Exercise 9.8.23 (p. 1110)34 .Solution to Exercise 9.8.25 (p. 1110)38 .Solution to Exercise 9.8.27 (p. 1110)18 .Solution to Exercise 9.8.29 (p. 1110)1516 .Solution to Exercise 9.8.31 (p. 1110)58 .Solution to Exercise 9.8.33 (p. 1110)

113 .Solution to Exercise 9.8.35 (p. 1110)

126 .Solution to Exercise 9.8.37 (p. 1110)1213 .Solution to Exercise 9.8.39 (p. 1110)

1 2 3 4 5 6

1 (1, 1)2 (1, 2)3 (1, 3)4 (1, 4) 5 (1, 5) 6 (1, 6) 7

2 (2, 1)3 (2, 2)4 (2, 3)5 (2, 4) 6 (2, 5) 7 (2, 6) 8

3 (3, 1)4 (3, 2)5 (3, 3)6 (3, 4) 7 (3, 5) 8 (3, 6) 9

4 (4, 1)5 (4, 2)6 (4, 3)7 (4, 4) 8 (4, 5) 9 (4, 6)10

5 (5, 1)6 (5, 2)7 (5, 3)8 (5, 4) 9 (5, 5)10 (5, 6)11

6 (6, 1)7 (6, 2)8 (6, 3)9 (6, 4)10 (6, 5)11 (6, 6)12

Table 9.16

Available for free at Connexions <http://legacy.cnx.org/content/col11759/1.2>

1139

Solution to Exercise 9.8.41 (p. 1111)512 .Solution to Exercise 9.8.43 (p. 1111)0.Solution to Exercise 9.8.45 (p. 1111)49 .Solution to Exercise 9.8.47 (p. 1111)14 .Solution to Exercise 9.8.49 (p. 1111)34Solution to Exercise 9.8.51 (p. 1111)2126Solution to Exercise 9.8.53 (p. 1111)C(12,5)C(48,5) = 1

2162

Solution to Exercise 9.8.55 (p. 1111)C(12,3)C(36,2)

C(48,5) = 1752162

Solution to Exercise 9.8.57 (p. 1112)C(20,3)C(60,17)

C(80,20) ≈ 12.49%Solution to Exercise 9.8.59 (p. 1112)C(20,5)C(60,15)

C(80,20) ≈ 23.33%Solution to Exercise 9.8.61 (p. 1112)20.50 + 23.33− 12.49 = 31.34%Solution to Exercise 9.8.63 (p. 1112)C(40000000,1)C(277000000,4)

C(317000000,5) = 36.78%Solution to Exercise 9.8.65 (p. 1112)C(40000000,4)C(277000000,1)

C(317000000,5) = 0.11%Solution to Exercise 9.8.67 (p. 1113)2, 4, 7, 11Solution to Exercise 9.8.69 (p. 1113)13, 103, 1003, 10003Solution to Exercise 9.8.71 (p. 1113)The sequence is arithmetic. The common di�erence is d = 5

3 .Solution to Exercise 9.8.73 (p. 1113)18, 10, 2,−6,−14Solution to Exercise 9.8.75 (p. 1113)a1 = −20, an = an−1 + 10Solution to Exercise 9.8.77 (p. 1113)an = 1

3n+ 1324

Solution to Exercise 9.8.79 (p. 1113)r = 2Solution to Exercise 9.8.81 (p. 1113)4, 16, 64, 256, 1024Solution to Exercise 9.8.83 (p. 1113)3, 12, 48, 192, 768Solution to Exercise 9.8.85 (p. 1114)

an = − 15 ·(

13

)n−1

Solution to Exercise 9.8.87 (p. 1114)∑5m=0

(12m+ 5

).

Available for free at Connexions <http://legacy.cnx.org/content/col11759/1.2>

1140 CHAPTER 9. SEQUENCES, PROBABILITY, AND COUNTING THEORY

Solution to Exercise 9.8.89 (p. 1114)S11 = 110Solution to Exercise 9.8.91 (p. 1114)S9 ≈ 23.95Solution to Exercise 9.8.93 (p. 1114)S = 135

4Solution to Exercise 9.8.95 (p. 1114)$5,617.61Solution to Exercise 9.8.97 (p. 1115)6Solution to Exercise 9.8.99 (p. 1115)104 = 10,000Solution to Exercise 9.8.101 (p. 1115)P (18, 4) = 73,440Solution to Exercise 9.8.103 (p. 1115)C (15, 6) = 5005Solution to Exercise 9.8.105 (p. 1115)250 = 1.13× 1015

Solution to Exercise 9.8.107 (p. 1115)8!

3!2! = 3360Solution to Exercise 9.8.109 (p. 1116)490,314Solution to Exercise 9.8.111 (p. 1116)131,072a17+1,114,112a16b+4,456,448a15b2

Solution to Exercise 9.8.113 (p. 1116)

1 2 3 4 5 6

1 1, 1 1, 2 1, 3 1, 4 1, 5 1, 6

2 2, 1 2, 2 2, 3 2, 4 2, 5 2, 6

3 3, 1 3, 2 3, 3 3, 4 3, 5 3, 6

4 4, 1 4, 2 4, 3 4, 4 4, 5 4, 6

5 5, 1 5, 2 5, 3 5, 4 5, 5 5, 6

6 6, 1 6, 2 6, 3 6, 4 6, 5 6, 6

Table 9.17

Solution to Exercise 9.8.115 (p. 1116)16Solution to Exercise 9.8.117 (p. 1116)

59Solution to Exercise 9.8.119 (p. 1116)

49Solution to Exercise 9.8.121 (p. 1116)

1− C(350,8)C(500,8) ≈ 94.4%

Solution to Exercise 9.8.123 (p. 1116)C(150,3)C(350,5)

C(500,8) ≈ 25.6%

Available for free at Connexions <http://legacy.cnx.org/content/col11759/1.2>

1141

Solution to Exercise 9.8.124 (p. 1117)−14,−6,−2, 0Solution to Exercise 9.8.126 (p. 1117)The sequence is arithmetic. The common di�erence is d = 0.9.Solution to Exercise 9.8.128 (p. 1117)a1 = −2, an = an−1 − 3

2 ; a22 = − 672

Solution to Exercise 9.8.130 (p. 1117)The sequence is geometric. The common ratio is r = 1

2 .Solution to Exercise 9.8.132 (p. 1117)a1 = 1, an = − 1

2 · an−1

Solution to Exercise 9.8.134 (p. 1117)∑15k=−3

(3k2 − 5

6k)

Solution to Exercise 9.8.136 (p. 1117)S7 = −2604.2Solution to Exercise 9.8.138 (p. 1117)Total in account: $140, 355.75; Interest earned: $14, 355.75Solution to Exercise 9.8.140 (p. 1117)5× 3× 2× 3× 2 = 180Solution to Exercise 9.8.142 (p. 1118)C (15, 3) = 455Solution to Exercise 9.8.144 (p. 1118)

10!2!3!2! = 151,200Solution to Exercise 9.8.146 (p. 1118)

429x14

16Solution to Exercise 9.8.148 (p. 1119)

47Solution to Exercise 9.8.150 (p. 1119)

57Solution to Exercise 9.8.152 (p. 1119)C(14,3)C(26,4)

C(40,7) ≈ 29.2%

Available for free at Connexions <http://legacy.cnx.org/content/col11759/1.2>

1142 GLOSSARY

Glossary

A absolute value equation

an equation in which the variable appears in absolute value bars, typically with two solutions, oneaccounting for the positive expression and one for the negative expression

addition method

an algebraic technique used to solve systems of linear equations in which the equations are addedin a way that eliminates one variable, allowing the resulting equation to be solved for theremaining variable; substitution is then used to solve for the �rst variable

Addition Principle

if one event can occur in m ways and a second event with no common outcomes can occur in nways, then the �rst or second event can occur in m+ n ways

algebraic expression

constants and variables combined using addition, subtraction, multiplication, and division

angle of rotation

an acute angle formed by a set of axes rotated from the Cartesian plane where,if cot (2θ) > 0,then θ is between (0 ◦, 45 ◦) ;if cot (2θ) < 0,then θ is between (45 ◦, 90 ◦) ; andif cot (2θ) = 0,then θ = 45 ◦

annual percentage rate (APR)

the yearly interest rate earned by an investment account, also called nominal rate

annuity

an investment in which the purchaser makes a sequence of periodic, equal payments

area

in square units, the area formula used in this section is used to �nd the area of anytwo-dimensional rectangular region:A = LW

arithmetic sequence

a sequence in which the di�erence between any two consecutive terms is a constant

arithmetic series

the sum of the terms in an arithmetic sequence

associative property of addition

the sum of three numbers may be grouped di�erently without a�ecting the result; insymbols, a+ (b+ c) = (a+ b) + c

associative property of multiplication

the product of three numbers may be grouped di�erently without a�ecting the result; insymbols, a · (b · c) = (a · b) · c

augmented matrix

a coe�cient matrix adjoined with the constant column separated by a vertical line within thematrix brackets

Available for free at Connexions <http://legacy.cnx.org/content/col11759/1.2>

GLOSSARY 1143

B base

in exponential notation, the expression that is being multiplied

binomial

a polynomial containing two terms

binomial coe�cient

the number of ways to choose r objects from n objects where order does not matter; equivalent

toC (n, r) , denoted

n

r

binomial expansion

the result of expanding (x+ y)n by multiplying

Binomial Theorem

a formula that can be used to expand any binomial

break-even point

the point at which a cost function intersects a revenue function; where pro�t is zero

C carrying capacity

in a logistic model, the limiting value of the output

center of a hyperbola

the midpoint of both the transverse and conjugate axes of a hyperbola

center of an ellipse

the midpoint of both the major and minor axes

change-of-base formula

a formula for converting a logarithm with any base to a quotient of logarithms with any otherbase.

coe�cient

any real number ai in a polynomial in the form anxn + ...+ a2x

2 + a1x+ a0

coe�cient matrix

a matrix that contains only the coe�cients from a system of equations

column

a set of numbers aligned vertically in a matrix

combination

a selection of objects in which order does not matter

common di�erence

the di�erence between any two consecutive terms in an arithmetic sequence

common logarithm

the exponent to which 10 must be raised to getx; log10 (x) is written simply as log (x) .

common ratio

the ratio between any two consecutive terms in a geometric sequence

commutative property of addition

Available for free at Connexions <http://legacy.cnx.org/content/col11759/1.2>

1144 GLOSSARY

two numbers may be added in either order without a�ecting the result; in symbols, a+ b = b+ a

commutative property of multiplication

two numbers may be multiplied in any order without a�ecting the result; in symbols, a · b = b · acomplement of an event

the set of outcomes in the sample space that are not in the eventE

completing the square

a process for solving quadratic equations in which terms are added to or subtracted from bothsides of the equation in order to make one side a perfect square

complex conjugate

a complex number containing the same terms as another complex number, but with the oppositeoperator. Multiplying a complex number by its conjugate yields a real number.

complex number

the sum of a real number and an imaginary number; the standard form is a+ bi,where a is thereal part and b is the complex part.

complex plane

the coordinate plane in which the horizontal axis represents the real component of a complexnumber, and the vertical axis represents the imaginary component, labeled i.

compound inequality

a problem or a statement that includes two inequalities

compound interest

interest earned on the total balance, not just the principal

conditional equation

an equation that is true for some values of the variable

conic section

any shape resulting from the intersection of a right circular cone with a plane

conjugate axis

the axis of a hyperbola that is perpendicular to the transverse axis and has the co-vertices as itsendpoints

consistent system

a system for which there is a single solution to all equations in the system and it is anindependent system, or if there are an in�nite number of solutions and it is a dependent system

constant

a quantity that does not change value

cost function

the function used to calculate the costs of doing business; it usually has two parts, �xed costs andvariable costs

Cramer's Rule

a method for solving systems of equations that have the same number of equations as variablesusing determinants

D degenerate conic sections

Available for free at Connexions <http://legacy.cnx.org/content/col11759/1.2>

GLOSSARY 1145

any of the possible shapes formed when a plane intersects a double cone through the apex. Typesof degenerate conic sections include a point, a line, and intersecting lines.

degree

the highest power of the variable that occurs in a polynomial

dependent system

a system of linear equations in which the two equations represent the same line; there are anin�nite number of solutions to a dependent system

dependent variable

an output variable

determinant

a number calculated using the entries of a square matrix that determines such information aswhether there is a solution to a system of equations

di�erence of squares

the binomial that results when a binomial is multiplied by a binomial with the same terms, butthe opposite sign

directrix

a line perpendicular to the axis of symmetry of a parabola; a line such that the ratio of thedistance between the points on the conic and the focus to the distance to the directrix is constant

discriminant

the expression under the radical in the quadratic formula that indicates the nature of thesolutions, real or complex, rational or irrational, single or double roots.

distributive property

the product of a factor times a sum is the sum of the factor times each term in the sum; insymbols, a · (b+ c) = a · b+ a · c

diverge

a series is said to diverge if the sum is not a real number

domain

the set of all possible input values for a relation

doubling time

the time it takes for a quantity to double

E eccentricity

the ratio of the distances from a pointP on the graph to the focusF and to thedirectrixD represented by e = PF

PD ,where e is a positive real number

ellipse

the set of all points (x, y) in a plane such that the sum of their distances from two �xed points isa constant

entry

an element, coe�cient, or constant in a matrix

equation

a mathematical statement indicating that two expressions are equal

equations in quadratic form

Available for free at Connexions <http://legacy.cnx.org/content/col11759/1.2>

1146 GLOSSARY

equations with a power other than 2 but with a middle term with an exponent that is one-halfthe exponent of the leading term

event

any subset of a sample space

experiment

an activity with an observable result

explicit formula

a formula that de�nes each term of a sequence in terms of its position in the sequence

exponent

in exponential notation, the raised number or variable that indicates how many times the base isbeing multiplied

exponential growth

a model that grows by a rate proportional to the amount present

exponential notation

a shorthand method of writing products of the same factor

extraneous solution

a solution introduced while solving an equation that does not satisfy the conditions of the originalequation

extraneous solutions

any solutions obtained that are not valid in the original equation

F factor by grouping

a method for factoring a trinomial in the form ax2 + bx+ c by dividing the x term into the sum oftwo terms, factoring each portion of the expression separately, and then factoring out the GCFof the entire expression

feasible region

the solution to a system of nonlinear inequalities that is the region of the graph where the shadedregions of each inequality intersect

�nite sequence

a function whose domain consists of a �nite subset of the positive integers {1, 2, . . . n} for somepositive integern

foci

plural of focus

focus (of a parabola)

a �xed point in the interior of a parabola that lies on the axis of symmetry

focus (of an ellipse)

one of the two �xed points on the major axis of an ellipse such that the sum of the distances fromthese points to any point (x, y) on the ellipse is a constant

formula

an equation expressing a relationship between constant and variable quantities

function

Available for free at Connexions <http://legacy.cnx.org/content/col11759/1.2>

GLOSSARY 1147

a relation in which each input value yields a unique output value

Fundamental Counting Principle

if one event can occur in m ways and a second event can occur in n ways after the �rst event hasoccurred, then the two events can occur in m× n ways; also known as the MultiplicationPrinciple

G Gaussian elimination

using elementary row operations to obtain a matrix in row-echelon form

geometric sequence

a sequence in which the ratio of a term to a previous term is a constant

geometric series

the sum of the terms in a geometric sequence

greatest common factor

the largest polynomial that divides evenly into each polynomial

H half-life

the length of time it takes for a substance to exponentially decay to half of its original quantity

horizontal line test

a method of testing whether a function is one-to-one by determining whether any horizontal lineintersects the graph more than once

hyperbola

the set of all points (x, y) in a plane such that the di�erence of the distances between (x, y) andthe foci is a positive constant

I identity equation

an equation that is true for all values of the variable

identity matrix

a square matrix containing ones down the main diagonal and zeros everywhere else; it acts as a 1in matrix algebra

identity property of addition

there is a unique number, called the additive identity, 0, which, when added to a number, resultsin the original number; in symbols, a+ 0 = a

identity property of multiplication

there is a unique number, called the multiplicative identity, 1, which, when multiplied by anumber, results in the original number; in symbols, a · 1 = a

imaginary number

the square root of − 1 : i =√−1.

inconsistent equation

an equation producing a false result

inconsistent system

a system of linear equations with no common solution because they represent parallel lines, whichhave no point or line in common

Available for free at Connexions <http://legacy.cnx.org/content/col11759/1.2>

1148 GLOSSARY

independent system

a system of linear equations with exactly one solution pair (x, y)

independent variable

an input variable

index of summation

in summation notation, the variable used in the explicit formula for the terms of a series andwritten below the sigma with the lower limit of summation

index

the number above the radical sign indicating the nth root

in�nite sequence

a function whose domain is the set of positive integers

in�nite series

the sum of the terms in an in�nite sequence

input

each object or value in a domain that relates to another object or value by a relationship knownas a function

integers

the set consisting of the natural numbers, their opposites, and 0:{. . . ,−3,−2,−1, 0, 1, 2, 3, . . . }interval

an interval describes a set of numbers within which a solution falls

interval notation

a mathematical statement that describes a solution set and uses parentheses or brackets toindicate where an interval begins and ends

inverse property of addition

for every real number a,there is a unique number, called the additive inverse (or opposite),denoted − a,which, when added to the original number, results in the additive identity, 0; insymbols, a+ (−a) = 0

inverse property of multiplication

for every non-zero real number a,there is a unique number, called the multiplicative inverse (orreciprocal), denoted 1

a ,which, when multiplied by the original number, results in themultiplicative identity, 1; in symbols, a · 1

a = 1

irrational numbers

the set of all numbers that are not rational; they cannot be written as either a terminating orrepeating decimal; they cannot be expressed as a fraction of two integers

L latus rectum

the line segment that passes through the focus of a parabola parallel to the directrix, withendpoints on the parabola

leading coe�cient

the coe�cient of the leading term

leading term

the term containing the highest degree

Available for free at Connexions <http://legacy.cnx.org/content/col11759/1.2>

GLOSSARY 1149

least common denominator

the smallest multiple that two denominators have in common

linear equation

an algebraic equation in which each term is either a constant or the product of a constant and the�rst power of a variable

linear inequality

similar to a linear equation except that the solutions will include sets of numbers

logarithm

the exponent to which bmust be raised to getx; written y = logb (x)

logistic growth model

a function of the form f (x) = c1+ae−bx where

c1+a is the initial value, c is the carrying capacity, or

limiting value, and b is a constant determined by the rate of growth

lower limit of summation

the number used in the explicit formula to �nd the �rst term in a series

M main diagonal

entries from the upper left corner diagonally to the lower right corner of a square matrix

major axis

the longer of the two axes of an ellipse

matrix

a rectangular array of numbers

minor axis

the shorter of the two axes of an ellipse

monomial

a polynomial containing one term

Multiplication Principle

if one event can occur inm ways and a second event can occur in n ways after the �rst event hasoccurred, then the two events can occur in m× n ways; also known as the FundamentalCounting Principle

multiplicative inverse of a matrix

a matrix that, when multiplied by the original, equals the identity matrix

mutually exclusive events

events that have no outcomes in common

N n factorial

the product of all the positive integers from 1 ton

natural logarithm

the exponent to which the number emust be raised to getx; loge (x) is written as ln (x) .

natural numbers

the set of counting numbers: {1, 2, 3, . . . }Newton's Law of Cooling

Available for free at Connexions <http://legacy.cnx.org/content/col11759/1.2>

1150 GLOSSARY

the scienti�c formula for temperature as a function of time as an object's temperature isequalized with the ambient temperature

nominal rate

the yearly interest rate earned by an investment account, also called annual percentage rate

nondegenerate conic section

a shape formed by the intersection of a plane with a double right cone such that the plane doesnot pass through the apex; nondegenerate conics include circles, ellipses, hyperbolas, andparabolas

nonlinear inequality

an inequality containing a nonlinear expression

nth partial sum

the sum of the �rstnterms of a sequence

nth term of a sequence

a formula for the general term of a sequence

O one-to-one function

a function for which each value of the output is associated with a unique input value

order of magnitude

the power of ten, when a number is expressed in scienti�c notation, with one non-zero digit to theleft of the decimal

order of operations

a set of rules governing how mathematical expressions are to be evaluated, assigning priorities tooperations

outcomes

the possible results of an experiment

output

each object or value in the range that is produced when an input value is entered into a function

P parabola

the set of all points (x, y) in a plane that are the same distance from a �xed line, called thedirectrix, and a �xed point (the focus) not on the directrix

partial fraction decomposition

the process of returning a simpli�ed rational expression to its original form, a sum or di�erence ofsimpler rational expressions

partial fractions

the individual fractions that make up the sum or di�erence of a rational expression beforecombining them into a simpli�ed rational expression

perfect square trinomial

the trinomial that results when a binomial is squared

perimeter

in linear units, the perimeter formula is used to �nd the linear measurement, or outside lengthand width, around a two-dimensional regular object; for a rectangle:P = 2L+ 2W

Available for free at Connexions <http://legacy.cnx.org/content/col11759/1.2>

GLOSSARY 1151

permutation

a selection of objects in which order matters

polar equation

an equation of a curve in polar coordinates r and θ

polynomial

a sum of terms each consisting of a variable raised to a nonnegative integer power

polynomial equation

an equation containing a string of terms including numerical coe�cients and variables raised towhole-number exponents

power rule for logarithms

a rule of logarithms that states that the log of a power is equal to the product of the exponentand the log of its base

principal nth root

the number with the same sign as a that when raised to the nth power equals a

principal square root

the nonnegative square root of a number a that, when multiplied by itself, equals a

probability

a number from 0 to 1 indicating the likelihood of an event

probability model

a mathematical description of an experiment listing all possible outcomes and their associatedprobabilities

product rule for logarithms

a rule of logarithms that states that the log of a product is equal to a sum of logarithms

pro�t function

the pro�t function is written asP (x) = R (x)− C (x) ,revenue minus cost

Pythagorean Theorem

a theorem that states the relationship among the lengths of the sides of a right triangle, used tosolve right triangle problems

Q quadratic equation

an equation containing a second-degree polynomial; can be solved using multiple methods

quadratic formula

a formula that will solve all quadratic equations

quotient rule for logarithms

a rule of logarithms that states that the log of a quotient is equal to a di�erence of logarithms

R radical equation

an equation containing at least one radical term where the variable is part of the radicand

radical expression

an expression containing a radical symbol

radical

Available for free at Connexions <http://legacy.cnx.org/content/col11759/1.2>

1152 GLOSSARY

the symbol used to indicate a root

radicand

the number under the radical symbol

range

the set of output values that result from the input values in a relation

rational equation

an equation consisting of a fraction of polynomials

rational expression

the quotient of two polynomial expressions

rational numbers

the set of all numbers of the form mn ,wherem andn are integers andn 6= 0.Any rational number

may be written as a fraction or a terminating or repeating decimal.

real number line

a horizontal line used to represent the real numbers. An arbitrary �xed point is chosen torepresent 0; positive numbers lie to the right of 0 and negative numbers to the left.

real numbers

the sets of rational numbers and irrational numbers taken together

recursive formula

a formula that de�nes each term of a sequence using previous term(s)

relation

a set of ordered pairs

revenue function

the function that is used to calculate revenue, simply written asR = xp,wherex = quantityand p = price

row

a set of numbers aligned horizontally in a matrix

row operations

adding one row to another row, multiplying a row by a constant, interchanging rows, and so on,with the goal of achieving row-echelon form

row-echelon form

after performing row operations, the matrix form that contains ones down the main diagonal andzeros at every space below the diagonal

row-equivalent

two matricesA andB are row-equivalent if one can be obtained from the other by performingbasic row operations

S sample space

the set of all possible outcomes of an experiment

scalar multiple

an entry of a matrix that has been multiplied by a scalar

scienti�c notation

Available for free at Connexions <http://legacy.cnx.org/content/col11759/1.2>

GLOSSARY 1153

a shorthand notation for writing very large or very small numbers in theform a × 10n where 1 ≤ |a| < 10 andn is an integer

sequence

a function whose domain is a subset of the positive integers

series

the sum of the terms in a sequence

slope

the change in y-values over the change in x-values

solution set

the set of all ordered pairs or triples that satisfy all equations in a system of equations

solution set

the set of all solutions to an equation

square root property

one of the methods used to solve a quadratic equation, in which thex2 term is isolated so that thesquare root of both sides of the equation can be taken to solve for x

substitution method

an algebraic technique used to solve systems of linear equations in which one of the two equationsis solved for one variable and then substituted into the second equation to solve for the secondvariable

summation notation

a notation for series using the Greek letter sigma; it includes an explicit formula and speci�es the�rst and last terms in the series

system of linear equations

a set of two or more equations in two or more variables that must be considered simultaneously.

system of nonlinear equations

a system of equations containing at least one equation that is of degree larger than one

system of nonlinear inequalities

a system of two or more inequalities in two or more variables containing at least one inequalitythat is not linear

T term

a number in a sequence

term of a polynomial

any aixi of a polynomial in the form anx

n + ...+ a2x2 + a1x+ a0

transverse axis

the axis of a hyperbola that includes the foci and has the vertices as its endpoints

trinomial

a polynomial containing three terms

U union of two events

the event that occurs if either or both events occur

Available for free at Connexions <http://legacy.cnx.org/content/col11759/1.2>

1154 GLOSSARY

upper limit of summation

the number used in the explicit formula to �nd the last term in a series

V variable

a quantity that may change value

vertical line test

a method of testing whether a graph represents a function by determining whether a vertical lineintersects the graph no more than once

volume

in cubic units, the volume measurement includes length, width, and depth:V = LWH

W whole numbers

the set consisting of 0 plus the natural numbers: {0, 1, 2, 3, . . . }

Z zero-product property

the property that formally states that multiplication by zero is zero, so that each factor of aquadratic equation can be set equal to zero to solve equations

Available for free at Connexions <http://legacy.cnx.org/content/col11759/1.2>

INDEX 1155

Index of Keywords and Terms

Keywords are listed by the section with that keyword (page numbers are in parentheses). Keywordsdo not necessarily appear in the text of the page. They are merely associated with that section. Ex.apples, � 1.1 (1) Terms are referenced by the page they appear on. Ex. apples, 1

A absolute maximum, � (1), � 3.1(273)absolute minimum, � (1), � 3.1(273)absolute value, � (1), � 3.1(273)absolute value equation, 214, 215absolute value equations, � 2.7(205)absolute value inequality, � 2.8(223), 230, 230absolute-value equation, � (1), � 3.1(273)absolute-value function, � (1), � 3.1(273)absolute-value inequality, � (1), � 3.1(273)addition method, 607, 609, 617Addition Principle, � (1), � 9.1(997),� 9.6(1073), 1074addition property, � 2.8(223), 226adjacent side, � (1)algebraic expression, � 1.1(9), � 1.2(10), 23altitude, � (1)ambiguous case, � (1)amplitude, � (1)angle, � (1)angle of elevation, � (1)angle of rotation, � 8.1(809), � 8.5(896), 902angular speed, � (1)annual interest, 1067annual percentage rate, � (1), � 6.1(339),� 6.2(340)annual percentage rate (APR), 354annuity, � (1), � 9.1(997), � 9.5(1056), 1067apoapsis, 919arc length, � (1)arccosine, � (1)Archimedes' spiral, � (1)arcsine, � (1)arctangent, � (1)area, � 2.4(155), 159area of a sector, � (1)argument, � (1)arithmetic sequence, � (1), � 9.1(997),� 9.3(1023), 1023, 1025, 1028, 1029, 1058arithmetic series, � (1), � 9.1(997), � 9.5(1056),1059arrow notation, � (1)

associative property of addition, � 1.1(9),� 1.2(10), 19associative property of multiplication, � 1.1(9),� 1.2(10), 19asymptotes, 840augmented matrix, � (1), � 7.1(599),� 7.7(706), 707, 712, 714, 734average rate of change, � (1), � 3.1(273)axes of symmetry, 840axis of symmetry, � (1), � 8.1(809), � 8.4(869),880, 883

B base, � 1.2(10), 16binomial, � 1.1(9), � 1.5(71), 72binomial coe�cient, � (1), � 9.1(997),� 9.7(1088), 1088binomial expansion, � (1), � 9.1(997),� 9.7(1088), 1090, 1094Binomial Theorem, � (1), � 9.1(997),� 9.7(1088), 1092break-even point, � (1), � 7.1(599), � 7.2(600),619

C cardioid, � (1)carrying capacity, � (1), � 6.1(339), � 6.8(485),496Cartesian equations, � (1)center of a hyperbola, � 8.1(809), � 8.3(839),840center of an ellipse, � (1), � 8.1(809),� 8.2(810), 812central rectangle, 841change-of-base formula, � (1), � 6.1(339),� 6.6(448), 459, 459circle, 656, 659circular motion, � (1)co-vertex, 812co-vertices, � 8.1(809), � 8.2(810), 815coe�cient, � (1), � 1.1(9), � 1.5(71), 72, 72coe�cient matrix, � (1), � 7.1(599), � 7.7(706),707, 710, 737, 742cofunction, � (1)

Available for free at Connexions <http://legacy.cnx.org/content/col11759/1.2>

1156 INDEX

cofunction identities, � (1)column, � (1), � 7.1(599), � 7.6(686), 687column matrix, 687combination, � (1), � 9.1(997), � 9.6(1073)combinations, 1080, � 9.7(1088), 1088combining functions, � 3.1(273)common base, 467common di�erence, � (1), � 9.1(997),� 9.3(1023), 1023, 1058common logarithm, � (1), � 6.1(339),� 6.4(397), 404common ratio, � (1), � 9.1(997), � 9.4(1041),1042, 1061commutative property of addition, � 1.1(9),� 1.2(10), 19commutative property of multiplication,� 1.1(9), � 1.2(10), 19complement, � (1), � 9.1(997), � 9.8(1099)complement of an event, 1104completing the square, � 2.6(184), 194, 195complex conjugate, � (1), � 2.5(168), 177complex number, � (1), � 2.5(168), 169complex plane, � (1), � 2.5(168), 170component form, � (1)composite function, � (1), � 3.1(273)composition of functions, � (1), � 3.1(273)compound inequality, � 2.8(223), 228compound interest, � (1), � 6.1(339),� 6.2(340), 354compressing factor, � (1)compression, 377, 425conditional equation, � 2.3(130), 131conic, � (1), � 8.1(809), � 8.2(810), 811, 839,� 8.5(896), 931conjugate axis, � 8.1(809), � 8.3(839), 840consistent system, � (1), � 7.1(599), � 7.2(600),601constant, � 1.1(9), � 1.2(10), 23constant of variation, � (1)continuous function, � (1)coordinate plane, 871correlation coe�cient, � (1)cosecant function, � (1)cosine function, � (1)cost function, � (1), � 3.1(273), � 7.1(599),� 7.2(600), 619cotangent, � (1)cotangent function, � (1)coterminal angles, � (1)Cramer, � 7.1(599), � 7.9(750)Cramer's rule, � (1), � 7.1(599), � 7.9(750),

751, 752, 755, 761cubic functions, � (1)curvilinear path, � (1)

D damped harmonic motion, � (1)De Moivre's Theorem, � (1)decomposing composite functions, � 3.1(273)decomposition, � (1), � 7.1(599), � 7.5(671),672decreasing function, � (1), � 3.1(273)degenerate conic sections, 896degree, � (1), � 1.1(9), � 1.5(71), 72, 72dependent system, � (1), � 7.1(599),� 7.2(600), 601, 616, 640dependent variable, � (1), � 3.1(273),� 3.2(274), 274derivative, � (1)Descartes' Rule of Signs, � (1)determinant, � (1), � 7.1(599), � 7.9(750), 750,754, 755di�erence of squares, � 1.1(9), � 1.5(71), 78di�erentiable, � (1)dimpled limaçon, � (1)direct variation, � (1)directrix, � 8.1(809), � 8.4(869), 870, 875, 876,878, 880, � 8.6(919), 920, 921, 928, 931, 931discontinuous function, � (1)discriminant, � 2.6(184), 197, 197distance formula, � (1), 842, 872distributive property, � 1.1(9), � 1.2(10), 20diverges, 1063Division Algorithm, � (1)domain, � (1), � 3.1(273), � 3.2(274), 275, 287domain of a composite function, � 3.1(273)dot product, � (1)double-angle formulas, � (1)doubling time, � (1), � 6.1(339), � 6.8(485),493

E eccentricity, � 8.1(809), � 8.6(919), 921, 921elimination, � (1), � 7.1(599), � 7.3(632), 659ellipse, � (1), 659, � 8.1(809), � 8.2(810), 811,813, 815, 818, 824, 870, � 8.6(919), 922, 928ellipsis, 998end behavior, � (1)entry, � (1), � 7.1(599), � 7.6(686), 687equation, � 1.1(9), � 1.2(10), 26, 285equation in quadratic form, 216equations in quadratic form, � 2.7(205)even function, � (1), � 3.1(273)even-odd identities, � (1)event, � (1), � 9.1(997), � 9.8(1099), 1099

Available for free at Connexions <http://legacy.cnx.org/content/col11759/1.2>

INDEX 1157

events, � 9.1(997)experiment, � (1), � 9.1(997), � 9.8(1099), 1099explicit formula, � (1), � 9.1(997), � 9.2(998),998, 1001, 1003, � 9.3(1023), 1030, � 9.4(1041),1047exponent, � 1.2(10), 16Exponential decay, 341, 351, 367, 486, 491,494, 512exponential equation, 465exponential function, � (1), � 6.1(339), 341exponential functions, � 6.2(340), � 6.3(366)exponential growth, � (1), � 6.1(339),� 6.2(340), 341, 345, 370, 486, 493, 496exponential model, � (1), � 6.1(339), � 6.9(511)exponential notation, � 1.1(9), � 1.2(10), 16extraneous solution, � (1), � 6.1(339),� 6.7(465), 472extraneous solutions, � 2.7(205), 212extrapolation, � (1)

F factor by grouping, � 1.1(9), � 1.6(83), 87Factor Theorem, � (1)factorial, 1012factoring, � (1), � 2.6(184), 184feasible region, � (1), � 7.1(599), � 7.4(651),663�nite arithmetic sequence, 1032�nite sequence, � (1), � 9.1(997), � 9.2(998),1000foci, � (1), � 8.1(809), � 8.2(810), 811, 814,815, � 8.3(839), 841focus, � (1), � 8.1(809), � 8.2(810), 811,� 8.4(869), 870, 875, 876, 878, 880, � 8.6(919),920, 928, 931, 931formula, � 1.1(9), � 1.2(10), 26, 285function, � (1), � 3.1(273), � 3.2(274), 275function notation, 278Fundamental Counting, � 9.1(997), � 9.6(1073)Fundamental Counting Principle, � (1), 1076Fundamental Theorem of Algebra, � (1)

G Gauss, � (1), � 7.1(599), � 7.3(632), 633, 707Gaussian elimination, � (1), � 7.1(599),� 7.3(632), 633, � 7.7(706), 710, 711general form of a quadratic function, � (1)Generalized Pythagorean Theorem, � (1)geometric sequence, � (1), � 9.1(997),� 9.4(1041), 1042, 1061geometric series, � (1), � 9.1(997), � 9.5(1056),1062global maximum, � (1)global minimum, � (1)

greatest common factor, � 1.1(9), � 1.6(83), 84,85, � 2.6(184), 185

H half-angle formulas, � (1)half-life, � (1), � 6.1(339), 477, � 6.8(485), 486harmonic motion, � (1)Heaviside method, � (1), � 7.1(599),� 7.5(671), 674Heron's formula, � (1)horizontal asymptote, � (1)horizontal compression, � (1), � 3.1(273)horizontal line, � (1), � 2.3(130), 142horizontal line test, � (1), � 3.1(273),� 3.2(274), 296horizontal re�ection, � (1), � 3.1(273)horizontal shift, � (1), � 3.1(273), 374, 419horizontal stretch, � (1), � 3.1(273)hyperbola, � 8.1(809), � 8.3(839), 839, 845,846, 847, 849, 851, 853, 857, 870, � 8.6(919),922, 926hypotenuse, � (1)

I identities, � (1)identity equation, � 2.3(130), 131identity matrix, � (1), � 7.1(599), � 7.8(727),727, 727, 734identity property of addition, � 1.1(9),� 1.2(10), 21identity property of multiplication, � 1.1(9),� 1.2(10), 21imaginary number, � (1), � 2.5(168), 169inconsistent equation, � 2.3(130), 131inconsistent system, � (1), � 7.1(599),� 7.2(600), 601, 615, 639inconsistent systems, � 7.3(632)increasing function, � (1), � 3.1(273)independent system, � (1), � 7.1(599),� 7.2(600), 601independent variable, � (1), � 3.1(273),� 3.2(274), 274index, � 1.1(9), � 1.4(56), 63, 63index of summation, � (1), � 9.1(997),� 9.5(1056), 1058inequality, 661in�nite geometric series, 1064in�nite sequence, � (1), � 9.1(997), � 9.2(998),1001in�nite series, � (1), � 9.1(997), � 9.5(1056),1063initial point, � (1)initial side, � (1)inner-loop limaçon, � (1)

Available for free at Connexions <http://legacy.cnx.org/content/col11759/1.2>

1158 INDEX

input, � (1), � 3.1(273), � 3.2(274), 275instantaneous rate of change, � (1)instantaneous velocity, � (1)integers, � 1.1(9), � 1.2(10), 10, 15Intermediate Value Theorem, � (1)interpolation, � (1)intersection, 1102interval, � 2.8(223), 224interval notation, � (1), � 2.8(223), 224,� 3.1(273)inverse cosine function, � (1)inverse function, � (1), � 3.1(273)inverse matrix, 734, 737inverse of a polynomial function, � (1)inverse of a rational function, � (1)inverse property of addition, � 1.1(9),� 1.2(10), 21inverse property of multiplication, � 1.1(9),� 1.2(10), 21inverse sine function, � (1)inverse tangent function, � (1)inverse trigonometric functions, � (1)inverse variation, � (1)inversely proportional, � (1)invertible functions, � (1)invertible matrix, � (1), � 7.1(599), 727,� 7.9(750), 750irrational numbers, � 1.1(9), � 1.2(10), 12, 15

J joint variation, � (1)jump discontinuity, � (1)

K Kronecker, � (1)

L latus rectum, � 8.1(809), � 8.4(869), 871, 875,876, 880Law of Cosines, � (1)Law of Sines, � (1)leading coe�cient, � (1), � 1.1(9), � 1.5(71),72, 72leading term, � (1), � 1.1(9), � 1.5(71), 72, 72least common denominator, � 1.1(9), � 1.7(96),100, � 2.3(130), 133least squares regression, � (1)left-hand limit, � (1)lemniscate, � (1)limaçon, � (1)limit, � (1)limit notation, � (1)limit of a di�erence, � (1)limit of a power, � (1)limit of a product, � (1)

limit of a root, � (1)limit of a sum, � (1)linear equation, � 2.3(130), 131Linear Factorization Theorem, � (1)linear function, � (1)linear growth, � 6.2(340), 341linear speed, � (1)local extrema, � (1), � 3.1(273)local maximum, � (1), � 3.1(273)local minimum, � (1), � 3.1(273)logarithm, � (1), � 6.1(339), � 6.4(397), 400logarithmic equation, 473logarithmic function, � (1), � 6.1(339),� 6.5(411)logarithmic model, � (1), � 6.1(339),� 6.9(511), 516logistic growth model, � (1), � 6.1(339),� 6.8(485), 496logistic model, � (1), � 6.1(339), � 6.9(511)long division, � (1)lower limit of, � 9.1(997)lower limit of summation, � (1), � 9.5(1056),1058

M magnitude, � (1)main diagonal, � (1), � 7.1(599), � 7.7(706),709major and minor axes, 815major axis, � (1), � 8.1(809), � 8.2(810), 812,818matrices, � 7.1(599), � 7.6(686)matrix, � (1), � 7.1(599), � 7.6(686), 687, 688,707matrix multiplication, � (1), � 7.1(599),� 7.6(686), 695, 729, 737matrix operations, � (1), � 7.1(599),� 7.6(686), 688measure of an angle, � (1)midline, � (1)minor axis, � (1), � 8.1(809), � 8.2(810), 812model breakdown, � (1)modulus, � (1)monomial, � 1.1(9), � 1.5(71), 72Multiplication, � 9.1(997), � 9.6(1073)Multiplication Principle, � (1), 1076multiplication property, � 2.8(223), 226multiplicative inverse, 731, 732multiplicative inverse of a matrix, � (1),� 7.1(599), � 7.8(727), 727, 728multiplicity, � (1)mutually exclusive, � 9.1(997)mutually exclusive events, � (1), � 9.8(1099),

Available for free at Connexions <http://legacy.cnx.org/content/col11759/1.2>

INDEX 1159

1103

N n factorial, � (1), � 9.1(997), � 9.2(998)natural logarithm, � (1), � 6.1(339), � 6.4(397),406, 470natural numbers, � (1), � 1.1(9), � 1.2(10), 10,15, � 3.1(273), � 3.2(274), 274negative angle, � (1)Newton's Law of Cooling, � (1), � 6.1(339),� 6.8(485), 494nominal rate, � (1), � 6.1(339), � 6.2(340), 354non-right triangles, � (1)nondegenerate conic section, � 8.1(809),� 8.5(896), 898nondegenerate conic sections, 896nonlinear inequality, � (1), � 7.1(599),� 7.4(651), 661nth partial sum, � (1), � 9.1(997), � 9.5(1056)nth term of a sequence, � (1), � 9.1(997),� 9.2(998)nth term of the sequence, 998, 1001nth partial sum, 1057

O oblique triangle, � (1)odd function, � (1), � 3.1(273)one-loop limaçon, � (1)one-to-one, 369, 399, 449, 459one-to-one function, � (1), � 3.1(273),� 3.2(274), 292one-to-one functions, � 6.3(366)opposite side, � (1)order of magnitude, � (1), � 6.1(339),� 6.8(485), 488order of operations, � 1.1(9), � 1.2(10), 16ordered pair, � (1), � 3.1(273), � 3.2(274), 274ordered triple, � (1), � 7.1(599), � 7.3(632), 633outcome, � (1), � 9.1(997), � 9.8(1099)outcomes, 1099output, � (1), � 3.1(273), � 3.2(274), 275

P parabola, � (1), 652, � 8.1(809), � 8.4(869),870, 875, 877, 878, 882, � 8.6(919), 920, 922,923parabolic curves, � (1)parallel, � 2.3(130), 144parallel lines, � (1)parameter, � (1)parameterize a curve, � (1)parametric equations, � (1)parent function, 419partial fraction decomposition, � (1),� 7.1(599), � 7.5(671), 672, 672

partial fractions, � (1), � 7.1(599), � 7.5(671),672Pascal's Triangle, 1091Pascal's Triangle, � 9.7(1088)perfect square trinomial, � 1.1(9), � 1.5(71), 77periapsis, 919perimeter, � 2.4(155), 159period, � (1)periodic function, � (1)periodic motion, � (1)permutation, � (1), � 9.1(997), � 9.6(1073),1076perpendicular, � 2.3(130), 146perpendicular lines, � (1)pH, 448phase shift, � (1)piecewise function, � (1), � 3.1(273)piecewise functions, 1005plane curves, � (1)point-slope form, � (1)point-slope formula, � 2.3(130), 148, 848polar axis, � (1)polar coordinates, � (1)polar equation, � 8.1(809), � 8.6(919), 921polar equations, � (1)polar form, � (1)polar form of a complex number, � (1)polar form of a conic, 931pole, � (1)polynomial, � 1.1(9), � 1.5(71), 72, 72polynomial equation, � 2.7(205), 208polynomial function, � (1)position vector, � (1)positive angle, � (1)power function, � (1)power rule for logarithms, � (1), � 6.1(339),� 6.6(448), 453, 459principal nth root, � 1.1(9), � 1.4(56), 63, 63principal square root, � 1.1(9), � 1.4(56), 57, 57Principle, � 9.1(997), � 9.6(1073)probability, � (1), � 9.1(997), � 9.8(1099), 1099probability model, � (1), � 9.1(997),� 9.8(1099), 1099product of two matrices, � (1), � 7.1(599),� 7.6(686), 695product rule for logarithms, � (1), � 6.1(339),� 6.6(448), 450, 452product-to-sum formulas, � (1)pro�t function, � (1), � 7.1(599), � 7.2(600),619projectile motion, � (1)

Available for free at Connexions <http://legacy.cnx.org/content/col11759/1.2>

1160 INDEX

properties of determinants, 760properties of limits, � (1)Proxima Centauri, 488Pythagorean identities, � (1)Pythagorean Theorem, � (1), � 2.6(184), 198

Q quadrantal angle, � (1)quadratic, 677, 679quadratic equation, � (1), � 2.6(184), 184, 185,194, 195quadratic formula, � (1), � 2.6(184), 195, 196,197quotient identities, � (1)quotient rule for logarithms, � (1), � 6.1(339),� 6.6(448), 451

R radian, � (1)radian measure, � (1)radical, � 1.1(9), � 1.4(56), 57, 57radical equation, 212radical equations, � 2.7(205)radical expression, � 1.1(9), � 1.4(56), 57, 57radical functions, � (1)radicand, � 1.1(9), � 1.4(56), 57, 57, � 2.7(205),212radiocarbon dating, 492range, � (1), � 3.1(273), � 3.2(274), 275rate of change, � (1), � 3.1(273)rational equation, � 2.3(130), 134rational expression, � 1.1(9), � 1.7(96), 97,� 2.3(130), 133, 672, 673, 679rational function, � (1)rational number, � 2.3(130), 133rational numbers, � 1.1(9), � 1.2(10), 10, 15Rational Zero Theorem, � (1)rational zeros, � (1)ray, � (1)real number line, � 1.1(9), � 1.2(10), 13real numbers, � 1.1(9), � 1.2(10), 13reciprocal, � 2.3(130), 146reciprocal identities, � (1)rectangular coordinates, � (1)rectangular equations, � (1)rectangular form, � (1)recursive formula, � (1), � 9.1(997), � 9.2(998),1009, � 9.3(1023), 1027, 1028, � 9.4(1041), 1044reduction formulas, � (1)reference angle, � (1)re�ection, 380, 429regression analysis, � (1), � 6.1(339),� 6.9(511), 511, 516, 520relation, � (1), � 3.1(273), � 3.2(274), 274

Remainder Theorem, � (1)removable discontinuity, � (1)resultant, � (1)revenue function, � (1), � 7.1(599), � 7.2(600),619Richter Scale, 398right triangle, � (1)right-hand limit, � (1)roots, � (1)rose curve, � (1)rotated conics, � 8.1(809), � 8.5(896)rotation of axes, � 8.1(809), � 8.5(896)row, � (1), � 7.1(599), � 7.6(686), 687row matrix, 687row operations, � (1), � 7.1(599), � 7.7(706),709, 712, 715, 731, 732, 734, 734row-echelon form, � (1), � 7.1(599), � 7.7(706),709, 712, 714row-equivalent, � (1), � 7.1(599), � 7.7(706),709

S sample space, � (1), � 9.1(997), � 9.8(1099),1099SAS, � (1)scalar, � (1), 692scalar multiple, � (1), � 7.1(599), � 7.6(686),692scalar multiplication, � (1), � 7.1(599),� 7.6(686), 692scienti�c notation, � 1.1(9), � 1.3(34), 46, 47,48secant, � (1)secant function, � (1)secant line, � (1)sequence, � (1), � 9.1(997), � 9.2(998), 998,1000, � 9.3(1023), 1023series, � (1), � 9.1(997), � 9.5(1056), 1057set-builder notation, � (1), � 2.8(223), 224,� 3.1(273)sigma, 1057simple harmonic motion, � (1)sine function, � (1)sinusoidal, � (1)sinusoidal function, � (1)slope, � (1), � 2.3(130), 137slope-intercept form, � (1)smooth curve, � (1)solution, � (1), � 7.1(599), � 7.2(600)solution set, � (1), � 2.3(130), 131, � 7.1(599),� 7.3(632), 633solving systems of linear equations, 607special angles, � (1)

Available for free at Connexions <http://legacy.cnx.org/content/col11759/1.2>

INDEX 1161

speed, � (1)square matrix, � (1), � 7.1(599), 687,� 7.9(750), 750square root function, � (1)square root of a negative number, � (1)square root property, � 2.6(184), 192square roots, � (1)SSS, � (1)standard form, � 2.3(130), 141standard form of a quadratic function, � (1)standard position, � (1)stretch, 377stretching factor, � (1)substitution method, � (1), � 7.1(599),� 7.2(600), 606sum and di�erence formulas for cosine, � (1)sum and di�erence formulas for sine, � (1)sum and di�erence formulas for tangent, � (1)sum-to-product formulas, � (1)summation, � 9.1(997)summation notation, � (1), � 9.1(997),� 9.5(1056), 1057symmetry test, � (1)synthetic division, � (1)system of dependent equations, � (1),� 7.1(599), � 7.3(632)system of equations, 707, 708, 711, 714, 738system of linear equations, � (1), � 7.1(599),� 7.2(600), 601, 604, 606system of nonlinear equations, � (1),� 7.1(599), � 7.4(651), 652system of nonlinear inequalities, � (1),� 7.1(599), � 7.4(651), 663system of three equations in three variables,� (1), � 7.1(599), � 7.9(750), 755systems of linear equations in three variables,� (1), � 7.1(599), � 7.3(632)systems of three equations, � (1), � 7.1(599),� 7.3(632)

T tangent function, � (1)tangent line, � (1)term, 998, � 9.3(1023), 1023term of a polynomial, � 1.1(9), � 1.5(71), 72, 72term of a polynomial function, � (1)terminal point, � (1)terminal side, � (1)

terms, � (1), � 9.1(997), � 9.2(998)translation, 818transverse axis, � 8.1(809), � 8.3(839), 840trigonometric parametric equations, � (1)trinomial, � 1.1(9), � 1.5(71), 72turning point, � (1)two-sided limit, � (1)

U union of two events, � (1), � 9.1(997),� 9.8(1099), 1101unit circle, � (1)unit vector, � (1)upper limit of, � 9.1(997)upper limit of summation, � (1), � 9.5(1056),1058upper triangular form, � (1), � 7.1(599),� 7.3(632), 633

V variable, � 1.1(9), � 1.2(10), 23varies directly, � (1)varies inversely, � (1)vector, � (1)vector addition, � (1)vertex, � (1), � 8.1(809), 812, 814, � 8.4(869),871, 880vertex form of a quadratic function, � (1)vertical asymptote, � (1)vertical compression, � (1), � 3.1(273)vertical line, � (1), � 2.3(130), 142vertical line test, � (1), � 3.1(273), � 3.2(274),293vertical re�ection, � (1), � 3.1(273)vertical shift, � (1), � 3.1(273), 372, 421, 494vertical stretch, � (1), � 3.1(273), 425vertices, � 8.1(809), � 8.2(810), 812, 815,� 8.3(839)volume, � 2.4(155), 159

W whole numbers, � 1.1(9), � 1.2(10), 10, 15

X x-intercept, � (1)

Y y-intercept, � (1)

Z zero product property, � 2.6(184)zero-product property, 185zeros, � (1)

n factorial, 1012

Available for free at Connexions <http://legacy.cnx.org/content/col11759/1.2>

1162 ATTRIBUTIONS

Attributions

Collection: College AlgebraEdited by: OpenStax College AlgebraURL: http://legacy.cnx.org/content/col11759/1.2/License: http://creativecommons.org/licenses/by/4.0/

Module: "Preface"By: OpenStax College AlgebraURL: http://legacy.cnx.org/content/m53195/1.1/Pages: 1-8Copyright: OpenStax College AlgebraLicense: http://creativecommons.org/licenses/by/4.0/Based on: PrefaceBy: OpenStax College, OpenStax CATURL: http://legacy.cnx.org/content/m53193/1.1/

Module: "Introduction to Prerequisites"By: OpenStax CollegeURL: http://legacy.cnx.org/content/m51240/1.2/Pages: 9-10Copyright: Rice UniversityLicense: http://creativecommons.org/licenses/by/4.0/

Module: "Real Numbers: Algebra Essentials"By: OpenStax CollegeURL: http://legacy.cnx.org/content/m51239/1.2/Pages: 10-34Copyright: Rice UniversityLicense: http://creativecommons.org/licenses/by/4.0/

Module: "Exponents and Scienti�c Notation"By: OpenStax CollegeURL: http://legacy.cnx.org/content/m51241/1.2/Pages: 34-56Copyright: Rice UniversityLicense: http://creativecommons.org/licenses/by/4.0/

Module: "Radicals and Rational Expressions"By: OpenStax CollegeURL: http://legacy.cnx.org/content/m51242/1.2/Pages: 56-70Copyright: Rice UniversityLicense: http://creativecommons.org/licenses/by/4.0/

Module: "Polynomials"By: OpenStax CollegeURL: http://legacy.cnx.org/content/m51246/1.2/Pages: 71-83Copyright: Rice UniversityLicense: http://creativecommons.org/licenses/by/4.0/

Available for free at Connexions <http://legacy.cnx.org/content/col11759/1.2>

ATTRIBUTIONS 1163

Module: "Factoring Polynomials"By: OpenStax CollegeURL: http://legacy.cnx.org/content/m51247/1.2/Pages: 83-96Copyright: Rice UniversityLicense: http://creativecommons.org/licenses/by/4.0/

Module: "Rational Expressions"By: OpenStax CollegeURL: http://legacy.cnx.org/content/m51248/1.2/Pages: 96-112Copyright: Rice UniversityLicense: http://creativecommons.org/licenses/by/4.0/

Module: "Introduction to Equations and Inequalities"By: OpenStax CollegeURL: http://legacy.cnx.org/content/m51251/1.2/Pages: 129-130Copyright: Rice UniversityLicense: http://creativecommons.org/licenses/by/4.0/

Module: "The Rectangular Coordinate Systems and Graphs"By: OpenStax CATURL: http://legacy.cnx.org/content/m51252/1.1/Page: 130Copyright: OpenStax CATLicense: http://creativecommons.org/licenses/by/4.0/

Module: "Linear Equations in One Variable"By: OpenStax CollegeURL: http://legacy.cnx.org/content/m51253/1.2/Pages: 130-155Copyright: Rice UniversityLicense: http://creativecommons.org/licenses/by/4.0/

Module: "Models and Applications"By: OpenStax CollegeURL: http://legacy.cnx.org/content/m51254/1.2/Pages: 155-168Copyright: Rice UniversityLicense: http://creativecommons.org/licenses/by/4.0/

Module: "Complex Numbers"By: OpenStax CollegeURL: http://legacy.cnx.org/content/m51255/1.2/Pages: 168-183Copyright: Rice UniversityLicense: http://creativecommons.org/licenses/by/4.0/

Module: "Quadratic Equations"By: OpenStax CollegeURL: http://legacy.cnx.org/content/m51256/1.2/Pages: 184-205Copyright: Rice UniversityLicense: http://creativecommons.org/licenses/by/4.0/

Available for free at Connexions <http://legacy.cnx.org/content/col11759/1.2>

1164 ATTRIBUTIONS

Module: "Other Types of Equations"By: OpenStax CollegeURL: http://legacy.cnx.org/content/m51258/1.2/Pages: 205-222Copyright: Rice UniversityLicense: http://creativecommons.org/licenses/by/4.0/

Module: "Linear Inequalities and Absolute Value Inequalities"By: OpenStax CollegeURL: http://legacy.cnx.org/content/m51259/1.2/Pages: 223-244Copyright: Rice UniversityLicense: http://creativecommons.org/licenses/by/4.0/

Module: "Introduction to Functions"By: OpenStax CollegeURL: http://legacy.cnx.org/content/m51260/1.2/Page: 273Copyright: Rice UniversityLicense: http://creativecommons.org/licenses/by/4.0/

Module: "Functions and Function Notation"By: OpenStax CollegeURL: http://legacy.cnx.org/content/m51261/1.2/Pages: 274-326Copyright: Rice UniversityLicense: http://creativecommons.org/licenses/by/4.0/

Module: "Domain and Range"By: OpenStax CATURL: http://legacy.cnx.org/content/m51262/1.1/Page: 326Copyright: OpenStax CATLicense: http://creativecommons.org/licenses/by/4.0/

Module: "Rates of Change and Behavior of Graphs"By: OpenStax CATURL: http://legacy.cnx.org/content/m51263/1.1/Page: 326Copyright: OpenStax CATLicense: http://creativecommons.org/licenses/by/4.0/

Module: "Composition of Functions"By: OpenStax CATURL: http://legacy.cnx.org/content/m51265/1.1/Page: 326Copyright: OpenStax CATLicense: http://creativecommons.org/licenses/by/4.0/

Module: "Transformation of Functions"By: OpenStax CATURL: http://legacy.cnx.org/content/m51266/1.1/Page: 326Copyright: OpenStax CATLicense: http://creativecommons.org/licenses/by/4.0/

Available for free at Connexions <http://legacy.cnx.org/content/col11759/1.2>

ATTRIBUTIONS 1165

Module: "Absolute Value Functions"By: OpenStax CATURL: http://legacy.cnx.org/content/m51267/1.1/Page: 326Copyright: OpenStax CATLicense: http://creativecommons.org/licenses/by/4.0/

Module: "Inverse Functions"By: OpenStax CATURL: http://legacy.cnx.org/content/m51268/1.1/Page: 326Copyright: OpenStax CATLicense: http://creativecommons.org/licenses/by/4.0/

Module: "Introduction to Linear Functions"By: OpenStax CATURL: http://legacy.cnx.org/content/m51269/1.1/Page: 335Copyright: OpenStax CATLicense: http://creativecommons.org/licenses/by/4.0/

Module: "Linear Functions"By: OpenStax CATURL: http://legacy.cnx.org/content/m51270/1.1/Page: 335Copyright: OpenStax CATLicense: http://creativecommons.org/licenses/by/4.0/

Module: "Modeling with Linear Functions"By: OpenStax CATURL: http://legacy.cnx.org/content/m51271/1.1/Page: 335Copyright: OpenStax CATLicense: http://creativecommons.org/licenses/by/4.0/

Module: "Fitting Linear Models to Data"By: OpenStax CATURL: http://legacy.cnx.org/content/m51272/1.1/Page: 335Copyright: OpenStax CATLicense: http://creativecommons.org/licenses/by/4.0/

Module: "Introduction to Polynomial and Rational Functions"By: OpenStax CATURL: http://legacy.cnx.org/content/m51273/1.1/Page: 337Copyright: OpenStax CATLicense: http://creativecommons.org/licenses/by/4.0/

Module: "Quadratic Functions"By: OpenStax CATURL: http://legacy.cnx.org/content/m51274/1.1/Page: 337Copyright: OpenStax CATLicense: http://creativecommons.org/licenses/by/4.0/

Available for free at Connexions <http://legacy.cnx.org/content/col11759/1.2>

1166 ATTRIBUTIONS

Module: "Power Functions and Polynomial Functions"By: OpenStax CATURL: http://legacy.cnx.org/content/m51275/1.1/Page: 337Copyright: OpenStax CATLicense: http://creativecommons.org/licenses/by/4.0/

Module: "Graphs of Polynomial Functions"By: OpenStax CATURL: http://legacy.cnx.org/content/m51276/1.1/Page: 337Copyright: OpenStax CATLicense: http://creativecommons.org/licenses/by/4.0/

Module: "Dividing Polynomials"By: OpenStax CATURL: http://legacy.cnx.org/content/m51277/1.1/Page: 337Copyright: OpenStax CATLicense: http://creativecommons.org/licenses/by/4.0/

Module: "Zeros of Polynomial Functions"By: OpenStax CATURL: http://legacy.cnx.org/content/m51278/1.1/Page: 337Copyright: OpenStax CATLicense: http://creativecommons.org/licenses/by/4.0/

Module: "Rational Functions"By: OpenStax CATURL: http://legacy.cnx.org/content/m51279/1.1/Page: 337Copyright: OpenStax CATLicense: http://creativecommons.org/licenses/by/4.0/

Module: "Inverses and Radical Functions"By: OpenStax CATURL: http://legacy.cnx.org/content/m51280/1.1/Page: 337Copyright: OpenStax CATLicense: http://creativecommons.org/licenses/by/4.0/

Module: "Modeling Using Variation"By: OpenStax CATURL: http://legacy.cnx.org/content/m51281/1.1/Page: 337Copyright: OpenStax CATLicense: http://creativecommons.org/licenses/by/4.0/

Module: "Introduction to Exponential and Logarithmic Functions"By: OpenStax CollegeURL: http://legacy.cnx.org/content/m49356/1.2/Pages: 339-340Copyright: Rice UniversityLicense: http://creativecommons.org/licenses/by/4.0/

Available for free at Connexions <http://legacy.cnx.org/content/col11759/1.2>

ATTRIBUTIONS 1167

Module: "Exponential Functions"By: OpenStax CollegeURL: http://legacy.cnx.org/content/m49361/1.5/Pages: 340-366Copyright: Rice UniversityLicense: http://creativecommons.org/licenses/by/4.0/

Module: "Graphs of Exponential Functions"By: OpenStax CollegeURL: http://legacy.cnx.org/content/m49362/1.5/Pages: 366-396Copyright: Rice UniversityLicense: http://creativecommons.org/licenses/by/4.0/

Module: "Logarithmic Functions"By: OpenStax CollegeURL: http://legacy.cnx.org/content/m49363/1.3/Pages: 397-411Copyright: Rice UniversityLicense: http://creativecommons.org/licenses/by/4.0/

Module: "Graphs of Logarithmic Functions"By: OpenStax CollegeURL: http://legacy.cnx.org/content/m49364/1.5/Pages: 411-447Copyright: Rice UniversityLicense: http://creativecommons.org/licenses/by/4.0/

Module: "Logarithmic Properties"By: OpenStax CollegeURL: http://legacy.cnx.org/content/m49365/1.4/Pages: 448-464Copyright: Rice UniversityLicense: http://creativecommons.org/licenses/by/4.0/

Module: "Exponential and Logarithmic Equations"By: OpenStax CollegeURL: http://legacy.cnx.org/content/m49366/1.3/Pages: 465-484Copyright: Rice UniversityLicense: http://creativecommons.org/licenses/by/4.0/

Module: "Exponential and Logarithmic Models"By: OpenStax CollegeURL: http://legacy.cnx.org/content/m49367/1.5/Pages: 485-511Copyright: Rice UniversityLicense: http://creativecommons.org/licenses/by/4.0/

Module: "Fitting Exponential Models to Data"By: OpenStax CollegeURL: http://legacy.cnx.org/content/m49368/1.4/Pages: 511-544Copyright: Rice UniversityLicense: http://creativecommons.org/licenses/by/4.0/

Available for free at Connexions <http://legacy.cnx.org/content/col11759/1.2>

1168 ATTRIBUTIONS

Module: "Introduction to Systems of Equations and Inequalities"By: OpenStax CollegeURL: http://legacy.cnx.org/content/m49418/1.4/Pages: 599-600Copyright: Rice UniversityLicense: http://creativecommons.org/licenses/by/4.0/

Module: "Systems of Linear Equations: Two Variables"By: OpenStax CollegeURL: http://legacy.cnx.org/content/m49420/1.4/Pages: 600-631Copyright: Rice UniversityLicense: http://creativecommons.org/licenses/by/4.0/

Module: "Systems of Linear Equations: Three Variables"By: OpenStax CollegeURL: http://legacy.cnx.org/content/m49419/1.4/Pages: 632-651Copyright: Rice UniversityLicense: http://creativecommons.org/licenses/by/4.0/

Module: "Systems of Nonlinear Equations and Inequalities: Two Variables"By: OpenStax CollegeURL: http://legacy.cnx.org/content/m49431/1.5/Pages: 651-671Copyright: Rice UniversityLicense: http://creativecommons.org/licenses/by/4.0/

Module: "Partial Fractions"By: OpenStax CollegeURL: http://legacy.cnx.org/content/m49432/1.3/Pages: 671-685Copyright: Rice UniversityLicense: http://creativecommons.org/licenses/by/4.0/

Module: "Matrices and Matrix Operations"By: OpenStax CollegeURL: http://legacy.cnx.org/content/m49433/1.3/Pages: 686-705Copyright: Rice UniversityLicense: http://creativecommons.org/licenses/by/4.0/

Module: "Solving Systems with Gaussian Elimination"By: OpenStax CollegeURL: http://legacy.cnx.org/content/m49434/1.3/Pages: 706-727Copyright: Rice UniversityLicense: http://creativecommons.org/licenses/by/4.0/

Module: "Solving Systems with Inverses"By: OpenStax CollegeURL: http://legacy.cnx.org/content/m49435/1.4/Pages: 727-750Copyright: Rice UniversityLicense: http://creativecommons.org/licenses/by/4.0/

Available for free at Connexions <http://legacy.cnx.org/content/col11759/1.2>

ATTRIBUTIONS 1169

Module: "Solving Systems with Cramer's Rule"By: OpenStax CollegeURL: http://legacy.cnx.org/content/m49436/1.4/Pages: 750-780Copyright: Rice UniversityLicense: http://creativecommons.org/licenses/by/4.0/

Module: "Introduction to Analytic Geometry"By: OpenStax CollegeURL: http://legacy.cnx.org/content/m49437/1.2/Pages: 809-810Copyright: Rice UniversityLicense: http://creativecommons.org/licenses/by/4.0/

Module: "The Ellipse"By: OpenStax CollegeURL: http://legacy.cnx.org/content/m49438/1.3/Pages: 810-838Copyright: Rice UniversityLicense: http://creativecommons.org/licenses/by/4.0/

Module: "The Hyperbola"By: OpenStax CollegeURL: http://legacy.cnx.org/content/m49439/1.3/Pages: 839-868Copyright: Rice UniversityLicense: http://creativecommons.org/licenses/by/4.0/

Module: "The Parabola"By: OpenStax CollegeURL: http://legacy.cnx.org/content/m49440/1.3/Pages: 869-895Copyright: Rice UniversityLicense: http://creativecommons.org/licenses/by/4.0/

Module: "Rotation of Axes"By: OpenStax CollegeURL: http://legacy.cnx.org/content/m49441/1.3/Pages: 896-919Copyright: Rice UniversityLicense: http://creativecommons.org/licenses/by/4.0/

Module: "Conic Sections in Polar Coordinates"By: OpenStax CollegeURL: http://legacy.cnx.org/content/m49442/1.4/Pages: 919-941Copyright: Rice UniversityLicense: http://creativecommons.org/licenses/by/4.0/

Module: "Introduction to Sequences, Probability and Counting Theory"By: OpenStax CollegeURL: http://legacy.cnx.org/content/m49443/1.3/Pages: 997-998Copyright: Rice UniversityLicense: http://creativecommons.org/licenses/by/4.0/

Available for free at Connexions <http://legacy.cnx.org/content/col11759/1.2>

1170 ATTRIBUTIONS

Module: "Sequences and Their Notations"By: OpenStax CollegeURL: http://legacy.cnx.org/content/m49444/1.4/Pages: 998-1022Copyright: Rice UniversityLicense: http://creativecommons.org/licenses/by/4.0/

Module: "Arithmetic Sequences"By: OpenStax CollegeURL: http://legacy.cnx.org/content/m49445/1.7/Pages: 1023-1041Copyright: Rice UniversityLicense: http://creativecommons.org/licenses/by/4.0/

Module: "Geometric Sequences"By: OpenStax CollegeURL: http://legacy.cnx.org/content/m49446/1.6/Pages: 1041-1056Copyright: Rice UniversityLicense: http://creativecommons.org/licenses/by/4.0/

Module: "Series and Their Notations"By: OpenStax CollegeURL: http://legacy.cnx.org/content/m49447/1.6/Pages: 1056-1073Copyright: Rice UniversityLicense: http://creativecommons.org/licenses/by/4.0/

Module: "Counting Principles"By: OpenStax CollegeURL: http://legacy.cnx.org/content/m49448/1.5/Pages: 1073-1088Copyright: Rice UniversityLicense: http://creativecommons.org/licenses/by/4.0/

Module: "Binomial Theorem"By: OpenStax CollegeURL: http://legacy.cnx.org/content/m49449/1.7/Pages: 1088-1098Copyright: Rice UniversityLicense: http://creativecommons.org/licenses/by/4.0/

Module: "Probability"By: OpenStax CollegeURL: http://legacy.cnx.org/content/m49450/1.4/Pages: 1099-1119Copyright: Rice UniversityLicense: http://creativecommons.org/licenses/by/4.0/

Available for free at Connexions <http://legacy.cnx.org/content/col11759/1.2>

About OpenStax-CNXRhaptos is a web-based collaborative publishing system for educational material.